Anda di halaman 1dari 272

G.R. No. 183591 October 14, 2008 MEJORADA II, HON. EDIONAR M. ZAMORAS, HON. EDGAR J. BAGUIO, HON.

MORAS, HON. EDGAR J. BAGUIO, HON. CEDRIC L.


ADRIATICO, HON. FELIXBERTO C. BOLANDO, HON. JOSEPH BRENDO C. AJERO, HON.
NORBIDEIRI B. EDDING, HON. ANECITO S. DARUNDAY, HON. ANGELICA J. CARREON
THE PROVINCE OF NORTH COTABATO, duly represented by GOVERNOR JESUS
and HON. LUZVIMINDA E. TORRINO, petitioners,
SACDALAN and/or VICE-GOVERNOR EMMANUEL PIÑOL, for and in his own
vs.
behalf, petitioners,
THE GOVERNMENT OF THE REPUBLIC OF THE PHILIPPINES PEACE NEGOTIATING
vs.
PANEL [GRP], as represented by HON. RODOLFO C. GARCIA and HON. HERMOGENES
THE GOVERNMENT OF THE REPUBLIC OF THE PHILIPPINES PEACE PANEL ON
ESPERON, in his capacity as the Presidential Adviser of Peace Process, respondents.
ANCESTRAL DOMAIN (GRP), represented by SEC. RODOLFO GARCIA, ATTY. LEAH
ARMAMENTO, ATTY. SEDFREY CANDELARIA, MARK RYAN SULLIVAN and/or GEN.
HERMOGENES ESPERON, JR., the latter in his capacity as the present and duly- x--------------------------------------------x
appointed Presidential Adviser on the Peace Process (OPAPP) or the so-called Office of
the Presidential Adviser on the Peace Process, respondents.
G.R. No. 183962 October 14, 2008

x--------------------------------------------x
ERNESTO M. MACEDA, JEJOMAR C. BINAY, and AQUILINO L. PIMENTEL III, petitioners,
vs.
G.R. No. 183752 October 14, 2008 THE GOVERNMENT OF THE REPUBLIC OF THE PHILIPPINES PEACE NEGOTIATING
PANEL, represented by its Chairman RODOLFO C. GARCIA, and the MORO ISLAMIC
LIBERATION FRONT PEACE NEGOTIATING PANEL, represented by its Chairman
CITY GOVERNMENT OF ZAMBOANGA, as represented by HON. CELSO L. LOBREGAT,
MOHAGHER IQBAL, respondents.
City Mayor of Zamboanga, and in his personal capacity as resident of the City of
Zamboanga, Rep. MA. ISABELLE G. CLIMACO, District 1, and Rep. ERICO BASILIO A.
FABIAN, District 2, City of Zamboanga, petitioners, x--------------------------------------------x
vs.
THE GOVERNMENT OF THE REPUBLIC OF THE PHILIPPINES PEACE NEGOTIATING
FRANKLIN M. DRILON and ADEL ABBAS TAMANO, petitioners-in-intervention.
PANEL (GRP), as represented by RODOLFO C. GARCIA, LEAH ARMAMENTO, SEDFREY
CANDELARIA, MARK RYAN SULLIVAN and HERMOGENES ESPERON, in his capacity as
the Presidential Adviser on Peace Process,respondents. x--------------------------------------------x

x--------------------------------------------x SEN. MANUEL A. ROXAS, petitioners-in-intervention.

G.R. No. 183893 October 14, 2008 x--------------------------------------------x

THE CITY OF ILIGAN, duly represented by CITY MAYOR LAWRENCE LLUCH MUNICIPALITY OF LINAMON duly represented by its Municipal Mayor NOEL N.
CRUZ, petitioner, DEANO, petitioners-in-intervention,
vs.
THE GOVERNMENT OF THE REPUBLIC OF THE PHILIPPINES PEACE PANEL ON
ANCESTRAL DOMAIN (GRP), represented by SEC. RODOLFO GARCIA, ATTY. LEAH x--------------------------------------------x
ARMAMENTO, ATTY. SEDFREY CANDELARIA, MARK RYAN SULLIVAN; GEN.
HERMOGENES ESPERON, JR., in his capacity as the present and duly appointed THE CITY OF ISABELA, BASILAN PROVINCE, represented by MAYOR CHERRYLYN P.
Presidential Adviser on the Peace Process; and/or SEC. EDUARDO ERMITA, in his SANTOS-AKBAR,petitioners-in-intervention.
capacity as Executive Secretary. respondents.
x--------------------------------------------x
x--------------------------------------------x
THE PROVINCE OF SULTAN KUDARAT, rep. by HON. SUHARTO T. MANGUDADATU, in
G.R. No. 183951 October 14, 2008 his capacity as Provincial Governor and a resident of the Province of Sultan
Kudarat, petitioner-in-intervention.
THE PROVINCIAL GOVERNMENT OF ZAMBOANGA DEL NORTE, as represented by HON.
ROLANDO E. YEBES, in his capacity as Provincial Governor, HON. FRANCIS H. OLVIS, in x-------------------------------------------x
his capacity as Vice-Governor and Presiding Officer of the Sangguniang Panlalawigan,
HON. CECILIA JALOSJOS CARREON, Congresswoman, 1st Congressional District, HON.
CESAR G. JALOSJOS, Congressman, 3rd Congressional District, and Members of the RUY ELIAS LOPEZ, for and in his own behalf and on behalf of Indigenous Peoples in
Sangguniang Panlalawigan of the Province of Zamboanga del Norte, namely, HON. SETH Mindanao Not Belonging to the MILF, petitioner-in-intervention.
FREDERICK P. JALOSJOS, HON. FERNANDO R. CABIGON, JR., HON. ULDARICO M.
x--------------------------------------------x The MOA-AD was preceded by a long process of negotiation and the concluding of several prior
agreements between the two parties beginning in 1996, when the GRP-MILF peace negotiations
began. On July 18, 1997, the GRP and MILF Peace Panels signed the Agreement on General
CARLO B. GOMEZ, GERARDO S. DILIG, NESARIO G. AWAT, JOSELITO C. ALISUAG and
Cessation of Hostilities. The following year, they signed the General Framework of Agreement of
RICHALEX G. JAGMIS, as citizens and residents of Palawan, petitioners-in-intervention.
Intent on August 27, 1998.

x--------------------------------------------x
The Solicitor General, who represents respondents, summarizes the MOA-AD by stating that the
same contained, among others, the commitment of the parties to pursue peace negotiations,
MARINO RIDAO and KISIN BUXANI, petitioners-in-intervention. protect and respect human rights, negotiate with sincerity in the resolution and pacific settlement
of the conflict, and refrain from the use of threat or force to attain undue advantage while the
peace negotiations on the substantive agenda are on-going.2
x--------------------------------------------x

Early on, however, it was evident that there was not going to be any smooth sailing in the GRP-
MUSLIM LEGAL ASSISTANCE FOUNDATION, INC (MUSLAF), respondent-in-intervention.
MILF peace process. Towards the end of 1999 up to early 2000, the MILF attacked a number of
municipalities in Central Mindanao and, in March 2000, it took control of the town hall of
x--------------------------------------------x Kauswagan, Lanao del Norte.3 In response, then President Joseph Estrada declared and carried
out an "all-out-war" against the MILF.
MUSLIM MULTI-SECTORAL MOVEMENT FOR PEACE & DEVELOPMENT
(MMMPD), respondent-in-intervention. When President Gloria Macapagal-Arroyo assumed office, the military offensive against the
MILF was suspended and the government sought a resumption of the peace talks. The MILF,
according to a leading MILF member, initially responded with deep reservation, but when
x--------------------------------------------x President Arroyo asked the Government of Malaysia through Prime Minister Mahathir
Mohammad to help convince the MILF to return to the negotiating table, the MILF convened its
DECISION Central Committee to seriously discuss the matter and, eventually, decided to meet with the
GRP.4
CARPIO MORALES, J.:
The parties met in Kuala Lumpur on March 24, 2001, with the talks being facilitated by the
Malaysian government, the parties signing on the same date the Agreement on the General
Subject of these consolidated cases is the extent of the powers of the President in pursuing the Framework for the Resumption of Peace Talks Between the GRP and the MILF. The MILF
peace process.While the facts surrounding this controversy center on the armed conflict in thereafter suspended all its military actions.5
Mindanao between the government and the Moro Islamic Liberation Front (MILF), the legal issue
involved has a bearing on all areas in the country where there has been a long-standing armed
conflict. Yet again, the Court is tasked to perform a delicate balancing act. It must Formal peace talks between the parties were held in Tripoli, Libya from June 20-22, 2001, the
uncompromisingly delineate the bounds within which the President may lawfully exercise her outcome of which was the GRP-MILF Tripoli Agreement on Peace (Tripoli Agreement 2001)
discretion, but it must do so in strict adherence to the Constitution, lest its ruling unduly restricts containing the basic principles and agenda on the following aspects of the
the freedom of action vested by that same Constitution in the Chief Executive precisely to enable negotiation: Security Aspect, Rehabilitation Aspect, and Ancestral Domain Aspect. With
her to pursue the peace process effectively. regard to the Ancestral Domain Aspect, the parties in Tripoli Agreement 2001 simply agreed
"that the same be discussed further by the Parties in their next meeting."
I. FACTUAL ANTECEDENTS OF THE PETITIONS
A second round of peace talks was held in Cyberjaya, Malaysia on August 5-7, 2001 which
ended with the signing of the Implementing Guidelines on the Security Aspect of the Tripoli
On August 5, 2008, the Government of the Republic of the Philippines (GRP) and the MILF, Agreement 2001 leading to a ceasefire status between the parties. This was followed by the
through the Chairpersons of their respective peace negotiating panels, were scheduled to sign a Implementing Guidelines on the Humanitarian Rehabilitation and Development Aspects of the
Memorandum of Agreement on the Ancestral Domain (MOA-AD) Aspect of the GRP-MILF Tripoli Agreement 2001, which was signed on May 7, 2002 at Putrajaya, Malaysia. Nonetheless,
Tripoli Agreement on Peace of 2001 in Kuala Lumpur, Malaysia. there were many incidence of violence between government forces and the MILF from 2002 to
2003.
The MILF is a rebel group which was established in March 1984 when, under the leadership of
the late Salamat Hashim, it splintered from the Moro National Liberation Front (MNLF) then Meanwhile, then MILF Chairman Salamat Hashim passed away on July 13, 2003 and he was
headed by Nur Misuari, on the ground, among others, of what Salamat perceived to be the replaced by Al Haj Murad, who was then the chief peace negotiator of the MILF. Murad's
manipulation of the MNLF away from an Islamic basis towards Marxist-Maoist orientations.1 position as chief peace negotiator was taken over by Mohagher Iqbal.6

The signing of the MOA-AD between the GRP and the MILF was not to materialize, however, for In 2005, several exploratory talks were held between the parties in Kuala Lumpur, eventually
upon motion of petitioners, specifically those who filed their cases before the scheduled signing leading to the crafting of the draft MOA-AD in its final form, which, as mentioned, was set to be
of the MOA-AD, this Court issued a Temporary Restraining Order enjoining the GRP from signed last August 5, 2008.
signing the same.
II. STATEMENT OF THE PROCEEDINGS Cherrylyn Santos-Akbar, the Province of Sultan Kudarat22 and Gov. Suharto Mangudadatu, the
Municipality of Linamon in Lanao del Norte,23 Ruy Elias Lopez of Davao City and of the Bagobo
tribe, Sangguniang Panlungsod member Marino Ridao and businessman Kisin Buxani, both of
Before the Court is what is perhaps the most contentious "consensus" ever embodied in an
Cotabato City; and lawyers Carlo Gomez, Gerardo Dilig, Nesario Awat, Joselito Alisuag,
instrument - the MOA-AD which is assailed principally by the present petitions bearing docket
Richalex Jagmis, all of Palawan City. The Muslim Legal Assistance Foundation, Inc. (Muslaf)
numbers 183591, 183752, 183893, 183951 and 183962.
and the Muslim Multi-Sectoral Movement for Peace and Development (MMMPD) filed their
respective Comments-in-Intervention.
Commonly impleaded as respondents are the GRP Peace Panel on Ancestral Domain 7 and the
Presidential Adviser on the Peace Process (PAPP) Hermogenes Esperon, Jr.
By subsequent Resolutions, the Court ordered the consolidation of the petitions. Respondents
filed Comments on the petitions, while some of petitioners submitted their respective Replies.
On July 23, 2008, the Province of North Cotabato8 and Vice-Governor Emmanuel Piñol filed a
petition, docketed as G.R. No. 183591, for Mandamus and Prohibition with Prayer for the
Respondents, by Manifestation and Motion of August 19, 2008, stated that the Executive
Issuance of Writ of Preliminary Injunction and Temporary Restraining Order. 9 Invoking the right
Department shall thoroughly review the MOA-AD and pursue further negotiations to address the
to information on matters of public concern, petitioners seek to compel respondents to disclose
issues hurled against it, and thus moved to dismiss the cases. In the succeeding exchange of
and furnish them the complete and official copies of the MOA-AD including its attachments, and
pleadings, respondents' motion was met with vigorous opposition from petitioners.
to prohibit the slated signing of the MOA-AD, pending the disclosure of the contents of the MOA-
AD and the holding of a public consultation thereon. Supplementarily, petitioners pray that the
MOA-AD be declared unconstitutional.10 The cases were heard on oral argument on August 15, 22 and 29, 2008 that tackled the
following principal issues:
This initial petition was followed by another one, docketed as G.R. No. 183752, also for
Mandamus and Prohibition11 filed by the City of Zamboanga,12 Mayor Celso Lobregat, Rep. Ma. 1. Whether the petitions have become moot and academic
Isabelle Climaco and Rep. Erico Basilio Fabian who likewise pray for similar injunctive reliefs.
Petitioners herein moreover pray that the City of Zamboanga be excluded from the Bangsamoro
(i) insofar as the mandamus aspect is concerned, in view of the disclosure
Homeland and/or Bangsamoro Juridical Entity and, in the alternative, that the MOA-AD be
of official copies of the final draft of the Memorandum of Agreement (MOA);
declared null and void.
and

By Resolution of August 4, 2008, the Court issued a Temporary Restraining Order commanding
(ii) insofar as the prohibition aspect involving the Local Government Units is
and directing public respondents and their agents to cease and desist from formally signing the
concerned, if it is considered that consultation has become fait
MOA-AD.13 The Court also required the Solicitor General to submit to the Court and petitioners
accompli with the finalization of the draft;
the official copy of the final draft of the MOA-AD,14 to which she complied.15

2. Whether the constitutionality and the legality of the MOA is ripe for adjudication;
Meanwhile, the City of Iligan16 filed a petition for Injunction and/or Declaratory Relief, docketed
as G.R. No. 183893, praying that respondents be enjoined from signing the MOA-AD or, if the
same had already been signed, from implementing the same, and that the MOA-AD be declared 3. Whether respondent Government of the Republic of the Philippines Peace Panel
unconstitutional. Petitioners herein additionally implead Executive Secretary Eduardo Ermita as committed grave abuse of discretion amounting to lack or excess of jurisdiction when
respondent. it negotiated and initiated the MOA vis-à-vis ISSUES Nos. 4 and 5;

The Province of Zamboanga del Norte,17 Governor Rolando Yebes, Vice-Governor Francis 4. Whether there is a violation of the people's right to information on matters of public
Olvis, Rep. Cecilia Jalosjos-Carreon, Rep. Cesar Jalosjos, and the members18 of the concern (1987 Constitution, Article III, Sec. 7) under a state policy of full disclosure of
Sangguniang Panlalawigan of Zamboanga del Norte filed on August 15, 2008 a petition for all its transactions involving public interest (1987 Constitution, Article II, Sec. 28)
Certiorari, Mandamus and Prohibition,19 docketed as G.R. No. 183951. They pray, inter alia, that including public consultation under Republic Act No. 7160 (LOCAL GOVERNMENT
the MOA-AD be declared null and void and without operative effect, and that respondents be CODE OF 1991)[;]
enjoined from executing the MOA-AD.
If it is in the affirmative, whether prohibition under Rule 65 of the 1997 Rules of Civil
On August 19, 2008, Ernesto Maceda, Jejomar Binay, and Aquilino Pimentel III filed a petition Procedure is an appropriate remedy;
for Prohibition,20docketed as G.R. No. 183962, praying for a judgment prohibiting and
permanently enjoining respondents from formally signing and executing the MOA-AD and or any
other agreement derived therefrom or similar thereto, and nullifying the MOA-AD for being 5. Whether by signing the MOA, the Government of the Republic of the Philippines
unconstitutional and illegal. Petitioners herein additionally implead as respondent the MILF would be BINDING itself
Peace Negotiating Panel represented by its Chairman Mohagher Iqbal.
a) to create and recognize the Bangsamoro Juridical Entity (BJE) as a
Various parties moved to intervene and were granted leave of court to file their petitions- separate state, or a juridical, territorial or political subdivision not recognized
by law;
/comments-in-intervention. Petitioners-in-Intervention include Senator Manuel A. Roxas, former
Senate President Franklin Drilon and Atty. Adel Tamano, the City of Isabela21 and Mayor
b) to revise or amend the Constitution and existing laws to conform to the ineffective.27 This way of viewing the world, however, became more complex through the
MOA; centuries as the Islamic world became part of the international community of nations.

c) to concede to or recognize the claim of the Moro Islamic Liberation Front As Muslim States entered into treaties with their neighbors, even with distant States and inter-
for ancestral domain in violation of Republic Act No. 8371 (THE governmental organizations, the classical division of the world into dar-ul-Islam and dar-ul-
INDIGENOUS PEOPLES RIGHTS ACT OF 1997), particularly Section 3(g) harb eventually lost its meaning. New terms were drawn up to describe novel ways of perceiving
& Chapter VII (DELINEATION, RECOGNITION OF ANCESTRAL non-Muslim territories. For instance, areas like dar-ul-mua'hada (land of compact) and dar-ul-
DOMAINS)[;] sulh (land of treaty) referred to countries which, though under a secular regime, maintained
peaceful and cooperative relations with Muslim States, having been bound to each other by
treaty or agreement. Dar-ul-aman (land of order), on the other hand, referred to countries which,
If in the affirmative, whether the Executive Branch has the authority to so bind the
though not bound by treaty with Muslim States, maintained freedom of religion for Muslims. 28
Government of the Republic of the Philippines;

It thus appears that the "compact rights entrenchment" emanating from the regime of dar-ul-
6. Whether the inclusion/exclusion of the Province of North Cotabato, Cities of
mua'hada and dar-ul-sulh simply refers to all other agreements between the MILF and the
Zamboanga, Iligan and Isabela, and the Municipality of Linamon, Lanao del Norte
Philippine government - the Philippines being the land of compact and peace agreement - that
in/from the areas covered by the projected Bangsamoro Homeland is a justiciable
partake of the nature of a treaty device, "treaty" being broadly defined as "any solemn
question; and
agreement in writing that sets out understandings, obligations, and benefits for both parties
which provides for a framework that elaborates the principles declared in the [MOA-AD]."29
7. Whether desistance from signing the MOA derogates any prior valid commitments
of the Government of the Republic of the Philippines.24
The MOA-AD states that the Parties "HAVE AGREED AND ACKNOWLEDGED AS FOLLOWS,"
and starts with its main body.
The Court, thereafter, ordered the parties to submit their respective Memoranda. Most of the
parties submitted their memoranda on time.
The main body of the MOA-AD is divided into four strands, namely, Concepts and
Principles, Territory, Resources, and Governance.
III. OVERVIEW OF THE MOA-AD
A. CONCEPTS AND PRINCIPLES
As a necessary backdrop to the consideration of the objections raised in the subject five
petitions and six petitions-in-intervention against the MOA-AD, as well as the two comments-in-
This strand begins with the statement that it is "the birthright of all Moros and all Indigenous
intervention in favor of the MOA-AD, the Court takes an overview of the MOA.
peoples of Mindanao to identify themselves and be accepted as ‘Bangsamoros.'" It defines
"Bangsamoro people" as the natives or original inhabitants of Mindanao and its adjacent
The MOA-AD identifies the Parties to it as the GRP and the MILF. islands including Palawan and the Sulu archipelago at the time of conquest or colonization, and
their descendants whether mixed or of full blood, including their spouses.30
Under the heading "Terms of Reference" (TOR), the MOA-AD includes not only four earlier
agreements between the GRP and MILF, but also two agreements between the GRP and the Thus, the concept of "Bangsamoro," as defined in this strand of the MOA-AD, includes not only
MNLF: the 1976 Tripoli Agreement, and the Final Peace Agreement on the Implementation of "Moros" as traditionally understood even by Muslims,31 but all indigenous peoples of Mindanao
the 1976 Tripoli Agreement, signed on September 2, 1996 during the administration of President and its adjacent islands. The MOA-AD adds that the freedom of choice of indigenous peoples
Fidel Ramos. shall be respected. What this freedom of choice consists in has not been specifically defined.

The MOA-AD also identifies as TOR two local statutes - the organic act for the Autonomous The MOA-AD proceeds to refer to the "Bangsamoro homeland," the ownership of which is
Region in Muslim Mindanao (ARMM)25 and the Indigenous Peoples Rights Act (IPRA),26 and vested exclusively in the Bangsamoro people by virtue of their prior rights of occupation.32 Both
several international law instruments - the ILO Convention No. 169 Concerning Indigenous and parties to the MOA-AD acknowledge that ancestral domain does not form part of the public
Tribal Peoples in Independent Countries in relation to the UN Declaration on the Rights of the domain.33
Indigenous Peoples, and the UN Charter, among others.
The Bangsamoro people are acknowledged as having the right to self-governance, which right is
The MOA-AD includes as a final TOR the generic category of "compact rights entrenchment said to be rooted on ancestral territoriality exercised originally under the suzerain authority of
emanating from the regime of dar-ul-mua'hada (or territory under compact) and dar-ul-sulh (or their sultanates and the Pat a Pangampong ku Ranaw. The sultanates were described as states
territory under peace agreement) that partakes the nature of a treaty device." or "karajaan/kadatuan" resembling a body politic endowed with all the elements of a nation-state
in the modern sense.34
During the height of the Muslim Empire, early Muslim jurists tended to see the world through a
simple dichotomy: there was the dar-ul-Islam (the Abode of Islam) and dar-ul-harb (the Abode The MOA-AD thus grounds the right to self-governance of the Bangsamoro people on the past
of War). The first referred to those lands where Islamic laws held sway, while the second suzerain authority of the sultanates. As gathered, the territory defined as the Bangsamoro
denoted those lands where Muslims were persecuted or where Muslim laws were outlawed or homeland was ruled by several sultanates and, specifically in the case of the Maranao, by
the Pat a Pangampong ku Ranaw, a confederation of independent principalities (pangampong) C. RESOURCES
each ruled by datus and sultans, none of whom was supreme over the others.35
The MOA-AD states that the BJE is free to enter into any economic cooperation and trade
The MOA-AD goes on to describe the Bangsamoro people as "the ‘First Nation' with defined relations with foreign countries and shall have the option to establish trade missions in those
territory and with a system of government having entered into treaties of amity and commerce countries. Such relationships and understandings, however, are not to include aggression
with foreign nations." against the GRP. The BJE may also enter into environmental cooperation agreements. 46

The term "First Nation" is of Canadian origin referring to the indigenous peoples of that territory, The external defense of the BJE is to remain the duty and obligation of the Central Government.
particularly those known as Indians. In Canada, each of these indigenous peoples is equally The Central Government is also bound to "take necessary steps to ensure the BJE's
entitled to be called "First Nation," hence, all of them are usually described collectively by the participation in international meetings and events" like those of the ASEAN and the specialized
plural "First Nations."36 To that extent, the MOA-AD, by identifying the Bangsamoro people as agencies of the UN. The BJE is to be entitled to participate in Philippine official missions and
"the First Nation" - suggesting its exclusive entitlement to that designation - departs from the delegations for the negotiation of border agreements or protocols for environmental protection
Canadian usage of the term. and equitable sharing of incomes and revenues involving the bodies of water adjacent to or
between the islands forming part of the ancestral domain.47
The MOA-AD then mentions for the first time the "Bangsamoro Juridical Entity" (BJE) to which
it grants the authority and jurisdiction over the Ancestral Domain and Ancestral Lands of the With regard to the right of exploring for, producing, and obtaining all potential sources of energy,
Bangsamoro.37 petroleum, fossil fuel, mineral oil and natural gas, the jurisdiction and control thereon is to be
vested in the BJE "as the party having control within its territorial jurisdiction." This right carries
the proviso that, "in times of national emergency, when public interest so requires," the Central
B. TERRITORY
Government may, for a fixed period and under reasonable terms as may be agreed upon by
both Parties, assume or direct the operation of such resources. 48
The territory of the Bangsamoro homeland is described as the land mass as well as the
maritime, terrestrial, fluvial and alluvial domains, including the aerial domain and the
The sharing between the Central Government and the BJE of total production pertaining to
atmospheric space above it, embracing the Mindanao-Sulu-Palawan geographic region.38
natural resources is to be 75:25 in favor of the BJE.49

More specifically, the core of the BJE is defined as the present geographic area of the ARMM -
The MOA-AD provides that legitimate grievances of the Bangsamoro people arising from any
thus constituting the following areas: Lanao del Sur, Maguindanao, Sulu, Tawi-Tawi, Basilan,
unjust dispossession of their territorial and proprietary rights, customary land tenures, or their
and Marawi City. Significantly, this core also includes certain municipalities of Lanao del Norte
marginalization shall be acknowledged. Whenever restoration is no longer possible, reparation is
that voted for inclusion in the ARMM in the 2001 plebiscite.39
to be in such form as mutually determined by the Parties.50

Outside of this core, the BJE is to cover other provinces, cities, municipalities and barangays,
The BJE may modify or cancel the forest concessions, timber licenses, contracts or
which are grouped into two categories, Category A and Category B. Each of these areas is to be
agreements, mining concessions, Mineral Production and Sharing Agreements (MPSA),
subjected to a plebiscite to be held on different dates, years apart from each other. Thus,
Industrial Forest Management Agreements (IFMA), and other land tenure instruments granted
Category A areas are to be subjected to a plebiscite not later than twelve (12) months following
by the Philippine Government, including those issued by the present ARMM.51
the signing of the MOA-AD.40 Category B areas, also called "Special Intervention Areas," on the
other hand, are to be subjected to a plebiscite twenty-five (25) years from the signing of a
separate agreement - the Comprehensive Compact.41 D. GOVERNANCE

The Parties to the MOA-AD stipulate that the BJE shall have jurisdiction over all natural The MOA-AD binds the Parties to invite a multinational third-party to observe and monitor the
resources within its "internalwaters," defined as extending fifteen (15) kilometers from the implementation of the Comprehensive Compact. This compact is to embody the "details for the
coastline of the BJE area;42 that the BJE shall also have "territorial waters," which shall stretch effective enforcement" and "the mechanisms and modalities for the actual implementation" of
beyond the BJE internal waters up to the baselines of the Republic of the Philippines (RP) south the MOA-AD. The MOA-AD explicitly provides that the participation of the third party shall not in
east and south west of mainland Mindanao; and that within these territorial waters, the BJE and any way affect the status of the relationship between the Central Government and the BJE.52
the "Central Government" (used interchangeably with RP) shall exercise joint jurisdiction,
authority and management over all natural resources.43 Notably, the jurisdiction over
The "associative" relationship
the internal waters is not similarly described as "joint."
between the Central Government
and the BJE
The MOA-AD further provides for the sharing of minerals on the territorial waters between the
Central Government and the BJE, in favor of the latter, through production sharing and
The MOA-AD describes the relationship of the Central Government and the BJE as
economic cooperation agreement.44 The activities which the Parties are allowed to conduct on
"associative," characterized by shared authority and responsibility. And it states that the
the territorial waters are enumerated, among which are the exploration and utilization of natural
structure of governance is to be based on executive, legislative, judicial, and administrative
resources, regulation of shipping and fishing activities, and the enforcement of police and safety
institutions with defined powers and functions in the Comprehensive Compact.
measures.45 There is no similar provision on the sharing of minerals and allowed activities with
respect to the internal waters of the BJE.
The MOA-AD provides that its provisions requiring "amendments to the existing legal The Solicitor General argues that there is no justiciable controversy that is ripe for judicial review
framework" shall take effect upon signing of the Comprehensive Compact and upon effecting the in the present petitions, reasoning that
aforesaid amendments, with due regard to the non-derogation of prior agreements and within
the stipulated timeframe to be contained in the Comprehensive Compact. As will be discussed
The unsigned MOA-AD is simply a list of consensus points subject to further
later, much of the present controversy hangs on the legality of this provision.
negotiations and legislative enactments as well as constitutional processes aimed at
attaining a final peaceful agreement. Simply put, the MOA-AD remains to be
The BJE is granted the power to build, develop and maintain its own institutions inclusive of civil a proposal that does not automatically create legally demandable rights and
service, electoral, financial and banking, education, legislation, legal, economic, police and obligations until the list of operative acts required have been duly complied with. x x x
internal security force, judicial system and correctional institutions, the details of which shall be
discussed in the negotiation of the comprehensive compact.
xxxx

As stated early on, the MOA-AD was set to be signed on August 5, 2008 by Rodolfo Garcia and
In the cases at bar, it is respectfully submitted that this Honorable Court has no
Mohagher Iqbal, Chairpersons of the Peace Negotiating Panels of the GRP and the MILF,
authority to pass upon issues based on hypothetical or feigned constitutional problems
respectively. Notably, the penultimate paragraph of the MOA-AD identifies the signatories as
or interests with no concrete bases. Considering the preliminary character of the
"the representatives of the Parties," meaning the GRP and MILF themselves, and not merely of
MOA-AD, there are no concrete acts that could possibly violate petitioners' and
the negotiating panels.53 In addition, the signature page of the MOA-AD states that it is
intervenors' rights since the acts complained of are mere contemplated steps toward
"WITNESSED BY" Datuk Othman Bin Abd Razak, Special Adviser to the Prime Minister of
the formulation of a final peace agreement. Plainly, petitioners and intervenors'
Malaysia, "ENDORSED BY" Ambassador Sayed Elmasry, Adviser to Organization of the Islamic
perceived injury, if at all, is merely imaginary and illusory apart from being unfounded
Conference (OIC) Secretary General and Special Envoy for Peace Process in Southern
and based on mere conjectures. (Underscoring supplied)
Philippines, and SIGNED "IN THE PRESENCE OF" Dr. Albert G. Romulo, Secretary of Foreign
Affairs of RP and Dato' Seri Utama Dr. Rais Bin Yatim, Minister of Foreign Affairs, Malaysia, all
of whom were scheduled to sign the Agreement last August 5, 2008. The Solicitor General cites63 the following provisions of the MOA-AD:

Annexed to the MOA-AD are two documents containing the respective lists cum maps of the TERRITORY
provinces, municipalities, and barangays under Categories A and B earlier mentioned in the
discussion on the strand on TERRITORY.
xxxx

IV. PROCEDURAL ISSUES


2. Toward this end, the Parties enter into the following stipulations:

A. RIPENESS
xxxx

The power of judicial review is limited to actual cases or controversies. 54 Courts decline to issue
advisory opinions or to resolve hypothetical or feigned problems, or mere academic d. Without derogating from the requirements of prior agreements, the Government
questions.55 The limitation of the power of judicial review to actual cases and controversies stipulates to conduct and deliver, using all possible legal measures, within twelve (12)
defines the role assigned to the judiciary in a tripartite allocation of power, to assure that the months following the signing of the MOA-AD, a plebiscite covering the areas as
courts will not intrude into areas committed to the other branches of government.56 enumerated in the list and depicted in the map as Category A attached herein (the
"Annex"). The Annex constitutes an integral part of this framework agreement. Toward
this end, the Parties shall endeavor to complete the negotiations and resolve all
An actual case or controversy involves a conflict of legal rights, an assertion of opposite legal outstanding issues on the Comprehensive Compact within fifteen (15) months from
claims, susceptible of judicial resolution as distinguished from a hypothetical or abstract the signing of the MOA-AD.
difference or dispute. There must be a contrariety of legal rights that can be interpreted and
enforced on the basis of existing law and jurisprudence.57 The Court can decide the
constitutionality of an act or treaty only when a proper case between opposing parties is xxxx
submitted for judicial determination.58
GOVERNANCE
Related to the requirement of an actual case or controversy is the requirement of ripeness. A
question is ripe for adjudication when the act being challenged has had a direct adverse effect xxxx
on the individual challenging it.59 For a case to be considered ripe for adjudication, it is a
prerequisite that something had then been accomplished or performed by either branch before a
court may come into the picture,60 and the petitioner must allege the existence of an immediate 7. The Parties agree that mechanisms and modalities for the actual implementation of
or threatened injury to itself as a result of the challenged action.61 He must show that he has this MOA-AD shall be spelt out in the Comprehensive Compact to mutually take such
sustained or is immediately in danger of sustaining some direct injury as a result of the act steps to enable it to occur effectively.
complained of.62
Any provisions of the MOA-AD requiring amendments to the existing legal framework informing them of the proceedings. As will be discussed in greater detail later, such omission, by
shall come into force upon the signing of a Comprehensive Compact and upon itself, constitutes a departure by respondents from their mandate under E.O. No. 3.
effecting the necessary changes to the legal framework with due regard to non-
derogation of prior agreements and within the stipulated timeframe to be contained in
Furthermore, the petitions allege that the provisions of the MOA-AD violate the Constitution. The
the Comprehensive Compact.64 (Underscoring supplied)
MOA-AD provides that "any provisions of the MOA-AD requiring amendments to the existing
legal framework shall come into force upon the signing of a Comprehensive Compact and upon
The Solicitor General's arguments fail to persuade. effecting the necessary changes to the legal framework," implying an amendment of the
Constitution to accommodate the MOA-AD. This stipulation, in effect, guaranteed to the MILF
the amendment of the Constitution. Such act constitutes another violation of its authority. Again,
Concrete acts under the MOA-AD are not necessary to render the present controversy ripe.
these points will be discussed in more detail later.
In Pimentel, Jr. v. Aguirre,65 this Court held:

As the petitions allege acts or omissions on the part of respondent that exceed their authority,
x x x [B]y the mere enactment of the questioned law or the approval of the challenged
by violating their duties under E.O. No. 3 and the provisions of the Constitution and statutes, the
action, the dispute is said to have ripened into a judicial controversy even without any
petitions make a prima facie case for Certiorari, Prohibition, and Mandamus, and an actual case
other overt act. Indeed, even a singular violation of the Constitution and/or the law is
or controversy ripe for adjudication exists. When an act of a branch of government is
enough to awaken judicial duty.
seriously alleged to have infringed the Constitution, it becomes not only the right but in
fact the duty of the judiciary to settle the dispute.77
xxxx
B. LOCUS STANDI
By the same token, when an act of the President, who in our constitutional scheme is
a coequal of Congress, is seriously alleged to have infringed the Constitution and the
For a party to have locus standi, one must allege "such a personal stake in the outcome of the
laws x x x settling the dispute becomes the duty and the responsibility of the courts. 66
controversy as to assure that concrete adverseness which sharpens the presentation of issues
upon which the court so largely depends for illumination of difficult constitutional questions." 78
In Santa Fe Independent School District v. Doe,67 the United States Supreme Court held that the
challenge to the constitutionality of the school's policy allowing student-led prayers and
Because constitutional cases are often public actions in which the relief sought is likely to affect
speeches before games was ripe for adjudication, even if no public prayer had yet been led
other persons, a preliminary question frequently arises as to this interest in the constitutional
under the policy, because the policy was being challenged as unconstitutional on its face. 68
question raised.79

That the law or act in question is not yet effective does not negate ripeness. For example,
When suing as a citizen, the person complaining must allege that he has been or is about to be
in New York v. United States,69 decided in 1992, the United States Supreme Court held that the
denied some right or privilege to which he is lawfully entitled or that he is about to be subjected
action by the State of New York challenging the provisions of the Low-Level Radioactive Waste
to some burdens or penalties by reason of the statute or act complained of. 80 When the issue
Policy Act was ripe for adjudication even if the questioned provision was not to take effect until
concerns a public right, it is sufficient that the petitioner is a citizen and has an interest in the
January 1, 1996, because the parties agreed that New York had to take immediate action to
execution of the laws.81
avoid the provision's consequences.70

For a taxpayer, one is allowed to sue where there is an assertion that public funds are illegally
The present petitions pray for Certiorari,71 Prohibition, and Mandamus. Certiorari and Prohibition
disbursed or deflected to an illegal purpose, or that there is a wastage of public funds through
are remedies granted by law when any tribunal, board or officer has acted, in the case of
the enforcement of an invalid or unconstitutional law.82 The Court retains discretion whether or
certiorari, or is proceeding, in the case of prohibition, without or in excess of its jurisdiction or
not to allow a taxpayer's suit.83
with grave abuse of discretion amounting to lack or excess of jurisdiction. 72 Mandamus is a
remedy granted by law when any tribunal, corporation, board, officer or person unlawfully
neglects the performance of an act which the law specifically enjoins as a duty resulting from an In the case of a legislator or member of Congress, an act of the Executive that injures the
office, trust, or station, or unlawfully excludes another from the use or enjoyment of a right or institution of Congress causes a derivative but nonetheless substantial injury that can be
office to which such other is entitled.73 Certiorari, Mandamus and Prohibition are appropriate questioned by legislators. A member of the House of Representatives has standing to maintain
remedies to raise constitutional issues and to review and/or prohibit/nullify, when proper, acts of inviolate the prerogatives, powers and privileges vested by the Constitution in his office.84
legislative and executive officials.74
An organization may be granted standing to assert the rights of its members, 85 but the mere
The authority of the GRP Negotiating Panel is defined by Executive Order No. 3 (E.O. No. 3), invocation by the Integrated Bar of the Philippines or any member of the legal profession of the
issued on February 28, 2001.75 The said executive order requires that "[t]he government's policy duty to preserve the rule of law does not suffice to clothe it with standing.86
framework for peace, including the systematic approach and the administrative structure for
carrying out the comprehensive peace process x x x be governed by this Executive Order." 76
As regards a local government unit (LGU), it can seek relief in order to protect or vindicate an
interest of its own, and of the other LGUs.87
The present petitions allege that respondents GRP Panel and PAPP Esperon drafted the terms
of the MOA-AD without consulting the local government units or communities affected, nor
Intervenors, meanwhile, may be given legal standing upon showing of facts that satisfy the Intervening respondents Muslim Multi-Sectoral Movement for Peace and Development, an
requirements of the law authorizing intervention,88 such as a legal interest in the matter in advocacy group for justice and the attainment of peace and prosperity in Muslim Mindanao;
litigation, or in the success of either of the parties. and Muslim Legal Assistance Foundation Inc., a non-government organization of Muslim
lawyers, allege that they stand to be benefited or prejudiced, as the case may be, in the
resolution of the petitions concerning the MOA-AD, and prays for the denial of the petitions on
In any case, the Court has discretion to relax the procedural technicality on locus standi, given
the grounds therein stated. Such legal interest suffices to clothe them with standing.
the liberal attitude it has exercised, highlighted in the case of David v. Macapagal-
Arroyo,89 where technicalities of procedure were brushed aside, the constitutional issues raised
being of paramount public interest or of transcendental importance deserving the attention of the B. MOOTNESS
Court in view of their seriousness, novelty and weight as precedents.90 The Court's forbearing
stance on locus standi on issues involving constitutional issues has for its purpose the protection
Respondents insist that the present petitions have been rendered moot with the satisfaction of
of fundamental rights.
all the reliefs prayed for by petitioners and the subsequent pronouncement of the Executive
Secretary that "[n]o matter what the Supreme Court ultimately decides[,] the government will not
In not a few cases, the Court, in keeping with its duty under the Constitution to determine sign the MOA."92
whether the other branches of government have kept themselves within the limits of the
Constitution and the laws and have not abused the discretion given them, has brushed aside
In lending credence to this policy decision, the Solicitor General points out that the President had
technical rules of procedure.91
already disbanded the GRP Peace Panel.93

In the petitions at bar, petitioners Province of North Cotabato (G.R. No. 183591) Province of
In David v. Macapagal-Arroyo,94 this Court held that the "moot and academic" principle not being
Zamboanga del Norte (G.R. No. 183951), City of Iligan (G.R. No. 183893) and City of
a magical formula that automatically dissuades courts in resolving a case, it will decide cases,
Zamboanga (G.R. No. 183752) and petitioners-in-intervention Province of Sultan
otherwise moot and academic, if it finds that (a) there is a grave violation of the
Kudarat, City of Isabela and Municipality of Linamon have locus standi in view of the direct
Constitution;95 (b) the situation is of exceptional character and paramount public interest is
and substantial injury that they, as LGUs, would suffer as their territories, whether in whole or in
involved;96 (c) the constitutional issue raised requires formulation of controlling principles to
part, are to be included in the intended domain of the BJE. These petitioners allege that they did
guide the bench, the bar, and the public;97 and (d) the case is capable of repetition yet evading
not vote for their inclusion in the ARMM which would be expanded to form the BJE territory.
review.98
Petitioners' legal standing is thus beyond doubt.

Another exclusionary circumstance that may be considered is where there is


In G.R. No. 183962, petitioners Ernesto Maceda, Jejomar Binay and Aquilino Pimentel
a voluntary cessation of the activity complained of by the defendant or doer. Thus, once a suit is
III would have no standing as citizens and taxpayers for their failure to specify that they would be
filed and the doer voluntarily ceases the challenged conduct, it does not automatically deprive
denied some right or privilege or there would be wastage of public funds. The fact that they are a
the tribunal of power to hear and determine the case and does not render the case moot
former Senator, an incumbent mayor of Makati City, and a resident of Cagayan de Oro,
especially when the plaintiff seeks damages or prays for injunctive relief against the possible
respectively, is of no consequence. Considering their invocation of the transcendental
recurrence of the violation.99
importance of the issues at hand, however, the Court grants them standing.

The present petitions fall squarely into these exceptions to thus thrust them into the domain of
Intervenors Franklin Drilon and Adel Tamano, in alleging their standing as taxpayers, assert
judicial review. The grounds cited above in David are just as applicable in the present cases as
that government funds would be expended for the conduct of an illegal and unconstitutional
they were, not only in David, but also in Province of Batangas v. Romulo100 and Manalo v.
plebiscite to delineate the BJE territory. On that score alone, they can be given legal standing.
Calderon101 where the Court similarly decided them on the merits, supervening events that
Their allegation that the issues involved in these petitions are of "undeniable transcendental
would ordinarily have rendered the same moot notwithstanding.
importance" clothes them with added basis for their personality to intervene in these petitions.

Petitions not mooted


With regard to Senator Manuel Roxas, his standing is premised on his being a member of the
Senate and a citizen to enforce compliance by respondents of the public's constitutional right to
be informed of the MOA-AD, as well as on a genuine legal interest in the matter in litigation, or in Contrary then to the asseverations of respondents, the non-signing of the MOA-AD and the
the success or failure of either of the parties. He thus possesses the requisite standing as an eventual dissolution of the GRP Peace Panel did not moot the present petitions. It bears
intervenor. emphasis that the signing of the MOA-AD did not push through due to the Court's issuance of a
Temporary Restraining Order.
With respect to Intervenors Ruy Elias Lopez, as a former congressman of the 3rd district of
Davao City, a taxpayer and a member of the Bagobo tribe; Carlo B. Gomez, et al., as members Contrary too to respondents' position, the MOA-AD cannot be considered a mere "list of
of the IBP Palawan chapter, citizens and taxpayers; Marino Ridao, as taxpayer, resident and consensus points," especially given its nomenclature, the need to have it signed or
member of the Sangguniang Panlungsod of Cotabato City; and Kisin Buxani, as taxpayer, they initialed by all the parties concerned on August 5, 2008, and the far-reaching Constitutional
failed to allege any proper legal interest in the present petitions. Just the same, the Court implications of these "consensus points," foremost of which is the creation of the BJE.
exercises its discretion to relax the procedural technicality on locus standi given the paramount
public interest in the issues at hand.
In fact, as what will, in the main, be discussed, there is a commitment on the part of
respondents to amend and effect necessary changes to the existing legal framework for
certain provisions of the MOA-AD to take effect. Consequently, the present petitions are not Respondents invite the Court's attention to the separate opinion of then Chief Justice Artemio
confined to the terms and provisions of the MOA-AD, but to other on- Panganiban in Sanlakas v. Reyes104 in which he stated that the doctrine of "capable of repetition
going and future negotiations and agreements necessary for its realization. The petitions have yet evading review" can override mootness, "provided the party raising it in a proper case has
not, therefore, been rendered moot and academic simply by the public disclosure of the MOA- been and/or continue to be prejudiced or damaged as a direct result of their issuance." They
AD,102 the manifestation that it will not be signed as well as the disbanding of the GRP Panel not contend that the Court must have jurisdiction over the subject matter for the doctrine to be
withstanding. invoked.

Petitions are imbued with paramount public interest The present petitions all contain prayers for Prohibition over which this Court exercises original
jurisdiction. While G.R. No. 183893 (City of Iligan v. GRP) is a petition for Injunction and
Declaratory Relief, the Court will treat it as one for Prohibition as it has far reaching implications
There is no gainsaying that the petitions are imbued with paramount public interest, involving a
and raises questions that need to be resolved.105 At all events, the Court has jurisdiction over
significant part of the country's territory and the wide-ranging political modifications of affected
most if not the rest of the petitions.
LGUs. The assertion that the MOA-AD is subject to further legal enactments including
possible Constitutional amendments more than ever provides impetus for the Court
to formulate controlling principles to guide the bench, the bar, the public and, in this Indeed, the present petitions afford a proper venue for the Court to again apply the doctrine
case, the government and its negotiating entity. immediately referred to as what it had done in a number of landmark cases. 106 There is
a reasonable expectation that petitioners, particularly the Provinces of North Cotabato,
Zamboanga del Norte and Sultan Kudarat, the Cities of Zamboanga, Iligan and Isabela, and the
Respondents cite Suplico v. NEDA, et al.103 where the Court did not "pontificat[e] on issues
Municipality of Linamon, will again be subjected to the same problem in the future as
which no longer legitimately constitute an actual case or controversy [as this] will do more harm
respondents' actions are capable of repetition, in another or any form.
than good to the nation as a whole."

It is with respect to the prayers for Mandamus that the petitions have become moot, respondents
The present petitions must be differentiated from Suplico. Primarily, in Suplico, what was
having, by Compliance of August 7, 2008, provided this Court and petitioners with official copies
assailed and eventually cancelled was a stand-alone government procurement contract for a
of the final draft of the MOA-AD and its annexes. Too, intervenors have been furnished, or have
national broadband network involving a one-time contractual relation between two parties-the
procured for themselves, copies of the MOA-AD.
government and a private foreign corporation. As the issues therein involved specific
government procurement policies and standard principles on contracts, the majority opinion
in Suplico found nothing exceptional therein, the factual circumstances being peculiar only to the V. SUBSTANTIVE ISSUES
transactions and parties involved in the controversy.
As culled from the Petitions and Petitions-in-Intervention, there are basically two SUBSTANTIVE
The MOA-AD is part of a series of agreements issues to be resolved, one relating to the manner in which the MOA-AD was negotiated and
finalized, the other relating to its provisions, viz:
In the present controversy, the MOA-AD is a significant part of a series of
agreements necessary to carry out the Tripoli Agreement 2001. The MOA-AD which dwells on 1. Did respondents violate constitutional and statutory provisions on public consultation and the
the Ancestral Domain Aspect of said Tripoli Agreement is the third such component to be right to information when they negotiated and later initialed the MOA-AD?
undertaken following the implementation of the Security Aspect in August 2001 and
the Humanitarian, Rehabilitation and Development Aspect in May 2002.
2. Do the contents of the MOA-AD violate the Constitution and the laws?

Accordingly, even if the Executive Secretary, in his Memorandum of August 28, 2008 to the
ON THE FIRST SUBSTANTIVE ISSUE
Solicitor General, has stated that "no matter what the Supreme Court ultimately decides[,] the
government will not sign the MOA[-AD]," mootness will not set in in light of the terms of the
Tripoli Agreement 2001. Petitioners invoke their constitutional right to information on matters of public concern, as
provided in Section 7, Article III on the Bill of Rights:
Need to formulate principles-guidelines
Sec. 7. The right of the people to information on matters of public concern shall be
recognized. Access to official records, and to documents, and papers pertaining to
Surely, the present MOA-AD can be renegotiated or another one will be drawn up to carry out
official acts, transactions, or decisions, as well as to government research data used
the Ancestral Domain Aspect of the Tripoli Agreement 2001, in another or in any form, which
as basis for policy development, shall be afforded the citizen, subject to such
could contain similar or significantly drastic provisions. While the Court notes the word of the
limitations as may be provided by law.107
Executive Secretary that the government "is committed to securing an agreement that is both
constitutional and equitable because that is the only way that long-lasting peace can be
assured," it is minded to render a decision on the merits in the present petitions to formulate As early as 1948, in Subido v. Ozaeta,108 the Court has recognized the statutory right to examine
controlling principles to guide the bench, the bar, the public and, most especially, the and inspect public records, a right which was eventually accorded constitutional status.
government in negotiating with the MILF regarding Ancestral Domain.
The right of access to public documents, as enshrined in both the 1973 Constitution and the the Bill of Rights. We can allow neither an emasculation of a constitutional right, nor a
1987 Constitution, has been recognized as a self-executory constitutional right.109 retreat by the State of its avowed "policy of full disclosure of all its transactions
involving public interest."122 (Emphasis and italics in the original)
In the 1976 case of Baldoza v. Hon. Judge Dimaano,110 the Court ruled that access to public
records is predicated on the right of the people to acquire information on matters of public Intended as a "splendid symmetry"123 to the right to information under the Bill of Rights is the
concern since, undoubtedly, in a democracy, the pubic has a legitimate interest in matters of policy of public disclosure under Section 28, Article II of the Constitution reading:
social and political significance.
Sec. 28. Subject to reasonable conditions prescribed by law, the State adopts and
x x x The incorporation of this right in the Constitution is a recognition of the fundamental role of implements a policy of full public disclosure of all its transactions involving public
free exchange of information in a democracy. There can be no realistic perception by the public interest.124
of the nation's problems, nor a meaningful democratic decision-making if they are denied access
to information of general interest. Information is needed to enable the members of society to
The policy of full public disclosure enunciated in above-quoted Section 28 complements the right
cope with the exigencies of the times. As has been aptly observed: "Maintaining the flow of such
of access to information on matters of public concern found in the Bill of Rights. The right to
information depends on protection for both its acquisition and its dissemination since, if either
information guarantees the right of the people to demand information, while Section 28
process is interrupted, the flow inevitably ceases." x x x111
recognizes the duty of officialdom to give information even if nobody demands.125

In the same way that free discussion enables members of society to cope with the exigencies of
The policy of public disclosure establishes a concrete ethical principle for the conduct of public
their time, access to information of general interest aids the people in democratic decision-
affairs in a genuinely open democracy, with the people's right to know as the centerpiece. It is a
making by giving them a better perspective of the vital issues confronting the nation112 so that
mandate of the State to be accountable by following such policy. 126 These provisions are vital to
they may be able to criticize and participate in the affairs of the government in a responsible,
the exercise of the freedom of expression and essential to hold public officials at all times
reasonable and effective manner. It is by ensuring an unfettered and uninhibited exchange of
accountable to the people.127
ideas among a well-informed public that a government remains responsive to the changes
desired by the people.113
Whether Section 28 is self-executory, the records of the deliberations of the Constitutional
Commission so disclose:
The MOA-AD is a matter of public concern

MR. SUAREZ. And since this is not self-executory, this policy will not be enunciated or
That the subject of the information sought in the present cases is a matter of public
will not be in force and effect until after Congress shall have provided it.
concern114 faces no serious challenge. In fact, respondents admit that the MOA-AD is indeed of
public concern.115 In previous cases, the Court found that the regularity of real estate
transactions entered in the Register of Deeds,116 the need for adequate notice to the public of MR. OPLE. I expect it to influence the climate of public ethics immediately but, of
the various laws,117 the civil service eligibility of a public employee,118 the proper management of course, the implementing law will have to be enacted by Congress, Mr. Presiding
GSIS funds allegedly used to grant loans to public officials,119 the recovery of the Marcoses' Officer.128
alleged ill-gotten wealth,120 and the identity of party-list nominees,121 among others, are matters
of public concern. Undoubtedly, the MOA-AD subject of the present cases is of public
concern, involving as it does the sovereignty and territorial integrity of the State, which The following discourse, after Commissioner Hilario Davide, Jr., sought clarification on the issue,
directly affects the lives of the public at large. is enlightening.

Matters of public concern covered by the right to information include steps and negotiations MR. DAVIDE. I would like to get some clarifications on this. Mr. Presiding Officer, did I
leading to the consummation of the contract. In not distinguishing as to the executory nature or get the Gentleman correctly as having said that this is not a self-executing provision?
It would require a legislation by Congress to implement?
commercial character of agreements, the Court has categorically ruled:

x x x [T]he right to information "contemplates inclusion of negotiations leading to MR. OPLE. Yes. Originally, it was going to be self-executing, but I accepted an
amendment from Commissioner Regalado, so that the safeguards on national interest
the consummation of the transaction." Certainly, a consummated contract is not a
requirement for the exercise of the right to information. Otherwise, the people can are modified by the clause "as may be provided by law"
never exercise the right if no contract is consummated, and if one is consummated, it
may be too late for the public to expose its defects. MR. DAVIDE. But as worded, does it not mean that this will immediately take
effect and Congress may provide for reasonable safeguards on the sole ground
Requiring a consummated contract will keep the public in the dark until the contract, national interest?
which may be grossly disadvantageous to the government or even illegal,
becomes fait accompli. This negates the State policy of full transparency on matters of MR. OPLE. Yes. I think so, Mr. Presiding Officer, I said earlier that it should
public concern, a situation which the framers of the Constitution could not have immediately influence the climate of the conduct of public affairs but, of course,
intended. Such a requirement will prevent the citizenry from participating in the public Congress here may no longer pass a law revoking it, or if this is approved, revoking
discussion of any proposed contract, effectively truncating a basic right enshrined in this principle, which is inconsistent with this policy.129 (Emphasis supplied)
Indubitably, the effectivity of the policy of public disclosure need not await the passing of Clearly, E.O. No. 3 contemplates not just the conduct of a plebiscite to effectuate
a statute. As Congress cannot revoke this principle, it is merely directed to provide for "continuing" consultations, contrary to respondents' position that plebiscite is "more
"reasonable safeguards." The complete and effective exercise of the right to information than sufficient consultation."136
necessitates that its complementary provision on public disclosure derive the same self-
executory nature. Since both provisions go hand-in-hand, it is absurd to say that the
Further, E.O. No. 3 enumerates the functions and responsibilities of the PAPP, one of which is to
broader130 right to information on matters of public concern is already enforceable while the
"[c]onduct regular dialogues with the National Peace Forum (NPF) and other peace partners to
correlative duty of the State to disclose its transactions involving public interest is not
seek relevant information, comments, recommendations as well as to render appropriate and
enforceable until there is an enabling law. Respondents cannot thus point to the absence of an
timely reports on the progress of the comprehensive peace process."137 E.O. No. 3 mandates
implementing legislation as an excuse in not effecting such policy.
the establishment of the NPF to be "the principal forum for the PAPP to consult with and seek
advi[c]e from the peace advocates, peace partners and concerned sectors of society on both
An essential element of these freedoms is to keep open a continuing dialogue or process of national and local levels, on the implementation of the comprehensive peace process, as well as
communication between the government and the people. It is in the interest of the State that the for government[-]civil society dialogue and consensus-building on peace agenda and
channels for free political discussion be maintained to the end that the government may perceive initiatives."138
and be responsive to the people's will.131Envisioned to be corollary to the twin rights to
information and disclosure is the design for feedback mechanisms.
In fine, E.O. No. 3 establishes petitioners' right to be consulted on the peace agenda, as a
corollary to the constitutional right to information and disclosure.
MS. ROSARIO BRAID. Yes. And lastly, Mr. Presiding Officer, will the people be able
to participate? Will the government provide feedback mechanisms so that the
PAPP Esperon committed grave abuse of discretion
people can participate and can react where the existing media facilities are not
able to provide full feedback mechanisms to the government? I suppose this
will be part of the government implementing operational mechanisms. The PAPP committed grave abuse of discretion when he failed to carry out the pertinent
consultation. The furtive process by which the MOA-AD was designed and crafted runs
contrary to and in excess of the legal authority, and amounts to a whimsical, capricious,
MR. OPLE. Yes. I think through their elected representatives and that is how these
oppressive, arbitrary and despotic exercise thereof.
courses take place. There is a message and a feedback, both ways.

The Court may not, of course, require the PAPP to conduct the consultation in a particular way
xxxx
or manner. It may, however, require him to comply with the law and discharge the
functions within the authority granted by the President.139
MS. ROSARIO BRAID. Mr. Presiding Officer, may I just make one last sentence?
Petitioners are not claiming a seat at the negotiating table, contrary to respondents' retort in
I think when we talk about the feedback network, we are not talking about public justifying the denial of petitioners' right to be consulted. Respondents' stance manifests the
officials but also network of private business o[r] community-based manner by which they treat the salient provisions of E.O. No. 3 on people's participation. Such
organizations that will be reacting. As a matter of fact, we will put more credence or disregard of the express mandate of the President is not much different from superficial conduct
credibility on the private network of volunteers and voluntary community-based toward token provisos that border on classic lip service. 140 It illustrates a gross evasion of
organizations. So I do not think we are afraid that there will be another OMA in the positive duty and a virtual refusal to perform the duty enjoined.
making.132(Emphasis supplied)
As for respondents' invocation of the doctrine of executive privilege, it is not tenable under the
The imperative of a public consultation, as a species of the right to information, is evident in the premises. The argument defies sound reason when contrasted with E.O. No. 3's explicit
"marching orders" to respondents. The mechanics for the duty to disclose information and to provisions on continuing consultation and dialogue on both national and local levels.
conduct public consultation regarding the peace agenda and process is manifestly provided by The executive order even recognizes the exercise of the public's right even before the GRP
E.O. No. 3.133 The preambulatory clause of E.O. No. 3 declares that there is a need to further makes its official recommendations or before the government proffers its definite
enhance the contribution of civil society to the comprehensive peace process by institutionalizing propositions.141 It bear emphasis that E.O. No. 3 seeks to elicit relevant advice, information,
the people's participation. comments and recommendations from the people through dialogue.

One of the three underlying principles of the comprehensive peace process is that it "should be AT ALL EVENTS, respondents effectively waived the defense of executive privilege in view of
community-based, reflecting the sentiments, values and principles important to all Filipinos" and their unqualified disclosure of the official copies of the final draft of the MOA-AD. By
"shall be defined not by the government alone, nor by the different contending groups only, but unconditionally complying with the Court's August 4, 2008 Resolution, without a prayer for the
by all Filipinos as one community."134 Included as a component of the comprehensive peace document's disclosure in camera, or without a manifestation that it was complying therewith ex
process is consensus-building and empowerment for peace, which includes "continuing abundante ad cautelam.
consultations on both national and local levels to build consensus for a peace agenda and
process, and the mobilization and facilitation of people's participation in the peace process."135
Petitioners' assertion that the Local Government Code (LGC) of 1991 declares it a State policy
to "require all national agencies and offices to conduct periodic consultations with appropriate
local government units, non-governmental and people's organizations, and other concerned
sectors of the community before any project or program is implemented in their respective
jurisdictions"142 is well-taken. The LGC chapter on intergovernmental relations puts flesh into this The MOA-AD is inconsistent with the Constitution and laws as presently worded.
avowed policy:
In general, the objections against the MOA-AD center on the extent of the powers conceded
Prior Consultations Required. - No project or program shall be implemented by therein to the BJE. Petitioners assert that the powers granted to the BJE exceed those granted
government authorities unlessthe consultations mentioned in Sections 2 (c) and 26 to any local government under present laws, and even go beyond those of the present ARMM.
hereof are complied with, and prior approval of the sanggunian concerned is obtained: Before assessing some of the specific powers that would have been vested in the BJE, however,
Provided, That occupants in areas where such projects are to be implemented shall it would be useful to turn first to a general idea that serves as a unifying link to the different
not be evicted unless appropriate relocation sites have been provided, in accordance provisions of the MOA-AD, namely, the international law concept of association. Significantly,
with the provisions of the Constitution.143 (Italics and underscoring supplied) the MOA-AD explicitly alludes to this concept, indicating that the Parties actually framed its
provisions with it in mind.
In Lina, Jr. v. Hon. Paño,144 the Court held that the above-stated policy and above-quoted
provision of the LGU apply only to national programs or projects which are to be implemented in Association is referred to in paragraph 3 on TERRITORY, paragraph 11 on RESOURCES, and
a particular local community. Among the programs and projects covered are those that are paragraph 4 on GOVERNANCE. It is in the last mentioned provision, however, that the MOA-AD
critical to the environment and human ecology including those that may call for the eviction of a most clearly uses it to describe the envisioned relationship between the BJE and the Central
particular group of people residing in the locality where these will be implemented.145 The MOA- Government.
AD is one peculiar program that unequivocally and unilaterally vests ownership of a vast
territory to the Bangsamoro people,146 which could pervasively and drastically result to
4. The relationship between the Central Government and the Bangsamoro
the diaspora or displacement of a great number of inhabitants from their total
juridical entity shall be associative characterized by shared authority and
environment.
responsibility with a structure of governance based on executive, legislative, judicial
and administrative institutions with defined powers and functions in the comprehensive
With respect to the indigenous cultural communities/indigenous peoples (ICCs/IPs), whose compact. A period of transition shall be established in a comprehensive peace
interests are represented herein by petitioner Lopez and are adversely affected by the MOA-AD, compact specifying the relationship between the Central Government and the BJE.
the ICCs/IPs have, under the IPRA, the right to participate fully at all levels of decision-making in (Emphasis and underscoring supplied)
matters which may affect their rights, lives and destinies.147 The MOA-AD, an instrument
recognizing ancestral domain, failed to justify its non-compliance with the clear-cut mechanisms
The nature of the "associative" relationship may have been intended to be defined more
ordained in said Act,148 which entails, among other things, the observance of the free and prior
precisely in the still to be forged Comprehensive Compact. Nonetheless, given that there is a
informed consent of the ICCs/IPs.
concept of "association" in international law, and the MOA-AD - by its inclusion of international
law instruments in its TOR- placed itself in an international legal context, that concept of
Notably, the IPRA does not grant the Executive Department or any government agency the association may be brought to bear in understanding the use of the term "associative" in the
power to delineate and recognize an ancestral domain claim by mere agreement or compromise. MOA-AD.
The recognition of the ancestral domain is the raison d'etre of the MOA-AD, without which all
other stipulations or "consensus points" necessarily must fail. In proceeding to make a sweeping
Keitner and Reisman state that
declaration on ancestral domain, without complying with the IPRA, which is cited as one of the
TOR of the MOA-AD, respondents clearly transcended the boundaries of their authority.
As it seems, even the heart of the MOA-AD is still subject to necessary changes to the legal [a]n association is formed when two states of unequal power voluntarily establish
framework. While paragraph 7 on Governance suspends the effectivity of all provisions requiring durable links. In the basic model, one state, the associate, delegates certain
changes to the legal framework, such clause is itself invalid, as will be discussed in the following responsibilities to the other, the principal, while maintaining its international
section. status as a state. Free associations represent a middle ground between
integration and independence. x x x150 (Emphasis and underscoring supplied)
Indeed, ours is an open society, with all the acts of the government subject to public scrutiny and
available always to public cognizance. This has to be so if the country is to remain democratic, For purposes of illustration, the Republic of the Marshall Islands and the Federated States of
with sovereignty residing in the people and all government authority emanating from them. 149 Micronesia (FSM), formerly part of the U.S.-administered Trust Territory of the Pacific
Islands,151 are associated states of the U.S. pursuant to a Compact of Free Association. The
currency in these countries is the U.S. dollar, indicating their very close ties with the U.S., yet
ON THE SECOND SUBSTANTIVE ISSUE
they issue their own travel documents, which is a mark of their statehood. Their international
legal status as states was confirmed by the UN Security Council and by their admission to UN
With regard to the provisions of the MOA-AD, there can be no question that they cannot all be membership.
accommodated under the present Constitution and laws. Respondents have admitted as much
in the oral arguments before this Court, and the MOA-AD itself recognizes the need to amend
According to their compacts of free association, the Marshall Islands and the FSM generally
the existing legal framework to render effective at least some of its provisions. Respondents,
have the capacity to conduct foreign affairs in their own name and right, such capacity extending
nonetheless, counter that the MOA-AD is free of any legal infirmity because any provisions
to matters such as the law of the sea, marine resources, trade, banking, postal, civil aviation,
therein which are inconsistent with the present legal framework will not be effective until the
and cultural relations. The U.S. government, when conducting its foreign affairs, is obligated to
necessary changes to that framework are made. The validity of this argument will be considered
consult with the governments of the Marshall Islands or the FSM on matters which it (U.S.
later. For now, the Court shall pass upon how
government) regards as relating to or affecting either government.
In the event of attacks or threats against the Marshall Islands or the FSM, the U.S. government SECTION 15. There shall be created autonomous regions in Muslim Mindanao and in
has the authority and obligation to defend them as if they were part of U.S. territory. The U.S. the Cordilleras consisting of provinces, cities, municipalities, and geographical areas
government, moreover, has the option of establishing and using military areas and facilities sharing common and distinctive historical and cultural heritage, economic and social
within these associated states and has the right to bar the military personnel of any third country structures, and other relevant characteristics within the framework of this
from having access to these territories for military purposes. Constitution and the national sovereignty as well as territorial integrity of the
Republic of the Philippines.
It bears noting that in U.S. constitutional and international practice, free association is
understood as an international association between sovereigns. The Compact of Free The BJE is a far more powerful
Association is a treaty which is subordinate to the associated nation's national constitution, and entity than the autonomous region
each party may terminate the association consistent with the right of independence. It has been recognized in the Constitution
said that, with the admission of the U.S.-associated states to the UN in 1990, the UN recognized
that the American model of free association is actually based on an underlying status of
It is not merely an expanded version of the ARMM, the status of its relationship with the national
independence.152
government being fundamentally different from that of the ARMM. Indeed, BJE is a state in all
but name as it meets the criteria of a state laid down in the Montevideo
In international practice, the "associated state" arrangement has usually been used as Convention,154 namely, a permanent population, a defined territory, a government, and
a transitional device of former colonies on their way to full independence. Examples of states a capacity to enter into relations with other states.
that have passed through the status of associated states as a transitional phase are Antigua, St.
Kitts-Nevis-Anguilla, Dominica, St. Lucia, St. Vincent and Grenada. All have since become
Even assuming arguendo that the MOA-AD would not necessarily sever any portion of Philippine
independent states.153
territory, the spirit animating it - which has betrayed itself by its use of the concept of
association - runs counter to the national sovereignty and territorial integrity of the
Back to the MOA-AD, it contains many provisions which are consistent with the international Republic.
legal concept of association, specifically the following: the BJE's capacity to enter into economic
and trade relations with foreign countries, the commitment of the Central Government to ensure
The defining concept underlying the relationship between the national government and
the BJE's participation in meetings and events in the ASEAN and the specialized UN agencies,
the BJE being itself contrary to the present Constitution, it is not surprising that many of
and the continuing responsibility of the Central Government over external defense. Moreover,
the specific provisions of the MOA-AD on the formation and powers of the BJE are in
the BJE's right to participate in Philippine official missions bearing on negotiation of border
conflict with the Constitution and the laws.
agreements, environmental protection, and sharing of revenues pertaining to the bodies of water
adjacent to or between the islands forming part of the ancestral domain, resembles the right of
the governments of FSM and the Marshall Islands to be consulted by the U.S. government on Article X, Section 18 of the Constitution provides that "[t]he creation of the autonomous region
any foreign affairs matter affecting them. shall be effective when approved by a majority of the votes cast by the constituent units in a
plebiscite called for the purpose, provided that only provinces, cities, and geographic areas
voting favorably in such plebiscite shall be included in the autonomous region."
These provisions of the MOA indicate, among other things, that the Parties aimed to vest in the
(Emphasis supplied)
BJE the status of an associated state or, at any rate, a status closely approximating it.

As reflected above, the BJE is more of a state than an autonomous region. But even assuming
The concept of association is not recognized under the present Constitution
that it is covered by the term "autonomous region" in the constitutional provision just quoted, the
MOA-AD would still be in conflict with it. Under paragraph 2(c) on TERRITORY in relation to 2(d)
No province, city, or municipality, not even the ARMM, is recognized under our laws as having and 2(e), the present geographic area of the ARMM and, in addition, the municipalities of Lanao
an "associative" relationship with the national government. Indeed, the concept implies powers del Norte which voted for inclusion in the ARMM during the 2001 plebiscite - Baloi, Munai,
that go beyond anything ever granted by the Constitution to any local or regional government. It Nunungan, Pantar, Tagoloan and Tangkal - are automatically part of the BJE without need of
also implies the recognition of the associated entity as a state. The Constitution, however, does another plebiscite, in contrast to the areas under Categories A and B mentioned earlier in the
not contemplate any state in this jurisdiction other than the Philippine State, much less does it overview. That the present components of the ARMM and the above-mentioned municipalities
provide for a transitory status that aims to prepare any part of Philippine territory for voted for inclusion therein in 2001, however, does not render another plebiscite unnecessary
independence. under the Constitution, precisely because what these areas voted for then was their inclusion in
the ARMM, not the BJE.
Even the mere concept animating many of the MOA-AD's provisions, therefore, already requires
for its validity the amendment of constitutional provisions, specifically the following provisions of The MOA-AD, moreover, would not
Article X: comply with Article X, Section 20 of
the Constitution
SECTION 1. The territorial and political subdivisions of the Republic of the Philippines
are the provinces, cities, municipalities, and barangays. There shall since that provision defines the powers of autonomous regions as follows:
be autonomous regions in Muslim Mindanao and the Cordilleras as hereinafter
provided.
SECTION 20. Within its territorial jurisdiction and subject to the provisions of this status which, in international practice, has generally been a preparation for independence, is
Constitution and national laws, the organic act of autonomous regions shall provide for certainly not conducive to national unity.
legislative powers over:
Besides being irreconcilable with the Constitution, the MOA-AD is also inconsistent with
(1) Administrative organization; prevailing statutory law, among which are R.A. No. 9054156 or the Organic Act of the ARMM,
and the IPRA.157
(2) Creation of sources of revenues;
Article X, Section 3 of the Organic Act of the ARMM is a bar to the adoption of the
definition of "Bangsamoro people" used in the MOA-AD. Paragraph 1 on Concepts and
(3) Ancestral domain and natural resources;
Principles states:

(4) Personal, family, and property relations;


1. It is the birthright of all Moros and all Indigenous peoples of Mindanao to
identify themselves and be accepted as "Bangsamoros". The Bangsamoro people
(5) Regional urban and rural planning development; refers to those who are natives or original inhabitants of Mindanao and its
adjacent islands including Palawan and the Sulu archipelago at the time of conquest
or colonization of its descendants whether mixed or of full blood. Spouses and their
(6) Economic, social, and tourism development;
descendants are classified as Bangsamoro. The freedom of choice of the Indigenous
people shall be respected. (Emphasis and underscoring supplied)
(7) Educational policies;
This use of the term Bangsamoro sharply contrasts with that found in the Article X, Section 3 of
(8) Preservation and development of the cultural heritage; and the Organic Act, which, rather than lumping together the identities of the Bangsamoro and other
indigenous peoples living in Mindanao, clearly distinguishes between Bangsamoro people
and Tribal peoples, as follows:
(9) Such other matters as may be authorized by law for the promotion of the general
welfare of the people of the region. (Underscoring supplied)
"As used in this Organic Act, the phrase "indigenous cultural community" refers
to Filipino citizens residing in the autonomous region who are:
Again on the premise that the BJE may be regarded as an autonomous region, the MOA-AD
would require an amendment that would expand the above-quoted provision. The mere passage
of new legislation pursuant to sub-paragraph No. 9 of said constitutional provision would not (a) Tribal peoples. These are citizens whose social, cultural and economic conditions
suffice, since any new law that might vest in the BJE the powers found in the MOA-AD must, distinguish them from other sectors of the national community; and
itself, comply with other provisions of the Constitution. It would not do, for instance, to merely
pass legislation vesting the BJE with treaty-making power in order to accommodate paragraph 4 (b) Bangsa Moro people. These are citizens who are believers in Islam and who
of the strand on RESOURCES which states: "The BJE is free to enter into any economic have retained some or all of their own social, economic, cultural, and political
cooperation and trade relations with foreign countries: provided, however, that such relationships institutions."
and understandings do not include aggression against the Government of the Republic of the
Philippines x x x." Under our constitutional system, it is only the President who has that
power. Pimentel v. Executive Secretary155 instructs: Respecting the IPRA, it lays down the prevailing procedure for the delineation and recognition of
ancestral domains. The MOA-AD's manner of delineating the ancestral domain of the
Bangsamoro people is a clear departure from that procedure. By paragraph 1 of Territory, the
In our system of government, the President, being the head of state, is regarded Parties simply agree that, subject to the delimitations in the agreed Schedules, "[t]he
as the sole organ and authority in external relations and is the country's sole
Bangsamoro homeland and historic territory refer to the land mass as well as the maritime,
representative with foreign nations. As the chief architect of foreign policy, the
terrestrial, fluvial and alluvial domains, and the aerial domain, the atmospheric space above it,
President acts as the country's mouthpiece with respect to international affairs. embracing the Mindanao-Sulu-Palawan geographic region."
Hence, the President is vested with the authority to deal with foreign states and
governments, extend or withhold recognition, maintain diplomatic relations, enter
into treaties, and otherwise transact the business of foreign relations. In the Chapter VIII of the IPRA, on the other hand, lays down a detailed procedure, as illustrated in the
realm of treaty-making, the President has the sole authority to negotiate with following provisions thereof:
other states. (Emphasis and underscoring supplied)
SECTION 52. Delineation Process. - The identification and delineation of ancestral
Article II, Section 22 of the Constitution must also be amended if the scheme envisioned domains shall be done in accordance with the following procedures:
in the MOA-AD is to be effected. That constitutional provision states: "The State recognizes
and promotes the rights of indigenous cultural communities within the framework of national
xxxx
unity and development." (Underscoring supplied) An associative arrangement does not uphold
national unity. While there may be a semblance of unity because of the associative ties between
the BJE and the national government, the act of placing a portion of Philippine territory in a
b) Petition for Delineation. - The process of delineating a specific perimeter may be g) Notice and Publication. - A copy of each document, including a translation in the
initiated by the NCIP with the consent of the ICC/IP concerned, or through a Petition native language of the ICCs/IPs concerned shall be posted in a prominent place
for Delineation filed with the NCIP, by a majority of the members of the ICCs/IPs; therein for at least fifteen (15) days. A copy of the document shall also be posted at
the local, provincial and regional offices of the NCIP, and shall be published in a
newspaper of general circulation once a week for two (2) consecutive weeks to allow
c) Delineation Proper. - The official delineation of ancestral domain boundaries
other claimants to file opposition thereto within fifteen (15) days from date of such
including census of all community members therein, shall be immediately undertaken
publication: Provided, That in areas where no such newspaper exists, broadcasting in
by the Ancestral Domains Office upon filing of the application by the ICCs/IPs
a radio station will be a valid substitute: Provided, further, That mere posting shall be
concerned. Delineation will be done in coordination with the community concerned
deemed sufficient if both newspaper and radio station are not available;
and shall at all times include genuine involvement and participation by the members of
the communities concerned;
h) Endorsement to NCIP. - Within fifteen (15) days from publication, and of the
inspection process, the Ancestral Domains Office shall prepare a report to the NCIP
d) Proof Required. - Proof of Ancestral Domain Claims shall include the testimony of
endorsing a favorable action upon a claim that is deemed to have sufficient proof.
elders or community under oath, and other documents directly or indirectly attesting to
However, if the proof is deemed insufficient, the Ancestral Domains Office shall
the possession or occupation of the area since time immemorial by such ICCs/IPs in
require the submission of additional evidence: Provided, That the Ancestral Domains
the concept of owners which shall be any one (1) of the following authentic
Office shall reject any claim that is deemed patently false or fraudulent after inspection
documents:
and verification: Provided, further, That in case of rejection, the Ancestral Domains
Office shall give the applicant due notice, copy furnished all concerned, containing the
1) Written accounts of the ICCs/IPs customs and traditions; grounds for denial. The denial shall be appealable to the NCIP: Provided, furthermore,
That in cases where there are conflicting claims among ICCs/IPs on the boundaries of
ancestral domain claims, the Ancestral Domains Office shall cause the contending
2) Written accounts of the ICCs/IPs political structure and institution; parties to meet and assist them in coming up with a preliminary resolution of the
conflict, without prejudice to its full adjudication according to the section below.
3) Pictures showing long term occupation such as those of old
improvements, burial grounds, sacred places and old villages; xxxx

4) Historical accounts, including pacts and agreements concerning To remove all doubts about the irreconcilability of the MOA-AD with the present legal system, a
boundaries entered into by the ICCs/IPs concerned with other ICCs/IPs; discussion of not only the Constitution and domestic statutes, but also of international law is in
order, for
5) Survey plans and sketch maps;
Article II, Section 2 of the Constitution states that the Philippines "adopts the generally
6) Anthropological data; accepted principles of international law as part of the law of the land."

7) Genealogical surveys; Applying this provision of the Constitution, the Court, in Mejoff v. Director of Prisons,158 held that
the Universal Declaration of Human Rights is part of the law of the land on account of which it
ordered the release on bail of a detained alien of Russian descent whose deportation order had
8) Pictures and descriptive histories of traditional communal forests and not been executed even after two years. Similarly, the Court in Agustin v. Edu159 applied the
hunting grounds; aforesaid constitutional provision to the 1968 Vienna Convention on Road Signs and Signals.

9) Pictures and descriptive histories of traditional landmarks such as International law has long recognized the right to self-determination of "peoples," understood not
mountains, rivers, creeks, ridges, hills, terraces and the like; and merely as the entire population of a State but also a portion thereof. In considering the question
of whether the people of Quebec had a right to unilaterally secede from Canada, the Canadian
10) Write-ups of names and places derived from the native dialect of the Supreme Court in REFERENCE RE SECESSION OF QUEBEC160 had occasion to acknowledge
community. that "the right of a people to self-determination is now so widely recognized in international
conventions that the principle has acquired a status beyond ‘convention' and is considered a
general principle of international law."
e) Preparation of Maps. - On the basis of such investigation and the findings of fact
based thereon, the Ancestral Domains Office of the NCIP shall prepare a perimeter
map, complete with technical descriptions, and a description of the natural features Among the conventions referred to are the International Covenant on Civil and Political
and landmarks embraced therein; Rights161 and the International Covenant on Economic, Social and Cultural Rights 162 which state,
in Article 1 of both covenants, that all peoples, by virtue of the right of self-determination, "freely
determine their political status and freely pursue their economic, social, and cultural
f) Report of Investigation and Other Documents. - A complete copy of the preliminary development."
census and a report of investigation, shall be prepared by the Ancestral Domains
Office of the NCIP;
The people's right to self-determination should not, however, be understood as extending to a every State. Positive International Law does not recognize the right of national
unilateral right of secession. A distinction should be made between the right of internal and groups, as such, to separate themselves from the State of which they form part
external self-determination. REFERENCE RE SECESSION OF QUEBEC is again instructive: by the simple expression of a wish, any more than it recognizes the right of other
States to claim such a separation. Generally speaking, the grant or refusal of the
right to a portion of its population of determining its own political fate by
"(ii) Scope of the Right to Self-determination
plebiscite or by some other method, is, exclusively, an attribute of the
sovereignty of every State which is definitively constituted. A dispute between
126. The recognized sources of international law establish that the right to self- two States concerning such a question, under normal conditions therefore, bears upon
determination of a people is normally fulfilled through internal self- a question which International Law leaves entirely to the domestic jurisdiction of one of
determination - a people's pursuit of its political, economic, social and cultural the States concerned. Any other solution would amount to an infringement of
development within the framework of an existing state. A right to external self- sovereign rights of a State and would involve the risk of creating difficulties and a lack
determination (which in this case potentially takes the form of the assertion of a of stability which would not only be contrary to the very idea embodied in term "State,"
right to unilateral secession) arises in only the most extreme of cases and, even but would also endanger the interests of the international community. If this right is not
then, under carefully defined circumstances. x x x possessed by a large or small section of a nation, neither can it be held by the State to
which the national group wishes to be attached, nor by any other State. (Emphasis
and underscoring supplied)
External self-determination can be defined as in the following statement from
the Declaration on Friendly Relations, supra, as
The Committee held that the dispute concerning the Aaland Islands did not refer to a question
which is left by international law to the domestic jurisdiction of Finland, thereby applying the
The establishment of a sovereign and independent State, the free association or
exception rather than the rule elucidated above. Its ground for departing from the general rule,
integration with an independent State or the emergence into any other political
however, was a very narrow one, namely, the Aaland Islands agitation originated at a time when
status freely determined by a peopleconstitute modes of implementing the right of
Finland was undergoing drastic political transformation. The internal situation of Finland was,
self-determination by that people. (Emphasis added)
according to the Committee, so abnormal that, for a considerable time, the conditions required
for the formation of a sovereign State did not exist. In the midst of revolution, anarchy, and civil
127. The international law principle of self-determination has evolved within a war, the legitimacy of the Finnish national government was disputed by a large section of the
framework of respect for the territorial integrity of existing states. The various people, and it had, in fact, been chased from the capital and forcibly prevented from carrying out
international documents that support the existence of a people's right to self- its duties. The armed camps and the police were divided into two opposing forces. In light of
determination also contain parallel statements supportive of the conclusion that the these circumstances, Finland was not, during the relevant time period, a "definitively constituted"
exercise of such a right must be sufficiently limited to prevent threats to an existing sovereign state. The Committee, therefore, found that Finland did not possess the right to
state's territorial integrity or the stability of relations between sovereign states. withhold from a portion of its population the option to separate itself - a right which sovereign
nations generally have with respect to their own populations.
x x x x (Emphasis, italics and underscoring supplied)
Turning now to the more specific category of indigenous peoples, this term has been used, in
scholarship as well as international, regional, and state practices, to refer to groups with distinct
The Canadian Court went on to discuss the exceptional cases in which the right to external self- cultures, histories, and connections to land (spiritual and otherwise) that have been forcibly
determination can arise, namely, where a people is under colonial rule, is subject to foreign incorporated into a larger governing society. These groups are regarded as "indigenous" since
domination or exploitation outside a colonial context, and - less definitely but asserted by a they are the living descendants of pre-invasion inhabitants of lands now dominated by others.
number of commentators - is blocked from the meaningful exercise of its right to internal self- Otherwise stated, indigenous peoples, nations, or communities are culturally distinctive groups
determination. The Court ultimately held that the population of Quebec had no right to that find themselves engulfed by settler societies born of the forces of empire and
secession, as the same is not under colonial rule or foreign domination, nor is it being deprived conquest.164 Examples of groups who have been regarded as indigenous peoples are the Maori
of the freedom to make political choices and pursue economic, social and cultural development, of New Zealand and the aboriginal peoples of Canada.
citing that Quebec is equitably represented in legislative, executive and judicial institutions within
Canada, even occupying prominent positions therein.
As with the broader category of "peoples," indigenous peoples situated within states do not have
a general right to independence or secession from those states under international law, 165 but
The exceptional nature of the right of secession is further exemplified in the REPORT OF THE they do have rights amounting to what was discussed above as the right to internal self-
INTERNATIONAL COMMITTEE OF JURISTS ON THE LEGAL ASPECTS OF THE AALAND determination.
ISLANDS QUESTION.163 There, Sweden presented to the Council of the League of Nations the
question of whether the inhabitants of the Aaland Islands should be authorized to determine by
plebiscite if the archipelago should remain under Finnish sovereignty or be incorporated in the In a historic development last September 13, 2007, the UN General Assembly adopted the
kingdom of Sweden. The Council, before resolving the question, appointed an International United Nations Declaration on the Rights of Indigenous Peoples (UN DRIP) through General
Committee composed of three jurists to submit an opinion on the preliminary issue of whether Assembly Resolution 61/295. The vote was 143 to 4, the Philippines being included among
the dispute should, based on international law, be entirely left to the domestic jurisdiction of those in favor, and the four voting against being Australia, Canada, New Zealand, and the U.S.
Finland. The Committee stated the rule as follows: The Declaration clearly recognized the right of indigenous peoples to self-determination,
encompassing the right to autonomy or self-government, to wit:
x x x [I]n the absence of express provisions in international treaties, the right of
disposing of national territory is essentially an attribute of the sovereignty of
Article 3 employment, vocational training and retraining, housing, sanitation, health and social
security.
Indigenous peoples have the right to self-determination. By virtue of that right they
freely determine their political status and freely pursue their economic, social and 2. States shall take effective measures and, where appropriate, special measures to
cultural development. ensure continuing improvement of their economic and social conditions. Particular
attention shall be paid to the rights and special needs of indigenous elders, women,
youth, children and persons with disabilities.
Article 4

Article 26
Indigenous peoples, in exercising their right to self-determination, have the right
to autonomy or self-government in matters relating to their internal and local
affairs, as well as ways and means for financing their autonomous functions. 1. Indigenous peoples have the right to the lands, territories and resources
which they have traditionally owned, occupied or otherwise used or acquired.
Article 5
2. Indigenous peoples have the right to own, use, develop and control the lands,
territories and resources that they possess by reason of traditional ownership or other
Indigenous peoples have the right to maintain and strengthen their distinct political,
traditional occupation or use, as well as those which they have otherwise acquired.
legal, economic, social and cultural institutions, while retaining their right to participate
fully, if they so choose, in the political, economic, social and cultural life of the State.
3. States shall give legal recognition and protection to these lands, territories and
resources. Such recognition shall be conducted with due respect to the customs,
Self-government, as used in international legal discourse pertaining to indigenous peoples, has
traditions and land tenure systems of the indigenous peoples concerned.
been understood as equivalent to "internal self-determination."166 The extent of self-
determination provided for in the UN DRIP is more particularly defined in its subsequent articles,
some of which are quoted hereunder: Article 30

Article 8 1. Military activities shall not take place in the lands or territories of indigenous
peoples, unless justified by a relevant public interest or otherwise freely agreed with or
requested by the indigenous peoples concerned.
1. Indigenous peoples and individuals have the right not to be subjected to forced
assimilation or destruction of their culture.
2. States shall undertake effective consultations with the indigenous peoples
concerned, through appropriate procedures and in particular through their
2. States shall provide effective mechanisms for prevention of, and redress for:
representative institutions, prior to using their lands or territories for military activities.

(a) Any action which has the aim or effect of depriving them of their integrity as
Article 32
distinct peoples, or of their cultural values or ethnic identities;

1. Indigenous peoples have the right to determine and develop priorities and
(b) Any action which has the aim or effect of dispossessing them of their lands,
strategies for the development or use of their lands or territories and other resources.
territories or resources;

2. States shall consult and cooperate in good faith with the indigenous peoples
(c) Any form of forced population transfer which has the aim or effect of
concerned through their own representative institutions in order to obtain their free and
violating or undermining any of their rights;
informed consent prior to the approval of any project affecting their lands or territories
and other resources, particularly in connection with the development, utilization or
(d) Any form of forced assimilation or integration; exploitation of mineral, water or other resources.

(e) Any form of propaganda designed to promote or incite racial or ethnic 3. States shall provide effective mechanisms for just and fair redress for any such
discrimination directed against them. activities, and appropriate measures shall be taken to mitigate adverse environmental,
economic, social, cultural or spiritual impact.
Article 21
Article 37
1. Indigenous peoples have the right, without discrimination, to the improvement of
their economic and social conditions, including, inter alia, in the areas of education, 1. Indigenous peoples have the right to the recognition, observance and enforcement
of treaties, agreements and other constructive arrangements concluded with States or
their successors and to have States honour and respect such treaties, agreements 7. The Parties agree that the mechanisms and modalities for the actual
and other constructive arrangements. implementation of this MOA-AD shall be spelt out in the Comprehensive Compact to
mutually take such steps to enable it to occur effectively.
2. Nothing in this Declaration may be interpreted as diminishing or eliminating the
rights of indigenous peoples contained in treaties, agreements and other constructive Any provisions of the MOA-AD requiring amendments to the existing legal framework
arrangements. shall come into force upon signing of a Comprehensive Compact and upon effecting
the necessary changes to the legal framework with due regard to non derogation of
prior agreements and within the stipulated timeframe to be contained in the
Article 38
Comprehensive Compact.

States in consultation and cooperation with indigenous peoples, shall take the
Indeed, the foregoing stipulation keeps many controversial provisions of the MOA-AD from
appropriate measures, including legislative measures, to achieve the ends of this
coming into force until the necessary changes to the legal framework are effected. While the
Declaration.
word "Constitution" is not mentioned in the provision now under consideration or
anywhere else in the MOA-AD, the term "legal framework" is certainly broad enough to
Assuming that the UN DRIP, like the Universal Declaration on Human Rights, must now be include the Constitution.
regarded as embodying customary international law - a question which the Court need not
definitively resolve here - the obligations enumerated therein do not strictly require the Republic
Notwithstanding the suspensive clause, however, respondents, by their mere act of
to grant the Bangsamoro people, through the instrumentality of the BJE, the particular rights and
incorporating in the MOA-AD the provisions thereof regarding the associative relationship
powers provided for in the MOA-AD. Even the more specific provisions of the UN DRIP are
between the BJE and the Central Government, have already violated the Memorandum of
general in scope, allowing for flexibility in its application by the different States.
Instructions From The President dated March 1, 2001, which states that the "negotiations shall
be conducted in accordance with x x x the principles of the sovereignty and territorial
There is, for instance, no requirement in the UN DRIP that States now guarantee indigenous integrityof the Republic of the Philippines." (Emphasis supplied) Establishing an associative
peoples their own police and internal security force. Indeed, Article 8 presupposes that it is the relationship between the BJE and the Central Government is, for the reasons already discussed,
State which will provide protection for indigenous peoples against acts like the forced a preparation for independence, or worse, an implicit acknowledgment of an independent status
dispossession of their lands - a function that is normally performed by police officers. If the already prevailing.
protection of a right so essential to indigenous people's identity is acknowledged to be the
responsibility of the State, then surely the protection of rights less significant to them as such
Even apart from the above-mentioned Memorandum, however, the MOA-AD is defective
peoples would also be the duty of States. Nor is there in the UN DRIP an acknowledgement of
because the suspensive clause is invalid, as discussed below.
the right of indigenous peoples to the aerial domain and atmospheric space. What it upholds, in
Article 26 thereof, is the right of indigenous peoples to the lands, territories and resources which
they have traditionally owned, occupied or otherwise used or acquired. The authority of the GRP Peace Negotiating Panel to negotiate with the MILF is founded on E.O.
No. 3, Section 5(c), which states that there shall be established Government Peace Negotiating
Panels for negotiations with different rebel groups to be "appointed by the President as her
Moreover, the UN DRIP, while upholding the right of indigenous peoples to autonomy, does not
official emissaries to conduct negotiations, dialogues, and face-to-face discussions with rebel
obligate States to grant indigenous peoples the near-independent status of an associated state.
groups." These negotiating panels are to report to the President, through the PAPP on the
All the rights recognized in that document are qualified in Article 46 as follows:
conduct and progress of the negotiations.

1. Nothing in this Declaration may be interpreted as implying for any State, people,
It bears noting that the GRP Peace Panel, in exploring lasting solutions to the Moro Problem
group or person any right to engage in any activity or to perform any act contrary to
through its negotiations with the MILF, was not restricted by E.O. No. 3 only to those options
the Charter of the United Nations or construed as authorizing or encouraging any
available under the laws as they presently stand. One of the components of a comprehensive
action which would dismember or impair, totally or in part, the territorial
peace process, which E.O. No. 3 collectively refers to as the "Paths to Peace," is the pursuit of
integrity or political unity of sovereign and independent States.
social, economic, and political reforms which may require new legislation or even constitutional
amendments. Sec. 4(a) of E.O. No. 3, which reiterates Section 3(a), of E.O. No. 125, 167 states:
Even if the UN DRIP were considered as part of the law of the land pursuant to Article II, Section
2 of the Constitution, it would not suffice to uphold the validity of the MOA-AD so as to render its
SECTION 4. The Six Paths to Peace. - The components of the comprehensive peace
compliance with other laws unnecessary.
process comprise the processes known as the "Paths to Peace". These component
processes are interrelated and not mutually exclusive, and must therefore be pursued
It is, therefore, clear that the MOA-AD contains numerous provisions that cannot be simultaneously in a coordinated and integrated fashion. They shall include, but may
reconciled with the Constitution and the laws as presently worded. Respondents proffer, not be limited to, the following:
however, that the signing of the MOA-AD alone would not have entailed any violation of law or
grave abuse of discretion on their part, precisely because it stipulates that the provisions thereof
a. PURSUIT OF SOCIAL, ECONOMIC AND POLITICAL REFORMS. This component
inconsistent with the laws shall not take effect until these laws are amended. They cite
involves the vigorous implementation of various policies, reforms, programs and
paragraph 7 of the MOA-AD strand on GOVERNANCE quoted earlier, but which is reproduced
projects aimed at addressing the root causes of internal armed conflicts and
below for convenience:
social unrest. This may require administrative action, new legislation or even As the experience of nations which have similarly gone through internal armed conflict will show,
constitutional amendments. however, peace is rarely attained by simply pursuing a military solution. Oftentimes, changes as
far-reaching as a fundamental reconfiguration of the nation's constitutional structure is required.
The observations of Dr. Kirsti Samuels are enlightening, to wit:
x x x x (Emphasis supplied)

x x x [T]he fact remains that a successful political and governance transition must form
The MOA-AD, therefore, may reasonably be perceived as an attempt of respondents to address,
the core of any post-conflict peace-building mission. As we have observed in Liberia
pursuant to this provision of E.O. No. 3, the root causes of the armed conflict in Mindanao. The
and Haiti over the last ten years, conflict cessation without modification of the political
E.O. authorized them to "think outside the box," so to speak. Hence, they negotiated and were
environment, even where state-building is undertaken through technical electoral
set on signing the MOA-AD that included various social, economic, and political reforms which
assistance and institution- or capacity-building, is unlikely to succeed. On average,
cannot, however, all be accommodated within the present legal framework, and which thus
more than 50 percent of states emerging from conflict return to conflict. Moreover, a
would require new legislation and constitutional amendments.
substantial proportion of transitions have resulted in weak or limited democracies.

The inquiry on the legality of the "suspensive clause," however, cannot stop here, because it
The design of a constitution and its constitution-making process can play an important
must be asked whether the President herself may exercise the power delegated to the GRP
role in the political and governance transition. Constitution-making after conflict is an
Peace Panel under E.O. No. 3, Sec. 4(a).
opportunity to create a common vision of the future of a state and a road map on how
to get there. The constitution can be partly a peace agreement and partly a framework
The President cannot delegate a power that she herself does not possess. May the President, in setting up the rules by which the new democracy will operate. 170
the course of peace negotiations, agree to pursue reforms that would require new legislation and
constitutional amendments, or should the reforms be restricted only to those solutions which the
In the same vein, Professor Christine Bell, in her article on the nature and legal status of peace
present laws allow? The answer to this question requires a discussion of the extent of the
agreements, observed that the typical way that peace agreements establish or confirm
President's power to conduct peace negotiations.
mechanisms for demilitarization and demobilization is by linking them to new constitutional
structures addressing governance, elections, and legal and human rights institutions. 171
That the authority of the President to conduct peace negotiations with rebel groups is not
explicitly mentioned in the Constitution does not mean that she has no such authority.
In the Philippine experience, the link between peace agreements and constitution-making has
In Sanlakas v. Executive Secretary,168 in issue was the authority of the President to declare a
been recognized by no less than the framers of the Constitution. Behind the provisions of the
state of rebellion - an authority which is not expressly provided for in the Constitution. The Court
Constitution on autonomous regions172 is the framers' intention to implement a particular peace
held thus:
agreement, namely, the Tripoli Agreement of 1976 between the GRP and the MNLF, signed by
then Undersecretary of National Defense Carmelo Z. Barbero and then MNLF Chairman Nur
"In her ponencia in Marcos v. Manglapus, Justice Cortes put her thesis into Misuari.
jurisprudence. There, the Court, by a slim 8-7 margin, upheld the President's power to
forbid the return of her exiled predecessor. The rationale for the majority's ruling
MR. ROMULO. There are other speakers; so, although I have some more questions, I
rested on the President's
will reserve my right to ask them if they are not covered by the other speakers. I have
only two questions.
. . . unstated residual powers which are implied from the grant of
executive power and which are necessary for her to comply with her
I heard one of the Commissioners say that local autonomy already exists in the
duties under the Constitution. The powers of the President are not
Muslim region; it is working very well; it has, in fact, diminished a great deal of the
limited to what are expressly enumerated in the article on the
problems. So, my question is: since that already exists, why do we have to go into
Executive Department and in scattered provisions of the
something new?
Constitution. This is so, notwithstanding the avowed intent of the members
of the Constitutional Commission of 1986 to limit the powers of the
President as a reaction to the abuses under the regime of Mr. Marcos, for MR. OPLE. May I answer that on behalf of Chairman Nolledo. Commissioner Yusup
the result was a limitation of specific powers of the President, particularly Abubakar is right that certain definite steps have been taken to implement the
those relating to the commander-in-chief clause, but not a diminution of the provisions of the Tripoli Agreement with respect to an autonomous region in
general grant of executive power. Mindanao. This is a good first step, but there is no question that this is merely a
partial response to the Tripoli Agreement itself and to the fuller standard of
regional autonomy contemplated in that agreement, and now by state
Thus, the President's authority to declare a state of rebellion springs in the main
policy.173(Emphasis supplied)
from her powers as chief executive and, at the same time, draws strength from
her Commander-in-Chief powers. x x x (Emphasis and underscoring supplied)
The constitutional provisions on autonomy and the statutes enacted pursuant to them have, to
the credit of their drafters, been partly successful. Nonetheless, the Filipino people are still faced
Similarly, the President's power to conduct peace negotiations is implicitly included in her
with the reality of an on-going conflict between the Government and the MILF. If the President is
powers as Chief Executive and Commander-in-Chief. As Chief Executive, the President has the
to be expected to find means for bringing this conflict to an end and to achieve lasting peace in
general responsibility to promote public peace, and as Commander-in-Chief, she has the more
Mindanao, then she must be given the leeway to explore, in the course of peace negotiations,
specific duty to prevent and suppress rebellion and lawless violence.169
solutions that may require changes to the Constitution for their implementation. Being uniquely what President Marcos did in Sanidad, but for their independent consideration of whether these
vested with the power to conduct peace negotiations with rebel groups, the President is in a recommendations merit being formally proposed through initiative.
singular position to know the precise nature of their grievances which, if resolved, may bring an
end to hostilities.
These recommendations, however, may amount to nothing more than the President's
suggestions to the people, for any further involvement in the process of initiative by the Chief
The President may not, of course, unilaterally implement the solutions that she considers viable, Executive may vitiate its character as a genuine "people's initiative." The only initiative
but she may not be prevented from submitting them as recommendations to Congress, which recognized by the Constitution is that which truly proceeds from the people. As the Court stated
could then, if it is minded, act upon them pursuant to the legal procedures for constitutional in Lambino v. COMELEC:177
amendment and revision. In particular, Congress would have the option, pursuant to Article XVII,
Sections 1 and 3 of the Constitution, to propose the recommended amendments or revision to
"The Lambino Group claims that their initiative is the ‘people's voice.' However, the
the people, call a constitutional convention, or submit to the electorate the question of calling
Lambino Group unabashedly states in ULAP Resolution No. 2006-02, in the
such a convention.
verification of their petition with the COMELEC, that ‘ULAP maintains its unqualified
support to the agenda of Her Excellency President Gloria Macapagal-Arroyo for
While the President does not possess constituent powers - as those powers may be exercised constitutional reforms.' The Lambino Group thus admits that their ‘people's' initiative
only by Congress, a Constitutional Convention, or the people through initiative and referendum - is an ‘unqualified support to the agenda' of the incumbent President to change the
she may submit proposals for constitutional change to Congress in a manner that does not Constitution. This forewarns the Court to be wary of incantations of ‘people's voice' or
involve the arrogation of constituent powers. ‘sovereign will' in the present initiative."

In Sanidad v. COMELEC,174 in issue was the legality of then President Marcos' act of directly It will be observed that the President has authority, as stated in her oath of office,178 only
submitting proposals for constitutional amendments to a referendum, bypassing the interim to preserve and defend the Constitution. Such presidential power does not, however, extend to
National Assembly which was the body vested by the 1973 Constitution with the power to allowing her to change the Constitution, but simply to recommend proposed amendments or
propose such amendments. President Marcos, it will be recalled, never convened the interim revision. As long as she limits herself to recommending these changes and submits to the
National Assembly. The majority upheld the President's act, holding that "the urges of absolute proper procedure for constitutional amendments and revision, her mere recommendation need
necessity" compelled the President as the agent of the people to act as he did, there being no not be construed as an unconstitutional act.
interim National Assembly to propose constitutional amendments. Against this ruling, Justices
Teehankee and Muñoz Palma vigorously dissented. The Court's concern at present, however, is
The foregoing discussion focused on the President's authority to
not with regard to the point on which it was then divided in that controversial case, but on that
propose constitutional amendments, since her authority to propose new legislation is not in
which was not disputed by either side.
controversy. It has been an accepted practice for Presidents in this jurisdiction to propose new
legislation. One of the more prominent instances the practice is usually done is in the yearly
Justice Teehankee's dissent,175 in particular, bears noting. While he disagreed that the President State of the Nation Address of the President to Congress. Moreover, the annual general
may directly submit proposed constitutional amendments to a referendum, implicit in his opinion appropriations bill has always been based on the budget prepared by the President, which - for
is a recognition that he would have upheld the President's action along with the majority had the all intents and purposes - is a proposal for new legislation coming from the President.179
President convened the interim National Assembly and coursed his proposals through it. Thus
Justice Teehankee opined:
The "suspensive clause" in the MOA-AD viewed in light of the above-discussed
standards
"Since the Constitution provides for the organization of the essential departments of
government, defines and delimits the powers of each and prescribes the manner of
Given the limited nature of the President's authority to propose constitutional amendments,
the exercise of such powers, and the constituent power has not been granted to but
she cannot guaranteeto any third party that the required amendments will eventually be put in
has been withheld from the President or Prime Minister, it follows that the President's
place, nor even be submitted to a plebiscite. The most she could do is submit these proposals
questioned decrees proposing and submitting constitutional amendments directly to
as recommendations either to Congress or the people, in whom constituent powers are vested.
the people (without the intervention of the interim National Assembly in whom
the power is expressly vested) are devoid of constitutional and legal
basis."176 (Emphasis supplied) Paragraph 7 on Governance of the MOA-AD states, however, that all provisions thereof which
cannot be reconciled with the present Constitution and laws "shall come into force upon signing
of a Comprehensive Compact and upon effecting the necessary changes to the legal
From the foregoing discussion, the principle may be inferred that the President - in the course of
framework." This stipulation does not bear the marks of a suspensive condition - defined in civil
conducting peace negotiations - may validly consider implementing even those policies that
law as a future and uncertain event - but of a term. It is not a question of whether the necessary
require changes to the Constitution, but she may not unilaterally implement them without the
changes to the legal framework will be effected, but when. That there is no uncertainty being
intervention of Congress, or act in any way as if the assent of that body were assumed as
contemplated is plain from what follows, for the paragraph goes on to state that the
a certainty.
contemplated changes shall be "with due regard to non derogation of prior
agreements and within the stipulated timeframe to be contained in the Comprehensive
Since, under the present Constitution, the people also have the power to directly propose Compact."
amendments through initiative and referendum, the President may also submit her
recommendations to the people, not as a formal proposal to be voted on in a plebiscite similar to
Pursuant to this stipulation, therefore, it is mandatory for the GRP to effect the changes to the signed on July 7, 1999 between the Government of Sierra Leone and the Revolutionary United
legal framework contemplated in the MOA-AD - which changes would include constitutional Front (RUF), a rebel group with which the Sierra Leone Government had been in armed conflict
amendments, as discussed earlier. It bears noting that, for around eight years at the time of signing. There were non-contracting signatories to the
agreement, among which were the Government of the Togolese Republic, the Economic
Community of West African States, and the UN.
By the time these changes are put in place, the MOA-AD itself would be counted among
the "prior agreements" from which there could be no derogation.
On January 16, 2002, after a successful negotiation between the UN Secretary-General and the
Sierra Leone Government, another agreement was entered into by the UN and that Government
What remains for discussion in the Comprehensive Compact would merely be the implementing
whereby the Special Court of Sierra Leone was established. The sole purpose of the Special
details for these "consensus points" and, notably, the deadline for effecting the contemplated
Court, an international court, was to try persons who bore the greatest responsibility for serious
changes to the legal framework.
violations of international humanitarian law and Sierra Leonean law committed in the territory of
Sierra Leone since November 30, 1996.
Plainly, stipulation-paragraph 7 on GOVERNANCE is inconsistent with the limits of the
President's authority to propose constitutional amendments, it being a virtual guarantee
Among the stipulations of the Lomé Accord was a provision for the full pardon of the members of
that the Constitution and the laws of the Republic of the Philippines will certainly be adjusted to
the RUF with respect to anything done by them in pursuit of their objectives as members of that
conform to all the "consensus points" found in the MOA-AD. Hence, it must be struck down
organization since the conflict began.
as unconstitutional.

In the Lomé Accord case, the Defence argued that the Accord created an internationally
A comparison between the "suspensive clause" of the MOA-AD with a similar provision
binding obligation not to prosecute the beneficiaries of the amnesty provided therein, citing,
appearing in the 1996 final peace agreement between the MNLF and the GRP is most
among other things, the participation of foreign dignitaries and international organizations in the
instructive.
finalization of that agreement. The Special Court, however, rejected this argument, ruling that
the Lome Accord is not a treaty and that it can only create binding obligations and rights
As a backdrop, the parties to the 1996 Agreement stipulated that it would be implemented in two between the parties in municipal law, not in international law. Hence, the Special Court held, it is
phases. Phase Icovered a three-year transitional period involving the putting up of new ineffective in depriving an international court like it of jurisdiction.
administrative structures through Executive Order, such as the Special Zone of Peace and
Development (SZOPAD) and the Southern Philippines Council for Peace and Development
"37. In regard to the nature of a negotiated settlement of an internal armed conflict it
(SPCPD), while Phase II covered the establishment of the new regional autonomous
is easy to assume and to argue with some degree of plausibility, as Defence
government through amendment or repeal of R.A. No. 6734, which was then the Organic Act of
counsel for the defendants seem to have done, that the mere fact that in
the ARMM.
addition to the parties to the conflict, the document formalizing the settlement is
signed by foreign heads of state or their representatives and representatives of
The stipulations on Phase II consisted of specific agreements on the structure of the expanded international organizations, means the agreement of the parties is
autonomous region envisioned by the parties. To that extent, they are similar to the provisions of internationalized so as to create obligations in international law.
the MOA-AD. There is, however, a crucial difference between the two agreements. While the
MOA-AD virtually guarantees that the "necessary changes to the legal framework" will be
xxxx
put in place, the GRP-MNLF final peace agreement states thus: "Accordingly, these provisions
[on Phase II] shall be recommended by the GRP to Congress for incorporation in the
amendatory or repealing law." 40. Almost every conflict resolution will involve the parties to the conflict and the
mediator or facilitator of the settlement, or persons or bodies under whose auspices
the settlement took place but who are not at all parties to the conflict, are not
Concerns have been raised that the MOA-AD would have given rise to a binding international
contracting parties and who do not claim any obligation from the contracting parties or
law obligation on the part of the Philippines to change its Constitution in conformity thereto, on
incur any obligation from the settlement.
the ground that it may be considered either as a binding agreement under international law, or a
unilateral declaration of the Philippine government to the international community that it would
grant to the Bangsamoro people all the concessions therein stated. Neither ground finds 41. In this case, the parties to the conflict are the lawful authority of the State
sufficient support in international law, however. and the RUF which has no status of statehood and is to all intents and purposes
a faction within the state. The non-contracting signatories of the Lomé
Agreement were moral guarantors of the principle that, in the terms of Article
The MOA-AD, as earlier mentioned in the overview thereof, would have included foreign
XXXIV of the Agreement, "this peace agreement is implemented with integrity
dignitaries as signatories. In addition, representatives of other nations were invited to witness its
and in good faith by both parties". The moral guarantors assumed no legal
signing in Kuala Lumpur. These circumstances readily lead one to surmise that the MOA-AD
obligation. It is recalled that the UN by its representative appended, presumably for
would have had the status of a binding international agreement had it been signed. An
avoidance of doubt, an understanding of the extent of the agreement to be
examination of the prevailing principles in international law, however, leads to the contrary
implemented as not including certain international crimes.
conclusion.

42. An international agreement in the nature of a treaty must create rights and
The Decision on Challenge to Jurisdiction: Lomé Accord Amnesty180 (the Lomé Accord case) of
obligations regulated by international law so that a breach of its terms will be a breach
the Special Court of Sierra Leone is enlightening. The Lomé Accord was a peace agreement
determined under international law which will also provide principle means of 44. Of course, not all unilateral acts imply obligation; but a State may choose to
enforcement. The Lomé Agreement created neither rights nor obligations take up a certain position in relation to a particular matter with the intention of
capable of being regulated by international law. An agreement such as the Lomé being bound-the intention is to be ascertained by interpretation of the act. When
Agreement which brings to an end an internal armed conflict no doubt creates a States make statements by which their freedom of action is to be limited, a restrictive
factual situation of restoration of peace that the international community acting interpretation is called for.
through the Security Council may take note of. That, however, will not convert it
to an international agreement which creates an obligation enforceable in
xxxx
international, as distinguished from municipal, law. A breach of the terms of such
a peace agreement resulting in resumption of internal armed conflict or creating a
threat to peace in the determination of the Security Council may indicate a reversal of 51. In announcing that the 1974 series of atmospheric tests would be the last,
the factual situation of peace to be visited with possible legal consequences arising the French Government conveyed to the world at large, including the Applicant,
from the new situation of conflict created. Such consequences such as action by the its intention effectively to terminate these tests. It was bound to assume that
Security Council pursuant to Chapter VII arise from the situation and not from the other States might take note of these statements and rely on their being
agreement, nor from the obligation imposed by it. Such action cannot be regarded as effective. The validity of these statements and their legal consequences must be
a remedy for the breach. A peace agreement which settles an internal armed considered within the general framework of the security of international
conflict cannot be ascribed the same status as one which settles an intercourse, and the confidence and trust which are so essential in the relations
international armed conflict which, essentially, must be between two or more among States. It is from the actual substance of these statements, and from the
warring States. The Lomé Agreement cannot be characterised as an circumstances attending their making, that the legal implications of the
international instrument. x x x" (Emphasis, italics and underscoring supplied) unilateral act must be deduced. The objects of these statements are clear and
they were addressed to the international community as a whole, and the Court
holds that they constitute an undertaking possessing legal effect. The Court
Similarly, that the MOA-AD would have been signed by representatives of States and
considers *270 that the President of the Republic, in deciding upon the effective
international organizations not parties to the Agreement would not have sufficed to vest in it a
cessation of atmospheric tests, gave an undertaking to the international community to
binding character under international law.
which his words were addressed. x x x (Emphasis and underscoring supplied)

In another vein, concern has been raised that the MOA-AD would amount to a unilateral
As gathered from the above-quoted ruling of the ICJ, public statements of a state representative
declaration of the Philippine State, binding under international law, that it would comply with all
may be construed as a unilateral declaration only when the following conditions are present: the
the stipulations stated therein, with the result that it would have to amend its Constitution
statements were clearly addressed to the international community, the state intended to be
accordingly regardless of the true will of the people. Cited as authority for this view is Australia v.
bound to that community by its statements, and that not to give legal effect to those statements
France,181 also known as the Nuclear Tests Case, decided by the International Court of Justice
would be detrimental to the security of international intercourse. Plainly, unilateral declarations
(ICJ).
arise only in peculiar circumstances.

In the Nuclear Tests Case, Australia challenged before the ICJ the legality of France's nuclear
The limited applicability of the Nuclear Tests Case ruling was recognized in a later case decided
tests in the South Pacific. France refused to appear in the case, but public statements from its
by the ICJ entitled Burkina Faso v. Mali,183 also known as the Case Concerning the Frontier
President, and similar statements from other French officials including its Minister of Defence,
Dispute. The public declaration subject of that case was a statement made by the President of
that its 1974 series of atmospheric tests would be its last, persuaded the ICJ to dismiss the
Mali, in an interview by a foreign press agency, that Mali would abide by the decision to be
case.182 Those statements, the ICJ held, amounted to a legal undertaking addressed to the
issued by a commission of the Organization of African Unity on a frontier dispute then pending
international community, which required no acceptance from other States for it to become
between Mali and Burkina Faso.
effective.

Unlike in the Nuclear Tests Case, the ICJ held that the statement of Mali's President was not a
Essential to the ICJ ruling is its finding that the French government intended to be bound to the
unilateral act with legal implications. It clarified that its ruling in the Nuclear Tests case rested on
international community in issuing its public statements, viz:
the peculiar circumstances surrounding the French declaration subject thereof, to wit:

43. It is well recognized that declarations made by way of unilateral acts, concerning
40. In order to assess the intentions of the author of a unilateral act, account must be
legal or factual situations, may have the effect of creating legal obligations.
taken of all the factual circumstances in which the act occurred. For example, in the
Declarations of this kind may be, and often are, very specific. When it is the intention
Nuclear Tests cases, the Court took the view that since the applicant States
of the State making the declaration that it should become bound according to
were not the only ones concerned at the possible continuance of atmospheric
its terms, that intention confers on the declaration the character of a legal
testing by the French Government, that Government's unilateral declarations
undertaking, the State being thenceforth legally required to follow a course of
had ‘conveyed to the world at large, including the Applicant, its intention
conduct consistent with the declaration. An undertaking of this kind, if given
effectively to terminate these tests‘ (I.C.J. Reports 1974, p. 269, para. 51; p. 474,
publicly, and with an intent to be bound, even though not made within the context of
para. 53). In the particular circumstances of those cases, the French
international negotiations, is binding. In these circumstances, nothing in the nature of
Government could not express an intention to be bound otherwise than by
a quid pro quo nor any subsequent acceptance of the declaration, nor even any reply
unilateral declarations. It is difficult to see how it could have accepted the terms
or reaction from other States, is required for the declaration to take effect, since such
of a negotiated solution with each of the applicants without thereby jeopardizing
a requirement would be inconsistent with the strictly unilateral nature of the juridical
its contention that its conduct was lawful. The circumstances of the present
act by which the pronouncement by the State was made.
case are radically different. Here, there was nothing to hinder the Parties from SUMMARY
manifesting an intention to accept the binding character of the conclusions of
the Organization of African Unity Mediation Commission by the normal method:
The petitions are ripe for adjudication. The failure of respondents to consult the local government
a formal agreement on the basis of reciprocity. Since no agreement of this kind
units or communities affected constitutes a departure by respondents from their mandate under
was concluded between the Parties, the Chamber finds that there are no grounds to
E.O. No. 3. Moreover, respondents exceeded their authority by the mere act of guaranteeing
interpret the declaration made by Mali's head of State on 11 April 1975 as a unilateral
amendments to the Constitution. Any alleged violation of the Constitution by any branch of
act with legal implications in regard to the present case. (Emphasis and underscoring
government is a proper matter for judicial review.
supplied)

As the petitions involve constitutional issues which are of paramount public interest or of
Assessing the MOA-AD in light of the above criteria, it would not have amounted to a unilateral
transcendental importance, the Court grants the petitioners, petitioners-in-intervention and
declaration on the part of the Philippine State to the international community. The Philippine
intervening respondents the requisite locus standi in keeping with the liberal stance adopted
panel did not draft the same with the clear intention of being bound thereby to the international
in David v. Macapagal-Arroyo.
community as a whole or to any State, but only to the MILF. While there were States and
international organizations involved, one way or another, in the negotiation and projected signing
of the MOA-AD, they participated merely as witnesses or, in the case of Malaysia, as facilitator. Contrary to the assertion of respondents that the non-signing of the MOA-AD and the eventual
As held in the Lomé Accord case, the mere fact that in addition to the parties to the conflict, the dissolution of the GRP Peace Panel mooted the present petitions, the Court finds that the
peace settlement is signed by representatives of states and international organizations present petitions provide an exception to the "moot and academic" principle in view of (a) the
does not mean that the agreement is internationalized so as to create obligations in international grave violation of the Constitution involved; (b) the exceptional character of the situation and
law. paramount public interest; (c) the need to formulate controlling principles to guide the bench, the
bar, and the public; and (d) the fact that the case is capable of repetition yet evading review.
Since the commitments in the MOA-AD were not addressed to States, not to give legal effect to
such commitments would not be detrimental to the security of international intercourse - to the The MOA-AD is a significant part of a series of agreements necessary to carry out the GRP-
trust and confidence essential in the relations among States. MILF Tripoli Agreement on Peace signed by the government and the MILF back in June 2001.
Hence, the present MOA-AD can be renegotiated or another one drawn up that could contain
similar or significantly dissimilar provisions compared to the original.
In one important respect, the circumstances surrounding the MOA-AD are closer to that
of Burkina Faso wherein, as already discussed, the Mali President's statement was not held to
be a binding unilateral declaration by the ICJ. As in that case, there was also nothing to hinder The Court, however, finds that the prayers for mandamus have been rendered moot in view of
the Philippine panel, had it really been its intention to be bound to other States, to manifest that the respondents' action in providing the Court and the petitioners with the official copy of the final
intention by formal agreement. Here, that formal agreement would have come about by the draft of the MOA-AD and its annexes.
inclusion in the MOA-AD of a clear commitment to be legally bound to the international
community, not just the MILF, and by an equally clear indication that the signatures of the
participating states-representatives would constitute an acceptance of that commitment. The people's right to information on matters of public concern under Sec. 7, Article III of the
Entering into such a formal agreement would not have resulted in a loss of face for the Philippine Constitution is in splendid symmetry with the state policy of full public disclosure of all its
government before the international community, which was one of the difficulties that prevented transactions involving public interest under Sec. 28, Article II of the Constitution. The right to
the French Government from entering into a formal agreement with other countries. That the information guarantees the right of the people to demand information, while Section 28
Philippine panel did not enter into such a formal agreement suggests that it had no intention to recognizes the duty of officialdom to give information even if nobody demands. The complete
be bound to the international community. On that ground, the MOA-AD may not be considered a and effective exercise of the right to information necessitates that its complementary provision
unilateral declaration under international law. on public disclosure derive the same self-executory nature, subject only to reasonable
safeguards or limitations as may be provided by law.

The MOA-AD not being a document that can bind the Philippines under international law
notwithstanding, respondents' almost consummated act of guaranteeing amendments to the The contents of the MOA-AD is a matter of paramount public concern involving public interest in
legal framework is, by itself, sufficient to constitute grave abuse of discretion. The grave the highest order. In declaring that the right to information contemplates steps and negotiations
leading to the consummation of the contract, jurisprudence finds no distinction as to the
abuse lies not in the fact that they considered, as a solution to the Moro Problem, the creation of
a state within a state, but in their brazen willingness to guarantee that Congress and the executory nature or commercial character of the agreement.
sovereign Filipino people would give their imprimatur to their solution. Upholding such an
act would amount to authorizing a usurpation of the constituent powers vested only in Congress, An essential element of these twin freedoms is to keep a continuing dialogue or process of
a Constitutional Convention, or the people themselves through the process of initiative, for the communication between the government and the people. Corollary to these twin rights is the
only way that the Executive can ensure the outcome of the amendment process is through an design for feedback mechanisms. The right to public consultation was envisioned to be a
undue influence or interference with that process. species of these public rights.

The sovereign people may, if it so desired, go to the extent of giving up a portion of its own At least three pertinent laws animate these constitutional imperatives and justify the exercise of
territory to the Moros for the sake of peace, for it can change the Constitution in any it wants, so the people's right to be consulted on relevant matters relating to the peace agenda.
long as the change is not inconsistent with what, in international law, is known as Jus
Cogens.184 Respondents, however, may not preempt it in that decision.
One, E.O. No. 3 itself is replete with mechanics for continuing consultations on both national and amendments is, by itself, already a constitutional violation that renders the MOA-AD fatally
local levels and for a principal forum for consensus-building. In fact, it is the duty of the defective.
Presidential Adviser on the Peace Process to conduct regular dialogues to seek relevant
information, comments, advice, and recommendations from peace partners and concerned
WHEREFORE, respondents' motion to dismiss is DENIED. The main and intervening petitions
sectors of society.
are GIVEN DUE COURSE and hereby GRANTED.

Two, Republic Act No. 7160 or the Local Government Code of 1991 requires all national offices
The Memorandum of Agreement on the Ancestral Domain Aspect of the GRP-MILF Tripoli
to conduct consultations before any project or program critical to the environment and human
Agreement on Peace of 2001 is declared contrary to law and the Constitution.
ecology including those that may call for the eviction of a particular group of people residing in
such locality, is implemented therein. The MOA-AD is one peculiar program that unequivocally
and unilaterally vests ownership of a vast territory to the Bangsamoro people, which could SO ORDERED.
pervasively and drastically result to the diaspora or displacement of a great number of
inhabitants from their total environment.

Three, Republic Act No. 8371 or the Indigenous Peoples Rights Act of 1997 provides for clear-
cut procedure for the recognition and delineation of ancestral domain, which entails, among
other things, the observance of the free and prior informed consent of the Indigenous Cultural
Communities/Indigenous Peoples. Notably, the statute does not grant the Executive Department
or any government agency the power to delineate and recognize an ancestral domain claim by
mere agreement or compromise.

The invocation of the doctrine of executive privilege as a defense to the general right to
information or the specific right to consultation is untenable. The various explicit legal provisions
fly in the face of executive secrecy. In any event, respondents effectively waived such defense
after it unconditionally disclosed the official copies of the final draft of the MOA-AD, for judicial
compliance and public scrutiny.

In sum, the Presidential Adviser on the Peace Process committed grave abuse of discretion
when he failed to carry out the pertinent consultation process, as mandated by E.O. No. 3,
Republic Act No. 7160, and Republic Act No. 8371. The furtive process by which the MOA-AD
was designed and crafted runs contrary to and in excess of the legal authority, and amounts to a
whimsical, capricious, oppressive, arbitrary and despotic exercise thereof. It illustrates a gross
evasion of positive duty and a virtual refusal to perform the duty enjoined.

The MOA-AD cannot be reconciled with the present Constitution and laws. Not only its specific
provisions but the very concept underlying them, namely, the associative relationship envisioned
between the GRP and the BJE, are unconstitutional, for the concept presupposes that the
associated entity is a state and implies that the same is on its way to independence.

While there is a clause in the MOA-AD stating that the provisions thereof inconsistent with the
present legal framework will not be effective until that framework is amended, the same does not
cure its defect. The inclusion of provisions in the MOA-AD establishing an associative
relationship between the BJE and the Central Government is, itself, a violation of the
Memorandum of Instructions From The President dated March 1, 2001, addressed to the
government peace panel. Moreover, as the clause is worded, it virtually guarantees that the
necessary amendments to the Constitution and the laws will eventually be put in place. Neither
the GRP Peace Panel nor the President herself is authorized to make such a guarantee.
Upholding such an act would amount to authorizing a usurpation of the constituent powers
vested only in Congress, a Constitutional Convention, or the people themselves through the
process of initiative, for the only way that the Executive can ensure the outcome of the
amendment process is through an undue influence or interference with that process.

While the MOA-AD would not amount to an international agreement or unilateral declaration
binding on the Philippines under international law, respondents' act of guaranteeing
G.R No. 187167 August 16, 2011 for the filing of application for the extended continental shelf.8 Complying with these
requirements, RA 9522 shortened one baseline, optimized the location of some basepoints
around the Philippine archipelago and classified adjacent territories, namely, the Kalayaan
PROF. MERLIN M. MAGALLONA, AKBAYAN PARTY-LIST REP. RISA HONTIVEROS,
Island Group (KIG) and the Scarborough Shoal, as "regimes of islands" whose islands generate
PROF. HARRY C. ROQUE, JR., AND UNIVERSITY OF THE PHILIPPINES COLLEGE OF
their own applicable maritime zones.
LAW STUDENTS, ALITHEA BARBARA ACAS, VOLTAIRE ALFERES, CZARINA MAY
ALTEZ, FRANCIS ALVIN ASILO, SHERYL BALOT, RUBY AMOR BARRACA, JOSE JAVIER
BAUTISTA, ROMINA BERNARDO, VALERIE PAGASA BUENAVENTURA, EDAN MARRI Petitioners, professors of law, law students and a legislator, in their respective capacities as
CAÑETE, VANN ALLEN DELA CRUZ, RENE DELORINO, PAULYN MAY DUMAN, SHARON "citizens, taxpayers or x x x legislators,"9 as the case may be, assail the constitutionality of RA
ESCOTO, RODRIGO FAJARDO III, GIRLIE FERRER, RAOULLE OSEN FERRER, CARLA 9522 on two principal grounds, namely: (1) RA 9522 reduces Philippine maritime territory, and
REGINA GREPO, ANNA MARIE CECILIA GO, IRISH KAY KALAW, MARY ANN JOY LEE, logically, the reach of the Philippine state’s sovereign power, in violation of Article 1 of the 1987
MARIA LUISA MANALAYSAY, MIGUEL RAFAEL MUSNGI, MICHAEL OCAMPO, JAKLYN Constitution,10 embodying the terms of the Treaty of Paris11 and ancillary treaties,12 and (2) RA
HANNA PINEDA, WILLIAM RAGAMAT, MARICAR RAMOS, ENRIK FORT REVILLAS, 9522 opens the country’s waters landward of the baselines to maritime passage by all vessels
JAMES MARK TERRY RIDON, JOHANN FRANTZ RIVERA IV, CHRISTIAN RIVERO, DIANNE and aircrafts, undermining Philippine sovereignty and national security, contravening the
MARIE ROA, NICHOLAS SANTIZO, MELISSA CHRISTINA SANTOS, CRISTINE MAE country’s nuclear-free policy, and damaging marine resources, in violation of relevant
TABING, VANESSA ANNE TORNO, MARIA ESTER VANGUARDIA, and MARCELINO constitutional provisions.13
VELOSO III, Petitioners,
vs.
In addition, petitioners contend that RA 9522’s treatment of the KIG as "regime of islands" not
HON. EDUARDO ERMITA, IN HIS CAPACITY AS EXECUTIVE SECRETARY, HON.
only results in the loss of a large maritime area but also prejudices the livelihood of subsistence
ALBERTO ROMULO, IN HIS CAPACITY AS SECRETARY OF THE DEPARTMENT OF
fishermen.14 To buttress their argument of territorial diminution, petitioners facially attack RA
FOREIGN AFFAIRS, HON. ROLANDO ANDAYA, IN HIS CAPACITY AS SECRETARY OF
9522 for what it excluded and included – its failure to reference either the Treaty of Paris or
THE DEPARTMENT OF BUDGET AND MANAGEMENT, HON. DIONY VENTURA, IN HIS
Sabah and its use of UNCLOS III’s framework of regime of islands to determine the maritime
CAPACITY AS ADMINISTRATOR OF THE NATIONAL MAPPING & RESOURCE
zones of the KIG and the Scarborough Shoal.
INFORMATION AUTHORITY, and HON. HILARIO DAVIDE, JR., IN HIS CAPACITY AS
REPRESENTATIVE OF THE PERMANENT MISSION OF THE REPUBLIC OF THE
PHILIPPINES TO THE UNITED NATIONS,Respondents. Commenting on the petition, respondent officials raised threshold issues questioning (1) the
petition’s compliance with the case or controversy requirement for judicial review grounded on
petitioners’ alleged lack of locus standi and (2) the propriety of the writs of certiorari and
DECISION
prohibition to assail the constitutionality of RA 9522. On the merits, respondents defended RA
9522 as the country’s compliance with the terms of UNCLOS III, preserving Philippine territory
CARPIO, J.: over the KIG or Scarborough Shoal. Respondents add that RA 9522 does not undermine the
country’s security, environment and economic interests or relinquish the Philippines’ claim over
Sabah.
The Case

Respondents also question the normative force, under international law, of petitioners’ assertion
This original action for the writs of certiorari and prohibition assails the constitutionality of
that what Spain ceded to the United States under the Treaty of Paris were the islands and all the
Republic Act No. 95221(RA 9522) adjusting the country’s archipelagic baselines and classifying
waters found within the boundaries of the rectangular area drawn under the Treaty of Paris.
the baseline regime of nearby territories.

We left unacted petitioners’ prayer for an injunctive writ.


The Antecedents

The Issues
In 1961, Congress passed Republic Act No. 3046 (RA 3046)2 demarcating the maritime
baselines of the Philippines as an archipelagic State.3 This law followed the framing of the
Convention on the Territorial Sea and the Contiguous Zone in 1958 (UNCLOS I), 4 codifying, The petition raises the following issues:
among others, the sovereign right of States parties over their "territorial sea," the breadth of
which, however, was left undetermined. Attempts to fill this void during the second round of
1. Preliminarily –
negotiations in Geneva in 1960 (UNCLOS II) proved futile. Thus, domestically, RA 3046
remained unchanged for nearly five decades, save for legislation passed in 1968 (Republic Act
No. 5446 [RA 5446]) correcting typographical errors and reserving the drawing of baselines 1. Whether petitioners possess locus standi to bring this suit; and
around Sabah in North Borneo.
2. Whether the writs of certiorari and prohibition are the proper remedies to
In March 2009, Congress amended RA 3046 by enacting RA 9522, the statute now under assail the constitutionality of RA 9522.
scrutiny. The change was prompted by the need to make RA 3046 compliant with the terms of
the United Nations Convention on the Law of the Sea (UNCLOS III), 5 which the Philippines
2. On the merits, whether RA 9522 is unconstitutional.
ratified on 27 February 1984.6 Among others, UNCLOS III prescribes the water-land ratio,
length, and contour of baselines of archipelagic States like the Philippines 7 and sets the deadline
The Ruling of the Court sovereignty over territorial waters extends hundreds of nautical miles around the Philippine
archipelago, embracing the rectangular area delineated in the Treaty of Paris. 22
On the threshold issues, we hold that (1) petitioners possess locus standi to bring this suit as
citizens and (2) the writs of certiorari and prohibition are proper remedies to test the Petitioners’ theory fails to persuade us.
constitutionality of RA 9522. On the merits, we find no basis to declare RA 9522 unconstitutional.
UNCLOS III has nothing to do with the acquisition (or loss) of territory. It is a multilateral treaty
On the Threshold Issues regulating, among others, sea-use rights over maritime zones (i.e., the territorial waters [12
Petitioners Possess Locus nautical miles from the baselines], contiguous zone [24 nautical miles from the baselines],
Standi as Citizens exclusive economic zone [200 nautical miles from the baselines]), and continental shelves that
UNCLOS III delimits.23 UNCLOS III was the culmination of decades-long negotiations among
United Nations members to codify norms regulating the conduct of States in the world’s oceans
Petitioners themselves undermine their assertion of locus standi as legislators and taxpayers
and submarine areas, recognizing coastal and archipelagic States’ graduated authority over a
because the petition alleges neither infringement of legislative prerogative15 nor misuse of public
limited span of waters and submarine lands along their coasts.
funds,16 occasioned by the passage and implementation of RA 9522. Nonetheless, we recognize
petitioners’ locus standi as citizens with constitutionally sufficient interest in the resolution of the
merits of the case which undoubtedly raises issues of national significance necessitating urgent On the other hand, baselines laws such as RA 9522 are enacted by UNCLOS III States parties
resolution. Indeed, owing to the peculiar nature of RA 9522, it is understandably difficult to find to mark-out specific basepoints along their coasts from which baselines are drawn, either
other litigants possessing "a more direct and specific interest" to bring the suit, thus satisfying straight or contoured, to serve as geographic starting points to measure the breadth of the
one of the requirements for granting citizenship standing.17 maritime zones and continental shelf. Article 48 of UNCLOS III on archipelagic States like ours
could not be any clearer:
The Writs of Certiorari and Prohibition
Are Proper Remedies to Test Article 48. Measurement of the breadth of the territorial sea, the contiguous zone, the exclusive
the Constitutionality of Statutes economic zone and the continental shelf. – The breadth of the territorial sea, the contiguous
zone, the exclusive economic zone and the continental shelf shall be measured from
archipelagic baselines drawn in accordance with article 47. (Emphasis supplied)
In praying for the dismissal of the petition on preliminary grounds, respondents seek a strict
observance of the offices of the writs of certiorari and prohibition, noting that the writs cannot
issue absent any showing of grave abuse of discretion in the exercise of judicial, quasi-judicial or Thus, baselines laws are nothing but statutory mechanisms for UNCLOS III States parties to
ministerial powers on the part of respondents and resulting prejudice on the part of petitioners. 18 delimit with precision the extent of their maritime zones and continental shelves. In turn, this
gives notice to the rest of the international community of the scope of the maritime space and
submarine areas within which States parties exercise treaty-based rights, namely, the exercise
Respondents’ submission holds true in ordinary civil proceedings. When this Court exercises its
of sovereignty over territorial waters (Article 2), the jurisdiction to enforce customs, fiscal,
constitutional power of judicial review, however, we have, by tradition, viewed the writs of
immigration, and sanitation laws in the contiguous zone (Article 33), and the right to exploit the
certiorari and prohibition as proper remedial vehicles to test the constitutionality of
living and non-living resources in the exclusive economic zone (Article 56) and continental shelf
statutes,19 and indeed, of acts of other branches of government.20 Issues of constitutional import
(Article 77).
are sometimes crafted out of statutes which, while having no bearing on the personal interests of
the petitioners, carry such relevance in the life of this nation that the Court inevitably finds itself
constrained to take cognizance of the case and pass upon the issues raised, non-compliance Even under petitioners’ theory that the Philippine territory embraces the islands and all the
with the letter of procedural rules notwithstanding. The statute sought to be reviewed here is one waters within the rectangular area delimited in the Treaty of Paris, the baselines of the
such law. Philippines would still have to be drawn in accordance with RA 9522 because this is the only
way to draw the baselines in conformity with UNCLOS III. The baselines cannot be drawn from
the boundaries or other portions of the rectangular area delineated in the Treaty of Paris, but
RA 9522 is Not Unconstitutional
from the "outermost islands and drying reefs of the archipelago." 24
RA 9522 is a Statutory Tool
to Demarcate the Country’s
Maritime Zones and Continental UNCLOS III and its ancillary baselines laws play no role in the acquisition, enlargement or, as
Shelf Under UNCLOS III, not to petitioners claim, diminution of territory. Under traditional international law typology, States
Delineate Philippine Territory acquire (or conversely, lose) territory through occupation, accretion, cession and
prescription,25 not by executing multilateral treaties on the regulations of sea-use rights or
enacting statutes to comply with the treaty’s terms to delimit maritime zones and continental
Petitioners submit that RA 9522 "dismembers a large portion of the national territory"21 because
shelves. Territorial claims to land features are outside UNCLOS III, and are instead governed by
it discards the pre-UNCLOS III demarcation of Philippine territory under the Treaty of Paris and
the rules on general international law.26
related treaties, successively encoded in the definition of national territory under the 1935, 1973
and 1987 Constitutions. Petitioners theorize that this constitutional definition trumps any treaty or
statutory provision denying the Philippines sovereign control over waters, beyond the territorial
sea recognized at the time of the Treaty of Paris, that Spain supposedly ceded to the United
States. Petitioners argue that from the Treaty of Paris’ technical description, Philippine
RA 9522’s Use of the Framework Thus, as the map below shows, the reach of the exclusive economic zone drawn under RA 9522
of Regime of Islands to Determine the even extends way beyond the waters covered by the rectangular demarcation under the Treaty
Maritime Zones of the KIG and the of Paris. Of course, where there are overlapping exclusive economic zones of opposite or
Scarborough Shoal, not Inconsistent adjacent States, there will have to be a delineation of maritime boundaries in accordance with
with the Philippines’ Claim of Sovereignty UNCLOS III.30
Over these Areas

Petitioners next submit that RA 9522’s use of UNCLOS III’s regime of islands framework to draw
the baselines, and to measure the breadth of the applicable maritime zones of the KIG,
"weakens our territorial claim" over that area.27Petitioners add that the KIG’s (and Scarborough
Shoal’s) exclusion from the Philippine archipelagic baselines results in the loss of "about 15,000
square nautical miles of territorial waters," prejudicing the livelihood of subsistence
fishermen.28 A comparison of the configuration of the baselines drawn under RA 3046 and RA
9522 and the extent of maritime space encompassed by each law, coupled with a reading of the
text of RA 9522 and its congressional deliberations, vis-à-vis the Philippines’ obligations under
UNCLOS III, belie this view.1avvphi1

The configuration of the baselines drawn under RA 3046 and RA 9522 shows that RA 9522
merely followed the basepoints mapped by RA 3046, save for at least nine basepoints that RA
9522 skipped to optimize the location of basepoints and adjust the length of one baseline (and
thus comply with UNCLOS III’s limitation on the maximum length of baselines). Under RA 3046,
as under RA 9522, the KIG and the Scarborough Shoal lie outside of the baselines drawn
around the Philippine archipelago. This undeniable cartographic fact takes the wind out of
petitioners’ argument branding RA 9522 as a statutory renunciation of the Philippines’ claim over
the KIG, assuming that baselines are relevant for this purpose.

Petitioners’ assertion of loss of "about 15,000 square nautical miles of territorial waters" under
RA 9522 is similarly unfounded both in fact and law. On the contrary, RA 9522, by optimizing the
location of basepoints, increased the Philippines’ total maritime space (covering its internal
waters, territorial sea and exclusive economic zone) by 145,216 square nautical miles, as shown
in the table below:29

Extent of maritime area


using RA 3046, as Extent of maritime area
amended, taking into using RA 9522, taking into
account the Treaty of Paris’ account UNCLOS III (in
delimitation (in square square nautical miles)
Further, petitioners’ argument that the KIG now lies outside Philippine territory because the
nautical miles)
baselines that RA 9522 draws do not enclose the KIG is negated by RA 9522 itself. Section 2 of
Internal or archipelagic the law commits to text the Philippines’ continued claim of sovereignty and jurisdiction over the
KIG and the Scarborough Shoal:
waters 166,858 171,435

SEC. 2. The baselines in the following areas over which the Philippines likewise exercises
Territorial Sea 274,136 32,106 sovereignty and jurisdiction shall be determined as "Regime of Islands" under the Republic of
the Philippines consistent with Article 121 of the United Nations Convention on the Law of the
Sea (UNCLOS):
Exclusive Economic
Zone 382,669 a) The Kalayaan Island Group as constituted under Presidential Decree No. 1596 and

TOTAL 440,994 586,210 b) Bajo de Masinloc, also known as Scarborough Shoal. (Emphasis supplied)

Had Congress in RA 9522 enclosed the KIG and the Scarborough Shoal as part of the Philippine
archipelago, adverse legal effects would have ensued. The Philippines would have committed a
breach of two provisions of UNCLOS III. First, Article 47 (3) of UNCLOS III requires that "[t]he west coasts of Luzon down to Palawan were later found to be located either inland or
drawing of such baselines shall not depart to any appreciable extent from the general on water, not on low-water line and drying reefs as prescribed by Article 47.35
configuration of the archipelago." Second, Article 47 (2) of UNCLOS III requires that "the length
of the baselines shall not exceed 100 nautical miles," save for three per cent (3%) of the total
Hence, far from surrendering the Philippines’ claim over the KIG and the Scarborough Shoal,
number of baselines which can reach up to 125 nautical miles. 31
Congress’ decision to classify the KIG and the Scarborough Shoal as "‘Regime[s] of Islands’
under the Republic of the Philippines consistent with Article 121"36 of UNCLOS III manifests the
Although the Philippines has consistently claimed sovereignty over the KIG32 and the Philippine State’s responsible observance of its pacta sunt servanda obligation under UNCLOS
Scarborough Shoal for several decades, these outlying areas are located at an appreciable III. Under Article 121 of UNCLOS III, any "naturally formed area of land, surrounded by water,
distance from the nearest shoreline of the Philippine archipelago, 33 such that any straight which is above water at high tide," such as portions of the KIG, qualifies under the category of
baseline loped around them from the nearest basepoint will inevitably "depart to an appreciable "regime of islands," whose islands generate their own applicable maritime zones. 37
extent from the general configuration of the archipelago."
Statutory Claim Over Sabah under
The principal sponsor of RA 9522 in the Senate, Senator Miriam Defensor-Santiago, took pains RA 5446 Retained
to emphasize the foregoing during the Senate deliberations:
Petitioners’ argument for the invalidity of RA 9522 for its failure to textualize the Philippines’
What we call the Kalayaan Island Group or what the rest of the world call[] the Spratlys and the claim over Sabah in North Borneo is also untenable. Section 2 of RA 5446, which RA 9522 did
Scarborough Shoal are outside our archipelagic baseline because if we put them inside our not repeal, keeps open the door for drawing the baselines of Sabah:
baselines we might be accused of violating the provision of international law which states: "The
drawing of such baseline shall not depart to any appreciable extent from the general
Section 2. The definition of the baselines of the territorial sea of the Philippine Archipelago as
configuration of the archipelago." So sa loob ng ating baseline, dapat magkalapit ang mga
provided in this Act is without prejudice to the delineation of the baselines of the territorial
islands. Dahil malayo ang Scarborough Shoal, hindi natin masasabing malapit sila sa atin
sea around the territory of Sabah, situated in North Borneo, over which the Republic of
although we are still allowed by international law to claim them as our own.
the Philippines has acquired dominion and sovereignty. (Emphasis supplied)

This is called contested islands outside our configuration. We see that our archipelago is defined
UNCLOS III and RA 9522 not
by the orange line which [we] call[] archipelagic baseline. Ngayon, tingnan ninyo ang maliit na
Incompatible with the Constitution’s
circle doon sa itaas, that is Scarborough Shoal, itong malaking circle sa ibaba, that is Kalayaan
Delineation of Internal Waters
Group or the Spratlys. Malayo na sila sa ating archipelago kaya kung ilihis pa natin ang dating
archipelagic baselines para lamang masama itong dalawang circles, hindi na sila magkalapit at
baka hindi na tatanggapin ng United Nations because of the rule that it should follow the natural As their final argument against the validity of RA 9522, petitioners contend that the law
configuration of the archipelago.34 (Emphasis supplied) unconstitutionally "converts" internal waters into archipelagic waters, hence subjecting these
waters to the right of innocent and sea lanes passage under UNCLOS III, including overflight.
Petitioners extrapolate that these passage rights indubitably expose Philippine internal waters to
Similarly, the length of one baseline that RA 3046 drew exceeded UNCLOS III’s
nuclear and maritime pollution hazards, in violation of the Constitution. 38
limits.1avvphi1 The need to shorten this baseline, and in addition, to optimize the location of
basepoints using current maps, became imperative as discussed by respondents:
Whether referred to as Philippine "internal waters" under Article I of the Constitution 39 or as
"archipelagic waters" under UNCLOS III (Article 49 [1]), the Philippines exercises sovereignty
[T]he amendment of the baselines law was necessary to enable the Philippines to draw the outer
over the body of water lying landward of the baselines, including the air space over it and the
limits of its maritime zones including the extended continental shelf in the manner provided by
submarine areas underneath. UNCLOS III affirms this:
Article 47 of [UNCLOS III]. As defined by R.A. 3046, as amended by R.A. 5446, the baselines
suffer from some technical deficiencies, to wit:
Article 49. Legal status of archipelagic waters, of the air space over archipelagic waters and of
their bed and subsoil. –
1. The length of the baseline across Moro Gulf (from Middle of 3 Rock Awash to
Tongquil Point) is 140.06 nautical miles x x x. This exceeds the maximum length
allowed under Article 47(2) of the [UNCLOS III], which states that "The length of such 1. The sovereignty of an archipelagic State extends to the waters enclosed by
baselines shall not exceed 100 nautical miles, except that up to 3 per cent of the total the archipelagic baselines drawn in accordance with article 47, described as
number of baselines enclosing any archipelago may exceed that length, up to a archipelagic waters, regardless of their depth or distance from the coast.
maximum length of 125 nautical miles."
2. This sovereignty extends to the air space over the archipelagic waters, as
2. The selection of basepoints is not optimal. At least 9 basepoints can be skipped or well as to their bed and subsoil, and the resources contained therein.
deleted from the baselines system. This will enclose an additional 2,195 nautical miles
of water.
xxxx

3. Finally, the basepoints were drawn from maps existing in 1968, and not established
by geodetic survey methods. Accordingly, some of the points, particularly along the
4. The regime of archipelagic sea lanes passage established in this Part shall not in UNCLOS III favors States with a long coastline like the Philippines. UNCLOS III creates a sui
other respects affect the status of the archipelagic waters, including the sea generis maritime space – the exclusive economic zone – in waters previously part of the high
lanes, or the exercise by the archipelagic State of its sovereignty over such seas. UNCLOS III grants new rights to coastal States to exclusively exploit the resources found
waters and their air space, bed and subsoil, and the resources contained within this zone up to 200 nautical miles.53 UNCLOS III, however, preserves the traditional
therein. (Emphasis supplied) freedom of navigation of other States that attached to this zone beyond the territorial sea before
UNCLOS III.
The fact of sovereignty, however, does not preclude the operation of municipal and international
law norms subjecting the territorial sea or archipelagic waters to necessary, if not marginal, RA 9522 and the Philippines’ Maritime Zones
burdens in the interest of maintaining unimpeded, expeditious international navigation,
consistent with the international law principle of freedom of navigation. Thus, domestically, the
Petitioners hold the view that, based on the permissive text of UNCLOS III, Congress was not
political branches of the Philippine government, in the competent discharge of their constitutional
bound to pass RA 9522.54 We have looked at the relevant provision of UNCLOS III55 and we find
powers, may pass legislation designating routes within the archipelagic waters to regulate
petitioners’ reading plausible. Nevertheless, the prerogative of choosing this option belongs to
innocent and sea lanes passage.40 Indeed, bills drawing nautical highways for sea lanes
Congress, not to this Court. Moreover, the luxury of choosing this option comes at a very steep
passage are now pending in Congress.41
price. Absent an UNCLOS III compliant baselines law, an archipelagic State like the Philippines
will find itself devoid of internationally acceptable baselines from where the breadth of its
In the absence of municipal legislation, international law norms, now codified in UNCLOS III, maritime zones and continental shelf is measured. This is recipe for a two-fronted disaster: first,
operate to grant innocent passage rights over the territorial sea or archipelagic waters, subject to it sends an open invitation to the seafaring powers to freely enter and exploit the resources in
the treaty’s limitations and conditions for their exercise.42 Significantly, the right of innocent the waters and submarine areas around our archipelago; and second, it weakens the country’s
passage is a customary international law,43 thus automatically incorporated in the corpus of case in any international dispute over Philippine maritime space. These are consequences
Philippine law.44 No modern State can validly invoke its sovereignty to absolutely forbid innocent Congress wisely avoided.
passage that is exercised in accordance with customary international law without risking
retaliatory measures from the international community.
The enactment of UNCLOS III compliant baselines law for the Philippine archipelago and
adjacent areas, as embodied in RA 9522, allows an internationally-recognized delimitation of the
The fact that for archipelagic States, their archipelagic waters are subject to both the right of breadth of the Philippines’ maritime zones and continental shelf. RA 9522 is therefore a most
innocent passage and sea lanes passage45 does not place them in lesser footing vis-à- vital step on the part of the Philippines in safeguarding its maritime zones, consistent with the
vis continental coastal States which are subject, in their territorial sea, to the right of innocent Constitution and our national interest.
passage and the right of transit passage through international straits. The imposition of these
passage rights through archipelagic waters under UNCLOS III was a concession by archipelagic
WHEREFORE, we DISMISS the petition.
States, in exchange for their right to claim all the waters landward of their baselines, regardless
of their depth or distance from the coast, as archipelagic waters subject to their territorial
sovereignty. More importantly, the recognition of archipelagic States’ archipelago and the waters SO ORDERED.
enclosed by their baselines as one cohesive entity prevents the treatment of their islands as
separate islands under UNCLOS III.46 Separate islands generate their own maritime zones,
placing the waters between islands separated by more than 24 nautical miles beyond the States’
territorial sovereignty, subjecting these waters to the rights of other States under UNCLOS III. 47

Petitioners’ invocation of non-executory constitutional provisions in Article II (Declaration of


Principles and State Policies)48 must also fail. Our present state of jurisprudence considers the
provisions in Article II as mere legislative guides, which, absent enabling legislation, "do not
embody judicially enforceable constitutional rights x x x."49 Article II provisions serve as guides in
formulating and interpreting implementing legislation, as well as in interpreting executory
provisions of the Constitution. Although Oposa v. Factoran50 treated the right to a healthful and
balanced ecology under Section 16 of Article II as an exception, the present petition lacks factual
basis to substantiate the claimed constitutional violation. The other provisions petitioners cite,
relating to the protection of marine wealth (Article XII, Section 2, paragraph 251 ) and subsistence
fishermen (Article XIII, Section 752 ), are not violated by RA 9522.

In fact, the demarcation of the baselines enables the Philippines to delimit its exclusive
economic zone, reserving solely to the Philippines the exploitation of all living and non-living
resources within such zone. Such a maritime delineation binds the international community since
the delineation is in strict observance of UNCLOS III. If the maritime delineation is contrary to
UNCLOS III, the international community will of course reject it and will refuse to be bound by it.
G.R. No. L-9959 December 13, 1916 5. That the court erred in holding in its decision that there is no title for the prescription
of this suit brought by the Insular Government against the Monte de Piedad y Caja de
Ahorros for the reimbursement of the eighty thousand dollars ($80,000) given to it by
THE GOVERNMENT OF THE PHILIPPINE ISLANDS, represented by the Treasurer of the
the late Spanish Government of these Islands.
Philippine Islands,plaintiff-appellee,
vs.
EL MONTE DE PIEDAD Y CAJA DE AHORRAS DE MANILA, defendant-appellant. 6. That the court erred in sentencing the Monte de Piedad y Caja de Ahorros to
reimburse the Philippine Government in the sum of eighty thousand dollars ($80,000)
gold coin, or the equivalent thereof in the present legal tender currency in circulation,
TRENT, J.:
with legal interest thereon from February 28th, 1912, and the costs of this suit.

About $400,000, were subscribed and paid into the treasury of the Philippine Islands by the
In the royal order of June 29, 1879, the Governor-General of the Philippine Islands was directed
inhabitants of the Spanish Dominions of the relief of those damaged by the earthquake which
to inform the home Government in what manner the indemnity might be paid to which, by virtue
took place in the Philippine Islands on June 3, 1863. Subsequent thereto and on October 6 of
of the resolutions of the relief board, the persons who suffered damage by the earthquake might
that year, a central relief board was appointed, by authority of the King of Spain, to distribute the
be entitled, in order to perform the sacred obligation which the Government of Spain had
moneys thus voluntarily contributed. After a thorough investigation and consideration, the relief
assumed toward the donors.
board allotted $365,703.50 to the various sufferers named in its resolution, dated September 22,
1866, and, by order of the Governor-General of the Philippine Islands, a list of these allotments,
together with the names of those entitled thereto, was published in the Official Gazette of Manila The next pertinent document in order is the defendant's petition, dated February 1, 1883,
dated April 7, 1870. There was later distributed, inaccordance with the above-mentioned addressed to the Governor-General of the Philippine Islands, which reads:
allotments, the sum of $30,299.65, leaving a balance of S365,403.85 for distribution. Upon the
petition of the governing body of the Monte de Piedad, dated February 1, 1833, the Philippine
Board of Directors of the Monte de Piedad of Manila Presidencia.
Government, by order dated the 1st of that month, directed its treasurer to turn over to the Monte
de Piedad the sum of $80,000 of the relief fund in installments of $20,000 each. These amounts
were received on the following dates: February 15, March 12, April 14, and June 2, 1883, and Excellency: The Board of Directors of the Monte de Piedad y Caja de Ahorros of
are still in the possession of the Monte de Piedad. On account of various petitions of the Manila informs your Excellency, First: That the funds which it has up to the present
persons, and heirs of others to whom the above-mentioned allotments were made by the central been able to dispose of have been exhausted in loans on jewelry, and there only
relief board for the payment of those amounts, the Philippine Islands to bring suit against remains the sum of one thousand and odd pesos, which will be expended between to-
the Monte de Piedad a recover, "through the Attorney-General and in representation of the day and day after tomorrow. Second: That, to maintain the credit of the establishment,
Government of the Philippine Islands," the $80.000, together with interest, for the benefit of which would be greatly injured were its operations suspended, it is necessary to
those persons or their heirs appearing in the list of names published in the Official Gazette procure money. Third: That your Excellency has proposed to His Majesty's
instituted on May 3, 1912, by the Government of the Philippine Islands, represented by the Government to apply to the funds of the Monte de Piedad a part of the funds held in
Insular Treasurer, and after due trial, judgment was entered in favor of the plaintiff for the sum of the treasury derived form the national subscription for the relief of the distress caused
$80,000 gold or its equivalent in Philippine currency, together with legal interest from February by the earthquake of 1863. Fourth: That in the public treasury there is held at the
28, 1912, and the costs of the cause. The defendant appealed and makes the following disposal of the central earthquake relief board over $1090,000 which was deposited in
assignment of errors: the said treasury by order of your general Government, it having been transferred
thereto from the Spanish-Filipino Bank where it had been held. fifth: That in the
straightened circumstances of the moment, your Excellency can, to avert impending
1. The court erred in not finding that the eighty thousand dollars ($80,000), give to
disaster to the Monte de Piedad, order that, out of that sum of one hundred thousand
the Monte de Piedad y Caja de Ahorros, were so given as a donation subject to one
pesos held in the Treasury at the disposal of the central relief board, there be
condition, to wit: the return of such sum of money to the Spanish Government of these
transferred to the Monte de Piedad the sum of $80,000, there to be held under the
Islands, within eight days following the day when claimed, in case the Supreme
same conditions as at present in the Treasury, to wit, at the disposal of the Relief
Government of Spain should not approve the action taken by the former government.
Board. Sixth: That should this transfer not be approved for any reason, either because
of the failure of His Majesty's Government to approve the proposal made by your
2. The court erred in not having decreed that this donation had been cleared; said Excellency relative to the application to the needs of the Monte de Piedad of a pat of
eighty thousand dollars ($80,000) being at present the exclusive property of the the subscription intended to believe the distress caused by the earthquake of 1863, or
appellant the Monte de Piedad y Caja de Ahorros. for any other reason, the board of directors of the Monte de Piedad obligates itself to
return any sums which it may have received on account of the eighty thousand pesos,
or the whole thereof, should it have received the same, by securing a loan from
3. That the court erred in stating that the Government of the Philippine Islands has
whichever bank or banks may lend it the money at the cheapest rate upon the security
subrogated the Spanish Government in its rights, as regards an important sum of
of pawned jewelry. — This is an urgent measure to save the Monte de Piedad in the
money resulting from a national subscription opened by reason of the earthquake of
present crisis and the board of directors trusts to secure your Excellency's entire
June 3, 1863, in these Island.
cooperation and that of the other officials who have take part in the transaction.

4. That the court erred in not declaring that Act Numbered 2109, passed by the
The Governor-General's resolution on the foregoing petition is as follows:
Philippine Legislature on January 30, 1912, is unconstitutional.
GENERAL GOVERNMENT OF THE PHILIPPINES. Third. The Intendencia General de Hacienda shall forthwith, and in preference to all
MANILA, February 1, 1883. other work, proceed to prepare the necessary papers so that with the least possible
delay the payment referred to may be made and the danger that menaces the Monte
de Piedad of having to suspend its operations may be averted.
In view of the foregoing petition addressed to me by the board of directors of
the Monte de Piedad of this city, in which it is stated that the funds which the said
institution counted upon are nearly all invested in loans on jewelry and that the small H. M. Government shall be advised hereof.lawphi1.net
account remaining will scarcely suffice to cover the transactions of the next two days, (Signed) P. DE RIVERA.
for which reason it entreats the general Government that, in pursuance of its
telegraphic advice to H. M. Government, the latter direct that there be turned over to
By the royal order of December 3, 1892, the Governor-General of the Philippine Islands was
said Monte de Piedad $80,000 out of the funds in the public treasury obtained from
ordered to "inform this ministerio what is the total sum available at the present time, taking into
the national subscription for the relief of the distress caused by the earthquake of
consideration the sums delivered to the Monte de Piedad pursuant to the decree issued by your
1863, said board obligating itself to return this sum should H. M. Government, for any
general Government on February 1, 1883," and after the rights of the claimants, whose names
reason, not approve the said proposal, and for this purpose it will procure funds by
were published in the Official Gazette of Manila on April 7, 1870, and their heirs had been
means of loans raised on pawned jewelry; it stated further that if the aid so solicited is
established, as therein provided, as such persons "have an unquestionable right to be paid the
not furnished, it will be compelled to suspend operations, which would seriously injure
donations assigned to them therein, your general Government shall convoke them all within a
the credit of so beneficient an institution; and in view of the report upon the matter
reasonable period and shall pay their shares to such as shall identify themselves, without regard
made by the Intendencia General de Hacienda; and considering the fact that the
to their financial status," and finally "that when all the proceedings and operations herein
public treasury has on hand a much greater sum from the source mentioned than that
mentioned have been concluded and the Government can consider itself free from all kinds of
solicited; and considering that this general Government has submitted for the
claims on the part of those interested in the distribution of the funds deposited in the vaults of the
determination of H. M. Government that the balance which, after strictly applying the
Treasury, such action may be taken as the circumstances shall require, after first consulting the
proceeds obtained from the subscription referred to, may remain as a surplus should
relief board and your general Government and taking account of what sums have been delivered
be delivered to the Monte de Piedad, either as a donation, or as a loan upon the
to the Monte de Piedad and those that were expended in 1888 to relieve public calamities," and
security of the credit of the institution, believing that in so doing the wishes of the
"in order that all the points in connection with the proceedings had as a result of the earthquake
donors would be faithfully interpreted inasmuch as those wishes were no other than to
be clearly understood, it is indispensable that the offices hereinbefore mentioned comply with
relieve distress, an act of charity which is exercised in the highest degree by
the provisions contained in paragraphs 2 and 3 of the royal order of June 25, 1879." On receipt
the Monte de Piedad, for it liberates needy person from the pernicious effects of usury;
of this Finance order by the Governor-General, the Department of Finance was called upon for a
and
report in reference to the $80,000 turned over to the defendant, and that Department's report to
the Governor-General dated June 28, 1893, reads:
Considering that the lofty purposes that brought about the creation of the pious
institution referred to would be frustrated, and that the great and laudable work of its
Intendencia General de Hacienda de Filipinas (General Treasury of the Philippines) —
establishment, and that the great and laudable and valuable if the aid it urgently seeks
Excellency. — By Royal Order No. 1044 of December 3, last, it is provided that the
is not granted, since the suspension of its operations would seriously and regrettably
persons who sustained losses by the earthquakes that occurred in your capital in the
damage the ever-growing credit of the Monte de Piedad; and
year 1863 shall be paid the amounts allotted to them out of the sums sent from Spain
for this purpose, with observance of the rules specified in the said royal order, one of
Considering that if such a thing would at any time cause deep distress in the public them being that before making the payment to the interested parties the assets shall
mind, it might be said that at the present juncture it would assume the nature of a be reduced to money. These assets, during the long period of time that has elapsed
disturbance of public order because of the extreme poverty of the poorer classes since they were turned over to the Treasury of the Philippine Islands, were used to
resulting from the late calamities, and because it is the only institution which can cover the general needs of the appropriation, a part besides being invested in the
mitigate the effects of such poverty; and relief of charitable institutions and another part to meet pressing needs occasioned by
public calamities. On January 30, last, your Excellency was please to order the
fulfillment of that sovereign mandate and referred the same to this Intendencia for its
Considering that no reasonable objection can be made to granting the request herein
information and the purposes desired (that is, for compliance with its directions and,
contained, for the funds in question are sufficiently secured in the unlikely event that
as aforesaid, one of these being the liquidation, recovery, and deposit with the
H> M. Government does not approve the recommendation mentioned, this general
Treasury of the sums paid out of that fund and which were expended in a different way
Government, in the exercise of the extraordinary powers conferred upon it and in
from that intended by the donors) and this Intendencia believed the moment had
conformity with the report of the Intendencia de Hacienda, resolves as follows:
arrived to claim from the board of directors of the Monte de Piedad y Caja de
Ahorros the sum of 80,000 pesos which, by decree of your general Government of the
First. Authority is hereby given to deliver to the Monte de Piedad, out of the sum held date of February 1, 1883, was loaned to it out of the said funds, the (Monte de Piedad)
in the public treasury of these Islands obtained from the national subscription opened obligating itself to return the same within the period of eight days if H. M. Government
by reason of the earthquakes of 1863, amounts up to the sum $80,000, as its needs did not approve the delivery. On this Intendencia's demanding from the Monte de
may require, in installments of $20,000. Piedad the eighty thousand pesos, thus complying with the provisions of the Royal
Order, it was to be supposed that no objection to its return would be made by
the Monte de Piedad for, when it received the loan, it formally engaged itself to return
Second. The board of directors of the Monte de Piedad is solemnly bound to return, it; and, besides, it was indisputable that the moment to do so had arrived, inasmuch as
within eight days after demand, the sums it may have so received, if H. M.
H. M. Government, in ordering that the assets of the earthquake relief fund should he
Government does not approve this resolution. collected, makes express mention of the 80,000 pesos loaned to the Monte de
Piedad, without doubt considering as sufficient the period of ten years during which it To the Attorney-General of the Department of Justice of the Philippine
has been using this large sum which lawfully belongs to their persons. Islands.
This Intendencia also supposed that the Monte de Piedad no longer needed the
amount of that loan, inasmuch as, far from investing it in beneficient transactions, it
SIR: In reply to your courteous letter of the 16th inst., in which you request information
had turned the whole amount into the voluntary deposit funds bearing 5 per cent
from this office as to when and for what purpose the Spanish Government delivered to
interests, the result of this operation being that the debtor loaned to the creditor on
the Monte de Piedad eighty thousand pesos obtained from the subscription opened in
interest what the former had gratuitously received. But the Monte de Piedad, instead
connection with the earthquake of 1863, as well as any other information that might be
of fulfilling the promise it made on receiving the sum, after repeated demands refused
useful for the report which your office is called upon to furnish, I must state to your
to return the money on the ground that only your Excellency, and not
department that the books kept in these Pious Institutions, and which have been
the Intendencia (Treasury), is entitled to order the reimbursement, taking no account
consulted for the purpose, show that on the 15th of February, 1883, they received as a
of the fact that this Intendencia was acting in the discharge of a sovereign command,
reimbursable loan and without interest, twenty thousand pesos, which they deposited
the fulfillment of which your Excellency was pleased to order; and on the further
with their own funds. On the same account and on each of the dates of March 12,
ground that the sum of 80,000 pesos which it received from the fund intended for the
April 14 and June 2 of the said year, 1883, they also received and turned into their
earthquake victims was not received as a loan, but as a donation, this in the opinion of
funds a like sum of twenty thousand pesos, making a total of eighty thousand pesos.
this Intendencia, erroneously interpreting both the last royal order which directed the
— (Signed) Emilio Moreta.
apportionment of the amount of the subscription raised in the year 1863 and the
superior decree which granted the loan, inasmuch as in this letter no donation is made
to the Monte de Piedad of the 80,000 pesos, but simply a loan; besides, no donation I hereby certify that the foregoing is a literal copy of that found in the letter book No. 2
whatever could be made of funds derived from a private subscription raised for a of those Pious Institutions.
specific purpose, which funds are already distributed and the names of the
beneficiaries have been published in the Gaceta, there being lacking only the mere
material act of the delivery, which has been unduly delayed. In view of the unexpected Manila, November 19, 1913
(Sgd.) EMILIO LAZCANOTEGUI,
reply made by the Monte de Piedad, and believing it useless to insist further in the
matter of the claim for the aforementioned loan, or to argue in support thereof, Secretary
this Intendencia believes the intervention of your Excellency necessary in this matter,
if the royal Order No. 1044 of December 3, last, is to be complied with, and for this (Sgd.) O. K. EMILIO MORETA,
purpose I beg your Excellency kindly to order the Monte de Piedad to reimburse within Managing Director.
the period of eight days the 80,000 which it owes, and that you give this Intendencia
power to carry out the provisions of the said royal order. I must call to the attention of
your Excellency that the said pious establishment, during the last few days and after The foregoing documentary evidence shows the nature of the transactions which took place
demand was made upon it, has endorsed to the Spanish-Filipino Bank nearly the between the Government of Spain and the Philippine Government on the one side and
whole of the sum which it had on deposit in the general deposit funds. the Monte de Piedad on the other, concerning the $80,000. The Monte de Piedad, after setting
forth in its petition to the Governor-General its financial condition and its absolute necessity for
more working capital, asked that out of the sum of $100,000 held in the Treasury of the
The record in the case under consideration fails to disclose any further definite action taken by Philippine Islands, at the disposal of the central relief board, there be transferred to it the sum of
either the Philippine Government or the Spanish Government in regard to the $80,000 turned $80,000 to be held under the same conditions, to wit, "at the disposal of the relief board." The
over to the Monte de Piedad. Monte de Piedad agreed that if the transfer of these funds should not be approved by the
Government of Spain, the same would be returned forthwith. It did not ask that the $80,000 be
In the defendant's general ledger the following entries appear: "Public Treasury: February 15, given to it as a donation. The Governor-General, after reciting the substance of the petition,
1883, $20,000; March 12, 1883, $20,000; April 14, 1883, $20,000; June 2, 1883, $20,000, total stated that "this general Government has submitted for the determination of H. M. Government
that the balance which, after strictly applying the proceeds obtained from the subscription
$80,000." The book entry for this total is as follows: "To the public Treasury derived from the
subscription for the earthquake of 1863, $80,000 received from general Treasury as a returnable referred to, may remain as a surplus, should be delivered to the Monte de Piedad, either as a
loan, and without interest." The account was carried in this manner until January 1, 1899, when donation, or as a loan upon the security of the credit of the institution," and "considering that no
reasonable objection can be made to granting the request herein contained," directed the
it was closed by transferring the amount to an account called "Sagrada Mitra," which latter
account was a loan of $15,000 made to the defendant by the Archbishop of Manila, without transfer of the $80,000 to be made with the understanding that "the Board of Directors of
interest, thereby placing the "Sagrada Mitra" account at $95,000 instead of $15,000. The above- the Monte de Piedad is solemnly bound to return, within eight days after demand, the sums it
may have so received, if H. M. Government does not approve this resolution." It will be noted
mentioned journal entry for January 1, 1899, reads: "Sagrada Mitra and subscription, balance of
these two account which on this date are united in accordance with an order of the Exmo. Sr. that the first and only time the word "donation" was used in connection with the $80,000 appears
Presidente of the Council transmitted verbally to the Presidente Gerente of these institutions, in this resolution of the Governor-General. It may be inferred from the royal orders that the
Madrid Government did tacitly approve of the transfer of the $80,000 to the Monte de Piedad as
$95,000."
a loan without interest, but that Government certainly did not approve such transfer as a
donation for the reason that the Governor-General was directed by the royal order of December
On March 16, 1902, the Philippine government called upon the defendant for information 3, 1892, to inform the Madrid Government of the total available sum of the earthquake fund,
concerning the status of the $80,000 and received the following reply: "taking into consideration the sums delivered to the Monte de Piedad pursuant to the decree
issued by your general Government on February 1, 1883." This language, nothing else
appearing, might admit of the interpretation that the Madrid Government did not intend that the
MANILA, March 31, 1902.
Governor-General of the Philippine Islands should include the $80,000 in the total available sum,
but when considered in connection with the report of the Department of Finance there can be no
doubt that it was so intended. That report refers expressly to the royal order of December 3d, The contention of counsel, as thus stated, in untenable for two reason, (1) because such
and sets forth in detail the action taken in order to secure the return of the $80,000. The contention is based upon the erroneous theory that the sum in question was a donation to
Department of Finance, acting under the orders of the Governor-General, understood that the the Monte de Piedad and not a loan, and (2) because the charity founded by the donations for
$80,000 was transferred to the Monte de Piedad well knew that it received this sum as a loan the earthquake sufferers is not and never was intended to be an ecclesiastical pious work. The
interest." The amount was thus carried in its books until January, 1899, when it was transferred first proposition has already been decided adversely to the defendant's contention. As to the
to the account of the "Sagrada Mitra" and was thereafter known as the "Sagrada Mitra and second, the record shows clearly that the fund was given by the donors for a specific and
subscription account." Furthermore, the Monte de Piedad recognized and considered as late as definite purpose — the relief of the earthquake sufferers — and for no other purpose. The
March 31, 1902, that it received the $80,000 "as a returnable loan, and without interest." money was turned over to the Spanish Government to be devoted to that purpose. The Spanish
Therefore, there cannot be the slightest doubt the fact that the Monte de Piedad received the Government remitted the money to the Philippine Government to be distributed among the
$80,000 as a mere loan or deposit and not as a donation. Consequently, the first alleged error is suffers. All officials, including the King of Spain and the Governor-General of the Philippine
entirely without foundation. Islands, who took part in the disposal of the fund, acted in their purely civil, official capacity, and
the fact that they might have belonged to a certain church had nothing to do with their acts in this
matter. The church, as such, had nothing to do with the fund in any way whatever until the
Counsel for the defendant, in support of their third assignment of error, say in their principal brief
$80,000 reached the coffers of the Monte de Piedad (an institution under the control of the
that:
church) as a loan or deposit. If the charity in question had been founded as an ecclesiastical
pious work, the King of Spain and the Governor-General, in their capacities as vicar-general of
The Spanish nation was professedly Roman Catholic and its King enjoyed the the Indies and as royal vice-patron, respectively, would have disposed of the fund as such and
distinction of being deputy ex officio of the Holy See and Apostolic Vicar-General of not in their civil capacities, and such functions could not have been transferred to the present
the Indies, and as such it was his duty to protect all pious works and charitable Philippine Government, because the right to so act would have arisen out of the special
institutions in his kingdoms, especially those of the Indies; among the latter was agreement between the Government of Spain and the Holy See, based on the union of the
the Monte de Piedad of the Philippines, of which said King and his deputy the church and state which was completely separated with the change of sovereignty.
Governor-General of the Philippines, as royal vice-patron, were, in a special and
peculiar manner, the protectors; the latter, as a result of the cession of the Philippine
And in their supplemental brief counsel say:
Islands, Implicitly renounced this high office and tacitly returned it to the Holy See,
now represented by the Archbishop of Manila; the national subscription in question
was a kind of foundation or pious work, for a charitable purpose in these Islands; and By the conceded facts the money in question is part of a charitable subscription. The
the entire subscription not being needed for its original purpose, the royal vice-patron, donors were persons in Spain, the trustee was the Spanish Government, the donees,
with the consent of the King, gave the surplus thereof to an analogous purpose; the the cestuis que trustent, were certain persons in the Philippine Islands. The whole
fulfillment of all these things involved, in the majority, if not in all cases, faithful matter is one of trusteeship. This is undisputed and indisputable. It follows that the
compliance with the duty imposed upon him by the Holy See, when it conferred upon Spanish Government at no time was the owner of the fund. Not being the owner of the
him the royal patronage of the Indies, a thing that touched him very closely in his fund it could not transfer the ownership. Whether or not it could transfer its trusteeship
conscience and religion; the cessionary Government though Christian, was not it certainly never has expressly done so and the general terms of property transfer in
Roman Catholic and prided itself on its policy of non-interference in religious matters, the Treaty of Paris are wholly insufficient for such a purpose even could Spain have
and inveterately maintained a complete separation between the ecclesiastical and civil transferred its trusteeship without the consent of the donors and even could the United
powers. States, as a Government, have accepted such a trust under any power granted to it by
the thirteen original States in the Constitution, which is more than doubtful. It follows
further that this Government is not a proper party to the action. The only persons who
In view of these circumstances it must be quite clear that, even without the express
could claim to be damaged by this payment to the Monte, if it was unlawful, are the
provisions of the Treaty of Paris, which apparently expressly exclude such an idea, it
donors or the cestuis que trustent, and this Government is neither.
did not befit the honor of either of the contracting parties to subrogate to the American
Government in lieu of the Spanish Government anything respecting the disposition of
the funds delivered by the latter to the Monte de Piedad. The same reasons that If "the whole matter is one of trusteeship," and it being true that the Spanish Government could
induced the Spanish Government to take over such things would result in great not, as counsel say, transfer the ownership of the fund to the Monte de Piedad, the question
inconvenience to the American Government in attempting to do so. The question was arises, who may sue to recover this loan? It needs no argument to show that the Spanish or
such a delicate one, for the reason that it affected the conscience, deeply religious, of Philippine Government, as trustee, could maintain an action for this purpose had there been no
the King of Spain, that it cannot be believed that it was ever his intention to confide the change of sovereignty and if the right of action has not prescribed. But those governments were
exercise thereof to a Government like the American. (U. S. vs. Arredondo, 6 Pet. [U. something more than mere common law trustees of the fund. In order to determine their exact
S.], 711.) status with reference to this fund, it is necessary to examine the law in force at the time there
transactions took place, which are the law of June 20, 1894, the royal decree of April 27. 1875,
and the instructions promulgated on the latter date. These legal provisions were applicable to
It is thus seen that the American Government did not subrogate the Spanish
the Philippine Islands (Benedicto vs. De la Rama, 3 Phil. Rep., 34)
Government or rather, the King of Spain, in this regard; and as the condition annexed
to the donation was lawful and possible of fulfillment at the time the contract was
made, but became impossible of fulfillment by the cession made by the Spanish The funds collected as a result of the national subscription opened in Spain by royal order of the
Government in these Islands, compliance therewith is excused and the contract has Spanish Government and which were remitted to the Philippine Government to be distributed
been cleared thereof. among the earthquake sufferers by the Central Relief Board constituted, under article 1 of the
law of June 20, 1894, and article 2 of the instructions of April 27, 1875, a special charity of a
temporary nature as distinguished from a permanent public charitable institution. As the Spanish
Government initiated the creation of the fund and as the donors turned their contributions over to regulate, and to some extent control charities and charitable institutions. The present sovereign,
that Government, it became the duty of the latter, under article 7 of the instructions, to exercise in exempting "provident institutions, savings banks, etc.," all of which are in the nature of
supervision and control over the moneys thus collected to the end that the will of the donors charitable institutions, from taxation, placed such institutions, in so far as the investment in
should be carried out. The relief board had no power whatever to dispose of the funds confided securities are concerned, under the general supervision of the Insular Treasurer (paragraph 4 of
to its charge for other purposes than to distribute them among the sufferers, because paragraph section 111 of Act No. 1189; see also Act No. 701).
3 of article 11 of the instructions conferred the power upon the secretary of the interior of Spain,
and no other, to dispose of the surplus funds, should there be any, by assigning them to some
Furthermore, upon the cession of the Philippine Islands the prerogatives of he crown of Spain
other charitable purpose or institution. The secretary could not dispose of any of the funds in this
devolved upon he United States. In Magill vs. Brown (16 Fed. Cas., 408), quoted with approval
manner so long as they were necessary for the specific purpose for which they were contributed.
in Mormon Charch vs. United States (136 U. S.,1, 57), the court said:
The secretary had the power, under the law above mentioned to appoint and totally or partially
change the personnel of the relief board and to authorize the board to defend the rights of the
charity in the courts. The authority of the board consisted only in carrying out the will of the The Revolution devolved on the State all the transcendent power of Parliament, and
donors as directed by the Government whose duty it was to watch over the acts of the board and the prerogative of the crown, and gave their Acts the same force and effect.
to see that the funds were applied to the purposes for which they were contributed .The
secretary of the interior, as the representative of His Majesty's Government, exercised these
powers and duties through the Governor-General of the Philippine Islands. The Governments of In Fontain vs. Ravenel (17 Hw., 369, 384), Mr. Justice McLean, delivering the opinion of the
court in a charity case, said:
Spain and of the Philippine Islands in complying with their duties conferred upon them by law,
acted in their governmental capacities in attempting to carry out the intention of the contributors.
It will this be seen that those governments were something more, as we have said, than mere When this country achieved its independence, the prerogatives of the crown devolved
trustees of the fund. upon the people of the States. And this power still remains with them except so fact as
they have delegated a portion of it to the Federal Government. The sovereign will is
It is further contended that the obligation on the part of the Monte de Piedad to return the made known to us by legislative enactment. The State as a sovereign, is the parens
patriae.
$80,000 to the Government, even considering it a loan, was wiped out on the change of
sovereignty, or inn other words, the present Philippine Government cannot maintain this action
for that reason. This contention, if true, "must result from settled principles of rigid law," as it Chancelor Kent says:
cannot rest upon any title to the fund in the Monte de Piedad acquired prior to such change.
While the obligation to return the $80,000 to the Spanish Government was still pending, war
between the United States and Spain ensued. Under the Treaty of Paris of December 10, 1898, In this country, the legislature or government of the State, as parens patriae, has the
the Archipelago, known as the Philippine Islands, was ceded to the United States, the latter right to enforce all charities of public nature, by virtue of its general superintending
agreeing to pay Spain the sum of $20,000,000. Under the first paragraph of the eighth article, authority over the public interests, where no other person is entrusted with it. (4 Kent
Spain relinquished to the United States "all buildings, wharves, barracks, forts, structures, public Com., 508, note.)
highways, and other immovable property which, in conformity with law, belonged to the public
domain, and as such belonged to the crown of Spain." As the $80,000 were not included therein, The Supreme Court of the United States in Mormon Church vs. United States, supra, after
it is said that the right to recover this amount did not, therefore, pass to the present sovereign. approving also the last quotations, said:
This, in our opinion, does not follow as a necessary consequence, as the right to recover does
not rest upon the proposition that the $80,000 must be "other immovable property" mentioned in
article 8 of the treaty, but upon contractual obligations incurred before the Philippine Islands This prerogative of parens patriae is inherent in the supreme power of every State,
were ceded to the United States. We will not inquire what effect his cession had upon the law of whether that power is lodged in a royal person or in the legislature, and has no affinity
June 20, 1849, the royal decree of April 27, 1875, and the instructions promulgated on the latter to those arbitrary powers which are sometimes exerted by irresponsible monarchs to
date. In Vilas vs.Manila (220 U. S., 345), the court said: the great detriment of the people and the destruction of their liberties. On the contrary,
it is a most beneficient functions, and often necessary to be exercised in the interest of
humanity, and for the prevention of injury to those who cannot protect themselves.
That there is a total abrogation of the former political relations of the inhabitants of the
ceded region is obvious. That all laws theretofore in force which are in conflict with the
political character, constitution, or institutions of the substituted sovereign, lose their The court in the same case, after quoting from Sohier vs. Mass. General Hospital (3 Cush., 483,
force, is also plain. (Alvarez y Sanchez vs. United States, 216 U. S., 167.) But it is 497), wherein the latter court held that it is deemed indispensible that there should be a power in
equally settled in the same public law that the great body of municipal law which the legislature to authorize the same of the estates of in facts, idiots, insane persons, and
regulates private and domestic rights continues in force until abrogated or changed by persons not known, or not in being, who cannot act for themselves, said:
the new ruler.
These remarks in reference to in facts, insane persons and person not known, or not
If the above-mentioned legal provisions are in conflict with the political character, constitution or in being, apply to the beneficiaries of charities, who are often in capable of vindicating
institutions of the new sovereign, they became inoperative or lost their force upon the cession of their rights, and justly look for protection to the sovereign authority, acting as parens
the Philippine Islands to the United States, but if they are among "that great body of municipal patriae. They show that this beneficient functions has not ceased t exist under the
law which regulates private and domestic rights," they continued in force and are still in force change of government from a monarchy to a republic; but that it now resides in the
unless they have been repealed by the present Government. That they fall within the latter class legislative department, ready to be called into exercise whenever required for the
is clear from their very nature and character. They are laws which are not political in any sense purposes of justice and right, and is a clearly capable of being exercised in cases of
of the word. They conferred upon the Spanish Government the right and duty to supervise, charities as in any other cases whatever.
In People vs. Cogswell (113 Cal. 129, 130), it was urged that the plaintiff was not the real party Court of the United States, in reversing this judgment and in holding the city liable for the old
in interest; that the Attorney-General had no power to institute the action; and that there must be debt, said:
an allegation and proof of a distinct right of the people as a whole, as distinguished from the
rights of individuals, before an action could be brought by the Attorney-General in the name of
The juristic identity of the corporation has been in no wise affected, and, in law, the
the people. The court, in overruling these contentions, held that it was not only the right but the
present city is, in every legal sense, the successor of the old. As such it is entitled to
duty of the Attorney-General to prosecute the action, which related to charities, and approved
the property and property rights of the predecessor corporation, and is, in law, subject
the following quotation from Attorney-General vs. Compton (1 Younge & C. C., 417):
to all of its liabilities.

Where property affected by a trust for public purposes is in the hands of those who
In support of the fifth assignment of error counsel for the defendant argue that as the Monte de
hold it devoted to that trust, it is the privilege of the public that the crown should be
Piedad declined to return the $80,000 when ordered to do so by the Department of Finance in
entitled to intervene by its officers for the purpose of asserting, on behalf on the public
June, 1893, the plaintiff's right of action had prescribed at the time this suit was instituted on May
generally, the public interest and the public right, which, probably, no individual could
3, 1912, citing and relying upon article 1961, 1964 and 1969 of the Civil Code. While on the
be found effectually to assert, even if the interest were such as to allow it. (2 Knet's
other hand, the Attorney-General contends that the right of action had not prescribed (a)
Commentaries, 10th ed., 359; Lewin on Trusts, sec. 732.)
because the defense of prescription cannot be set up against the Philippine Government, (b)
because the right of action to recover a deposit or trust funds does not prescribe, and (c) even if
It is further urged, as above indicated, that "the only persons who could claim to be damaged by the defense of prescription could be interposed against the Government and if the action had, in
this payment to the Monte, if it was unlawful, are the donors or the cestuis que trustent, and this fact, prescribed, the same was revived by Act No. 2109.
Government is neither. Consequently, the plaintiff is not the proper party to bring the action."
The earthquake fund was the result or the accumulation of a great number of small
The material facts relating to this question are these: The Monte de Piedad received the $80,000
contributions. The names of the contributors do not appear in the record. Their whereabouts are
in 1883 "to be held under the same conditions as at present in the treasury, to wit, at the
unknown. They parted with the title to their respective contributions. The beneficiaries, consisting
disposal of the relief board." In compliance with the provisions of the royal order of December 3,
of the original sufferers and their heirs, could have been ascertained. They are quite numerous
1892, the Department of Finance called upon the Monte de Piedadin June, 1893, to return the
also. And no doubt a large number of the original sufferers have died, leaving various heirs. It
$80,000. The Monte declined to comply with this order upon the ground that only the Governor-
would be impracticable for them to institute an action or actions either individually or collectively
General of the Philippine Islands and not the Department of Finance had the right to order the
to recover the $80,000. The only course that can be satisfactorily pursued is for the Government
reimbursement. The amount was carried on the books of the Monte as a returnable loan until
to again assume control of the fund and devote it to the object for which it was originally
January 1, 1899, when it was transferred to the account of the "Sagrada Mitra." On March 31,
destined.
1902, the Monte, through its legal representative, stated in writing that the amount in question
was received as a reimbursable loan, without interest. Act No. 2109 became effective January
The impracticability of pursuing a different course, however, is not the true ground upon which 30, 1912, and the action was instituted on May 3rd of that year.
the right of the Government to maintain the action rests. The true ground is that the money being
given to a charity became, in a measure, public property, only applicable, it is true, to the specific
Counsel for the defendant treat the question of prescription as if the action was one between
purposes to which it was intended to be devoted, but within those limits consecrated to the
individuals or corporations wherein the plaintiff is seeking to recover an ordinary loan. Upon this
public use, and became part of the public resources for promoting the happiness and welfare of
theory June, 1893, cannot be taken as the date when the statute of limitations began to run, for
the Philippine Government. (Mormon Church vs. U. S., supra.) To deny the Government's right
the reason that the defendant acknowledged in writing on March 31, 1902, that the $80,000
to maintain this action would be contrary to sound public policy, as tending to discourage the
were received as a loan, thereby in effect admitting that it still owed the amount. (Section 50,
prompt exercise of similar acts of humanity and Christian benevolence in like instances in the
Code of Civil Procedure.) But if counsels' theory is the correct one the action may have
future.
prescribed on May 3, 1912, because more than ten full years had elapsed after March 31, 1902.
(Sections 38 and 43, Code of Civil Procedure.)
As to the question raised in the fourth assignment of error relating to the constitutionality of Act
No. 2109, little need be said for the reason that we have just held that the present Philippine
Is the Philippine Government bound by the statute of limitations? The Supreme Court of the
Government is the proper party to the action. The Act is only a manifestation on the part of the
United States in U. S. vs. Nashville, Chattanooga & St. Louis Railway Co. (118 U. S., 120, 125),
Philippine Government to exercise the power or right which it undoubtedly had. The Act is not,
said:
as contended by counsel, in conflict with the fifth section of the Act of Congress of July 1, 1902,
because it does not take property without due process of law. In fact, the defendant is not the
owner of the $80,000, but holds it as a loan subject to the disposal of the central relief board. It is settled beyond doubt or controversy — upon the foundation of the great principle
Therefor, there can be nothing in the Act which transcends the power of the Philippine of public policy, applicable to all governments alike, which forbids that the public
Legislature. interests should be prejudiced by the negligence of the officers or agents to whose
care they are confided — that the United States, asserting rights vested in it as a
sovereign government, is not bound by any statute of limitations, unless Congress has
In Vilas vs. Manila, supra, the plaintiff was a creditor of the city of Manila as it existed before the
clearly manifested its intention that it should be so bound. (Lindsey vs. Miller, 6 Pet.
cession of the Philippine Islands to the United States by the Treaty of Paris of December 10,
666; U. S. vs. Knight, 14 Pet., 301; Gibson vs. Chouteau, 13 Wall., 92; U.
1898. The action was brought upon the theory that the city, under its present charter from the
S. vs. Thompson, 98 U. S., 486; Fink vs. O'Neil, 106 U. S., 272, 281.)
Government of the Philippine Islands, was the same juristic person, and liable upon the
obligations of the old city. This court held that the present municipality is a totally different
corporate entity and in no way liable for the debts of the Spanish municipality. The Supreme In Gibson vs. Choteau, supra, the court said:
It is a matter of common knowledge that statutes of limitation do not run against the
State. That no laches can be imputed to the King, and that no time can bar his rights,
was the maxim of the common laws, and was founded on the principle of public policy,
that as he was occupied with the cares of government he ought not to suffer from the
negligence of his officer and servants. The principle is applicable to all governments,
which must necessarily act through numerous agents, and is essential to a
preservation of the interests and property of the public. It is upon this principle that in
this country the statutes of a State prescribing periods within which rights must be
prosecuted are not held to embrace the State itself, unless it is expressly designated
or the mischiefs to be remedied are of such a nature that it must necessarily be
included. As legislation of a State can only apply to persons and thing over which the
State has jurisdiction, the United States are also necessarily excluded from the
operation of such statutes.

In 25 Cyc., 1006, the rule, supported by numerous authorities, is stated as follows:

In the absence of express statutory provision to the contrary, statute of limitations do


not as a general rule run against the sovereign or government, whether state or
federal. But the rule is otherwise where the mischiefs to be remedied are of such a
nature that the state must necessarily be included, where the state goes into business
in concert or in competition with her citizens, or where a party seeks to enforces his
private rights by suit in the name of the state or government, so that the latter is only a
nominal party.

In the instant case the Philippine Government is not a mere nominal party because it, in bringing
and prosecuting this action, is exercising its sovereign functions or powers and is seeking to
carry out a trust developed upon it when the Philippine Islands were ceded to the United States.
The United States having in 1852, purchased as trustee for the Chickasaw Indians under treaty
with that tribe, certain bonds of the State of Tennessee, the right of action of the Government on
the coupons of such bonds could not be barred by the statute of limitations of Tennessee, either
while it held them in trust for the Indians, or since it became the owner of such coupons. (U.
S. vs. Nashville, etc., R. Co., supra.) So where lands are held in trust by the state and the
beneficiaries have no right to sue, a statute does not run against the State's right of action for
trespass on the trust lands. (Greene Tp. vs. Campbell, 16 Ohio St., 11; see also Atty.-
Gen. vs. Midland R. Co., 3 Ont., 511 [following Reg. vs. Williams, 39 U. C. Q. B., 397].)

These principles being based "upon the foundation of the great principle of public policy" are, in
the very nature of things, applicable to the Philippine Government.

Counsel in their argument in support of the sixth and last assignments of error do not question
the amount of the judgment nor do they question the correctness of the judgment in so far as it
allows interest, and directs its payment in gold coin or in the equivalent in Philippine currency.

For the foregoing reasons the judgment appealed from is affirmed, with costs against the
appellant. So ordered.
G.R. No. 127325 March 19, 1997 dates to be designated for the purpose be first fixed in an order to be issued by the COMELEC;
and that to adequately inform the people of the electoral process involved, it is likewise
necessary that the said order, as well as the Petition on which the signatures shall be affixed, be
MIRIAM DEFENSOR SANTIAGO, ALEXANDER PADILLA, and MARIA ISABEL
published in newspapers of general and local circulation, under the control and supervision of
ONGPIN, petitioners,
the COMELEC.
vs.
COMMISSION ON ELECTIONS, JESUS DELFIN, ALBERTO PEDROSA & CARMEN
PEDROSA, in their capacities as founding members of the People's Initiative for Reforms, The Delfin Petition further alleged that the provisions sought to be amended are Sections 4 and
Modernization and Action (PIRMA), respondents. 7 of Article VI,7Section 4 of Article VII,8 and Section 8 of Article X9 of the Constitution. Attached
to the petition is a copy of a "Petition for Initiative on the 1987 Constitution" 10 embodying the
proposed amendments which consist in the deletion from the aforecited sections of the
SENATOR RAUL S. ROCO, DEMOKRASYA-IPAGTANGGOL ANG KONSTITUSYON (DIK),
provisions concerning term limits, and with the following proposition:
MOVEMENT OF ATTORNEYS FOR BROTHERHOOD INTEGRITY AND NATIONALISM, INC.
(MABINI), INTEGRATED BAR OF THE PHILIPPINES (IBP), and LABAN NG
DEMOKRATIKONG PILIPINO (LABAN), petitioners-intervenors. DO YOU APPROVE OF LIFTING THE TERM LIMITS OF ALL ELECTIVE
GOVERNMENT OFFICIALS, AMENDING FOR THE PURPOSE
SECTIONS 4 AND 7 OF ARTICLE VI, SECTION 4 OF ARTICLE VII, AND
DAVIDE, JR., J.:
SECTION 8 OF ARTICLE X OF THE 1987 PHILIPPINE CONSTITUTION?

The heart of this controversy brought to us by way of a petition for prohibition under Rule 65 of
According to Delfin, the said Petition for Initiative will first be submitted to the people, and after it
the Rules of Court is the right of the people to directly propose amendments to the Constitution
is signed by at least twelve per cent of the total number of registered voters in the country it will
through the system of initiative under Section 2 of Article XVII of the 1987 Constitution.
be formally filed with the COMELEC.
Undoubtedly, this demands special attention, as this system of initiative was unknown to the
people of this country, except perhaps to a few scholars, before the drafting of the 1987
Constitution. The 1986 Constitutional Commission itself, through the original proponent 1 and the Upon the filing of the Delfin Petition, which was forthwith given the number UND 96-037
main sponsor2 of the proposed Article on Amendments or Revision of the Constitution, (INITIATIVE), the COMELEC, through its Chairman, issued an Order 11 (a) directing Delfin "to
characterized this system as "innovative".3 Indeed it is, for both under the 1935 and 1973 cause the publication of the petition, together with the attached Petition for Initiative on the 1987
Constitutions, only two methods of proposing amendments to, or revision of, the Constitution Constitution (including the proposal, proposed constitutional amendment, and the signature
were recognized, viz., (1) by Congress upon a vote of three-fourths of all its members and (2) by form), and the notice of hearing in three (3) daily newspapers of general circulation at his own
a constitutional convention.4 For this and the other reasons hereafter discussed, we resolved to expense" not later than 9 December 1996; and (b) setting the case for hearing on 12 December
give due course to this petition. 1996 at 10:00 a.m.

On 6 December 1996, private respondent Atty. Jesus S. Delfin filed with public respondent At the hearing of the Delfin Petition on 12 December 1996, the following appeared: Delfin and
Commission on Elections (hereafter, COMELEC) a "Petition to Amend the Constitution, to Lift Atty. Pete Q. Quadra; representatives of the People's Initiative for Reforms, Modernization and
Term Limits of Elective Officials, by People's Initiative" (hereafter, Delfin Petition)5 wherein Delfin Action (PIRMA); intervenor-oppositor Senator Raul S. Roco, together with his two other lawyers,
asked the COMELEC for an order and representatives of, or counsel for, the Integrated Bar of the Philippines (IBP), Demokrasya-
Ipagtanggol ang Konstitusyon (DIK), Public Interest Law Center, and Laban ng Demokratikong
Pilipino (LABAN). 12 Senator Roco, on that same day, filed a Motion to Dismiss the Delfin
1. Fixing the time and dates for signature gathering all over the country;
Petition on the ground that it is not the initiatory petition properly cognizable by the COMELEC.

2. Causing the necessary publications of said Order and the attached


After hearing their arguments, the COMELEC directed Delfin and the oppositors to file their
"Petition for Initiative on the 1987 Constitution, in newspapers of general
"memoranda and/or oppositions/memoranda" within five days. 13
and local circulation;

On 18 December 1996, the petitioners herein — Senator Miriam Defensor Santiago, Alexander
3. Instructing Municipal Election Registrars in all Regions of the Philippines,
Padilla, and Maria Isabel Ongpin — filed this special civil action for prohibition raising the
to assist Petitioners and volunteers, in establishing signing stations at the
following arguments:
time and on the dates designated for the purpose.

(1) The constitutional provision on people's initiative to amend the


Delfin alleged in his petition that he is a founding member of the Movement for People's
Constitution can only be implemented by law to be passed by Congress. No
Initiative,6 a group of citizens desirous to avail of the system intended to institutionalize people
such law has been passed; in fact, Senate Bill No. 1290 entitled An Act
power; that he and the members of the Movement and other volunteers intend to exercise the
Prescribing and Regulating Constitution Amendments by People's Initiative,
power to directly propose amendments to the Constitution granted under Section 2, Article XVII
which petitioner Senator Santiago filed on 24 November 1995, is still
of the Constitution; that the exercise of that power shall be conducted in proceedings under the
pending before the Senate Committee on Constitutional Amendments.
control and supervision of the COMELEC; that, as required in COMELEC Resolution No. 2300,
signature stations shall be established all over the country, with the assistance of municipal
election registrars, who shall verify the signatures affixed by individual signatories; that before (2) It is true that R.A. No. 6735 provides for three systems of initiative,
the Movement and other volunteers can gather signatures, it is necessary that the time and namely, initiative on the Constitution, on statutes, and on local legislation.
However, it failed to provide any subtitle on initiative on the Constitution, (P180,000,000.00)" IF THE "COMELEC GRANTS THE PETITION FILED
unlike in the other modes of initiative, which are specifically provided for in BY RESPONDENT DELFIN BEFORE THE COMELEC.
Subtitle II and Subtitle III. This deliberate omission indicates that the matter
of people's initiative to amend the Constitution was left to some future law.
2. NOT A SINGLE CENTAVO WOULD BE SPENT BY THE NATIONAL
Former Senator Arturo Tolentino stressed this deficiency in the law in his
GOVERNMENT IF THE COMELEC GRANTS THE PETITION OF
privilege speech delivered before the Senate in 1994: "There is not a single
RESPONDENT DELFIN. ALL EXPENSES IN THE SIGNATURE
word in that law which can be considered as implementing [the provision on
GATHERING ARE ALL FOR THE ACCOUNT OF RESPONDENT DELFIN
constitutional initiative]. Such implementing provisions have been obviously
AND HIS VOLUNTEERS PER THEIR PROGRAM OF ACTIVITIES AND
left to a separate law.
EXPENDITURES SUBMITTED TO THE COMELEC. THE ESTIMATED
COST OF THE DAILY PER DIEM OF THE SUPERVISING SCHOOL
(3) Republic Act No. 6735 provides for the effectivity of the law after TEACHERS IN THE SIGNATURE GATHERING TO BE DEPOSITED and
publication in print media. This indicates that the Act covers only laws and TO BE PAID BY DELFIN AND HIS VOLUNTEERS IS P2,571,200.00;
not constitutional amendments because the latter take effect only upon
ratification and not after publication.
3. THE PENDING PETITION BEFORE THE COMELEC IS ONLY ON THE
SIGNATURE GATHERING WHICH BY LAW COMELEC IS DUTY BOUND
(4) COMELEC Resolution No. 2300, adopted on 16 January 1991 to govern "TO SUPERVISE CLOSELY" PURSUANT TO ITS "INITIATORY
"the conduct of initiative on the Constitution and initiative and referendum on JURISDICTION" UPHELD BY THE HONORABLE COURT IN ITS RECENT
national and local laws, is ultra vires insofar as initiative on amendments to SEPTEMBER 26, 1996 DECISION IN THE CASE OF SUBIC BAY
the Constitution is concerned, since the COMELEC has no power to provide METROPOLITAN AUTHORITY VS. COMELEC, ET AL. G.R. NO. 125416;
rules and regulations for the exercise of the right of initiative to amend the
Constitution. Only Congress is authorized by the Constitution to pass the
4. REP. ACT NO. 6735 APPROVED ON AUGUST 4, 1989 IS THE
implementing law.
ENABLING LAW IMPLEMENTING THE POWER OF PEOPLE INITIATIVE
TO PROPOSE AMENDMENTS TO THE CONSTITUTION. SENATOR
(5) The people's initiative is limited to amendments to the Constitution, not DEFENSOR-SANTIAGO'S SENATE BILL NO. 1290 IS A DUPLICATION
to revision thereof. Extending or lifting of term limits constitutes OF WHAT ARE ALREADY PROVIDED FOR IN REP. ACT NO. 6735;
a revision and is, therefore, outside the power of the people's initiative.
5. COMELEC RESOLUTION NO. 2300 PROMULGATED ON JANUARY
(6) Finally, Congress has not yet appropriated funds for people's initiative; 16, 1991 PURSUANT TO REP. ACT 6735 WAS UPHELD BY THE
neither the COMELEC nor any other government department, agency, or HONORABLE COURT IN THE RECENT SEPTEMBER 26, 1996
office has realigned funds for the purpose. DECISION IN THE CASE OF SUBIC BAY METROPOLITAN AUTHORITY
VS. COMELEC, ET AL. G.R. NO. 125416 WHERE THE HONORABLE
COURT SAID: "THE COMMISSION ON ELECTIONS CAN DO NO LESS
To justify their recourse to us via the special civil action for prohibition, the petitioners allege that
BY SEASONABLY AND JUDICIOUSLY PROMULGATING GUIDELINES
in the event the COMELEC grants the Delfin Petition, the people's initiative spearheaded by
AND RULES FOR BOTH NATIONAL AND LOCAL USE, IN
PIRMA would entail expenses to the national treasury for general re-registration of voters
IMPLEMENTING OF THESE LAWS."
amounting to at least P180 million, not to mention the millions of additional pesos in expenses
which would be incurred in the conduct of the initiative itself. Hence, the transcendental
importance to the public and the nation of the issues raised demands that this petition for 6. EVEN SENATOR DEFENSOR-SANTIAGO'S SENATE BILL NO. 1290
prohibition be settled promptly and definitely, brushing aside technicalities of procedure and CONTAINS A PROVISION DELEGATING TO THE COMELEC THE
calling for the admission of a taxpayer's and legislator's suit. 14 Besides, there is no other plain, POWER TO "PROMULGATE SUCH RULES AND REGULATIONS AS MAY
speedy, and adequate remedy in the ordinary course of law. BE NECESSARY TO CARRY OUT THE PURPOSES OF THIS ACT."
(SEC. 12, S.B. NO. 1290, ENCLOSED AS ANNEX E, PETITION);
On 19 December 1996, this Court (a) required the respondents to comment on the petition within
a non-extendible period of ten days from notice; and (b) issued a temporary restraining order, 7. THE LIFTING OF THE LIMITATION ON THE TERM OF OFFICE OF
effective immediately and continuing until further orders, enjoining public respondent COMELEC ELECTIVE OFFICIALS PROVIDED UNDER THE 1987 CONSTITUTION IS
from proceeding with the Delfin Petition, and private respondents Alberto and Carmen Pedrosa NOT A "REVISION" OF THE CONSTITUTION. IT IS ONLY AN
from conducting a signature drive for people's initiative to amend the Constitution. AMENDMENT. "AMENDMENT ENVISAGES AN ALTERATION OF ONE
OR A FEW SPECIFIC PROVISIONS OF THE CONSTITUTION. REVISION
15 CONTEMPLATES A RE-EXAMINATION OF THE ENTIRE DOCUMENT TO
On 2 January 1997, private respondents, through Atty Quadra, filed their Comment on the
DETERMINE HOW AND TO WHAT EXTENT IT SHOULD BE ALTERED."
petition. They argue therein that:
(PP. 412-413, 2ND. ED. 1992, 1097 PHIL. CONSTITUTION, BY JOAQUIN
G. BERNAS, S.J.).
1. IT IS NOT TRUE THAT "IT WOULD ENTAIL EXPENSES TO THE
NATIONAL TREASURY FOR GENERAL REGISTRATION OF VOTERS
Also on 2 January 1997, private respondent Delfin filed in his own behalf a Comment 16 which
AMOUNTING TO AT LEAST PESOS: ONE HUNDRED EIGHTY MILLION
starts off with an assertion that the instant petition is a "knee-jerk reaction to a draft 'Petition for
Initiative on the 1987 Constitution'. . . which is not formally filed yet." What he filed on 6 (3) Senate Bill No. 1290 is neither a competent nor a material proof that
December 1996 was an "Initiatory Pleading" or "Initiatory Petition," which was legally necessary R.A. No. 6735 does not deal with initiative on the Constitution.
to start the signature campaign to amend the Constitution or to put the movement to gather
signatures under COMELEC power and function. On the substantive allegations of the
(4) Extension of term limits of elected officials constitutes a mere
petitioners, Delfin maintains as follows:
amendment to the Constitution, not a revision thereof.

(1) Contrary to the claim of the petitioners, there is a law, R.A. No. 6735,
(5) COMELEC Resolution No. 2300 was validly issued under Section 20 of
which governs the conduct of initiative to amend the Constitution. The
R.A. No. 6735 and under the Omnibus Election Code. The rule-making
absence therein of a subtitle for such initiative is not fatal, since subtitles are
power of the COMELEC to implement the provisions of R.A. No. 6735 was
not requirements for the validity or sufficiency of laws.
in fact upheld by this Court in Subic Bay Metropolitan Authority
vs. COMELEC.
(2) Section 9(b) of R.A. No. 6735 specifically provides that the proposition in
an initiative to amend the Constitution approved by the majority of the votes
On 14 January 1997, this Court (a) confirmed nunc pro tunc the temporary restraining order; (b)
cast in the plebiscite shall become effective as of the day of the plebiscite.
noted the aforementioned Comments and the Motion to Lift Temporary Restraining Order filed
by private respondents through Atty. Quadra, as well as the latter's Manifestation stating that he
(3) The claim that COMELEC Resolution No. 2300 is ultra vires is is the counsel for private respondents Alberto and Carmen Pedrosa only and the Comment he
contradicted by (a) Section 2, Article IX-C of the Constitution, which grants filed was for the Pedrosas; and (c) granted the Motion for Intervention filed on 6 January 1997 by
the COMELEC the power to enforce and administer all laws and regulations Senator Raul Roco and allowed him to file his Petition in Intervention not later than 20 January
relative to the conduct of an election, plebiscite, initiative, referendum, and 1997; and (d) set the case for hearing on 23 January 1997 at 9:30 a.m.
recall; and (b) Section 20 of R.A. 6735, which empowers the COMELEC to
promulgate such rules and regulations as may be necessary to carry out the
On 17 January 1997, the Demokrasya-Ipagtanggol ang Konstitusyon (DIK) and the Movement of
purposes of the Act.
Attorneys for Brotherhood Integrity and Nationalism, Inc. (MABINI), filed a Motion for
Intervention. Attached to the motion was their Petition in Intervention, which was later replaced
(4) The proposed initiative does not involve a revision of, but by an Amended Petition in Intervention wherein they contend that:
mere amendment to, the Constitution because it seeks to alter only a few
specific provisions of the Constitution, or more specifically, only those which
(1) The Delfin proposal does not involve a mere amendment to, but
lay term limits. It does not seek to reexamine or overhaul the entire
a revision of, the Constitution because, in the words of Fr. Joaquin Bernas,
document.
S.J., 18 it would involve a change from a political philosophy that rejects
unlimited tenure to one that accepts unlimited tenure; and although the
As to the public expenditures for registration of voters, Delfin considers petitioners' estimate of change might appear to be an isolated one, it can affect other provisions,
P180 million as unreliable, for only the COMELEC can give the exact figure. Besides, if there will such as, on synchronization of elections and on the State policy of
be a plebiscite it will be simultaneous with the 1997 Barangay Elections. In any event, fund guaranteeing equal access to opportunities for public service and prohibiting
requirements for initiative will be a priority government expense because it will be for the political dynasties. 19 A revision cannot be done by initiative which, by
exercise of the sovereign power of the people. express provision of Section 2 of Article XVII of the Constitution, is limited
to amendments.
In the Comment 17 for the public respondent COMELEC, filed also on 2 January 1997, the Office
of the Solicitor General contends that: (2) The prohibition against reelection of the President and the limits
provided for all other national and local elective officials are based on the
philosophy of governance, "to open up the political arena to as many as
(1) R.A. No. 6735 deals with, inter alia, people's initiative to amend the
there are Filipinos qualified to handle the demands of leadership, to break
Constitution. Its Section 2 on Statement of Policy explicitly affirms,
the concentration of political and economic powers in the hands of a few,
recognizes, and guarantees that power; and its Section 3, which
and to promote effective proper empowerment for participation in policy and
enumerates the three systems of initiative, includes initiative on the
decision-making for the common good"; hence, to remove the term limits is
Constitution and defines the same as the power to propose amendments to
to negate and nullify the noble vision of the 1987 Constitution.
the Constitution. Likewise, its Section 5 repeatedly mentions initiative on the
Constitution.
(3) The Delfin proposal runs counter to the purpose of initiative, particularly
in a conflict-of-interest situation. Initiative is intended as a fallback position
(2) A separate subtitle on initiative on the Constitution is not necessary in
that may be availed of by the people only if they are dissatisfied with the
R.A. No. 6735 because, being national in scope, that system of initiative is
performance of their elective officials, but not as a premium for good
deemed included in the subtitle on National Initiative and Referendum; and
performance. 20
Senator Tolentino simply overlooked pertinent provisions of the law when he
claimed that nothing therein was provided for initiative on the Constitution.
(4) R.A. No. 6735 is deficient and inadequate in itself to be called the
enabling law that implements the people's initiative on amendments to the
Constitution. It fails to state (a) the proper parties who may file the petition, On 21 January 1997, we promulgated a Resolution (a) granting the Motions for Intervention filed
(b) the appropriate agency before whom the petition is to be filed, (c) the by the DIK and MABINI and by the IBP, as well as the Motion for Leave to Intervene filed by
contents of the petition, (d) the publication of the same, (e) the ways and LABAN; (b) admitting the Amended Petition in Intervention of DIK and MABINI, and the Petitions
means of gathering the signatures of the voters nationwide and 3% per in Intervention of Senator Roco and of the IBP; (c) requiring the respondents to file within a
legislative district, (f) the proper parties who may oppose or question the nonextendible period of five days their Consolidated Comments on the aforesaid Petitions in
veracity of the signatures, (g) the role of the COMELEC in the verification of Intervention; and (d) requiring LABAN to file its Petition in Intervention within a nonextendible
the signatures and the sufficiency of the petition, (h) the appeal from any period of three days from notice, and the respondents to comment thereon within a
decision of the COMELEC, (I) the holding of a plebiscite, and (g) the nonextendible period of five days from receipt of the said Petition in Intervention.
appropriation of funds for such people's initiative. Accordingly, there being
no enabling law, the COMELEC has no jurisdiction to hear Delfin's petition.
At the hearing of the case on 23 January 1997, the parties argued on the following pivotal
issues, which the Court formulated in light of the allegations and arguments raised in the
(5) The deficiency of R.A. No. 6735 cannot be rectified or remedied by pleadings so far filed:
COMELEC Resolution No. 2300, since the COMELEC is without authority to
legislate the procedure for a people's initiative under Section 2 of Article
1. Whether R.A. No. 6735, entitled An Act Providing for a System of
XVII of the Constitution. That function exclusively pertains to Congress.
Initiative and Referendum and Appropriating Funds Therefor, was intended
Section 20 of R.A. No. 6735 does not constitute a legal basis for the
to include or cover initiative on amendments to the Constitution; and if so,
Resolution, as the former does not set a sufficient standard for a valid
whether the Act, as worded, adequately covers such initiative.
delegation of power.

2. Whether that portion of COMELEC Resolution No. 2300 (In re: Rules and
On 20 January 1997, Senator Raul Roco filed his Petition in
Regulations Governing the Conduct of Initiative on the Constitution, and
Intervention. 21 He avers that R.A. No. 6735 is the enabling law that implements the people's
Initiative and Referendum on National and Local Laws) regarding the
right to initiate constitutional amendments. This law is a consolidation of Senate Bill No. 17 and
conduct of initiative on amendments to the Constitution is valid, considering
House Bill No. 21505; he co-authored the House Bill and even delivered a sponsorship speech
the absence in the law of specific provisions on the conduct of such
thereon. He likewise submits that the COMELEC was empowered under Section 20 of that law
initiative.
to promulgate COMELEC Resolution No. 2300. Nevertheless, he contends that the respondent
Commission is without jurisdiction to take cognizance of the Delfin Petition and to order its
publication because the said petition is not the initiatory pleading contemplated under the 3. Whether the lifting of term limits of elective national and local officials, as
Constitution, Republic Act No. 6735, and COMELEC Resolution No. 2300. What vests proposed in the draft "Petition for Initiative on the 1987 Constitution," would
jurisdiction upon the COMELEC in an initiative on the Constitution is the filing of a petition for constitute a revision of, or an amendment to, the Constitution.
initiative which is signed by the required number of registered voters. He also submits that the
proponents of a constitutional amendment cannot avail of the authority and resources of the
COMELEC to assist them is securing the required number of signatures, as the COMELEC's 4. Whether the COMELEC can take cognizance of, or has jurisdiction over,
role in an initiative on the Constitution is limited to the determination of the sufficiency of the a petition solely intended to obtain an order (a) fixing the time and dates for
signature gathering; (b) instructing municipal election officers to assist
initiative petition and the call and supervision of a plebiscite, if warranted.
Delfin's movement and volunteers in establishing signature stations; and (c)
directing or causing the publication of, inter alia, the unsigned proposed
On 20 January 1997, LABAN filed a Motion for Leave to Intervene. Petition for Initiative on the 1987 Constitution.

The following day, the IBP filed a Motion for Intervention to which it attached a Petition in 5. Whether it is proper for the Supreme Court to take cognizance of the
Intervention raising the following arguments: petition when there is a pending case before the COMELEC.

(1) Congress has failed to enact an enabling law mandated under Section 2, After hearing them on the issues, we required the parties to submit simultaneously their
Article XVII of the 1987 Constitution. respective memoranda within twenty days and requested intervenor Senator Roco to submit
copies of the deliberations on House Bill No. 21505.
(2) COMELEC Resolution No. 2300 cannot substitute for the required
implementing law on the initiative to amend the Constitution. On 27 January 1997, LABAN filed its Petition in Intervention wherein it adopts the allegations
and arguments in the main Petition. It further submits that the COMELEC should have dismissed
the Delfin Petition for failure to state a sufficient cause of action and that the Commission's
(3) The Petition for Initiative suffers from a fatal defect in that it does not
failure or refusal to do so constituted grave abuse of discretion amounting to lack of jurisdiction.
have the required number of signatures.

On 28 January 1997, Senator Roco submitted copies of portions of both the Journal and the
(4) The petition seeks, in effect a revision of the Constitution, which can be
Record of the House of Representatives relating to the deliberations of House Bill No. 21505, as
proposed only by Congress or a constitutional convention. 22
well as the transcripts of stenographic notes on the proceedings of the Bicameral Conference
Committee, Committee on Suffrage and Electoral Reforms, of 6 June 1989 on House Bill No.
21505 and Senate Bill No. 17.
Private respondents Alberto and Carmen Pedrosa filed their Consolidated Comments on the ministerial, are without or in excess of its or his jurisdiction, or with grave
Petitions in Intervention of Senator Roco, DIK and MABINI, and IBP. 23 The parties thereafter abuse of discretion, and there is no appeal or any other plain, speedy and
filed, in due time, their separate memoranda. 24 adequate remedy in the ordinary course of law, a person aggrieved thereby
may file a verified petition in the proper court alleging the facts with certainty
and praying that judgment be rendered commanding the defendant to desist
As we stated in the beginning, we resolved to give due course to this special civil action.
from further proceedings in the action or matter specified therein.

For a more logical discussion of the formulated issues, we shall first take up the fifth issue which
It must also be noted that intervenor Roco claims that the COMELEC has no jurisdiction over the
appears to pose a prejudicial procedural question.
Delfin Petition because the said petition is not supported by the required minimum number of
signatures of registered voters. LABAN also asserts that the COMELEC gravely abused its
I discretion in refusing to dismiss the Delfin Petition, which does not contain the required number
of signatures. In light of these claims, the instant case may likewise be treated as a special civil
action for certiorari under Section I of Rule 65 of the Rules of Court.
THE INSTANT PETITION IS VIABLE DESPITE THE PENDENCY IN THE COMELEC
OF THE DELFIN PETITION.
In any event, as correctly pointed out by intervenor Roco in his Memorandum, this Court may
brush aside technicalities of procedure in
Except for the petitioners and intervenor Roco, the parties paid no serious attention to the fifth
cases of transcendental importance. As we stated in Kilosbayan, Inc. v. Guingona, Jr. 28
issue, i.e., whether it is proper for this Court to take cognizance of this special civil action when
there is a pending case before the COMELEC. The petitioners provide an affirmative answer.
Thus: A party's standing before this Court is a procedural technicality which it may,
in the exercise of its discretion, set aside in view of the importance of issues
raised. In the landmark Emergency Powers Cases, this Court brushed aside
28. The Comelec has no jurisdiction to take cognizance of the petition filed this technicality because the transcendental importance to the public of
by private respondent Delfin. This being so, it becomes imperative to stop
these cases demands that they be settled promptly and definitely, brushing
the Comelec from proceeding any further, and under the Rules of Court, aside, if we must, technicalities of procedure.
Rule 65, Section 2, a petition for prohibition is the proper remedy.

II
29. The writ of prohibition is an extraordinary judicial writ issuing out of a
court of superior jurisdiction and directed to an inferior court, for the purpose
of preventing the inferior tribunal from usurping a jurisdiction with which it is R.A. NO. 6735 INTENDED TO INCLUDE THE SYSTEM OF INITIATIVE ON
not legally vested. (People v. Vera, supra., p. 84). In this case the writ is an AMENDMENTS TO THE CONSTITUTION, BUT IS, UNFORTUNATELY,
urgent necessity, in view of the highly divisive and adverse environmental INADEQUATE TO COVER THAT SYSTEM.
consequences on the body politic of the questioned Comelec order. The
consequent climate of legal confusion and political instability begs for
Section 2 of Article XVII of the Constitution provides:
judicial statesmanship.

Sec. 2. Amendments to this Constitution may likewise be directly proposed


30. In the final analysis, when the system of constitutional law is threatened
by the people through initiative upon a petition of at least twelve per
by the political ambitions of man, only the Supreme Court
centum of the total number of registered voters, of which every legislative
can save a nation in peril and uphold the paramount majesty of the
district must be represented by at least three per centum of the registered
Constitution. 25
voters therein. No amendment under this section shall be authorized within
five years following the ratification of this Constitution nor oftener than once
It must be recalled that intervenor Roco filed with the COMELEC a motion to dismiss the Delfin every five years thereafter.
Petition on the ground that the COMELEC has no jurisdiction or authority to entertain the
petition. 26 The COMELEC made no ruling thereon evidently because after having heard the
The Congress shall provide for the implementation of the exercise of this right.
arguments of Delfin and the oppositors at the hearing on 12 December 1996, it required them to
submit within five days their memoranda or oppositions/memoranda. 27 Earlier, or specifically on
29
6 December 1996, it practically gave due course to the Delfin Petition by ordering Delfin to This provision is not self-executory. In his book, Joaquin Bernas, a member of the 1986
cause the publication of the petition, together with the attached Petition for Initiative, the Constitutional Commission, stated:
signature form, and the notice of hearing; and by setting the case for hearing. The COMELEC's
failure to act on Roco's motion to dismiss and its insistence to hold on to the petition rendered
Without implementing legislation Section 2 cannot operate. Thus, although
ripe and viable the instant petition under Section 2 of Rule 65 of the Rules of Court, which
this mode of amending the Constitution is a mode of amendment which
provides:
bypasses congressional action, in the last analysis it still is dependent on
congressional action.
Sec. 2. Petition for prohibition. — Where the proceedings of any tribunal,
corporation, board, or person, whether exercising functions judicial or
Bluntly stated, the right of the people to directly propose amendments to the therefore, that we are leaving this matter to the
Constitution through the system of initiative would remain entombed in the cold niche legislature?
of the Constitution until Congress provides for its implementation. Stated otherwise,
while the Constitution has recognized or granted that right, the people cannot exercise
MR. SUAREZ. That is right, Madam President.
it if Congress, for whatever reason, does not provide for its implementation.

FR. BERNAS. And do we also understand, therefore,


This system of initiative was originally included in Section 1 of the draft Article on Amendment or
that for as long as the legislature does not pass the
Revision proposed by the Committee on Amendments and Transitory Provisions of the 1986
necessary implementing law on this, this will not
Constitutional Commission in its Committee Report No. 7 (Proposed Resolution No.
operate?
332). 30 That section reads as follows:

MR. SUAREZ. That matter was also taken up during


Sec. 1. Any amendment to, or revision of, this Constitution may be proposed:
the committee hearing, especially with respect to the
budget appropriations which would have to be
(a) by the National Assembly upon a vote of three-fourths of all its members; legislated so that the plebiscite could be called. We
or deemed it best that this matter be left to the legislature.
The Gentleman is right. In any event, as envisioned, no
amendment through the power of initiative can be
(b) by a constitutional convention; or
called until after five years from the date of the
ratification of this Constitution. Therefore, the first
(c) directly by the people themselves thru initiative as provided for in amendment that could be proposed through the
Article___ Section ___of the Constitution. 31 exercise of this initiative power would be after five
years. It is reasonably expected that within that five-
year period, the National Assembly can come up with
After several interpellations, but before the period of amendments, the Committee the appropriate rules governing the exercise of this
submitted a new formulation of the concept of initiative which it denominated as power.
Section 2; thus:

FR. BERNAS. Since the matter is left to the legislature


MR. SUAREZ. Thank you, Madam President. May we — the details on how this is to be carried out — is it
respectfully call attention of the Members of the
possible that, in effect, what will be presented to the
Commission that pursuant to the mandate given to us people for ratification is the work of the legislature
last night, we submitted this afternoon a complete rather than of the people? Does this provision exclude
Committee Report No. 7 which embodies the proposed that possibility?
provision governing the matter of initiative. This is now
covered by Section 2 of the complete committee report.
With the permission of the Members, may I quote MR. SUAREZ. No, it does not exclude that possibility
Section 2: because even the legislature itself as a body could
propose that amendment, maybe individually or
collectively, if it fails to muster the three-fourths vote in
The people may, after five years from the date of the last plebiscite held, order to constitute itself as a constituent assembly and
directly propose amendments to this Constitution thru initiative upon petition submit that proposal to the people for ratification
of at least ten percent of the registered voters.
through the process of an initiative.

This completes the blanks appearing in the original Committee Report No. xxx xxx xxx
7. 32

MS. AQUINO. Do I understand from the sponsor that


The interpellations on Section 2 showed that the details for carrying out Section 2 are left to the the intention in the proposal is to vest constituent power
legislature. Thus:
in the people to amend the Constitution?

FR. BERNAS. Madam President, just two simple, MR. SUAREZ. That is absolutely correct, Madam
clarificatory questions.
President.

First, on Section 1 on the matter of initiative upon MS. AQUINO. I fully concur with the underlying precept
petition of at least 10 percent, there are no details in the
of the proposal in terms of institutionalizing popular
provision on how to carry this out. Do we understand, participation in the drafting of the Constitution or in the
amendment thereof, but I would have a lot of difficulties MS. AQUINO. In which case, I am seriously bothered
in terms of accepting the draft of Section 2, as written. by providing this process of initiative as a separate
Would the sponsor agree with me that in the hierarchy section in the Article on Amendment. Would the
of legal mandate, constituent power has primacy over sponsor be amenable to accepting an amendment in
all other legal mandates? terms of realigning Section 2 as another subparagraph
(c) of Section 1, instead of setting it up as another
separate section as if it were a self-executing
MR. SUAREZ. The Commissioner is right, Madam
provision?
President.

MR. SUAREZ. We would be amenable except that, as


MS. AQUINO. And would the sponsor agree with me
we clarified a while ago, this process of initiative is
that in the hierarchy of legal values, the Constitution is
limited to the matter of amendment and should not
source of all legal mandates and that therefore we
expand into a revision which contemplates a total
require a great deal of circumspection in the drafting
overhaul of the Constitution. That was the sense that
and in the amendments of the Constitution?
was conveyed by the Committee.

MR. SUAREZ. That proposition is nondebatable.


MS. AQUINO. In other words, the Committee was
attempting to distinguish the coverage of modes (a) and
MS. AQUINO. Such that in order to underscore the (b) in Section 1 to include the process of revision;
primacy of constituent power we have a separate article whereas the process of initiation to amend, which is
in the constitution that would specifically cover the given to the public, would only apply to amendments?
process and the modes of amending the Constitution?
MR. SUAREZ. That is right. Those were the terms
MR. SUAREZ. That is right, Madam President. envisioned in the Committee. 35

MS. AQUINO. Therefore, is the sponsor inclined, as the Amendments to the proposed Section 2 were thereafter introduced by then Commissioner
provisions are drafted now, to again concede to the Hilario G. Davide, Jr., which the Committee accepted. Thus:
legislature the process or the requirement of
determining the mechanics of amending the
MR. DAVIDE. Thank you Madam President. I propose
Constitution by people's initiative?
to substitute the entire Section 2 with the following:

MR. SUAREZ. The matter of implementing this could


MR. DAVIDE. Madam President, I have modified the
very well be placed in the hands of the National
proposed amendment after taking into account the
Assembly, not unless we can incorporate into this
modifications submitted by the sponsor himself and the
provision the mechanics that would adequately cover all
honorable Commissioners Guingona, Monsod, Rama,
the conceivable situations. 33
Ople, de los Reyes and Romulo. The modified
amendment in substitution of the proposed Section 2
It was made clear during the interpellations that the aforementioned Section 2 is limited to will now read as follows: "SECTION 2. —
proposals to AMEND — not to REVISE — the Constitution; thus: AMENDMENTS TO THIS CONSTITUTION MAY
LIKEWISE BE DIRECTLY PROPOSED BY THE
PEOPLE THROUGH INITIATIVE UPON A PETITION
MR. SUAREZ. . . . This proposal was suggested on the OF AT LEAST TWELVE PERCENT OF THE TOTAL
theory that this matter of initiative, which came about
NUMBER Of REGISTERED VOTERS, OF WHICH
because of the extraordinary developments this year, EVERY LEGISLATIVE DISTRICT MUST BE
has to be separated from the traditional modes of REPRESENTED BY AT LEAST THREE PERCENT OF
amending the Constitution as embodied in Section 1.
THE REGISTERED VOTERS THEREOF. NO
The committee members felt that this system of AMENDMENT UNDER THIS SECTION SHALL BE
initiative should not extend to the revision of the entire AUTHORIZED WITHIN FIVE YEARS FOLLOWING
Constitution, so we removed it from the operation of
THE RATIFICATION OF THIS CONSTITUTION NOR
Section 1 of the proposed Article on Amendment or OFTENER THAN ONCE EVERY FIVE YEARS
Revision. 34 THEREAFTER.

xxx xxx xxx


THE NATIONAL ASSEMBLY SHALL BY LAW "amendment." Does it not cover the word "revision" as
PROVIDE FOR THE IMPLEMENTATION OF THE defined by Commissioner Padilla when he made the
EXERCISE OF THIS RIGHT. distinction between the words "amendments" and
"revision"?
MR. SUAREZ. Madam President, considering that the
proposed amendment is reflective of the sense MR. DAVIDE. No, it does not, because "amendments"
contained in Section 2 of our completed Committee and "revision" should be covered by Section 1. So
Report No. 7, we accept the proposed amendment. 36 insofar as initiative is concerned, it can only relate to
"amendments" not "revision." 38
The interpellations which ensued on the proposed modified amendment to Section 2 clearly
showed that it was a legislative act which must implement the exercise of the right. Thus: Commissioner Davide further emphasized that the process of proposing amendments
through initiative must be more rigorous and difficult than the initiative on legislation. Thus:
MR. ROMULO. Under Commissioner Davide's
amendment, is it possible for the legislature to set forth MR. DAVIDE. A distinction has to be made that under
certain procedures to carry out the initiative. . .? this proposal, what is involved is an amendment to the
Constitution. To amend a Constitution would ordinarily
require a proposal by the National Assembly by a vote
MR. DAVIDE. It can.
of three-fourths; and to call a constitutional convention
would require a higher number. Moreover, just to
xxx xxx xxx submit the issue of calling a constitutional convention, a
majority of the National Assembly is required, the
import being that the process of amendment must be
MR. ROMULO. But the Commissioner's amendment
made more rigorous and difficult than probably initiating
does not prevent the legislature from asking another an ordinary legislation or putting an end to a law
body to set the proposition in proper form. proposed by the National Assembly by way of a
referendum. I cannot agree to reducing the requirement
MR. DAVIDE. The Commissioner is correct. In other approved by the Committee on the Legislative because
words, the implementation of this particular right would it would require another voting by the Committee, and
be subject to legislation, provided the legislature cannot the voting as precisely based on a requirement of 10
determine anymore the percentage of the requirement. percent. Perhaps, I might present such a proposal, by
way of an amendment, when the Commission shall take
up the Article on the Legislative or on the National
MR. ROMULO. But the procedures, including the Assembly on plenary sessions. 39
determination of the proper form for submission to the
people, may be subject to legislation.
The Davide modified amendments to Section 2 were subjected to amendments, and the final
version, which the Commission approved by a vote of 31 in favor and 3 against, reads as
MR. DAVIDE. As long as it will not destroy the follows:
substantive right to initiate. In other words, none of the
procedures to be proposed by the legislative body must
diminish or impair the right conceded here. MR. DAVIDE. Thank you Madam President. Section 2,
as amended, reads as follows: "AMENDMENT TO
THIS CONSTITUTION MAY LIKEWISE BE DIRECTLY
MR. ROMULO. In that provision of the Constitution can PROPOSED BY THE PEOPLE THROUGH INITIATIVE
the procedures which I have discussed be legislated? UPON A PETITION OF AT LEAST TWELVE
PERCENT OF THE TOTAL NUMBER OF
MR. DAVIDE. Yes. 37 REGISTERED VOTERS, OF WHICH EVERY
LEGISLATIVE DISTRICT MUST BE REPRESENTED
BY AT LEAST THREE PERCENT OF THE
Commissioner Davide also reaffirmed that his modified amendment strictly confines initiative to REGISTERED VOTERS THEREOF. NO AMENDMENT
AMENDMENTS to — NOT REVISION of — the Constitution. Thus: UNDER THIS SECTION SHALL BE AUTHORIZED
WITHIN FIVE YEARS FOLLOWING THE
MR. DAVIDE. With pleasure, Madam President. RATIFICATION OF THIS CONSTITUTION NOR
OFTENER THAN ONCE EVERY FIVE YEARS
THEREAFTER.
MR. MAAMBONG. My first question: Commissioner
Davide's proposed amendment on line 1 refers to
THE NATIONAL ASSEMBLY SHALL BY LAW subsequently approved on 8 June 1989 by the Senate 50 and by the House of
PROVIDE Representatives. 51 This approved bill is now R.A. No. 6735.
FOR THE IMPLEMENTATION OF THE EXERCISE OF
THIS RIGHT. 40
But is R.A. No. 6735 a full compliance with the power and duty of Congress to "provide for the
implementation of the exercise of the right?"
The entire proposed Article on Amendments or Revisions was approved on second
reading on 9 July 1986. 41Thereafter, upon his motion for reconsideration,
A careful scrutiny of the Act yields a negative answer.
Commissioner Gascon was allowed to introduce an amendment to Section 2 which,
nevertheless, was withdrawn. In view thereof, the Article was again approved on
Second and Third Readings on 1 August 1986. 42 First. Contrary to the assertion of public respondent COMELEC, Section 2 of the Act does not
suggest an initiative on amendments to the Constitution. The said section reads:
However, the Committee on Style recommended that the approved Section 2 be amended by
changing "percent" to "per centum" and "thereof" to "therein" and deleting the phrase "by law" in Sec. 2. Statement and Policy. — The power of the people under a system of
the second paragraph so that said paragraph reads: The Congress 43 shall provide for the initiative and referendum to directly propose, enact, approve or reject, in
implementation of the exercise of this right. 44 This amendment was approved and is the text of whole or in part, the Constitution, laws, ordinances, or resolutions passed by
the present second paragraph of Section 2. any legislative body upon compliance with the requirements of this Act is
hereby affirmed, recognized and guaranteed. (Emphasis supplied).
The conclusion then is inevitable that, indeed, the system of initiative on the Constitution under
Section 2 of Article XVII of the Constitution is not self-executory. The inclusion of the word "Constitution" therein was a delayed afterthought. That word
is neither germane nor relevant to said section, which exclusively relates to initiative
and referendum on national laws and local laws, ordinances, and resolutions. That
Has Congress "provided" for the implementation of the exercise of this right? Those who answer
section is silent as to amendments on the Constitution. As pointed out earlier, initiative
the question in the affirmative, like the private respondents and intervenor Senator Roco, point to
on the Constitution is confined only to proposals to AMEND. The people are not
us R.A. No. 6735.
accorded the power to "directly propose, enact, approve, or reject, in whole or in part,
the Constitution" through the system of initiative. They can only do so with respect to
There is, of course, no other better way for Congress to implement the exercise of the right than "laws, ordinances, or resolutions."
through the passage of a statute or legislative act. This is the essence or rationale of the last
minute amendment by the Constitutional Commission to substitute the last paragraph of Section
The foregoing conclusion is further buttressed by the fact that this section was lifted from Section
2 of Article XVII then reading:
1 of Senate Bill No. 17, which solely referred to a statement of policy on local initiative and
referendum and appropriately used the phrases "propose and enact," "approve or reject" and "in
The Congress 45 shall by law provide for the implementation of the exercise whole or in part." 52
of this right.
Second. It is true that Section 3 (Definition of Terms) of the Act defines initiative on amendments
with to the Constitution and mentions it as one of the three systems of initiative, and that Section 5
(Requirements) restates the constitutional requirements as to the percentage of the registered
voters who must submit the proposal. But unlike in the case of the other systems of initiative, the
The Congress shall provide for the implementation of the exercise of this
Act does not provide for the contents of a petition for initiative on the Constitution. Section 5,
right.
paragraph (c) requires, among other things, statement of the proposed law sought to be
enacted, approved or rejected, amended or repealed, as the case may be. It does not include,
This substitute amendment was an investiture on Congress of a power to provide for as among the contents of the petition, the provisions of the Constitution sought to be amended,
the rules implementing the exercise of the right. The "rules" means "the details on how in the case of initiative on the Constitution. Said paragraph (c) reads in full as follows:
[the right] is to be carried out." 46
(c) The petition shall state the following:
We agree that R.A. No. 6735 was, as its history reveals, intended to cover initiative to propose
amendments to the Constitution. The Act is a consolidation of House Bill No. 21505 and Senate
c.1 contents or text of the proposed law sought to be enacted, approved or
Bill No. 17. The former was prepared by the Committee on Suffrage and Electoral Reforms of
rejected, amended or repealed, as the case may be;
the House of Representatives on the basis of two House Bills referred to it, viz., (a) House Bill
No. 497, 47 which dealt with the initiative and referendum mentioned
in Sections 1 and 32 of Article VI of the Constitution; and (b) House Bill No. 988, 48 which dealt c.2 the proposition;
with the subject matter of House Bill No. 497, as well as with initiative and referendum under
Section 3 of Article X (Local Government) and initiative provided for in Section 2 of Article XVII of
c.3 the reason or reasons therefor;
the Constitution. Senate Bill No. 17 49 solely dealt with initiative and referendum concerning
ordinances or resolutions of local government units. The Bicameral Conference Committee
consolidated Senate Bill No. 17 and House Bill No. 21505 into a draft bill, which was c.4 that it is not one of the exceptions provided therein;
c.5 signatures of the petitioners or registered voters; and A further examination of the Act even reveals that the subtitling is not accurate. Provisions not
germane to the subtitle on National Initiative and Referendum are placed therein, like (1)
paragraphs (b) and (c) of Section 9, which reads:
c.6 an abstract or summary proposition is not more than one hundred (100)
words which shall be legibly written or printed at the top of every page of the
petition. (Emphasis supplied). (b) The proposition in an initiative on the Constitution approved by the
majority of the votes cast in the plebiscite shall become effective as to the
day of the plebiscite.
The use of the clause "proposed laws sought to be enacted, approved or rejected,
amended or repealed" only strengthens the conclusion that Section 2, quoted earlier,
excludes initiative on amendments to the Constitution. (c) A national or local initiative proposition approved by majority of the votes
cast in an election called for the purpose shall become effective fifteen (15)
days after certification and proclamation of the Commission. (Emphasis
Third. While the Act provides subtitles for National Initiative and Referendum (Subtitle II) and for
supplied).
Local Initiative and Referendum (Subtitle III), no subtitle is provided for initiative on the
Constitution. This conspicuous silence as to the latter simply means that the main thrust of the
Act is initiative and referendum on national and local laws. If Congress intended R.A. No. 6735 (2) that portion of Section 11 (Indirect Initiative) referring to indirect initiative with the legislative
to fully provide for the implementation of the initiative on amendments to the Constitution, it bodies of local governments; thus:
could have provided for a subtitle therefor, considering that in the order of things, the primacy of
interest, or hierarchy of values, the right of the people to directly propose amendments to the
Sec. 11. Indirect Initiative. — Any duly accredited people's organization, as
Constitution is far more important than the initiative on national and local laws.
defined by law, may file a petition for indirect initiative with the House of
Representatives, and other legislative bodies. . . .
We cannot accept the argument that the initiative on amendments to the Constitution is
subsumed under the subtitle on National Initiative and Referendum because it is national in
and (3) Section 12 on Appeal, since it applies to decisions of the COMELEC on the
scope. Our reading of Subtitle II (National Initiative and Referendum) and Subtitle III (Local
findings of sufficiency or insufficiency of the petition for initiative or referendum, which
Initiative and Referendum) leaves no room for doubt that the classification is not based on
could be petitions for both national and local initiative and referendum.
the scope of the initiative involved, but on its nature and character. It is "national initiative," if
what is proposed to be adopted or enacted is a national law, or a law which only Congress can
pass. It is "local initiative" if what is proposed to be adopted or enacted is a law, ordinance, or Upon the other hand, Section 18 on "Authority of Courts" under subtitle III on Local Initiative and
resolution which only the legislative bodies of the governments of the autonomous regions, Referendum is misplaced, 54 since the provision therein applies to both national and local
provinces, cities, municipalities, and barangays can pass. This classification of initiative initiative and referendum. It reads:
into national and local is actually based on Section 3 of the Act, which we quote for emphasis
and clearer understanding:
Sec. 18. Authority of Courts. — Nothing in this Act shall prevent or preclude
the proper courts from declaring null and void any proposition approved
Sec. 3. Definition of terms — pursuant to this Act for violation of the Constitution or want of capacity of the
local legislative body to enact the said measure.
xxx xxx xxx
Curiously, too, while R.A. No. 6735 exerted utmost diligence and care in providing for the details
in the implementation of initiative and referendum on national and local legislation thereby giving
There are three (3) systems of initiative, namely:
them special attention, it failed, rather intentionally, to do so on the system of initiative on
amendments to the Constitution. Anent the initiative on national legislation, the Act provides for
a.1 Initiative on the Constitution which refers to a petition proposing the following:
amendments to the Constitution;
(a) The required percentage of registered voters to sign the petition and the contents of the
a.2 Initiative on Statutes which refers to a petition proposing to enact petition;
a national legislation; and
(b) The conduct and date of the initiative;
a.3 Initiative on local legislation which refers to a petition proposing to enact
a regional, provincial, city, municipal, or barangay law, resolution or
(c) The submission to the electorate of the proposition and the required number of votes for its
ordinance. (Emphasis supplied).
approval;

Hence, to complete the classification under subtitles there should have been a subtitle on
(d) The certification by the COMELEC of the approval of the proposition;
initiative on amendments to the Constitution. 53

(e) The publication of the approved proposition in the Official Gazette or in a newspaper of
general circulation in the Philippines; and
55
(f) The effects of the approval or rejection of the proposition. The rule is that what has been delegated, cannot be delegated or as expressed in a Latin
maxim: potestas delegata non delegari potest. 59 The recognized exceptions to the rule are as
follows:
As regards local initiative, the Act provides for the following:

(1) Delegation of tariff powers to the President under Section 28(2) of Article VI of the
(a) The preliminary requirement as to the number of signatures of registered voters for the
Constitution;
petition;

(2) Delegation of emergency powers to the President under Section 23(2) of Article VI of the
(b) The submission of the petition to the local legislative body concerned;
Constitution;

(c) The effect of the legislative body's failure to favorably act thereon, and the invocation of the
(3) Delegation to the people at large;
power of initiative as a consequence thereof;

(4) Delegation to local governments; and


(d) The formulation of the proposition;

(5) Delegation to administrative bodies. 60


(e) The period within which to gather the signatures;

Empowering the COMELEC, an administrative body exercising quasi-judicial functions, to


(f) The persons before whom the petition shall be signed;
promulgate rules and regulations is a form of delegation of legislative authority under no. 5
above. However, in every case of permissible delegation, there must be a showing that the
(g) The issuance of a certification by the COMELEC through its official in the local government delegation itself is valid. It is valid only if the law (a) is complete in itself, setting forth therein the
unit concerned as to whether the required number of signatures have been obtained; policy to be executed, carried out, or implemented by the delegate; and (b) fixes a standard —
the limits of which are sufficiently determinate and determinable — to which the delegate must
conform in the performance of his functions. 61 A sufficient standard is one which defines
(h) The setting of a date by the COMELEC for the submission of the proposition to the registered legislative policy, marks its limits, maps out its boundaries and specifies the public agency to
voters for their approval, which must be within the period specified therein; apply it. It indicates the circumstances under which the legislative command is to be effected. 62

(i) The issuance of a certification of the result; Insofar as initiative to propose amendments to the Constitution is concerned, R.A. No. 6735
miserably failed to satisfy both requirements in subordinate legislation. The delegation of the
(j) The date of effectivity of the approved proposition; power to the COMELEC is then invalid.

(k) The limitations on local initiative; and III

56 COMELEC RESOLUTION NO. 2300, INSOFAR AS IT PRESCRIBES RULES AND


(l) The limitations upon local legislative bodies.
REGULATIONS ON THE CONDUCT OF INITIATIVE ON AMENDMENTS TO THE
CONSTITUTION, IS VOID.
Upon the other hand, as to initiative on amendments to the Constitution, R.A. No. 6735, in all of
its twenty-three sections, merely (a) mentions, the word "Constitution" in Section 2; (b) defines
"initiative on the Constitution" and includes it in the enumeration of the three systems of initiative It logically follows that the COMELEC cannot validly promulgate rules and regulations to
in Section 3; (c) speaks of "plebiscite" as the process by which the proposition in an initiative on implement the exercise of the right of the people to directly propose amendments to the
the Constitution may be approved or rejected by the people; (d) reiterates the constitutional Constitution through the system of initiative. It does not have that power under R.A. No. 6735.
requirements as to the number of voters who should sign the petition; and (e) provides for the Reliance on the COMELEC's power under Section 2(1) of Article IX-C of the Constitution is
date of effectivity of the approved proposition. misplaced, for the laws and regulations referred to therein are those promulgated by the
COMELEC under (a) Section 3 of Article IX-C of the Constitution, or (b) a law where subordinate
legislation is authorized and which satisfies the "completeness" and the "sufficient standard"
There was, therefore, an obvious downgrading of the more important or the paramount system tests.
of initiative. RA. No. 6735 thus delivered a humiliating blow to the system of initiative on
amendments to the Constitution by merely paying it a reluctant lip service. 57
IV
The foregoing brings us to the conclusion that R.A. No. 6735 is incomplete, inadequate, or
wanting in essential terms and conditions insofar as initiative on amendments to the Constitution COMELEC ACTED WITHOUT JURISDICTION OR WITH GRAVE ABUSE OF
is concerned. Its lacunae on this substantive matter are fatal and cannot be cured by DISCRETION IN ENTERTAINING THE DELFIN PETITION.
"empowering" the COMELEC "to promulgate such rules and regulations as may be necessary to
carry out the purposes of [the] Act. 58
Even if it be conceded ex gratia that R.A. No. 6735 is a full compliance with the power of a) GRANTING the instant petition;
Congress to implement the right to initiate constitutional amendments, or that it has validly
vested upon the COMELEC the power of subordinate legislation and that COMELEC Resolution
b) DECLARING R.A. No. 6735 inadequate to cover the system of initiative on amendments to
No. 2300 is valid, the COMELEC acted without jurisdiction or with grave abuse of discretion in
the Constitution, and to have failed to provide sufficient standard for subordinate legislation;
entertaining the Delfin Petition.

c) DECLARING void those parts of Resolution No. 2300 of the Commission on Elections
Under Section 2 of Article XVII of the Constitution and Section 5(b) of R.A. No. 6735, a petition
prescribing rules and regulations on the conduct of initiative or amendments to the Constitution;
for initiative on the Constitution must be signed by at least 12% of the total number of registered
and
voters of which every legislative district is represented by at least 3% of the registered voters
therein. The Delfin Petition does not contain signatures of the required number of voters. Delfin
himself admits that he has not yet gathered signatures and that the purpose of his petition is d) ORDERING the Commission on Elections to forthwith DISMISS the DELFIN petition (UND-
primarily to obtain assistance in his drive to gather signatures. Without the required signatures, 96-037).
the petition cannot be deemed validly initiated.
The Temporary Restraining Order issued on 18 December 1996 is made permanent as against
The COMELEC acquires jurisdiction over a petition for initiative only after its filing. The petition the Commission on Elections, but is LIFTED as against private respondents.
then is the initiatory pleading. Nothing before its filing is cognizable by the COMELEC, sitting en
banc. The only participation of the COMELEC or its personnel before the filing of such petition
are (1) to prescribe the form of the petition; 63 (2) to issue through its Election Records and Resolution on the matter of contempt is hereby reserved.
Statistics Office a certificate on the total number of registered voters in each legislative
district; 64 (3) to assist, through its election registrars, in the establishment of signature SO ORDERED.
stations; 65 and (4) to verify, through its election registrars, the signatures on the basis of the
registry list of voters, voters' affidavits, and voters' identification cards used in the immediately
preceding election. 66

Since the Delfin Petition is not the initiatory petition under R.A. No. 6735 and COMELEC
Resolution No. 2300, it cannot be entertained or given cognizance of by the COMELEC. The
respondent Commission must have known that the petition does not fall under any of the actions
or proceedings under the COMELEC Rules of Procedure or under Resolution No. 2300, for
which reason it did not assign to the petition a docket number. Hence, the said petition was
merely entered as UND, meaning, undocketed. That petition was nothing more than a mere
scrap of paper, which should not have been dignified by the Order of 6 December 1996, the
hearing on 12 December 1996, and the order directing Delfin and the oppositors to file their
memoranda or oppositions. In so dignifying it, the COMELEC acted without jurisdiction or with
grave abuse of discretion and merely wasted its time, energy, and resources.

The foregoing considered, further discussion on the issue of whether the proposal to lift the term
limits of elective national and local officials is an amendment to, and not a revision of, the
Constitution is rendered unnecessary, if not academic.

CONCLUSION

This petition must then be granted, and the COMELEC should be permanently enjoined from
entertaining or taking cognizance of any petition for initiative on amendments to the Constitution
until a sufficient law shall have been validly enacted to provide for the implementation of the
system.

We feel, however, that the system of initiative to propose amendments to the Constitution should
no longer be kept in the cold; it should be given flesh and blood, energy and strength. Congress
should not tarry any longer in complying with the constitutional mandate to provide for the
implementation of the right of the people under that system.

WHEREFORE, judgment is hereby rendered


G.R. No. 174153 October 25, 2006 LUWALHATI RICASA ANTONINO, Intervenor.

RAUL L. LAMBINO and ERICO B. AUMENTADO, TOGETHER WITH 6,327,952 x ------------------------------------------------------- x


REGISTERED VOTERS,Petitioners,
vs.
PHILIPPINE CONSTITUTION ASSOCIATION (PHILCONSA), CONRADO F. ESTRELLA,
THE COMMISSION ON ELECTIONS, Respondent.
TOMAS C. TOLEDO, MARIANO M. TAJON, FROILAN M. BACUNGAN, JOAQUIN T. VENUS,
JR., FORTUNATO P. AGUAS, and AMADO GAT INCIONG, Intervenors.
x--------------------------------------------------------x
x ------------------------------------------------------- x
ALTERNATIVE LAW GROUPS, INC., Intervenor.
RONALD L. ADAMAT, ROLANDO MANUEL RIVERA, and RUELO BAYA, Intervenors.
x ------------------------------------------------------ x
x -------------------------------------------------------- x
ONEVOICE INC., CHRISTIAN S.MONSOD, RENE B. AZURIN, MANUEL L. QUEZON III,
BENJAMIN T. TOLOSA, JR., SUSAN V. OPLE, and CARLOS P. MEDINA, JR., Intervenors.
PHILIPPINE TRANSPORT AND GENERAL WORKERS ORGANIZATION (PTGWO) and MR.
VICTORINO F. BALAIS, Intervenors.
x------------------------------------------------------ x
x -------------------------------------------------------- x
ATTY. PETE QUIRINO QUADRA, Intervenor.
SENATE OF THE PHILIPPINES, represented by its President, MANUEL VILLAR,
x--------------------------------------------------------x JR., Intervenor.

BAYAN represented by its Chairperson Dr. Carolina Pagaduan-Araullo, BAYAN MUNA x ------------------------------------------------------- x
represented by its Chairperson Dr. Reynaldo Lesaca, KILUSANG MAYO UNO represented
by its Secretary General Joel Maglunsod, HEAD represented by its Secretary General Dr.
SULONG BAYAN MOVEMENT FOUNDATION, INC., Intervenor.
Gene Alzona Nisperos, ECUMENICAL BISHOPS FORUM represented by Fr. Dionito
Cabillas, MIGRANTE represented by its Chairperson Concepcion Bragas-Regalado,
GABRIELA represented by its Secretary General Emerenciana de Jesus, GABRIELA x ------------------------------------------------------- x
WOMEN'S PARTY represented by Sec. Gen. Cristina Palabay, ANAKBAYAN represented
by Chairperson Eleanor de Guzman, LEAGUE OF FILIPINO STUDENTS represented by
Chair Vencer Crisostomo Palabay, JOJO PINEDA of the League of Concerned JOSE ANSELMO I. CADIZ, BYRON D. BOCAR, MA. TANYA KARINA A. LAT, ANTONIO L.
SALVADOR, and RANDALL TABAYOYONG, Intervenors.
Professionals and Businessmen, DR. DARBY SANTIAGO of the Solidarity of Health
Against Charter Change, DR. REGINALD PAMUGAS of Health Action for Human
Rights,Intervenors. x -------------------------------------------------------- x

x--------------------------------------------------------x INTEGRATED BAR OF THE PHILIPPINES, CEBU CITY AND CEBU PROVINCE
CHAPTERS, Intervenors.
LORETTA ANN P. ROSALES, MARIO JOYO AGUJA, and ANA THERESA HONTIVEROS-
BARAQUEL,Intervenors. x --------------------------------------------------------x

x--------------------------------------------------------x SENATE MINORITY LEADER AQUILINO Q. PIMENTEL, JR. and SENATORS SERGIO R.
OSMENA III, JAMBY MADRIGAL, JINGGOY ESTRADA, ALFREDO S. LIM and PANFILO
ARTURO M. DE CASTRO, Intervenor. LACSON, Intervenors.

x ------------------------------------------------------- x x -----------------------------------------------------x

TRADE UNION CONGRESS OF THE PHILIPPINES, Intervenor. JOSEPH EJERCITO ESTRADA and PWERSA NG MASANG PILIPINO, Intervenors.

x -----------------------------------------------------x
x---------------------------------------------------------x
G.R. No. 174299 October 25, 2006 The Ruling of the COMELEC

MAR-LEN ABIGAIL BINAY, SOFRONIO UNTALAN, JR., and RENE A.V. On 31 August 2006, the COMELEC issued its Resolution denying due course to the Lambino
SAGUISAG, Petitioners, Group's petition for lack of an enabling law governing initiative petitions to amend the
vs. Constitution. The COMELEC invoked this Court's ruling in Santiago v. Commission on
COMMISSION ON ELECTIONS, represented by Chairman BENJAMIN S. ABALOS, SR., and Elections8 declaring RA 6735 inadequate to implement the initiative clause on proposals to
Commissioners RESURRECCION Z. BORRA, FLORENTINO A. TUASON, JR., ROMEO A. amend the Constitution.9
BRAWNER, RENE V. SARMIENTO, NICODEMO T. FERRER, and John Doe and Peter
Doe,, Respondent.
In G.R. No. 174153, the Lambino Group prays for the issuance of the writs of certiorari and
mandamus to set aside the COMELEC Resolution of 31 August 2006 and to compel the
DECISION COMELEC to give due course to their initiative petition. The Lambino Group contends that the
COMELEC committed grave abuse of discretion in denying due course to their petition
since Santiago is not a binding precedent. Alternatively, the Lambino Group claims
CARPIO, J.:
that Santiago binds only the parties to that case, and their petition deserves cognizance as an
expression of the "will of the sovereign people."
The Case
In G.R. No. 174299, petitioners ("Binay Group") pray that the Court require respondent
These are consolidated petitions on the Resolution dated 31 August 2006 of the Commission on COMELEC Commissioners to show cause why they should not be cited in contempt for the
Elections ("COMELEC") denying due course to an initiative petition to amend the 1987 COMELEC's verification of signatures and for "entertaining" the Lambino Group's petition
Constitution. despite the permanent injunction in Santiago. The Court treated the Binay Group's petition as
an opposition-in-intervention.
Antecedent Facts
In his Comment to the Lambino Group's petition, the Solicitor General joined causes with the
petitioners, urging the Court to grant the petition despite the Santiago ruling. The Solicitor
On 15 February 2006, petitioners in G.R. No. 174153, namely Raul L. Lambino and Erico B. General proposed that the Court treat RA 6735 and its implementing rules "as temporary
Aumentado ("Lambino Group"), with other groups1 and individuals, commenced gathering
devises to implement the system of initiative."
signatures for an initiative petition to change the 1987 Constitution. On 25 August 2006, the
Lambino Group filed a petition with the COMELEC to hold a plebiscite that will ratify their
initiative petition under Section 5(b) and (c)2 and Section 73 of Republic Act No. 6735 or the Various groups and individuals sought intervention, filing pleadings supporting or opposing the
Initiative and Referendum Act ("RA 6735"). Lambino Group's petition. The supporting intervenors10 uniformly hold the view that the
COMELEC committed grave abuse of discretion in relying on Santiago. On the other hand, the
opposing intervenors11 hold the contrary view and maintain that Santiago is a binding
The Lambino Group alleged that their petition had the support of 6,327,952 individuals precedent. The opposing intervenors also challenged (1) the Lambino Group's standing to file
constituting at least twelve per centum (12%) of all registered voters, with each legislative district the petition; (2) the validity of the signature gathering and verification process; (3) the Lambino
represented by at least three per centum (3%) of its registered voters. The Lambino Group also Group's compliance with the minimum requirement for the percentage of voters supporting an
claimed that COMELEC election registrars had verified the signatures of the 6.3 million initiative petition under Section 2, Article XVII of the 1987 Constitution;12 (4) the nature of the
individuals. proposed changes as revisions and not mere amendments as provided under Section 2, Article
XVII of the 1987 Constitution; and (5) the Lambino Group's compliance with the requirement in
The Lambino Group's initiative petition changes the 1987 Constitution by modifying Sections 1-7 Section 10(a) of RA 6735 limiting initiative petitions to only one subject.
of Article VI (Legislative Department)4 and Sections 1-4 of Article VII (Executive
Department)5 and by adding Article XVIII entitled "Transitory Provisions."6 These proposed
The Court heard the parties and intervenors in oral arguments on 26 September 2006. After
changes will shift the present Bicameral-Presidential system to a Unicameral-Parliamentary form receiving the parties' memoranda, the Court considered the case submitted for resolution.
of government. The Lambino Group prayed that after due publication of their petition, the
COMELEC should submit the following proposition in a plebiscite for the voters' ratification:
The Issues
DO YOU APPROVE THE AMENDMENT OF ARTICLES VI AND VII OF THE 1987
CONSTITUTION, CHANGING THE FORM OF GOVERNMENT FROM THE The petitions raise the following issues:
PRESENT BICAMERAL-PRESIDENTIAL TO A UNICAMERAL-PARLIAMENTARY
SYSTEM, AND PROVIDING ARTICLE XVIII AS TRANSITORY PROVISIONS FOR
1. Whether the Lambino Group's initiative petition complies with Section 2, Article XVII of the
THE ORDERLY SHIFT FROM ONE SYSTEM TO THE OTHER?
Constitution on amendments to the Constitution through a people's initiative;

On 30 August 2006, the Lambino Group filed an Amended Petition with the COMELEC
2. Whether this Court should revisit its ruling in Santiago declaring RA 6735 "incomplete,
indicating modifications in the proposed Article XVIII (Transitory Provisions) of their initiative. 7
inadequate or wanting in essential terms and conditions" to implement the initiative clause on
proposals to amend the Constitution; and
3. Whether the COMELEC committed grave abuse of discretion in denying due course to the The essence of amendments "directly proposed by the people through initiative upon a
Lambino Group's petition. petition" is that the entire proposal on its face is a petition by the people. This means two
essential elements must be present. First, the people must author and thus sign the entire
proposal. No agent or representative can sign on their behalf. Second, as an initiative upon a
The Ruling of the Court
petition, the proposal must be embodied in a petition.

There is no merit to the petition.


These essential elements are present only if the full text of the proposed amendments is first
shown to the people who express their assent by signing such complete proposal in a
The Lambino Group miserably failed to comply with the basic requirements of the Constitution petition. Thus, an amendment is "directly proposed by the people through initiative upon a
for conducting a people's initiative. Thus, there is even no need to revisit Santiago, as the petition" only if the people sign on a petition that contains the full text of the proposed
present petition warrants dismissal based alone on the Lambino Group's glaring failure to amendments.
comply with the basic requirements of the Constitution. For following the Court's ruling in
Santiago, no grave abuse of discretion is attributable to the Commision on Elections.
The full text of the proposed amendments may be either written on the face of the petition, or
attached to it. If so attached, the petition must state the fact of such attachment. This is an
1. The Initiative Petition Does Not Comply with Section 2, Article XVII of the Constitution assurance that every one of the several millions of signatories to the petition had seen the full
on Direct Proposal by the People text of the proposed amendments before signing. Otherwise, it is physically impossible, given
the time constraint, to prove that every one of the millions of signatories had seen the full text of
the proposed amendments before signing.
Section 2, Article XVII of the Constitution is the governing constitutional provision that allows a
people's initiative to propose amendments to the Constitution. This section states:
The framers of the Constitution directly borrowed14 the concept of people's initiative from the
United States where various State constitutions incorporate an initiative clause. In almost all
Sec. 2. Amendments to this Constitution may likewise be directly proposed by the States15 which allow initiative petitions, the unbending requirement is that the people must
people through initiative upon a petition of at least twelve per centum of the total
first see the full text of the proposed amendments before they sign to signify their assent,
number of registered voters of which every legislative district must be represented by and that the people must sign on an initiative petition that contains the full text of the
at least three per centum of the registered voters therein. x x x x (Emphasis supplied) proposed amendments.16

The deliberations of the Constitutional Commission vividly explain the meaning of an The rationale for this requirement has been repeatedly explained in several decisions of various
amendment "directly proposed by the people through initiative upon a petition," thus:
courts. Thus, in Capezzuto v. State Ballot Commission, the Supreme Court of
Massachusetts, affirmed by the First Circuit Court of Appeals, declared:
MR. RODRIGO: Let us look at the mechanics. Let us say some voters want to
propose a constitutional amendment. Is the draft of the proposed constitutional [A] signature requirement would be meaningless if the person supplying the
amendment ready to be shown to the people when they are asked to sign? signature has not first seen what it is that he or she is signing. Further, and more
importantly, loose interpretation of the subscription requirement can pose a significant
MR. SUAREZ: That can be reasonably assumed, Madam President. potential for fraud. A person permitted to describe orally the contents of an initiative
petition to a potential signer, without the signer having actually examined the petition,
could easily mislead the signer by, for example, omitting, downplaying, or even flatly
MR. RODRIGO: What does the sponsor mean? The draft is ready and shown to misrepresenting, portions of the petition that might not be to the signer's liking. This
them before they sign. Now, who prepares the draft? danger seems particularly acute when, in this case, the person giving the
description is the drafter of the petition, who obviously has a vested interest in
MR. SUAREZ: The people themselves, Madam President. seeing that it gets the requisite signatures to qualify for the ballot.17 (Boldfacing
and underscoring supplied)
MR. RODRIGO: No, because before they sign there is already a draft shown to
them and they are asked whether or not they want to propose this constitutional Likewise, in Kerr v. Bradbury,18 the Court of Appeals of Oregon explained:
amendment.
The purposes of "full text" provisions that apply to amendments by initiative commonly
MR. SUAREZ: As it is envisioned, any Filipino can prepare that proposal and pass are described in similar terms. x x x (The purpose of the full text requirement is to
it around for signature.13 (Emphasis supplied) provide sufficient information so that registered voters can intelligently evaluate
whether to sign the initiative petition."); x x x (publication of full text of amended
constitutional provision required because it is "essential for the elector to have x x x
Clearly, the framers of the Constitution intended that the "draft of the proposed constitutional the section which is proposed to be added to or subtracted from. If he is to vote
amendment" should be "ready and shown" to the people "before" they sign such proposal. intelligently, he must have this knowledge. Otherwise in many instances he would be
The framers plainly stated that "before they sign there is already a draft shown to them." The required to vote in the dark.") (Emphasis supplied)
framers also "envisioned" that the people should sign on the proposal itself because the
proponents must "prepare that proposal and pass it around for signature."
Moreover, "an initiative signer must be informed at the time of signing of the nature and I hereby APPROVE the proposed amendment to the 1987 Constitution. My signature herein
effect of that which is proposed" and failure to do so is "deceptive and misleading" which which shall form part of the petition for initiative to amend the Constitution signifies my support
renders the initiative void.19 for the filing thereof.

Section 2, Article XVII of the Constitution does not expressly state that the petition must set forth Precinct Name Address Birthdate Signature Verification
the full text of the proposed amendments. However, the deliberations of the framers of our Number
Constitution clearly show that the framers intended to adopt the relevant American jurisprudence
on people's initiative. In particular, the deliberations of the Constitutional Commission explicitly Last Name, MM/DD/YY
reveal that the framers intended that the people must first see the full text of the proposed First Name,
amendments before they sign, and that the people must sign on a petition containing M.I.
such full text. Indeed, Section 5(b) of Republic Act No. 6735, the Initiative and Referendum Act 1
that the Lambino Group invokes as valid, requires that the people must sign the "petition x x x 2
as signatories." 3
4
5
The proponents of the initiative secure the signatures from the people. The proponents secure 6
the signatures in their private capacity and not as public officials. The proponents are not 7
disinterested parties who can impartially explain the advantages and disadvantages of the
8
proposed amendments to the people. The proponents present favorably their proposal to the
9
people and do not present the arguments against their proposal. The proponents, or their
supporters, often pay those who gather the signatures. 10
_________________ _________________ __________________
Thus, there is no presumption that the proponents observed the constitutional requirements in Barangay Official Witness Witness
gathering the signatures. The proponents bear the burden of proving that they complied with the (Print Name and Sign) (Print Name and Sign) (Print Name and Sign)
constitutional requirements in gathering the signatures - that the petition contained, or
incorporated by attachment, the full text of the proposed amendments.
There is not a single word, phrase, or sentence of text of the Lambino Group's proposed
changes in the signature sheet. Neither does the signature sheet state that the text of the
The Lambino Group did not attach to their present petition with this Court a copy of the paper proposed changes is attached to it. Petitioner Atty. Raul Lambino admitted this during the oral
that the people signed as their initiative petition. The Lambino Group submitted to this Court a arguments before this Court on 26 September 2006.
copy of a signature sheet20 after the oral arguments of 26 September 2006 when they filed their
Memorandum on 11 October 2006. The signature sheet with this Court during the oral
arguments was the signature sheet attached21 to the opposition in intervention filed on 7 The signature sheet merely asks a question whether the people approve a shift from the
September 2006 by intervenor Atty. Pete Quirino-Quadra. Bicameral-Presidential to the Unicameral-Parliamentary system of government. The signature
sheet does not show to the people the draft of the proposed changes before they are
asked to sign the signature sheet. Clearly, the signature sheet is not the "petition" that the
The signature sheet attached to Atty. Quadra's opposition and the signature sheet attached to framers of the Constitution envisioned when they formulated the initiative clause in Section 2,
the Lambino Group's Memorandum are the same. We reproduce below the signature sheet in Article XVII of the Constitution.
full:

Petitioner Atty. Lambino, however, explained that during the signature-gathering from February
Province: City/Municipality: No. of to August 2006, the Lambino Group circulated, together with the signature sheets, printed copies
Legislative Barangay: of the Lambino Group's draft petition which they later filed on 25 August 2006 with the
District: Verified COMELEC. When asked if his group also circulated the draft of their amended petition filed on
30 August 2006 with the COMELEC, Atty. Lambino initially replied that they circulated both.
However, Atty. Lambino changed his answer and stated that what his group circulated was the
Signatures: draft of the 30 August 2006 amended petition, not the draft of the 25 August 2006 petition.

PROPOSITION: "DO YOU APPROVE OF THE AMENDMENT OF ARTICLES VI AND VII OF The Lambino Group would have this Court believe that they prepared the draft of the 30 August
THE 1987 CONSTITUTION, CHANGING THE FORM OF GOVERNMENT FROM THE 2006 amended petition almost seven months earlier in February 2006 when they started
PRESENT BICAMERAL-PRESIDENTIAL TO A UNICAMERAL-PARLIAMENTARY SYSTEM OF gathering signatures. Petitioner Erico B. Aumentado's "Verification/Certification" of the 25
GOVERNMENT, IN ORDER TO ACHIEVE GREATER EFFICIENCY, SIMPLICITY AND August 2006 petition, as well as of the 30 August 2006 amended petition, filed with the
ECONOMY IN GOVERNMENT; AND PROVIDING AN ARTICLE XVIII AS TRANSITORY COMELEC, states as follows:
PROVISIONS FOR THE ORDERLY SHIFT FROM ONE SYSTEM TO ANOTHER?"
I have caused the preparation of the foregoing [Amended] Petition in my personal
capacity as a registered voter, for and on behalf of the Union of Local Authorities
of the Philippines, as shown by ULAP Resolution No. 2006-02 hereto attached, amending the 1987 Constitution." The proposals of the Consultative Commission24 are vastly
and as representative of the mass of signatories hereto. (Emphasis supplied) different from the proposed changes of the Lambino Group in the 25 August 2006 petition or 30
August 2006 amended petition filed with the COMELEC.
The Lambino Group failed to attach a copy of ULAP Resolution No. 2006-02 to the present
petition. However, the "Official Website of the Union of Local Authorities of the Philippines" 22 has For example, the proposed revisions of the Consultative Commission affect all provisions of the
posted the full text of Resolution No. 2006-02, which provides: existing Constitution, from the Preamble to the Transitory Provisions. The proposed
revisions have profound impact on the Judiciary and the National Patrimony provisions of the
existing Constitution, provisions that the Lambino Group's proposed changes do not touch. The
RESOLUTION NO. 2006-02
Lambino Group's proposed changes purport to affect only Articles VI and VII of the existing
Constitution, including the introduction of new Transitory Provisions.
RESOLUTION SUPPORTING THE PROPOSALS OF THE PEOPLE'S
CONSULTATIVE COMMISSION ON CHARTER CHANGE THROUGH PEOPLE'S
The ULAP adopted Resolution No. 2006-02 on 14 January 2006 or more than six months before
INITIATIVE AND REFERENDUM AS A MODE OF AMENDING THE 1987
the filing of the 25 August 2006 petition or the 30 August 2006 amended petition with the
CONSTITUTION
COMELEC. However, ULAP Resolution No. 2006-02 does not establish that ULAP or the
Lambino Group caused the circulation of the draft petition, together with the signature sheets, six
WHEREAS, there is a need for the Union of Local Authorities of the Philippines months before the filing with the COMELEC. On the contrary, ULAP Resolution No. 2006-02
(ULAP) to adopt a common stand on the approach to support the proposals of the casts grave doubt on the Lambino Group's claim that they circulated the draft petition
People's Consultative Commission on Charter Change; together with the signature sheets. ULAP Resolution No. 2006-02 does not refer at all to
the draft petition or to the Lambino Group's proposed changes.
WHEREAS, ULAP maintains its unqualified support to the agenda of Her Excellency
President Gloria Macapagal-Arroyo for constitutional reforms as embodied in the In their Manifestation explaining their amended petition before the COMELEC, the Lambino
ULAP Joint Declaration for Constitutional Reforms signed by the members of the Group declared:
ULAP and the majority coalition of the House of Representatives in Manila Hotel
sometime in October 2005;
After the Petition was filed, Petitioners belatedly realized that the proposed
amendments alleged in the Petition, more specifically, paragraph 3 of Section 4 and
WHEREAS, the People's Consultative Commission on Charter Change created by paragraph 2 of Section 5 of the Transitory Provisions were inaccurately stated and
Her Excellency to recommend amendments to the 1987 Constitution has submitted its failed to correctly reflect their proposed amendments.
final report sometime in December 2005;
The Lambino Group did not allege that they were amending the petition because the amended
WHEREAS, the ULAP is mindful of the current political developments in Congress petition was what they had shown to the people during the February to August 2006 signature-
which militates against the use of the expeditious form of amending the 1987 gathering. Instead, the Lambino Group alleged that the petition of 25 August 2006 "inaccurately
Constitution; stated and failed to correctly reflect their proposed amendments."

WHEREAS, subject to the ratification of its institutional members and the failure of The Lambino Group never alleged in the 25 August 2006 petition or the 30 August 2006
Congress to amend the Constitution as a constituent assembly, ULAP has amended petition with the COMELEC that they circulated printed copies of the draft petition
unanimously agreed to pursue the constitutional reform agenda through People's together with the signature sheets. Likewise, the Lambino Group did not allege in their present
Initiative and Referendum without prejudice to other pragmatic means to pursue the petition before this Court that they circulated printed copies of the draft petition together with the
same; signature sheets. The signature sheets do not also contain any indication that the draft petition is
attached to, or circulated with, the signature sheets.
WHEREFORE, BE IT RESOLVED AS IT IS HEREBY RESOLVED, THAT ALL THE
MEMBER-LEAGUES OF THE UNION OF LOCAL AUTHORITIES OF THE It is only in their Consolidated Reply to the Opposition-in-Interventions that the Lambino Group
PHILIPPINES (ULAP) SUPPORT THE PORPOSALS (SIC) OF THE PEOPLE'S first claimed that they circulated the "petition for initiative filed with the COMELEC," thus:
CONSULATATIVE (SIC) COMMISSION ON CHARTER CHANGE THROUGH
PEOPLE'S INITIATIVE AND REFERENDUM AS A MODE OF AMENDING THE 1987
[T]here is persuasive authority to the effect that "(w)here there is not (sic) fraud, a
CONSTITUTION;
signer who did not read the measure attached to a referendum petition cannot
question his signature on the ground that he did not understand the nature of
DONE, during the ULAP National Executive Board special meeting held on 14 the act." [82 C.J.S. S128h. Mo. State v. Sullivan, 224, S.W. 327, 283 Mo. 546.]
January 2006 at the Century Park Hotel, Manila.23 (Underscoring supplied) Thus, the registered voters who signed the signature sheets circulated together
with the petition for initiative filed with the COMELEC below, are presumed to
have understood the proposition contained in the petition. (Emphasis supplied)
ULAP Resolution No. 2006-02 does not authorize petitioner Aumentado to prepare the 25
August 2006 petition, or the 30 August 2006 amended petition, filed with the COMELEC. ULAP
Resolution No. 2006-02 "support(s) the porposals (sic) of the Consulatative (sic) The Lambino Group's statement that they circulated to the people "the petition for initiative
Commission on Charter Change through people's initiative and referendum as a mode of filed with the COMELEC" appears an afterthought, made after the intervenors Integrated Bar of
the Philippines (Cebu City Chapter and Cebu Province Chapters) and Atty. Quadra had pointed the 6.3 million signatories the full text of the proposed changes. If ever, not more than one
out that the signature sheets did not contain the text of the proposed changes. In their million signatories saw the petition before they signed the signature sheets.
Consolidated Reply, the Lambino Group alleged that they circulated "the petition for initiative"
but failed to mention the amended petition. This contradicts what Atty. Lambino finally stated
In any event, the Lambino Group's signature sheets do not contain the full text of the proposed
during the oral arguments that what they circulated was the draft of the amended petition of 30
changes, either on the face of the signature sheets, or as attachment with an indication in the
August 2006.
signature sheet of such attachment. Petitioner Atty. Lambino admitted this during the oral
arguments, and this admission binds the Lambino Group. This fact is also obvious from a
The Lambino Group cites as authority Corpus Juris Secundum, stating that "a signer who did not mere reading of the signature sheet. This omission is fatal. The failure to so include the text
read the measure attached to a referendum petition cannot question his signature on the of the proposed changes in the signature sheets renders the initiative void for non-compliance
ground that he did not understand the nature of the act." The Lambino Group quotes an with the constitutional requirement that the amendment must be "directly proposed by the
authority that cites a proposed change attached to the petition signed by the people. Even people through initiative upon a petition." The signature sheet is not the "petition"
the authority the Lambino Group quotes requires that the proposed change must be attached to envisioned in the initiative clause of the Constitution.
the petition. The same authority the Lambino Group quotes requires the people to sign on the
petition itself.
For sure, the great majority of the 6.3 million people who signed the signature sheets did not see
the full text of the proposed changes before signing. They could not have known the nature and
Indeed, it is basic in American jurisprudence that the proposed amendment must be effect of the proposed changes, among which are:
incorporated with, or attached to, the initiative petition signed by the people. In the present
initiative, the Lambino Group's proposed changes were not incorporated with, or attached to, the
1. The term limits on members of the legislature will be lifted and thus members
signature sheets. The Lambino Group's citation of Corpus Juris Secundumpulls the rug from
of Parliament can be re-elected indefinitely;26
under their feet.

2. The interim Parliament can continue to function indefinitely until its members, who
It is extremely doubtful that the Lambino Group prepared, printed, circulated, from February to
are almost all the present members of Congress, decide to call for new parliamentary
August 2006 during the signature-gathering period, the draft of the petition or amended petition
elections. Thus, the members of the interim Parliament will determine the
they filed later with the COMELEC. The Lambino Group are less than candid with this Court in
expiration of their own term of office; 27
their belated claim that they printed and circulated, together with the signature sheets, the
petition or amended petition. Nevertheless, even assuming the Lambino Group circulated
the amended petition during the signature-gathering period, the Lambino Group admitted 3. Within 45 days from the ratification of the proposed changes, the interim
circulating only very limited copies of the petition. Parliament shall convene to propose further amendments or revisions to the
Constitution.28
During the oral arguments, Atty. Lambino expressly admitted that they printed only 100,000
copies of the draft petition they filed more than six months later with the COMELEC. Atty. These three specific amendments are not stated or even indicated in the Lambino Group's
Lambino added that he also asked other supporters to print additional copies of the draft petition signature sheets. The people who signed the signature sheets had no idea that they were
but he could not state with certainty how many additional copies the other supporters proposing these amendments. These three proposed changes are highly controversial. The
printed. Atty. Lambino could only assure this Court of the printing of 100,000 copies people could not have inferred or divined these proposed changes merely from a reading or
because he himself caused the printing of these 100,000 copies. rereading of the contents of the signature sheets.

Likewise, in the Lambino Group's Memorandum filed on 11 October 2006, the Lambino Group During the oral arguments, petitioner Atty. Lambino stated that he and his group assured the
expressly admits that "petitioner Lambino initiated the printing and reproduction of people during the signature-gathering that the elections for the regular Parliament would
100,000 copies of the petition for initiative x x x."25 This admission binds the Lambino be held during the 2007 local elections if the proposed changes were ratified before the 2007
Group and establishes beyond any doubt that the Lambino Group failed to show the full local elections. However, the text of the proposed changes belies this.
text of the proposed changes to the great majority of the people who signed the signature
sheets.
The proposed Section 5(2), Article XVIII on Transitory Provisions, as found in the amended
petition, states:
Thus, of the 6.3 million signatories, only 100,000 signatories could have received with certainty
one copy each of the petition, assuming a 100 percent distribution with no wastage. If Atty.
Section 5(2). The interim Parliament shall provide for the election of the members of
Lambino and company attached one copy of the petition to each signature sheet, only 100,000
Parliament, which shall be synchronized and held simultaneously with the
signature sheets could have circulated with the petition. Each signature sheet contains space for
election of all local government officials. x x x x (Emphasis supplied)
ten signatures. Assuming ten people signed each of these 100,000 signature sheets with the
attached petition, the maximum number of people who saw the petition before they signed the
signature sheets would not exceed 1,000,000. Section 5(2) does not state that the elections for the regular Parliament will be held
simultaneously with the 2007 local elections. This section merely requires that the elections for
the regular Parliament shall be held simultaneously with the local elections without specifying
With only 100,000 printed copies of the petition, it would be physically impossible for all or a
the year.
great majority of the 6.3 million signatories to have seen the petition before they signed the
signature sheets. The inescapable conclusion is that the Lambino Group failed to show to
Petitioner Atty. Lambino, who claims to be the principal drafter of the proposed changes, could language in the instant case fails to do that. The very broadness of the proposal
have easily written the word "next" before the phrase "election of all local government officials." makes it impossible to state what it will affect and effect and violates the requirement
This would have insured that the elections for the regular Parliament would be held in the next that proposed amendments embrace only one subject. (Emphasis supplied)
local elections following the ratification of the proposed changes. However, the absence of the
word "next" allows the interim Parliament to schedule the elections for the regular Parliament
Logrolling confuses and even deceives the people. In Yute Air Alaska v. McAlpine,30 the
simultaneously with any future local elections.
Supreme Court of Alaska warned against "inadvertence, stealth and fraud" in logrolling:

Thus, the members of the interim Parliament will decide the expiration of their own term of office.
Whenever a bill becomes law through the initiative process, all of the problems that the single-
This allows incumbent members of the House of Representatives to hold office beyond their
subject rule was enacted to prevent are exacerbated. There is a greater danger of logrolling, or
current three-year term of office, and possibly even beyond the five-year term of office of regular
the deliberate intermingling of issues to increase the likelihood of an initiative's passage,
members of the Parliament. Certainly, this is contrary to the representations of Atty.
and there is a greater opportunity for "inadvertence, stealth and fraud" in the enactment-
Lambino and his group to the 6.3 million people who signed the signature sheets. Atty.
by-initiative process. The drafters of an initiative operate independently of any structured or
Lambino and his group deceived the 6.3 million signatories, and even the entire nation.
supervised process. They often emphasize particular provisions of their proposition, while
remaining silent on other (more complex or less appealing) provisions, when communicating to
This lucidly shows the absolute need for the people to sign an initiative petition that contains the the public. x x x Indeed, initiative promoters typically use simplistic advertising to present
full text of the proposed amendments to avoid fraud or misrepresentation. In the present their initiative to potential petition-signers and eventual voters. Many voters will never read
initiative, the 6.3 million signatories had to rely on the verbal representations of Atty. Lambino the full text of the initiative before the election. More importantly, there is no process for
and his group because the signature sheets did not contain the full text of the proposed amending or splitting the several provisions in an initiative proposal. These difficulties clearly
changes. The result is a grand deception on the 6.3 million signatories who were led to believe distinguish the initiative from the legislative process. (Emphasis supplied)
that the proposed changes would require the holding in 2007 of elections for the regular
Parliament simultaneously with the local elections.
Thus, the present initiative appears merely a preliminary step for further amendments or
revisions to be undertaken by the interim Parliament as a constituent assembly. The people who
The Lambino Group's initiative springs another surprise on the people who signed the signature signed the signature sheets could not have known that their signatures would be used to
sheets. The proposed changes mandate the interim Parliament to make further amendments or propose an amendment mandating the interim Parliament to propose further amendments or
revisions to the Constitution. The proposed Section 4(4), Article XVIII on Transitory Provisions, revisions to the Constitution.
provides:
Apparently, the Lambino Group inserted the proposed Section 4(4) to compel the interim
Section 4(4). Within forty-five days from ratification of these amendments, the interim Parliament to amend or revise again the Constitution within 45 days from ratification of the
Parliament shall convene to propose amendments to, or revisions of, this proposed changes, or before the May 2007 elections. In the absence of the proposed Section
Constitution consistent with the principles of local autonomy, decentralization and a 4(4), the interim Parliament has the discretion whether to amend or revise again the
strong bureaucracy. (Emphasis supplied) Constitution. With the proposed Section 4(4), the initiative proponents want the interim
Parliament mandated to immediately amend or revise again the Constitution.
During the oral arguments, Atty. Lambino stated that this provision is a "surplusage" and the
Court and the people should simply ignore it. Far from being a surplusage, this provision However, the signature sheets do not explain the reason for this rush in amending or revising
invalidates the Lambino Group's initiative. again so soon the Constitution. The signature sheets do not also explain what specific
amendments or revisions the initiative proponents want the interim Parliament to make, and why
there is a need for such further amendments or revisions. The people are again left in the dark
Section 4(4) is a subject matter totally unrelated to the shift from the Bicameral-Presidential to
to fathom the nature and effect of the proposed changes. Certainly, such an initiative is not
the Unicameral-Parliamentary system. American jurisprudence on initiatives outlaws this
"directly proposed by the people" because the people do not even know the nature and effect of
as logrolling - when the initiative petition incorporates an unrelated subject matter in the same
the proposed changes.
petition. This puts the people in a dilemma since they can answer only either yes or no to the
entire proposition, forcing them to sign a petition that effectively contains two propositions, one
of which they may find unacceptable. There is another intriguing provision inserted in the Lambino Group's amended petition of 30
August 2006. The proposed Section 4(3) of the Transitory Provisions states:
Under American jurisprudence, the effect of logrolling is to nullify the entire proposition and
not only the unrelated subject matter. Thus, in Fine v. Firestone,29 the Supreme Court of Florida Section 4(3). Senators whose term of office ends in 2010 shall be members of
declared: Parliament until noon of the thirtieth day of June 2010.

Combining multiple propositions into one proposal constitutes "logrolling," After 30 June 2010, not one of the present Senators will remain as member of Parliament if the
which, if our judicial responsibility is to mean anything, we cannot permit. The interim Parliament does not schedule elections for the regular Parliament by 30 June 2010.
very broadness of the proposed amendment amounts to logrolling because the However, there is no counterpart provision for the present members of the House of
electorate cannot know what it is voting on - the amendment's proponents' simplistic Representatives even if their term of office will all end on 30 June 2007, three years earlier than
explanation reveals only the tip of the iceberg. x x x x The ballot must give the that of half of the present Senators. Thus, all the present members of the House will remain
electorate fair notice of the proposed amendment being voted on. x x x x The ballot members of the interim Parliament after 30 June 2010.
The term of the incumbent President ends on 30 June 2010. Thereafter, the Prime Minister Article XVII of the Constitution speaks of three modes of amending the Constitution. The first
exercises all the powers of the President. If the interim Parliament does not schedule elections mode is through Congress upon three-fourths vote of all its Members. The second mode is
for the regular Parliament by 30 June 2010, the Prime Minister will come only from the present through a constitutional convention. The third mode is through a people's initiative.
members of the House of Representatives to the exclusion of the present Senators.
Section 1 of Article XVII, referring to the first and second modes, applies to "[A]ny amendment
The signature sheets do not explain this discrimination against the Senators. The 6.3 million to, or revision of, this Constitution." In contrast, Section 2 of Article XVII, referring to the third
people who signed the signature sheets could not have known that their signatures mode, applies only to "[A]mendments to this Constitution." This distinction was intentional as
would be used to discriminate against the Senators. They could not have known that their shown by the following deliberations of the Constitutional Commission:
signatures would be used to limit, after 30 June 2010, the interim Parliament's choice of
Prime Minister only to members of the existing House of Representatives.
MR. SUAREZ: Thank you, Madam President.

An initiative that gathers signatures from the people without first showing to the people the full
May we respectfully call the attention of the Members of the Commission that pursuant
text of the proposed amendments is most likely a deception, and can operate as a gigantic
to the mandate given to us last night, we submitted this afternoon a complete
fraud on the people. That is why the Constitution requires that an initiative must be "directly
Committee Report No. 7 which embodies the proposed provision governing the matter
proposed by the people x x x in a petition" - meaning that the people must sign on a petition
of initiative. This is now covered by Section 2 of the complete committee report. With
that contains the full text of the proposed amendments. On so vital an issue as amending the
the permission of the Members, may I quote Section 2:
nation's fundamental law, the writing of the text of the proposed amendments cannot be hidden
from the people under a general or special power of attorney to unnamed, faceless, and
unelected individuals. The people may, after five years from the date of the last plebiscite held, directly
propose amendments to this Constitution thru initiative upon petition of at least ten
percent of the registered voters.
The Constitution entrusts to the people the power to directly propose amendments to the
Constitution. This Court trusts the wisdom of the people even if the members of this Court do not
personally know the people who sign the petition. However, this trust emanates from a This completes the blanks appearing in the original Committee Report No. 7. This
fundamental assumption: the full text of the proposed amendment is first shown to the proposal was suggested on the theory that this matter of initiative, which came about
people before they sign the petition, not after they have signed the petition. because of the extraordinary developments this year, has to be separated from the
traditional modes of amending the Constitution as embodied in Section 1. The
committee members felt that this system of initiative should be limited to
In short, the Lambino Group's initiative is void and unconstitutional because it dismally fails to
amendments to the Constitution and should not extend to the revision of the
comply with the requirement of Section 2, Article XVII of the Constitution that the initiative must
entire Constitution, so we removed it from the operation of Section 1 of the
be "directly proposed by the people through initiative upon a petition."
proposed Article on Amendment or Revision. x x x x

2. The Initiative Violates Section 2, Article XVII of the Constitution Disallowing Revision
xxxx
through Initiatives

MS. AQUINO: [I] am seriously bothered by providing this process of initiative as a


A people's initiative to change the Constitution applies only to an amendment of the Constitution
separate section in the Article on Amendment. Would the sponsor be amenable to
and not to its revision. In contrast, Congress or a constitutional convention can propose both
accepting an amendment in terms of realigning Section 2 as another subparagraph (c)
amendments and revisions to the Constitution. Article XVII of the Constitution provides:
of Section 1, instead of setting it up as another separate section as if it were a self-
executing provision?
ARTICLE XVII
AMENDMENTS OR REVISIONS
MR. SUAREZ: We would be amenable except that, as we clarified a while ago, this
process of initiative is limited to the matter of amendment and should not
Sec. 1. Any amendment to, or revision of, this Constitution may be proposed by: expand into a revision which contemplates a total overhaul of the Constitution.
That was the sense that was conveyed by the Committee.
(1) The Congress, upon a vote of three-fourths of all its Members, or
MS. AQUINO: In other words, the Committee was attempting to distinguish the
coverage of modes (a) and (b) in Section 1 to include the process of revision;
(2) A constitutional convention.
whereas, the process of initiation to amend, which is given to the public, would
only apply to amendments?
Sec. 2. Amendments to this Constitution may likewise be directly proposed by the
people through initiative x x x. (Emphasis supplied)
MR. SUAREZ: That is right. Those were the terms envisioned in the Committee.

MS. AQUINO: I thank the sponsor; and thank you, Madam President.
xxxx To call it an amendment is a misnomer.

MR. MAAMBONG: My first question: Commissioner Davide's proposed Whether it be a revision or a new constitution, it is not such a measure as can be
amendment on line 1 refers to "amendments." Does it not cover the word submitted to the people through the initiative. If a revision, it is subject to the
"revision" as defined by Commissioner Padilla when he made the distinction requirements of Article XVII, Section 2(1); if a new constitution, it can only be
between the words "amendments" and "revision"? proposed at a convention called in the manner provided in Article XVII, Section 1. x x x
x
MR. DAVIDE: No, it does not, because "amendments" and "revision" should be
covered by Section 1. So insofar as initiative is concerned, it can only relate to Similarly, in this jurisdiction there can be no dispute that a people's initiative can only propose
"amendments" not "revision." amendments to the Constitution since the Constitution itself limits initiatives to amendments.
There can be no deviation from the constitutionally prescribed modes of revising the
Constitution. A popular clamor, even one backed by 6.3 million signatures, cannot justify a
MR. MAAMBONG: Thank you.31 (Emphasis supplied)
deviation from the specific modes prescribed in the Constitution itself.

There can be no mistake about it. The framers of the Constitution intended, and wrote, a clear
As the Supreme Court of Oklahoma ruled in In re Initiative Petition No. 364:34
distinction between "amendment" and "revision" of the Constitution. The framers intended, and
wrote, that only Congress or a constitutional convention may propose revisions to the
Constitution. The framers intended, and wrote, that a people's initiative may propose only It is a fundamental principle that a constitution can only be revised or amended
amendments to the Constitution. Where the intent and language of the Constitution clearly in the manner prescribed by the instrument itself, and that any attempt to revise
withhold from the people the power to propose revisions to the Constitution, the people cannot a constitution in a manner other than the one provided in the instrument is
propose revisions even as they are empowered to propose amendments. almost invariably treated as extra-constitutional and revolutionary. x x x x "While
it is universally conceded that the people are sovereign and that they have power to
adopt a constitution and to change their own work at will, they must, in doing so, act in
This has been the consistent ruling of state supreme courts in the United States. Thus,
an orderly manner and according to the settled principles of constitutional law. And
in McFadden v. Jordan,32the Supreme Court of California ruled:
where the people, in adopting a constitution, have prescribed the method by which the
people may alter or amend it, an attempt to change the fundamental law in violation of
The initiative power reserved by the people by amendment to the Constitution x the self-imposed restrictions, is unconstitutional." x x x x (Emphasis supplied)
x x applies only to the proposing and the adopting or rejecting of 'laws and
amendments to the Constitution' and does not purport to extend to a
This Court, whose members are sworn to defend and protect the Constitution, cannot shirk from
constitutional revision. x x x x It is thus clear that a revision of the Constitution may
its solemn oath and duty to insure compliance with the clear command of the Constitution ― that
be accomplished only through ratification by the people of a revised constitution
a people's initiative may only amend, never revise, the Constitution.
proposed by a convention called for that purpose as outlined hereinabove.
Consequently if the scope of the proposed initiative measure (hereinafter termed 'the
measure') now before us is so broad that if such measure became law a substantial The question is, does the Lambino Group's initiative constitute an amendment or revision of the
revision of our present state Constitution would be effected, then the measure may not Constitution? If the Lambino Group's initiative constitutes a revision, then the present petition
properly be submitted to the electorate until and unless it is first agreed upon by a should be dismissed for being outside the scope of Section 2, Article XVII of the Constitution.
constitutional convention, and the writ sought by petitioner should issue. x x x x
(Emphasis supplied)
Courts have long recognized the distinction between an amendment and a revision of a
constitution. One of the earliest cases that recognized the distinction described the fundamental
Likewise, the Supreme Court of Oregon ruled in Holmes v. Appling:33 difference in this manner:

It is well established that when a constitution specifies the manner in which it may be [T]he very term "constitution" implies an instrument of a permanent and abiding
amended or revised, it can be altered by those who favor amendments, revision, or nature, and the provisions contained therein for its revision indicate the will of
other change only through the use of one of the specified means. The constitution the people that the underlying principles upon which it rests, as well as the
itself recognizes that there is a difference between an amendment and a revision; and substantial entirety of the instrument, shall be of a like permanent and abiding
it is obvious from an examination of the measure here in question that it is not an nature. On the other hand, the significance of the term "amendment" implies such an
amendment as that term is generally understood and as it is used in Article IV, Section addition or change within the lines of the original instrument as will effect an
1. The document appears to be based in large part on the revision of the constitution improvement, or better carry out the purpose for which it was framed.35 (Emphasis
drafted by the 'Commission for Constitutional Revision' authorized by the 1961 supplied)
Legislative Assembly, x x x and submitted to the 1963 Legislative Assembly. It failed
to receive in the Assembly the two-third's majority vote of both houses required by
Revision broadly implies a change that alters a basic principle in the constitution, like altering
Article XVII, Section 2, and hence failed of adoption, x x x.
the principle of separation of powers or the system of checks-and-balances. There is also
revision if the change alters the substantial entirety of the constitution, as when the change
While differing from that document in material respects, the measure sponsored by the affects substantial provisions of the constitution. On the other hand, amendment broadly
plaintiffs is, nevertheless, a thorough overhauling of the present constitution x x x. refers to a change that adds, reduces, or deletes without altering the basic principle
involved. Revision generally affects several provisions of the constitution, while amendment system be because of its effect on other important provisions of the
generally affects only the specific provision being amended. Constitution.41 (Emphasis supplied)

In California where the initiative clause allows amendments but not revisions to the constitution In Adams v. Gunter,42 an initiative petition proposed the amendment of the Florida State
just like in our Constitution, courts have developed a two-part test: the quantitative test and the constitution to shift from a bicameral to a unicameral legislature. The issue turned on
qualitative test. The quantitative test asks whether the proposed change is "so extensive in its whether the initiative "was defective and unauthorized where [the] proposed amendment would x
provisions as to change directly the 'substantial entirety' of the constitution by the deletion or x x affect several other provisions of [the] Constitution." The Supreme Court of Florida, striking
alteration of numerous existing provisions."36 The court examines only the number of provisions down the initiative as outside the scope of the initiative clause, ruled as follows:
affected and does not consider the degree of the change.
The proposal here to amend Section 1 of Article III of the 1968 Constitution to provide
The qualitative test inquires into the qualitative effects of the proposed change in the for a Unicameral Legislature affects not only many other provisions of the
constitution. The main inquiry is whether the change will "accomplish such far reaching changes Constitution but provides for a change in the form of the legislative branch of
in the nature of our basic governmental plan as to amount to a revision."37 Whether there is an government, which has been in existence in the United States Congress and in all of
alteration in the structure of government is a proper subject of inquiry. Thus, "a change in the the states of the nation, except one, since the earliest days. It would be difficult to
nature of [the] basic governmental plan" includes "change in its fundamental framework or the visualize a more revolutionary change. The concept of a House and a Senate is
fundamental powers of its Branches."38 A change in the nature of the basic governmental plan basic in the American form of government. It would not only radically change the
also includes changes that "jeopardize the traditional form of government and the system of whole pattern of government in this state and tear apart the whole fabric of the
check and balances."39 Constitution, but would even affect the physical facilities necessary to carry on
government.
Under both the quantitative and qualitative tests, the Lambino Group's initiative is a revision and
not merely an amendment. Quantitatively, the Lambino Group's proposed changes overhaul two xxxx
articles - Article VI on the Legislature and Article VII on the Executive - affecting a total of 105
provisions in the entire Constitution.40Qualitatively, the proposed changes alter substantially the
We conclude with the observation that if such proposed amendment were adopted by
basic plan of government, from presidential to parliamentary, and from a bicameral to a
the people at the General Election and if the Legislature at its next session should fail
unicameral legislature.
to submit further amendments to revise and clarify the numerous inconsistencies and
conflicts which would result, or if after submission of appropriate amendments the
A change in the structure of government is a revision of the Constitution, as when the three great people should refuse to adopt them, simple chaos would prevail in the government of
co-equal branches of government in the present Constitution are reduced into two. This alters this State. The same result would obtain from an amendment, for instance, of Section
the separation of powers in the Constitution. A shift from the present Bicameral-Presidential 1 of Article V, to provide for only a Supreme Court and Circuit Courts-and there could
system to a Unicameral-Parliamentary system is a revision of the Constitution. Merging the be other examples too numerous to detail. These examples point unerringly to the
legislative and executive branches is a radical change in the structure of government. answer.

The abolition alone of the Office of the President as the locus of Executive Power alters the The purpose of the long and arduous work of the hundreds of men and women and
separation of powers and thus constitutes a revision of the Constitution. Likewise, the abolition many sessions of the Legislature in bringing about the Constitution of 1968 was to
alone of one chamber of Congress alters the system of checks-and-balances within the eliminate inconsistencies and conflicts and to give the State a workable, accordant,
legislature and constitutes a revision of the Constitution. homogenous and up-to-date document. All of this could disappear very quickly if we
were to hold that it could be amended in the manner proposed in the initiative petition
here.43(Emphasis supplied)
By any legal test and under any jurisdiction, a shift from a Bicameral-Presidential to a
Unicameral-Parliamentary system, involving the abolition of the Office of the President and the
abolition of one chamber of Congress, is beyond doubt a revision, not a mere amendment. On The rationale of the Adams decision applies with greater force to the present petition. The
the face alone of the Lambino Group's proposed changes, it is readily apparent that the changes Lambino Group's initiative not only seeks a shift from a bicameral to a unicameral legislature, it
will radically alter the framework of government as set forth in the Constitution. Father also seeks to merge the executive and legislative departments. The initiative in Adams did not
Joaquin Bernas, S.J., a leading member of the Constitutional Commission, writes: even touch the executive department.

An amendment envisages an alteration of one or a few specific and separable provisions. The In Adams, the Supreme Court of Florida enumerated 18 sections of the Florida Constitution that
guiding original intention of an amendment is to improve specific parts or to add new provisions would be affected by the shift from a bicameral to a unicameral legislature. In the Lambino
deemed necessary to meet new conditions or to suppress specific portions that may have Group's present initiative, no less than 105 provisions of the Constitution would be
become obsolete or that are judged to be dangerous. In revision, however, the guiding original affected based on the count of Associate Justice Romeo J. Callejo, Sr. 44 There is no doubt that
intention and plan contemplates a re-examination of the entire document, or of provisions of the the Lambino Group's present initiative seeks far more radical changes in the structure of
document which have over-all implications for the entire document, to determine how and to government than the initiative in Adams.
what extent they should be altered. Thus, for instance a switch from the presidential system
to a parliamentary system would be a revision because of its over-all impact on the entire
The Lambino Group theorizes that the difference between "amendment" and "revision" is
constitutional structure. So would a switch from a bicameral system to a unicameral
only one of procedure, not of substance. The Lambino Group posits that when a deliberative
body drafts and proposes changes to the Constitution, substantive changes are called Plaintiffs argue that the proposed ballot measure constitutes a wholesale change to
"revisions" because members of the deliberative body work full-time on the changes. the constitution that cannot be enacted through the initiative process. They assert that
However, the same substantive changes, when proposed through an initiative, are called the distinction between amendment and revision is determined by reviewing the scope
"amendments" because the changes are made by ordinary people who do not make an and subject matter of the proposed enactment, and that revisions are not limited to "a
"occupation, profession, or vocation" out of such endeavor. formal overhauling of the constitution." They argue that this ballot measure proposes
far reaching changes outside the lines of the original instrument, including profound
impacts on existing fundamental rights and radical restructuring of the government's
Thus, the Lambino Group makes the following exposition of their theory in their Memorandum:
relationship with a defined group of citizens. Plaintiffs assert that, because the
proposed ballot measure "will refashion the most basic principles of Oregon
99. With this distinction in mind, we note that the constitutional provisions expressly constitutional law," the trial court correctly held that it violated Article XVII, section 2,
provide for both "amendment" and "revision" when it speaks of legislators and and cannot appear on the ballot without the prior approval of the legislature.
constitutional delegates, while the same provisions expressly provide only for
"amendment" when it speaks of the people. It would seem that the apparent distinction
We first address Mabon's argument that Article XVII, section 2(1), does not prohibit
is based on the actual experience of the people, that on one hand the common people
revisions instituted by initiative. In Holmes v. Appling, x x x, the Supreme Court
in general are not expected to work full-time on the matter of correcting the
concluded that a revision of the constitution may not be accomplished by initiative,
constitution because that is not their occupation, profession or vocation; while on the
because of the provisions of Article XVII, section 2. After reviewing Article XVII,
other hand, the legislators and constitutional convention delegates are expected to
section1, relating to proposed amendments, the court said:
work full-time on the same matter because that is their occupation, profession or
vocation. Thus, the difference between the words "revision" and "amendment"
pertain only to the process or procedure of coming up with the corrections, for "From the foregoing it appears that Article IV, Section 1, authorizes the use of the
purposes of interpreting the constitutional provisions. initiative as a means of amending the Oregon Constitution, but it contains no similar
sanction for its use as a means of revising the constitution." x x x x
100. Stated otherwise, the difference between "amendment" and "revision"
cannot reasonably be in the substance or extent of the correction. x x x x It then reviewed Article XVII, section 2, relating to revisions, and said: "It is the only
(Underlining in the original; boldfacing supplied) section of the constitution which provides the means for constitutional revision and it
excludes the idea that an individual, through the initiative, may place such a measure
before the electorate." x x x x
The Lambino Group in effect argues that if Congress or a constitutional convention had drafted
the same proposed changes that the Lambino Group wrote in the present initiative, the changes
would constitute a revision of the Constitution. Thus, the Lambino Group concedes that the Accordingly, we reject Mabon's argument that Article XVII, section 2, does not
proposed changes in the present initiative constitute a revision if Congress or a apply to constitutional revisions proposed by initiative. (Emphasis supplied)
constitutional convention had drafted the changes. However, since the Lambino Group as
private individuals drafted the proposed changes, the changes are merely amendments to the
Similarly, this Court must reject the Lambino Group's theory which negates the express intent of
Constitution. The Lambino Group trivializes the serious matter of changing the fundamental law
the framers and the plain language of the Constitution.
of the land.

We can visualize amendments and revisions as a spectrum, at one end green for amendments
The express intent of the framers and the plain language of the Constitution contradict the
and at the other end red for revisions. Towards the middle of the spectrum, colors fuse and
Lambino Group's theory. Where the intent of the framers and the language of the Constitution
difficulties arise in determining whether there is an amendment or revision. The present initiative
are clear and plainly stated, courts do not deviate from such categorical intent and
is indisputably located at the far end of the red spectrum where revision begins. The present
language.45 Any theory espousing a construction contrary to such intent and language deserves
initiative seeks a radical overhaul of the existing separation of powers among the three co-equal
scant consideration. More so, if such theory wreaks havoc by creating inconsistencies in the
departments of government, requiring far-reaching amendments in several sections and articles
form of government established in the Constitution. Such a theory, devoid of any jurisprudential
of the Constitution.
mooring and inviting inconsistencies in the Constitution, only exposes the flimsiness of the
Lambino Group's position. Any theory advocating that a proposed change involving a radical
structural change in government does not constitute a revision justly deserves rejection. Where the proposed change applies only to a specific provision of the Constitution without
affecting any other section or article, the change may generally be considered an amendment
and not a revision. For example, a change reducing the voting age from 18 years to 15 years 47 is
The Lambino Group simply recycles a theory that initiative proponents in American jurisdictions
an amendment and not a revision. Similarly, a change reducing Filipino ownership of mass
have attempted to advance without any success. In Lowe v. Keisling,46 the Supreme Court of
media companies from 100 percent to 60 percent is an amendment and not a revision. 48 Also, a
Oregon rejected this theory, thus:
change requiring a college degree as an additional qualification for election to the Presidency is
an amendment and not a revision.49
Mabon argues that Article XVII, section 2, does not apply to changes to the
constitution proposed by initiative. His theory is that Article XVII, section 2 merely
The changes in these examples do not entail any modification of sections or articles of the
provides a procedure by which the legislature can propose a revision of the
Constitution other than the specific provision being amended. These changes do not also affect
constitution, but it does not affect proposed revisions initiated by the people.
the structure of government or the system of checks-and-balances among or within the three
branches. These three examples are located at the far green end of the spectrum, opposite the Now, what "unicameral parliamentary form of government" do the Lambino Group's
far red end where the revision sought by the present petition is located. proposed changes refer to ― the Bangladeshi, Singaporean, Israeli, or New Zealand models,
which are among the few countries with unicameral parliaments? The proposed changes
could not possibly refer to the traditional and well-known parliamentary forms of government ―
However, there can be no fixed rule on whether a change is an amendment or a revision. A
the British, French, Spanish, German, Italian, Canadian, Australian, or Malaysian models, which
change in a single word of one sentence of the Constitution may be a revision and not an
have all bicameral parliaments. Did the people who signed the signature sheets realize that
amendment. For example, the substitution of the word "republican" with "monarchic" or
they were adopting the Bangladeshi, Singaporean, Israeli, or New Zealand parliamentary form of
"theocratic" in Section 1, Article II50 of the Constitution radically overhauls the entire structure of
government?
government and the fundamental ideological basis of the Constitution. Thus, each specific
change will have to be examined case-by-case, depending on how it affects other provisions, as
well as how it affects the structure of government, the carefully crafted system of checks-and- This drives home the point that the people's initiative is not meant for revisions of the
balances, and the underlying ideological basis of the existing Constitution. Constitution but only for amendments. A shift from the present Bicameral-Presidential to a
Unicameral-Parliamentary system requires harmonizing several provisions in many articles of
the Constitution. Revision of the Constitution through a people's initiative will only result in gross
Since a revision of a constitution affects basic principles, or several provisions of a constitution,
absurdities in the Constitution.
a deliberative body with recorded proceedings is best suited to undertake a revision. A
revision requires harmonizing not only several provisions, but also the altered principles with
those that remain unaltered. Thus, constitutions normally authorize deliberative bodies like In sum, there is no doubt whatsoever that the Lambino Group's initiative is a revision and not an
constituent assemblies or constitutional conventions to undertake revisions. On the other hand, amendment. Thus, the present initiative is void and unconstitutional because it violates Section
constitutions allow people's initiatives, which do not have fixed and identifiable deliberative 2, Article XVII of the Constitution limiting the scope of a people's initiative to "[A]mendments to
bodies or recorded proceedings, to undertake only amendments and not revisions. this Constitution."

In the present initiative, the Lambino Group's proposed Section 2 of the Transitory Provisions 3. A Revisit of Santiago v. COMELEC is Not Necessary
states:
The present petition warrants dismissal for failure to comply with the basic requirements of
Section 2. Upon the expiration of the term of the incumbent President and Vice Section 2, Article XVII of the Constitution on the conduct and scope of a people's initiative to
President, with the exception of Sections 1, 2, 3, 4, 5, 6 and 7 of Article VI of the 1987 amend the Constitution. There is no need to revisit this Court's ruling in Santiago declaring RA
Constitution which shall hereby be amended and Sections 18 and 24 which shall be 6735 "incomplete, inadequate or wanting in essential terms and conditions" to cover the system
deleted, all other Sections of Article VI are hereby retained and renumbered of initiative to amend the Constitution. An affirmation or reversal of Santiago will not change the
sequentially as Section 2, ad seriatim up to 26, unless they are inconsistent with outcome of the present petition. Thus, this Court must decline to revisit Santiago which
the Parliamentary system of government, in which case, they shall be amended effectively ruled that RA 6735 does not comply with the requirements of the Constitution to
to conform with a unicameral parliamentary form of government; x x x x implement the initiative clause on amendments to the Constitution.
(Emphasis supplied)
This Court must avoid revisiting a ruling involving the constitutionality of a statute if the case
The basic rule in statutory construction is that if a later law is irreconcilably inconsistent with a before the Court can be resolved on some other grounds. Such avoidance is a logical
prior law, the later law prevails. This rule also applies to construction of constitutions. However, consequence of the well-settled doctrine that courts will not pass upon the constitutionality of a
the Lambino Group's draft of Section 2 of the Transitory Provisions turns on its head this rule of statute if the case can be resolved on some other grounds. 51
construction by stating that in case of such irreconcilable inconsistency, the earlier provision
"shall be amended to conform with a unicameral parliamentary form of government." The effect
Nevertheless, even assuming that RA 6735 is valid to implement the constitutional provision on
is to freeze the two irreconcilable provisions until the earlier one "shall be amended," which
initiatives to amend the Constitution, this will not change the result here because the present
requires a future separate constitutional amendment.
petition violates Section 2, Article XVII of the Constitution. To be a valid initiative, the present
initiative must first comply with Section 2, Article XVII of the Constitution even before complying
Realizing the absurdity of the need for such an amendment, petitioner Atty. Lambino readily with RA 6735.
conceded during the oral arguments that the requirement of a future amendment is a
"surplusage." In short, Atty. Lambino wants to reinstate the rule of statutory construction so that
Even then, the present initiative violates Section 5(b) of RA 6735 which requires that the
the later provision automatically prevails in case of irreconcilable inconsistency. However, it is
"petition for an initiative on the 1987 Constitution must have at least twelve per centum (12%) of
not as simple as that.
the total number of registered voters as signatories." Section 5(b) of RA 6735 requires that the
people must sign the "petition x x x as signatories."
The irreconcilable inconsistency envisioned in the proposed Section 2 of the Transitory
Provisions is not between a provision in Article VI of the 1987 Constitution and a provision in the
The 6.3 million signatories did not sign the petition of 25 August 2006 or the amended petition of
proposed changes. The inconsistency is between a provision in Article VI of the 1987
30 August 2006 filed with the COMELEC. Only Atty. Lambino, Atty. Demosthenes B. Donato,
Constitution and the "Parliamentary system of government," and the inconsistency shall be
and Atty. Alberto C. Agra signed the petition and amended petition as counsels for "Raul
resolved in favor of a "unicameral parliamentary form of government."
L. Lambino and Erico B. Aumentado, Petitioners." In the COMELEC, the Lambino Group,
claiming to act "together with" the 6.3 million signatories, merely attached the signature sheets to
the petition and amended petition. Thus, the petition and amended petition filed with the
COMELEC did not even comply with the basic requirement of RA 6735 that the Lambino Group Incantations of "people's voice," "people's sovereign will," or "let the people decide" cannot
claims as valid. override the specific modes of changing the Constitution as prescribed in the Constitution itself.
Otherwise, the Constitution ― the people's fundamental covenant that provides enduring
stability to our society ― becomes easily susceptible to manipulative changes by political groups
The Lambino Group's logrolling initiative also violates Section 10(a) of RA 6735 stating, "No
gathering signatures through false promises. Then, the Constitution ceases to be the bedrock of
petition embracing more than one (1) subject shall be submitted to the electorate; x x x."
the nation's stability.
The proposed Section 4(4) of the Transitory Provisions, mandating the interim Parliament to
propose further amendments or revisions to the Constitution, is a subject matter totally unrelated
to the shift in the form of government. Since the present initiative embraces more than one The Lambino Group claims that their initiative is the "people's voice." However, the Lambino
subject matter, RA 6735 prohibits submission of the initiative petition to the electorate. Thus, Group unabashedly states in ULAP Resolution No. 2006-02, in the verification of their petition
even if RA 6735 is valid, the Lambino Group's initiative will still fail. with the COMELEC, that "ULAP maintains its unqualified support to the agenda of Her
Excellency President Gloria Macapagal-Arroyo for constitutional reforms." The Lambino Group
thus admits that their "people's" initiative is an "unqualified support to the agenda" of the
4. The COMELEC Did Not Commit Grave Abuse of Discretion in Dismissing the Lambino
incumbent President to change the Constitution. This forewarns the Court to be wary of
Group's Initiative
incantations of "people's voice" or "sovereign will" in the present initiative.

In dismissing the Lambino Group's initiative petition, the COMELEC en banc merely followed
This Court cannot betray its primordial duty to defend and protect the Constitution. The
this Court's ruling in Santiago and People's Initiative for Reform, Modernization and Action
Constitution, which embodies the people's sovereign will, is the bible of this Court. This Court
(PIRMA) v. COMELEC.52 For following this Court's ruling, no grave abuse of discretion is
exists to defend and protect the Constitution. To allow this constitutionally infirm initiative,
attributable to the COMELEC. On this ground alone, the present petition warrants outright
propelled by deceptively gathered signatures, to alter basic principles in the Constitution is to
dismissal. Thus, this Court should reiterate its unanimous ruling in PIRMA:
allow a desecration of the Constitution. To allow such alteration and desecration is to lose this
Court's raison d'etre.
The Court ruled, first, by a unanimous vote, that no grave abuse of discretion could be
attributed to the public respondent COMELEC in dismissing the petition filed by
WHEREFORE, we DISMISS the petition in G.R. No. 174153.
PIRMA therein, it appearing that it only complied with the dispositions in the Decisions
of this Court in G.R. No. 127325, promulgated on March 19, 1997, and its Resolution
of June 10, 1997. SO ORDERED.

5. Conclusion

The Constitution, as the fundamental law of the land, deserves the utmost respect and
obedience of all the citizens of this nation. No one can trivialize the Constitution by cavalierly
amending or revising it in blatant violation of the clearly specified modes of amendment and
revision laid down in the Constitution itself.

To allow such change in the fundamental law is to set adrift the Constitution in unchartered
waters, to be tossed and turned by every dominant political group of the day. If this Court allows
today a cavalier change in the Constitution outside the constitutionally prescribed modes,
tomorrow the new dominant political group that comes will demand its own set of changes in the
same cavalier and unconstitutional fashion. A revolving-door constitution does not augur well for
the rule of law in this country.

An overwhelming majority − 16,622,111 voters comprising 76.3 percent of the total votes
cast53 − approved our Constitution in a national plebiscite held on 11 February 1987. That
approval is the unmistakable voice of the people, the full expression of the people's
sovereign will. That approval included the prescribed modes for amending or revising the
Constitution.

No amount of signatures, not even the 6,327,952 million signatures gathered by the Lambino
Group, can change our Constitution contrary to the specific modes that the people, in their
sovereign capacity, prescribed when they ratified the Constitution. The alternative is an extra-
constitutional change, which means subverting the people's sovereign will and discarding
the Constitution. This is one act the Court cannot and should never do. As the ultimate
guardian of the Constitution, this Court is sworn to perform its solemn duty to defend and protect
the Constitution, which embodies the real sovereign will of the people.
G.R. No. 122156 February 3, 1997 The Highest Bidder will be declared the Winning Bidder/Strategic Partner
after the following conditions are met:
MANILA PRINCE HOTEL petitioner,
vs. a. Execution of the necessary contracts with GSIS/MHC
GOVERNMENT SERVICE INSURANCE SYSTEM, MANILA HOTEL CORPORATION, not later than October 23, 1995 (reset to November 3,
COMMITTEE ON PRIVATIZATION and OFFICE OF THE GOVERNMENT CORPORATE 1995); and
COUNSEL, respondents.
b. Requisite approvals from the GSIS/MHC and COP
BELLOSILLO, J.: (Committee on Privatization)/OGCC (Office of the
Government Corporate Counsel) are obtained.3
The FiIipino First Policy enshrined in the 1987 Constitution, i.e., in the grant of rights, privileges,
and concessions covering the national economy and patrimony, the State shall give preference Pending the declaration of Renong Berhad as the winning bidder/strategic partner and the
to qualified Filipinos,1 is in oked by petitioner in its bid to acquire 51% of the shares of the Manila execution of the necessary contracts, petitioner in a letter to respondent GSIS dated 28
Hotel Corporation (MHC) which owns the historic Manila Hotel. Opposing, respondents maintain September 1995 matched the bid price of P44.00 per share tendered by Renong Berhad. 4 In a
that the provision is not self-executing but requires an implementing legislation for its subsequent letter dated 10 October 1995 petitioner sent a manager's check issued by Philtrust
enforcement. Corollarily, they ask whether the 51% shares form part of the national economy Bank for Thirty-three Million Pesos (P33.000.000.00) as Bid Security to match the bid of the
and patrimony covered by the protective mantle of the Constitution. Malaysian Group, Messrs. Renong Berhad . . .5 which respondent GSIS refused to accept.

The controversy arose when respondent Government Service Insurance System (GSIS), On 17 October 1995, perhaps apprehensive that respondent GSIS has disregarded the tender
pursuant to the privatization program of the Philippine Government under Proclamation No. 50 of the matching bid and that the sale of 51% of the MHC may be hastened by respondent GSIS
dated 8 December 1986, decided to sell through public bidding 30% to 51% of the issued and and consummated with Renong Berhad, petitioner came to this Court on prohibition
outstanding shares of respondent MHC. The winning bidder, or the eventual "strategic and mandamus. On 18 October 1995 the Court issued a temporary restraining order enjoining
partner," is to provide management expertise and/or an international marketing/reservation respondents from perfecting and consummating the sale to the Malaysian firm.
system, and financial support to strengthen the profitability and performance of the Manila
Hotel.2 In a close bidding held on 18 September 1995 only two (2) bidders participated:
On 10 September 1996 the instant case was accepted by the Court En Banc after it was referred
petitioner Manila Prince Hotel Corporation, a Filipino corporation, which offered to buy 51% of
to it by the First Division. The case was then set for oral arguments with former Chief Justice
the MHC or 15,300,000 shares at P41.58 per share, and Renong Berhad, a Malaysian firm, with
Enrique M. Fernando and Fr. Joaquin G. Bernas, S.J., as amici curiae.
ITT-Sheraton as its hotel operator, which bid for the same number of shares at P44.00 per
share, or P2.42 more than the bid of petitioner.
In the main, petitioner invokes Sec. 10, second par., Art. XII, of the 1987 Constitution and
submits that the Manila Hotel has been identified with the Filipino nation and has practically
Pertinent provisions of the bidding rules prepared by respondent GSIS state —
become a historical monument which reflects the vibrancy of Philippine heritage and culture. It is
a proud legacy of an earlier generation of Filipinos who believed in the nobility and sacredness
I. EXECUTION OF THE NECESSARY CONTRACTS of independence and its power and capacity to release the full potential of the Filipino people. To
WITH GSIS/MHC — all intents and purposes, it has become a part of the national patrimony.6 Petitioner also argues
that since 51% of the shares of the MHC carries with it the ownership of the business of the
hotel which is owned by respondent GSIS, a government-owned and controlled corporation, the
1. The Highest Bidder must comply with the conditions set forth below by
hotel business of respondent GSIS being a part of the tourism industry is unquestionably a part
October 23, 1995 (reset to November 3, 1995) or the Highest Bidder will
of the national economy. Thus, any transaction involving 51% of the shares of stock of the MHC
lose the right to purchase the Block of Shares and GSIS will instead offer
is clearly covered by the term national economy, to which Sec. 10, second par., Art. XII, 1987
the Block of Shares to the other Qualified Bidders:
Constitution, applies.7

a. The Highest Bidder must negotiate and execute with


It is also the thesis of petitioner that since Manila Hotel is part of the national patrimony and its
the GSIS/MHC the Management Contract, International
business also unquestionably part of the national economy petitioner should be preferred after it
Marketing/Reservation System Contract or other type of
has matched the bid offer of the Malaysian firm. For the bidding rules mandate that if for any
contract specified by the Highest Bidder in its strategic
reason, the Highest Bidder cannot be awarded the Block of Shares, GSIS may offer this to the
plan for the Manila Hotel. . . .
other Qualified Bidders that have validly submitted bids provided that these Qualified Bidders are
willing to match the highest bid in terms of price per share.8
b. The Highest Bidder must execute the Stock
Purchase and Sale Agreement with GSIS . . . .
Respondents except. They maintain that: First, Sec. 10, second par., Art. XII, of the 1987
Constitution is merely a statement of principle and policy since it is not a self-executing provision
K. DECLARATION OF THE WINNING and requires implementing legislation(s) . . . Thus, for the said provision to Operate, there must
BIDDER/STRATEGIC PARTNER — be existing laws "to lay down conditions under which business may be done."9
Second, granting that this provision is self-executing, Manila Hotel does not fall under the term Constitution, is usually not self-executing. But a provision which is complete in itself and
national patrimony which only refers to lands of the public domain, waters, minerals, coal, becomes operative without the aid of supplementary or enabling legislation, or that which
petroleum and other mineral oils, all forces of potential energy, fisheries, forests or timber, supplies sufficient rule by means of which the right it grants may be enjoyed or protected, is self-
wildlife, flora and fauna and all marine wealth in its territorial sea, and exclusive marine zone as executing. Thus a constitutional provision is self-executing if the nature and extent of the right
cited in the first and second paragraphs of Sec. 2, Art. XII, 1987 Constitution. According to conferred and the liability imposed are fixed by the constitution itself, so that they can be
respondents, while petitioner speaks of the guests who have slept in the hotel and the events determined by an examination and construction of its terms, and there is no language indicating
that have transpired therein which make the hotel historic, these alone do not make the hotel fall that the subject is referred to the legislature for action. 13
under the patrimony of the nation. What is more, the mandate of the Constitution is addressed to
the State, not to respondent GSIS which possesses a personality of its own separate and
As against constitutions of the past, modern constitutions have been generally drafted upon a
distinct from the Philippines as a State.
different principle and have often become in effect extensive codes of laws intended to operate
directly upon the people in a manner similar to that of statutory enactments, and the function of
Third, granting that the Manila Hotel forms part of the national patrimony, the constitutional constitutional conventions has evolved into one more like that of a legislative body. Hence,
provision invoked is still inapplicable since what is being sold is only 51% of the outstanding unless it is expressly provided that a legislative act is necessary to enforce a constitutional
shares of the corporation, not the hotel building nor the land upon which the building stands. mandate, the presumption now is that all provisions of the constitution are self-executing If the
Certainly, 51% of the equity of the MHC cannot be considered part of the national patrimony. constitutional provisions are treated as requiring legislation instead of self-executing, the
Moreover, if the disposition of the shares of the MHC is really contrary to the Constitution, legislature would have the power to ignore and practically nullify the mandate of the fundamental
petitioner should have questioned it right from the beginning and not after it had lost in the law.14 This can be cataclysmic. That is why the prevailing view is, as it has always been, that —
bidding.
. . . in case of doubt, the Constitution should be considered self-executing
Fourth, the reliance by petitioner on par. V., subpar. J. 1., of the bidding rules which provides rather than non-self-executing . . . . Unless the contrary is clearly intended,
that if for any reason, the Highest Bidder cannot be awarded the Block of Shares, GSIS may the provisions of the Constitution should be considered self-executing, as a
offer this to the other Qualified Bidders that have validly submitted bids provided that these contrary rule would give the legislature discretion to determine when, or
Qualified Bidders are willing to match the highest bid in terms of price per share, is misplaced. whether, they shall be effective. These provisions would be subordinated to
Respondents postulate that the privilege of submitting a matching bid has not yet arisen since it the will of the lawmaking body, which could make them entirely meaningless
only takes place if for any reason, the Highest Bidder cannot be awarded the Block of Shares. by simply refusing to pass the needed implementing statute. 15
Thus the submission by petitioner of a matching bid is premature since Renong Berhad could
still very well be awarded the block of shares and the condition giving rise to the exercise of the
Respondents argue that Sec. 10, second par., Art. XII, of the 1987 Constitution is clearly not
privilege to submit a matching bid had not yet taken place.
self-executing, as they quote from discussions on the floor of the 1986 Constitutional
Commission —
Finally, the prayer for prohibition grounded on grave abuse of discretion should fail since
respondent GSIS did not exercise its discretion in a capricious, whimsical manner, and if ever it
MR. RODRIGO. Madam President, I am asking this
did abuse its discretion it was not so patent and gross as to amount to an evasion of a positive
question as the Chairman of the Committee on Style. If
duty or a virtual refusal to perform a duty enjoined by law. Similarly, the petition
the wording of "PREFERENCE" is given to QUALIFIED
for mandamus should fail as petitioner has no clear legal right to what it demands and
FILIPINOS," can it be understood as a preference to
respondents do not have an imperative duty to perform the act required of them by petitioner.
qualified Filipinos vis-a-vis Filipinos who are not
qualified. So, why do we not make it clear? To qualified
We now resolve. A constitution is a system of fundamental laws for the governance and Filipinos as against aliens?
administration of a nation. It is supreme, imperious, absolute and unalterable except by the
authority from which it emanates. It has been defined as the fundamental and paramount law of
THE PRESIDENT. What is the question of
the nation. 10 It prescribes the permanent framework of a system of government, assigns to the
Commissioner Rodrigo? Is it to remove the word
different departments their respective powers and duties, and establishes certain fixed principles
"QUALIFIED?".
on which government is founded. The fundamental conception in other words is that it is a
supreme law to which all other laws must conform and in accordance with which all private rights
must be determined and all public authority administered. 11 Under the doctrine of constitutional MR. RODRIGO. No, no, but say definitely "TO
supremacy, if a law or contract violates any norm of the constitution that law or contract whether QUALIFIED FILIPINOS" as against whom? As against
promulgated by the legislative or by the executive branch or entered into by private persons for aliens or over aliens?
private purposes is null and void and without any force and effect. Thus, since the Constitution is
the fundamental, paramount and supreme law of the nation, it is deemed written in every statute
MR. NOLLEDO. Madam President, I think that is
and contract.
understood. We use the word "QUALIFIED" because
the existing laws or prospective laws will always lay
Admittedly, some constitutions are merely declarations of policies and principles. Their down conditions under which business may be
provisions command the legislature to enact laws and carry out the purposes of the framers who done. For example, qualifications on the setting up of
merely establish an outline of government providing for the different departments of the other financial structures, et cetera (emphasis supplied
governmental machinery and securing certain fundamental and inalienable rights of citizens. 12 A by respondents)
provision which lays down a general principle, such as those found in Art. II of the 1987
MR. RODRIGO. It is just a matter of style. require any legislation to put it in operation. It is per se judicially enforceable When our
Constitution mandates that [i]n the grant of rights, privileges, and concessions covering national
economy and patrimony, the State shall give preference to qualified Filipinos, it means just that
MR. NOLLEDO Yes, 16
— qualified Filipinos shall be preferred. And when our Constitution declares that a right exists in
certain specified circumstances an action may be maintained to enforce such right
Quite apparently, Sec. 10, second par., of Art XII is couched in such a way as not to make it notwithstanding the absence of any legislation on the subject; consequently, if there is no statute
appear that it is non-self-executing but simply for purposes of style. But, certainly, the legislature especially enacted to enforce such constitutional right, such right enforces itself by its own
is not precluded from enacting other further laws to enforce the constitutional provision so long inherent potency and puissance, and from which all legislations must take their bearings. Where
as the contemplated statute squares with the Constitution. Minor details may be left to the there is a right there is a remedy. Ubi jus ibi remedium.
legislature without impairing the self-executing nature of constitutional provisions.
As regards our national patrimony, a member of the 1986 Constitutional Commission 34 explains
In self-executing constitutional provisions, the legislature may still enact legislation to facilitate —
the exercise of powers directly granted by the constitution, further the operation of such a
provision, prescribe a practice to be used for its enforcement, provide a convenient remedy for
The patrimony of the Nation that should be conserved and developed refers
the protection of the rights secured or the determination thereof, or place reasonable safeguards
not only to out rich natural resources but also to the cultural heritage of out
around the exercise of the right. The mere fact that legislation may supplement and add to or
race. It also refers to our intelligence in arts, sciences and letters. Therefore,
prescribe a penalty for the violation of a self-executing constitutional provision does not render
we should develop not only our lands, forests, mines and other natural
such a provision ineffective in the absence of such legislation. The omission from a constitution
resources but also the mental ability or faculty of our people.
of any express provision for a remedy for enforcing a right or liability is not necessarily an
indication that it was not intended to be self-executing. The rule is that a self-executing provision
of the constitution does not necessarily exhaust legislative power on the subject, but any We agree. In its plain and ordinary meaning, the term patrimony pertains to heritage. 35 When
legislation must be in harmony with the constitution, further the exercise of constitutional right the Constitution speaks of national patrimony, it refers not only to the natural resources of the
and make it more available. 17 Subsequent legislation however does not necessarily mean that Philippines, as the Constitution could have very well used the term natural resources, but also to
the subject constitutional provision is not, by itself, fully enforceable. the cultural heritage of the Filipinos.

Respondents also argue that the non-self-executing nature of Sec. 10, second par., of Art. XII is Manila Hotel has become a landmark — a living testimonial of Philippine heritage. While it was
implied from the tenor of the first and third paragraphs of the same section which undoubtedly restrictively an American hotel when it first opened in 1912, it immediately evolved to be truly
are not self-executing. 18 The argument is flawed. If the first and third paragraphs are not self- Filipino, Formerly a concourse for the elite, it has since then become the venue of various
executing because Congress is still to enact measures to encourage the formation and operation significant events which have shaped Philippine history. It was called the Cultural Center of the
of enterprises fully owned by Filipinos, as in the first paragraph, and the State still needs 1930's. It was the site of the festivities during the inauguration of the Philippine Commonwealth.
legislation to regulate and exercise authority over foreign investments within its national Dubbed as the Official Guest House of the Philippine Government. it plays host to dignitaries
jurisdiction, as in the third paragraph, then a fortiori, by the same logic, the second paragraph and official visitors who are accorded the traditional Philippine hospitality. 36
can only be self-executing as it does not by its language require any legislation in order to give
preference to qualified Filipinos in the grant of rights, privileges and concessions covering the
The history of the hotel has been chronicled in the book The Manila Hotel: The Heart and
national economy and patrimony. A constitutional provision may be self-executing in one part
Memory of a City. 37During World War II the hotel was converted by the Japanese Military
and non-self-executing in another. 19
Administration into a military headquarters. When the American forces returned to recapture
Manila the hotel was selected by the Japanese together with Intramuros as the two (2) places fro
Even the cases cited by respondents holding that certain constitutional provisions are merely their final stand. Thereafter, in the 1950's and 1960's, the hotel became the center of political
statements of principles and policies, which are basically not self-executing and only placed in activities, playing host to almost every political convention. In 1970 the hotel reopened after a
the Constitution as moral incentives to legislation, not as judicially enforceable rights — are renovation and reaped numerous international recognitions, an acknowledgment of the Filipino
simply not in point. Basco v. Philippine Amusements and Gaming Corporation 20 speaks of talent and ingenuity. In 1986 the hotel was the site of a failed coup d' etat where an aspirant for
constitutional provisions on personal dignity, 21 the sanctity of family life, 22 the vital role of the vice-president was "proclaimed" President of the Philippine Republic.
youth in nation-building 23 the promotion of social justice, 24 and the values of
education. 25 Tolentino v. Secretary of Finance 26 refers to the constitutional provisions on social
For more than eight (8) decades Manila Hotel has bore mute witness to the triumphs and
justice and human rights 27 and on education. 28 Lastly, Kilosbayan, Inc. v. Morato 29 cites
failures, loves and frustrations of the Filipinos; its existence is impressed with public interest; its
provisions on the promotion of general welfare, 30 the sanctity of family life, 31 the vital role of the
own historicity associated with our struggle for sovereignty, independence and nationhood.
youth in nation-building 32 and the promotion of total human liberation and development. 33A
Verily, Manila Hotel has become part of our national economy and patrimony. For sure, 51% of
reading of these provisions indeed clearly shows that they are not judicially enforceable
the equity of the MHC comes within the purview of the constitutional shelter for it comprises the
constitutional rights but merely guidelines for legislation. The very terms of the provisions
majority and controlling stock, so that anyone who acquires or owns the 51% will have actual
manifest that they are only principles upon which the legislations must be based. Res ipsa
control and management of the hotel. In this instance, 51% of the MHC cannot be disassociated
loquitur.
from the hotel and the land on which the hotel edifice stands. Consequently, we cannot sustain
respondents' claim that the Filipino First Policy provision is not applicable since what is being
On the other hand, Sec. 10, second par., Art. XII of the of the 1987 Constitution is a mandatory, sold is only 51% of the outstanding shares of the corporation, not the Hotel building nor the land
positive command which is complete in itself and which needs no further guidelines or upon which the building stands. 38
implementing laws or rules for its enforcement. From its very words the provision does not
The argument is pure sophistry. The term qualified Filipinos as used in Our Constitution also the word "Filipinos" here, as intended by the
includes corporations at least 60% of which is owned by Filipinos. This is very clear from the proponents, will include not only individual Filipinos but
proceedings of the 1986 Constitutional Commission also Filipino-controlled entities or entities fully-controlled
by Filipinos. 40
THE PRESIDENT. Commissioner Davide is
recognized. The phrase preference to qualified Filipinos was explained thus —

MR. DAVIDE. I would like to introduce an amendment MR. FOZ. Madam President, I would like to request
to the Nolledo amendment. And the amendment would Commissioner Nolledo to please restate his
consist in substituting the words "QUALIFIED amendment so that I can ask a question.
FILIPINOS" with the following: "CITIZENS OF THE
PHILIPPINES OR CORPORATIONS OR
MR. NOLLEDO. "IN THE GRANT OF RIGHTS,
ASSOCIATIONS WHOSE CAPITAL OR
PRIVILEGES AND CONCESSIONS COVERING THE
CONTROLLING STOCK IS WHOLLY OWNED BY
NATIONAL ECONOMY AND PATRIMONY, THE
SUCH CITIZENS.
STATE SHALL GIVE PREFERENCE TO QUALIFIED
FILIPINOS."
xxx xxx xxx
MR FOZ. In connection with that amendment, if a
MR. MONSOD. Madam President, apparently the foreign enterprise is qualified and a Filipino enterprise is
proponent is agreeable, but we have to raise a also qualified, will the Filipino enterprise still be given a
question. Suppose it is a corporation that is 80-percent preference?
Filipino, do we not give it preference?
MR. NOLLEDO. Obviously.
MR. DAVIDE. The Nolledo amendment would refer to
an individual Filipino. What about a corporation wholly
MR. FOZ. If the foreigner is more qualified in some
owned by Filipino citizens?
aspects than the Filipino enterprise, will the Filipino still
be preferred?
MR. MONSOD. At least 60 percent, Madam President.
MR. NOLLEDO. The answer is "yes."
MR. DAVIDE. Is that the intention?
MR. FOZ. Thank you, 41
MR. MONSOD. Yes, because, in fact, we would be
limiting it if we say that the preference should only be
Expounding further on the Filipino First Policy provision Commissioner Nolledo continues —
100-percent Filipino.

MR. NOLLEDO. Yes, Madam President. Instead of "MUST," it will be


MR: DAVIDE. I want to get that meaning clear because
"SHALL — THE STATE SHALL GlVE PREFERENCE TO QUALIFIED
"QUALIFIED FILIPINOS" may refer only to individuals
FILIPINOS. This embodies the so-called "Filipino First" policy. That means
and not to juridical personalities or entities.
that Filipinos should be given preference in the grant of concessions,
privileges and rights covering the national patrimony. 42
39
MR. MONSOD. We agree, Madam President.
The exchange of views in the sessions of the Constitutional Commission regarding the subject
xxx xxx xxx provision was still further clarified by Commissioner Nolledo 43 —

MR. RODRIGO. Before we vote, may I request that the Paragraph 2 of Section 10 explicitly mandates the "Pro-Filipino" bias in all
amendment be read again. economic concerns. It is better known as the FILIPINO FIRST Policy . . .
This provision was never found in previous Constitutions . . . .
MR. NOLLEDO. The amendment will read: "IN THE
GRANT OF RIGHTS, PRIVILEGES AND The term "qualified Filipinos" simply means that preference shall be given to
CONCESSIONS COVERING THE NATIONAL those citizens who can make a viable contribution to the common good,
ECONOMY AND PATRIMONY, THE STATE SHALL because of credible competence and efficiency. It certainly does NOT
GIVE PREFERENCE TO QUALIFIED FILIPINOS." And mandate the pampering and preferential treatment to Filipino citizens or
organizations that are incompetent or inefficient, since such an State is correspondingly directed to the three(3) branches of government. It is undeniable that in
indiscriminate preference would be counter productive and inimical to the this case the subject constitutional injunction is addressed among others to the Executive
common good. Department and respondent GSIS, a government instrumentality deriving its authority from the
State.
In the granting of economic rights, privileges, and concessions, when a
choice has to be made between a "qualified foreigner" end a "qualified It should be stressed that while the Malaysian firm offered the higher bid it is not yet the winning
Filipino," the latter shall be chosen over the former." bidder. The bidding rules expressly provide that the highest bidder shall only be declared the
winning bidder after it has negotiated and executed the necessary contracts, and secured the
requisite approvals. Since the "Filipino First Policy provision of the Constitution bestows
Lastly, the word qualified is also determinable. Petitioner was so considered by respondent
preference on qualified Filipinos the mere tending of the highest bid is not an assurance that the
GSIS and selected as one of the qualified bidders. It was pre-qualified by respondent GSIS in
highest bidder will be declared the winning bidder. Resultantly, respondents are not bound to
accordance with its own guidelines so that the sole inference here is that petitioner has been
make the award yet, nor are they under obligation to enter into one with the highest bidder. For
found to be possessed of proven management expertise in the hotel industry, or it has significant
in choosing the awardee respondents are mandated to abide by the dictates of the 1987
equity ownership in another hotel company, or it has an overall management and marketing
Constitution the provisions of which are presumed to be known to all the bidders and other
proficiency to successfully operate the Manila Hotel. 44
interested parties.

The penchant to try to whittle away the mandate of the Constitution by arguing that the subject
Adhering to the doctrine of constitutional supremacy, the subject constitutional provision is, as it
provision is not self-executory and requires implementing legislation is quite disturbing. The
should be, impliedly written in the bidding rules issued by respondent GSIS, lest the bidding
attempt to violate a clear constitutional provision — by the government itself — is only too
rules be nullified for being violative of the Constitution. It is a basic principle in constitutional law
distressing. To adopt such a line of reasoning is to renounce the duty to ensure faithfulness to
that all laws and contracts must conform with the fundamental law of the land. Those which
the Constitution. For, even some of the provisions of the Constitution which evidently need
violate the Constitution lose their reason for being.
implementing legislation have juridical life of their own and can be the source of a judicial
remedy. We cannot simply afford the government a defense that arises out of the failure to enact
further enabling, implementing or guiding legislation. In fine, the discourse of Fr. Joaquin G. Paragraph V. J. 1 of the bidding rules provides that [if] for any reason the Highest Bidder cannot
Bernas, S.J., on constitutional government is apt — be awarded the Block of Shares, GSIS may offer this to other Qualified Bidders that have validly
submitted bids provided that these Qualified Bidders are willing to match the highest bid in terms
of price per
The executive department has a constitutional duty to implement laws,
share. 47 Certainly, the constitutional mandate itself is reason enough not to award the block of
including the Constitution, even before Congress acts — provided that there
shares immediately to the foreign bidder notwithstanding its submission of a higher, or even the
are discoverable legal standards for executive action. When the executive
highest, bid. In fact, we cannot conceive of a stronger reason than the constitutional injunction
acts, it must be guided by its own understanding of the constitutional
itself.
command and of applicable laws. The responsibility for reading and
understanding the Constitution and the laws is not the sole prerogative of
Congress. If it were, the executive would have to ask Congress, or perhaps In the instant case, where a foreign firm submits the highest bid in a public bidding concerning
the Court, for an interpretation every time the executive is confronted by a the grant of rights, privileges and concessions covering the national economy and patrimony,
constitutional command. That is not how constitutional government thereby exceeding the bid of a Filipino, there is no question that the Filipino will have to be
operates. 45 allowed to match the bid of the foreign entity. And if the Filipino matches the bid of a foreign firm
the award should go to the Filipino. It must be so if we are to give life and meaning to the Filipino
First Policy provision of the 1987 Constitution. For, while this may neither be expressly stated
Respondents further argue that the constitutional provision is addressed to the State, not to
nor contemplated in the bidding rules, the constitutional fiat is, omnipresent to be simply
respondent GSIS which by itself possesses a separate and distinct personality. This argument
disregarded. To ignore it would be to sanction a perilous skirting of the basic law.
again is at best specious. It is undisputed that the sale of 51% of the MHC could only be carried
out with the prior approval of the State acting through respondent Committee on Privatization. As
correctly pointed out by Fr. Joaquin G. Bernas, S.J., this fact alone makes the sale of the assets This Court does not discount the apprehension that this policy may discourage foreign investors.
of respondents GSIS and MHC a "state action." In constitutional jurisprudence, the acts of But the Constitution and laws of the Philippines are understood to be always open to public
persons distinct from the government are considered "state action" covered by the Constitution scrutiny. These are given factors which investors must consider when venturing into business in
(1) when the activity it engages in is a "public function;" (2) when the government is so a foreign jurisdiction. Any person therefore desiring to do business in the Philippines or with any
significantly involved with the private actor as to make the government responsible for his action; of its agencies or instrumentalities is presumed to know his rights and obligations under the
and, (3) when the government has approved or authorized the action. It is evident that the act of Constitution and the laws of the forum.
respondent GSIS in selling 51% of its share in respondent MHC comes under the second and
third categories of "state action." Without doubt therefore the transaction. although entered into
The argument of respondents that petitioner is now estopped from questioning the sale to
by respondent GSIS, is in fact a transaction of the State and therefore subject to the
Renong Berhad since petitioner was well aware from the beginning that a foreigner could
constitutional command. 46
participate in the bidding is meritless. Undoubtedly, Filipinos and foreigners alike were invited to
the bidding. But foreigners may be awarded the sale only if no Filipino qualifies, or if the qualified
When the Constitution addresses the State it refers not only to the people but also to the Filipino fails to match the highest bid tendered by the foreign entity. In the case before us, while
government as elements of the State. After all, government is composed of three (3) divisions of petitioner was already preferred at the inception of the bidding because of the constitutional
power — legislative, executive and judicial. Accordingly, a constitutional mandate directed to the mandate, petitioner had not yet matched the bid offered by Renong Berhad. Thus it did not have
the right or personality then to compel respondent GSIS to accept its earlier bid. Rightly, only material values. Indeed, the Court will always defer to the Constitution in the proper governance
after it had matched the bid of the foreign firm and the apparent disregard by respondent GSIS of a free society; after all, there is nothing so sacrosanct in any economic policy as to draw itself
of petitioner's matching bid did the latter have a cause of action. beyond judicial review when the Constitution is involved. 49

Besides, there is no time frame for invoking the constitutional safeguard unless perhaps the Nationalism is inherent, in the very concept of the Philippines being a democratic and republican
award has been finally made. To insist on selling the Manila Hotel to foreigners when there is a state, with sovereignty residing in the Filipino people and from whom all government authority
Filipino group willing to match the bid of the foreign group is to insist that government be treated emanates. In nationalism, the happiness and welfare of the people must be the goal. The nation-
as any other ordinary market player, and bound by its mistakes or gross errors of judgment, state can have no higher purpose. Any interpretation of any constitutional provision must adhere
regardless of the consequences to the Filipino people. The miscomprehension of the to such basic concept. Protection of foreign investments, while laudible, is merely a policy. It
Constitution is regrettable. Thus we would rather remedy the indiscretion while there is still an cannot override the demands of nationalism. 50
opportunity to do so than let the government develop the habit of forgetting that the Constitution
lays down the basic conditions and parameters for its actions.
The Manila Hotel or, for that matter, 51% of the MHC, is not just any commodity to be sold to the
highest bidder solely for the sake of privatization. We are not talking about an ordinary piece of
Since petitioner has already matched the bid price tendered by Renong Berhad pursuant to the property in a commercial district. We are talking about a historic relic that has hosted many of
bidding rules, respondent GSIS is left with no alternative but to award to petitioner the block of the most important events in the short history of the Philippines as a nation. We are talking about
shares of MHC and to execute the necessary agreements and documents to effect the sale in a hotel where heads of states would prefer to be housed as a strong manifestation of their desire
accordance not only with the bidding guidelines and procedures but with the Constitution as well. to cloak the dignity of the highest state function to their official visits to the Philippines. Thus the
The refusal of respondent GSIS to execute the corresponding documents with petitioner as Manila Hotel has played and continues to play a significant role as an authentic repository of
provided in the bidding rules after the latter has matched the bid of the Malaysian firm clearly twentieth century Philippine history and culture. In this sense, it has become truly a reflection of
constitutes grave abuse of discretion. the Filipino soul — a place with a history of grandeur; a most historical setting that has played a
part in the shaping of a country. 51
The Filipino First Policy is a product of Philippine nationalism. It is embodied in the 1987
Constitution not merely to be used as a guideline for future legislation but primarily to be This Court cannot extract rhyme nor reason from the determined efforts of respondents to sell
enforced; so must it be enforced. This Court as the ultimate guardian of the Constitution will the historical landmark — this Grand Old Dame of hotels in Asia — to a total stranger. For,
never shun, under any reasonable circumstance, the duty of upholding the majesty of the indeed, the conveyance of this epic exponent of the Filipino psyche to alien hands cannot be
Constitution which it is tasked to defend. It is worth emphasizing that it is not the intention of this less than mephistophelian for it is, in whatever manner viewed, a veritable alienation of a
Court to impede and diminish, much less undermine, the influx of foreign investments. Far from nation's soul for some pieces of foreign silver. And so we ask: What advantage, which cannot be
it, the Court encourages and welcomes more business opportunities but avowedly sanctions the equally drawn from a qualified Filipino, can be gained by the Filipinos Manila Hotel — and all
preference for Filipinos whenever such preference is ordained by the Constitution. The position that it stands for — is sold to a non-Filipino? How much of national pride will vanish if the
of the Court on this matter could have not been more appropriately articulated by Chief Justice nation's cultural heritage is entrusted to a foreign entity? On the other hand, how much dignity
Narvasa — will be preserved and realized if the national patrimony is safekept in the hands of a qualified,
zealous and well-meaning Filipino? This is the plain and simple meaning of the Filipino First
Policy provision of the Philippine Constitution. And this Court, heeding the clarion call of the
As scrupulously as it has tried to observe that it is not its function to
Constitution and accepting the duty of being the elderly watchman of the nation, will continue to
substitute its judgment for that of the legislature or the executive about the
respect and protect the sanctity of the Constitution.
wisdom and feasibility of legislation economic in nature, the Supreme Court
has not been spared criticism for decisions perceived as obstacles to
economic progress and development . . . in connection with a temporary WHEREFORE, respondents GOVERNMENT SERVICE INSURANCE SYSTEM, MANILA
injunction issued by the Court's First Division against the sale of the Manila HOTEL CORPORATION, COMMITTEE ON PRIVATIZATION and OFFICE OF THE
Hotel to a Malaysian Firm and its partner, certain statements were published GOVERNMENT CORPORATE COUNSEL are directed to CEASE and DESIST from selling 51%
in a major daily to the effect that injunction "again demonstrates that the of the shares of the Manila Hotel Corporation to RENONG BERHAD, and to ACCEPT the
Philippine legal system can be a major obstacle to doing business here. matching bid of petitioner MANILA PRINCE HOTEL CORPORATION to purchase the subject
51% of the shares of the Manila Hotel Corporation at P44.00 per share and thereafter to execute
the necessary clearances and to do such other acts and deeds as may be necessary for
Let it be stated for the record once again that while it is no business of the
purpose.
Court to intervene in contracts of the kind referred to or set itself up as the
judge of whether they are viable or attainable, it is its bounden duty to make
sure that they do not violate the Constitution or the laws, or are not adopted SO ORDERED.
or implemented with grave abuse of discretion amounting to lack or excess
of jurisdiction. It will never shirk that duty, no matter how buffeted by winds
of unfair and ill-informed criticism. 48

Privatization of a business asset for purposes of enhancing its business viability and preventing
further losses, regardless of the character of the asset, should not take precedence over non-
material values. A commercial, nay even a budgetary, objective should not be pursued at the
expense of national pride and dignity. For the Constitution enshrines higher and nobler non-
G.R. No. 171182 August 23, 2012 Wherefore, in the light of the foregoing, judgment is hereby rendered in favor of the plaintiff and
against the defendants ordering the latter to pay plaintiff, jointly and severally, the following, to
wit:
UNIVERSITY OF THE PHILIPPINES, JOSE V. ABUEVA, RAUL P. DE GUZMAN, RUBEN P.
ASPIRAS, EMMANUEL P. BELLO, WILFREDO P. DAVID, CASIANO S. ABRIGO, and
JOSEFINA R. LICUANAN,Petitioners, 1. ₱ 503,462.74 amount of the third billing, additional accomplished work and retention
vs. money
HON. AGUSTIN S. DIZON, his capacity as Presiding Judge of the Regional Trial Court of
Quezon City, Branch 80, STERN BUILDERS, INC., and SERVILLANO DELA
2. ₱ 5,716,729.00 in actual damages
CRUZ, Respondents.

3. ₱ 10,000,000.00 in moral damages


DECISION

4. ₱ 150,000.00 and ₱ 1,500.00 per appearance as attorney’s fees; and


BERSAMIN, J.:

5. Costs of suit.
Trial judges should not immediately issue writs of execution or garnishment against the
Government or any of its subdivisions, agencies and instrumentalities to enforce money
judgments.1 They should bear in mind that the primary jurisdiction to examine, audit and settle all SO ORDERED.
claims of any sort due from the Government or any of its subdivisions, agencies and
instrumentalities pertains to the Commission on Audit (COA) pursuant to Presidential Decree
Following the RTC’s denial of its motion for reconsideration on May 7, 2002,6 the UP filed a
No. 1445 (Government Auditing Code of the Philippines).
notice of appeal on June 3, 2002.7 Stern Builders and dela Cruz opposed the notice of appeal on
the ground of its filing being belated, and moved for the execution of the decision. The UP
The Case countered that the notice of appeal was filed within the reglementary period because the UP’s
Office of Legal Affairs (OLS) in Diliman, Quezon City received the order of denial only on May
31, 2002. On September 26, 2002, the RTC denied due course to the notice of appeal for having
On appeal by the University of the Philippines and its then incumbent officials (collectively, the
been filed out of time and granted the private respondents’ motion for execution. 8
UP) is the decision promulgated on September 16, 2005,2 whereby the Court of Appeals (CA)
upheld the order of the Regional Trial Court (RTC), Branch 80, in Quezon City that directed the
garnishment of public funds amounting to ₱ 16,370,191.74 belonging to the UP to satisfy the writ The RTC issued the writ of execution on October 4, 2002,9 and the sheriff of the RTC served the
of execution issued to enforce the already final and executory judgment against the UP. writ of execution and notice of demand upon the UP, through its counsel, on October 9,
2002.10 The UP filed an urgent motion to reconsider the order dated September 26, 2002, to
quash the writ of execution dated October 4, 2002, and to restrain the proceedings. 11 However,
Antecedents
the RTC denied the urgent motion on April 1, 2003.12

On August 30, 1990, the UP, through its then President Jose V. Abueva, entered into a General
On June 24, 2003, the UP assailed the denial of due course to its appeal through a petition
Construction Agreement with respondent Stern Builders Corporation (Stern Builders),
for certiorari in the Court of Appeals (CA), docketed as CA-G.R. No. 77395.13
represented by its President and General Manager Servillano dela Cruz, for the construction of
the extension building and the renovation of the College of Arts and Sciences Building in the
campus of the University of the Philippines in Los Baños (UPLB). 3 On February 24, 2004, the CA dismissed the petition for certiorari upon finding that the UP’s
notice of appeal had been filed late,14 stating:
In the course of the implementation of the contract, Stern Builders submitted three progress
billings corresponding to the work accomplished, but the UP paid only two of the billings. The Records clearly show that petitioners received a copy of the Decision dated November 28, 2001
third billing worth ₱ 273,729.47 was not paid due to its disallowance by the Commission on Audit and January 7, 2002, thus, they had until January 22, 2002 within which to file their appeal. On
(COA). Despite the lifting of the disallowance, the UP failed to pay the billing, prompting Stern January 16, 2002 or after the lapse of nine (9) days, petitioners through their counsel Atty.
Builders and dela Cruz to sue the UP and its co-respondent officials to collect the unpaid billing Nolasco filed a Motion for Reconsideration of the aforesaid decision, hence, pursuant to the
and to recover various damages. The suit, entitled Stern Builders Corporation and Servillano R. rules, petitioners still had six (6) remaining days to file their appeal. As admitted by the
Dela Cruz v. University of the Philippines Systems, Jose V. Abueva, Raul P. de Guzman, Ruben petitioners in their petition (Rollo, p. 25), Atty. Nolasco received a copy of the Order denying their
P. Aspiras, Emmanuel P. Bello, Wilfredo P. David, Casiano S. Abrigo, and Josefina R. motion for reconsideration on May 17, 2002, thus, petitioners still has until May 23, 2002 (the
Licuanan, was docketed as Civil Case No. Q-93-14971 of the Regional Trial Court in Quezon remaining six (6) days) within which to file their appeal. Obviously, petitioners were not able to
City (RTC).4 file their Notice of Appeal on May 23, 2002 as it was only filed on June 3, 2002.

After trial, on November 28, 2001, the RTC rendered its decision in favor of the plaintiffs, 5 viz: In view of the said circumstances, We are of the belief and so holds that the Notice of Appeal
filed by the petitioners was really filed out of time, the same having been filed seventeen (17)
days late of the reglementary period. By reason of which, the decision dated November 28, 2001
had already become final and executory. "Settled is the rule that the perfection of an appeal in
the manner and within the period permitted by law is not only mandatory but jurisdictional, and WHEREFORE, premises considered, there being no more legal impediment for the release of
failure to perfect that appeal renders the challenged judgment final and executory. This is not an the garnished amount in satisfaction of the judgment award in the instant case, let the amount
empty procedural rule but is grounded on fundamental considerations of public policy and sound garnished be immediately released by the Development Bank of the Philippines, Commonwealth
practice." (Ram’s Studio and Photographic Equipment, Inc. vs. Court of Appeals, 346 SCRA Branch, Quezon City in favor of the plaintiff.
691, 696). Indeed, Atty. Nolasco received the order of denial of the Motion for Reconsideration
on May 17, 2002 but filed a Notice of Appeal only on June 3, 3003. As such, the decision of the
SO ORDERED.
lower court ipso facto became final when no appeal was perfected after the lapse of the
reglementary period. This procedural caveat cannot be trifled with, not even by the High Court.15
The UP was served on January 3, 2005 with the order of December 21, 2004 directing DBP to
release the garnished funds.33
The UP sought a reconsideration, but the CA denied the UP’s motion for reconsideration on April
19, 2004.16
On January 6, 2005, Stern Builders and dela Cruz moved to cite DBP in direct contempt of court
for its non-compliance with the order of release.34
On May 11, 2004, the UP appealed to the Court by petition for review on certiorari (G.R. No.
163501).
Thereupon, on January 10, 2005, the UP brought a petition for certiorari in the CA to challenge
the jurisdiction of the RTC in issuing the order of December 21, 2004 (CA-G.R. CV No.
On June 23, 2004, the Court denied the petition for review.17 The UP moved for the
88125).35 Aside from raising the denial of due process, the UP averred that the RTC committed
reconsideration of the denial of its petition for review on August 29, 2004, 18 but the Court denied
grave abuse of discretion amounting to lack or excess of jurisdiction in ruling that there was no
the motion on October 6, 2004.19 The denial became final and executory on November 12,
longer any legal impediment to the release of the garnished funds. The UP argued that
2004.20
government funds and properties could not be seized by virtue of writs of execution or
garnishment, as held in Department of Agriculture v. National Labor Relations
In the meanwhile that the UP was exhausting the available remedies to overturn the denial of Commission,36 and citing Section 84 of Presidential Decree No. 1445 to the effect that "revenue
due course to the appeal and the issuance of the writ of execution, Stern Builders and dela Cruz funds shall not be paid out of any public treasury or depository except in pursuance of an
filed in the RTC their motions for execution despite their previous motion having already been appropriation law or other specific statutory authority;" and that the order of garnishment clashed
granted and despite the writ of execution having already issued. On June 11, 2003, the RTC with the ruling in University of the Philippines Board of Regents v. Ligot-Telan37 to the effect that
granted another motion for execution filed on May 9, 2003 (although the RTC had already issued the funds belonging to the UP were public funds.
the writ of execution on October 4, 2002).21
On January 19, 2005, the CA issued a temporary restraining order (TRO) upon application by
On June 23, 2003 and July 25, 2003, respectively, the sheriff served notices of garnishment on the UP.38
the UP’s depository banks, namely: Land Bank of the Philippines (Buendia Branch) and the
Development Bank of the Philippines (DBP), Commonwealth Branch.22 The UP assailed the
On March 22, 2005, Stern Builders and dela Cruz filed in the RTC their amended motion for
garnishment through an urgent motion to quash the notices of garnishment; 23 and a motion to
sheriff’s assistance to implement the release order dated December 21, 2004, stating that the
quash the writ of execution dated May 9, 2003.24
60-day period of the TRO of the CA had already lapsed.39 The UP opposed the amended motion
and countered that the implementation of the release order be suspended.40
On their part, Stern Builders and dela Cruz filed their ex parte motion for issuance of a release
order.25
On May 3, 2005, the RTC granted the amended motion for sheriff’s assistance and directed the
sheriff to proceed to the DBP to receive the check in satisfaction of the judgment.41
On October 14, 2003, the RTC denied the UP’s urgent motion to quash, and granted Stern
Builders and dela Cruz’s ex parte motion for issuance of a release order.26
The UP sought the reconsideration of the order of May 3, 2005.42

The UP moved for the reconsideration of the order of October 14, 2003, but the RTC denied the
On May 16, 2005, DBP filed a motion to consign the check representing the judgment award and
motion on November 7, 2003.27
to dismiss the motion to cite its officials in contempt of court. 43

On January 12, 2004, Stern Builders and dela Cruz again sought the release of the garnished
On May 23, 2005, the UP presented a motion to withhold the release of the payment of the
funds.28 Despite the UP’s opposition,29 the RTC granted the motion to release the garnished
judgment award.44
funds on March 16, 2004.30 On April 20, 2004, however, the RTC held in abeyance the
enforcement of the writs of execution issued on October 4, 2002 and June 3, 2003 and all the
ensuing notices of garnishment, citing Section 4, Rule 52, Rules of Court, which provided that On July 8, 2005, the RTC resolved all the pending matters,45 noting that the DBP had already
the pendency of a timely motion for reconsideration stayed the execution of the judgment. 31 delivered to the sheriff Manager’s Check No. 811941 for ₱ 16,370,191.74 representing the
garnished funds payable to the order of Stern Builders and dela Cruz as its compliance with the
RTC’s order dated December 21, 2004.46 However, the RTC directed in the same order that
On December 21, 2004, the RTC, through respondent Judge Agustin S. Dizon, authorized the
Stern Builders and dela Cruz should not encash the check or withdraw its amount pending the
release of the garnished funds of the UP,32 to wit:
final resolution of the UP’s petition for certiorari, to wit: 47
To enable the money represented in the check in question (No. 00008119411) to earn interest defendants and the plaintiff entered into the General Construction of Agreement there
during the pendency of the defendant University of the Philippines application for a writ of is an amount already allocated by the latter for the said project which is no longer
injunction with the Court of Appeals the same may now be deposited by the plaintiff at the subject of future appropriation."49
garnishee Bank (Development Bank of the Philippines), the disposition of the amount
represented therein being subject to the final outcome of the case of the University of the
After the CA denied their motion for reconsideration on December 23, 2005, the petitioners
Philippines et al., vs. Hon. Agustin S. Dizon et al., (CA G.R. 88125) before the Court of Appeals.
appealed by petition for review.

Let it be stated herein that the plaintiff is not authorized to encash and withdraw the amount
Matters Arising During the Pendency of the Petition
represented in the check in question and enjoy the same in the fashion of an owner during the
pendency of the case between the parties before the Court of Appeals which may or may not be
resolved in plaintiff’s favor. On January 30, 2006, Judge Dizon of the RTC (Branch 80) denied Stern Builders and dela
Cruz’s motion to withdraw the deposit, in consideration of the UP’s intention to appeal to the
CA,50 stating:
With the end in view of seeing to it that the check in question is deposited by the plaintiff at the
Development Bank of the Philippines (garnishee bank), Branch Sheriff Herlan Velasco is
directed to accompany and/or escort the plaintiff in making the deposit of the check in question. Since it appears that the defendants are intending to file a petition for review of the Court of
Appeals resolution in CA-G.R. No. 88125 within the reglementary period of fifteen (15) days from
receipt of resolution, the Court agrees with the defendants stand that the granting of plaintiffs’
SO ORDERED.
subject motion is premature.

On September 16, 2005, the CA promulgated its assailed decision dismissing the UP’s petition
Let it be stated that what the Court meant by its Order dated July 8, 2005 which states in part
for certiorari, ruling that the UP had been given ample opportunity to contest the motion to direct
that the "disposition of the amount represented therein being subject to the final outcome of the
the DBP to deposit the check in the name of Stern Builders and dela Cruz; and that the
case of the University of the Philippines, et. al., vs. Hon. Agustin S. Dizon et al., (CA G.R. No.
garnished funds could be the proper subject of garnishment because they had been already
88125 before the Court of Appeals) is that the judgment or resolution of said court has to be final
earmarked for the project, with the UP holding the funds only in a fiduciary capacity, 48 viz:
and executory, for if the same will still be elevated to the Supreme Court, it will not attain finality
yet until the highest court has rendered its own final judgment or resolution. 51
Petitioners next argue that the UP funds may not be seized for execution or garnishment to
satisfy the judgment award. Citing Department of Agriculture vs. NLRC, University of the
However, on January 22, 2007, the UP filed an Urgent Application for A Temporary Restraining
Philippines Board of Regents vs. Hon. Ligot-Telan, petitioners contend that UP deposits at Land
Order and/or A Writ of Preliminary Injunction,52 averring that on January 3, 2007, Judge Maria
Bank and the Development Bank of the Philippines, being government funds, may not be
Theresa dela Torre-Yadao (who had meanwhile replaced Judge Dizon upon the latter’s
released absent an appropriations bill from Congress.
appointment to the CA) had issued another order allowing Stern Builders and dela Cruz to
withdraw the deposit,53 to wit:
The argument is specious. UP entered into a contract with private respondents for the expansion
and renovation of the Arts and Sciences Building of its campus in Los Baños, Laguna.
It bears stressing that defendants’ liability for the payment of the judgment obligation has
Decidedly, there was already an appropriations earmarked for the said project. The said funds
become indubitable due to the final and executory nature of the Decision dated November 28,
are retained by UP, in a fiduciary capacity, pending completion of the construction project.
2001. Insofar as the payment of the [sic] judgment obligation is concerned, the Court believes
that there is nothing more the defendant can do to escape liability. It is observed that there is
We agree with the trial Court [sic] observation on this score: nothing more the defendant can do to escape liability. It is observed that defendant U.P. System
had already exhausted all its legal remedies to overturn, set aside or modify the decision (dated
November 28, 2001( rendered against it. The way the Court sees it, defendant U.P. System’s
"4. Executive Order No. 109 (Directing all National Government Agencies to Revert
petition before the Supreme Court concerns only with the manner by which said judgment award
Certain Accounts Payable to the Cumulative Result of Operations of the National
should be satisfied. It has nothing to do with the legality or propriety thereof, although it prays for
Government and for Other Purposes) Section 9. Reversion of Accounts Payable,
the deletion of [sic] reduction of the award of moral damages.
provides that, all 1995 and prior years documented accounts payable and all
undocumented accounts regardless of the year they were incurred shall be reverted to
the Cumulative Result of Operations of the National Government (CROU). This shall It must be emphasized that this Court’s finding, i.e., that there was sufficient appropriation
apply to accounts payable of all funds, except fiduciary funds, as long as the purpose earmarked for the project, was upheld by the Court of Appeals in its decision dated September
for which the funds were created have not been accomplished and accounts payable 16, 2005. Being a finding of fact, the Supreme Court will, ordinarily, not disturb the same was
under foreign assisted projects for the duration of the said project. In this regard, the said Court is not a trier of fact. Such being the case, defendants’ arguments that there was no
Department of Budget and Management issued Joint-Circular No. 99-6 4.0 (4.3) sufficient appropriation for the payment of the judgment obligation must fail.
Procedural Guidelines which provides that all accounts payable that reverted to the
CROU may be considered for payment upon determination thru administrative
While it is true that the former Presiding Judge of this Court in its Order dated January 30, 2006
process, of the existence, validity and legality of the claim. Thus, the allegation of the
had stated that:
defendants that considering no appropriation for the payment of any amount awarded
to plaintiffs appellee the funds of defendant-appellants may not be seized pursuant to
a writ of execution issued by the regular court is misplaced. Surely when the
Let it be stated that what the Court meant by its Order dated July 8, 2005 which states in part could have enjoined the release of plaintiffs’ deposit. The Court also took into account the
that the "disposition of the amount represented therein being subject to the final outcome of the following factors:
case of the University of the Philippines, et. al., vs. Hon. Agustin S. Dizon et al., (CA G.R. No.
88125 before the Court of Appeals) is that the judgment or resolution of said court has to be final
a) the Decision in this case had long been final and executory after it was rendered on
and executory, for if the same will still be elevated to the Supreme Court, it will not attain finality
November 28, 2001;
yet until the highest court has rendered its own final judgment or resolution.

b) the propriety of the dismissal of U.P. System’s appeal was upheld by the Supreme
it should be noted that neither the Court of Appeals nor the Supreme Court issued a preliminary
Court;
injunction enjoining the release or withdrawal of the garnished amount. In fact, in its present
petition for review before the Supreme Court, U.P. System has not prayed for the issuance of a
writ of preliminary injunction. Thus, the Court doubts whether such writ is forthcoming. c) a writ of execution had been issued;

The Court honestly believes that if defendants’ petition assailing the Order of this Court dated d) defendant U.P. System’s deposit with DBP was garnished pursuant to a lawful writ
December 31, 2004 granting the motion for the release of the garnished amount was of execution issued by the Court; and
meritorious, the Court of Appeals would have issued a writ of injunction enjoining the same.
Instead, said appellate court not only refused to issue a wit of preliminary injunction prayed for
e) the garnished amount had already been turned over to the plaintiffs and deposited
by U.P. System but denied the petition, as well.54
in their account with DBP.

The UP contended that Judge Yadao thereby effectively reversed the January 30, 2006 order of
The garnished amount, as discussed in the Order dated January 16, 2007, was already owned
Judge Dizon disallowing the withdrawal of the garnished amount until after the decision in the
case would have become final and executory. by the plaintiffs, having been delivered to them by the Deputy Sheriff of this Court pursuant to
par. (c), Section 9, Rule 39 of the 1997 Rules of Civil Procedure. Moreover, the judgment
obligation has already been fully satisfied as per Report of the Deputy Sheriff.
Although the Court issued a TRO on January 24, 2007 to enjoin Judge Yadao and all persons
acting pursuant to her authority from enforcing her order of January 3, 2007, 55 it appears that on
January 16, 2007, or prior to the issuance of the TRO, she had already directed the DBP to Anent the Temporary Restraining Order issued by the Supreme Court, the same has become
functus oficio, having been issued after the garnished amount had been released to the
forthwith release the garnished amount to Stern Builders and dela Cruz; 56 and that DBP had
forthwith complied with the order on January 17, 2007 upon the sheriff’s service of the order of plaintiffs. The judgment debt was released to the plaintiffs on January 17, 2007, while the
Judge Yadao.57 Temporary Restraining Order issued by the Supreme Court was received by this Court on
February 2, 2007. At the time of the issuance of the Restraining Order, the act sought to be
restrained had already been done, thereby rendering the said Order ineffectual.
These intervening developments impelled the UP to file in this Court a supplemental petition on
January 26, 2007,58alleging that the RTC (Judge Yadao) gravely erred in ordering the immediate
release of the garnished amount despite the pendency of the petition for review in this Court. After a careful and thorough study of the arguments advanced by the parties, the Court is of the
considered opinion that there is no legal basis to grant defendant U.P. System’s motion to
redeposit the judgment amount. Granting said motion is not only contrary to law, but it will also
The UP filed a second supplemental petition59 after the RTC (Judge Yadao) denied the UP’s render this Court’s final executory judgment nugatory. Litigation must end and terminate
motion for the redeposit of the withdrawn amount on April 10, 2007, 60 to wit: sometime and somewhere, and it is essential to an effective administration of justice that once a
judgment has become final the issue or cause involved therein should be laid to rest. This
doctrine of finality of judgment is grounded on fundamental considerations of public policy and
This resolves defendant U.P. System’s Urgent Motion to Redeposit Judgment Award praying
sound practice. In fact, nothing is more settled in law than that once a judgment attains finality it
that plaintiffs be directed to redeposit the judgment award to DBP pursuant to the Temporary
thereby becomes immutable and unalterable. It may no longer be modified in any respect, even
Restraining Order issued by the Supreme Court. Plaintiffs opposed the motion and countered
if the modification is meant to correct what is perceived to be an erroneous conclusion of fact or
that the Temporary Restraining Order issued by the Supreme Court has become moot and
law, and regardless of whether the modification is attempted to be made by the court rendering it
academic considering that the act sought to be restrained by it has already been performed.
or by the highest court of the land.
They also alleged that the redeposit of the judgment award was no longer feasible as they have
already spent the same.
WHEREFORE, premises considered, finding defendant U.P. System’s Urgent Motion to
Redeposit Judgment Award devoid of merit, the same is hereby DENIED.
It bears stressing, if only to set the record straight, that this Court did not – in its Order dated
January 3, 2007 (the implementation of which was restrained by the Supreme Court in its
Resolution dated January 24, 2002) – direct that that garnished amount "be deposited with the SO ORDERED.
garnishee bank (Development Bank of the Philippines)". In the first place, there was no need to
order DBP to make such deposit, as the garnished amount was already deposited in the account
Issues
of plaintiffs with the DBP as early as May 13, 2005. What the Court granted in its Order dated
January 3, 2007 was plaintiff’s motion to allow the release of said deposit. It must be recalled
that the Court found plaintiff’s motion meritorious and, at that time, there was no restraining The UP now submits that:
order or preliminary injunction from either the Court of Appeals or the Supreme Court which
I reduction of the UP’s limited budget allocated for the remuneration, job satisfaction and
fulfillment of the best available teachers; that Judge Yadao should have exhibited judicial
courtesy towards the Court due to the pendency of the UP’s petition for review; and that she
THE COURT OF APPEALS COMMITTED GRAVE ERROR IN DISMISSING THE PETITION,
should have also desisted from declaring that the TRO issued by this Court had become functus
ALLOWING IN EFFECT THE GARNISHMENT OF UP FUNDS, WHEN IT RULED THAT
officio.
FUNDS HAVE ALREADY BEEN EARMARKED FOR THE CONSTRUCTION PROJECT; AND
THUS, THERE IS NO NEED FOR FURTHER APPROPRIATIONS.
Lastly, the UP states that the awards of actual damages of ₱ 5,716,729.00 and moral damages
of ₱ 10 million should be reduced, if not entirely deleted, due to its being unconscionable,
II
inequitable and detrimental to public service.

THE COURT OF APPEALS COMMITTED GRAVE ERROR IN ALLOWING GARNISHMENT OF


In contrast, Stern Builders and dela Cruz aver that the petition for review was fatally defective for
A STATE UNIVERSITY’S FUNDS IN VIOLATION OF ARTICLE XIV, SECTION 5(5) OF THE
its failure to mention the other cases upon the same issues pending between the parties (i.e.,
CONSTITUTION.
CA-G.R. No. 77395 and G.R No. 163501); that the UP was evidently resorting to forum
shopping, and to delaying the satisfaction of the final judgment by the filing of its petition for
III review; that the ruling in Commissioner of Public Works v. San Diego had no application
because there was an appropriation for the project; that the UP retained the funds allotted for the
project only in a fiduciary capacity; that the contract price had been meanwhile adjusted to ₱
IN THE ALTERNATIVE, THE UNIVERSITY INVOKES EQUITY AND THE REVIEW POWERS 22,338,553.25, an amount already more than sufficient to cover the judgment award; that the
OF THIS HONORABLE COURT TO MODIFY, IF NOT TOTALLY DELETE THE AWARD OF ₱ UP’s prayer to reduce or delete the award of damages had no factual basis, because they had
10 MILLION AS MORAL DAMAGES TO RESPONDENTS.
been gravely wronged, had been deprived of their source of income, and had suffered untold
miseries, discomfort, humiliation and sleepless years; that dela Cruz had even been constrained
IV to sell his house, his equipment and the implements of his trade, and together with his family
had been forced to live miserably because of the wrongful actuations of the UP; and that the
RTC correctly declared the Court’s TRO to be already functus officio by reason of the withdrawal
THE RTC-BRANCH 80 COMMITTED GRAVE ERROR IN ORDERING THE IMMEDIATE of the garnished amount from the DBP.
RELEASE OF THE JUDGMENT AWARD IN ITS ORDER DATED 3 JANUARY 2007 ON THE
GROUND OF EQUITY AND JUDICIAL COURTESY.
The decisive issues to be considered and passed upon are, therefore:
V
(a) whether the funds of the UP were the proper subject of garnishment in order to satisfy the
judgment award; and (b) whether the UP’s prayer for the deletion of the awards of actual
THE RTC-BRANCH 80 COMMITTED GRAVE ERROR IN ORDERING THE IMMEDIATE damages of ₱ 5,716,729.00, moral damages of ₱ 10,000,000.00 and attorney’s fees of ₱
RELEASE OF THE JUDGMENT AWARD IN ITS ORDER DATED 16 JANUARY 2007 ON THE 150,000.00 plus ₱ 1,500.00 per appearance could be granted despite the finality of the judgment
GROUND THAT PETITIONER UNIVERSITY STILL HAS A PENDING MOTION FOR of the RTC.
RECONSIDERATION OF THE ORDER DATED 3 JANUARY 2007.

Ruling
VI

The petition for review is meritorious.


THE RTC-BRANCH 80 COMMITTED GRAVE ERROR IN NOT ORDERING THE REDEPOSIT
OF THE GARNISHED AMOUNT TO THE DBP IN VIOLATION OF THE CLEAR LANGUAGE OF
THE SUPREME COURT RESOLUTION DATED 24 JANUARY 2007. I.
UP’s funds, being government funds,
are not subject to garnishment
The UP argues that the amount earmarked for the construction project had been purposely set
aside only for the aborted project and did not include incidental matters like the awards of actual
damages, moral damages and attorney’s fees. In support of its argument, the UP cited Article The UP was founded on June 18, 1908 through Act 1870 to provide advanced instruction in
12.2 of the General Construction Agreement, which stipulated that no deductions would be literature, philosophy, the sciences, and arts, and to give professional and technical training to
allowed for the payment of claims, damages, losses and expenses, including attorney’s fees, in deserving students.63 Despite its establishment as a body corporate,64 the UP remains to be a
case of any litigation arising out of the performance of the work. The UP insists that the CA "chartered institution"65 performing a legitimate government function. It is an institution of higher
decision was inconsistent with the rulings in Commissioner of Public Highways v. San learning, not a corporation established for profit and declaring any dividends. 66 In enacting
Diego61 and Department of Agriculture v. NLRC62 to the effect that government funds and Republic Act No. 9500 (The University of the Philippines Charter of 2008), Congress has
properties could not be seized under writs of execution or garnishment to satisfy judgment declared the UP as the national university67 "dedicated to the search for truth and knowledge as
awards. well as the development of future leaders."68

Furthermore, the UP contends that the CA contravened Section 5, Article XIV of the Constitution Irrefragably, the UP is a government instrumentality,69 performing the State’s constitutional
by allowing the garnishment of UP funds, because the garnishment resulted in a substantial mandate of promoting quality and accessible education.70 As a government instrumentality, the
UP administers special funds sourced from the fees and income enumerated under Act No. authorization by Congress or by the head of a government agency in control of the funds, and
1870 and Section 1 of Executive Order No. 714,71 and from the yearly appropriations, to achieve subject to pertinent budgetary laws, rules and regulations.83
the purposes laid down by Section 2 of Act 1870, as expanded in Republic Act No. 9500. 72 All
the funds going into the possession of the UP, including any interest accruing from the deposit of
Indeed, an appropriation by Congress was required before the judgment that rendered the UP
such funds in any banking institution, constitute a "special trust fund," the disbursement of which
liable for moral and actual damages (including attorney’s fees) would be satisfied considering
should always be aligned with the UP’s mission and purpose,73 and should always be subject to
that such monetary liabilities were not covered by the "appropriations earmarked for the said
auditing by the COA.74
project." The Constitution strictly mandated that "(n)o money shall be paid out of the Treasury
except in pursuance of an appropriation made by law."84
Presidential Decree No. 1445 defines a "trust fund" as a fund that officially comes in the
possession of an agency of the government or of a public officer as trustee, agent or
II
administrator, or that is received for the fulfillment of some obligation.75 A trust fund may be
COA must adjudicate private respondents’ claim
utilized only for the "specific purpose for which the trust was created or the funds received." 76
before execution should proceed

The funds of the UP are government funds that are public in character. They include the income
The execution of the monetary judgment against the UP was within the primary jurisdiction of the
accruing from the use of real property ceded to the UP that may be spent only for the attainment
COA. This was expressly provided in Section 26 of Presidential Decree No. 1445, to wit:
of its institutional objectives.77 Hence, the funds subject of this action could not be validly made
the subject of the RTC’s writ of execution or garnishment. The adverse judgment rendered
against the UP in a suit to which it had impliedly consented was not immediately enforceable by Section 26. General jurisdiction. - The authority and powers of the Commission shall extend to
execution against the UP,78 because suability of the State did not necessarily mean its liability. 79 and comprehend all matters relating to auditing procedures, systems and controls, the keeping
of the general accounts of the Government, the preservation of vouchers pertaining thereto for a
period of ten years, the examination and inspection of the books, records, and papers relating to
A marked distinction exists between suability of the State and its liability. As the Court succinctly
those accounts; and the audit and settlement of the accounts of all persons respecting funds or
stated in Municipality of San Fernando, La Union v. Firme:80
property received or held by them in an accountable capacity, as well as the examination, audit,
and settlement of all debts and claims of any sort due from or owing to the Government or any of
A distinction should first be made between suability and liability. "Suability depends on the its subdivisions, agencies and instrumentalities. The said jurisdiction extends to all government-
consent of the state to be sued, liability on the applicable law and the established facts. The owned or controlled corporations, including their subsidiaries, and other self-governing boards,
circumstance that a state is suable does not necessarily mean that it is liable; on the other hand, commissions, or agencies of the Government, and as herein prescribed, including non
it can never be held liable if it does not first consent to be sued. Liability is not conceded by the governmental entities subsidized by the government, those funded by donations through the
mere fact that the state has allowed itself to be sued. When the state does waive its sovereign government, those required to pay levies or government share, and those for which the
immunity, it is only giving the plaintiff the chance to prove, if it can, that the defendant is liable. government has put up a counterpart fund or those partly funded by the government.

Also, in Republic v. Villasor,81 where the issuance of an alias writ of execution directed against It was of no moment that a final and executory decision already validated the claim against the
the funds of the Armed Forces of the Philippines to satisfy a final and executory judgment was UP. The settlement of the monetary claim was still subject to the primary jurisdiction of the COA
nullified, the Court said: despite the final decision of the RTC having already validated the claim. 85 As such, Stern
Builders and dela Cruz as the claimants had no alternative except to first seek the approval of
the COA of their monetary claim.
xxx The universal rule that where the State gives its consent to be sued by private parties either
by general or special law, it may limit claimant’s action "only up to the completion of proceedings
anterior to the stage of execution" and that the power of the Courts ends when the judgment is On its part, the RTC should have exercised utmost caution, prudence and judiciousness in
rendered, since government funds and properties may not be seized under writs of execution or dealing with the motions for execution against the UP and the garnishment of the UP’s funds.
garnishment to satisfy such judgments, is based on obvious considerations of public policy. The RTC had no authority to direct the immediate withdrawal of any portion of the garnished
Disbursements of public funds must be covered by the corresponding appropriation as required funds from the depository banks of the UP. By eschewing utmost caution, prudence and
by law. The functions and public services rendered by the State cannot be allowed to be judiciousness in dealing with the execution and garnishment, and by authorizing the withdrawal
paralyzed or disrupted by the diversion of public funds from their legitimate and specific objects, of the garnished funds of the UP, the RTC acted beyond its jurisdiction, and all its orders and
as appropriated by law. issuances thereon were void and of no legal effect, specifically: (a) the order Judge Yadao
issued on January 3, 2007 allowing Stern Builders and dela Cruz to withdraw the deposited
garnished amount; (b) the order Judge Yadao issued on January 16, 2007 directing DBP to
The UP correctly submits here that the garnishment of its funds to satisfy the judgment awards
forthwith release the garnish amount to Stern Builders and dela Cruz; (c) the sheriff’s report of
of actual and moral damages (including attorney’s fees) was not validly made if there was no
January 17, 2007 manifesting the full satisfaction of the writ of execution; and (d) the order of
special appropriation by Congress to cover the liability. It was, therefore, legally unwarranted for
April 10, 2007 deying the UP’s motion for the redeposit of the withdrawn amount. Hence, such
the CA to agree with the RTC’s holding in the order issued on April 1, 2003 that no appropriation
orders and issuances should be struck down without exception.
by Congress to allocate and set aside the payment of the judgment awards was necessary
because "there (were) already an appropriations (sic) earmarked for the said project." 82 The CA
and the RTC thereby unjustifiably ignored the legal restriction imposed on the trust funds of the Nothing extenuated Judge Yadao’s successive violations of Presidential Decree No. 1445. She
Government and its agencies and instrumentalities to be used exclusively to fulfill the purposes was aware of Presidential Decree No. 1445, considering that the Court circulated to all judges its
for which the trusts were created or for which the funds were received except upon express Administrative Circular No. 10-2000,86 issued on October 25, 2000, enjoining them "to observe
utmost caution, prudence and judiciousness in the issuance of writs of execution to satisfy
money judgments against government agencies and local government units" precisely in order 3. Property held for public purposes is not subject to execution merely because it is temporarily
to prevent the circumvention of Presidential Decree No. 1445, as well as of the rules and used for private purposes. If the public use is wholly abandoned, such property becomes subject
procedures of the COA, to wit: to execution.

In order to prevent possible circumvention of the rules and procedures of the This Administrative Circular shall take effect immediately and the Court Administrator shall see
Commission on Audit, judges are hereby enjoined to observe utmost caution, prudence to it that it is faithfully implemented.
and judiciousness in the issuance of writs of execution to satisfy money judgments
against government agencies and local government units.
Although Judge Yadao pointed out that neither the CA nor the Court had issued as of then any
writ of preliminary injunction to enjoin the release or withdrawal of the garnished amount, she did
Judges should bear in mind that in Commissioner of Public Highways v. San Diego (31 SCRA not need any writ of injunction from a superior court to compel her obedience to the law. The
617, 625 1970), this Court explicitly stated: Court is disturbed that an experienced judge like her should look at public laws like Presidential
Decree No. 1445 dismissively instead of loyally following and unquestioningly implementing
them. That she did so turned her court into an oppressive bastion of mindless tyranny instead of
"The universal rule that where the State gives its consent to be sued by private parties either by
having it as a true haven for the seekers of justice like the UP.
general or special law, it may limit claimant’s action ‘only up to the completion of proceedings
anterior to the stage of execution’ and that the power of the Court ends when the judgment is
rendered, since government funds and properties may not be seized under writs of execution or III
garnishment to satisfy such judgments, is based on obvious considerations of public policy. Period of appeal did not start without effective
Disbursements of public funds must be covered by the corresponding appropriation as required service of decision upon counsel of record;
by law. The functions and public services rendered by the State cannot be allowed to be Fresh-period rule announced in
paralyzed or disrupted by the diversion of public funds from their legitimate and specific objects, Neypes v. Court of Appeals
as appropriated by law. can be given retroactive application

Moreover, it is settled jurisprudence that upon determination of State liability, the The UP next pleads that the Court gives due course to its petition for review in the name of
prosecution, enforcement or satisfaction thereof must still be pursued in accordance with equity in order to reverse or modify the adverse judgment against it despite its finality. At stake in
the rules and procedures laid down in P.D. No. 1445, otherwise known as the Government the UP’s plea for equity was the return of the amount of ₱ 16,370,191.74 illegally garnished from
Auditing Code of the Philippines (Department of Agriculture v. NLRC, 227 SCRA 693, 701- its trust funds. Obstructing the plea is the finality of the judgment based on the supposed
02 1993 citing Republic vs. Villasor, 54 SCRA 84 1973). All money claims against the tardiness of UP’s appeal, which the RTC declared on September 26, 2002. The CA upheld the
Government must first be filed with the Commission on Audit which must act upon it declaration of finality on February 24, 2004, and the Court itself denied the UP’s petition for
within sixty days. Rejection of the claim will authorize the claimant to elevate the matter review on that issue on May 11, 2004 (G.R. No. 163501). The denial became final on November
to the Supreme Court on certiorari and in effect, sue the State thereby (P.D. 1445, 12, 2004.
Sections 49-50).
It is true that a decision that has attained finality becomes immutable and unalterable, and
However, notwithstanding the rule that government properties are not subject to levy and cannot be modified in any respect,87 even if the modification is meant to correct erroneous
execution unless otherwise provided for by statute (Republic v. Palacio, 23 SCRA 899 1968; conclusions of fact and law, and whether the modification is made by the court that rendered it or
Commissioner of Public Highways v. San Diego, supra) or municipal ordinance (Municipality of by this Court as the highest court of the land.88 Public policy dictates that once a judgment
Makati v. Court of Appeals, 190 SCRA 206 1990), the Court has, in various instances, becomes final, executory and unappealable, the prevailing party should not be deprived of the
distinguished between government funds and properties for public use and those not held for fruits of victory by some subterfuge devised by the losing party. Unjustified delay in the
public use. Thus, in Viuda de Tan Toco v. Municipal Council of Iloilo (49 Phil 52 1926, the Court enforcement of such judgment sets at naught the role and purpose of the courts to resolve
ruled that "where property of a municipal or other public corporation is sought to be subjected to justiciable controversies with finality.89Indeed, all litigations must at some time end, even at the
execution to satisfy judgments recovered against such corporation, the question as to whether risk of occasional errors.
such property is leviable or not is to be determined by the usage and purposes for which it is
held." The following can be culled from Viuda de Tan Toco v. Municipal Council of Iloilo:
But the doctrine of immutability of a final judgment has not been absolute, and has admitted
several exceptions, among them: (a) the correction of clerical errors; (b) the so-called nunc pro
1. Properties held for public uses – and generally everything held for governmental tunc entries that cause no prejudice to any party; (c) void judgments; and (d) whenever
purposes – are not subject to levy and sale under execution against such corporation. circumstances transpire after the finality of the decision that render its execution unjust and
The same rule applies to funds in the hands of a public officer and taxes due to a inequitable.90 Moreover, in Heirs of Maura So v. Obliosca,91 we stated that despite the absence
municipal corporation. of the preceding circumstances, the Court is not precluded from brushing aside procedural
norms if only to serve the higher interests of justice and equity. Also, in Gumaru v. Quirino State
College,92 the Court nullified the proceedings and the writ of execution issued by the RTC for the
2. Where a municipal corporation owns in its proprietary capacity, as distinguished from its
reason that respondent state college had not been represented in the litigation by the Office of
public or government capacity, property not used or used for a public purpose but for quasi-
the Solicitor General.
private purposes, it is the general rule that such property may be seized and sold under
execution against the corporation.
We rule that the UP’s plea for equity warrants the Court’s exercise of the exceptional power to that the filing of a motion for reconsideration interrupted the running of the period for filing the
disregard the declaration of finality of the judgment of the RTC for being in clear violation of the appeal; and that the period resumed upon notice of the denial of the motion for reconsideration.
UP’s right to due process. For that reason, the CA and the RTC might not be taken to task for strictly adhering to the rule
then prevailing.
Both the CA and the RTC found the filing on June 3, 2002 by the UP of the notice of appeal to
be tardy. They based their finding on the fact that only six days remained of the UP’s However, equity calls for the retroactive application in the UP’s favor of the fresh-period rule that
reglementary 15-day period within which to file the notice of appeal because the UP had filed a the Court first announced in mid-September of 2005 through its ruling in Neypes v. Court of
motion for reconsideration on January 16, 2002 vis-à-vis the RTC’s decision the UP received on Appeals,98 viz:
January 7, 2002; and that because the denial of the motion for reconsideration had been served
upon Atty. Felimon D. Nolasco of the UPLB Legal Office on May 17, 2002, the UP had only until
To standardize the appeal periods provided in the Rules and to afford litigants fair opportunity to
May 23, 2002 within which to file the notice of appeal.
appeal their cases, the Court deems it practical to allow a fresh period of 15 days within which to
file the notice of appeal in the Regional Trial Court, counted from receipt of the order dismissing
The UP counters that the service of the denial of the motion for reconsideration upon Atty. a motion for a new trial or motion for reconsideration.
Nolasco was defective considering that its counsel of record was not Atty. Nolasco of the UPLB
Legal Office but the OLS in Diliman, Quezon City; and that the period of appeal should be
The retroactive application of the fresh-period rule, a procedural law that aims "to regiment or
reckoned from May 31, 2002, the date when the OLS received the order. The UP submits that
make the appeal period uniform, to be counted from receipt of the order denying the motion for
the filing of the notice of appeal on June 3, 2002 was well within the reglementary period to
new trial, motion for reconsideration (whether full or partial) or any final order or resolution,"99 is
appeal.
impervious to any serious challenge. This is because there are no vested rights in rules of
procedure.100 A law or regulation is procedural when it prescribes rules and forms of procedure in
We agree with the submission of the UP. order that courts may be able to administer justice.101 It does not come within the legal
conception of a retroactive law, or is not subject of the general rule prohibiting the retroactive
operation of statues, but is given retroactive effect in actions pending and undetermined at the
Firstly, the service of the denial of the motion for reconsideration upon Atty. Nolasco of the UPLB
time of its passage without violating any right of a person who may feel that he is adversely
Legal Office was invalid and ineffectual because he was admittedly not the counsel of record of
affected.
the UP. The rule is that it is on the counsel and not the client that the service should be made.93

We have further said that a procedural rule that is amended for the benefit of litigants in
That counsel was the OLS in Diliman, Quezon City, which was served with the denial only on
furtherance of the administration of justice shall be retroactively applied to likewise favor actions
May 31, 2002. As such, the running of the remaining period of six days resumed only on June 1,
then pending, as equity delights in equality.102 We may even relax stringent procedural rules in
2002,94 rendering the filing of the UP’s notice of appeal on June 3, 2002 timely and well within
order to serve substantial justice and in the exercise of this Court’s equity jurisdiction. 103 Equity
the remaining days of the UP’s period to appeal.
jurisdiction aims to do complete justice in cases where a court of law is unable to adapt its
judgments to the special circumstances of a case because of the inflexibility of its statutory or
Verily, the service of the denial of the motion for reconsideration could only be validly made upon legal jurisdiction.104
the OLS in Diliman, and no other. The fact that Atty. Nolasco was in the employ of the UP at the
UPLB Legal Office did not render the service upon him effective. It is settled that where a party
It is cogent to add in this regard that to deny the benefit of the fresh-period rule to the UP would
has appeared by counsel, service must be made upon such counsel.95 Service on the party or
amount to injustice and absurdity – injustice, because the judgment in question was issued on
the party’s employee is not effective because such notice is not notice in law. 96 This is clear
November 28, 2001 as compared to the judgment in Neypes that was rendered in 1998;
enough from Section 2, second paragraph, of Rule 13, Rules of Court, which explicitly states
absurdity, because parties receiving notices of judgment and final orders issued in the year 1998
that: "If any party has appeared by counsel, service upon him shall be made upon his counsel or
would enjoy the benefit of the fresh-period rule but the later rulings of the lower courts like that
one of them, unless service upon the party himself is ordered by the court. Where one counsel
herein would not.105
appears for several parties, he shall only be entitled to one copy of any paper served upon him
by the opposite side." As such, the period to appeal resumed only on June 1, 2002, the date
following the service on May 31, 2002 upon the OLS in Diliman of the copy of the decision of the Consequently, even if the reckoning started from May 17, 2002, when Atty. Nolasco received the
RTC, not from the date when the UP was notified.97 denial, the UP’s filing on June 3, 2002 of the notice of appeal was not tardy within the context of
the fresh-period rule. For the UP, the fresh period of 15-days counted from service of the denial
of the motion for reconsideration would end on June 1, 2002, which was a Saturday. Hence, the
Accordingly, the declaration of finality of the judgment of the RTC, being devoid of factual and
UP had until the next working day, or June 3, 2002, a Monday, within which to appeal,
legal bases, is set aside.
conformably with Section 1 of Rule 22, Rules of Court, which holds that: "If the last day of the
period, as thus computed, falls on a Saturday, a Sunday, or a legal holiday in the place where
Secondly, even assuming that the service upon Atty. Nolasco was valid and effective, such that the court sits, the time shall not run until the next working day."
the remaining period for the UP to take a timely appeal would end by May 23, 2002, it would still
not be correct to find that the judgment of the RTC became final and immutable thereafter due to
the notice of appeal being filed too late on June 3, 2002.

In so declaring the judgment of the RTC as final against the UP, the CA and the RTC applied the
rule contained in the second paragraph of Section 3, Rule 41 of the Rules of Court to the effect
IV Here, the decision of the RTC justified the grant of actual and moral damages, and attorney’s
Awards of monetary damages, fees in the following terse manner, viz:
being devoid of factual and legal bases,
did not attain finality and should be deleted
xxx The Court is not unmindful that due to defendants’ unjustified refusal to pay their outstanding
obligation to plaintiff, the same suffered losses and incurred expenses as he was forced to re-
Section 14 of Article VIII of the Constitution prescribes that express findings of fact and of law mortgage his house and lot located in Quezon City to Metrobank (Exh. "CC") and BPI Bank just
should be made in the decision rendered by any court, to wit: to pay its monetary obligations in the form of interest and penalties incurred in the course of the
construction of the subject project.109
Section 14. No decision shall be rendered by any court without expressing therein clearly and
distinctly the facts and the law on which it is based. The statement that "due to defendants’ unjustified refusal to pay their outstanding obligation to
plaintiff, the same suffered losses and incurred expenses as he was forced to re-mortgage his
house and lot located in Quezon City to Metrobank (Exh. "CC") and BPI Bank just to pay its
No petition for review or motion for reconsideration of a decision of the court shall be refused
monetary obligations in the form of interest and penalties incurred in the course of the
due course or denied without stating the legal basis therefor.
construction of the subject project" was only a conclusion of fact and law that did not comply with
the constitutional and statutory prescription. The statement specified no detailed expenses or
Implementing the constitutional provision in civil actions is Section 1 of Rule 36, Rules of Court, losses constituting the ₱ 5,716,729.00 actual damages sustained by Stern Builders in relation to
viz: the construction project or to other pecuniary hardships. The omission of such expenses or
losses directly indicated that Stern Builders did not prove them at all, which then contravened
Article 2199, Civil Code, the statutory basis for the award of actual damages, which entitled a
Section 1. Rendition of judgments and final orders. — A judgment or final order determining the
person to an adequate compensation only for such pecuniary loss suffered by him as he has
merits of the case shall be in writing personally and directly prepared by the judge, stating clearly duly proved. As such, the actual damages allowed by the RTC, being bereft of factual support,
and distinctly the facts and the law on which it is based, signed by him, and filed with the clerk of were speculative and whimsical. Without the clear and distinct findings of fact and law, the
the court. (1a)
award amounted only to an ipse dixit on the part of the RTC, 110 and did not attain finality.

The Constitution and the Rules of Court apparently delineate two main essential parts of a There was also no clear and distinct statement of the factual and legal support for the award of
judgment, namely: the body and the decretal portion. Although the latter is the controlling
moral damages in the substantial amount of ₱ 10,000,000.00. The award was thus also
part,106 the importance of the former is not to be lightly regarded because it is there where the speculative and whimsical. Like the actual damages, the moral damages constituted another
court clearly and distinctly states its findings of fact and of law on which the decision is based. judicial ipse dixit, the inevitable consequence of which was to render the award of moral
To state it differently, one without the other is ineffectual and useless. The omission of either
damages incapable of attaining finality. In addition, the grant of moral damages in that manner
inevitably results in a judgment that violates the letter and the spirit of the Constitution and the contravened the law that permitted the recovery of moral damages as the means to assuage
Rules of Court. "physical suffering, mental anguish, fright, serious anxiety, besmirched reputation, wounded
feelings, moral shock, social humiliation, and similar injury."111 The contravention of the law was
The term findings of fact that must be found in the body of the decision refers to statements of manifest considering that Stern Builders, as an artificial person, was incapable of experiencing
fact, not to conclusions of law.107 Unlike in pleadings where ultimate facts alone need to be pain and moral sufferings.112 Assuming that in granting the substantial amount of ₱
stated, the Constitution and the Rules of Court require not only that a decision should state the 10,000,000.00 as moral damages, the RTC might have had in mind that dela Cruz had himself
ultimate facts but also that it should specify the supporting evidentiary facts, for they are what suffered mental anguish and anxiety. If that was the case, then the RTC obviously disregarded
are called the findings of fact. his separate and distinct personality from that of Stern Builders. 113 Moreover, his moral and
emotional sufferings as the President of Stern Builders were not the sufferings of Stern Builders.
Lastly, the RTC violated the basic principle that moral damages were not intended to enrich the
The importance of the findings of fact and of law cannot be overstated. The reason and purpose plaintiff at the expense of the defendant, but to restore the plaintiff to his status quo ante as
of the Constitution and the Rules of Court in that regard are obviously to inform the parties why much as possible. Taken together, therefore, all these considerations exposed the substantial
they win or lose, and what their rights and obligations are. Only thereby is the demand of due amount of ₱ 10,000,000.00 allowed as moral damages not only to be factually baseless and
process met as to the parties. As Justice Isagani A. Cruz explained in Nicos Industrial legally indefensible, but also to be unconscionable, inequitable and unreasonable.
Corporation v. Court of Appeals:108

Like the actual and moral damages, the ₱ 150,000.00, plus ₱ 1,500.00 per appearance, granted
It is a requirement of due process that the parties to a litigation be informed of how it was as attorney’s fees were factually unwarranted and devoid of legal basis. The general rule is that
decided, with an explanation of the factual and legal reasons that led to the conclusions of the a successful litigant cannot recover attorney’s fees as part of the damages to be assessed
court. The court cannot simply say that judgment is rendered in favor of X and against Y and just against the losing party because of the policy that no premium should be placed on the right to
leave it at that without any justification whatsoever for its action. The losing party is entitled to litigate.114 Prior to the effectivity of the present Civil Code, indeed, such fees could be recovered
know why he lost, so he may appeal to a higher court, if permitted, should he believe that the only when there was a stipulation to that effect. It was only under the present Civil Code that the
decision should be reversed. A decision that does not clearly and distinctly state the facts and right to collect attorney’s fees in the cases mentioned in Article 2208115 of the Civil Code came to
the law on which it is based leaves the parties in the dark as to how it was reached and is be recognized.116 Nonetheless, with attorney’s fees being allowed in the concept of actual
especially prejudicial to the losing party, who is unable to pinpoint the possible errors of the court damages,117 their amounts must be factually and legally justified in the body of the decision and
for review by a higher tribunal. not stated for the first time in the decretal portion.118 Stating the amounts only in the dispositive
portion of the judgment is not enough;119 a rendition of the factual and legal justifications for them
must also be laid out in the body of the decision.120

That the attorney’s fees granted to the private respondents did not satisfy the foregoing
requirement suffices for the Court to undo them.121 The grant was ineffectual for being contrary
to law and public policy, it being clear that the express findings of fact and law were intended to
bring the case within the exception and thereby justify the award of the attorney’s fees. Devoid of
such express findings, the award was a conclusion without a premise, its basis being improperly
left to speculation and conjecture.122

Nonetheless, the absence of findings of fact and of any statement of the law and jurisprudence
on which the awards of actual and moral damages, as well as of attorney’s fees, were based
was a fatal flaw that invalidated the decision of the RTC only as to such awards. As the Court
declared in Velarde v. Social Justice Society,123 the failure to comply with the constitutional
requirement for a clear and distinct statement of the supporting facts and law "is a grave abuse
of discretion amounting to lack or excess of jurisdiction" and that "(d)ecisions or orders issued in
careless disregard of the constitutional mandate are a patent nullity and must be struck down as
void."124 The other item granted by the RTC (i.e., ₱ 503,462.74) shall stand, subject to the action
of the COA as stated herein.

WHEREFORE, the Court GRANTS the petition for review on certiorari; REVERSES and SETS
ASIDE the decision of the Court of Appeals under review; ANNULS the orders for the
garnishment of the funds of the University of the Philippines and for the release of the garnished
amount to Stern Builders Corporation and Servillano dela Cruz; and DELETES from the decision
of the Regional Trial Court dated November 28, 2001 for being void only the awards of actual
damages of ₱ 5,716,729.00, moral damages of ₱ 10,000,000.00, and attorney's fees of ₱
150,000.00, plus ₱ 1,500.00 per appearance, in favor of Stern Builders Corporation and
Servillano dela Cruz.

The Court ORDERS Stem Builders Corporation and Servillano dela Cruz to redeposit the
amount of ₱ 16,370,191.74 within 10 days from receipt of this decision.

Costs of suit to be paid by the private respondents.

SO ORDERED.
G.R. No. 85284 February 28, 1990 2. Similarly, the shares of stock in Bacolod Real Estate Development
Corporation appealing under PERSONAL PROPERTY on page two of
Annex A of the complaint t are ' carried not in the names of any of the
REPUBLIC OF THE PHILIPPINES, petitioner
defendants, but in the name of Marsteel Consolidated Inc. and were
vs.
acquired under the circumstances averred more in detail in the
SANDIGANBAYAN, Third Division, SIMPLICIO A. PALANCA in his own behalf as a
accompanying Answer in Intervention by reason of which detail shares
stockholder of Bacolod Real Estate Development Corporation (BREDCO), and other
should not be involved in the present action.
stockholders similarly situated, respondents.

3. If intervention is allowed, intervenors are prepared to prove that if ever


RESOLUTION
any of the defendants through Marsteel Consolidated, Inc. and Marsteel
Corporation came to have any interest in Bacolod Real Estate Development
PADILLA, J.: Corporation, it was only by way of accommodation on the part of BREDCO
stockholders who transferred their shareholdings aggregating 70% of the
subscribed capital to enable Marsteel Consolidated to secure adequate
This is a petition for certiorari to annul and set aside the resolution of the Sandiganbayan (Third financing for the reclamation and port development project . 4
Division), dated 3 June 1988, granting the private respondents' motion to intervene in Civil Case
No. 0025 and admitting their answer in intervention, as well as its resolution, dated 25 August
1988, denying the petitioner's motion for reconsideration; PROHIBITION to order the respondent The foregoing allegations were further expanded and elaborated in the private respondents'
court to cease and desist from proceeding with the intervention filed with it; and alternatively, Answer in Intervention.
mandamus to compel the respondent court to dismiss the intervention case.
On 2 December 1987, petitioner filed its Reply 5 to Answer In Intervention, while private
The antecedents are as follows: respondents filed a "Rejoinder to Reply With Motion To Release BREDCO Lots 6 and also a
"Motion To Calendar For Hearing" the motion to release BREDCO lots. 7
On 29 July 1987, the Republic of the Philippines, as Plaintiff, through its governmental
instrumentality the Presidential Commission on Good Government (PCGG) filed with the On 22 January 1988, respondent court promulgated a resolution 8 holding in abeyance action on
respondent Sandiganbayan a complaint against Ferdinand E. Marcos, et al. for reconveyance, the private respondents' "Rejoinder to Reply with Motion to Release BREDCO lots", and set the
reversion, accounting, restitution and damages, docketed therein as Civil Case No. 0025 (PCGG Motion for Leave to Intervene for hearing on 2 February 1988.
No. 26). 1
On 11 March 1988, respondent court issued an order 9 giving petitioner fifteen (1 5) days from
On or about 3 September 1987, before the said Civil Case No. 0025 could be set for hearing, 11 March 1988 within which to file its opposition and/or comment on the motion to intervene and
private respondent Simplicio A. Palanca in his own behalf as a stockholder of Bacolod Real giving the private respondents in turn ten (10) days within which to file their reply thereto.
Estate Development Corporation (BREDCO) and other stockholders similarly situated, filed with
the respondent Sandiganbayan a "Motion For Leave To Intervene" 2 attaching thereto their
On 23 March 1988, petitioner filed its Motion to Dismiss "Answer In Intervention," on the grounds
"Answer in Intervention ." 3
that; (1) respondent court lacks jurisdiction and (2) intervenors have no legal interest in the
matter in litigation, 10 which the private respondents opposed. 11
In their motion, private respondents alleged that they be —
On 6 June 1988, respondent court promulgated a Resolution dated 3 June 1988 12 granting the
... allowed to intervene in the present action and to file the Answer in private respondents' motion to intervene and admitting their Answer in Intervention.
intervention hereto attached as Annex 'A', the said stockholders having a
legal interest in the matter in litigation and in the disposition of the properties
Petitioner moved for reconsideration but this was denied by respondent court in its resolution of
listed in Annex 'A' of the Complaint as BREDCO LOTS and shares of stock
25 August 1989.13
in Bacolod Real Estate Development Corporation.

Hence, the instant petition.


In justification, it is further respectfully alleged that.

The petitioner, through the Solicitor General, contends that in issuing the questioned resolutions
1. Close examination of the Complaint, in particular par. 12 thereto under 'V.
granting the Motion to Intervene and admitting the Answer-in-Intervention, respondent
SPECIFIC AVERMENTS OF DEFENDANTS' ILLEGAL ACTS', makes no
Sandiganbayan acted in contravention of a national or public policy embedded in Executive
mention at all about BREDCO being the subject of any anomalous
Order Nos. 1, 2, 4 and related issuances, or otherwise acted in a way not in accord with law or
transaction engaged in by any of the defendants, in consequence of which
with the applicable decisions of this Court, because:
the listed BREDCO lots could have been gotten illegally. It is to be
observed, on the other hand, that the titles mentioned in aforesaid Annex of
the complaint covering the lots in question are not registered in the names (a) Petitioner, being the sovereign state, cannot be sued without its consent, and the Intervention
of any of the defendants but in the name of Bacolod Real Estate is, in legal effect, a suit or counter- suit against the sovereign state, the Republic of the
Development Corporation. Philippines;
(b) The cause of action of intervenors does not fall within the jurisdiction of the Sandiganbayan Sandiganbayan's exclusive and original jurisdiction, subject to review on
as expressly spelled out in P.D. No. 1606 and Executive Order No. 14; certiorari exclusively by the Supreme Court. (emphasis supplied)

16
(c) Intervenors have no legal interest in the matter in litigation, and the subject matter is not in In reiterating the aforequoted ruling in six (6) subsequent cases which were decided jointly,
custodia legis of respondent court; and again, the Court held that-

(d) Intervenors' claims, as contained in their Motion for Intervention and Answer-in-Intervention, ... the exclusive jurisdiction conferred on the Sandiganbayan would
are claims between and/or among Ferdinand and Imelda Marcos and their cronies, i.e., evidently extend not only to the principal causes of action, i.e., the recovery
"members of their immediate family close relatives, subordinates, and/or business associates, of alleged ill-gotten wealth, but also to 'all incidents arising from, incidental
dummies, agents and nominees" and are cognizable not by respondent court but by the regular to, or related to, such cases,' such as the dispute over the sale of the
courts or other for a Even if there would be multiple litigations, as among themselves, the legal shares, the propriety of the issuance of ancillary writs or provisional
effect remains, i.e., that there is only one case filed by the Republic against the named remedies relative thereto, the sequestration thereof, which may not be made
defendants in Civil Case No. 0025, grounded on causes of action entirely distinct from any the subject of separate actions or proceedings in another forum.
cause of action which intervenors may have against Mr. Marcos and his cronies.
Intervention is not an independent action, but is ancillary and supplemental to an existing
The petition is not impressed with merit. litigation. 17 Hence, the private respondents' action for intervention in Civil Case No. 0025, not
being an independent action, is merely incidental to, or related to, the said civil case. Since the
respondent Sandiganbayan has the exclusive and original jurisdiction over Civil Case No. 0025,
The Rules of Court permit an aggrieved party, generally, to take a cause and apply for relief with
it has likewise original and exclusive jurisdiction over the private respondents' action for
the appellate courts by way of either of two distinct and dissimilar modes through the broad
intervention therein.
process of appeal or the limited special civil action of certiorari. An appeal brings up for
review errors of judgment committed by a court of competent jurisdiction over the subject of the
suit or the persons of the parties or any such error committed by the court in the exercise of its Now, considering that respondent Sandiganbayan has jurisdiction not only over Civil Case No.
jurisdiction amounting to nothing more than an error of judgment. On the other hand, the writ of 0025 but also over the private respondents' action for intervention, any error or irregularity that it
certiorari issues for the correction of errors of jurisdiction only or grave abuse of discretion may have committed in rendering its questioned resolutions, in the exercise of its jurisdiction,
amounting to lack or excess of jurisdiction. The writ of certiorari cannot legally be used for any amounts to an error of judgment, which is not correctable in the present petition for certiorari but
other purpose. In terms of its function, the writ of certiorari serves to keep a lower court within by appeal.
the bounds of its jurisdiction or to prevent it from committing such a grave abuse of discretion
amounting to excess of jurisdiction or to relieve parties from arbitrary acts of courts — acts which
Accordingly, this case may be dismissed outright without the Court having to pass upon the
courts have no power or authority in law to perform. 14
other issues raised in the petition. However, considering that the litigation below is of great
public interest and involves a matter of public policy, the Court has decided to review the other
Hence, the main issue to be resolved in the present case, which is principally a petition for errors allegedly committed by respondent court in rendering its questioned resolutions.
certiorari to annul and set aside the questioned resolutions of respondent court is, whether or not
the Sandiganbayan has jurisdiction over the action for intervention, or if it has, whether
In this jurisdiction, the law on "intervention" is found in the Rules of Court. 18 Thus, a person
respondent court acted with grave abuse of discretion amounting to lack or excess of its
may, before or during a trial, be permitted by the court, in its discretion, to intervene in an action,
jurisdiction in rendering the questioned resolutions.
if he has legal interest in the matter in litigation, or in the success of either of the parties or an
interest against both, or when he is so situated as to be adversely affected by a distribution or
In the present case, petitioner merely contends that the cause of action of intervenors does not other disposition of property in the custody of the court or of an officer thereof. 19
fall within the jurisdiction of the Sandiganbayan as expressly spelled out in Presidential Decree
No. 1606 and Executive Order No. 14; it does not claim that respondent court committed grave
The Court is not impressed with the contention of petitioner that the intervenors have no legal
abuse of discretion amounting to lack or excess of its jurisdiction in rendering the questioned
interest in the matter in litigation. In this connection, it would suffice to quote what the
resolutions.
respondent court said in holding that the intervenors have a legal interest in the matter in
litigation. Thus —
The jurisdiction of the Sandiganbayan has already been settled in Presidential Commission on
Good Government vs. Hon. Emmanuel G. Penal, etc., et al. 15 where the Court held that —
Has Palanca shown a proper case for intervention by him and his co-
stockholders who are similarly situated as he is?
... Under Section 2 of the President's Executive Order No. 14 issued on May
7, 1986, all cases of the Commission regarding 'the funds, Moneys, Assets,
A narration of the pertinent facts alleged by Palanca and the plaintiff
and Properties Illegally Acquired or I Misappropriated by Former President
indicates the answer.
Ferdinand Marcos, Mrs. Imelda Romualdez Marcos, their Close Relatives,
Subordinates, Business Associates, Dummies, Agents, or Nominees
whether civil or criminal, are lodged within the 'exclusive and original In 1961, BREDCO was awarded by Bacolod City a
jurisdiction of the Sandiganbayan' and all incidents arising from, incidental contract to undertake the reclamation and port
to, or related to, such cases necessarily fall likewise under the development of the city. As of 1975, a sizeable portion
of land had already been reclaimed from the sea and interest in the property which is the subject of litigation,
corresponding torrens titles issued in BREDCO's name. and this without particular regard to the value of the
property or the right claimed therein. A third party may
intervene in a sequestration suit involving title to
In that year, BREDCO engaged MARSTEEL as a
personal property, and have his claims to the
contractor to complete the project with power to
possession of the property vindicated therein So, in an
negotiate in its name or jointly and/or severally with
action for possession of real or personal property, an
BREDCO for loans to finance the reclamation and port
intervenor may be admitted on the ground that he is an
development, and to mortgage all reclaimed lots and
owner thereof, either to assist in the defense, or to
other assets of the project as security. For its services,
claim the property for himself, or to obtain some other
MARSTEEL shall receive 65% of the excess of all
relief germane to the action.' (59 Am Jur 2d, Parties,
revenues over all disbursements. Accordingly,
Sec. 152, p. 585,
BREDCO conveyed to MARSTEEL 65% of each lot
already reclaimed and that to be reclaimed.
Secondly, the same Section 2, Rule 12, further provides that intervention by
a person may be permitted 'when he is so situated as to be adversely
In 1977, MARSTEEL assigned to MCI, which owned
affected by a distribution or other disposition of property in the custody of
100% of its capital stock, all its rights, interests,
the court or of an officer thereof.' On this point, the Supreme Court
obligations, and undertakings in the project. To enable
observed:
MCI to expand its base of negotiation for loans needed
in the reclamation and port development the BREDCO
stockholders transferred to MCI their respective shares We shall now speak of the case where the stranger
of stock amounting to 70% of the capital stock of desires to intervene for the purpose of asserting a
BREDCO. In return, they 'shall be entitled to a share of property right in the res, or thing, which is the subject-
35% in excess of all revenues over all disbursements of matter of the ligitation, without becoming a formal
the projects,' it being understood that payment of the plaintiff or defendant, and without acquiring the control
corresponding share shall be due to BREDCO over the course of a litigation, which is conceded to the
stockholders as owners of existing interests in the main actions (sic) therein. The mode of intervention to
project, regardless of the fact that by implementation of which reference is now made is denominated in equity
this AGREEMENT, they ceased to be stockholders of procedure the intervention pro interesse suo and is
BREDCO. somewhat analogous to the trial of a right of property in
an action of law, its purpose being to enable a person
whose property gets into the clutches of a court, in a
In September 1986, the Presidential Commission on
controversy between others, to go into court and to
Good Government (PCGG) sequestered all assets,
procure it or its proceeds to be surrendered to him. It
properties, records and documents' of MARSTEEL,
often happens that a person who really owns property,
MCI, and BREDCO'. In July 1987, the complaint at bar
or has a superior lien or other interest in it, sees a
was filed and expanded in March 1988. The pleadings,
litigation spring up between others who assert rights in
original and expanded, allege that the defendants,
or concerning it. If the court takes possession of the
acting singly or collectively, amassed ill-gotten wealth
res, or otherwise gets jurisdiction over it in such a
listed in Annex 'A' thereof, among which are the
controversy, the real owner is not compelled to stand
BREDCO lots and shares of stock, and pray that the ill-
Idly by and see the property disposed of without
gotten wealth be reconveyed to the plaintiff, plus
asserting his rights. Though it be granted that the
damages. Significantly, however, the bodies of the
litigation would not be technically binding on him,
complaints do not mention anything about BREDCO, its
because of his not being a party, yet it might well
project, lots, and stocks, nor about MCI.
happen that complications would ensue whereby his
rights would be materially prejudiced. For instance, the
Under these alleged facts, Palanca has established a proper case for subject-matter of the litigation might consist of a fund to
intervention. Firstly, he and his co-stockholders have a legal interest in the he distributed, and the conditions might be such that if it
matter in litigation, namely, their 70% of the capital stock of BREDCO, which were turned over to the particular litigant who should
they transferred to MCI by way of alleged accommodation, or its equivalent appear to have the better right in the original action, the
of 35% of the excess of all revenues over all disbursements, to which they person really having a superior title might be left without
are entitled 'as owners of existing interests in the project.' Section 2, Rule redress. Accordingly provision is made whereby
12, Revised Rules of Court, provides that a person may be permitted 'to persons who have not been joined as parties in the
intervene in an action, if he has legal interest in the matter in litigation.' original proceedings may intervene and assert a right
antagonistic or superior to that of one or both of the
parties. (Bosworth vs. Terminal etc. Assoc. of St. Louis,
As a general rule the right to intervene exists in favor of 174 U.S. 182,187, 43 L. ed., 941, 943). As regards the
one who claims to be the owner or to have some
right to intervene in this manner, it may be stated that if The claim for damages for the use of the property against the intervenor
the party desiring to intervene shows a legitimate and defendant Republic of the Philippines to which it was transferred, likewise,
proper interest in the fund or property in question, the cannot be maintained because of the immunity of the state from suit. The
motion to intervene should be granted, especially if claim obviously constitutes a charge against, or financial liability to, the
such interest cannot be otherwise properly protected. Government and consequently cannot be entertained by the courts except
(Joaquin v. Herrera, 37 Phil. 705, 722-724) with the consent of said government. (Syquia vs. Almeda Lopez, 84 Phil.
312; 47 Off. Gaz., 665; Compania General de Tabacos vs. Govt. of the PI
45 Phil., 663). Plaintiff argues that by its intervention, the Republic of the
Here, the BREDCO lots and stocks were sequestered and are now in
Philippines, in effect, waived its right of non-suability, but it will be
custodia legis (Bernas, The Constitution of the Republic of the Philippines,
remembered that the Republic intervened in the case merely to unite with
An Annotated Text, 1987 ed., p. 129, footnote 42). From the facts averred
the defendant Attorney General of the United States in resisting plaintiffs
by Palanca and the plaintiff, it is easy to see that in the event We decide to
claims, and for that reason asked no affirmative relief against any party in
order the reconveyance of those assets to the plaintiff, Palanca and his co-
the answer in intervention. x x x. Clearly, this is not a case where the State
stockholders in BREDCO stand to be adversely affected.
takes the initiative in an action against a private party by filing a complaint in
intervention, thereby surrendering its privileged position and coming down to
And thirdly, the legal interest of Palanca and his co-stockholders in the the level of the defendants what happened in the case of Froilan vs. Pan
matter in litigation and the possibility of a judgment ordering reconveyance Oriental Shipping Co., et al.-95 Phil. 905 cited by the plaintiff but one where
in favor of the plaintiff, invest them with legal interest in the success of the the State, as one of the defendants merely resisted a claim against it
defendants, at least insofar as the BREDCO lots and shares are concerned. precisely on the ground, among others, of its privileged position which
Section 2, Rule 12, also permits intervention by a person who has legal exempts it from suit. (emphasis supplied).
interest in the success of either of the parties. 20
In the present case, the private respondents intervened in Civil Case No. 0025 merely to unite
The petitioner's contention that the State cannot be sued without its consent and that private with the defendants therein in resisting the claims of petitioner, as plaintiff, and for that reason
respondents' action for intervention is, in legal effect, a suit or counter-suit against the sovereign asked for no affirmative relief against any party in their answer in intervention. In other words,
is also untenable. this is not a case where the private respondents take the initiative in an action against petitioner
by filing a complaint in intervention or a complaint. As observed by respondent Sandiganbayan:
The Rules of Court 21 provide that the intervention shall be made by complaint filed and served
in regular form, and may be answered as if it where an original complaint; but where the In intervening, Palanca and his co-stockholders have for their purpose to
intervenor unites with the defendant in resisting the claims of the plaintiff, the intervention may exclude the BREDCO lots and stocks or, at least, their 35% interest in the
be made in the form of an answer to the complaint. In order words, a third person who makes BREDCO project from any possible judgment directing reconveyance of the
himself a party to an existing litigation, may either join the plaintiff in claiming what is sought in alleged ill-gotten wealth to the plaintiff. They do not pray for damages
the filing a complaint in intervention, or by uniting with the defendant in resisting the claims of the against the latter. In effect, they occupy a defensive position as regards
plaintiff, by filing an answer in intervention. those shares of stock or interest. The fact that they interjected themselves
into his litigation at their own initiative does not alter the essential nature of
their intervention." 24
In Froilan v. Pan Oriental Shipping Co., 22 the plaintiff therein Fernando A. Froilan filed a
complaint against the defendant, Pan Oriental Shipping Co. The Republic of the Philippines
intervened by filing a complaint in intervention. Thereafter, the defendant filed its answer to the Private respondents' action for intervention in Civil Case No. 0025 is not, therefore, a suit or
complaint in intervention, and set up a counterclaim against the Republic of the Philippines. The counter-suit against petitioner Republic of the Philippines.
trial court dismissed the defendants counterclaim against the Republic on the ground, among
others, that the state is immune from suit. On appeal, this Court held that the dismissal of the
Having arrived at the above conclusions, the Court finds no need to further discuss the
counterclaim was untenable, because by filing its complaint in intervention the Government in
petitioner's pretense that the private respondents' claims are claims as between and/or among
effect waived its right to non-suability.
Ferdinand and Imelda Marcos, et al., and that the same is not cognizable by respondent
Sandiganbayan but by the regular courts. It suffices to state that, as already stated, in
In another case, Lim vs. Brownell, Jr. and Kagawa, 23 the plaintiff Benito E. Lim, as administrator intervening in Civil Case No. 0025, private respondents merely joined the defendants therein in
of the intestate estate of Arsenia Enriquez, filed a complaint in the Court of First Instance of resisting the claims of petitioner, as plaintiff, and that they asked no affirmative relief against any
Manila against the Alien Property Administrator (later substituted by the Attorney General of the party in their answer in intervention. They do not appear to have any controversy with the
United States) for the recovery of four (4) parcels of land (which were subsequently transferred defendants, Ferdinand and Imelda Marcos, et al.
to the Republic of the Philippines) with a prayer for the payment of back rentals. The Republic of
the Philippines intervened in the case. The defendant Attorney General of the United States and
ACCORDINGLY, the petition in the present case is hereby DISMISSED.
the defendant- intervenor Republic of the Philippines each filed an answer, alleging by way of
affirmative defense, among others, that the lower court had no jurisdiction over the claim for
rentals since the action in that regard constituted a suit against the Republic to which it had not SO ORDERED.
given its consent. The trial court dismissed the complaint for lack of jurisdiction. On appeal, this
Court affirmed, with the following reasons:
G.R. No. L-48214 December 19, 1978
ILDEFONSO SANTIAGO, represented by his Attorney-in-Fact, ALFREDO T. doctrine of non-suability recognized in this jurisdiction even prior to the effectivity of the [1935]
SANTIAGO, petitioner, Constitution is a logical corollary of the positivist concept of law which, to paraphrase Holmes,
vs. negates the assertion of any legal right as against the state, in itself the source of the law on
THE GOVERNMENT OF THE REPUBLIC OF THE PHILIPPINES, represented by the which such a right may be predicated. Nor is this all. Even if such a principle does give rise to
Director, Bureau of Plant Industry, and the Regional Director, Region IX, Zamboanga problems, considering the vastly expanded role of government enabling it to engage in business
pursuits to promote the general welfare, it is not obeisance to the analytical school of thought
alone that calls for its continued applicability. 16 That is the teaching of the leading case of Mobil
FERNANDO, J.:
Philippines Exploration, Inc. v. Customs Arrastre Service, 17 promulgated in December of 1966.
As a matter of fact, the Switzerland General Insurance Co. decision was the thirty-seventh of its
The first impression yielded by a perusal of this petition for certiorari is its inherent weakness kind after Mobil. Clearly, then, the contention that to dismiss the suit would be to give the
considering the explicit provision in the present Constitution prohibiting a suit against the applicable constitutional provision a retroactive effect is, to put it at its mildest, untenable.
Republic without its consent. 1 Here petitioner Ildefonso Santiago 2 filed on August 9, 1976 an
action in the Court of First Instance of Zamboanga City naming as defendant the government of
3. Petitioner's counsel invoked Santos v. Santos, 18 a 1952 decision. A more thorough analysis
the Republic of the Philippines represented by the Director of the Bureau of Plant Industry. 3 His
ought to have cautioned him against reliance on such a case. It was therein clearly pointed out
plea was for the revocation of a deed of donation executed by him and his spouse in January of
that the government entity involved was originally the National Airports Corporation. Thereafter,
1971, 4 with the Bureau of Plant Industry as the donee. As alleged in such complaint, such
it "was abolished by Executive Order No. 365, series of 1950, and in its place and stead the Civil
Bureau, contrary to the terms of the donation, failed to "install lighting facilities and water system
Aeronautics Administration was created and took over all the assets and assumed all the
on the property donated and to build an office building and parking [lot] thereon which should
liabilities of the abolished corporation. The Civil Aeronautics Administration, even if it is not a
have been constructed and ready for occupancy on or before December 7, 1974. 5 That led him
juridical entity, cannot legally prevent a party or parties from enforcing their proprietary rights
to conclude that under the circumstances, he was exempt from compliance with such an explicit
under the cloak or shield of lack of juridical personality, because to took over all the powers and
constitutional command. The lower court, in the order challenged in this petition, was of a
assumed all the obligations of the defunct corporation which had entered into the contract in
different view. It sustained a motion to dismiss on the part of the defendant Republic of the
question." 19 Then came National Shipyard and Steel Corporation v. Court of Industrial
Philippines, now named as one of the respondents, the other respondent being the Court of First
Relations, 20 a 1963 decision, where the then Justice, later Chief Justice, Concepcion, as
Instance of Zamboanga City, Branch II. It premised such an order on the settled "rule that the
ponente, stated that a government-owned and controlled corporation "has a personality of its
state cannot be sued without its consent. This is so, because the New Constitution of the
own distinct and separate from that of the government. ... Accordingly, it may sue and be sued
Philippines expressly provides that the state may not be sued without its consent. 6 Solicitor
and may be subjected to court processes just like any other corporation. (Section 13, Act 1459,
General Estelito P. Mendoza, 7 in the com ment on the petition filed with this Court, is for the
as amended). 21 In three recent decisions, Philippine National Bank v. Court of Industrial
affirmance of the order of dismissal of respondent Court precisely to accord deference to the
Relations, 22 Philippine National Bank v. Honorable Judge Pabalan, 23and Philippine National
above categorical constitutional mandate.
Railways v. Union de Maquinistas, 24 this constitutional provision on non-suability was unavailing
in view of the suit being against a government-owned or controlled corporation. That point
On its face, such a submission carries persuasion. Upon further reflection, this Tribunal is apparently escaped the attention of counsel for petitioner. Hence Santos v. Santos is hardly
impressed with the unique aspect of this petition for certiorari, dealing as it does with a suit for controlling.
the revocation of a donation to the Republic, which allegedly fatted to conform with what was
agreed to by the donee. If an order of dismissal would suffice, then the element of unfairness
4. It is to be noted further that the trend against the interpretation sought to be fastened in the
enters, the facts alleged being hypothetically admitted. It is the considered opinion of this Court
broad language of Santos v. Santos is quite discernible. Not long after, in Araneta v. Hon. M.
then that to conform to the high dictates of equity and justice, the presumption of consent could
Gatmaitan, 25 decided in 1957, it was held that an action [against] Government officials, is
be indulged in safely. That would serve to accord to petitioner as plaintiff, at the very least, the
essentially one against the Government, ... . 26 In the same year, this Court, in Angat River
right to be heard. certiorari lies.
Irrigation System v. Angat River Workers 27 Union, after referring to the "basic and fundamental
principle of the law that the Government cannot be sued before courts of justice without its
1. This is not to deny the obstacle posed by the constitutional provision. It is expressed in consent," pointed out that "this privilege of non-suability of the Government" covers with the
language plain and unmistakable: "The State may not be sued without its consent. 8 The mantle of its protection "an entity," in this case, the Angat River Irrigation System. 28 Then, in
Republic cannot be proceeded against unless it allows itself to be sued. Neither can a 1960, came Lim v. Brownell, Jr., 29 where there was a reaffirmation of the doctrine that a "claim
department, bureau, agency, office, or instrumentality of the government where the suit, [constituting] a charge against, or financial liability to, the Government cannot be entertained by
according to the then Justice, now Chief Justice, Castro in Del Mar v. Philippine Veterans the courts except with the consent of said government. 30 Bureau of Printing v. Bureau of
Administration, 9 may result "in adverse consequences to the public treasury, whether in the Printing Employees Association 31 came a year later; it reiterated such a doctrine. It was not
disbursements of funds or loss of property. 10 Such a doctrine was reiterated in the following surprising therefore that in 1966, Mobil Philippines Exploration, Inc. was decided the way it was.
cases: Republic v. Villasor, 11 Sayson v. Singson, 12 Director of the Bureau of Printing v. The remedy, where the liability is based on contract, according to this Court, speaking through
Francisco, 13 and Republic v. Purisima. 14 Justice J. P. Bengzon, is for plaintiff to file a claim with the general office in accordance with the
controlling statute, Commonwealth Act No. 327. 32 To repeat, that doctrine has been adhered to
ever since. The latest case in point is Travelers Indemnity Company v. Barber Steamship Lines,
2. It is contended by counsel for petitioner that the above constitutional provision would be given Inc. 33 Justice Aquino's opinion concluded with this paragraph: "It is settled that the Bureau of
a retroactive application in this case if the suit for the revocation of donation were dismissed.
Customs, acting as part of the machinery of the national government in the operation of the
That is not the case at all. In Republic v. Purisima, this Court made clear that such a basic arrastre service, is immune from suit under the doctrine of non-suability of the State. The
postulate is part and parcel of the system of government implanted in the Philippines from the claimant's remedy to recover the loss or damage to the goods under the custody of the customs
time of the acquisition of sovereignty by the United States, and therefore, was implicit in the
arrastre service is to file a claim with the Commission in Audit as contemplated in Act No. 3083
1935 Constitution even in the absence of any explicit language to that effect. This it did in a
citation from Switzerland General Insurance Co., Ltd. v. Republic of the Philippines: 15 "The
and Commonwealth Act No. 327. 34 With the explicit provision found in the present Constitution,
the fundamental principle of non-suability becomes even more exigent in its command.

5. The reliance on Santos v. Santos as a prop for this petition having failed, it would ordinarily
follow that this suit cannot prosper. Nonetheless, as set forth at the outset, there is a novel
aspect that suffices to call for a contrary conclusion. It would be manifestly unfair for the
Republic, as donee, alleged to have violated the conditions under which it received gratuitously
certain property, thereafter to put as a barrier the concept of non-suitability. That would be a
purely one-sided arrangement offensive to one's sense of justice. Such conduct, whether
proceeding from an individual or governmental agency, is to be condemned. As a matter of fact,
in case it is the latter that is culpable, the affront to decency is even more manifest. The
government, to paraphrase Justice Brandeis, should set the example. If it is susceptible to the
charge of having acted dishonorably, then it forfeits public trust-and rightly so.

6. Fortunately, the constitutional provision itself snows a waiver. Where there is consent, a suit
may be filed. Consent need not be express. It can be implied. So it was more than implied
in Ministerio v. Court of First Instance of Cebu: 35 "The doctrine of governmental immunity from
suit cannot serve as an instrument for perpetrating an injustice on a citizen. 36 The fact that this
decision arose from a suit against the Public Highways Commissioner and the Auditor General
for failure of the government to pay for land necessary to widen a national highway, the defense
of immunity without the consent proving unavailing, is not material. The analogy is quite obvious.
Where the government ordinarily benefited by the taking of the land, the failure to institute the
necessary condemnation proceedings should not be a bar to an ordinary action for the collection
of the just compensation due. Here, the alleged failure to abide by the conditions under which a
donation was given should not prove an insuperable obstacle to a civil action, the consent
likewise being presumed. This conclusion is strengthened by the fact that while a donation
partakes of a contract, there is no money claim, and therefore reliance on Commonwealth Act
No. 327 would be futile.

7. Our decision, it must be emphasized, goes no further than to rule that a donor, with the
Republic or any of its agency being the donee, is entitled to go to court in case of an alleged
breach of the conditions of such donation. He has the right to be heard. Under the
circumstances, the fundamental postulate of non-suability cannot stand in the way. It is made to
accommodate itself to the demands of procedural due process, which is the negation of
arbitrariness and inequity. The government, in the final analysis, is the beneficiary. It thereby
manifests its adherence to the highest ethical standards, which can only be ignored at the risk of
losing the confidence of the people, the repository of the sovereign power. The judiciary under
this circumstance has the grave responsibility of living up to the ideal of objectivity and
impartiality, the very essence of the rule of law. Only by displaying the neutrality expected of an
arbiter, even if it happens to be one of the departments of a litigant, can the decision arrived at,
whatever it may be, command respect and be entitled to acceptance.

WHEREFORE, the writ of certiorari prayed for is granted and the order of dismissal of October
20, 1977 is nullified, set aside and declared to be without force and effect. The Court of First
Instance of Zamboanga City, Branch II, is hereby directed to proceed with this case, observing
the procedure set forth in the Rules of Court. No costs.
G.R. No. 206484 June 29, 2016 On September 3, 2003, the respondent spouses filed an accion publiciana complaint 6 against
the DOTC and Digitel for recovery of possession and damages. The complaint was docketed
as Civil Case No. 7355.
DEPARTMENT OF TRANSPORTATION AND COMMUNICATIONS (DOTC), Petitioner,
vs.
SPOUSES VICENTE ABECINA and MARIA CLEOFE ABECINA, Respondents. In its answer, the DOTC claimed immunity from suit and ownership over the subject
properties.7 Nevertheless, during the pre-trial conference, the DOTC admitted that the Abecinas
were the rightful owners of the properties and opted to rely instead on state immunity from suit.8
DECISION

On March 12, 2007, the respondent spouses and Digitel executed a Compromise Agreement
BRION, J.:
and entered into a Contract of Lease. The RTC rendered a partial decision and approved the
Compromise Agreement on March 22, 2007.9
This petition for review on certiorari seeks to reverse and set aside the March 20, 2013 decision
of the Court of Appeals (CA) in CA-G.R. CV No. 93795 1 affirming the decision of the Regional
On May 20, 2009, the RTC rendered its decision against the DOTC.10 It brushed aside the
Trial Court (RTC) of Daet, Camarines Norte, Branch 39, in Civil Case No. 7355.2The RTC
defense of state immunity. Citing Ministerio v. Court of First Instance11and Amigable v.
ordered the Department of Transportation and Communications (DOTC) to vacate the
Cuenca,12it held that government immunity from suit could not be used as an instrument to
respondents' properties and to pay them actual and moral damages.
perpetuate an injustice on a citizen.13

ANTECEDENTS
The RTC held that as the lawful owners of the properties, the respondent spouses enjoyed the
right to use and to possess them – rights that were violated by the DOTC’s unauthorized entry,
Respondent spouses Vicente and Maria Cleofe Abecina (respondents/spouses Abecina) are the construction, and refusal to vacate. The RTC (1) ordered the Department – as a builder in bad
registered owners of five parcels of land in Sitio Paltik, Barrio Sta. Rosa, Jose Panganiban, faith – to forfeit the improvements and vacate the properties; and (2) awarded the spouses with
Camarines Norte. ₱1,200,000.00 as actual damages, ₱200,000.00 as moral damages, and ₱200,000.00 as
exemplary damages plus attorney’s fees and
The properties are covered by Transfer Certificates of Title (TCT) Nos. T-25094, T-25095, T-
25096, T-25097, and T-25098.3 costs of suit.

In February 1993, the DOTC awarded Digitel Telecommunications Philippines, Inc. (Digitel) a The DOTC elevated the case to the CA arguing: (1) that the RTC never acquired jurisdiction
contract for the management, operation, maintenance, and development of a Regional over it due to state immunity from suit; (2) that the suit against it should have been dismissed
Telecommunications Development Project (RTDP) under the National Telephone Program, after the spouses Abecina and Digitel executed a compromise agreement; and (3) that the RTC
Phase I, Tranche 1 (NTPI-1).4 erred in awarding actual, moral, and exemplary damages against it.14 The appeal was docketed
as CA-G.R. CV No. 93795.
The DOTC and Digitel subsequently entered into several Facilities Management Agreements
(FMA) for Digitel to manage, operate, maintain, and develop the RTDP and NTPI-1 facilities On March 20, 2013, the CA affirmed the RTC’s decision but deleted the award of exemplary
comprising local telephone exchange lines in various municipalities in Luzon. The FMAs were damages. The CA upheld the RTC’s jurisdiction over cases for accion publiciana where the
later converted into Financial Lease Agreements (FLA) in 1995. assessed value exceeds ₱20,000.00.15 It likewise denied the DOTC’s claim of state immunity
from suit, reasoning that the DOTC removed its cloak of immunity after entering into a
proprietary contract – the Financial Lease Agreement with Digitel.16 It also adopted the RTC’s
Later on, the municipality of Jose Panganiban, Camarines Norte, donated a one thousand two position that state immunity cannot be used to defeat a valid claim for compensation arising from
hundred (1,200) square-meter parcel of land to the DOTC for the implementation of the RDTP in an unlawful taking without the proper expropriation proceedings. 17 The CA affirmed the award of
the municipality. However, the municipality erroneously included portions of the respondents’
actual and moral damages due to the DOTC’s neglect to verify the perimeter of the telephone
property in the donation. Pursuant to the FLAs, Digitel constructed a telephone exchange on the exchange construction but found no valid justification for the award of exemplary damages. 18
property which encroached on the properties of the respondent spouses. 5

On April 16, 2013, the DOTC filed the present petition for review on certiorari.
Sometime in the mid-1990s, the spouses Abecina discovered Digitel’s occupation over portions
of their properties. They required Digitel to vacate their properties and pay damages, but the
latter refused, insisting that it was occupying the property of the DOTC pursuant to their FLA. THE PARTIES’ ARGUMENTS

On April 29, 2003, the respondent spouses sent a final demand letter to both the DOTC and The DOTC asserts that its Financial Lease Agreement with Digitel was entered into in pursuit of
Digitel to vacate the premises and to pay unpaid rent/damages in the amount of one million two its governmental functions to promote and develop networks of communication
hundred thousand pesos (₱1,200,000.00). Neither the DOTC nor Digitel complied with the systems.19 Therefore, it cannot be interpreted as a waiver of state immunity.
demand.
The DOTC also maintains that while it was regrettable that the construction of the telephone
exchange erroneously encroached on portions of the respondent’s properties, the RTC erred in
ordering the return of the property. 20 It argues that while the DOTC, in good faith and in the The Constitution identifies the limitations to the awesome and near-limitless powers of the State.
performance of its mandate, took private property without formal expropriation proceedings, the Chief among these limitations are the principles that no person shall be deprived of life, liberty,
taking was nevertheless an exercise of eminent domain.21 or property without due process of law and that private property shall not be taken for public use
without just compensation.37 These limitations are enshrined in no less than the Bill of Rights
that guarantees the citizen protection from abuse by the State.
Citing the 2007 case of Heirs of Mateo Pidacan v. Air Transportation Office (ATO),22the
Department prays that instead of allowing recovery of the property, the case should be
remanded to the RTC for determination of just compensation. Consequently, our laws38 require that the State’s power of eminent domain shall be exercised
through expropriation proceedings in court. Whenever private property is taken for public use, it
becomes the ministerial duty of the concerned office or agency to initiate expropriation
On the other hand, the respondents counter that the state immunity cannot be invoked to
proceedings. By necessary implication, the filing of a complaint for expropriation is a waiver of
perpetrate an injustice against its citizens.23 They also maintain that because the subject
State immunity.
properties are titled, the DOTC is a builder in bad faith who is deemed to have lost the
improvements it introduced.24 Finally, they differentiate their case from Heirs of Mateo Pidacan v.
ATO because Pidacan originated from a complaint for payment of the value of the property and If the DOTC had correctly followed the regular procedure upon discovering that it had
rentals while their case originated from a complaint for recovery of possession and damages. 25 encroached on the respondents’ property, it would have initiated expropriation proceedings
instead of insisting on its immunity from suit. The petitioners would not have had to resort to
filing its complaint for reconveyance. As this Court said in Ministerio:
OUR RULING

It is unthinkable then that precisely because there was a failure to abide by what the law
We find no merit in the petition.
requires, the government would stand to benefit. It is just as important, if not more so, that there
be fidelity to legal norms on the part of officialdom if the rule of law were to be maintained. It is
The State may not be sued without its consent.26 This fundamental doctrine stems from the not too much to say that when the government takes any property for public use, which is
principle that there can be no legal right against the authority which makes the law on which the conditioned upon the payment of just compensation, to be judicially ascertained, it
right depends.27 This generally accepted principle of law has been explicitly expressed in both makes manifest that it submits to the jurisdiction of a court. There is no thought then that
the 197328 and the present Constitutions. the doctrine of immunity from suit could still be appropriately invoked. 39 [emphasis supplied]

But as the principle itself implies, the doctrine of state immunity is not absolute. The State may We hold, therefore, that the Department’s entry into and taking of possession of the
waive its cloak of immunity and the waiver may be made expressly or by implication. respondents’ property amounted to an implied waiver of its governmental immunity from suit.

Over the years, the State’s participation in economic and commercial activities gradually We also find no merit in the DOTC’s contention that the RTC should not have ordered the
expanded beyond its sovereign function as regulator and governor.1âwphi1 The evolution of the reconveyance of the respondent spouses’ property because the property is being used for a vital
State’s activities and degree of participation in commerce demanded a parallel evolution in the governmental function, that is, the operation and maintenance of a safe and efficient
traditional rule of state immunity. Thus, it became necessary to distinguish between the State’s communication system.40
sovereign and governmental acts (jure imperii) and its private, commercial, and proprietary
acts (jure gestionis). Presently, state immunity restrictively extends only to acts jure imperii while
The exercise of eminent domain requires a genuine necessity to take the property for public use
acts jure gestionis are considered as a waiver of immunity.29
and the consequent payment of just compensation. The property is evidently being used for a
public purpose. However, we also note that the respondent spouses willingly entered into a
The Philippines recognizes the vital role of information and communication in nation lease agreement with Digitel for the use of the subject properties.
building.30 As a consequence, we have adopted a policy environment that aspires for the full
development of communications infrastructure to facilitate the flow of information into, out of, and
If in the future the factual circumstances should change and the respondents refuse to continue
across the country.31 To this end, the DOTC has been mandated with the promotion,
the lease, then the DOTC may initiate expropriation proceedings. But as matters now stand, the
development, and regulation of dependable and coordinated networks of communication.32
respondents are clearly willing to lease the property. Therefore, we find no genuine necessity for
the DOTC to actually take the property at this point.
The DOTC encroached on the respondents’ properties when it constructed the local telephone
exchange in Daet, Camarines Norte. The exchange was part of the RTDP pursuant to the
Lastly, we find that the CA erred when it affirmed the RTC's decision without deleting the
National Telephone Program. We have no doubt that when the DOTC constructed the
forfeiture of the improvements made by the DOTC through Digitel. Contrary to the RTC's
encroaching structures and subsequently entered into the FLA with Digitel for their maintenance,
findings, the DOTC was not a builder in bad faith when the improvements were constructed. The
it was carrying out a sovereign function. Therefore, we agree with the DOTC’s contention that
CA itself found that the Department's encroachment over the respondents' properties was a
these are acts jure imperii that fall within the cloak of state immunity.
result of a mistaken implementation of the donation from the municipality of Jose Panganiban. 41

However, as the respondents repeatedly pointed out, this Court has long established
Good faith consists in the belief of the builder that the land he is building on is his and [of] his
in Ministerio v CFI,33 Amigable v. Cuenca, 34the 2010 case Heirs of Pidacan v. ATO, 35and more
ignorance of any defect or flaw in his title.42 While the DOTC later realized its error and admitted
recently in Vigilar v. Aquino36that the doctrine of state immunity cannot serve as an instrument
its encroachment over the respondents' property, there is no evidence that it acted maliciously or
for perpetrating an injustice to a citizen.
in bad faith when the construction was done.
Article 52743 of the Civil Code presumes good faith. Without proof that. the Department's mistake
was made in bad faith, its construction is presumed to have been made in good faith. Therefore,
the forfeiture of the improvements in favor of the respondent spouses is unwarranted.

WHEREFORE, we hereby DENY the petition for lack of merit. The May 20, 2009 decision of the
Regional Trial Court in Civil Case No. 7355, as modified by the March 20, 2013 decision of the
Court of Appeals in CA-G.R. CV No. 93795, is AFFIRMED with further MODIFICATION that the
forfeiture of the improvements made by the DOTC in favor of the respondents is DELETED. No
costs.

SO ORDERED.
G.R. No. 206510 September 16, 2014 life. The 97,030-hectare protected marine park is also an important habitat for internationally
threatened and endangered marine species. UNESCO cited Tubbataha's outstanding universal
value as an important and significant natural habitat for in situ conservation of biological
MOST REV. PEDRO D. ARIGO, Vicar Apostolic of Puerto Princesa D.D.; MOST REV.
diversity; an example representing significant on-going ecological and biological processes; and
DEOGRACIAS S. INIGUEZ, JR., Bishop-Emeritus of Caloocan, FRANCES Q. QUIMPO,
an area of exceptional natural beauty and aesthetic importance. 2
CLEMENTE G. BAUTISTA, JR., Kalikasan-PNE, MARIA CAROLINA P. ARAULLO, RENATO
M. REYES, JR., Bagong Alyansang Makabayan, HON. NERI JAVIER COLMENARES, Bayan
Muna Partylist, ROLAND G. SIMBULAN, PH.D., Junk VF A Movement, TERESITA R. On April 6, 2010, Congress passed Republic Act (R.A.) No. 10067, 3 otherwise known as the
PEREZ, PH.D., HON. RAYMOND V. PALATINO, Kabataan Party-list, PETER SJ. "Tubbataha Reefs Natural Park (TRNP) Act of 2009" "to ensure the protection and conservation
GONZALES, Pamalakaya, GIOVANNI A. TAPANG, PH. D., Agham, ELMER C. LABOG, of the globally significant economic, biological, sociocultural, educational and scientific values of
Kilusang Mayo Uno, JOAN MAY E. SALVADOR, Gabriela, JOSE ENRIQUE A. AFRICA, the Tubbataha Reefs into perpetuity for the enjoyment of present and future generations." Under
THERESA A. CONCEPCION, MARY JOAN A. GUAN, NESTOR T. BAGUINON, PH.D., A. the "no-take" policy, entry into the waters of TRNP is strictly regulated and many human
EDSEL F. TUPAZ, Petitioners, activities are prohibited and penalized or fined, including fishing, gathering, destroying and
vs. disturbing the resources within the TRNP. The law likewise created the Tubbataha Protected
SCOTT H. SWIFT in his capacity as Commander of the US. 7th Fleet, MARK A. RICE in his Area Management Board (TPAMB) which shall be the sole policy-making and permit-granting
capacity as Commanding Officer of the USS Guardian, PRESIDENT BENIGNO S. AQUINO body of the TRNP.
III in his capacity as Commander-in-Chief of the Armed Forces of the Philippines, HON.
ALBERT F. DEL ROSARIO, Secretary, pepartment of Foreign Affair.s, HON. PAQUITO
The USS Guardian is an Avenger-class mine countermeasures ship of the US Navy. In
OCHOA, JR., Executiv~.:Secretary, Office of the President, . HON. VOLTAIRE T. GAZMIN,
December 2012, the US Embassy in the Philippines requested diplomatic clearance for the said
Secretary, Department of National Defense, HON. RAMON JESUS P. P AJE, Secretary,
vessel "to enter and exit the territorial waters of the Philippines and to arrive at the port of Subic
Department of Environment and Natural Resoz!rces, VICE ADMIRAL JOSE LUIS M.
Bay for the purpose of routine ship replenishment, maintenance, and crew liberty." 4 On January
ALANO, Philippine Navy Flag Officer in Command, Armed Forces of the Philippines,
6, 2013, the ship left Sasebo, Japan for Subic Bay, arriving on January 13, 2013 after a brief
ADMIRAL RODOLFO D. ISO RENA, Commandant, Philippine Coast Guard, COMMODORE
stop for fuel in Okinawa, Japan.1âwphi1
ENRICO EFREN EVANGELISTA, Philippine Coast Guard Palawan, MAJOR GEN. VIRGILIO
0. DOMINGO, Commandant of Armed Forces of the Philippines Command and LT. GEN.
TERRY G. ROBLING, US Marine Corps Forces. Pacific and Balikatan 2013 Exercise Co- On January 15, 2013, the USS Guardian departed Subic Bay for its next port of call in Makassar,
Director, Respondents. Indonesia. On January 17, 2013 at 2:20 a.m. while transiting the Sulu Sea, the ship ran aground
on the northwest side of South Shoal of the Tubbataha Reefs, about 80 miles east-southeast of
Palawan. No cine was injured in the incident, and there have been no reports of leaking fuel or
DECISION
oil.

VILLARAMA, JR, J.:


On January 20, 2013, U.S. 7th Fleet Commander, Vice Admiral Scott Swift, expressed regret for
the incident in a press statement.5 Likewise, US Ambassador to the Philippines Harry K.
Before us is a petition for the issuance of a Writ of Kalikasan with prayer for the issuance of a Thomas, Jr., in a meeting at the Department of Foreign Affairs (DFA) on February 4, "reiterated
Temporary Environmental Protection Order (TEPO) under Rule 7 of A.M. No. 09-6-8-SC, his regrets over the grounding incident and assured Foreign Affairs Secretazy Albert F. del
otherwise known as the Rules of Procedure for Environmental Cases (Rules), involving Rosario that the United States will provide appropriate compensation for damage to the reef
violations of environmental laws and regulations in relation to the grounding of the US military caused by the ship."6 By March 30, 2013, the US Navy-led salvage team had finished removing
ship USS Guardian over the Tubbataha Reefs. the last piece of the grounded ship from the coral reef.

Factual Background On April 1 7, 2013, the above-named petitioners on their behalf and in representation of their
respective sector/organization and others, including minors or generations yet unborn, filed the
present petition agairtst Scott H. Swift in his capacity as Commander of the US 7th Fleet, Mark
The name "Tubbataha" came from the Samal (seafaring people of southern Philippines)
A. Rice in his capacity as Commanding Officer of the USS Guardian and Lt. Gen. Terry G.
language which means "long reef exposed at low tide." Tubbataha is composed of two huge
Robling, US Marine Corps Forces, Pacific and Balikatan 2013 Exercises Co-Director ("US
coral atolls - the north atoll and the south atoll - and the Jessie Beazley Reef, a smaller coral
respondents"); President Benigno S. Aquino III in his capacity as Commander-in-Chief of the
structure about 20 kilometers north of the atolls. The reefs of Tubbataha and Jessie Beazley are
Armed Forces of the Philippines (AFP), DF A Secretary Albert F. Del Rosario, Executive
considered part of Cagayancillo, a remote island municipality of Palawan. 1
Secretary Paquito Ochoa, Jr., Secretary Voltaire T. Gazmin (Department of National Defense),
Secretary Jesus P. Paje (Department of Environment and Natural Resources), Vice-Admiral
In 1988, Tubbataha was declared a National Marine Park by virtue of Proclamation No. 306 Jose Luis M. Alano (Philippine Navy Flag Officer in Command, AFP), Admiral Rodolfo D. Isorena
issued by President Corazon C. Aquino on August 11, 1988. Located in the middle of Central (Philippine Coast Guard Commandant), Commodore Enrico Efren Evangelista (Philippine Coast
Sulu Sea, 150 kilometers southeast of Puerto Princesa City, Tubbataha lies at the heart of the Guard-Palawan), and Major General Virgilio 0. Domingo (AFP Commandant), collectively the
Coral Triangle, the global center of marine biodiversity. "Philippine respondents."

In 1993, Tubbataha was inscribed by the United Nations Educational Scientific and Cultural
Organization (UNESCO) as a World Heritage Site. It was recognized as one of the Philippines'
oldest ecosystems, containing excellent examples of pristine reefs and a high diversity of marine
The Petition c. Declare that Philippine authorities may exercise primary and exclusive criminal
jurisdiction over erring U.S. personnel under the circumstances of this case;
Petitioners claim that the grounding, salvaging and post-salvaging operations of the USS
Guardian cause and continue to cause environmental damage of such magnitude as to affect d. Require Respondents to pay just and reasonable compensation in the settlement of
the provinces of Palawan, Antique, Aklan, Guimaras, Iloilo, Negros Occidental, Negros Oriental, all meritorious claims for damages caused to the Tubbataha Reef on terms and
Zamboanga del Norte, Basilan, Sulu, and Tawi-Tawi, which events violate their constitutional conditions no less severe than those applicable to other States, and damages for
rights to a balanced and healthful ecology. They also seek a directive from this Court for the personal injury or death, if such had been the case;
institution of civil, administrative and criminal suits for acts committed in violation of
environmental laws and regulations in connection with the grounding incident.
e. Direct Respondents to cooperate in providing for the attendance of witnesses and in
the collection and production of evidence, including seizure and delivery of objects
Specifically, petitioners cite the following violations committed by US respondents under R.A. connected with the offenses related to the grounding of the Guardian;
No. 10067: unauthorized entry (Section 19); non-payment of conservation fees (Section 21 );
obstruction of law enforcement officer (Section 30); damages to the reef (Section 20); and
f. Require the authorities of the Philippines and the United States to notify each other
destroying and disturbing resources (Section 26[g]). Furthermore, petitioners assail certain
of the disposition of all cases, wherever heard, related to the grounding of the
provisions of the Visiting Forces Agreement (VFA) which they want this Court to nullify for being
Guardian;
unconstitutional.

g. Restrain Respondents from proceeding with any purported restoration, repair,


The numerous reliefs sought in this case are set forth in the final prayer of the petition, to wit:
salvage or post salvage plan or plans, including cleanup plans covering the damaged
WHEREFORE, in view of the foregoing, Petitioners respectfully pray that the Honorable Court:
area of the Tubbataha Reef absent a just settlement approved by the Honorable
1. Immediately issue upon the filing of this petition a Temporary Environmental Protection Order
Court;
(TEPO) and/or a Writ of Kalikasan, which shall, in particular,

h. Require Respondents to engage in stakeholder and LOU consultations in


a. Order Respondents and any person acting on their behalf, to cease and desist all
accordance with the Local Government Code and R.A. 10067;
operations over the Guardian grounding incident;

i. Require Respondent US officials and their representatives to place a deposit to the


b. Initially demarcating the metes and bounds of the damaged area as well as an
TRNP Trust Fund defined under Section 17 of RA 10067 as a bona .fide gesture
additional buffer zone;
towards full reparations;

c. Order Respondents to stop all port calls and war games under 'Balikatan' because
j. Direct Respondents to undertake measures to rehabilitate the areas affected by the
of the absence of clear guidelines, duties, and liability schemes for breaches of those
grounding of the Guardian in light of Respondents' experience in the Port Royale
duties, and require Respondents to assume responsibility for prior and future
grounding in 2009, among other similar grounding incidents;
environmental damage in general, and environmental damage under the Visiting
Forces Agreement in particular.
k. Require Respondents to regularly publish on a quarterly basis and in the name of
transparency and accountability such environmental damage assessment, valuation,
d. Temporarily define and describe allowable activities of ecotourism, diving,
and valuation methods, in all stages of negotiation;
recreation, and limited commercial activities by fisherfolk and indigenous communities
near or around the TRNP but away from the damaged site and an additional buffer
zone; l. Convene a multisectoral technical working group to provide scientific and technical
support to the TPAMB;
2. After summary hearing, issue a Resolution extending the TEPO until further orders
of the Court; m. Order the Department of Foreign Affairs, Department of National Defense, and the
Department of Environment and Natural Resources to review the Visiting Forces
Agreement and the Mutual Defense Treaty to consider whether their provisions allow
3. After due proceedings, render a Decision which shall include, without limitation:
for the exercise of erga omnes rights to a balanced and healthful ecology and for
damages which follow from any violation of those rights;
a. Order Respondents Secretary of Foreign Affairs, following the dispositive portion of
Nicolas v. Romulo, "to forthwith negotiate with the United States representatives for
n. Narrowly tailor the provisions of the Visiting Forces Agreement for purposes of
the appropriate agreement on [environmental guidelines and environmental
protecting the damaged areas of TRNP;
accountability] under Philippine authorities as provided in Art. V[] of the VFA ... "

o. Declare the grant of immunity found in Article V ("Criminal Jurisdiction") and Article
b. Direct Respondents and appropriate agencies to commence administrative, civil,
VI of the Visiting Forces Agreement unconstitutional for violating equal protection
and criminal proceedings against erring officers and individuals to the full extent of the
law, and to make such proceedings public;
and/or for violating the preemptory norm of nondiscrimination incorporated as part of Petitioners minors assert that they represent their generation as well as generations yet unborn.
the law of the land under Section 2, Article II, of the Philippine Constitution; We find no difficulty in ruling that they can, for themselves, for others of their generation and for
the succeeding generations, file a class suit. Their personality to sue in behalf of the succeeding
generations can only be based on the concept of intergenerational responsibility insofar as the
p. Allow for continuing discovery measures;
right to a balanced and healthful ecology is concerned. Such a right, as hereinafter expounded,
considers the "rhythm and harmony of nature." Nature means the created world in its entirety.
q. Supervise marine wildlife rehabilitation in the Tubbataha Reefs in all other respects; Such rhythm and harmony indispensably include, inter alia, the judicious disposition, utilization,
and management, renewal and conservation of the country's forest, mineral, land, waters, fisheries,
wildlife, off-shore areas and other natural resources to the end that their exploration,
development and utilization be equitably accessible to the present a:: well as future generations.
4. Provide just and equitable environmental rehabilitation measures and such other
Needless to say, every generation has a responsibility to the next to preserve that rhythm and
reliefs as are just and equitable under the premises.7 (Underscoring supplied.) harmony for the full 1:njoyment of a balanced and healthful ecology. Put a little differently, the
minors' assertion of their right to a sound environment constitutes, at the same time, the
Since only the Philippine respondents filed their comment8 to the petition, petitioners also filed a performance of their obligation to ensure the protection of that right for the generations to
motion for early resolution and motion to proceed ex parte against the US respondents.9 come.15 (Emphasis supplied.)

Respondents' Consolidated Comment The liberalization of standing first enunciated in Oposa, insofar as it refers to minors and
generations yet unborn, is now enshrined in the Rules which allows the filing of a citizen suit in
environmental cases. The provision on citizen suits in the Rules "collapses the traditional rule on
In their consolidated comment with opposition to the application for a TEPO and ocular personal and direct interest, on the principle that humans are stewards of nature."16
inspection and production orders, respondents assert that: ( 1) the grounds relied upon for the
issuance of a TEPO or writ of Kalikasan have become fait accompli as the salvage operations
on the USS Guardian were already completed; (2) the petition is defective in form and Having settled the issue of locus standi, we shall address the more fundamental question of
substance; (3) the petition improperly raises issues involving the VFA between the Republic of whether this Court has jurisdiction over the US respondents who did not submit any pleading or
the Philippines and the United States of America; and ( 4) the determination of the extent of manifestation in this case.
responsibility of the US Government as regards the damage to the Tubbataha Reefs rests
exdusively with the executive branch.
The immunity of the State from suit, known also as the doctrine of sovereign immunity or non-
suability of the State,17is expressly provided in Article XVI of the 1987 Constitution which states:
The Court's Ruling
Section 3. The State may not be sued without its consent.
As a preliminary matter, there is no dispute on the legal standing of petitioners to file the present
petition. In United States of America v. Judge Guinto,18 we discussed the principle of state immunity from
suit, as follows:
Locus standi is "a right of appearance in a court of justice on a given question."10 Specifically, it
is "a party's personal and substantial interest in a case where he has sustained or will sustain The rule that a state may not be sued without its consent, now · expressed in Article XVI,
direct injury as a result" of the act being challenged, and "calls for more than just a generalized Section 3, of the 1987 Constitution, is one of the generally accepted principles of international
grievance."11 However, the rule on standing is a procedural matter which this Court has relaxed law that we have adopted as part of the law of our land under Article II, Section 2. x x x.
for non-traditional plaintiffs like ordinary citizens, taxpayers and legislators when the public
interest so requires, such as when the subject matter of the controversy is of transcendental
importance, of overreaching significance to society, or of paramount public interest.12 Even without such affirmation, we would still be bound by the generally accepted principles of
international law under the doctrine of incorporation. Under this doctrine, as accepted by the
majority of states, such principles are deemed incorporated in the law of every civilized state as
In the landmark case of Oposa v. Factoran, Jr.,13 we recognized the "public right" of citizens to a condition and consequence of its membership in the society of nations. Upon its admission to
"a balanced and healthful ecology which, for the first time in our constitutional history, is such society, the state is automatically obligated to comply with these principles in its relations
solemnly incorporated in the fundamental law." We declared that the right to a balanced and
with other states.
healthful ecology need not be written in the Constitution for it is assumed, like other civil and
polittcal rights guaranteed in the Bill of Rights, to exist from the inception of mankind and it is an
issue of transcendental importance with intergenerational implications.1âwphi1 Such right As applied to the local state, the doctrine of state immunity is based on the justification given by
carries with it the correlative duty to refrain from impairing the environment. 14 Justice Holmes that ''there can be no legal right against the authority which makes the law on
which the right depends." [Kawanakoa v. Polybank, 205 U.S. 349] There are other practical
reasons for the enforcement of the doctrine. In the case of the foreign state sought to be
On the novel element in the class suit filed by the petitioners minors in Oposa, this Court ruled
impleaded in the local jurisdiction, the added inhibition is expressed in the maxim par in parem,
that not only do ordinary citizens have legal standing to sue for the enforcement of non habet imperium. All states are sovereign equals and cannot assert jurisdiction over one
environmental rights, they can do so in representation of their own and future generations. Thus: another. A contrary disposition would, in the language of a celebrated case, "unduly vex the
peace of nations." [De Haber v. Queen of Portugal, 17 Q. B. 171]
While the doctrine appears to prohibit only suits against the state without its consent, it is also distinguishes sovereign and governmental acts (Jure imperil") from private, commercial and
applicable to complaints filed against officials of the state for acts allegedly performed by them in proprietary acts (Jure gestionis). Under the restrictive rule of State immunity, State immunity
the discharge of their duties. The rule is that if the judgment against such officials will require the extends only to acts Jure imperii. The restrictive application of State immunity is proper only
state itself to perform an affirmative act to satisfy the same,. such as the appropriation of the when the proceedings arise out of commercial transactions of the foreign sovereign, its
amount needed to pay the damages awarded against them, the suit must be regarded as commercial activities or economic affairs.24
against the state itself although it has not been formally impleaded. [Garcia v. Chief of Staff, 16
SCRA 120] In such a situation, the state may move to dismiss the comp.taint on the ground that
In Shauf v. Court of Appeals,25 we discussed the limitations of the State immunity principle, thus:
it has been filed without its consent.19 (Emphasis supplied.)

It is a different matter where the public official is made to account in his capacity as such for acts
Under the American Constitution, the doctrine is expressed in the Eleventh Amendment which
contrary to law and injurious to the rights of plaintiff. As was clearly set forth by JustiGe Zaldivar
reads:
in Director of the Bureau of Telecommunications, et al. vs. Aligaen, etc., et al. : "Inasmuch as the
State authorizes only legal acts by its officers, unauthorized acts of government officials or
The Judicial power of the United States shall not be construed to extend to any suit in law or officers are not acts of the State, and an action against the officials or officers by one whose
equity, commenced or prosecuted against one of the United States by Citizens of another State, rights have been invaded or violated by such acts, for the protection of his rights, is not a suit
or by Citizens or Subjects of any Foreign State. against the State within the rule of immunity of the State from suit. In the same tenor, it has been
said that an action at law or suit in equity against a State officer or the director of a State
department on the ground that, while claiming to act for the State, he violates or invades the
In the case of Minucher v. Court of Appeals,20 we further expounded on the immunity of foreign
personal and property rights of the plaintiff, under an unconstitutional act or under an
states from the jurisdiction of local courts, as follows:
assumption of authority which he does not have, is not a suit against the State within the
constitutional provision that the State may not be sued without its consent." The rationale for this
The precept that a State cannot be sued in the courts of a foreign state is a long-standing rule of ruling is that the doctrine of state immunity cannot be used as an instrument for perpetrating an
customary international law then closely identified with the personal immunity of a foreign injustice.
sovereign from suit and, with the emergence of democratic states, made to attach not just to the
person of the head of state, or his representative, but also distinctly to the state itself in its
xxxx
sovereign capacity. If the acts giving rise to a suit arc those of a foreign government done by its
foreign agent, although not necessarily a diplomatic personage, but acting in his official capacity,
the complaint could be barred by the immunity of the foreign sovereign from suit without its The aforecited authorities are clear on the matter. They state that the doctrine of immunity from
consent. Suing a representative of a state is believed to be, in effect, suing the state itself. The suit will not apply and may not be invoked where the public official is being sued in his private
proscription is not accorded for the benefit of an individual but for the State, in whose service he and personal capacity as an ordinary citizen. The cloak of protection afforded the officers and
is, under the maxim -par in parem, non habet imperium -that all states are soverr~ign equals and agents of the government is removed the moment they are sued in their individual capacity. This
cannot assert jurisdiction over one another. The implication, in broad terms, is that if the situation usually arises where the public official acts without authority or in excess of the powers
judgment against an official would rec 1uire the state itself to perform an affirmative act to satisfy vested in him. It is a well-settled principle of law that a public official may be liable in his personal
the award, such as the appropriation of the amount needed to pay the damages decreed against private capacity for whatever damage he may have caused by his act done with malice and in
him, the suit must be regarded as being against the state itself, although it has not been formally bad faith, or beyond the scope of his authority or jurisdiction. 26 (Emphasis supplied.) In this case,
impleaded.21 (Emphasis supplied.) the US respondents were sued in their official capacity as commanding officers of the US Navy
who had control and supervision over the USS Guardian and its crew. The alleged act or
omission resulting in the unfortunate grounding of the USS Guardian on the TRNP was
In the same case we also mentioned that in the case of diplomatic immunity, the privilege is not
committed while they we:re performing official military duties. Considering that the satisfaction of
an immunity from the observance of the law of the territorial sovereign or from ensuing legal
a judgment against said officials will require remedial actions and appropriation of funds by the
liability; it is, rather, an immunity from the exercise of territorial jurisdiction. 22
US government, the suit is deemed to be one against the US itself. The principle of State
immunity therefore bars the exercise of jurisdiction by this Court over the persons of
In United States of America v. Judge Guinto,23 one of the consolidated cases therein involved a respondents Swift, Rice and Robling.
Filipino employed at Clark Air Base who was arrested following a buy-bust operation conducted
by two officers of the US Air Force, and was eventually dismissed from his employment when he
During the deliberations, Senior Associate Justice Antonio T. Carpio took the position that the
was charged in court for violation of R.A. No. 6425. In a complaint for damages filed by the said
conduct of the US in this case, when its warship entered a restricted area in violation of R.A. No.
employee against the military officers, the latter moved to dismiss the case on the ground that
10067 and caused damage to the TRNP reef system, brings the matter within the ambit of
the suit was against the US Government which had not given its consent. The RTC denied the
Article 31 of the United Nations Convention on the Law of the Sea (UNCLOS). He explained that
motion but on a petition for certiorari and prohibition filed before this Court, we reversed the RTC
while historically, warships enjoy sovereign immunity from suit as extensions of their flag State,
and dismissed the complaint. We held that petitioners US military officers were acting in the
Art. 31 of the UNCLOS creates an exception to this rule in cases where they fail to comply with
exercise of their official functions when they conducted the buy-bust operation against the
the rules and regulations of the coastal State regarding passage through the latter's internal
complainant and thereafter testified against him at his trial. It follows that for discharging their
waters and the territorial sea.
duties as agents of the United States, they cannot be directly impleaded for acts imputable to
their principal, which has not given its consent to be sued.
According to Justice Carpio, although the US to date has not ratified the UNCLOS, as a matter
of long-standing policy the US considers itself bound by customary international rules on the
This traditional rule of State immunity which exempts a State from being sued in the courts of
"traditional uses of the oceans" as codified in UNCLOS, as can be gleaned from previous
another State without the former's consent or waiver has evolved into a restrictive doctrine which
declarations by former Presidents Reagan and Clinton, and the US judiciary in the case of provisions may apply. But what if the offending warship is a non-party to the UNCLOS, as in this
United States v. Royal Caribbean Cruise Lines, Ltd.27 case, the US?

The international law of the sea is generally defined as "a body of treaty rules arid customary An overwhelming majority - over 80% -- of nation states are now members of UNCLOS, but
norms governing the uses of the sea, the exploitation of its resources, and the exercise of despite this the US, the world's leading maritime power, has not ratified it.
jurisdiction over maritime regimes. It is a branch of public international law, regulating the
relations of states with respect to the uses of the oceans." 28 The UNCLOS is a multilateral treaty
While the Reagan administration was instrumental in UNCLOS' negotiation and drafting, the
which was opened for signature on December 10, 1982 at Montego Bay, Jamaica. It was ratified
U.S. delegation ultimately voted against and refrained from signing it due to concerns over deep
by the Philippines in 1984 but came into force on November 16, 1994 upon the submission of
seabed mining technology transfer provisions contained in Part XI. In a remarkable, multilateral
the 60th ratification.
effort to induce U.S. membership, the bulk of UNCLOS member states cooperated over the
succeeding decade to revise the objection.able provisions. The revisions satisfied the Clinton
The UNCLOS is a product of international negotiation that seeks to balance State sovereignty administration, which signed the revised Part XI implementing agreement in 1994. In the fall of
(mare clausum) and the principle of freedom of the high seas (mare liberum). 29 The freedom to 1994, President Clinton transmitted UNCLOS and the Part XI implementing agreement to the
use the world's marine waters is one of the oldest customary principles of international Senate requesting its advice and consent. Despite consistent support from President Clinton,
law.30 The UNCLOS gives to the coastal State sovereign rights in varying degrees over the each of his successors, and an ideologically diverse array of stakeholders, the Senate has since
different zones of the sea which are: 1) internal waters, 2) territorial sea, 3) contiguous zone, 4) withheld the consent required for the President to internationally bind the United States to
exclusive economic zone, and 5) the high seas. It also gives coastal States more or less UNCLOS.
jurisdiction over foreign vessels depending on where the vessel is located.31
While UNCLOS cleared the Senate Foreign Relations Committee (SFRC) during the 108th and
Insofar as the internal waters and territorial sea is concerned, the Coastal State exercises 110th Congresses, its progress continues to be hamstrung by significant pockets of political
sovereignty, subject to the UNCLOS and other rules of international law. Such sovereignty ambivalence over U.S. participation in international institutions. Most recently, 111 th Congress
extends to the air space over the territorial sea as well as to its bed and subsoil. 32 SFRC Chairman Senator John Kerry included "voting out" UNCLOS for full Senate consideration
among his highest priorities. This did not occur, and no Senate action has been taken on
UNCLOS by the 112th Congress.34
In the case of warships,33 as pointed out by Justice Carpio, they continue to enjoy sovereign
immunity subject to the following exceptions:
Justice Carpio invited our attention to the policy statement given by President Reagan on March
10, 1983 that the US will "recognize the rights of the other , states in the waters off their coasts,
Article 30
as reflected in the convention [UNCLOS], so long as the rights and freedom of the United States
Non-compliance by warships with the laws and regulations of the coastal State
and others under international law are recognized by such coastal states", and President
Clinton's reiteration of the US policy "to act in a manner consistent with its [UNCLOS] provisions
If any warship does not comply with the laws and regulations of the coastal State concerning relating to traditional uses of the oceans and to encourage other countries to do likewise." Since
passage through the territorial sea and disregards any request for compliance therewith which is Article 31 relates to the "traditional uses of the oceans," and "if under its policy, the US
made to it, the coastal State may require it to leave the territorial sea immediately. 'recognize[s] the rights of the other states in the waters off their coasts,"' Justice Carpio
postulates that "there is more reason to expect it to recognize the rights of other states in their
internal waters, such as the Sulu Sea in this case."
Article 31
Responsibility of the flag State for damage caused by a warship
As to the non-ratification by the US, Justice Carpio emphasizes that "the US' refusal to join the
UN CLOS was centered on its disagreement with UN CLOS' regime of deep seabed mining
or other government ship operated for non-commercial purposes (Part XI) which considers the oceans and deep seabed commonly owned by mankind," pointing
out that such "has nothing to do with its [the US'] acceptance of customary international rules on
The flag State shall bear international responsibility for any loss or damage to the coastal State navigation."
resulting from the non-compliance by a warship or other government ship operated for non-
commercial purposes with the laws and regulations of the coastal State concerning passage
It may be mentioned that even the US Navy Judge Advocate General's Corps publicly endorses
through the territorial sea or with the provisions of this Convention or other rules of international the ratification of the UNCLOS, as shown by the following statement posted on its official
law. website:

Article 32 The Convention is in the national interest of the United States because it establishes stable
Immunities of warships and other government ships operated for non-commercial purposes maritime zones, including a maximum outer limit for territorial seas; codifies innocent passage,
transit passage, and archipelagic sea lanes passage rights; works against "jurisdictiomtl creep"
With such exceptions as are contained in subsection A and in articles 30 and 31, nothing in this by preventing coastal nations from expanding their own maritime zones; and reaffirms sovereign
Convention affects the immunities of warships and other government ships operated for non- immunity of warships, auxiliaries anJ government aircraft.
commercial purposes. (Emphasis supplied.) A foreign warship's unauthorized entry into our
internal waters with resulting damage to marine resources is one situation in which the above xxxx
Economically, accession to the Convention would support our national interests by enhancing can be inferred from Section 17, Rule 7 of the Rules that a criminal case against a person
the ability of the US to assert its sovereign rights over the resources of one of the largest charged with a violation of an environmental law is to be filed separately:
continental shelves in the world. Further, it is the Law of the Sea Convention that first
established the concept of a maritime Exclusive Economic Zone out to 200 nautical miles, and
SEC. 17. Institution of separate actions.-The filing of a petition for the issuance of the writ of
recognized the rights of coastal states to conserve and manage the natural resources in this
kalikasan shall not preclude the filing of separate civil, criminal or administrative actions.
Zone.35

In any case, it is our considered view that a ruling on the application or non-application of
We fully concur with Justice Carpio's view that non-membership in the UNCLOS does not mean
criminal jurisdiction provisions of the VF A to US personnel who may be found responsible for
that the US will disregard the rights of the Philippines as a Coastal State over its internal waters
the grounding of the USS Guardian, would be premature and beyond the province of a petition
and territorial sea. We thus expect the US to bear "international responsibility" under Art. 31 in
for a writ of Kalikasan. We also find it unnecessary at this point to determine whether such
connection with the USS Guardian grounding which adversely affected the Tubbataha reefs.
waiver of State immunity is indeed absolute. In the same vein, we cannot grant damages which
Indeed, it is difficult to imagine that our long-time ally and trading partner, which has been
have resulted from the violation of environmental laws. The Rules allows the recovery of
actively supporting the country's efforts to preserve our vital marine resources, would shirk from
damages, including the collection of administrative fines under R.A. No. 10067, in a separate
its obligation to compensate the damage caused by its warship while transiting our internal
civil suit or that deemed instituted with the criminal action charging the same violation of an
waters. Much less can we comprehend a Government exercising leadership in international
environmental law.37
affairs, unwilling to comply with the UNCLOS directive for all nations to cooperate in the global
task to protect and preserve the marine environment as provided in Article 197, viz:
Section 15, Rule 7 enumerates the reliefs which may be granted in a petition for issuance of a
writ of Kalikasan, to wit:
Article 197
Cooperation on a global or regional basis
SEC. 15. Judgment.-Within sixty (60) days from the time the petition is submitted for decision,
the court shall render judgment granting or denying the privilege of the writ of kalikasan.
States shall cooperate on a global basis and, as appropriate, on a regional basis, directly or
through competent international organizations, in formulating and elaborating international rules,
standards and recommended practices and procedures consistent with this Convention, for the The reliefs that may be granted under the writ are the following:
protection and preservation of the marine environment, taking into account characteristic
regional features.
(a) Directing respondent to permanently cease and desist from committing acts or
neglecting the performance of a duty in violation of environmental laws resulting in
In fine, the relevance of UNCLOS provisions to the present controversy is beyond dispute. environmental destruction or damage;
Although the said treaty upholds the immunity of warships from the jurisdiction of Coastal States
while navigating the.latter's territorial sea, the flag States shall be required to leave the territorial
'::;ea immediately if they flout the laws and regulations of the Coastal State, and they will be (b) Directing the respondent public official, govemment agency, private person or
liable for damages caused by their warships or any other government vessel operated for non- entity to protect, preserve, rehabilitate or restore the environment;
commercial purposes under Article 31.
(c) Directing the respondent public official, government agency, private person or
Petitioners argue that there is a waiver of immunity from suit found in the VFA. Likewise, they entity to monitor strict compliance with the decision and orders of the court;
invoke federal statutes in the US under which agencies of the US have statutorily waived their
immunity to any action. Even under the common law tort claims, petitioners asseverate that the (d) Directing the respondent public official, government agency, or private person or
US respondents are liable for negligence, trespass and nuisance. entity to make periodic reports on the execution of the final judgment; and

We are not persuaded. (e) Such other reliefs which relate to the right of the people to a balanced and healthful
ecology or to the protection, preservation, rehabilitation or restoration of the
The VFA is an agreement which defines the treatment of United States troops and personnel environment, except the award of damages to individual petitioners. (Emphasis
supplied.)
visiting the Philippines to promote "common security interests" between the US and the
Philippines in the region. It provides for the guidelines to govern such visits of military personnel,
and further defines the rights of the United States and the Philippine government in the matter of We agree with respondents (Philippine officials) in asserting that this petition has become moot
criminal jurisdiction, movement of vessel and aircraft, importation and exportation of equipment, in the sense that the salvage operation sought to be enjoined or restrained had already been
materials and supplies.36 The invocation of US federal tort laws and even common law is thus accomplished when petitioners sought recourse from this Court. But insofar as the directives to
improper considering that it is the VF A which governs disputes involving US military ships and Philippine respondents to protect and rehabilitate the coral reef stn icture and marine habitat
crew navigating Philippine waters in pursuance of the objectives of the agreement. adversely affected by the grounding incident are concerned, petitioners are entitled to these
reliefs notwithstanding the completion of the removal of the USS Guardian from the coral reef.
As it is, the waiver of State immunity under the VF A pertains only to criminal jurisdiction and not However, we are mindful of the fact that the US and Philippine governments both expressed
readiness to negotiate and discuss the matter of compensation for the damage caused by the
to special civil actions such as the present petition for issuance of a writ of Kalikasan. In fact, it
USS Guardian. The US Embassy has also declared it is closely coordinating with local scientists
and experts in assessing the extent of the damage and appropriate methods of rehabilitation.
Exploring avenues for settlement of environmental cases is not proscribed by the Rules. As can To underscore that the US government is prepared to pay appropriate compensation for the
be gleaned from the following provisions, mediation and settlement are available for the damage caused by the USS Guardian grounding, the US Embassy in the Philippines has
consideration of the parties, and which dispute resolution methods are encouraged by the court, announced the formation of a US interdisciplinary scientific team which will "initiate discussions
to wit: with the Government of the Philippines to review coral reef rehabilitation options in Tubbataha,
based on assessments by Philippine-based marine scientists." The US team intends to "help
assess damage and remediation options, in coordination with the Tubbataha Management
RULE3
Office, appropriate Philippine government entities, non-governmental organizations, and
scientific experts from Philippine universities."39
xxxx
A rehabilitation or restoration program to be implemented at the cost of the violator is also a
SEC. 3. Referral to mediation.-At the start of the pre-trial conference, the court shall inquire from major relief that may be obtained under a judgment rendered in a citizens' suit under the Rules,
the parties if they have settled the dispute; otherwise, the court shall immediately refer the viz:
parties or their counsel, if authorized by their clients, to the Philippine Mediation Center (PMC)
unit for purposes of mediation. If not available, the court shall refer the case to the clerk of court
RULES
or legal researcher for mediation.

SECTION 1. Reliefs in a citizen suit.-If warranted, the court may grant to the plaintiff proper
Mediation must be conducted within a non-extendible period of thirty (30) days from receipt of
reliefs which shall include the protection, preservation or rehabilitation of the environment and
notice of referral to mediation.
the payment of attorney's fees, costs of suit and other litigation expenses. It may also require the
violator to submit a program of rehabilitation or restoration of the environment, the costs of which
The mediation report must be submitted within ten (10) days from the expiration of the 30-day shall be borne by the violator, or to contribute to a special trust fund for that purpose subject to
period. the control of the court.1âwphi1

SEC. 4. Preliminary conference.-If mediation fails, the court will schedule the continuance of the In the light of the foregoing, the Court defers to the Executive Branch on the matter of
pre-trial. Before the scheduled date of continuance, the court may refer the case to the branch compensation and rehabilitation measures through diplomatic channels. Resolution of these
clerk of court for a preliminary conference for the following purposes: issues impinges on our relations with another State in the context of common security interests
under the VFA. It is settled that "[t]he conduct of the foreign relations of our government is
committed by the Constitution to the executive and legislative-"the political" --departments of the
(a) To assist the parties in reaching a settlement; government, and the propriety of what may be done in the exercise of this political power is not
subject to judicial inquiry or decision."40
xxxx
On the other hand, we cannot grant the additional reliefs prayed for in the petition to order a
SEC. 5. Pre-trial conference; consent decree.-The judge shall put the parties and their counsels review of the VFA and to nullify certain immunity provisions thereof.
under oath, and they shall remain under oath in all pre-trial conferences.
As held in BAYAN (Bagong Alyansang Makabayan) v. Exec. Sec. Zamora, 41 the VFA was duly
The judge shall exert best efforts to persuade the parties to arrive at a settlement of the dispute. concurred in by the Philippine Senate and has been recognized as a treaty by the United States
The judge may issue a consent decree approving the agreement between the parties in as attested and certified by the duly authorized representative of the United States government.
accordance with law, morals, public order and public policy to protect the right of the people to a The VF A being a valid and binding agreement, the parties are required as a matter of
balanced and healthful ecology. international law to abide by its terms and provisions.42 The present petition under the Rules is
not the proper remedy to assail the constitutionality of its provisions. WHEREFORE, the petition
for the issuance of the privilege of the Writ of Kalikasan is hereby DENIED.
xxxx

No pronouncement as to costs.
SEC. 10. Efforts to settle.- The court shall endeavor to make the parties to agree to compromise
or settle in accordance with law at any stage of the proceedings before rendition of judgment.
(Underscoring supplied.) SO ORDERED.

The Court takes judicial notice of a similar incident in 2009 when a guided-missile cruiser, the
USS Port Royal, ran aground about half a mile off the Honolulu Airport Reef Runway and
remained stuck for four days. After spending $6.5 million restoring the coral reef, the US
government was reported to have paid the State of Hawaii $8.5 million in settlement over coral
reef damage caused by the grounding.38
G.R. No. 142396 February 11, 2003 concentrated on politics, carpets and caviar. Thereafter, the defendant promised to see plaintiff
again.
KHOSROW MINUCHER, petitioner,
vs. "On May 19, 1986, the defendant called the plaintiff and invited the latter for dinner at Mario's
HON. COURT OF APPEALS and ARTHUR SCALZO, respondents. Restaurant at Makati. He wanted to buy 200 grams of caviar. Plaintiff brought the merchandize
but for the reason that the defendant was not yet there, he requested the restaurant people to x
x x place the same in the refrigerator. Defendant, however, came and plaintiff gave him the
DECISION
caviar for which he was paid. Then their conversation was again focused on politics and
business.
VITUG, J.:
"On May 26, 1986, defendant visited plaintiff again at the latter's residence for 18 years at
Sometime in May 1986, an Information for violation of Section 4 of Republic Act No. 6425, Kapitolyo, Pasig. The defendant wanted to buy a pair of carpets which plaintiff valued at
otherwise also known as the "Dangerous Drugs Act of 1972," was filed against petitioner $27,900.00. After some haggling, they agreed at $24,000.00. For the reason that defendant did
Khosrow Minucher and one Abbas Torabian with the Regional Trial Court, Branch 151, of Pasig not yet have the money, they agreed that defendant would come back the next day. The
City. The criminal charge followed a "buy-bust operation" conducted by the Philippine police following day, at 1:00 p.m., he came back with his $24,000.00, which he gave to the plaintiff, and
narcotic agents in the house of Minucher, an Iranian national, where a quantity of heroin, a the latter, in turn, gave him the pair of carpets.1awphi1.nét
prohibited drug, was said to have been seized. The narcotic agents were accompanied by
private respondent Arthur Scalzo who would, in due time, become one of the principal witnesses
"At about 3:00 in the afternoon of May 27, 1986, the defendant came back again to plaintiff's
for the prosecution. On 08 January 1988, Presiding Judge Eutropio Migrino rendered a decision
house and directly proceeded to the latter's bedroom, where the latter and his countryman,
acquitting the two accused.
Abbas Torabian, were playing chess. Plaintiff opened his safe in the bedroom and obtained
$2,000.00 from it, gave it to the defendant for the latter's fee in obtaining a visa for plaintiff's wife.
On 03 August 1988, Minucher filed Civil Case No. 88-45691 before the Regional Trial Court The defendant told him that he would be leaving the Philippines very soon and requested him to
(RTC), Branch 19, of Manila for damages on account of what he claimed to have been trumped- come out of the house for a while so that he can introduce him to his cousin waiting in a cab.
up charges of drug trafficking made by Arthur Scalzo. The Manila RTC detailed what it had Without much ado, and without putting on his shirt as he was only in his pajama pants, he
found to be the facts and circumstances surrounding the case. followed the defendant where he saw a parked cab opposite the street. To his complete
surprise, an American jumped out of the cab with a drawn high-powered gun. He was in the
company of about 30 to 40 Filipino soldiers with 6 Americans, all armed. He was handcuffed and
"The testimony of the plaintiff disclosed that he is an Iranian national. He came to the Philippines after about 20 minutes in the street, he was brought inside the house by the defendant. He was
to study in the University of the Philippines in 1974. In 1976, under the regime of the Shah of
made to sit down while in handcuffs while the defendant was inside his bedroom. The defendant
Iran, he was appointed Labor Attaché for the Iranian Embassies in Tokyo, Japan and Manila, came out of the bedroom and out from defendant's attaché case, he took something and placed
Philippines. When the Shah of Iran was deposed by Ayatollah Khomeini, plaintiff became a it on the table in front of the plaintiff. They also took plaintiff's wife who was at that time at the
refugee of the United Nations and continued to stay in the Philippines. He headed the Iranian boutique near his house and likewise arrested Torabian, who was playing chess with him in the
National Resistance Movement in the Philippines. bedroom and both were handcuffed together. Plaintiff was not told why he was being handcuffed
and why the privacy of his house, especially his bedroom was invaded by defendant. He was not
"He came to know the defendant on May 13, 1986, when the latter was brought to his house and allowed to use the telephone. In fact, his telephone was unplugged. He asked for any warrant,
introduced to him by a certain Jose Iñigo, an informer of the Intelligence Unit of the military. Jose but the defendant told him to `shut up.’ He was nevertheless told that he would be able to call for
Iñigo, on the other hand, was met by plaintiff at the office of Atty. Crisanto Saruca, a lawyer for his lawyer who can defend him.
several Iranians whom plaintiff assisted as head of the anti-Khomeini movement in the
Philippines. "The plaintiff took note of the fact that when the defendant invited him to come out to meet his
cousin, his safe was opened where he kept the $24,000.00 the defendant paid for the carpets
"During his first meeting with the defendant on May 13, 1986, upon the introduction of Jose and another $8,000.00 which he also placed in the safe together with a bracelet worth
Iñigo, the defendant expressed his interest in buying caviar. As a matter of fact, he bought two $15,000.00 and a pair of earrings worth $10,000.00. He also discovered missing upon his
kilos of caviar from plaintiff and paid P10,000.00 for it. Selling caviar, aside from that of Persian release his 8 pieces hand-made Persian carpets, valued at $65,000.00, a painting he bought for
carpets, pistachio nuts and other Iranian products was his business after the Khomeini P30,000.00 together with his TV and betamax sets. He claimed that when he was handcuffed,
government cut his pension of over $3,000.00 per month. During their introduction in that the defendant took his keys from his wallet. There was, therefore, nothing left in his house.
meeting, the defendant gave the plaintiff his calling card, which showed that he is working at the
US Embassy in the Philippines, as a special agent of the Drug Enforcement Administration, "That his arrest as a heroin trafficker x x x had been well publicized throughout the world, in
Department of Justice, of the United States, and gave his address as US Embassy, Manila. At various newspapers, particularly in Australia, America, Central Asia and in the Philippines. He
the back of the card appears a telephone number in defendant’s own handwriting, the number of
was identified in the papers as an international drug trafficker. x x x
which he can also be contacted.

In fact, the arrest of defendant and Torabian was likewise on television, not only in the
"It was also during this first meeting that plaintiff expressed his desire to obtain a US Visa for his
Philippines, but also in America and in Germany. His friends in said places informed him that
wife and the wife of a countryman named Abbas Torabian. The defendant told him that he they saw him on TV with said news.
[could] help plaintiff for a fee of $2,000.00 per visa. Their conversation, however, was more
"After the arrest made on plaintiff and Torabian, they were brought to Camp Crame handcuffed (now Chief Justice) Hilario Davide, Jr., this Court reversed the decision of the appellate court
together, where they were detained for three days without food and water." 1 and remanded the case to the lower court for trial. The remand was ordered on the theses (a)
that the Court of Appeals erred in granting the motion to dismiss of Scalzo for lack of jurisdiction
over his person without even considering the issue of the authenticity of Diplomatic Note No. 414
During the trial, the law firm of Luna, Sison and Manas, filed a special appearance for Scalzo
and (b) that the complaint contained sufficient allegations to the effect that Scalzo committed the
and moved for extension of time to file an answer pending a supposed advice from the United
imputed acts in his personal capacity and outside the scope of his official duties and, absent any
States Department of State and Department of Justice on the defenses to be raised. The trial
evidence to the contrary, the issue on Scalzo’s diplomatic immunity could not be taken up.
court granted the motion. On 27 October 1988, Scalzo filed another special appearance to
quash the summons on the ground that he, not being a resident of the Philippines and the action
being one in personam, was beyond the processes of the court. The motion was denied by the The Manila RTC thus continued with its hearings on the case. On 17 November 1995, the trial
court, in its order of 13 December 1988, holding that the filing by Scalzo of a motion for court reached a decision; it adjudged:
extension of time to file an answer to the complaint was a voluntary appearance equivalent to
service of summons which could likewise be construed a waiver of the requirement of formal
"WHEREFORE, and in view of all the foregoing considerations, judgment is hereby rendered for
notice. Scalzo filed a motion for reconsideration of the court order, contending that a motion for
the plaintiff, who successfully established his claim by sufficient evidence, against the defendant
an extension of time to file an answer was not a voluntary appearance equivalent to service of
in the manner following:
summons since it did not seek an affirmative relief. Scalzo argued that in cases involving the
United States government, as well as its agencies and officials, a motion for extension was
peculiarly unavoidable due to the need (1) for both the Department of State and the Department "`Adjudging defendant liable to plaintiff in actual and compensatory damages of P520,000.00;
of Justice to agree on the defenses to be raised and (2) to refer the case to a Philippine lawyer moral damages in the sum of P10 million; exemplary damages in the sum of P100,000.00;
who would be expected to first review the case. The court a quo denied the motion for attorney's fees in the sum of P200,000.00 plus costs.
reconsideration in its order of 15 October 1989.
`The Clerk of the Regional Trial Court, Manila, is ordered to take note of the lien of the Court on
Scalzo filed a petition for review with the Court of Appeals, there docketed CA-G.R. No. 17023, this judgment to answer for the unpaid docket fees considering that the plaintiff in this case
assailing the denial. In a decision, dated 06 October 1989, the appellate court denied the petition instituted this action as a pauper litigant.’"2
and affirmed the ruling of the trial court. Scalzo then elevated the incident in a petition for review
on certiorari, docketed G.R. No. 91173, to this Court. The petition, however, was denied for its
While the trial court gave credence to the claim of Scalzo and the evidence presented by him
failure to comply with SC Circular No. 1-88; in any event, the Court added, Scalzo had failed to
show that the appellate court was in error in its questioned judgment. that he was a diplomatic agent entitled to immunity as such, it ruled that he, nevertheless, should
be held accountable for the acts complained of committed outside his official duties. On appeal,
the Court of Appeals reversed the decision of the trial court and sustained the defense of Scalzo
Meanwhile, at the court a quo, an order, dated 09 February 1990, was issued (a) declaring that he was sufficiently clothed with diplomatic immunity during his term of duty and thereby
Scalzo in default for his failure to file a responsive pleading (answer) and (b) setting the case for immune from the criminal and civil jurisdiction of the "Receiving State" pursuant to the terms of
the reception of evidence. On 12 March 1990, Scalzo filed a motion to set aside the order of the Vienna Convention.
default and to admit his answer to the complaint. Granting the motion, the trial court set the case
for pre-trial. In his answer, Scalzo denied the material allegations of the complaint and raised the
affirmative defenses (a) of Minucher’s failure to state a cause of action in his complaint and (b) Hence, this recourse by Minucher. The instant petition for review raises a two-fold issue: (1)
that Scalzo had acted in the discharge of his official duties as being merely an agent of the Drug whether or not the doctrine of conclusiveness of judgment, following the decision rendered by
Enforcement Administration of the United States Department of Justice. Scalzo interposed a this Court in G.R. No. 97765, should have precluded the Court of Appeals from resolving the
appeal to it in an entirely different manner, and (2) whether or not Arthur Scalzo is indeed
counterclaim of P100,000.00 to answer for attorneys' fees and expenses of litigation.
entitled to diplomatic immunity.

Then, on 14 June 1990, after almost two years since the institution of the civil case, Scalzo filed
The doctrine of conclusiveness of judgment, or its kindred rule of res judicata, would require 1)
a motion to dismiss the complaint on the ground that, being a special agent of the United States
Drug Enforcement Administration, he was entitled to diplomatic immunity. He attached to his the finality of the prior judgment, 2) a valid jurisdiction over the subject matter and the parties on
motion Diplomatic Note No. 414 of the United States Embassy, dated 29 May 1990, addressed the part of the court that renders it, 3) a judgment on the merits, and 4) an identity of the parties,
subject matter and causes of action.3 Even while one of the issues submitted in G.R. No. 97765 -
to the Department of Foreign Affairs of the Philippines and a Certification, dated 11 June 1990,
of Vice Consul Donna Woodward, certifying that the note is a true and faithful copy of its original. "whether or not public respondent Court of Appeals erred in ruling that private respondent
In an order of 25 June 1990, the trial court denied the motion to dismiss. Scalzo is a diplomat immune from civil suit conformably with the Vienna Convention on
Diplomatic Relations" - is also a pivotal question raised in the instant petition, the ruling in G.R.
No. 97765, however, has not resolved that point with finality. Indeed, the Court there has made
On 27 July 1990, Scalzo filed a petition for certiorari with injunction with this Court, docketed this observation -
G.R. No. 94257 and entitled "Arthur W. Scalzo, Jr., vs. Hon. Wenceslao Polo, et al.," asking that
the complaint in Civil Case No. 88-45691 be ordered dismissed. The case was referred to the
Court of Appeals, there docketed CA-G.R. SP No. 22505, per this Court’s resolution of 07 "It may be mentioned in this regard that private respondent himself, in his Pre-trial Brief filed on
August 1990. On 31 October 1990, the Court of Appeals promulgated its decision sustaining the 13 June 1990, unequivocally states that he would present documentary evidence consisting of
DEA records on his investigation and surveillance of plaintiff and on his position and duties as
diplomatic immunity of Scalzo and ordering the dismissal of the complaint against him. Minucher
filed a petition for review with this Court, docketed G.R. No. 97765 and entitled "Khosrow DEA special agent in Manila. Having thus reserved his right to present evidence in support of his
Minucher vs. the Honorable Court of Appeals, et. al." (cited in 214 SCRA 242), appealing the position, which is the basis for the alleged diplomatic immunity, the barren self-serving claim in
judgment of the Court of Appeals. In a decision, dated 24 September 1992, penned by Justice
the belated motion to dismiss cannot be relied upon for a reasonable, intelligent and fair trafficking in a prohibited drug, and (c) that the Philippine Department of Foreign Affairs itself
resolution of the issue of diplomatic immunity."4 recognized that Scalzo during his tour of duty in the Philippines (14 October 1985 up to 10
August 1988) was listed as being an Assistant Attaché of the United States diplomatic mission
and accredited with diplomatic status by the Government of the Philippines. In his Exhibit 12,
Scalzo contends that the Vienna Convention on Diplomatic Relations, to which the Philippines is
Scalzo described the functions of the overseas office of the United States Drugs Enforcement
a signatory, grants him absolute immunity from suit, describing his functions as an agent of the
Agency, i.e., (1) to provide criminal investigative expertise and assistance to foreign law
United States Drugs Enforcement Agency as "conducting surveillance operations on suspected
enforcement agencies on narcotic and drug control programs upon the request of the host
drug dealers in the Philippines believed to be the source of prohibited drugs being shipped to the
country, 2) to establish and maintain liaison with the host country and counterpart foreign law
U.S., (and) having ascertained the target, (he then) would inform the Philippine narcotic agents
enforcement officials, and 3) to conduct complex criminal investigations involving international
(to) make the actual arrest." Scalzo has submitted to the trial court a number of documents -
criminal conspiracies which affect the interests of the United States.

1. Exh. '2' - Diplomatic Note No. 414 dated 29 May 1990;


The Vienna Convention on Diplomatic Relations was a codification of centuries-old customary
law and, by the time of its ratification on 18 April 1961, its rules of law had long become stable.
2. Exh. '1' - Certification of Vice Consul Donna K. Woodward dated 11 June 1990; Among the city states of ancient Greece, among the peoples of the Mediterranean before the
establishment of the Roman Empire, and among the states of India, the person of the herald in
time of war and the person of the diplomatic envoy in time of peace were universally held
3. Exh. '5' - Diplomatic Note No. 757 dated 25 October 1991;
sacrosanct.7 By the end of the 16th century, when the earliest treatises on diplomatic law were
published, the inviolability of ambassadors was firmly established as a rule of customary
4. Exh. '6' - Diplomatic Note No. 791 dated 17 November 1992; and international law.8Traditionally, the exercise of diplomatic intercourse among states was
undertaken by the head of state himself, as being the preeminent embodiment of the state he
represented, and the foreign secretary, the official usually entrusted with the external affairs of
5. Exh. '7' - Diplomatic Note No. 833 dated 21 October 1988. the state. Where a state would wish to have a more prominent diplomatic presence in the
receiving state, it would then send to the latter a diplomatic mission. Conformably with the
6. Exh. '3' - 1st Indorsement of the Hon. Jorge R. Coquia, Legal Adviser, Department Vienna Convention, the functions of the diplomatic mission involve, by and large, the
of Foreign Affairs, dated 27 June 1990 forwarding Embassy Note No. 414 to the Clerk representation of the interests of the sending state and promoting friendly relations with the
of Court of RTC Manila, Branch 19 (the trial court); receiving state.9

7. Exh. '4' - Diplomatic Note No. 414, appended to the 1st Indorsement (Exh. '3'); and The Convention lists the classes of heads of diplomatic missions to include (a) ambassadors or
nuncios accredited to the heads of state,10 (b) envoys,11 ministers or internuncios accredited to
the heads of states; and (c) charges d' affairs12 accredited to the ministers of foreign
8. Exh. '8' - Letter dated 18 November 1992 from the Office of the Protocol, affairs.13 Comprising the "staff of the (diplomatic) mission" are the diplomatic staff, the
Department of Foreign Affairs, through Asst. Sec. Emmanuel Fernandez, addressed administrative staff and the technical and service staff. Only the heads of missions, as well as
to the Chief Justice of this Court.5 members of the diplomatic staff, excluding the members of the administrative, technical and
service staff of the mission, are accorded diplomatic rank. Even while the Vienna Convention on
The documents, according to Scalzo, would show that: (1) the United States Embassy Diplomatic Relations provides for immunity to the members of diplomatic missions, it does so,
accordingly advised the Executive Department of the Philippine Government that Scalzo was a nevertheless, with an understanding that the same be restrictively applied. Only "diplomatic
member of the diplomatic staff of the United States diplomatic mission from his arrival in the agents," under the terms of the Convention, are vested with blanket diplomatic immunity from
Philippines on 14 October 1985 until his departure on 10 August 1988; (2) that the United States civil and criminal suits. The Convention defines "diplomatic agents" as the heads of missions or
Government was firm from the very beginning in asserting the diplomatic immunity of Scalzo members of the diplomatic staff, thus impliedly withholding the same privileges from all others. It
with respect to the case pursuant to the provisions of the Vienna Convention on Diplomatic might bear stressing that even consuls, who represent their respective states in concerns of
Relations; and (3) that the United States Embassy repeatedly urged the Department of Foreign commerce and navigation and perform certain administrative and notarial duties, such as the
Affairs to take appropriate action to inform the trial court of Scalzo’s diplomatic immunity. The issuance of passports and visas, authentication of documents, and administration of oaths, do
other documentary exhibits were presented to indicate that: (1) the Philippine government itself, not ordinarily enjoy the traditional diplomatic immunities and privileges accorded diplomats,
through its Executive Department, recognizing and respecting the diplomatic status of Scalzo, mainly for the reason that they are not charged with the duty of representing their states in
formally advised the "Judicial Department" of his diplomatic status and his entitlement to all political matters. Indeed, the main yardstick in ascertaining whether a person is a diplomat
diplomatic privileges and immunities under the Vienna Convention; and (2) the Department of entitled to immunity is the determination of whether or not he performs duties of diplomatic
Foreign Affairs itself authenticated Diplomatic Note No. 414. Scalzo additionally presented nature.
Exhibits "9" to "13" consisting of his reports of investigation on the surveillance and subsequent
arrest of Minucher, the certification of the Drug Enforcement Administration of the United States Scalzo asserted, particularly in his Exhibits "9" to "13," that he was an Assistant Attaché of the
Department of Justice that Scalzo was a special agent assigned to the Philippines at all times United States diplomatic mission and was accredited as such by the Philippine Government. An
relevant to the complaint, and the special power of attorney executed by him in favor of his attaché belongs to a category of officers in the diplomatic establishment who may be in charge
previous counsel6 to show (a) that the United States Embassy, affirmed by its Vice Consul, of its cultural, press, administrative or financial affairs. There could also be a class of attaches
acknowledged Scalzo to be a member of the diplomatic staff of the United States diplomatic belonging to certain ministries or departments of the government, other than the foreign ministry
mission from his arrival in the Philippines on 14 October 1985 until his departure on 10 August or department, who are detailed by their respective ministries or departments with the
1988, (b) that, on May 1986, with the cooperation of the Philippine law enforcement officials and embassies such as the military, naval, air, commercial, agricultural, labor, science, and customs
in the exercise of his functions as member of the mission, he investigated Minucher for alleged attaches, or the like. Attaches assist a chief of mission in his duties and are administratively
under him, but their main function is to observe, analyze and interpret trends and developments law of the territorial sovereign or from ensuing legal liability; it is, rather, an immunity from the
in their respective fields in the host country and submit reports to their own ministries or exercise of territorial jurisdiction.16 The government of the United States itself, which Scalzo
departments in the home government.14 These officials are not generally regarded as members claims to be acting for, has formulated its standards for recognition of a diplomatic agent. The
of the diplomatic mission, nor are they normally designated as having diplomatic rank. State Department policy is to only concede diplomatic status to a person who possesses an
acknowledged diplomatic title and "performs duties of diplomatic nature."17 Supplementary
criteria for accreditation are the possession of a valid diplomatic passport or, from States which
In an attempt to prove his diplomatic status, Scalzo presented Diplomatic Notes Nos. 414, 757
do not issue such passports, a diplomatic note formally representing the intention to assign the
and 791, all issued post litem motam, respectively, on 29 May 1990, 25 October 1991 and 17
person to diplomatic duties, the holding of a non-immigrant visa, being over twenty-one years of
November 1992. The presentation did nothing much to alleviate the Court's initial reservations in
age, and performing diplomatic functions on an essentially full-time basis.18 Diplomatic missions
G.R. No. 97765, viz:
are requested to provide the most accurate and descriptive job title to that which currently
applies to the duties performed. The Office of the Protocol would then assign each individual to
"While the trial court denied the motion to dismiss, the public respondent gravely abused its the appropriate functional category.19
discretion in dismissing Civil Case No. 88-45691 on the basis of an erroneous assumption that
simply because of the diplomatic note, the private respondent is clothed with diplomatic
But while the diplomatic immunity of Scalzo might thus remain contentious, it was sufficiently
immunity, thereby divesting the trial court of jurisdiction over his person.
established that, indeed, he worked for the United States Drug Enforcement Agency and was
tasked to conduct surveillance of suspected drug activities within the country on the dates
"x x x x x x x x x pertinent to this case. If it should be ascertained that Arthur Scalzo was acting well within his
assigned functions when he committed the acts alleged in the complaint, the present
controversy could then be resolved under the related doctrine of State Immunity from Suit.
"And now, to the core issue - the alleged diplomatic immunity of the private respondent. Setting
aside for the moment the issue of authenticity raised by the petitioner and the doubts that
surround such claim, in view of the fact that it took private respondent one (1) year, eight (8) The precept that a State cannot be sued in the courts of a foreign state is a long-standing
months and seventeen (17) days from the time his counsel filed on 12 September 1988 a rule of customary international law then closely identified with the personal immunity of a foreign
Special Appearance and Motion asking for a first extension of time to file the Answer because sovereign from suit20 and, with the emergence of democratic states, made to attach not just to
the Departments of State and Justice of the United States of America were studying the case for the person of the head of state, or his representative, but also distinctly to the state itself in its
the purpose of determining his defenses, before he could secure the Diplomatic Note from the sovereign capacity.21 If the acts giving rise to a suit are those of a foreign government done by its
US Embassy in Manila, and even granting for the sake of argument that such note is authentic, foreign agent, although not necessarily a diplomatic personage, but acting in his official capacity,
the complaint for damages filed by petitioner cannot be peremptorily dismissed. the complaint could be barred by the immunity of the foreign sovereign from suit without its
consent. Suing a representative of a state is believed to be, in effect, suing the state itself. The
proscription is not accorded for the benefit of an individual but for the State, in whose service he
"x x x x x x x x x is, under the maxim - par in parem, non habet imperium - that all states are sovereign equals
and cannot assert jurisdiction over one another.22 The implication, in broad terms, is that if the
"There is of course the claim of private respondent that the acts imputed to him were done in his judgment against an official would require the state itself to perform an affirmative act to satisfy
official capacity. Nothing supports this self-serving claim other than the so-called Diplomatic the award, such as the appropriation of the amount needed to pay the damages decreed against
Note. x x x. The public respondent then should have sustained the trial court's denial of the him, the suit must be regarded as being against the state itself, although it has not been formally
motion to dismiss. Verily, it should have been the most proper and appropriate recourse. It impleaded.23
should not have been overwhelmed by the self-serving Diplomatic Note whose belated issuance
is even suspect and whose authenticity has not yet been proved. The undue haste with which
In United States of America vs. Guinto,24 involving officers of the United States Air Force and
respondent Court yielded to the private respondent's claim is arbitrary." special officers of the Air Force Office of Special Investigators charged with the duty of
preventing the distribution, possession and use of prohibited drugs, this Court has ruled -
A significant document would appear to be Exhibit No. 08, dated 08 November 1992, issued by
the Office of Protocol of the Department of Foreign Affairs and signed by Emmanuel C. "While the doctrine (of state immunity) appears to prohibit only suits against the state without its
Fernandez, Assistant Secretary, certifying that "the records of the Department (would) show that consent, it is also applicable to complaints filed against officials of the state for acts allegedly
Mr. Arthur W. Scalzo, Jr., during his term of office in the Philippines (from 14 October 1985 up to
performed by them in the discharge of their duties. x x x. It cannot for a moment be imagined
10 August 1988) was listed as an Assistant Attaché of the United States diplomatic mission and that they were acting in their private or unofficial capacity when they apprehended and later
was, therefore, accredited diplomatic status by the Government of the Philippines." No certified testified against the complainant. It follows that for discharging their duties as agents of the
true copy of such "records," the supposed bases for the belated issuance, was presented in
United States, they cannot be directly impleaded for acts imputable to their principal, which has
evidence. not given its consent to be sued. x x x As they have acted on behalf of the government, and
within the scope of their authority, it is that government, and not the petitioners personally, [who
Concededly, vesting a person with diplomatic immunity is a prerogative of the executive branch were] responsible for their acts."25
of the government. In World Health Organization vs. Aquino,15 the Court has recognized that, in
such matters, the hands of the courts are virtually tied. Amidst apprehensions of indiscriminate This immunity principle, however, has its limitations. Thus, Shauf vs. Court of
and incautious grant of immunity, designed to gain exemption from the jurisdiction of courts, it
Appeals26 elaborates:
should behoove the Philippine government, specifically its Department of Foreign Affairs, to be
most circumspect, that should particularly be no less than compelling, in its post litem motam
issuances. It might be recalled that the privilege is not an immunity from the observance of the
"It is a different matter where the public official is made to account in his capacity as such for SO ORDERED.
acts contrary to law and injurious to the rights of the plaintiff. As was clearly set forth by Justice
Zaldivar in Director of the Bureau of Telecommunications, et al., vs. Aligaen, et al. (33 SCRA
368): `Inasmuch as the State authorizes only legal acts by its officers, unauthorized acts of
government officials or officers are not acts of the State, and an action against the officials or
officers by one whose rights have been invaded or violated by such acts, for the protection of his
rights, is not a suit against the State within the rule of immunity of the State from suit. In the
same tenor, it has been said that an action at law or suit in equity against a State officer or the
director of a State department on the ground that, while claiming to act for the State, he violates
or invades the personal and property rights of the plaintiff, under an unconstitutional act or under
an assumption of authority which he does not have, is not a suit against the State within the
constitutional provision that the State may not be sued without its consent. The rationale for this
ruling is that the doctrine of state immunity cannot be used as an instrument for perpetrating an
injustice.

"x x x x x x x x x

"(T)he doctrine of immunity from suit will not apply and may not be invoked where the public
official is being sued in his private and personal capacity as an ordinary citizen. The cloak of
protection afforded the officers and agents of the government is removed the moment they are
sued in their individual capacity. This situation usually arises where the public official acts
without authority or in excess of the powers vested in him. It is a well-settled principle of law that
a public official may be liable in his personal private capacity for whatever damage he may have
caused by his act done with malice and in bad faith or beyond the scope of his authority and
jurisdiction."27

A foreign agent, operating within a territory, can be cloaked with immunity from suit but only as
long as it can be established that he is acting within the directives of the sending state. The
consent of the host state is an indispensable requirement of basic courtesy between the two
sovereigns. Guinto and Shauf both involve officers and personnel of the United States, stationed
within Philippine territory, under the RP-US Military Bases Agreement. While evidence is wanting
to show any similar agreement between the governments of the Philippines and of the United
States (for the latter to send its agents and to conduct surveillance and related activities of
suspected drug dealers in the Philippines), the consent or imprimatur of the Philippine
government to the activities of the United States Drug Enforcement Agency, however, can be
gleaned from the facts heretofore elsewhere mentioned. The official exchanges of
communication between agencies of the government of the two countries, certifications from
officials of both the Philippine Department of Foreign Affairs and the United States Embassy, as
well as the participation of members of the Philippine Narcotics Command in the "buy-bust
operation" conducted at the residence of Minucher at the behest of Scalzo, may be inadequate
to support the "diplomatic status" of the latter but they give enough indication that the Philippine
government has given its imprimatur, if not consent, to the activities within Philippine territory of
agent Scalzo of the United States Drug Enforcement Agency. The job description of Scalzo has
tasked him to conduct surveillance on suspected drug suppliers and, after having ascertained
the target, to inform local law enforcers who would then be expected to make the arrest. In
conducting surveillance activities on Minucher, later acting as the poseur-buyer during the buy-
bust operation, and then becoming a principal witness in the criminal case against Minucher,
Scalzo hardly can be said to have acted beyond the scope of his official function or duties.

All told, this Court is constrained to rule that respondent Arthur Scalzo, an agent of the United
States Drug Enforcement Agency allowed by the Philippine government to conduct activities in
the country to help contain the problem on the drug traffic, is entitled to the defense of state
immunity from suit.

WHEREFORE, on the foregoing premises, the petition is DENIED. No costs.


G.R. No. 154705 June 26, 2003 Counsellor Kasim are diplomatic agents as defined under the Vienna Convention on Diplomatic
Relations and therefore enjoy diplomatic immunity.4 In turn, respondent filed on March 20, 2001,
an Opposition to the said motion alleging that the Republic of Indonesia has expressly waived its
THE REPUBLIC OF INDONESIA, HIS EXCELLENCY AMBASSADOR SOERATMIN, and
immunity from suit. He based this claim upon the following provision in the Maintenance
MINISTER COUNSELLOR AZHARI KASIM, Petitioners,
Agreement:
vs.
JAMES VINZON, doing business under the name and style of VINZON TRADE AND
SERVICES, Respondent. "Any legal action arising out of this Maintenance Agreement shall be settled according to the
laws of the Philippines and by the proper court of Makati City, Philippines."
DECISION
Respondent’s Opposition likewise alleged that Ambassador Soeratmin and Minister Counsellor
Kasim can be sued and held liable in their private capacities for tortious acts done with malice
AZCUNA, J:
and bad faith.5

This is a petition for review on certiorari to set aside the Decision of the Court of Appeals dated
On May 17, 2001, the trial court denied herein petitioners’ Motion to Dismiss. It likewise denied
May 30, 2002 and its Resolution dated August 16, 2002, in CA-G.R. SP No. 66894 entitled "The
the Motion for Reconsideration subsequently filed.
Republic of Indonesia, His Excellency Ambassador Soeratmin and Minister Counselor Azhari
Kasim v. Hon. Cesar Santamaria, Presiding Judge, RTC Branch 145, Makati City, and James
Vinzon, doing business under the name and style of Vinzon Trade and Services." The trial court’s denial of the Motion to Dismiss was brought up to the Court of Appeals by herein
petitioners in a petition for certiorari and prohibition. Said petition, docketed as CA-G.R. SP No.
66894, alleged that the trial court gravely abused its discretion in ruling that the Republic of
Petitioner, Republic of Indonesia, represented by its Counsellor, Siti Partinah, entered into a
Indonesia gave its consent to be sued and voluntarily submitted itself to the laws and jurisdiction
Maintenance Agreement in August 1995 with respondent James Vinzon, sole proprietor of
of Philippine courts and that petitioners Ambassador Soeratmin and Minister Counsellor Kasim
Vinzon Trade and Services. The Maintenance Agreement stated that respondent shall, for a
waived their immunity from suit.
consideration, maintain specified equipment at the Embassy Main Building, Embassy Annex
Building and the Wisma Duta, the official residence of petitioner Ambassador Soeratmin. The
equipment covered by the Maintenance Agreement are air conditioning units, generator sets, On May 30, 2002, the Court of Appeals rendered its assailed decision denying the petition for
electrical facilities, water heaters, and water motor pumps. It is likewise stated therein that the lack of merit.6 On August 16, 2002, it denied herein petitioners’ motion for reconsideration.7
agreement shall be effective for a period of four years and will renew itself automatically unless
cancelled by either party by giving thirty days prior written notice from the date of expiry. 1
Hence, this petition.

Petitioners claim that sometime prior to the date of expiration of the said agreement, or before
In the case at bar, petitioners raise the sole issue of whether or not the Court of Appeals erred in
August 1999, they informed respondent that the renewal of the agreement shall be at the
sustaining the trial court’s decision that petitioners have waived their immunity from suit by using
discretion of the incoming Chief of Administration, Minister Counsellor Azhari Kasim, who was
as its basis the abovementioned provision in the Maintenance Agreement.
expected to arrive in February 2000. When Minister Counsellor Kasim assumed the position of
Chief of Administration in March 2000, he allegedly found respondent’s work and services
unsatisfactory and not in compliance with the standards set in the Maintenance Agreement. The petition is impressed with merit.
Hence, the Indonesian Embassy terminated the agreement in a letter dated August 31,
2000.2 Petitioners claim, moreover, that they had earlier verbally informed respondent of their
decision to terminate the agreement. International law is founded largely upon the principles of reciprocity, comity, independence, and
equality of States which were adopted as part of the law of our land under Article II, Section 2 of
the 1987 Constitution.8 The rule that a State may not be sued without its consent is a necessary
On the other hand, respondent claims that the aforesaid termination was arbitrary and unlawful. consequence of the principles of independence and equality of States. 9 As enunciated in
Respondent cites various circumstances which purportedly negated petitioners’ alleged Sanders v. Veridiano II,10 the practical justification for the doctrine of sovereign immunity is that
dissatisfaction over respondent’s services: (a) in July 2000, Minister Counsellor Kasim still there can be no legal right against the authority that makes the law on which the right depends.
requested respondent to assign to the embassy an additional full-time worker to assist one of his In the case of foreign States, the rule is derived from the principle of the sovereign equality of
other workers; (b) in August 2000, Minister Counsellor Kasim asked respondent to donate a States, as expressed in the maxim par in parem non habet imperium. All states are sovereign
prize, which the latter did, on the occasion of the Indonesian Independence Day golf tournament; equals and cannot assert jurisdiction over one another.11 A contrary attitude would "unduly vex
and (c) in a letter dated August 22, 2000, petitioner Ambassador Soeratmin thanked respondent the peace of nations."12
for sponsoring a prize and expressed his hope that the cordial relations happily existing between
them will continue to prosper and be strengthened in the coming years.
The rules of International Law, however, are neither unyielding nor impervious to change. The
increasing need of sovereign States to enter into purely commercial activities remotely
3
Hence, on December 15, 2000, respondent filed a complaint against petitioners docketed as connected with the discharge of their governmental functions brought about a new concept of
Civil Case No. 18203 in the Regional Trial Court (RTC) of Makati, Branch 145. On February 20, sovereign immunity. This concept, the restrictive theory, holds that the immunity of the sovereign
2001, petitioners filed a Motion to Dismiss, alleging that the Republic of Indonesia, as a foreign is recognized only with regard to public acts or acts jure imperii, but not with regard to private
sovereign State, has sovereign immunity from suit and cannot be sued as a party-defendant in acts or acts jure gestionis.13
the Philippines. The said motion further alleged that Ambassador Soeratmin and Minister
In United States v. Ruiz,14 for instance, we held that the conduct of public bidding for the repair of xxx
a wharf at a United States Naval Station is an act jure imperii. On the other hand, we considered
as an act jure gestionis the hiring of a cook in the recreation center catering to American
1. A diplomatic agent shall enjoy immunity from the criminal jurisidiction of the receiving State.
servicemen and the general public at the John Hay Air Station in Baguio City, 15 as well as the
He shall also enjoy immunity from its civil and administrative jurisdiction, except in the case of:
bidding for the operation of barber shops in Clark Air Base in Angeles City.16

(a) a real action relating to private immovable property situated in the territory of the
Apropos the present case, the mere entering into a contract by a foreign State with a private
receiving State, unless he holds it on behalf of the sending State for the purposes of
party cannot be construed as the ultimate test of whether or not it is an act jure imperii or jure
the mission;
gestionis. Such act is only the start of the inquiry. Is the foreign State engaged in the regular
conduct of a business? If the foreign State is not engaged regularly in a business or commercial
activity, and in this case it has not been shown to be so engaged, the particular act or (b) an action relating to succession in which the diplomatic agent is involved as
transaction must then be tested by its nature. If the act is in pursuit of a sovereign activity, or an executor, administrator, heir or legatee as a private person and not on behalf of the
incident thereof, then it is an act jure imperii.17 sending State;

Hence, the existence alone of a paragraph in a contract stating that any legal action arising out (c) an action relating to any professional or commercial activity exercised by the
of the agreement shall be settled according to the laws of the Philippines and by a specified diplomatic agent in the receiving State outside his official functions.
court of the Philippines is not necessarily a waiver of sovereign immunity from suit. The
aforesaid provision contains language not necessarily inconsistent with sovereign immunity. On
the other hand, such provision may also be meant to apply where the sovereign party elects to xxx
sue in the local courts, or otherwise waives its immunity by any subsequent act. The applicability
of Philippine laws must be deemed to include Philippine laws in its totality, including the principle The act of petitioners Ambassador Soeratmin and Minister Counsellor Kasim in terminating the
recognizing sovereign immunity. Hence, the proper court may have no proper action, by way of Maintenance Agreement is not covered by the exceptions provided in the abovementioned
settling the case, except to dismiss it. provision.

Submission by a foreign state to local jurisdiction must be clear and unequivocal. It must be The Solicitor General believes that said act may fall under subparagraph (c) thereof, 20 but said
given explicitly or by necessary implication. We find no such waiver in this case. provision clearly applies only to a situation where the diplomatic agent engages in any
professional or commercial activity outside official functions, which is not the case herein.
Respondent concedes that the establishment of a diplomatic mission is a sovereign
function.1âwphi1 On the other hand, he argues that the actual physical maintenance of the WHEREFORE, the petition is hereby GRANTED. The decision and resolution of the Court of
premises of the diplomatic mission, such as the upkeep of its furnishings and equipment, is no Appeals in CA G.R. SP No. 66894 are REVERSED and SET ASIDE and the complaint in Civil
longer a sovereign function of the State.18 Case No. 18203 against petitioners is DISMISSED.

We disagree. There is no dispute that the establishment of a diplomatic mission is an act jure No costs.
imperii. A sovereign State does not merely establish a diplomatic mission and leave it at that; the
establishment of a diplomatic mission encompasses its maintenance and upkeep. Hence, the
State may enter into contracts with private entities to maintain the premises, furnishings and SO ORDERED.
equipment of the embassy and the living quarters of its agents and officials. It is therefore clear
that petitioner Republic of Indonesia was acting in pursuit of a sovereign activity when it entered
into a contract with respondent for the upkeep or maintenance of the air conditioning units,
generator sets, electrical facilities, water heaters, and water motor pumps of the Indonesian
Embassy and the official residence of the Indonesian ambassador.

The Solicitor General, in his Comment, submits the view that, "the Maintenance Agreement was
entered into by the Republic of Indonesia in the discharge of its governmental functions. In such
a case, it cannot be deemed to have waived its immunity from suit." As to the paragraph in the
agreement relied upon by respondent, the Solicitor General states that it "was not a waiver of
their immunity from suit but a mere stipulation that in the event they do waive their immunity,
Philippine laws shall govern the resolution of any legal action arising out of the agreement and
the proper court in Makati City shall be the agreed venue thereof. 19

On the matter of whether or not petitioners Ambassador Soeratmin and Minister Counsellor
Kasim may be sued herein in their private capacities, Article 31 of the Vienna Convention on
Diplomatic Relations provides:
G.R. No. 178551 October 11, 2010 As to Ikdal’s liability, the appellate court held that under Sec. 10 of Republic Act No. 8042, the
"Migrant and Overseas Filipinos’ Act of 1995," corporate officers, directors and partners of a
recruitment agency may themselves be jointly and solidarily liable with the recruitment agency
ATCI OVERSEAS CORPORATION, AMALIA G. IKDAL and MINISTRY OF PUBLIC HEALTH-
for money claims and damages awarded to overseas workers.
KUWAITPetitioners,
vs.
MA. JOSEFA ECHIN, Respondent. Petitioners’ motion for reconsideration having been denied by the appellate court by
Resolution7 of June 27, 2007, the present petition for review on certiorari was filed.
DECISION
Petitioners maintain that they should not be held liable because respondent’s employment
contract specifically stipulates that her employment shall be governed by the Civil Service Law
CARPIO MORALES, J.:
and Regulations of Kuwait. They thus conclude that it was patent error for the labor tribunals and
the appellate court to apply the Labor Code provisions governing probationary employment in
Josefina Echin (respondent) was hired by petitioner ATCI Overseas Corporation in behalf of its deciding the present case.
principal-co-petitioner, the Ministry of Public Health of Kuwait (the Ministry), for the position of
medical technologist under a two-year contract, denominated as a Memorandum of Agreement
Further, petitioners argue that even the Philippine Overseas Employment Act (POEA) Rules
(MOA), with a monthly salary of US$1,200.00.
relative to master employment contracts (Part III, Sec. 2 of the POEA Rules and Regulations)
accord respect to the "customs, practices, company policies and labor laws and legislation of the
Under the MOA,1 all newly-hired employees undergo a probationary period of one (1) year and host country."
are covered by Kuwait’s Civil Service Board Employment Contract No. 2.
Finally, petitioners posit that assuming arguendo that Philippine labor laws are applicable, given
Respondent was deployed on February 17, 2000 but was terminated from employment on that the foreign principal is a government agency which is immune from suit, as in fact it did not
February 11, 2001, she not having allegedly passed the probationary period. sign any document agreeing to be held jointly and solidarily liable, petitioner ATCI cannot
likewise be held liable, more so since the Ministry’s liability had not been judicially determined as
jurisdiction was not acquired over it.
As the Ministry denied respondent’s request for reconsideration, she returned to the Philippines
on March 17, 2001, shouldering her own air fare.
The petition fails.
On July 27, 2001, respondent filed with the National Labor Relations Commission (NLRC) a
complaint2 for illegal dismissal against petitioner ATCI as the local recruitment agency, Petitioner ATCI, as a private recruitment agency, cannot evade responsibility for the money
represented by petitioner, Amalia Ikdal (Ikdal), and the Ministry, as the foreign principal. claims of Overseas Filipino workers (OFWs) which it deploys abroad by the mere expediency of
claiming that its foreign principal is a government agency clothed with immunity from suit, or that
such foreign principal’s liability must first be established before it, as agent, can be held jointly
By Decision3 of November 29, 2002, the Labor Arbiter, finding that petitioners neither showed
and solidarily liable.
that there was just cause to warrant respondent’s dismissal nor that she failed to qualify as a
regular employee, held that respondent was illegally dismissed and accordingly ordered
petitioners to pay her US$3,600.00, representing her salary for the three months unexpired In providing for the joint and solidary liability of private recruitment agencies with their foreign
portion of her contract. principals, Republic Act No. 8042 precisely affords the OFWs with a recourse and assures them
of immediate and sufficient payment of what is due them. Skippers United Pacific v.
Maguad8 explains:
On appeal of petitioners ATCI and Ikdal, the NLRC affirmed the Labor Arbiter’s decision by
Resolution4 of January 26, 2004. Petitioners’ motion for reconsideration having been denied by
Resolution5 of April 22, 2004, they appealed to the Court of Appeals, contending that their . . . [T]he obligations covenanted in the recruitment agreement entered into by and
principal, the Ministry, being a foreign government agency, is immune from suit and, as such, the between the local agent and its foreign principal are not coterminous with the term of
immunity extended to them; and that respondent was validly dismissed for her failure to meet the such agreement so that if either or both of the parties decide to end the agreement, the
performance rating within the one-year period as required under Kuwait’s Civil Service Laws. responsibilities of such parties towards the contracted employees under the agreement do not at
Petitioners further contended that Ikdal should not be liable as an officer of petitioner ATCI. all end, but the same extends up to and until the expiration of the employment contracts of the
employees recruited and employed pursuant to the said recruitment agreement. Otherwise, this
will render nugatory the very purpose for which the law governing the employment of
By Decision6 of March 30, 2007, the appellate court affirmed the NLRC Resolution. workers for foreign jobs abroad was enacted. (emphasis supplied)

In brushing aside petitioners’ contention that they only acted as agent of the Ministry and that The imposition of joint and solidary liability is in line with the policy of the state to protect and
they cannot be held jointly and solidarily liable with it, the appellate court noted that under the
alleviate the plight of the working class.9 Verily, to allow petitioners to simply invoke the immunity
law, a private employment agency shall assume all responsibilities for the implementation of the from suit of its foreign principal or to wait for the judicial determination of the foreign principal’s
contract of employment of an overseas worker, hence, it can be sued jointly and severally with liability before petitioner can be held liable renders the law on joint and solidary liability inutile.
the foreign principal for any violation of the recruitment agreement or contract of employment.
As to petitioners’ contentions that Philippine labor laws on probationary employment are not To prove the Kuwaiti law, petitioners submitted the following: MOA between respondent and the
applicable since it was expressly provided in respondent’s employment contract, which she Ministry, as represented by ATCI, which provides that the employee is subject to a probationary
voluntarily entered into, that the terms of her engagement shall be governed by prevailing period of one (1) year and that the host country’s Civil Service Laws and Regulations apply; a
Kuwaiti Civil Service Laws and Regulations as in fact POEA Rules accord respect to such rules, translated copy11 (Arabic to English) of the termination letter to respondent stating that she did
customs and practices of the host country, the same was not substantiated. not pass the probation terms, without specifying the grounds therefor, and a translated copy of
the certificate of termination,12 both of which documents were certified by Mr. Mustapha Alawi,
Head of the Department of Foreign Affairs-Office of Consular Affairs Inslamic Certification and
Indeed, a contract freely entered into is considered the law between the parties who can
Translation Unit; and respondent’s letter13 of reconsideration to the Ministry, wherein she noted
establish stipulations, clauses, terms and conditions as they may deem convenient, including the
that in her first eight (8) months of employment, she was given a rating of "Excellent" albeit it
laws which they wish to govern their respective obligations, as long as they are not contrary to
changed due to changes in her shift of work schedule.
law, morals, good customs, public order or public policy.

These documents, whether taken singly or as a whole, do not sufficiently prove that respondent
It is hornbook principle, however, that the party invoking the application of a foreign law has the
was validly terminated as a probationary employee under Kuwaiti civil service laws. Instead of
burden of proving the law, under the doctrine of processual presumption which, in this case,
submitting a copy of the pertinent Kuwaiti labor laws duly authenticated and translated
petitioners failed to discharge. The Court’s ruling in EDI-Staffbuilders Int’l., v.
by Embassy officials thereat, as required under the Rules, what petitioners submitted
NLRC10 illuminates:
were mere certifications attesting only to the correctness of the translations of the MOA
and the termination letter which does not prove at all that Kuwaiti civil service laws differ
In the present case, the employment contract signed by Gran specifically states that Saudi Labor from Philippine laws and that under such Kuwaiti laws, respondent was validly
Laws will govern matters not provided for in the contract (e.g. specific causes for termination, terminated. Thus the subject certifications read:
termination procedures, etc.). Being the law intended by the parties (lex loci intentiones) to apply
to the contract, Saudi Labor Laws should govern all matters relating to the termination of the
xxxx
employment of Gran.

This is to certify that the herein attached translation/s from Arabic to English/Tagalog and or vice
In international law, the party who wants to have a foreign law applied to a dispute or case has
versa was/were presented to this Office for review and certification and the same was/were
the burden of proving the foreign law. The foreign law is treated as a question of fact to be
found to be in order. This Office, however, assumes no responsibility as to the contents of
properly pleaded and proved as the judge or labor arbiter cannot take judicial notice of a foreign
the document/s.
law. He is presumed to know only domestic or forum law.

This certification is being issued upon request of the interested party for whatever legal purpose
Unfortunately for petitioner, it did not prove the pertinent Saudi laws on the matter; thus, the
it may serve. (emphasis supplied)1avvphi1
International Law doctrine of presumed-identity approach or processual presumption comes into
play. Where a foreign law is not pleaded or, even if pleaded, is not proved, the presumption is
that foreign law is the same as ours. Thus, we apply Philippine labor laws in determining the Respecting Ikdal’s joint and solidary liability as a corporate officer, the same is in order too
issues presented before us. (emphasis and underscoring supplied) following the express provision of R.A. 8042 on money claims, viz:

The Philippines does not take judicial notice of foreign laws, hence, they must not only be SEC. 10. Money Claims.—Notwithstanding any provision of law to the contrary, the Labor
alleged; they must be proven. To prove a foreign law, the party invoking it must present a copy Arbiters of the National Labor Relations Commission (NLRC) shall have the original and
thereof and comply with Sections 24 and 25 of Rule 132 of the Revised Rules of Court which exclusive jurisdiction to hear and decide, within ninety (90) calendar days after the filing of the
reads: complaint, the claims arising out of an employer-employee relationship or by virtue of any law or
contract involving Filipino workers for overseas deployment including claims for actual moral,
exemplary and other forms of damages.
SEC. 24. Proof of official record. — The record of public documents referred to in paragraph (a)
of Section 19, when admissible for any purpose, may be evidenced by an official publication
thereof or by a copy attested by the officer having the legal custody of the record, or by his The liability of the principal/employer and the recruitment/placement agency for any and all
deputy, and accompanied, if the record is not kept in the Philippines, with a certificate that such claims under this section shall be joint and several. This provision shall be incorporated in the
officer has the custody. If the office in which the record is kept is in a foreign country, the contract for overseas employment and shall be a condition precedent for its approval. The
certificate may be made by a secretary of the embassy or legation, consul general, consul, vice performance bond to be filed by the recruitment/placement agency, as provided by law, shall be
consul, or consular agent or by any officer in the foreign service of the Philippines stationed in answerable for all money claims or damages that may be awarded to the workers. If the
the foreign country in which the record is kept, and authenticated by the seal of his office. recruitment/placement agency is a juridical being, the corporate officers and directors and
(emphasis supplied) partners as the case may be, shall themselves be jointly and solidarily liable with the corporation
or partnership for the aforesaid claims and damages. (emphasis and underscoring supplied)
SEC. 25. What attestation of copy must state. — Whenever a copy of a document or record is
attested for the purpose of the evidence, the attestation must state, in substance, that the copy WHEREFORE, the petition is DENIED.
is a correct copy of the original, or a specific part thereof, as the case may be. The attestation
must be under the official seal of the attesting officer, if there be any, or if he be the clerk of a
SO ORDERED.
court having a seal, under the seal of such court.
G.R. No. 156208 September 26, 2006 On 27 February 2002, the Secretary of the Department of Energy (DOE) promulgated the
Implementing Rules and Regulations (IRR) of the EPIRA Law, pursuant to Section 77 3 thereof.
Said IRR were approved by the Joint Congressional Power Commission on even date.
NPC DRIVERS AND MECHANICS ASSOCIATION, (NPC DAMA), represented by Its
Meanwhile, also in pursuant to the provisions of the EPIRA Law, the DOE created the Energy
President ROGER S. SAN JUAN, SR., NPC EMPLOYEES & WORKERS UNION (NEWU) –
Restructuring Steering Committee (Restructuring Committee) to manage the privatization and
NORTHERN LUZON REGIONAL CENTER, represented by its Regional President JIMMY D.
restructuring of the NPC, the National Transmission Corporation (TRANSCO), and the Power
SALMAN, in their own individual capacities and in behalf of the members of the
Sector Assets and Liabilities Corporation (PSALM).
associations and all affected officers and employees of National Power Corporation
(NPC), ZOL D. MEDINA, NARCISO M. MAGANTE, VICENTE B. CIRIO, JR., NECITAS B.
CAMAMA, in their individual capacities as employees of National Power To serve as the overall organizational framework for the realigned functions of the NPC
Corporation, petitioners, mandated under the EPIRA Law, the Restructuring Committee proposed a new NPC Table of
vs. Organization which was approved by the NPB through NPB Resolution No. 2002-53 dated 11
THE NATIONAL POWER CORPORATION (NPC), NATIONAL POWER BOARD OF April 2002. Likewise, the Restructuring Committee reviewed the proposed 2002 NPC
DIRECTORS (NPB), JOSE ISIDRO N. CAMACHO as Chairman of the National Power Board Restructuring Plan and assisted in the implementation of Phase I (Realignment) of said Plan,
of Directors (NPB), ROLANDO S. QUILALA, as President – Officer-in-charge/CEO of and thereafter recommended to the NPB for approval the adoption of measures pertaining to the
National Power Corporation and Member of National Power Board, and VINCENT S. separation and hiring of NPC personnel. The NPB, taking into consideration the
PEREZ, JR., EMILIA T. BONCODIN, MARIUS P. CORPUS, RUBEN S. REINOSO, JR., recommendation of the Restructuring Committee, thus amended the Restructuring Plan
GREGORY L. DOMINGO and NIEVES L. OSORIO, respondents. approved under NPB Resolution No. 2002-53.

DECISION On 18 November 2002, pursuant to Section 634 of the EPIRA Law and Rule 335 of the IRR, the
NPB passed NPB Resolution No. 2002-124 which provided for the Guidelines on the Separation
Program of the NPC and the Selection and Placement of Personnel in the NPC Table of
CHICO-NAZARIO, J.:
Organization. Under said Resolution, all NPC personnel shall be legally terminated on 31
January 2003, and shall be entitled to separation benefits. On the same day, the NPB approved
Before Us is a special civil action for Injunction to enjoin public respondents from implementing NPB Resolution No. 2002-125, whereby a Transition Team was constituted to manage and
the National Power Board (NPB) Resolutions No. 2002-124 and No. 2002-125, both dated 18 implement the NPC's Separation Program.
November 2002, directing, among other things, the termination of all employees of the National
Power Corporation (NPC) on 31 January 2003 in line with the restructuring of the NPC.
In a Memorandum dated 21 November 2002, the NPC OIC-President and CEO Rolando S.
Quilala circulated the assailed Resolutions and directed the concerned NPC officials to
On 8 June 2001, Republic Act No. 9136, otherwise known as the "Electric Power Industry disseminate and comply with said Resolutions and implement the same within the period
Reform Act of 2001" (EPIRA Law), was approved and signed into law by President Gloria provided for in the timetable set in NPB Resolution No. 2002-125. As a result thereof, Mr.
Macapagal-Arroyo, and took effect on 26 June 2001. Section 2(i) and Section 3 of the EPIRA Paquito F. Garcia, Manager – HRSD and Resources and Administration Coordinator of NPC,
Law states: circulated a Memorandum dated 22 November 2002 to all NPC officials and employees
providing for a checklist of the documents required for securing clearances for the processing of
separation benefits of all employees who shall be terminated under the Restructuring Plan.
Section 2. Declaration of Policy. – It is hereby declared the policy of the State:

Contending that the assailed NPB Resolutions are void and without force and effect, herein
xxxx
petitioners, in their individual and representative capacities, filed the present Petition for
Injunction to restrain respondents from implementing NPB Resolutions No. 2002-124 and No.
(i) To provide for an orderly and transparent privatization of the assets and liabilities of 2002-125. In support thereof, petitioners invoke Section 78 of the EPIRA Law, to wit:
the National Power Corporation (NPC);
Section 78. Injunction and Restraining Order. – The implementation of the provisions
xxxx of this Act shall not be restrained or enjoined except by an order issued by the
Supreme Court of the Philippines.
Section 3. Scope. – This Act shall provide a framework for the restructuring of the
electric power industry, including the privatization of the assets of NPC, the transition In assailing the validity of NPB Resolutions No. 2002-124 and No. 2002-125, petitioners
to the desired competitive structure, and the definition of the responsibilities of the maintain that said Resolutions were not passed and issued by a majority of the members of the
various government agencies and private entities.1 duly constituted Board of Directors since only three of its members, as provided under Section
486 of the EPIRA Law, were present, namely: DOE Secretary Vincent S. Perez, Jr.; Department
of Budget and Management Secretary Emilia T. Boncodin; and NPC OIC-President Rolando S.
Under the EPIRA Law,2 a new National Power Board of Directors was constituted composed of Quilala. According to petitioners, the other four members who were present at the meeting and
the Secretary of Finance as Chairman, with the Secretary of Energy, the Secretary of Budget signed the Resolutions were not the secretaries of their respective departments but were merely
and Management, the Secretary of Agriculture, the Director-General of the National Economic representatives or designated alternates of the officials who were named under the EPIRA Law
and Development Authority, the Secretary of Environment and Natural Resources, the Secretary to sit as members of the NPB. Petitioners claim that the acts of these representatives are
of Interior and Local Government, the Secretary of the Department of Trade and Industry, and violative of the well-settled principle that "delegated power cannot be further delegated." Thus,
the President of the National Power Corporation as members.
petitioners conclude that the questioned Resolutions have been illegally issued as it were not defined as "the act or the liberty to decide according to the principles of justice and one's ideas
issued by a duly constituted board since no quorum existed because only three of the nine of what is right and proper under the circumstances, without willfulness or favor. 8 Discretion,
members, as provided under Section 48 of the EPIRA Law, were present and qualified to sit and when applied to public functionaries, means a power or right conferred upon them by law of
vote. acting officially in certain circumstances, according to the dictates of their own judgment and
conscience, uncontrolled by the judgment or conscience of others. 9 It is to be presumed that in
naming the respective department heads as members of the board of directors, the legislature
It is petitioners' submission that even assuming arguendo that there was no undue delegation of
chose these secretaries of the various executive departments on the basis of their personal
power to the four representatives who signed the assailed Resolutions, said Resolutions cannot
qualifications and acumen which made them eligible to occupy their present positions as
still be given legal effect because the same did not comply with the mandatory requirement of
department heads. Thus, the department secretaries cannot delegate their duties as members of
endorsement by the Joint Congressional Power Commission and approval of the President of
the NPB, much less their power to vote and approve board resolutions, because it is their
the Philippines, as provided under Section 47 of the EPIRA Law which states that:
personal judgment that must be exercised in the fulfillment of such responsibility.

Section 47. NPC Privatization. – Except for the assets of SPUG, the generation
There is no question that the enactment of the assailed Resolutions involves the exercise of
assets, real estate, and other disposable assets as well as IPP contracts of NPC shall
discretion and not merely a ministerial act that could be validly performed by a delegate, thus,
be privatized in accordance with this Act. Within six (6) months from effectivity of this
the rule enunciated in the case of Binamira v. Garrucho10 is relevant in the present controversy,
Act, the PSALM Corp. shall submit a plan for the endorsement by the Joint
to wit:
Congressional Power Commission and the approval of the President of the
Philippines, on the total privatization of the generation assets, real estate, other
disposable assets as well as existing IPP contracts of NPC and thereafter, implement An officer to whom a discretion is entrusted cannot delegate it to another, the
the same, in accordance with the following guidelines, except as provided for in presumption being that he was chosen because he was deemed fit and competent to
paragraph (f) herein: x x x. exercise that judgment and discretion, and unless the power to substitute another in
his place has been given to him, he cannot delegate his duties to another.
Petitioners insist that if ever there exists a valid wholesale abolition of their positions and their
concomitant separation form the service, such a process is an integral part of "privatization" and In those cases in which the proper execution of the office requires, on the part of the
"restructuring" as defined under the EPIRA Law and, therefore, must comply with the above- officer, the exercise of judgment or discretion, the presumption is that he was chosen
quoted provision requiring the endorsement of the Joint Congressional Power Commission and because he was deemed fit and competent to exercise that judgment and discretion,
the approval of the President of the Philippines. Furthermore, petitioner highlight the fact that and, unless power to substitute another in his place has been given to him, he cannot
said Resolutions will have an adverse effect on about 5,648 employees of the NPC and will delegate his duties to another.
result in the displacement of some 2,370 employees, which, petitioners argue, is contrary to the
mandate of the Constitution to promote full employment and security of tenure.
Respondents' assertion to the contrary is not tenable. The ruling in the case cited by
respondents to support their contention is not applicable in the case at bar. While it is true that
Respondents, on the other hand, uphold the validity of the assailed Resolutions by arguing that the Court has determined in the case of American Tobacco Company v. Director of
while it is true that four members of the National Power Board of Directors, particularly the Patents11 that a delegate may exercise his authority through persons he appoints to assist him in
respective Secretaries of the Department of Interior and Local Government, the Department of his functions, it must be stressed that the Court explicitly stated in the same case that said
Trade and Industry, and the Department of Finance, as well as the Director-General of the practice is permissible only when the judgment and discretion finally exercised are those of
National Economic and Development Authority, were not the actual signatories in NPB the officer authorized by law. According to the Court, the rule that requires an administrative
Resolutions No. 2002-124 and No. 2002-125, they were, however, ably represented by their officer to exercise his own judgment and discretion does not preclude him from utilizing, as a
respective alternates. Respondents claim that the validity of such administrative practice matter of practical administrative procedure, the aid of subordinates, so long as it is the legally
whereby an authority is exercised by persons or subordinates appointed by the responsible authorized official who makes the final decision through the use of his own personal judgment.
official has long been settled. Respondents further contend that Section 48 of the EPIRA Law
does not in any way prohibit any member of the NPB from authorizing his representative to sign
In the case at bar, it is not difficult to comprehend that in approving NPB Resolutions No. 2002-
resolutions adopted by the Board.
124 and No. 2002-125, it is the representatives of the secretaries of the different executive
departments and not the secretaries themselves who exercised judgment in passing the
From the arguments put forward by herein parties, it is evident that the pivotal issue to be assailed Resolution, as shown by the fact that it is the signatures of the respective
resolved in this Petition for Injunction is whether or not NPB Resolutions No. 2002-124 and No. representatives that are affixed to the questioned Resolutions. This, to our mind, violates the
2002-125 were properly enacted. It is petitioners' contention that the failure of the four duty imposed upon the specifically enumerated department heads to employ their own sound
specifically identified department heads7 under Section 48 of the EPIRA Law to personally discretion in exercising the corporate powers of the NPC. Evidently, the votes cast by these
approve and sign the assailed Resolutions invalidates the adoption of said Resolutions. mere representatives in favor of the adoption of the said Resolutions must not be considered in
Petitioners maintain that there was undue delegation of delegated power when only the determining whether or not the necessary number of votes was garnered in order that the
representatives of certain members of the NPB attended the board meetings and passed and assailed Resolutions may be validly enacted. Hence, there being only three valid votes cast out
signed the questioned Resolutions. of the nine board members, namely those of DOE Secretary Vincent S. Perez, Jr.; Department
of Budget and Management Secretary Emilia T. Boncodin; and NPC OIC-President Rolando S.
Quilala, NPB Resolutions No. 2002-124 and No. 2002-125 are void and are of no legal effect.
We agree with petitioners. In enumerating under Section 48 those who shall compose the
National Power Board of Directors, the legislature has vested upon these persons the power to
exercise their judgment and discretion in running the affairs of the NPC. Discretion may be
Having determined that the assailed Resolutions are void as they lack the necessary number of
votes for their adoption, We no longer deem it necessary to pass upon the other issues raised in
the instant petition

WHEREFORE, premises considered, National Power Board Resolutions No. 2002-124 and No.
2002-125 are hereby declared VOID and WITHOUT LEGAL EFFECT. The Petition for Injunction
is hereby GRANTED and respondents are hereby ENJOINED from implementing said NPB
Resolutions No. 2002-124 and No. 2002-125.

SO ORDERED.
G.R. No. 177597 July 16, 2008 The incumbent elective provincial officials of the Province of Maguindanao shall continue to
serve their unexpired terms in the province that they will choose or where they are residents:
Provided, that where an elective position in both provinces becomes vacant as a consequence
BAI SANDRA S. A. SEMA, Petitioner,
of the creation of the Province of Shariff Kabunsuan, all incumbent elective provincial officials
vs.
shall have preference for appointment to a higher elective vacant position and for the time being
COMMISSION ON ELECTIONS and DIDAGEN P. DILANGALEN, Respondents.
be appointed by the Regional Governor, and shall hold office until their successors shall have
been elected and qualified in the next local elections; Provided, further, that they shall continue
x - - - - - - - - - - - - - - - - - - - - - - -x to receive the salaries they are receiving at the time of the approval of this Act until the new
readjustment of salaries in accordance with law. Provided, furthermore, that there shall be no
diminution in the number of the members of the Sangguniang Panlalawigan of the mother
G.R. No. 178628
province.

PERFECTO F. MARQUEZ, Petitioner,


Except as may be provided by national law, the existing legislative district, which includes
vs.
Cotabato as a part thereof, shall remain.
COMMISSION ON ELECTIONS, Respondent.

Later, three new municipalities6 were carved out of the original nine municipalities constituting
DECISION
Shariff Kabunsuan, bringing its total number of municipalities to 11. Thus, what was left of
Maguindanao were the municipalities constituting its second legislative district. Cotabato City,
CARPIO, J.: although part of Maguindanao’s first legislative district, is not part of the Province of
Maguindanao.
The Case
The voters of Maguindanao ratified Shariff Kabunsuan’s creation in a plebiscite held on 29
1 October 2006.
These consolidated petitions seek to annul Resolution No. 7902, dated 10 May 2007, of the
Commission on Elections (COMELEC) treating Cotabato City as part of the legislative district of
the Province of Shariff Kabunsuan.2 On 6 February 2007, the Sangguniang Panlungsod of Cotabato City passed Resolution No.
3999 requesting the COMELEC to "clarify the status of Cotabato City in view of the conversion
of the First District of Maguindanao into a regular province" under MMA Act 201.
The Facts

In answer to Cotabato City’s query, the COMELEC issued Resolution No. 07-0407 on 6 March
The Ordinance appended to the 1987 Constitution apportioned two legislative districts for the 2007 "maintaining the status quo with Cotabato City as part of Shariff Kabunsuan in the First
Province of Maguindanao. The first legislative district consists of Cotabato City and eight Legislative District of Maguindanao." Resolution No. 07-0407, which adopted the
municipalities.3 Maguindanao forms part of the Autonomous Region in Muslim Mindanao recommendation of the COMELEC’s Law Department under a Memorandum dated 27 February
(ARMM), created under its Organic Act, Republic Act No. 6734 (RA 6734), as amended by 2007,7 provides in pertinent parts:
Republic Act No. 9054 (RA 9054).4 Although under the Ordinance, Cotabato City forms part of
Maguindanao’s first legislative district, it is not part of the ARMM but of Region XII, having voted
against its inclusion in the ARMM in the plebiscite held in November 1989. Considering the foregoing, the Commission RESOLVED, as it hereby resolves, to adopt the
recommendation of the Law Department that pending the enactment of the appropriate law
by Congress, to maintain the status quo with Cotabato City as part of Shariff Kabunsuan in the
On 28 August 2006, the ARMM’s legislature, the ARMM Regional Assembly, exercising its First Legislative District of Maguindanao. (Emphasis supplied)
power to create provinces under Section 19, Article VI of RA 9054, 5 enacted Muslim Mindanao
Autonomy Act No. 201 (MMA Act 201) creating the Province of Shariff Kabunsuan composed of
the eight municipalities in the first district of Maguindanao. MMA Act 201 provides: However, in preparation for the 14 May 2007 elections, the COMELEC promulgated on 29
March 2007 Resolution No. 7845 stating that Maguindanao’s first legislative district is composed
only of Cotabato City because of the enactment of MMA Act 201. 8
Section 1. The Municipalities of Barira, Buldon, Datu Odin Sinsuat, Kabuntalan, Matanog,
Parang, Sultan Kudarat, Sultan Mastura, and Upi are hereby separated from the Province of
Maguindanao and constituted into a distinct and independent province, which is hereby created, On 10 May 2007, the COMELEC issued Resolution No. 7902, subject of these petitions,
to be known as the Province of Shariff Kabunsuan. amending Resolution No. 07-0407 by renaming the legislative district in question as "Shariff
Kabunsuan Province with Cotabato City (formerly First District of Maguindanao with Cotabato
City)."91avvphi1
xxxx

In G.R. No. 177597, Sema, who was a candidate in the 14 May 2007 elections for
Sec. 5. The corporate existence of this province shall commence upon the appointment by the Representative of "Shariff Kabunsuan with Cotabato City," prayed for the nullification of
Regional Governor or election of the governor and majority of the regular members of the COMELEC Resolution No. 7902 and the exclusion from canvassing of the votes cast in
Sangguniang Panlalawigan. Cotabato City for that office. Sema contended that Shariff Kabunsuan is entitled to one
representative in Congress under Section 5 (3), Article VI of the Constitution10 and Section 3 of
the Ordinance appended to the Constitution.11 Thus, Sema asserted that the COMELEC acted (3) Respondent Dilangalen answered the issue in the negative on the following
without or in excess of its jurisdiction in issuing Resolution No. 7902 which maintained the status grounds: (a) the "province" contemplated in Section 5 (3), Article VI of the Constitution
quo in Maguindanao’s first legislative district despite the COMELEC’s earlier directive in is one that is created by an act of Congress taking into account the provisions in RA
Resolution No. 7845 designating Cotabato City as the lone component of Maguindanao’s 7160 on the creation of provinces; (b) Section 3, Article IV of RA 9054 withheld from
reapportioned first legislative district.12 Sema further claimed that in issuing Resolution No. 7902, the ARMM Regional Assembly the power to enact measures relating to national
the COMELEC usurped Congress’ power to create or reapportion legislative districts. elections, which encompasses the apportionment of legislative districts for members
of the House of Representatives; (c) recognizing a legislative district in every province
the ARMM Regional Assembly creates will lead to the disproportionate representation
In its Comment, the COMELEC, through the Office of the Solicitor General (OSG), chose not to
of the ARMM in the House of Representatives as the Regional Assembly can create
reach the merits of the case and merely contended that (1) Sema wrongly availed of the writ of
provinces without regard to the requirements in Section 461 of RA 7160; and (d)
certiorari to nullify COMELEC Resolution No. 7902 because the COMELEC issued the same in
Cotabato City, which has a population of less than 250,000, is not entitled to a
the exercise of its administrative, not quasi-judicial, power and (2) Sema’s prayer for the writ of
representative in the House of Representatives.
prohibition in G.R. No. 177597 became moot with the proclamation of respondent Didagen P.
Dilangalen (respondent Dilangalen) on 1 June 2007 as representative of the legislative district of
Shariff Kabunsuan Province with Cotabato City. On 27 November 2007, the Court heard the parties in G.R. No. 177597 in oral arguments on the
following issues: (1) whether Section 19, Article VI of RA 9054, delegating to the ARMM
Regional Assembly the power to create provinces, is constitutional; and (2) if in the affirmative,
In his Comment, respondent Dilangalen countered that Sema is estopped from questioning
whether a province created under Section 19, Article VI of RA 9054 is entitled to one
COMELEC Resolution No. 7902 because in her certificate of candidacy filed on 29 March 2007,
representative in the House of Representatives without need of a national law creating a
Sema indicated that she was seeking election as representative of "Shariff Kabunsuan including
legislative district for such new province.15
Cotabato City." Respondent Dilangalen added that COMELEC Resolution No. 7902 is
constitutional because it did not apportion a legislative district for Shariff Kabunsuan or
reapportion the legislative districts in Maguindanao but merely renamed Maguindanao’s first In compliance with the Resolution dated 27 November 2007, the parties in G.R. No. 177597 filed
legislative district. Respondent Dilangalen further claimed that the COMELEC could not their respective Memoranda on the issues raised in the oral arguments. 16 On the question of the
reapportion Maguindanao’s first legislative district to make Cotabato City its sole component unit constitutionality of Section 19, Article VI of RA 9054, the parties in G.R. No. 177597 adopted the
as the power to reapportion legislative districts lies exclusively with Congress, not to mention following positions:
that Cotabato City does not meet the minimum population requirement under Section 5 (3),
Article VI of the Constitution for the creation of a legislative district within a city. 13
(1) Sema contended that Section 19, Article VI of RA 9054 is constitutional (a) as a
valid delegation by Congress to the ARMM of the power to create provinces under
Sema filed a Consolidated Reply controverting the matters raised in respondents’ Comments Section 20 (9), Article X of the Constitution granting to the autonomous regions,
and reiterating her claim that the COMELEC acted ultra vires in issuing Resolution No. 7902. through their organic acts, legislative powers over "other matters as may be
authorized by law for the promotion of the general welfare of the people of the region"
and (b) as an amendment to Section 6 of RA 7160.17 However, Sema concedes that, if
In the Resolution of 4 September 2007, the Court required the parties in G.R. No. 177597 to
taken literally, the grant in Section 19, Article VI of RA 9054 to the ARMM Regional
comment on the issue of whether a province created by the ARMM Regional Assembly under
Assembly of the power to "prescribe standards lower than those mandated" in RA
Section 19, Article VI of RA 9054 is entitled to one representative in the House of
7160 in the creation of provinces contravenes Section 10, Article X of the
Representatives without need of a national law creating a legislative district for such new
Constitution.18 Thus, Sema proposed that Section 19 "should be construed as
province. The parties submitted their compliance as follows:
prohibiting the Regional Assembly from prescribing standards x x x that do not comply
with the minimum criteria" under RA 7160.19
(1) Sema answered the issue in the affirmative on the following grounds: (a) the Court
in Felwa v. Salas14stated that "when a province is created by statute, the
(2) Respondent Dilangalen contended that Section 19, Article VI of RA 9054 is
corresponding representative district comes into existence neither by authority of that
unconstitutional on the following grounds: (a) the power to create provinces was not
statute — which cannot provide otherwise — nor by apportionment, but by operation
among those granted to the autonomous regions under Section 20, Article X of the
of the Constitution, without a reapportionment"; (b) Section 462 of Republic Act No.
Constitution and (b) the grant under Section 19, Article VI of RA 9054 to the ARMM
7160 (RA 7160) "affirms" the apportionment of a legislative district incident to the
Regional Assembly of the power to prescribe standards lower than those mandated in
creation of a province; and (c) Section 5 (3), Article VI of the Constitution and Section
Section 461 of RA 7160 on the creation of provinces contravenes Section 10, Article X
3 of the Ordinance appended to the Constitution mandate the apportionment of a
of the Constitution and the Equal Protection Clause; and
legislative district in newly created provinces.

(3) The COMELEC, through the OSG, joined causes with respondent Dilangalen (thus
(2) The COMELEC, again represented by the OSG, apparently abandoned its earlier
effectively abandoning the position the COMELEC adopted in its Compliance with the
stance on the propriety of issuing Resolution Nos. 07-0407 and 7902 and joined
Resolution of 4 September 2007) and contended that Section 19, Article VI of RA
causes with Sema, contending that Section 5 (3), Article VI of the Constitution is "self-
9054 is unconstitutional because (a) it contravenes Section 10 and Section 6, 20 Article
executing." Thus, every new province created by the ARMM Regional Assembly
X of the Constitution and (b) the power to create provinces was withheld from the
is ipso facto entitled to one representative in the House of Representatives even in the
autonomous regions under Section 20, Article X of the Constitution.
absence of a national law; and
On the question of whether a province created under Section 19, Article VI of RA 9054 is entitled The petitions have no merit. We rule that (1) Section 19, Article VI of RA 9054 is
to one representative in the House of Representatives without need of a national law creating a unconstitutional insofar as it grants to the ARMM Regional Assembly the power to
legislative district for such new province, Sema and respondent Dilangalen reiterated in their create provinces and cities; (2) MMA Act 201 creating the Province of Shariff
Memoranda the positions they adopted in their Compliance with the Resolution of 4 September Kabunsuan is void; and (3) COMELEC Resolution No. 7902 is valid.
2007. The COMELEC deemed it unnecessary to submit its position on this issue considering its
stance that Section 19, Article VI of RA 9054 is unconstitutional.
On the Preliminary Matters

The pendency of the petition in G.R. No. 178628 was disclosed during the oral arguments on 27
The Writ of Prohibition is Appropriate
November 2007. Thus, in the Resolution of 19 February 2008, the Court ordered G.R. No.
to Test the Constitutionality of
178628 consolidated with G.R. No. 177597. The petition in G.R. No. 178628 echoed Sema's
Election Laws, Rules and Regulations
contention that the COMELEC acted ultra vires in issuing Resolution No. 7902 depriving the
voters of Cotabato City of a representative in the House of Representatives. In its Comment to
the petition in G.R. No. 178628, the COMELEC, through the OSG, maintained the validity of The purpose of the writ of Certiorari is to correct grave abuse of discretion by "any tribunal,
COMELEC Resolution No. 7902 as a temporary measure pending the enactment by Congress board, or officer exercising judicial or quasi-judicial functions."21 On the other hand, the writ of
of the "appropriate law." Mandamus will issue to compel a tribunal, corporation, board, officer, or person to perform an
act "which the law specifically enjoins as a duty."22 True, the COMELEC did not issue Resolution
No. 7902 in the exercise of its judicial or quasi-judicial functions.23 Nor is there a law which
The Issues
specifically enjoins the COMELEC to exclude from canvassing the votes cast in Cotabato City
for representative of "Shariff Kabunsuan Province with Cotabato City." These, however, do not
The petitions raise the following issues: justify the outright dismissal of the petition in G.R. No. 177597 because Sema also prayed for
the issuance of the writ of Prohibition and we have long recognized this writ as proper for testing
the constitutionality of election laws, rules, and regulations.24
I. In G.R. No. 177597:

Respondent Dilangalen’s Proclamation


(A) Preliminarily –
Does Not Moot the Petition

(1) whether the writs of Certiorari, Prohibition, and Mandamus are proper to
There is also no merit in the claim that respondent Dilangalen’s proclamation as winner in the 14
test the constitutionality of COMELEC Resolution No. 7902; and
May 2007 elections for representative of "Shariff Kabunsuan Province with Cotabato City"
mooted this petition. This case does not concern respondent Dilangalen’s election. Rather, it
(2) whether the proclamation of respondent Dilangalen as representative of involves an inquiry into the validity of COMELEC Resolution No. 7902, as well as the
Shariff Kabunsuan Province with Cotabato City mooted the petition in G.R. constitutionality of MMA Act 201 and Section 19, Article VI of RA 9054. Admittedly, the outcome
No. 177597. of this petition, one way or another, determines whether the votes cast in Cotabato City for
representative of the district of "Shariff Kabunsuan Province with Cotabato City" will be included
in the canvassing of ballots. However, this incidental consequence is no reason for us not to
(B) On the merits – proceed with the resolution of the novel issues raised here. The Court’s ruling in these petitions
affects not only the recently concluded elections but also all the other succeeding elections for
(1) whether Section 19, Article VI of RA 9054, delegating to the ARMM the office in question, as well as the power of the ARMM Regional Assembly to create in the
Regional Assembly the power to create provinces, cities, municipalities and future additional provinces.
barangays, is constitutional; and
On the Main Issues
(2) if in the affirmative, whether a province created by the ARMM Regional Whether the ARMM Regional Assembly
Assembly under MMA Act 201 pursuant to Section 19, Article VI of RA 9054 Can Create the Province of Shariff Kabunsuan
is entitled to one representative in the House of Representatives without
need of a national law creating a legislative district for such province.
The creation of local government units is governed by Section 10, Article X of the Constitution,
which provides:
II. In G.R No. 177597 and G.R No. 178628, whether COMELEC Resolution No. 7902
is valid for maintaining the status quo in the first legislative district of Maguindanao (as
Sec. 10. No province, city, municipality, or barangay may be created, divided, merged, abolished
"Shariff Kabunsuan Province with Cotabato City [formerly First District of or its boundary substantially altered except in accordance with the criteria established in the
Maguindanao with Cotabato City]"), despite the creation of the Province of Shariff local government code and subject to approval by a majority of the votes cast in a plebiscite in
Kabunsuan out of such district (excluding Cotabato City).
the political units directly affected.

The Ruling of the Court


Thus, the creation of any of the four local government units – province, city, municipality or
barangay – must comply with three conditions. First, the creation of a local government unit must
follow the criteria fixed in the Local Government Code. Second, such creation must not conflict
with any provision of the Constitution. Third, there must be a plebiscite in the political units SECTION 5. (1) The House of Representatives shall be composed of not more than two
affected. hundred and fifty members, unless otherwise fixed by law, who shall be elected from
legislative districts apportioned among the provinces, cities, and the Metropolitan Manila area in
accordance with the number of their respective inhabitants, and on the basis of a uniform and
There is neither an express prohibition nor an express grant of authority in the Constitution for
progressive ratio, and those who, as provided by law, shall be elected through a party-list
Congress to delegate to regional or local legislative bodies the power to create local government
system of registered national, regional, and sectoral parties or organizations.
units. However, under its plenary legislative powers, Congress can delegate to local legislative
bodies the power to create local government units, subject to reasonable standards and
provided no conflict arises with any provision of the Constitution. In fact, Congress has xxxx
delegated to provincial boards, and city and municipal councils, the power to create barangays
within their jurisdiction,25 subject to compliance with the criteria established in the Local
(3) Each legislative district shall comprise, as far as practicable, contiguous, compact,
Government Code, and the plebiscite requirement in Section 10, Article X of the Constitution.
and adjacent territory. Each city with a population of at least two hundred fifty
However, under the Local Government Code, "only x x x an Act of Congress" can create
thousand, or each province, shall have at least one representative.
provinces, cities or municipalities.261avvphi1

(4) Within three years following the return of every census, the Congress shall make
Under Section 19, Article VI of RA 9054, Congress delegated to the ARMM Regional Assembly
a reapportionment of legislative districts based on the standards provided in this
the power to create provinces, cities, municipalities and barangays within the ARMM. Congress
section. (Emphasis supplied)
made the delegation under its plenary legislative powers because the power to create local
government units is not one of the express legislative powers granted by the Constitution to
regional legislative bodies.27 In the present case, the question arises whether the delegation to Section 5 (1), Article VI of the Constitution vests in Congress the power to increase, through a
the ARMM Regional Assembly of the power to create provinces, cities, municipalities and law, the allowable membership in the House of Representatives. Section 5 (4) empowers
barangays conflicts with any provision of the Constitution. Congress to reapportion legislative districts. The power to reapportion legislative districts
necessarily includes the power to create legislative districts out of existing ones. Congress
exercises these powers through a law that Congress itself enacts, and not through a law that
There is no provision in the Constitution that conflicts with the delegation to regional legislative
regional or local legislative bodies enact. The allowable membership of the House of
bodies of the power to create municipalities and barangays, provided Section 10, Article X of the
Representatives can be increased, and new legislative districts of Congress can be created, only
Constitution is followed. However, the creation of provinces and cities is another matter. Section
through a national law passed by Congress. In Montejo v. COMELEC,29 we held that the "power
5 (3), Article VI of the Constitution provides, "Each city with a population of at least two hundred
of redistricting x x x is traditionally regarded as part of the power (of Congress) to make laws,"
fifty thousand, or each province, shall have at least one representative" in the House of
and thus is vested exclusively in Congress.
Representatives. Similarly, Section 3 of the Ordinance appended to the Constitution provides,
"Any province that may hereafter be created, or any city whose population may hereafter
increase to more than two hundred fifty thousand shall be entitled in the immediately following This textual commitment to Congress of the exclusive power to create or reapportion legislative
election to at least one Member x x x." districts is logical. Congress is a national legislature and any increase in its allowable
membership or in its incumbent membership through the creation of legislative districts must be
embodied in a national law. Only Congress can enact such a law. It would be anomalous for
Clearly, a province cannot be created without a legislative district because it will violate Section
regional or local legislative bodies to create or reapportion legislative districts for a national
5 (3), Article VI of the Constitution as well as Section 3 of the Ordinance appended to the
legislature like Congress. An inferior legislative body, created by a superior legislative body,
Constitution. For the same reason, a city with a population of 250,000 or more cannot also be
cannot change the membership of the superior legislative body.
created without a legislative district. Thus, the power to create a province, or a city with a
population of 250,000 or more, requires also the power to create a legislative district. Even the
creation of a city with a population of less than 250,000 involves the power to create a legislative The creation of the ARMM, and the grant of legislative powers to its Regional Assembly under its
district because once the city’s population reaches 250,000, the city automatically becomes organic act, did not divest Congress of its exclusive authority to create legislative districts. This is
entitled to one representative under Section 5 (3), Article VI of the Constitution and Section 3 of clear from the Constitution and the ARMM Organic Act, as amended. Thus, Section 20, Article X
the Ordinance appended to the Constitution. Thus, the power to create a province or city of the Constitution provides:
inherently involves the power to create a legislative district.
SECTION 20. Within its territorial jurisdiction and subject to the provisions of this Constitution
For Congress to delegate validly the power to create a province or city, it must also validly and national laws, the organic act of autonomous regions shall provide for legislative powers
delegate at the same time the power to create a legislative district. The threshold issue then is, over:
can Congress validly delegate to the ARMM Regional Assembly the power to create legislative
districts for the House of Representatives? The answer is in the negative.
(1) Administrative organization;

Legislative Districts are Created or Reapportioned


Only by an Act of Congress (2) Creation of sources of revenues;

(3) Ancestral domain and natural resources;


Under the present Constitution, as well as in past28 Constitutions, the power to increase the
allowable membership in the House of Representatives, and to reapportion legislative districts, is
vested exclusively in Congress. Section 5, Article VI of the Constitution provides: (4) Personal, family, and property relations;
(5) Regional urban and rural planning development; Each legislative district shall comprise, as far as practicable, contiguous, compact, and adjacent
territory. Each city with a population of at least two hundred fifty thousand, or each province,
shall have at least one representative. (Emphasis supplied)
(6) Economic, social, and tourism development;

and Section 3 of the Ordinance appended to the Constitution, which states:


(7) Educational policies;

Any province that may hereafter be created, or any city whose population may hereafter
(8) Preservation and development of the cultural heritage; and
increase to more than two hundred fifty thousand shall be entitled in the immediately
following election to at least one Member or such number of Members as it may be
(9) Such other matters as may be authorized by law for the promotion of the general entitled to on the basis of the number of its inhabitants and according to the standards
welfare of the people of the region. set forth in paragraph (3), Section 5 of Article VI of the Constitution. The number of
Members apportioned to the province out of which such new province was created or where the
city, whose population has so increased, is geographically located shall be correspondingly
Nothing in Section 20, Article X of the Constitution authorizes autonomous regions,
adjusted by the Commission on Elections but such adjustment shall not be made within one
expressly or impliedly, to create or reapportion legislative districts for Congress.
hundred and twenty days before the election. (Emphasis supplied)

On the other hand, Section 3, Article IV of RA 9054 amending the ARMM Organic Act, provides,
serve as bases for the conclusion that the Province of Shariff Kabunsuan, created on 29 October
"The Regional Assembly may exercise legislative power x x x except on the following
2006, is automatically entitled to one member in the House of Representatives in the 14 May
matters: x x x (k) National elections. x x x." Since the ARMM Regional Assembly has no
2007 elections. As further support for her stance, petitioner invokes the statement in Felwa that
legislative power to enact laws relating to national elections, it cannot create a legislative district
"when a province is created by statute, the corresponding representative district comes into
whose representative is elected in national elections. Whenever Congress enacts a law creating existence neither by authority of that statute — which cannot provide otherwise — nor by
a legislative district, the first representative is always elected in the "next national elections" from apportionment, but by operation of the Constitution, without a reapportionment."
the effectivity of the law.30

The contention has no merit.


Indeed, the office of a legislative district representative to Congress is a national office, and its
occupant, a Member of the House of Representatives, is a national official.31 It would be
incongruous for a regional legislative body like the ARMM Regional Assembly to create a First. The issue in Felwa, among others, was whether Republic Act No. 4695 (RA 4695),
national office when its legislative powers extend only to its regional territory. The office of a creating the provinces of Benguet, Mountain Province, Ifugao, and Kalinga-Apayao and
district representative is maintained by national funds and the salary of its occupant is paid out of providing for congressional representation in the old and new provinces, was unconstitutional for
national funds. It is a self-evident inherent limitation on the legislative powers of every local or "creati[ng] congressional districts without the apportionment provided in the Constitution." The
regional legislative body that it can only create local or regional offices, respectively, and it can Court answered in the negative, thus:
never create a national office.
The Constitution ordains:
To allow the ARMM Regional Assembly to create a national office is to allow its legislative
powers to operate outside the ARMM’s territorial jurisdiction. This violates Section 20, Article X
"The House of Representatives shall be composed of not more than one hundred and twenty
of the Constitution which expressly limits the coverage of the Regional Assembly’s legislative
Members who shall be apportioned among the several provinces as nearly as may be according
powers "[w]ithin its territorial jurisdiction x x x."
to the number of their respective inhabitants, but each province shall have at least one Member.
The Congress shall by law make an apportionment within three years after the return of every
The ARMM Regional Assembly itself, in creating Shariff Kabunsuan, recognized the exclusive enumeration, and not otherwise. Until such apportionment shall have been made, the House of
nature of Congress’ power to create or reapportion legislative districts by abstaining from Representatives shall have the same number of Members as that fixed by law for the National
creating a legislative district for Shariff Kabunsuan. Section 5 of MMA Act 201 provides that: Assembly, who shall be elected by the qualified electors from the present Assembly districts.
Each representative district shall comprise as far as practicable, contiguous and compact
territory."
Except as may be provided by national law, the existing legislative district, which includes
Cotabato City as a part thereof, shall remain. (Emphasis supplied)
Pursuant to this Section, a representative district may come into existence: (a) indirectly,
through the creation of a province — for "each province shall have at least one member"
However, a province cannot legally be created without a legislative district because the in the House of Representatives; or (b) by direct creation of several representative
Constitution mandates that "each province shall have at least one representative." Thus, the districts within a province. The requirements concerning the apportionment of representative
creation of the Province of Shariff Kabunsuan without a legislative district is unconstitutional. districts and the territory thereof refer only to the second method of creation of representative
districts, and do not apply to those incidental to the creation of provinces, under the first method.
Sema, petitioner in G.R. No. 177597, contends that Section 5 (3), Article VI of the Constitution, This is deducible, not only from the general tenor of the provision above quoted, but, also, from
which provides: the fact that the apportionment therein alluded to refers to that which is made by an Act of
Congress. Indeed, when a province is created by statute, the corresponding representative
district, comes into existence neither by authority of that statute — which cannot provide
otherwise — nor by apportionment, but by operation of the Constitution, without a Regional Assembly need not comply with the requirement in Section 461(a)(ii) of RA
reapportionment. 7160 that every province created must have a population of at least 250,000; and

There is no constitutional limitation as to the time when, territory of, or other conditions under (3) Representatives from the ARMM provinces can become the majority in the House
which a province may be created, except, perhaps, if the consequence thereof were to exceed of Representatives through the ARMM Regional Assembly’s continuous creation of
the maximum of 120 representative districts prescribed in the Constitution, which is not the effect provinces or cities within the ARMM.
of the legislation under consideration. As a matter of fact, provinces have been created or
subdivided into other provinces, with the consequent creation of additional representative
The following exchange during the oral arguments of the petition in G.R. No. 177597 highlights
districts, without complying with the aforementioned requirements. 32 (Emphasis supplied)
the absurdity of Sema’s position that the ARMM Regional Assembly can create provinces:

Thus, the Court sustained the constitutionality of RA 4695 because (1) it validly created
Justice Carpio:
legislative districts "indirectly" through a special law enacted by Congress creating a province
and (2) the creation of the legislative districts will not result in breaching the maximum number of
legislative districts provided under the 1935 Constitution. Felwa does not apply to the present So, you mean to say [a] Local Government can create legislative district[s] and pack Congress
case because in Felwa the new provinces were created by a national law enacted by with their own representatives [?]
Congress itself. Here, the new province was created merely by a regional law enacted by the
ARMM Regional Assembly.
Atty. Vistan II:35

What Felwa teaches is that the creation of a legislative district by Congress does not emanate
Yes, Your Honor, because the Constitution allows that.
alone from Congress’ power to reapportion legislative districts, but also from Congress’ power to
create provinces which cannot be created without a legislative district. Thus, when a province is
created, a legislative district is created by operation of the Constitution because the Constitution Justice Carpio:
provides that "each province shall have at least one representative" in the House of
Representatives. This does not detract from the constitutional principle that the power to create
legislative districts belongs exclusively to Congress. It merely prevents any other legislative So, [the] Regional Assembly of [the] ARMM can create and create x x x provinces x x x and,
body, except Congress, from creating provinces because for a legislative body to create a therefore, they can have thirty-five (35) new representatives in the House of Representatives
province such legislative body must have the power to create legislative districts. In short, only without Congress agreeing to it, is that what you are saying? That can be done, under your
an act of Congress can trigger the creation of a legislative district by operation of the theory[?]
Constitution. Thus, only Congress has the power to create, or trigger the creation of, a legislative
district. Atty. Vistan II:

Moreover, if as Sema claims MMA Act 201 apportioned a legislative district to Shariff Kabunsuan Yes, Your Honor, under the correct factual circumstances.
upon its creation, this will leave Cotabato City as the lone component of the first legislative
district of Maguindanao. However, Cotabato City cannot constitute a legislative district by itself
because as of the census taken in 2000, it had a population of only 163,849. To constitute Justice Carpio:
Cotabato City alone as the surviving first legislative district of Maguindanao will violate Section 5
(3), Article VI of the Constitution which requires that "[E]ach city with a population of at least two Under your theory, the ARMM legislature can create thirty-five (35) new provinces, there may be
hundred fifty thousand x x x, shall have at least one representative." x x x [only] one hundred thousand (100,000) [population], x x x, and they will each have one
representative x x x to Congress without any national law, is that what you are saying?
Second. Sema’s theory also undermines the composition and independence of the House of
Representatives. Under Section 19,33 Article VI of RA 9054, the ARMM Regional Assembly can Atty. Vistan II:
create provinces and cities within the ARMM with or without regard to the criteria fixed in Section
461 of RA 7160, namely: minimum annual income of ₱20,000,000, and minimum contiguous
territory of 2,000 square kilometers or minimum population of 250,000. 34The following scenarios Without law passed by Congress, yes, Your Honor, that is what we are saying.
thus become distinct possibilities:
xxxx
(1) An inferior legislative body like the ARMM Regional Assembly can create 100 or
more provinces and thus increase the membership of a superior legislative body, the Justice Carpio:
House of Representatives, beyond the maximum limit of 250 fixed in the Constitution
(unless a national law provides otherwise);
So, they can also create one thousand (1000) new provinces, sen[d] one thousand (1000)
representatives to the House of Representatives without a national law[,] that is legally possible,
(2) The proportional representation in the House of Representatives based on one correct?
representative for at least every 250,000 residents will be negated because the ARMM
Atty. Vistan II: Resolution No. 7902 Complies with the Constitution

Yes, Your Honor.36 (Emphasis supplied) Consequently, we hold that COMELEC Resolution No. 7902, preserving the geographic and
legislative district of the First District of Maguindanao with Cotabato City, is valid as it merely
complies with Section 5 of Article VI and Section 20 of Article X of the Constitution, as well as
Neither the framers of the 1987 Constitution in adopting the provisions in Article X on regional
Section 1 of the Ordinance appended to the Constitution.
autonomy,37 nor Congress in enacting RA 9054, envisioned or intended these disastrous
consequences that certainly would wreck the tri-branch system of government under our
Constitution. Clearly, the power to create or reapportion legislative districts cannot be delegated WHEREFORE, we declare Section 19, Article VI of Republic Act No. 9054
by Congress but must be exercised by Congress itself. Even the ARMM Regional Assembly UNCONSTITUTIONAL insofar as it grants to the Regional Assembly of the Autonomous Region
recognizes this. in Muslim Mindanao the power to create provinces and cities. Thus, we declare VOID Muslim
Mindanao Autonomy Act No. 201 creating the Province of Shariff Kabunsuan. Consequently, we
rule that COMELEC Resolution No. 7902 is VALID.
The Constitution empowered Congress to create or reapportion legislative districts, not the
regional assemblies. Section 3 of the Ordinance to the Constitution which states, "[A]ny province
that may hereafter be created x x x shall be entitled in the immediately following election to at Let a copy of this ruling be served on the President of the Senate and the Speaker of the House
least one Member," refers to a province created by Congress itself through a national law. The of Representatives.
reason is that the creation of a province increases the actual membership of the House of
Representatives, an increase that only Congress can decide. Incidentally, in the present 14th
SO ORDERED.
Congress, there are 21938 district representatives out of the maximum 250 seats in the House of
Representatives. Since party-list members shall constitute 20 percent of total membership of the
House, there should at least be 50 party-list seats available in every election in case 50 party-list
candidates are proclaimed winners. This leaves only 200 seats for district representatives, much
less than the 219 incumbent district representatives. Thus, there is a need now for Congress to
increase by law the allowable membership of the House, even before Congress can create new
provinces.

It is axiomatic that organic acts of autonomous regions cannot prevail over the Constitution.
Section 20, Article X of the Constitution expressly provides that the legislative powers of regional
assemblies are limited "[w]ithin its territorial jurisdiction and subject to the provisions of
the Constitution and national laws, x x x." The Preamble of the ARMM Organic Act (RA 9054)
itself states that the ARMM Government is established "within the framework of the
Constitution." This follows Section 15, Article X of the Constitution which mandates that the
ARMM "shall be created x x x within the framework of this Constitution and the national
sovereignty as well as territorial integrity of the Republic of the Philippines."

The present case involves the creation of a local government unit that necessarily involves also
the creation of a legislative district. The Court will not pass upon the constitutionality of the
creation of municipalities and barangays that does not comply with the criteria established in
Section 461 of RA 7160, as mandated in Section 10, Article X of the Constitution, because the
creation of such municipalities and barangays does not involve the creation of legislative
districts. We leave the resolution of this issue to an appropriate case.

In summary, we rule that Section 19, Article VI of RA 9054, insofar as it grants to the ARMM
Regional Assembly the power to create provinces and cities, is void for being contrary to Section
5 of Article VI and Section 20 of Article X of the Constitution, as well as Section 3 of the
Ordinance appended to the Constitution. Only Congress can create provinces and cities
because the creation of provinces and cities necessarily includes the creation of legislative
districts, a power only Congress can exercise under Section 5, Article VI of the Constitution and
Section 3 of the Ordinance appended to the Constitution. The ARMM Regional Assembly cannot
create a province without a legislative district because the Constitution mandates that every
province shall have a legislative district. Moreover, the ARMM Regional Assembly cannot enact
a law creating a national office like the office of a district representative of Congress because the
legislative powers of the ARMM Regional Assembly operate only within its territorial jurisdiction
as provided in Section 20, Article X of the Constitution. Thus, we rule that MMA Act 201, enacted
by the ARMM Regional Assembly and creating the Province of Shariff Kabunsuan, is void.
G.R. No. L-13678 November 12, 1918 The following penalties shall be imposed upon those who play the above games on
days other than Sundays and official holidays:
THE UNITED STATES, plaintiff-appellee,
vs. For the owner of the house: A fine of from Ten to Two hundred pesos, or subsidiary
PRUDENCIO SALAVERIA, defendant-appellant. imprisonment in case of insolvency at the rate of one peso a day.

MALCOLM, J.: For the gamblers: A fine of from Five to Two hundred pesos each or subsidiary
imprisonment in case of insolvency at the rate of one peso a day.
The municipal council of Orion, Bataan, enacted, on February 28, 1917, an ordinance which,
among other things, prohibited the playing of panguingue on days not Sundays or legal holidays, The Philippine Legislature has granted to municipalities legislative powers of a dual character,
and penalized the violation thereof by a casero [housekeeper] by a fine of not less than P10 nor one class mandatory an the other discretionary. Of the first class is the provision of the
more than P200, and by jugadores [gamblers] by a fine of not less than P5 nor more than P200. Administrative Code which makes it the duty of the municipal council, conformably with law, "to
The justice of the peace of Orion, when this ordinance went into effect, was Prudencio Salaveria, prohibit and penalize . . . gambling." (Sec. 2188 [i], Adm. Code of 1916; sec. 2242 [i], Adm. Code
now the defendant and appellant. Notwithstanding his official station, on the evening of March 8, of 1917.) This is a more restricted power than that found in the original Municipal Code which
1917, not a Sunday or legal holiday, seven persons including the justice of the peace an his wife authorized a municipal council to "provide against the evils of gambling, gambling houses, and
were surprised by the police while indulging in a game of panguingue in the house of the justice disorderly houses of whatsoever sort." (Act No. 82, sec. 39 [u].) The present municipal law,
of the peace. The chief of police took possession of the cards, the counters (sigayes), a tray, an since making use of the word "gambling," must be construed with reference to the Insular Law,
P2.07 in money, used in the game. Act No. 1757, relating to the same subject. Act No. 1757 in section 1 defines "gambling" as "the
paying of any game for money or any representative of value or valuable consideration or thing,
the result of which game depends wholly or chiefly upon chance or hazard, or the use of any
These are facts fully proven by the evince and by the admissions of the accused. Convicted in
mechanical inventions or contrivance to determine by chance the loser or winner of money or of
the justice of the peace court of Orion, and again in the Court of First Instance of Bataan,
any representative of value or of any valuable consideration or thing." In the United
Salaveria appeals to this court, making five assignments of error. The three assignments, of a
States vs. Hilario ([1913], 24 Phil., 392), the Supreme Court went into the subject of the meaning
technical nature, are without merit, and a fourth, relating to the evidence, is not sustained by the
of "gambling" in this jurisdiction, and found that it includes those games the result of which
proof. The remaining assignment of error, questioning the validity of the ordinance under which
depend wholly or chiefly upon chance or hazard, and excludes those games the result of which
the accused was convicted, requires serious consideration and final resolution. This ordinance in
depend wholly or chiefly upon skill, with the result that sections 621 to 625 of the Revise
part reads:
Ordinances of the city of Manila (734-738 of the Revised Ordinances of 1917) were found to
prohibit only games of chance or hazard.
RESOLUTION NO. 28
The ordinance of Orion, Bataan, merely prohibits the playing of panguingue on certain days,
xxx xxx xxx without describing it. Further, although this court has considered the method by which many
other games are played, it has never as yet authoritatively decided whether panguingue was a
game of skill or hazard. Nor was any evidence on this point introduced in the present case.
Whereas, this Council is vested with certain powers by sections 2184 and 2185 of the However, a reading of the decision of the trial court and of official opinions of two Attorneys-
Administrative Code; General, of which we can take judicial cognizance, warrants the deduction that panguingue is
not a game of chance or hazard and is not prohibited by Act No. 1757. (See Opinions of the
Whereas, it is the moral duty of this body to safeguard the tranquillity and stability of Attorney-General of July 11, 1904; July 25, 1904; October 10, 1905; and September 7, 1911;
the Government and to foster the welfare and prosperity of each an all of the also Berriz, Diccionario de la Administracion, p. 35.) If, therefore, we were to restrict our
inhabitants of this municipality; therefore, investigation to those portions of the Administrative Code which authorize a municipal council to
prohibit and penalize gambling, there would exist grave doubt, to say the least, of the validity of
ordinance No. 3 of the municipality of Orion, Bataan.
Be it resolved to enact, as it hereby is enacted, the following ordinance:

There remains for consideration a different approach to the question.


Ordinance No. 3

While Philippine law gives to gambling a restricted meaning, it is to be noted that, in its broader
xxx xxx xxx signification, gambling relates to play by certain rules at cards, dice, or other contrivance, so that
one shall be the loser an the other the winner. (20 Cyc., 878; Bouvier's Law Dictionary;
Third. — The games known as "Panguingue" "Manilla," "Jung-kiang," "Paris-Paris," People vs. Todd [1889], 51 Hun [N. Y.], 446 451; 4 N. Y. Supp., 25.) As one example the
"Poker," "Tute," "Burro," and "Treinta-y-uno" shall be allowed only on Sundays an Charter of the town of Ruston, State of Louisiana, authorized it "to restrain, prohibit, an suppress
official holidays. . . . games and gambling houses and rooms . . ., and to provide for the punishment of the
persons engaged in the same." Under this power the town passed an ordinance prohibiting "all
games of chance, lottery, banking games, raffling, and all other species of gambling," indicating
xxx xxx xxx that there were other species of gambling in addition to games of chance. (See Town of
Ruston vs. Perkins [1905], 114 La., 851.) The common law notion of gambling, which only made
it an indictable offense when the play was attended by such circumstances as would in The general welfare clause has two branches. One branch attaches itself to the main trunk of
themselves amount to a riot or a nuisance or to an actual breach of the peace, has given way to municipal authority, and relates to such ordinances and regulations as may be necessary to
statutes and ordinances designed to restrain, suppress, or control gambling. carry into effect and discharge the powers and duties conferred upon the municipal council by
law. With this class we are not here directly concerned. The second branch of the clause is
much more independent of the specific functions of the council which are enumerated by law. It
Authority for the State or a municipality to take action to control gambling in this larger sense can
authorizes such ordinances "as shall seem necessary and proper to provide for the health and
be found in an analysis of what is calle the police power.
safety, promote the prosperity, improve the morals, peace, good order, comfort, and
convenience of the municipality and the inhabitants thereof, and for the protection of property
Any attempt to define the police power with circumstantial precision would savor of pedantry. therein."
The United States Supreme Court tritely describes it as "the most essential of all powers, at
times the most insistent, an always one of least limitable of the powers of government." (District
It is a general rule that ordinances passed by virtue of the implied power found in the general
of Columbia vs. Brooks [1909], 214 U.S., 138.) The police power is based on the maxim "salus
powers and purposes of the corporation, and not inconsistent with the laws or policy of the State.
populi est suprema lex" — the welfare of the people is the first law. The United States Supreme
The ordinance of the municipality of Orion does not seem in itself to be pernicious, or
Court has said that it extends "to the protection of the lives, health and property of the citizens,
unreasonable or discriminatory. Its purposes evidently are to improve the morals and stimulate
and to the preservation of good order and the public morals." (Beer Co. vs. Massachusetts
the industry of the people. A person is to be compelled to refrain from private acts injurious both
[1878] , 97 U.S., 25; Barbier vs. Connolly [1885], 113 U.S., 27.) The Supreme Court of these
to himself an his neighbors. These objects, to be attained by limiting the pastime to definite days,
Islands has said that it extends "the police power of the state includes not only the public health
do not infringe any law of the general government.
safety, but also the public welfare, protection against impositions, and generally the public's best
interest." (U.S. vs. Pompeya [1915], 31 Phil., 245.) Recent judicial decisions incline to give a
more extensive scope to the police power that the older cases. The public welfare is rightfully The constitutional provision that no person shall be deprived of liberty without due process of law
made the basis of construction. is not violated by this ordinance. Liberty of action by the individual is not unduly circumscribed;
that is, it is not unduly circumscribed if we have in mind the correct notion of this "the greatest of
all rights." That gravest of sociological questions — How far, consistently with freedom, may the
Not only does the State effectuate its purposes through the exercise of the police power but the
liberties of the individual member of society be subordinated to the will of the Government? —
municipality does also. Like the State, the police power of a municipal corporation extends to all
has been debated for centuries, in vain, if we can not now discount the time worn objection to
matters affecting the peace, order, health, morals, convenience, comfort, and safety of its
any and all interference with private rights in order to effectuate the public purpose. (See
citizens — the security of social order — the best and highest interests of the municipality.
Jacobson vs. Massachusetts [1905], 197 U. S., 11; State vs. Kreutzberg [1902], 58 L. R. A.,
(Case vs. Board of Health of Manila and Heiser [1913], 24 Phil., 250.) The best considered
748.) Almost countless are the governmental restrictions on the citizen.
decisions have tended to broaden the scope of action of the municipality in dealing with police
offenses. Within the general police powers of a municipal corporation is the suppression of
gambling. Ordinances aimed in a reasonable way at the accomplishment of this purpose are The presumption is all favor of validity. The inhabitants of a municipality are in themselves
undoubtedly valid. (See U.S. vs. Pacis [1915], 31 Phil., 524; 39 L. R. A., 523, Note; Cooley's miniature states. The action of the elected representatives of the people cannot be lightly set
Constitutional Limitations, 6th edition, pp. 138, 226, 742; Greenville vs.Kemmis [1900], 58 S. C., aside. The councilors must, in the very nature of things, be familiar with the necessities of their
427 [holding that under the general welfare clause a city may pass an ordinance prohibiting particular municipality an with all the facts and circumstances which surround the subject, and
gambling in any private house].) necessities of their particular municipality and with all the facts and circumstances which
surround the subject, and necessitate action. The local legislative body, by enacting the
ordinance, has in effect given notice that the regulations are essential to the well being of the
The Philippine Legislature, as before intimated, delegated to municipalities certain legislative
people. Who is in a better position to say whether the playing of panguingue is deleterious to
powers are named specifically. But in addition, and preceding both the specific powers of a
social order and the public interest in a certain municipality — the municipal council, or the
mandatory and discretionary character, is the general power of a municipal council to enact
courts? The answer is self-evident. The Judiciary should not lightly set aside legislative action
ordinances and make regulations. It is this grant that the preamble of the ordinance of Orion
when there is not a clear invasion of personal or property rights under the guise of police
assigns as authority for its enactment. Said section 2184 of the Administrative Code of 1916
regulation. (See U.S. vs. Joson [1913], 26 Phil., 1.)
(sec. 2238, Adm. Code of 1917) reads:

President McKinley's Instructions to the Commission still remain undisturbed by subsequent


The municipal council shall enact such ordinances and make such regulations, not
Acts of Congress dealing with Philippine affairs and yet constitute a portion of our constitutional
repugnant to law, as may be necessary to carry into effect and discharge the powers
law, as to the inviolable rule that "municipal governments . . . shall be afforded the opportunity to
an duties conferred upon it by law an such as shall seem necessary and proper to
manage their own affairs to the fullest extent of which they are capable." Again the same organic
provide for the health and safety, promote the prosperity, improve the morals, peace,
law says, "In the distribution of powers among the governments organized by the Commission,
good order, comfort, and convenience of the municipality and the inhabitants
the presumption is always to be in favor of the smaller subdivision, so that all the powers which
thereof, and for the protection of property therein.
can properly be exercised by the municipal government shall be vested in that government . . . ."
Let us never forget these principles so highly protective of local self-government.
This section, known as the general welfare clause, delegates in statutory form the police power
to a municipality. As above stated, this clause has been given wide application by municipal
The judiciary can very well take notice of the fact that municipalities are accustomed to enacting
authorities and has in its relation to the particular circumstances of the case been liberally
ordinances aimed at the regulation of gambling. The executive authorities an the Attorney-
construed by the courts. Such, it is well to recall, is the progressive view of Philippine
General have usually upheld the validity of such ordinances, especially those intended to restrict
jurisprudence.
the playing of panguingue. (Opinions of the Attorney-General, supra; Opinion of the Executive
Secretary, July 6, 1909; Indorsement of the Governor-General, July 21, 1904.) This general
municipal practice, indicative of a social cancer to be eradicated, should not be discouraged by
strict judicial construction.

More important still, the courts cannot but realize that gambling, in its larger sense as well as in
its restricted sense, is an act beyond the pale of good morals, which, for the welfare of the
Filipino people, should be exterminated. The suppression of the evil does not interfere with any
of the inherent rights of citizenship. The pernicious practice is rightfully regarded as the offspring
of idleness and the prolific parent of vice and immorality, demoralizing in its association and
tendencies, detrimental to the best interests of society, and encouraging wastefulness,
thriftlessness, and a belief that a livelihood may be earned by other means than honest industry.
To be condemned in itself, it has the further effect of causing poverty, dishonesty, fraud, and
deceit. Many a man has neglected his business and mortgaged his integrity to follow the fickle
Goddess of the cards. Many a woman has wasted her hours and squandered her substance at
the gambling board while home and children were forgotten. It is highly proper that this pastime
should be subject to the control of restraints imposed by the ordinances of local governments
peculiarly afflicted by the evil. (See In re Voss [1903], 11 N. D., 540; Ex parte Tuttle [1891], 91,
Cal., 589; Greenwood vs. State [1873], 6 Baxt., 567; 32 Am. Rep., 539; 12 R. C. L., 709-715.)

For the suppression of such an evil, coordinate and harmonious action must concur between the
three departments of Government. A law or ordinance enacted by the legislative body must
exist. Such an ordinance is before us. Vigorous executive enforcement must take place to make
the law or ordinance a reality. Such activity by the police has brought this case to the courts. And
finally the Judiciary, having full respect for the legislative action of the municipal council and for
the prosecution by the executive officials, must, by judicial construction, equally as progressive
and constructive, give effect to the action of the other two powers. Wherefore,
although panguingue is not entirely a game of chance, since it is a proper subject for regulation
by municipal authorities acting under their delegated police power, whose laudable intention is to
improve the public morals and promote the prosperity of their people, their action should be
upheld by the courts. Ordinance No. 3 of Orion, Bataan, is found to be valid.

The culprit in this case is himself a member of the Judiciary. Instead of enforcing the law, he has
scorned it. His example to the people of Orion has been pernicious in its influence. If gambling is
to be suppressed, not only the weak and ignorant must be punished, but those with full
knowledge of the law and the consequences of violation. We would accordingly suggest to
Courts of First Instance that in all cases arising under the Gambling Law or ordinances, except
for unusual circumstances, a prison sentence should be imposed, if permitted by the law or
ordinance. We further suggest that, where the defendant has been found guilty and is a man of
station, he be given the maximum penalty.lawphil.net

Applying the foregoing in this instance, it results that the defendant and appellant must be found
guilty of a violation of ordinance No. 3 of the municipality of Orion, Bataan; and, in accordance
therewith, shall be sentenced to the maximum penalty of the payment of a fine of P200, or to
subsidiary imprisonment in case of insolvency, with the costs of all three instances against him.
So ordered.
G.R. No. 161107 March 12, 2013 which shall embody up-to-date and modern technical design in the construction of fences of
residential, commercial and industrial buildings;
HON. MA. LOURDES C. FERNANDO, in her capacity as City Mayor of Marikina City,
JOSEPHINE C. EVANGELIST A, in her capacity as Chief, Permit Division, Office of the WHEREAS, Presidential Decree No. 1096, otherwise known as the National Building Code of
City Engineer, and ALFONSO ESPIRITU, in his capacity as City Engineer of Marikina the Philippines, does not adequately provide technical guidelines for the construction of fences,
City, Petitioners, in terms of design, construction, and criteria;
vs.
ST. SCHOLASTICA'S COLLEGE and ST. SCHOLASTICA'S ACADEMY-MARIKINA,
WHEREAS, the adoption of such technical standards shall provide more efficient and effective
INC., Respondents.
enforcement of laws on public safety and security;

DECISION
WHEREAS, it has occurred in not just a few occasions that high fences or walls did not actually
discourage but, in fact, even protected burglars, robbers, and other lawless elements from the
MENDOZA, J.: view of outsiders once they have gained ingress into these walls, hence, fences not necessarily
providing security, but becomes itself a "security problem";
Before this Court is a petition for review on certiorari under Rule 45 of the Rules of Court, which
seeks to set aside the December 1, 2003 Decision1 of the Court of Appeals (CA) in CA-G.R. SP WHEREAS, to discourage, suppress or prevent the concealment of prohibited or unlawful acts
No. 75691. earlier enumerated, and as guardian of the people of Marikina, the municipal government seeks
to enact and implement rules and ordinances to protect and promote the health, safety and
morals of its constituents;
The Facts

WHEREAS, consistent too, with the "Clean and Green Program" of the government, lowering of
Respondents St. Scholastica’s College (SSC) and St. Scholastica’s Academy-Marikina, Inc.
fences and walls shall encourage people to plant more trees and ornamental plants in their
(SSA-Marikina) are educational institutions organized under the laws of the Republic of the
yards, and when visible, such trees and ornamental plants are expected to create an aura of a
Philippines, with principal offices and business addresses at Leon Guinto Street, Malate, Manila,
clean, green and beautiful environment for Marikeños;
and at West Drive, Marikina Heights, Marikina City, respectively. 2

WHEREAS, high fences are unsightly that, in the past, people planted on sidewalks to "beautify"
Respondent SSC is the owner of four (4) parcels of land measuring a total of 56,306.80 square
the façade of their residences but, however, become hazards and obstructions to pedestrians;
meters, located in Marikina Heights and covered by Transfer Certificate Title (TCT) No. 91537.
Located within the property are SSA-Marikina, the residence of the sisters of the Benedictine
Order, the formation house of the novices, and the retirement house for the elderly sisters. The WHEREAS, high and solid walls as fences are considered "un-neighborly" preventing
property is enclosed by a tall concrete perimeter fence built some thirty (30) years ago. Abutting community members to easily communicate and socialize and deemed to create "boxed-in"
the fence along the West Drive are buildings, facilities, and other improvements. 3 mentality among the populace;

The petitioners are the officials of the City Government of Marikina. On September 30, 1994, the WHEREAS, to gather as wide-range of opinions and comments on this proposal, and as a
Sangguniang Panlungsod of Marikina City enacted Ordinance No. 192, 4 entitled "Regulating the requirement of the Local Government Code of 1991 (R.A. 7160), the Sangguniang Bayan of
Construction of Fences and Walls in the Municipality of Marikina." In 1995 and 1998, Ordinance Marikina invited presidents or officers of homeowners associations, and commercial and
Nos. 2175 and 2006 were enacted to amend Sections 7 and 5, respectively. Ordinance No. 192, industrial establishments in Marikina to two public hearings held on July 28, 1994 and August
as amended, is reproduced hereunder, as follows: 25, 1994;

ORDINANCE No. 192 WHEREAS, the rationale and mechanics of the proposed ordinance were fully presented to the
Series of 1994 attendees and no vehement objection was presented to the municipal government;

ORDINANCE REGULATING THE CONSTRUCTION OF FENCES AND WALLS IN THE NOW, THEREFORE, BE IT ORDAINED BY THE SANGGUINANG BAYAN OF MARIKINA IN
MUNICIPALITY OF MARIKINA SESSION DULY ASSEMBLED:

WHEREAS, under Section 447.2 of Republic Act No. 7160 otherwise known as the Local Section 1. Coverage: This Ordinance regulates the construction of all fences, walls and gates on
Government Code of 1991 empowers the Sangguniang Bayan as the local legislative body of lots classified or used for residential, commercial, industrial, or special purposes.
the municipality to "x x x Prescribe reasonable limits and restraints on the use of property within
the jurisdiction of the municipality, x x x";
Section 2. Definition of Terms:

WHEREAS the effort of the municipality to accelerate its economic and physical development,
a. Front Yard – refers to the area of the lot fronting a street, alley or public
coupled with urbanization and modernization, makes imperative the adoption of an ordinance
thoroughfare.
b. Back Yard – the part of the lot at the rear of the structure constructed therein. fencing permits, and demolition of non-conforming walls at the lapse of the grace period herein
provided.
c. Open fence – type of fence which allows a view of "thru-see" of the inner yard and
the improvements therein. (Examples: wrought iron, wooden lattice, cyclone wire) Section 10. Repealing Clause. All existing Ordinances and Resolutions, Rules and Regulations
inconsistent with the foregoing provisions are hereby repealed, amended or modified.
d. Front gate – refers to the gate which serves as a passage of persons or vehicles
fronting a street, alley, or public thoroughfare. Section 11. Separability Clause. If for any reason or reasons, local executive orders, rules and
regulations or parts thereof in conflict with this Ordinance are hereby repealed and/or modified
accordingly.
Section 3. The standard height of fences or walls allowed under this ordinance are as follows:

Section 12. Effectivity. This ordinance takes effect after publication.


(1) Fences on the front yard – shall be no more than one (1) meter in height. Fences
in excess of one (1) meter shall be of an open fence type, at least eighty percent
(80%) see-thru; and APPROVED: September 30, 1994

(2) Fences on the side and back yard – shall be in accordance with the provisions of (Emphases supplied)
P.D. 1096 otherwise known as the National Building Code.
On April 2, 2000, the City Government of Marikina sent a letter to the respondents ordering them
Section 4. No fence of any kind shall be allowed in areas specifically reserved or classified as to demolish and replace the fence of their Marikina property to make it 80% see-thru, and, at the
parks. same time, to move it back about six (6) meters to provide parking space for vehicles to
park.9 On April 26, 2000, the respondents requested for an extension of time to comply with the
directive.10 In response, the petitioners, through then City Mayor Bayani F. Fernando, insisted on
Section 5. In no case shall walls and fences be built within the five (5) meter parking area
the enforcement of the subject ordinance.
allowance located between the front monument line and the building line of commercial and
industrial establishments and educational and religious institutions. 7
Not in conformity, the respondents filed a petition for prohibition with an application for a writ of
preliminary injunction and temporary restraining order before the Regional Trial Court, Marikina,
Section 6. Exemption.
Branch 273 (RTC), docketed as SCA Case No. 2000-381-MK.11

(1) The Ordinance does not cover perimeter walls of residential subdivisions.
The respondents argued that the petitioners were acting in excess of jurisdiction in enforcing
Ordinance No. 192, asserting that such contravenes Section 1, Article III of the 1987
(2) When public safety or public welfare requires, the Sangguniang Bayan may allow Constitution. That demolishing their fence and constructing it six (6) meters back would result in
the construction and/or maintenance of walls higher than as prescribed herein and the loss of at least 1,808.34 square meters, worth about ₱9,041,700.00, along West Drive, and
shall issue a special permit or exemption. at least 1,954.02 square meters, worth roughly ₱9,770,100.00, along East Drive. It would also
result in the destruction of the garbage house, covered walk, electric house, storage house,
comfort rooms, guards’ room, guards’ post, waiting area for visitors, waiting area for students,
Section 7. Transitory Provision. Real property owners whose existing fences and walls do not Blessed Virgin Shrine, P.E. area, and the multi-purpose hall, resulting in the permanent loss of
conform to the specifications herein are allowed adequate period of time from the passage of their beneficial use. The respondents, thus, asserted that the implementation of the ordinance on
this Ordinance within which to conform, as follows: their property would be tantamount to an appropriation of property without due process of law;
and that the petitioners could only appropriate a portion of their property through eminent
(1) Residential houses – eight (8) years domain. They also pointed out that the goal of the provisions to deter lawless elements and
criminality did not exist as the solid concrete walls of the school had served as sufficient
protection for many years.12
(2) Commercial establishments – five (5) years

The petitioners, on the other hand, countered that the ordinance was a valid exercise of police
(3) Industrial establishments – three (3) years power, by virtue of which, they could restrain property rights for the protection of public safety,
health, morals, or the promotion of public convenience and general prosperity. 13
(4) Educational institutions – five (5) years8 (public and privately owned)
On June 30, 2000, the RTC issued a writ of preliminary injunction, enjoining the petitioners from
Section 8. Penalty. Walls found not conforming to the provisions of this Ordinance shall be implementing the demolition of the fence at SSC’s Marikina property.14
demolished by the municipal government at the expense of the owner of the lot or structure.
Ruling of the RTC
Section 9. The Municipal Engineering Office is tasked to strictly implement this ordinance,
including the issuance of the necessary implementing guidelines, issuance of building and
On the merits, the RTC rendered a Decision,15 dated October 2, 2002, granting the petition and Ruling of the CA
ordering the issuance of a writ of prohibition commanding the petitioners to permanently desist
from enforcing or implementing Ordinance No. 192 on the respondents’ property.
In its December 1, 2003 Decision, the CA dismissed the petitioners’ appeal and affirmed the
RTC decision.
The RTC agreed with the respondents that the order of the petitioners to demolish the fence at
the SSC property in Marikina and to move it back six (6) meters would amount to an
The CA reasoned out that the objectives stated in Ordinance No. 192 did not justify the exercise
appropriation of property which could only be done through the exercise of eminent domain. It
of police power, as it did not only seek to regulate, but also involved the taking of the
held that the petitioners could not take the respondents’ property under the guise of police power
respondents’ property without due process of law. The respondents were bound to lose an
to evade the payment of just compensation.
unquantifiable sense of security, the beneficial use of their structures, and a total of 3,762.36
square meters of property. It, thus, ruled that the assailed ordinance could not be upheld as valid
It did not give weight to the petitioners’ contention that the parking space was for the benefit of as it clearly invaded the personal and property rights of the respondents and "[f]or being
the students and patrons of SSA-Marikina, considering that the respondents were already unreasonable, and undue restraint of trade."17
providing for sufficient parking in compliance with the standards under Rule XIX of the National
Building Code.
It noted that although the petitioners complied with procedural due process in enacting
Ordinance No. 192, they failed to comply with substantive due process. Hence, the failure of the
It further found that the 80% see-thru fence requirement could run counter to the respondents’ respondents to attend the public hearings in order to raise objections did not amount to a waiver
right to privacy, considering that the property also served as a residence of the Benedictine of their right to question the validity of the ordinance.
sisters, who were entitled to some sense of privacy in their affairs. It also found that the
respondents were able to prove that the danger to security had no basis in their case. Moreover,
The CA also shot down the argument that the five-meter setback provision for parking was a
it held that the purpose of beautification could not be used to justify the exercise of police power.
legal easement, the use and ownership of which would remain with, and inure to, the benefit of
the respondents for whom the easement was primarily intended. It found that the real intent of
It also observed that Section 7 of Ordinance No. 192, as amended, provided for retroactive the setback provision was to make the parking space free for use by the public, considering that
application. It held, however, that such retroactive effect should not impair the respondents’ such would cease to be for the exclusive use of the school and its students as it would be
vested substantive rights over the perimeter walls, the six-meter strips of land along the walls, situated outside school premises and beyond the school administration’s control.
and the building, structures, facilities, and improvements, which would be destroyed by the
demolition of the walls and the seizure of the strips of land.
In affirming the RTC ruling that the ordinance was not a curative statute, the CA found that the
petitioner failed to point out any irregularity or invalidity in the provisions of the National Building
The RTC also found untenable the petitioners’ argument that Ordinance No. 192 was a remedial Code that required correction or cure. It noted that any correction in the Code should be properly
or curative statute intended to correct the defects of buildings and structures, which were undertaken by the Congress and not by the City Council of Marikina through an ordinance.
brought about by the absence or insufficiency of laws. It ruled that the assailed ordinance was
neither remedial nor curative in nature, considering that at the time the respondents’ perimeter
The CA, thus, disposed:
wall was built, the same was valid and legal, and the ordinance did not refer to any previous
legislation that it sought to correct.
WHEREFORE, all foregoing premises considered, the instant appeal is DENIED.1âwphi1 The
October 2, 2002 Decision and the January 13, 2003 Order of the Regional Trial Court (RTC) of
The RTC noted that the petitioners could still take action to expropriate the subject property
Marikina City, Branch 273, granting petitioners-appellees’ petition for Prohibition in SCA Case
through eminent domain.
No. 2000-381-MK are hereby AFFIRMED.

The RTC, thus, disposed:


SO ORDERED.18

WHEREFORE, the petition is GRANTED. The writ of prohibition is hereby issued commanding
Aggrieved by the decision of the CA, the petitioners are now before this Court presenting the
the respondents to permanently desist from enforcing or implementing Ordinance No. 192,
following
Series of 1994, as amended, on petitioners’ property in question located at Marikina Heights,
Marikina, Metro Manila.
ASSIGNMENT OF ERRORS
No pronouncement as to costs.
1. WHETHER OR NOT THE HONORABLE COURT OF APPEALS ERRED IN
DECLARING THAT CITY ORDINANCE NO. 192, SERIES OF 1994 IS NOT A VALID
SO ORDERED.16
EXERCISE OF POLICE POWER;

2. WHETHER OR NOT THE HONORABLE COURT OF APPEALS ERRED IN


RULING THAT THE AFOREMENTIONED ORDINANCE IS AN EXERCISE OF THE
CITY OF THE POWER OF EMINENT DOMAIN;
3. WHETHER OR NOT THE HONORABLE COURT OF APPEALS ERRED IN strict scrutiny, the focus is on the presence of compelling, rather than substantial, governmental
DECLARING THAT THE CITY VIOLATED THE DUE PROCESS CLAUSE IN interest and on the absence of less restrictive means for achieving that interest. 27
IMPLEMENTING ORDINANCE NO. 192, SERIES OF 1994; AND
Even without going to a discussion of the strict scrutiny test, Ordinance No. 192, series of 1994
4. WHETHER OR NOT THE HONORABLE COURT OF APPEALS ERRED IN must be struck down for not being reasonably necessary to accomplish the City’s purpose. More
RULING THAT THE ABOVE-MENTIONED ORDINANCE CANNOT BE GIVEN importantly, it is oppressive of private rights.
RETROACTIVE APPLICATION.19
Under the rational relationship test, an ordinance must pass the following requisites as
In this case, the petitioners admit that Section 5 of the assailed ordinance, pertaining to the five- discussed in Social Justice Society (SJS) v. Atienza, Jr.:28
meter setback requirement is, as held by the lower courts, invalid. 20 Nonetheless, the petitioners
argue that such invalidity was subsequently cured by Zoning Ordinance No. 303, series of 2000.
As with the State, local governments may be considered as having properly exercised their
They also contend that Section 3, relating to the 80% see-thru fence requirement, must be
police power only if the following requisites are met: (1) the interests of the public generally, as
complied with, as it remains to be valid.
distinguished from those of a particular class, require its exercise and (2) the means employed
are reasonably necessary for the accomplishment of the purpose and not unduly oppressive
Ruling of the Court upon individuals. In short, there must be a concurrence of a lawful subject and lawful method. 29

The ultimate question before the Court is whether Sections 3.1 and 5 of Ordinance No. 192 are Lacking a concurrence of these two requisites, the police power measure shall be struck down
valid exercises of police power by the City Government of Marikina. as an arbitrary intrusion into private rights and a violation of the due process clause. 30

"Police power is the plenary power vested in the legislature to make statutes and ordinances to Section 3.1 and 5 of the assailed ordinance are pertinent to the issue at hand, to wit:
promote the health, morals, peace, education, good order or safety and general welfare of the
people."21 The State, through the legislature, has delegated the exercise of police power to local
Section 3. The standard height of fences of walls allowed under this ordinance are as follows:
government units, as agencies of the State. This delegation of police power is embodied in
Section 1622 of the Local Government Code of 1991 (R.A. No. 7160), known as the General
Welfare Clause,23 which has two branches. "The first, known as the general legislative power, (1) Fences on the front yard – shall be no more than one (1) meter in height. Fences in excess
authorizes the municipal council to enact ordinances and make regulations not repugnant to law, of one (1) meter shall be an open fence type, at least eighty percent (80%) see-thru;
as may be necessary to carry into effect and discharge the powers and duties conferred upon
the municipal council by law. The second, known as the police power proper, authorizes the
xxx xxx xxx
municipality to enact ordinances as may be necessary and proper for the health and safety,
prosperity, morals, peace, good order, comfort, and convenience of the municipality and its
inhabitants, and for the protection of their property."24 Section 5. In no case shall walls and fences be built within the five (5) meter parking area
allowance located between the front monument line and the building line of commercial and
White Light Corporation v. City of Manila,25 discusses the test of a valid ordinance: industrial establishments and educational and religious institutions.

The test of a valid ordinance is well established. A long line of decisions including City of Manila The respondents, thus, sought to prohibit the petitioners from requiring them to (1) demolish
has held that for an ordinance to be valid, it must not only be within the corporate powers of the their existing concrete wall, (2) build a fence (in excess of one meter) which must be 80% see-
local government unit to enact and pass according to the procedure prescribed by law, it must thru, and (3) build the said fence six meters back in order to provide a parking area.
also conform to the following substantive requirements: (1) must not contravene the
Setback Requirement
Constitution or any statute; (2) must not be unfair or oppressive; (3) must not be partial or
discriminatory; (4) must not prohibit but may regulate trade; (5) must be general and consistent The Court first turns its attention to Section 5 which requires the five-meter setback of the fence
with public policy; and (6) must not be unreasonable.26 to provide for a parking area. The petitioners initially argued that the ownership of the parking
area to be created would remain with the respondents as it would primarily be for the use of its
Ordinance No. 192 was passed by the City Council of Marikina in the apparent exercise of its students and faculty, and that its use by the public on non-school days would only be incidental.
police power. To successfully invoke the exercise of police power as the rationale for the In their Reply, however, the petitioners admitted that Section 5 was, in fact, invalid for being
repugnant to the Constitution.31
enactment of an ordinance and to free it from the imputation of constitutional infirmity, two tests
have been used by the Court – the rational relationship test and the strict scrutiny test:
The Court agrees with the latter position.
We ourselves have often applied the rational basis test mainly in analysis of equal protection
challenges. Using the rational basis examination, laws or ordinances are upheld if they rationally The Court joins the CA in finding that the real intent of the setback requirement was to make the
further a legitimate governmental interest. Under intermediate review, governmental interest is parking space free for use by the public, considering that it would no longer be for the exclusive
extensively examined and the availability of less restrictive measures is considered. Applying use of the respondents as it would also be available for use by the general public. Section 9 of
Article III of the 1987 Constitution, a provision on eminent domain, provides that private property 80% See-Thru Fence Requirement
shall not be taken for public use without just compensation.
The petitioners argue that while Section 5 of Ordinance No. 192 may be invalid, Section 3.1
The petitioners cannot justify the setback by arguing that the ownership of the property will limiting the height of fences to one meter and requiring fences in excess of one meter to be at
continue to remain with the respondents. It is a settled rule that neither the acquisition of title nor least 80% see-thru, should remain valid and enforceable against the respondents.
the total destruction of value is essential to taking. In fact, it is usually in cases where the title
remains with the private owner that inquiry should be made to determine whether the impairment
The Court cannot accommodate the petitioner.
of a property is merely regulated or amounts to a compensable taking. 32 The Court is of the view
that the implementation of the setback requirement would be tantamount to a taking of a total of
3,762.36 square meters of the respondents’ private property for public use without just For Section 3.1 to pass the rational relationship test, the petitioners must show the reasonable
compensation, in contravention to the Constitution. relation between the purpose of the police power measure and the means employed for its
accomplishment, for even under the guise of protecting the public interest, personal rights and
those pertaining to private property will not be permitted to be arbitrarily invaded. 36
Anent the objectives of prevention of concealment of unlawful acts and "un-neighborliness," it is
obvious that providing for a parking area has no logical connection to, and is not reasonably
necessary for, the accomplishment of these goals. The principal purpose of Section 3.1 is "to discourage, suppress or prevent the concealment of
prohibited or unlawful acts." The ultimate goal of this objective is clearly the prevention of crime
to ensure public safety and security. The means employed by the petitioners, however, is not
Regarding the beautification purpose of the setback requirement, it has long been settled that
reasonably necessary for the accomplishment of this purpose and is unduly oppressive to
the State may not, under the guise of police power, permanently divest owners of the beneficial
private rights. The petitioners have not adequately shown, and it does not appear obvious to this
use of their property solely to preserve or enhance the aesthetic appearance of the
Court, that an 80% see-thru fence would provide better protection and a higher level of security,
community.33 The Court, thus, finds Section 5 to be unreasonable and oppressive as it will
or serve as a more satisfactory criminal deterrent, than a tall solid concrete wall. It may even be
substantially divest the respondents of the beneficial use of their property solely for aesthetic
argued that such exposed premises could entice and tempt would-be criminals to the property,
purposes. Accordingly, Section 5 of Ordinance No. 192 is invalid.
and that a see-thru fence would be easier to bypass and breach. It also appears that the
respondents’ concrete wall has served as more than sufficient protection over the last 40 years. `
The petitioners, however, argue that the invalidity of Section 5 was properly cured by Zoning
Ordinance No. 303,34Series of 2000, which classified the respondents’ property to be within an
As to the beautification purpose of the assailed ordinance, as previously discussed, the State
institutional zone, under which a five-meter setback has been required.
may not, under the guise of police power, infringe on private rights solely for the sake of the
aesthetic appearance of the community. Similarly, the Court cannot perceive how a see-thru
The petitioners are mistaken. Ordinance No. 303, Series of 2000, has no bearing to the case at fence will foster "neighborliness" between members of a community.
hand.
Compelling the respondents to construct their fence in accordance with the assailed ordinance
The Court notes with displeasure that this argument was only raised for the first time on appeal is, thus, a clear encroachment on their right to property, which necessarily includes their right to
in this Court in the petitioners’ Reply. Considering that Ordinance No. 303 was enacted on decide how best to protect their property.
December 20, 2000, the petitioners could very well have raised it in their defense before the
RTC in 2002. The settled rule in this jurisdiction is that a party cannot change the legal theory of
It also appears that requiring the exposure of their property via a see-thru fence is violative of
this case under which the controversy was heard and decided in the trial court. It should be the
their right to privacy, considering that the residence of the Benedictine nuns is also located
same theory under which the review on appeal is conducted. Points of law, theories, issues, and
within the property. The right to privacy has long been considered a fundamental right
arguments not adequately brought to the attention of the lower court will not be ordinarily
guaranteed by the Constitution that must be protected from intrusion or constraint. The right to
considered by a reviewing court, inasmuch as they cannot be raised for the first time on appeal.
privacy is essentially the right to be let alone,37 as governmental powers should stop short of
This will be offensive to the basic rules of fair play, justice, and due process. 35
certain intrusions into the personal life of its citizens.38 It is inherent in the concept of liberty,
enshrined in the Bill of Rights (Article III) in Sections 1, 2, 3(1), 6, 8, and 17, Article III of the
Furthermore, the two ordinances have completely different purposes and subjects. Ordinance 1987 Constitution.39
No. 192 aims to regulate the construction of fences, while Ordinance No. 303 is a zoning
ordinance which classifies the city into specific land uses. In fact, the five-meter setback required
The enforcement of Section 3.1 would, therefore, result in an undue interference with the
by Ordinance No. 303 does not even appear to be for the purpose of providing a parking area.
respondents’ rights to property and privacy. Section 3.1 of Ordinance No. 192 is, thus, also
invalid and cannot be enforced against the respondents.
By no stretch of the imagination, therefore, can Ordinance No. 303, "cure" Section 5 of
Ordinance No. 192.
No Retroactivity

In any case, the clear subject of the petition for prohibition filed by the respondents is Ordinance
Ordinance No. 217 amended Section 7 of Ordinance No. 192 by including the regulation of
No. 192 and, as such, the precise issue to be determined is whether the petitioners can be
educational institutions which was unintentionally omitted, and giving said educational
prohibited from enforcing the said ordinance, and no other, against the respondents.
institutions five (5) years from the passage of Ordinance No. 192 (and not Ordinance No. 217) to
conform to its provisions.40 The petitioners argued that the amendment could be retroactively WHEREFORE, the petition is GRANTED. The writ of prohibition is hereby issued commanding
applied because the assailed ordinance is a curative statute which is retroactive in nature. the respondents to permanently desist from enforcing or implementing Sections 3.1 and 5 of
Ordinance No. 192, Series of 1994, as amended, on the petitioners' property in question located
in Marikina Heights, Marikina, Metro Manila.
Considering that Sections 3.1 and 5 of Ordinance No. 192 cannot be enforced against the
respondents, it is no longer necessary to rule on the issue of retroactivity. The Court shall,
nevertheless, pass upon the issue for the sake of clarity. No pronouncement as to costs.

"Curative statutes are enacted to cure defects in a prior law or to validate legal proceedings SO ORDERED.
which would otherwise be void for want of conformity with certain legal requirements. They are
intended to supply defects, abridge superfluities and curb certain evils. They are intended to
enable persons to carry into effect that which they have designed or intended, but has failed of
expected legal consequence by reason of some statutory disability or irregularity in their own
action. They make valid that which, before the enactment of the statute was invalid. Their
purpose is to give validity to acts done that would have been invalid under existing laws, as if
existing laws have been complied with. Curative statutes, therefore, by their very essence, are
retroactive."41

The petitioners argue that Ordinance No. 192 is a curative statute as it aims to correct or cure a
defect in the National Building Code, namely, its failure to provide for adequate guidelines for the
construction of fences. They ultimately seek to remedy an insufficiency in the law. In aiming to
cure this insufficiency, the petitioners attempt to add lacking provisions to the National Building
Code. This is not what is contemplated by curative statutes, which intend to correct irregularities
or invalidity in the law. The petitioners fail to point out any irregular or invalid provision. As such,
the assailed ordinance cannot qualify as curative and retroactive in nature.

At any rate, there appears to be no insufficiency in the National Building Code with respect to
parking provisions in relation to the issue of the respondents. Paragraph 1.16.1, Rule XIX of the
Rules and Regulations of the said code requires an educational institution to provide one parking
slot for every ten classrooms. As found by the lower courts, the respondents provide a total of 76
parking slots for their 80 classrooms and, thus, had more than sufficiently complied with the law.

Ordinance No. 192, as amended, is, therefore, not a curative statute which may be applied
retroactively.

Separability

Sections 3.1 and 5 of Ordinance No. 192, as amended, are, thus, invalid and cannot be
enforced against the respondents. Nonetheless, "the general rule is that where part of a statute
is void as repugnant to the Constitution, while another part is valid, the valid portion, if
susceptible to being separated from the invalid, may stand and be enforced." 42 Thus, the other
sections of the assailed ordinance remain valid and enforceable.

Conclusion

Considering the invalidity of Sections 3.1 and 5, it is clear that the petitioners were acting in
excess of their jurisdiction in enforcing Ordinance No. 192 against the respondents. The CA was
correct in affirming the decision of the RTC in issuing the writ of prohibition. The petitioners must
permanently desist from enforcing Sections 3.1 and 5 of the assailed ordinance on the
respondents' property in Marikina City.

WHEREFORE, the petition is DENIED. The October 2, 2002 Decision of the Regional Trial
Court in SCA Case No. 2000-381-MK is AFFIRMED but MODIFIED to read as follows:
G.R. No. 208566 November 19, 2013 "Pork Barrel" is political parlance of American -English origin.3 Historically, its usage
may be traced to the degrading ritual of rolling out a barrel stuffed with pork to a
multitude of black slaves who would cast their famished bodies into the porcine feast
GRECO ANTONIOUS BEDA B. BELGICA JOSE M. VILLEGAS JR. JOSE L. GONZALEZ
to assuage their hunger with morsels coming from the generosity of their well-fed
REUBEN M. ABANTE and QUINTIN PAREDES SAN DIEGO, Petitioners,
master.4 This practice was later compared to the actions of American legislators in
vs.
trying to direct federal budgets in favor of their districts.5 While the advent of
HONORABLE EXECUTIVE SECRETARY PAQUITO N. OCHOA JR. SECRETARY OF
refrigeration has made the actual pork barrel obsolete, it persists in reference to
BUDGET AND MANAGEMENT FLORENCIO B. ABAD, NATIONAL TREASURER ROSALIA
political bills that "bring home the bacon" to a legislator‘s district and constituents.6 In a
V. DE LEON SENATE OF THE PHILIPPINES represented by FRANKLIN M. DRILON m his
more technical sense, "Pork Barrel" refers to an appropriation of government spending
capacity as SENATE PRESIDENT and HOUSE OF REPRESENTATIVES represented by
meant for localized projects and secured solely or primarily to bring money to a
FELICIANO S. BELMONTE, JR. in his capacity as SPEAKER OF THE
representative's district.7Some scholars on the subject further use it to refer to
HOUSE, Respondents.
legislative control of local appropriations.8

x-----------------------x
In the Philippines, "Pork Barrel" has been commonly referred to as lump-sum,
discretionary funds of Members of the Legislature,9 although, as will be later
G.R. No. 208493 discussed, its usage would evolve in reference to certain funds of the Executive.

SOCIAL JUSTICE SOCIETY (SJS) PRESIDENT SAMSON S. ALCANTARA, Petitioner, II. History of Congressional Pork Barrel in the Philippines.
vs.
HONORABLE FRANKLIN M. DRILON in his capacity as SENATE PRESIDENT and
A. Pre-Martial Law Era (1922-1972).
HONORABLE FELICIANO S. BELMONTE, JR., in his capacity as SPEAKER OF THE
HOUSE OF REPRESENTATIVES, Respondents.
Act 3044,10 or the Public Works Act of 1922, is considered11 as the earliest form of
"Congressional Pork Barrel" in the Philippines since the utilization of the funds
x-----------------------x
appropriated therein were subjected to post-enactment legislator approval.
Particularly, in the area of fund release, Section 312 provides that the sums
G.R. No. 209251 appropriated for certain public works projects 13 "shall be distributed x x x subject to the
approval of a joint committee elected by the Senate and the House of
Representatives. "The committee from each House may also authorize one of its
PEDRITO M. NEPOMUCENO, Former Mayor-Boac, Marinduque Former Provincial Board
members to approve the distribution made by the Secretary of Commerce and
Member -Province of Marinduque, Petitioner,
Communications."14 Also, in the area of fund realignment, the same section provides
vs. that the said secretary, "with the approval of said joint committee, or of the authorized
PRESIDENT BENIGNO SIMEON C. AQUINO III* and SECRETARY FLORENCIO BUTCH
members thereof, may, for the purposes of said distribution, transfer unexpended
ABAD, DEPARTMENT OF BUDGET AND MANAGEMENT, Respondents.
portions of any item of appropriation under this Act to any other item hereunder."

DECISION In 1950, it has been documented15 that post-enactment legislator participation


broadened from the areas of fund release and realignment to the area of project
PERLAS-BERNABE, J.: identification. During that year, the mechanics of the public works act was modified to
the extent that the discretion of choosing projects was transferred from the Secretary
of Commerce and Communications to legislators. "For the first time, the law carried a
"Experience is the oracle of truth."1 list of projects selected by Members of Congress, they ‘being the representatives of
the people, either on their own account or by consultation with local officials or civil
-James Madison leaders.‘"16 During this period, the pork barrel process commenced with local
government councils, civil groups, and individuals appealing to Congressmen or
Senators for projects. Petitions that were accommodated formed part of a legislator‘s
Before the Court are consolidated petitions2 taken under Rule 65 of the Rules of Court, all of allocation, and the amount each legislator would eventually get is determined in a
which assail the constitutionality of the Pork Barrel System. Due to the complexity of the subject caucus convened by the majority. The amount was then integrated into the
matter, the Court shall heretofore discuss the system‘s conceptual underpinnings before administration bill prepared by the Department of Public Works and Communications.
detailing the particulars of the constitutional challenge. Thereafter, the Senate and the House of Representatives added their own provisions
to the bill until it was signed into law by the President – the Public Works Act.17 In the
The Facts 1960‘s, however, pork barrel legislation reportedly ceased in view of the stalemate
between the House of Representatives and the Senate.18
I. Pork Barrel: General Concept.
B. Martial Law Era (1972-1986).
While the previous" Congressional Pork Barrel" was apparently discontinued in 1972 the Senate Committee on Finance and the House Committee on Appropriations on the
after Martial Law was declared, an era when "one man controlled the releases made from the funds.33
legislature,"19 the reprieve was only temporary. By 1982, the Batasang Pambansa had
already introduced a new item in the General Appropriations Act (GAA) called the"
Under the 199734 CDF Article, Members of Congress and the Vice-President, in
Support for Local Development Projects" (SLDP) under the article on "National Aid to
consultation with the implementing agency concerned, were directed to submit to the
Local Government Units". Based on reports,20 it was under the SLDP that the practice
DBM the list of 50% of projects to be funded from their respective CDF allocations
of giving lump-sum allocations to individual legislators began, with each assemblyman
which shall be duly endorsed by (a) the Senate President and the Chairman of the
receiving ₱500,000.00. Thereafter, assemblymen would communicate their project
Committee on Finance, in the case of the Senate, and (b) the Speaker of the House of
preferences to the Ministry of Budget and Management for approval. Then, the said
Representatives and the Chairman of the Committee on Appropriations, in the case of
ministry would release the allocation papers to the Ministry of Local Governments,
the House of Representatives; while the list for the remaining 50% was to be
which would, in turn, issue the checks to the city or municipal treasurers in the
submitted within six (6) months thereafter. The same article also stated that the project
assemblyman‘s locality. It has been further reported that "Congressional Pork Barrel"
list, which would be published by the DBM,35 "shall be the basis for the release of
projects under the SLDP also began to cover not only public works projects, or so-
funds" and that "no funds appropriated herein shall be disbursed for projects not
called "hard projects", but also "soft projects",21 or non-public works projects such as
included in the list herein required."
those which would fall under the categories of, among others, education, health and
livelihood.22
The following year, or in 1998,36 the foregoing provisions regarding the required lists
and endorsements were reproduced, except that the publication of the project list was
C. Post-Martial Law Era:
no longer required as the list itself sufficed for the release of CDF Funds.

Corazon Cojuangco Aquino Administration (1986-1992).


The CDF was not, however, the lone form of "Congressional Pork Barrel" at that time.
Other forms of "Congressional Pork Barrel" were reportedly fashioned and inserted
After the EDSA People Power Revolution in 1986 and the restoration of Philippine into the GAA (called "Congressional Insertions" or "CIs") in order to perpetuate the ad
democracy, "Congressional Pork Barrel" was revived in the form of the "Mindanao ministration‘s political agenda.37 It has been articulated that since CIs "formed part and
Development Fund" and the "Visayas Development Fund" which were created with parcel of the budgets of executive departments, they were not easily identifiable and
lump-sum appropriations of ₱480 Million and ₱240 Million, respectively, for the were thus harder to monitor." Nonetheless, the lawmakers themselves as well as the
funding of development projects in the Mindanao and Visayas areas in 1989. It has finance and budget officials of the implementing agencies, as well as the DBM,
been documented23 that the clamor raised by the Senators and the Luzon legislators purportedly knew about the insertions.38 Examples of these CIs are the Department of
for a similar funding, prompted the creation of the "Countrywide Development Fund" Education (DepEd) School Building Fund, the Congressional Initiative Allocations, the
(CDF) which was integrated into the 1990 GAA24 with an initial funding of ₱2.3 Billion Public Works Fund, the El Niño Fund, and the Poverty Alleviation Fund.39 The
to cover "small local infrastructure and other priority community projects." allocations for the School Building Fund, particularly, ―shall be made upon prior
consultation with the representative of the legislative district concerned.” 40 Similarly,
the legislators had the power to direct how, where and when these appropriations
Under the GAAs for the years 1991 and 1992,25 CDF funds were, with the approval of
were to be spent.41
the President, to be released directly to the implementing agencies but "subject to the
submission of the required list of projects and activities."Although the GAAs from 1990
to 1992 were silent as to the amounts of allocations of the individual legislators, as E. Joseph Ejercito Estrada (Estrada) Administration (1998-2001).
well as their participation in the identification of projects, it has been reported26 that by
1992, Representatives were receiving ₱12.5 Million each in CDF funds, while
In 1999,42 the CDF was removed in the GAA and replaced by three (3) separate forms
Senators were receiving ₱18 Million each, without any limitation or qualification, and
of CIs, namely, the "Food Security Program Fund,"43 the "Lingap Para Sa Mahihirap
that they could identify any kind of project, from hard or infrastructure projects such as
Program Fund,"44and the "Rural/Urban Development Infrastructure Program
roads, bridges, and buildings to "soft projects" such as textbooks, medicines, and
Fund,"45 all of which contained a special provision requiring "prior consultation" with
scholarships.27
the Member s of Congress for the release of the funds.

D. Fidel Valdez Ramos (Ramos) Administration (1992-1998).


It was in the year 200046 that the "Priority Development Assistance Fund" (PDAF)
appeared in the GAA. The requirement of "prior consultation with the respective
The following year, or in 1993,28 the GAA explicitly stated that the release of CDF Representative of the District" before PDAF funds were directly released to the
funds was to be made upon the submission of the list of projects and activities implementing agency concerned was explicitly stated in the 2000 PDAF Article.
identified by, among others, individual legislators. For the first time, the 1993 CDF Moreover, realignment of funds to any expense category was expressly allowed, with
Article included an allocation for the Vice-President.29 As such, Representatives were the sole condition that no amount shall be used to fund personal services and other
allocated ₱12.5 Million each in CDF funds, Senators, ₱18 Million each, and the Vice- personnel benefits.47 The succeeding PDAF provisions remained the same in view of
President, ₱20 Million. the re-enactment48 of the 2000 GAA for the year 2001.

In 1994,30 1995,31 and 1996,32 the GAAs contained the same provisions on project F. Gloria Macapagal-Arroyo (Arroyo) Administration (2001-2010).
identification and fund release as found in the 1993 CDF Article. In addition, however,
the Department of Budget and Management (DBM) was directed to submit reports to
The 200249 PDAF Article was brief and straightforward as it merely contained a single Social Welfare and Development, Interior and Local Government, Environment and
special provision ordering the release of the funds directly to the implementing agency Natural Resources, Energy, and Public Works and Highways to realign PDAF Funds,
or local government unit concerned, without further qualifications. The following year, with the further conditions that: (a) realignment is within the same implementing unit
2003,50 the same single provision was present, with simply an expansion of purpose and same project category as the original project, for infrastructure projects; (b)
and express authority to realign. Nevertheless, the provisions in the 2003 budgets of allotment released has not yet been obligated for the original scope of work, and (c)
the Department of Public Works and Highways51 (DPWH) and the DepEd52 required the request for realignment is with the concurrence of the legislator concerned. 71
prior consultation with Members of Congress on the aspects of implementation
delegation and project list submission, respectively. In 2004, the 2003 GAA was re-
In the 201272 and 201373 PDAF Articles, it is stated that the "identification of projects
enacted.53
and/or designation of beneficiaries shall conform to the priority list, standard or design
prepared by each implementing agency (priority list requirement) x x x." However, as
In 2005,54 the PDAF Article provided that the PDAF shall be used "to fund priority practiced, it would still be the individual legislator who would choose and identify the
programs and projects under the ten point agenda of the national government and project from the said priority list.74
shall be released directly to the implementing agencies." It also introduced the
program menu concept,55 which is essentially a list of general programs and
Provisions on legislator allocations75 as well as fund realignment76 were included in the
implementing agencies from which a particular PDAF project may be subsequently
2012 and 2013 PDAF Articles; but the allocation for the Vice-President, which was
chosen by the identifying authority. The 2005 GAA was re-enacted56 in 2006 and
pegged at ₱200 Million in the 2011 GAA, had been deleted. In addition, the 2013
hence, operated on the same bases. In similar regard, the program menu concept was
PDAF Article now allowed LGUs to be identified as implementing agencies if they
consistently integrated into the 2007,57 2008,58 2009,59 and 201060 GAAs.
have the technical capability to implement the projects.77 Legislators were also
allowed to identify programs/projects, except for assistance to indigent patients and
Textually, the PDAF Articles from 2002 to 2010 were silent with respect to the specific scholarships, outside of his legislative district provided that he secures the written
amounts allocated for the individual legislators, as well as their participation in the concurrence of the legislator of the intended outside-district, endorsed by the Speaker
proposal and identification of PDAF projects to be funded. In contrast to the PDAF of the House.78 Finally, any realignment of PDAF funds, modification and revision of
Articles, however, the provisions under the DepEd School Building Program and the project identification, as well as requests for release of funds, were all required to be
DPWH budget, similar to its predecessors, explicitly required prior consultation with favorably endorsed by the House Committee on Appropriations and the Senate
the concerned Member of Congress61anent certain aspects of project implementation. Committee on Finance, as the case may be.79

Significantly, it was during this era that provisions which allowed formal participation of III. History of Presidential Pork Barrel in the Philippines.
non-governmental organizations (NGO) in the implementation of government projects
were introduced. In the Supplemental Budget for 2006, with respect to the
While the term "Pork Barrel" has been typically associated with lump-sum,
appropriation for school buildings, NGOs were, by law, encouraged to participate. For
discretionary funds of Members of Congress, the present cases and the recent
such purpose, the law stated that "the amount of at least ₱250 Million of the ₱500
controversies on the matter have, however, shown that the term‘s usage has
Million allotted for the construction and completion of school buildings shall be made
expanded to include certain funds of the President such as the Malampaya Funds and
available to NGOs including the Federation of Filipino-Chinese Chambers of
the Presidential Social Fund.
Commerce and Industry, Inc. for its "Operation Barrio School" program, with capability
and proven track records in the construction of public school buildings x x x." 62 The
same allocation was made available to NGOs in the 2007 and 2009 GAAs under the On the one hand, the Malampaya Funds was created as a special fund under Section
DepEd Budget.63 Also, it was in 2007 that the Government Procurement Policy 880 of Presidential Decree No. (PD) 910,81 issued by then President Ferdinand E.
Board64 (GPPB) issued Resolution No. 12-2007 dated June 29, 2007 (GPPB Marcos (Marcos) on March 22, 1976. In enacting the said law, Marcos recognized the
Resolution 12-2007), amending the implementing rules and regulations65 of RA need to set up a special fund to help intensify, strengthen, and consolidate
9184,66 the Government Procurement Reform Act, to include, as a form of negotiated government efforts relating to the exploration, exploitation, and development of
procurement,67 the procedure whereby the Procuring Entity68(the implementing indigenous energy resources vital to economic growth.82 Due to the energy-related
agency) may enter into a memorandum of agreement with an NGO, provided that "an activities of the government in the Malampaya natural gas field in Palawan, or the
appropriation law or ordinance earmarks an amount to be specifically contracted out to "Malampaya Deep Water Gas-to-Power Project",83 the special fund created under PD
NGOs."69 910 has been currently labeled as Malampaya Funds.

G. Present Administration (2010-Present). On the other hand the Presidential Social Fund was created under Section 12, Title
IV84 of PD 1869,85 or the Charter of the Philippine Amusement and Gaming
Corporation (PAGCOR). PD 1869 was similarly issued by Marcos on July 11, 1983.
Differing from previous PDAF Articles but similar to the CDF Articles, the 201170 PDAF
More than two (2) years after, he amended PD 1869 and accordingly issued PD 1993
Article included an express statement on lump-sum amounts allocated for individual
on October 31, 1985,86 amending Section 1287 of the former law. As it stands, the
legislators and the Vice-President: Representatives were given ₱70 Million each,
Presidential Social Fund has been described as a special funding facility managed
broken down into ₱40 Million for "hard projects" and ₱30 Million for "soft projects";
and administered by the Presidential Management Staff through which the President
while ₱200 Million was given to each Senator as well as the Vice-President, with a
provides direct assistance to priority programs and projects not funded under the
₱100 Million allocation each for "hard" and "soft projects." Likewise, a provision on
regular budget. It is sourced from the share of the government in the aggregate gross
realignment of funds was included, but with the qualification that it may be allowed
earnings of PAGCOR.88
only once. The same provision also allowed the Secretaries of Education, Health,
IV. Controversies in the Philippines. Infrastructures including Local Projects (VILP)," were made public, the highlights of
which are as follows:103
Over the decades, "pork" funds in the Philippines have increased
tremendously,89 owing in no small part to previous Presidents who reportedly used the ● Amounts released for projects identified by a considerable number of
"Pork Barrel" in order to gain congressional support.90 It was in 1996 when the first legislators significantly exceeded their respective allocations.
controversy surrounding the "Pork Barrel" erupted. Former Marikina City
Representative Romeo Candazo (Candazo), then an anonymous source, "blew the lid
● Amounts were released for projects outside of legislative districts of
on the huge sums of government money that regularly went into the pockets of
sponsoring members of the Lower House.
legislators in the form of kickbacks."91 He said that "the kickbacks were ‘SOP‘
(standard operating procedure) among legislators and ranged from a low 19 percent to
a high 52 percent of the cost of each project, which could be anything from dredging, ● Total VILP releases for the period exceeded the total amount appropriated
rip rapping, sphalting, concreting, and construction of school buildings." 92 "Other under the 2007 to 2009 GAAs.
sources of kickbacks that Candazo identified were public funds intended for medicines
and textbooks. A few days later, the tale of the money trail became the banner story of
the Philippine Daily Inquirer issue of August 13, 1996, accompanied by an illustration ● Infrastructure projects were constructed on private lots without these
of a roasted pig."93 "The publication of the stories, including those about congressional having been turned over to the government.
initiative allocations of certain lawmakers, including ₱3.6 Billion for a Congressman,
sparked public outrage."94 ● Significant amounts were released to implementing agencies without the
latter‘s endorsement and without considering their mandated functions,
administrative and technical capabilities to implement projects.
Thereafter, or in 2004, several concerned citizens sought the nullification of the PDAF
as enacted in the 2004 GAA for being unconstitutional. Unfortunately, for lack of "any
pertinent evidentiary support that illegal misuse of PDAF in the form of kickbacks has ● Implementation of most livelihood projects was not undertaken by the
become a common exercise of unscrupulous Members of Congress," the petition was implementing agencies themselves but by NGOs endorsed by the
dismissed.95 proponent legislators to which the Funds were transferred.

Recently, or in July of the present year, the National Bureau of Investigation (NBI) ● The funds were transferred to the NGOs in spite of the absence of any
began its probe into allegations that "the government has been defrauded of some appropriation law or ordinance.
₱10 Billion over the past 10 years by a syndicate using funds from the pork barrel of
lawmakers and various government agencies for scores of ghost projects." 96 The
investigation was spawned by sworn affidavits of six (6) whistle-blowers who declared ● Selection of the NGOs were not compliant with law and regulations.
that JLN Corporation – "JLN" standing for Janet Lim Napoles (Napoles) – had
swindled billions of pesos from the public coffers for "ghost projects" using no fewer ● Eighty-Two (82) NGOs entrusted with implementation of seven hundred
than 20 dummy NGOs for an entire decade. While the NGOs were supposedly the seventy two (772) projects amount to ₱6.156 Billion were either found
ultimate recipients of PDAF funds, the whistle-blowers declared that the money was questionable, or submitted questionable/spurious documents, or failed to
diverted into Napoles‘ private accounts.97 Thus, after its investigation on the Napoles liquidate in whole or in part their utilization of the Funds.
controversy, criminal complaints were filed before the Office of the Ombudsman,
charging five (5) lawmakers for Plunder, and three (3) other lawmakers for
Malversation, Direct Bribery, and Violation of the Anti-Graft and Corrupt Practices Act. ● Procurement by the NGOs, as well as some implementing agencies, of
Also recommended to be charged in the complaints are some of the lawmakers‘ chiefs goods and services reportedly used in the projects were not compliant with
-of-staff or representatives, the heads and other officials of three (3) implementing law.
agencies, and the several presidents of the NGOs set up by Napoles.98
As for the "Presidential Pork Barrel", whistle-blowers alleged that" at least ₱900 Million
On August 16, 2013, the Commission on Audit (CoA) released the results of a three- from royalties in the operation of the Malampaya gas project off Palawan province
year audit investigation99covering the use of legislators' PDAF from 2007 to 2009, or intended for agrarian reform beneficiaries has gone into a dummy NGO." 104 According
during the last three (3) years of the Arroyo administration. The purpose of the audit to incumbent CoA Chairperson Maria Gracia Pulido Tan (CoA Chairperson), the CoA
was to determine the propriety of releases of funds under PDAF and the Various is, as of this writing, in the process of preparing "one consolidated report" on the
Infrastructures including Local Projects (VILP)100 by the DBM, the application of these Malampaya Funds.105
funds and the implementation of projects by the appropriate implementing agencies
and several government-owned-and-controlled corporations (GOCCs).101 The total V. The Procedural Antecedents.
releases covered by the audit amounted to ₱8.374 Billion in PDAF and ₱32.664 Billion
in VILP, representing 58% and 32%, respectively, of the total PDAF and VILP
releases that were found to have been made nationwide during the audit Spurred in large part by the findings contained in the CoA Report and the Napoles
period.102 Accordingly, the Co A‘s findings contained in its Report No. 2012-03 (CoA controversy, several petitions were lodged before the Court similarly seeking that the
Report), entitled "Priority Development Assistance Fund (PDAF) and Various "Pork Barrel System" be declared unconstitutional. To recount, the relevant procedural
antecedents in these cases are as follows:
On August 28, 2013, petitioner Samson S. Alcantara (Alcantara), President of the Social Justice On September 23, 2013, the Office of the Solicitor General (OSG) filed a Consolidated
Society, filed a Petition for Prohibition of even date under Rule 65 of the Rules of Court Comment (Comment) of even date before the Court, seeking the lifting, or in the alternative, the
(Alcantara Petition), seeking that the "Pork Barrel System" be declared unconstitutional, and a partial lifting with respect to educational and medical assistance purposes, of the Court‘s
writ of prohibition be issued permanently restraining respondents Franklin M. Drilon and September 10, 2013 TRO, and that the consolidated petitions be dismissed for lack of merit. 113
Feliciano S. Belmonte, Jr., in their respective capacities as the incumbent Senate President and
Speaker of the House of Representatives, from further taking any steps to enact legislation
On September 24, 2013, the Court issued a Resolution of even date directing petitioners to reply
appropriating funds for the "Pork Barrel System," in whatever form and by whatever name it may
to the Comment.
be called, and from approving further releases pursuant thereto. 106 The Alcantara Petition was
docketed as G.R. No. 208493.
Petitioners, with the exception of Nepomuceno, filed their respective replies to the Comment: (a)
on September 30, 2013, Villegas filed a separate Reply dated September 27, 2013 (Villegas
On September 3, 2013, petitioners Greco Antonious Beda B. Belgica, Jose L. Gonzalez, Reuben
Reply); (b) on October 1, 2013, Belgica, et al. filed a Reply dated September 30, 2013 (Belgica
M. Abante, Quintin Paredes San Diego (Belgica, et al.), and Jose M. Villegas, Jr. (Villegas) filed
Reply); and (c) on October 2, 2013, Alcantara filed a Reply dated October 1, 2013.
an Urgent Petition For Certiorari and Prohibition With Prayer For The Immediate Issuance of
Temporary Restraining Order (TRO) and/or Writ of Preliminary Injunction dated August 27, 2013
under Rule 65 of the Rules of Court (Belgica Petition), seeking that the annual "Pork Barrel On October 1, 2013, the Court issued an Advisory providing for the guidelines to be observed by
System," presently embodied in the provisions of the GAA of 2013 which provided for the 2013 the parties for the Oral Arguments scheduled on October 8, 2013. In view of the technicality of
PDAF, and the Executive‘s lump-sum, discretionary funds, such as the Malampaya Funds and the issues material to the present cases, incumbent Solicitor General Francis H. Jardeleza
the Presidential Social Fund,107 be declared unconstitutional and null and void for being acts (Solicitor General) was directed to bring with him during the Oral Arguments representative/s
constituting grave abuse of discretion. Also, they pray that the Court issue a TRO against from the DBM and Congress who would be able to competently and completely answer
respondents Paquito N. Ochoa, Jr., Florencio B. Abad (Secretary Abad) and Rosalia V. De questions related to, among others, the budgeting process and its implementation. Further, the
Leon, in their respective capacities as the incumbent Executive Secretary, Secretary of the CoA Chairperson was appointed as amicus curiae and thereby requested to appear before the
Department of Budget and Management (DBM), and National Treasurer, or their agents, for Court during the Oral Arguments.
them to immediately cease any expenditure under the aforesaid funds. Further, they pray that
the Court order the foregoing respondents to release to the CoA and to the public: (a) "the
complete schedule/list of legislators who have availed of their PDAF and VILP from the years On October 8 and 10, 2013, the Oral Arguments were conducted. Thereafter, the Court directed
the parties to submit their respective memoranda within a period of seven (7) days, or until
2003 to 2013, specifying the use of the funds, the project or activity and the recipient entities or
individuals, and all pertinent data thereto"; and (b) "the use of the Executive‘s lump-sum, October 17, 2013, which the parties subsequently did.
discretionary funds, including the proceeds from the x x x Malampaya Funds and remittances
from the PAGCOR x x x from 2003 to 2013, specifying the x x x project or activity and the The Issues Before the Court
recipient entities or individuals, and all pertinent data thereto."108 Also, they pray for the
"inclusion in budgetary deliberations with the Congress of all presently off-budget, lump-sum,
discretionary funds including, but not limited to, proceeds from the Malampaya Funds and Based on the pleadings, and as refined during the Oral Arguments, the following are the main
remittances from the PAGCOR."109 The Belgica Petition was docketed as G.R. No. 208566.110 issues for the Court‘s resolution:

Lastly, on September 5, 2013, petitioner Pedrito M. Nepomuceno (Nepomuceno), filed a Petition I. Procedural Issues.
dated August 23, 2012 (Nepomuceno Petition), seeking that the PDAF be declared
unconstitutional, and a cease and desist order be issued restraining President Benigno Simeon Whether or not (a) the issues raised in the consolidated petitions involve an actual and
S. Aquino III (President Aquino) and Secretary Abad from releasing such funds to Members of justiciable controversy; (b) the issues raised in the consolidated petitions are matters of policy
Congress and, instead, allow their release to fund priority projects identified and approved by the not subject to judicial review; (c) petitioners have legal standing to sue; and (d) the Court‘s
Local Development Councils in consultation with the executive departments, such as the DPWH, Decision dated August 19, 1994 in G.R. Nos. 113105, 113174, 113766, and 113888, entitled
the Department of Tourism, the Department of Health, the Department of Transportation, and "Philippine Constitution Association v. Enriquez"114 (Philconsa) and Decision dated April 24,
Communication and the National Economic Development Authority.111 The Nepomuceno Petition 2012 in G.R. No. 164987, entitled "Lawyers Against Monopoly and Poverty v. Secretary of
was docketed as UDK-14951.112 Budget and Management"115 (LAMP) bar the re-litigatio n of the issue of constitutionality of the
"Pork Barrel System" under the principles of res judicata and stare decisis.
On September 10, 2013, the Court issued a Resolution of even date (a) consolidating all cases;
(b) requiring public respondents to comment on the consolidated petitions; (c) issuing a TRO II. Substantive Issues on the "Congressional Pork Barrel."
(September 10, 2013 TRO) enjoining the DBM, National Treasurer, the Executive Secretary, or
any of the persons acting under their authority from releasing (1) the remaining PDAF allocated
to Members of Congress under the GAA of 2013, and (2) Malampaya Funds under the phrase Whether or not the 2013 PDAF Article and all other Congressional Pork Barrel Laws similar
"for such other purposes as may be hereafter directed by the President" pursuant to Section 8 of thereto are unconstitutional considering that they violate the principles of/constitutional
PD 910 but not for the purpose of "financing energy resource development and exploitation provisions on (a) separation of powers; (b) non-delegability of legislative power; (c) checks and
programs and projects of the government‖ under the same provision; and (d) setting the balances; (d) accountability; (e) political dynasties; and (f) local autonomy.
consolidated cases for Oral Arguments on October 8, 2013.
III. Substantive Issues on the "Presidential Pork Barrel."
Whether or not the phrases (a) "and for such other purposes as may be hereafter directed by the Funds and PD 1869, as amended by PD 1993, for the Presidential Social Fund – are currently
President" under Section 8 of PD 910,116 relating to the Malampaya Funds, and (b) "to finance existing and operational; hence, there exists an immediate or threatened injury to petitioners as
the priority infrastructure development projects and to finance the restoration of damaged or a result of the unconstitutional use of these public funds.
destroyed facilities due to calamities, as may be directed and authorized by the Office of the
President of the Philippines" under Section 12 of PD 1869, as amended by PD 1993, relating to
As for the PDAF, the Court must dispel the notion that the issues related thereto had been
the Presidential Social Fund, are unconstitutional insofar as they constitute undue delegations of
rendered moot and academic by the reforms undertaken by respondents. A case becomes moot
legislative power.
when there is no more actual controversy between the parties or no useful purpose can be
served in passing upon the merits.125 Differing from this description, the Court observes that
These main issues shall be resolved in the order that they have been stated. In addition, the respondents‘ proposed line-item budgeting scheme would not terminate the controversy nor
Court shall also tackle certain ancillary issues as prompted by the present cases. diminish the useful purpose for its resolution since said reform is geared towards the 2014
budget, and not the 2013 PDAF Article which, being a distinct subject matter, remains legally
effective and existing. Neither will the President‘s declaration that he had already "abolished the
The Court’s Ruling
PDAF" render the issues on PDAF moot precisely because the Executive branch of government
has no constitutional authority to nullify or annul its legal existence. By constitutional design, the
The petitions are partly granted. annulment or nullification of a law may be done either by Congress, through the passage of a
repealing law, or by the Court, through a declaration of unconstitutionality. Instructive on this
point is the following exchange between Associate Justice Antonio T. Carpio (Justice Carpio)
I. Procedural Issues. and the Solicitor General during the Oral Arguments:126

The prevailing rule in constitutional litigation is that no question involving the constitutionality or
Justice Carpio: The President has taken an oath to faithfully execute the law, 127 correct? Solicitor
validity of a law or governmental act may be heard and decided by the Court unless there is General Jardeleza: Yes, Your Honor.
compliance with the legal requisites for judicial inquiry,117 namely: (a) there must be an actual
case or controversy calling for the exercise of judicial power; (b) the person challenging the act
must have the standing to question the validity of the subject act or issuance; (c) the question of Justice Carpio: And so the President cannot refuse to implement the General Appropriations Act,
constitutionality must be raised at the earliest opportunity ; and (d) the issue of constitutionality correct?
must be the very lis mota of the case.118 Of these requisites, case law states that the first two are
the most important119and, therefore, shall be discussed forthwith.
Solicitor General Jardeleza: Well, that is our answer, Your Honor. In the case, for example of the
PDAF, the President has a duty to execute the laws but in the face of the outrage over PDAF,
A. Existence of an Actual Case or Controversy. the President was saying, "I am not sure that I will continue the release of the soft projects," and
that started, Your Honor. Now, whether or not that … (interrupted)
By constitutional fiat, judicial power operates only when there is an actual case or
controversy.120 This is embodied in Section 1, Article VIII of the 1987 Constitution which Justice Carpio: Yeah. I will grant the President if there are anomalies in the project, he has the
pertinently states that "judicial power includes the duty of the courts of justice to settle actual power to stop the releases in the meantime, to investigate, and that is Section 38 of Chapter 5 of
controversies involving rights which are legally demandable and enforceable x x x." Book 6 of the Revised Administrative Code128 x x x. So at most the President can suspend, now
Jurisprudence provides that an actual case or controversy is one which "involves a conflict of if the President believes that the PDAF is unconstitutional, can he just refuse to implement it?
legal rights, an assertion of opposite legal claims, susceptible of judicial resolution as
distinguished from a hypothetical or abstract difference or dispute. 121 In other words, "there must
Solicitor General Jardeleza: No, Your Honor, as we were trying to say in the specific case of the
be a contrariety of legal rights that can be interpreted and enforced on the basis of existing law
PDAF because of the CoA Report, because of the reported irregularities and this Court can take
and jurisprudence."122 Related to the requirement of an actual case or controversy is the
judicial notice, even outside, outside of the COA Report, you have the report of the whistle-
requirement of "ripeness," meaning that the questions raised for constitutional scrutiny are
blowers, the President was just exercising precisely the duty ….
already ripe for adjudication. "A question is ripe for adjudication when the act being challenged
has had a direct adverse effect on the individual challenging it. It is a prerequisite that something
had then been accomplished or performed by either branch before a court may come into the xxxx
picture, and the petitioner must allege the existence of an immediate or threatened injury to itself
as a result of the challenged action."123 "Withal, courts will decline to pass upon constitutional
Justice Carpio: Yes, and that is correct. You‘ve seen the CoA Report, there are anomalies, you
issues through advisory opinions, bereft as they are of authority to resolve hypothetical or moot
stop and investigate, and prosecute, he has done that. But, does that mean that PDAF has been
questions."124
repealed?

Based on these principles, the Court finds that there exists an actual and justiciable controversy
Solicitor General Jardeleza: No, Your Honor x x x.
in these cases.

xxxx
The requirement of contrariety of legal rights is clearly satisfied by the antagonistic positions of
the parties on the constitutionality of the "Pork Barrel System." Also, the questions in these
consolidated cases are ripe for adjudication since the challenged funds and the provisions Justice Carpio: So that PDAF can be legally abolished only in two (2) cases. Congress passes a
allowing for their utilization – such as the 2013 GAA for the PDAF, PD 910 for the Malampaya law to repeal it, or this Court declares it unconstitutional, correct?
Solictor General Jardeleza: Yes, Your Honor. The Court also finds the third exception to be applicable largely due to the practical need for a
definitive ruling on the system‘s constitutionality. As disclosed during the Oral Arguments, the
CoA Chairperson estimates that thousands of notices of disallowances will be issued by her
Justice Carpio: The President has no power to legally abolish PDAF. (Emphases supplied)
office in connection with the findings made in the CoA Report. In this relation, Associate Justice
Marvic Mario Victor F. Leonen (Justice Leonen) pointed out that all of these would eventually
Even on the assumption of mootness, jurisprudence, nevertheless, dictates that "the moot and find their way to the courts.132 Accordingly, there is a compelling need to formulate controlling
academic‘ principle is not a magical formula that can automatically dissuade the Court in principles relative to the issues raised herein in order to guide the bench, the bar, and the public,
resolving a case." The Court will decide cases, otherwise moot, if: first, there is a grave violation not just for the expeditious resolution of the anticipated disallowance cases, but more
of the Constitution; second, the exceptional character of the situation and the paramount public importantly, so that the government may be guided on how public funds should be utilized in
interest is involved; third, when the constitutional issue raised requires formulation of controlling accordance with constitutional principles.
principles to guide the bench, the bar, and the public; and fourth, the case is capable of
repetition yet evading review.129
Finally, the application of the fourth exception is called for by the recognition that the preparation
and passage of the national budget is, by constitutional imprimatur, an affair of annual
The applicability of the first exception is clear from the fundamental posture of petitioners – they occurrence.133 The relevance of the issues before the Court does not cease with the passage of
essentially allege grave violations of the Constitution with respect to, inter alia, the principles of a "PDAF -free budget for 2014."134 The evolution of the "Pork Barrel System," by its multifarious
separation of powers, non-delegability of legislative power, checks and balances, accountability iterations throughout the course of history, lends a semblance of truth to petitioners‘ claim that
and local autonomy. "the same dog will just resurface wearing a different collar." 135 In Sanlakas v. Executive
Secretary,136 the government had already backtracked on a previous course of action yet the
Court used the "capable of repetition but evading review" exception in order "to prevent similar
The applicability of the second exception is also apparent from the nature of the interests
questions from re- emerging."137 The situation similarly holds true to these cases. Indeed, the
involved myriad of issues underlying the manner in which certain public funds are spent, if not resolved at
this most opportune time, are capable of repetition and hence, must not evade judicial review.
– the constitutionality of the very system within which significant amounts of public funds have
been and continue to be utilized and expended undoubtedly presents a situation of exceptional B. Matters of Policy: the Political Question Doctrine.
character as well as a matter of paramount public interest. The present petitions, in fact, have
been lodged at a time when the system‘s flaws have never before been magnified. To the
Court‘s mind, the coalescence of the CoA Report, the accounts of numerous whistle-blowers, The "limitation on the power of judicial review to actual cases and controversies‖ carries the
and the government‘s own recognition that reforms are needed "to address the reported abuses assurance that "the courts will not intrude into areas committed to the other branches of
of the PDAF"130 demonstrates a prima facie pattern of abuse which only underscores the government."138 Essentially, the foregoing limitation is a restatement of the political question
importance of the matter. It is also by this finding that the Court finds petitioners‘ claims as not doctrine which, under the classic formulation of Baker v. Carr, 139applies when there is found,
merely theorized, speculative or hypothetical. Of note is the weight accorded by the Court to the among others, "a textually demonstrable constitutional commitment of the issue to a coordinate
findings made by the CoA which is the constitutionally-mandated audit arm of the government. In political department," "a lack of judicially discoverable and manageable standards for resolving
Delos Santos v. CoA,131 a recent case wherein the Court upheld the CoA‘s disallowance of it" or "the impossibility of deciding without an initial policy determination of a kind clearly for non-
irregularly disbursed PDAF funds, it was emphasized that: judicial discretion." Cast against this light, respondents submit that the "the political branches are
in the best position not only to perform budget-related reforms but also to do them in response to
the specific demands of their constituents" and, as such, "urge the Court not to impose a
The COA is endowed with enough latitude to determine, prevent, and disallow irregular, solution at this stage."140
unnecessary, excessive, extravagant or unconscionable expenditures of government funds. It is
tasked to be vigilant and conscientious in safeguarding the proper use of the government's, and
ultimately the people's, property. The exercise of its general audit power is among the The Court must deny respondents‘ submission.
constitutional mechanisms that gives life to the check and balance system inherent in our form of
government.
Suffice it to state that the issues raised before the Court do not present political but legal
questions which are within its province to resolve. A political question refers to "those questions
It is the general policy of the Court to sustain the decisions of administrative authorities, which, under the Constitution, are to be decided by the people in their sovereign capacity, or in
especially one which is constitutionally-created, such as the CoA, not only on the basis of the regard to which full discretionary authority has been delegated to the Legislature or executive
doctrine of separation of powers but also for their presumed expertise in the laws they are branch of the Government. It is concerned with issues dependent upon the wisdom, not legality,
entrusted to enforce. Findings of administrative agencies are accorded not only respect but also of a particular measure."141 The intrinsic constitutionality of the "Pork Barrel System" is not an
finality when the decision and order are not tainted with unfairness or arbitrariness that would issue dependent upon the wisdom of the political branches of government but rather a legal one
amount to grave abuse of discretion. It is only when the CoA has acted without or in excess of which the Constitution itself has commanded the Court to act upon. Scrutinizing the contours of
jurisdiction, or with grave abuse of discretion amounting to lack or excess of jurisdiction, that this the system along constitutional lines is a task that the political branches of government are
Court entertains a petition questioning its rulings. x x x. (Emphases supplied) incapable of rendering precisely because it is an exercise of judicial power. More importantly, the
present Constitution has not only vested the Judiciary the right to exercise judicial power but
essentially makes it a duty to proceed therewith. Section 1, Article VIII of the 1987 Constitution
Thus, if only for the purpose of validating the existence of an actual and justiciable controversy in
cannot be any clearer: "The judicial power shall be vested in one Supreme Court and in such
these cases, the Court deems the findings under the CoA Report to be sufficient. lower courts as may be established by law. It includes the duty of the courts of justice to settle
actual controversies involving rights which are legally demandable and enforceable, and to
determine whether or not there has been a grave abuse of discretion amounting to lack or
excess of jurisdiction on the part of any branch or instrumentality of the Government." In Estrada the fundamental law by the enforcement of an invalid statute.150 All told, petitioners have
v. Desierto,142 the expanded concept of judicial power under the 1987 Constitution and its effect sufficient locus standi to file the instant cases.
on the political question doctrine was explained as follows:143
D. Res Judicata and Stare Decisis.
To a great degree, the 1987 Constitution has narrowed the reach of the political question
doctrine when it expanded the power of judicial review of this court not only to settle actual
Res judicata (which means a "matter adjudged") and stare decisis non quieta et movere (or
controversies involving rights which are legally demandable and enforceable but also to
simply, stare decisis which means "follow past precedents and do not disturb what has been
determine whether or not there has been a grave abuse of discretion amounting to lack or
settled") are general procedural law principles which both deal with the effects of previous but
excess of jurisdiction on the part of any branch or instrumentality of government. Heretofore, the
factually similar dispositions to subsequent cases. For the cases at bar, the Court examines the
judiciary has focused on the "thou shalt not's" of the Constitution directed against the exercise of
applicability of these principles in relation to its prior rulings in Philconsa and LAMP.
its jurisdiction. With the new provision, however, courts are given a greater prerogative to
determine what it can do to prevent grave abuse of discretion amounting to lack or excess of
jurisdiction on the part of any branch or instrumentality of government. Clearly, the new provision The focal point of res judicata is the judgment. The principle states that a judgment on the merits
did not just grant the Court power of doing nothing. x x x (Emphases supplied) in a previous case rendered by a court of competent jurisdiction would bind a subsequent case
if, between the first and second actions, there exists an identity of parties, of subject matter, and
of causes of action.151 This required identity is not, however, attendant hereto since Philconsa
It must also be borne in mind that ― when the judiciary mediates to allocate constitutional
and LAMP, respectively involved constitutional challenges against the 1994 CDF Article and
boundaries, it does not assert any superiority over the other departments; does not in reality
2004 PDAF Article, whereas the cases at bar call for a broader constitutional scrutiny of the
nullify or invalidate an act of the legislature or the executive, but only asserts the solemn and
entire "Pork Barrel System." Also, the ruling in LAMP is essentially a dismissal based on a
sacred obligation assigned to it by the Constitution."144 To a great extent, the Court is laudably
procedural technicality – and, thus, hardly a judgment on the merits – in that petitioners therein
cognizant of the reforms undertaken by its co-equal branches of government. But it is by
failed to present any "convincing proof x x x showing that, indeed, there were direct releases of
constitutional force that the Court must faithfully perform its duty. Ultimately, it is the Court‘s
funds to the Members of Congress, who actually spend them according to their sole discretion"
avowed intention that a resolution of these cases would not arrest or in any manner impede the
or "pertinent evidentiary support to demonstrate the illegal misuse of PDAF in the form of
endeavors of the two other branches but, in fact, help ensure that the pillars of change are
kickbacks and has become a common exercise of unscrupulous Members of Congress." As
erected on firm constitutional grounds. After all, it is in the best interest of the people that each
such, the Court up held, in view of the presumption of constitutionality accorded to every law, the
great branch of government, within its own sphere, contributes its share towards achieving a
2004 PDAF Article, and saw "no need to review or reverse the standing pronouncements in the
holistic and genuine solution to the problems of society. For all these reasons, the Court cannot
said case." Hence, for the foregoing reasons, the res judicata principle, insofar as the Philconsa
heed respondents‘ plea for judicial restraint.
and LAMP cases are concerned, cannot apply.

C. Locus Standi.
On the other hand, the focal point of stare decisis is the doctrine created. The principle,
entrenched under Article 8152 of the Civil Code, evokes the general rule that, for the sake of
"The gist of the question of standing is whether a party alleges such personal stake in the certainty, a conclusion reached in one case should be doctrinally applied to those that follow if
outcome of the controversy as to assure that concrete adverseness which sharpens the the facts are substantially the same, even though the parties may be different. It proceeds from
presentation of issues upon which the court depends for illumination of difficult constitutional the first principle of justice that, absent any powerful countervailing considerations, like cases
questions. Unless a person is injuriously affected in any of his constitutional rights by the ought to be decided alike. Thus, where the same questions relating to the same event have
operation of statute or ordinance, he has no standing."145 been put forward by the parties similarly situated as in a previous case litigated and decided by a
competent court, the rule of stare decisis is a bar to any attempt to re-litigate the same issue.153
Petitioners have come before the Court in their respective capacities as citizen-taxpayers and
accordingly, assert that they "dutifully contribute to the coffers of the National Philconsa was the first case where a constitutional challenge against a Pork Barrel provision,
Treasury."146 Clearly, as taxpayers, they possess the requisite standing to question the validity of i.e., the 1994 CDF Article, was resolved by the Court. To properly understand its context,
the existing "Pork Barrel System" under which the taxes they pay have been and continue to be petitioners‘ posturing was that "the power given to the Members of Congress to propose and
utilized. It is undeniable that petitioners, as taxpayers, are bound to suffer from the identify projects and activities to be funded by the CDF is an encroachment by the legislature on
unconstitutional usage of public funds, if the Court so rules. Invariably, taxpayers have been executive power, since said power in an appropriation act is in implementation of the law" and
allowed to sue where there is a claim that public funds are illegally disbursed or that public that "the proposal and identification of the projects do not involve the making of laws or the
money is being deflected to any improper purpose, or that public funds are wasted through the repeal and amendment thereof, the only function given to the Congress by the
enforcement of an invalid or unconstitutional law,147 as in these cases. Constitution."154 In deference to the foregoing submissions, the Court reached the following main
conclusions: one, under the Constitution, the power of appropriation, or the "power of the purse,"
belongs to Congress; two, the power of appropriation carries with it the power to specify the
Moreover, as citizens, petitioners have equally fulfilled the standing requirement given that the
project or activity to be funded under the appropriation law and it can be detailed and as broad
issues they have raised may be classified as matters "of transcendental importance, of
as Congress wants it to be; and, three, the proposals and identifications made by Members of
overreaching significance to society, or of paramount public interest."148 The CoA Chairperson‘s
Congress are merely recommendatory. At once, it is apparent that the Philconsa resolution was
statement during the Oral Arguments that the present controversy involves "not merely a
a limited response to a separation of powers problem, specifically on the propriety of conferring
systems failure" but a "complete breakdown of controls"149 amplifies, in addition to the matters
post-enactment identification authority to Members of Congress. On the contrary, the present
above-discussed, the seriousness of the issues involved herein. Indeed, of greater import than
cases call for a more holistic examination of (a) the inter-relation between the CDF and PDAF
the damage caused by the illegal expenditure of public funds is the mortal wound inflicted upon
Articles with each other, formative as they are of the entire "Pork Barrel System" as well as (b)
the intra-relation of post-enactment measures contained within a particular CDF or PDAF Article,
including not only those related to the area of project identification but also to the areas of fund discretionary public funds: first, the Congressional (or Legislative) Pork Barrel, currently known
release and realignment. The complexity of the issues and the broader legal analyses herein as the PDAF;158 and, second, the Presidential (or Executive) Pork Barrel, specifically, the
warranted may be, therefore, considered as a powerful countervailing reason against a Malampaya Funds under PD 910 and the Presidential Social Fund under PD 1869, as amended
wholesale application of the stare decisis principle. by PD 1993.159

In addition, the Court observes that the Philconsa ruling was actually riddled with inherent Considering petitioners‘ submission and in reference to its local concept and legal history, the
constitutional inconsistencies which similarly countervail against a full resort to stare decisis. As Court defines the Pork Barrel System as the collective body of rules and practices that govern
may be deduced from the main conclusions of the case, Philconsa‘s fundamental premise in the manner by which lump-sum, discretionary funds, primarily intended for local projects, are
allowing Members of Congress to propose and identify of projects would be that the said utilized through the respective participations of the Legislative and Executive branches of
identification authority is but an aspect of the power of appropriation which has been government, including its members. The Pork Barrel System involves two (2) kinds of lump-sum
constitutionally lodged in Congress. From this premise, the contradictions may be easily seen. If discretionary funds:
the authority to identify projects is an aspect of appropriation and the power of appropriation is a
form of legislative power thereby lodged in Congress, then it follows that: (a) it is Congress
First, there is the Congressional Pork Barrel which is herein defined as a kind of lump-sum,
which should exercise such authority, and not its individual Members; (b) such authority must be
discretionary fund wherein legislators, either individually or collectively organized into
exercised within the prescribed procedure of law passage and, hence, should not be exercised
committees, are able to effectively control certain aspects of the fund’s utilization through various
after the GAA has already been passed; and (c) such authority, as embodied in the GAA, has
post-enactment measures and/or practices. In particular, petitioners consider the PDAF, as it
the force of law and, hence, cannot be merely recommendatory. Justice Vitug‘s Concurring
appears under the 2013 GAA, as Congressional Pork Barrel since it is, inter alia, a post-
Opinion in the same case sums up the Philconsa quandary in this wise: "Neither would it be
enactment measure that allows individual legislators to wield a collective power;160 and
objectionable for Congress, by law, to appropriate funds for such specific projects as it may be
minded; to give that authority, however, to the individual members of Congress in whatever
guise, I am afraid, would be constitutionally impermissible." As the Court now largely benefits Second, there is the Presidential Pork Barrel which is herein defined as a kind of lump-sum,
from hindsight and current findings on the matter, among others, the CoA Report, the Court must discretionary fund which allows the President to determine the manner of its utilization. For
partially abandon its previous ruling in Philconsa insofar as it validated the post-enactment reasons earlier stated,161 the Court shall delimit the use of such term to refer only to the
identification authority of Members of Congress on the guise that the same was merely Malampaya Funds and the Presidential Social Fund.
recommendatory. This postulate raises serious constitutional inconsistencies which cannot be
simply excused on the ground that such mechanism is "imaginative as it is innovative."
Moreover, it must be pointed out that the recent case of Abakada Guro Party List v. With these definitions in mind, the Court shall now proceed to discuss the substantive issues of
Purisima155(Abakada) has effectively overturned Philconsa‘s allowance of post-enactment these cases.
legislator participation in view of the separation of powers principle. These constitutional
inconsistencies and the Abakada rule will be discussed in greater detail in the ensuing section of B. Substantive Issues on the Congressional Pork Barrel.
this Decision.
1. Separation of Powers.
As for LAMP, suffice it to restate that the said case was dismissed on a procedural technicality
and, hence, has not set any controlling doctrine susceptible of current application to the
substantive issues in these cases. In fine, stare decisis would not apply. a. Statement of Principle.

II. Substantive Issues. The principle of separation of powers refers to the constitutional demarcation of the three
fundamental powers of government. In the celebrated words of Justice Laurel in Angara v.
Electoral Commission,162 it means that the "Constitution has blocked out with deft strokes and in
A. Definition of Terms. bold lines, allotment of power to the executive, the legislative and the judicial departments of the
government."163 To the legislative branch of government, through Congress,164belongs the power
Before the Court proceeds to resolve the substantive issues of these cases, it must first define to make laws; to the executive branch of government, through the President, 165 belongs the
the terms "Pork Barrel System," "Congressional Pork Barrel," and "Presidential Pork Barrel" as power to enforce laws; and to the judicial branch of government, through the Court,166 belongs
the power to interpret laws. Because the three great powers have been, by constitutional design,
they are essential to the ensuing discourse.
ordained in this respect, "each department of the government has exclusive cognizance of
matters within its jurisdiction, and is supreme within its own sphere." 167 Thus, "the legislature has
Petitioners define the term "Pork Barrel System" as the "collusion between the Legislative and no authority to execute or construe the law, the executive has no authority to make or construe
Executive branches of government to accumulate lump-sum public funds in their offices with the law, and the judiciary has no power to make or execute the law."168 The principle of
unchecked discretionary powers to determine its distribution as political largesse." 156 They assert separation of powers and its concepts of autonomy and independence stem from the notion that
that the following elements make up the Pork Barrel System: (a) lump-sum funds are allocated the powers of government must be divided to avoid concentration of these powers in any one
through the appropriations process to an individual officer; (b) the officer is given sole and broad branch; the division, it is hoped, would avoid any single branch from lording its power over the
discretion in determining how the funds will be used or expended; (c) the guidelines on how to other branches or the citizenry.169 To achieve this purpose, the divided power must be wielded
spend or use the funds in the appropriation are either vague, overbroad or inexistent; and (d) by co-equal branches of government that are equally capable of independent action in
projects funded are intended to benefit a definite constituency in a particular part of the country exercising their respective mandates. Lack of independence would result in the inability of one
and to help the political careers of the disbursing official by yielding rich patronage branch of government to check the arbitrary or self-interest assertions of another or others.170
benefits.157 They further state that the Pork Barrel System is comprised of two (2) kinds of
Broadly speaking, there is a violation of the separation of powers principle when one branch of (2) investigation and monitoring of the implementation of laws pursuant to the power of
government unduly encroaches on the domain of another. US Supreme Court decisions instruct Congress to conduct inquiries in aid of legislation.
that the principle of separation of powers may be violated in two (2) ways: firstly, "one branch
may interfere impermissibly with the other’s performance of its constitutionally assigned
Any action or step beyond that will undermine the separation of powers guaranteed by the
function";171 and "alternatively, the doctrine may be violated when one branch assumes a
Constitution. (Emphases supplied)
function that more properly is entrusted to another."172 In other words, there is a violation of the
principle when there is impermissible (a) interference with and/or (b) assumption of another
department‘s functions. b. Application.

The enforcement of the national budget, as primarily contained in the GAA, is indisputably a In these cases, petitioners submit that the Congressional Pork Barrel – among others, the 2013
function both constitutionally assigned and properly entrusted to the Executive branch of PDAF Article – "wrecks the assignment of responsibilities between the political branches" as it is
government. In Guingona, Jr. v. Hon. Carague173 (Guingona, Jr.), the Court explained that the designed to allow individual legislators to interfere "way past the time it should have ceased" or,
phase of budget execution "covers the various operational aspects of budgeting" and particularly, "after the GAA is passed."179 They state that the findings and recommendations in
accordingly includes "the evaluation of work and financial plans for individual activities," the the CoA Report provide "an illustration of how absolute and definitive the power of legislators
"regulation and release of funds" as well as all "other related activities" that comprise the budget wield over project implementation in complete violation of the constitutional principle of
execution cycle.174 This is rooted in the principle that the allocation of power in the three principal separation of powers."180 Further, they point out that the Court in the Philconsa case only
branches of government is a grant of all powers inherent in them. 175 Thus, unless the allowed the CDF to exist on the condition that individual legislators limited their role to
Constitution provides otherwise, the Executive department should exclusively exercise all roles recommending projects and not if they actually dictate their implementation.181
and prerogatives which go into the implementation of the national budget as provided under the
GAA as well as any other appropriation law.
For their part, respondents counter that the separations of powers principle has not been
violated since the President maintains "ultimate authority to control the execution of the GAA‖
In view of the foregoing, the Legislative branch of government, much more any of its members, and that he "retains the final discretion to reject" the legislators‘ proposals.182 They maintain that
should not cross over the field of implementing the national budget since, as earlier stated, the the Court, in Philconsa, "upheld the constitutionality of the power of members of Congress to
same is properly the domain of the Executive. Again, in Guingona, Jr., the Court stated that propose and identify projects so long as such proposal and identification are
"Congress enters the picture when it deliberates or acts on the budget proposals of the recommendatory."183 As such, they claim that "everything in the Special Provisions [of the 2013
President. Thereafter, Congress, "in the exercise of its own judgment and wisdom, formulates an PDAF Article follows the Philconsa framework, and hence, remains constitutional." 184
appropriation act precisely following the process established by the Constitution, which specifies
that no money may be paid from the Treasury except in accordance with an appropriation made
by law." Upon approval and passage of the GAA, Congress‘ law -making role necessarily comes The Court rules in favor of petitioners.
to an end and from there the Executive‘s role of implementing the national budget begins. So as
not to blur the constitutional boundaries between them, Congress must "not concern it self with As may be observed from its legal history, the defining feature of all forms of Congressional Pork
details for implementation by the Executive."176 Barrel would be the authority of legislators to participate in the post-enactment phases of project
implementation.
The foregoing cardinal postulates were definitively enunciated in Abakada where the Court held
that "from the moment the law becomes effective, any provision of law that empowers Congress At its core, legislators – may it be through project lists,185 prior consultations186 or program
or any of its members to play any role in the implementation or enforcement of the law violates menus187 – have been consistently accorded post-enactment authority to identify the projects
the principle of separation of powers and is thus unconstitutional." 177 It must be clarified, they desire to be funded through various Congressional Pork Barrel allocations. Under the 2013
however, that since the restriction only pertains to "any role in the implementation or PDAF Article, the statutory authority of legislators to identify projects post-GAA may be
enforcement of the law," Congress may still exercise its oversight function which is a mechanism construed from the import of Special Provisions 1 to 3 as well as the second paragraph of
of checks and balances that the Constitution itself allows. But it must be made clear that Special Provision 4. To elucidate, Special Provision 1 embodies the program menu feature
Congress‘ role must be confined to mere oversight. Any post-enactment-measure allowing which, as evinced from past PDAF Articles, allows individual legislators to identify PDAF projects
legislator participation beyond oversight is bereft of any constitutional basis and hence, for as long as the identified project falls under a general program listed in the said menu.
tantamount to impermissible interference and/or assumption of executive functions. As the Court Relatedly, Special Provision 2 provides that the implementing agencies shall, within 90 days
ruled in Abakada:178 from the GAA is passed, submit to Congress a more detailed priority list, standard or design
prepared and submitted by implementing agencies from which the legislator may make his
choice. The same provision further authorizes legislators to identify PDAF projects outside his
Any post-enactment congressional measure x x x should be limited to scrutiny and
investigation.1âwphi1 In particular, congressional oversight must be confined to the following: district for as long as the representative of the district concerned concurs in writing. Meanwhile,
Special Provision 3 clarifies that PDAF projects refer to "projects to be identified by
legislators"188 and thereunder provides the allocation limit for the total amount of projects
(1) scrutiny based primarily on Congress‘ power of appropriation and the budget identified by each legislator. Finally, paragraph 2 of Special Provision 4 requires that any
hearings conducted in connection with it, its power to ask heads of departments to modification and revision of the project identification "shall be submitted to the House Committee
appear before and be heard by either of its Houses on any matter pertaining to their on Appropriations and the Senate Committee on Finance for favorable endorsement to the DBM
departments and its power of confirmation; and or the implementing agency, as the case may be." From the foregoing special provisions, it
cannot be seriously doubted that legislators have been accorded post-enactment authority to
identify PDAF projects.
Aside from the area of project identification, legislators have also been accorded post-enactment Solicitor General Jardeleza: Yes, Your Honor.
authority in the areas of fund release and realignment. Under the 2013 PDAF Article, the
statutory authority of legislators to participate in the area of fund release through congressional
xxxx
committees is contained in Special Provision 5 which explicitly states that "all request for release
of funds shall be supported by the documents prescribed under Special Provision No. 1 and
favorably endorsed by House Committee on Appropriations and the Senate Committee on Justice Bernabe: In short, the act of identification is mandatory?
Finance, as the case may be"; while their statutory authority to participate in the area of fund
realignment is contained in: first , paragraph 2, Special Provision 4189 which explicitly state s,
among others, that "any realignment of funds shall be submitted to the House Committee on Solictor General Jardeleza: Yes, Your Honor. In the sense that if it is not done and then there is
Appropriations and the Senate Committee on Finance for favorable endorsement to the DBM or no identification.
the implementing agency, as the case may be‖ ; and, second , paragraph 1, also of Special
Provision 4 which authorizes the "Secretaries of Agriculture, Education, Energy, Interior and xxxx
Local Government, Labor and Employment, Public Works and Highways, Social Welfare and
Development and Trade and Industry190 x x x to approve realignment from one project/scope to
another within the allotment received from this Fund, subject to among others (iii) the request is Justice Bernabe: Now, would you know of specific instances when a project was implemented
with the concurrence of the legislator concerned." without the identification by the individual legislator?

Clearly, these post-enactment measures which govern the areas of project identification, fund Solicitor General Jardeleza: I do not know, Your Honor; I do not think so but I have no specific
release and fund realignment are not related to functions of congressional oversight and, hence, examples. I would doubt very much, Your Honor, because to implement, there is a need for a
allow legislators to intervene and/or assume duties that properly belong to the sphere of budget SARO and the NCA. And the SARO and the NCA are triggered by an identification from the
execution. Indeed, by virtue of the foregoing, legislators have been, in one form or another, legislator.
authorized to participate in – as Guingona, Jr. puts it – "the various operational aspects of
budgeting," including "the evaluation of work and financial plans for individual activities" and the xxxx
"regulation and release of funds" in violation of the separation of powers principle. The
fundamental rule, as categorically articulated in Abakada, cannot be overstated – from the
moment the law becomes effective, any provision of law that empowers Congress or any of its Solictor General Jardeleza: What we mean by mandatory, Your Honor, is we were replying to a
members to play any role in the implementation or enforcement of the law violates the principle question, "How can a legislator make sure that he is able to get PDAF Funds?" It is mandatory in
of separation of powers and is thus unconstitutional. 191 That the said authority is treated as the sense that he must identify, in that sense, Your Honor. Otherwise, if he does not identify, he
merely recommendatory in nature does not alter its unconstitutional tenor since the prohibition, cannot avail of the PDAF Funds and his district would not be able to have PDAF Funds, only in
to repeat, covers any role in the implementation or enforcement of the law. Towards this end, the that sense, Your Honor. (Emphases supplied)
Court must therefore abandon its ruling in Philconsa which sanctioned the conduct of legislator
identification on the guise that the same is merely recommendatory and, as such, respondents‘ Thus, for all the foregoing reasons, the Court hereby declares the 2013 PDAF Article as well as
reliance on the same falters altogether. all other provisions of law which similarly allow legislators to wield any form of post-enactment
authority in the implementation or enforcement of the budget, unrelated to congressional
Besides, it must be pointed out that respondents have nonetheless failed to substantiate their oversight, as violative of the separation of powers principle and thus unconstitutional. Corollary
position that the identification authority of legislators is only of recommendatory import. Quite the thereto, informal practices, through which legislators have effectively intruded into the proper
contrary, respondents – through the statements of the Solicitor General during the Oral phases of budget execution, must be deemed as acts of grave abuse of discretion amounting to
Arguments – have admitted that the identification of the legislator constitutes a mandatory lack or excess of jurisdiction and, hence, accorded the same unconstitutional treatment. That
requirement before his PDAF can be tapped as a funding source, thereby highlighting the such informal practices do exist and have, in fact, been constantly observed throughout the
indispensability of the said act to the entire budget execution process: 192 years has not been substantially disputed here. As pointed out by Chief Justice Maria Lourdes
P.A. Sereno (Chief Justice Sereno) during the Oral Arguments of these cases:193
Chief Justice Sereno:
Justice Bernabe: Now, without the individual legislator’s identification of the project, can the
PDAF of the legislator be utilized?
Now, from the responses of the representative of both, the DBM and two (2) Houses of
Congress, if we enforces the initial thought that I have, after I had seen the extent of this
Solicitor General Jardeleza: No, Your Honor. research made by my staff, that neither the Executive nor Congress frontally faced the question
of constitutional compatibility of how they were engineering the budget process. In fact, the
Justice Bernabe: It cannot? words you have been using, as the three lawyers of the DBM, and both Houses of Congress has
also been using is surprise; surprised that all of these things are now surfacing. In fact, I thought
that what the 2013 PDAF provisions did was to codify in one section all the past practice that
Solicitor General Jardeleza: It cannot… (interrupted) had been done since 1991. In a certain sense, we should be thankful that they are all now in the
PDAF Special Provisions. x x x (Emphasis and underscoring supplied)
Justice Bernabe: So meaning you should have the identification of the project by the individual
legislator? Ultimately, legislators cannot exercise powers which they do not have, whether through formal
measures written into the law or informal practices institutionalized in government agencies, else
the Executive department be deprived of what the Constitution has vested as its own.
2. Non-delegability of Legislative Power. individual legislators are given a personal lump-sum fund from which they are able to dictate (a)
how much from such fund would go to (b) a specific project or beneficiary that they themselves
also determine. As these two (2) acts comprise the exercise of the power of appropriation as
a. Statement of Principle.
described in Bengzon, and given that the 2013 PDAF Article authorizes individual legislators to
perform the same, undoubtedly, said legislators have been conferred the power to legislate
As an adjunct to the separation of powers principle,194 legislative power shall be exclusively which the Constitution does not, however, allow. Thus, keeping with the principle of non-
exercised by the body to which the Constitution has conferred the same. In particular, Section 1, delegability of legislative power, the Court hereby declares the 2013 PDAF Article, as well as all
Article VI of the 1987 Constitution states that such power shall be vested in the Congress of the other forms of Congressional Pork Barrel which contain the similar legislative identification
Philippines which shall consist of a Senate and a House of Representatives, except to the extent feature as herein discussed, as unconstitutional.
reserved to the people by the provision on initiative and referendum. 195 Based on this provision,
it is clear that only Congress, acting as a bicameral body, and the people, through the process of
3. Checks and Balances.
initiative and referendum, may constitutionally wield legislative power and no other. This premise
embodies the principle of non-delegability of legislative power, and the only recognized
exceptions thereto would be: (a) delegated legislative power to local governments which, by a. Statement of Principle; Item-Veto Power.
immemorial practice, are allowed to legislate on purely local matters;196 and (b) constitutionally-
grafted exceptions such as the authority of the President to, by law, exercise powers necessary
The fact that the three great powers of government are intended to be kept separate and distinct
and proper to carry out a declared national policy in times of war or other national
does not mean that they are absolutely unrestrained and independent of each other. The
emergency,197or fix within specified limits, and subject to such limitations and restrictions as
Constitution has also provided for an elaborate system of checks and balances to secure
Congress may impose, tariff rates, import and export quotas, tonnage and wharfage dues, and
coordination in the workings of the various departments of the government. 203
other duties or imposts within the framework of the national development program of the
Government.198
A prime example of a constitutional check and balance would be the President’s power to veto
an item written into an appropriation, revenue or tariff bill submitted to him by Congress for
Notably, the principle of non-delegability should not be confused as a restriction to delegate rule-
approval through a process known as "bill presentment." The President‘s item-veto power is
making authority to implementing agencies for the limited purpose of either filling up the details
found in Section 27(2), Article VI of the 1987 Constitution which reads as follows:
of the law for its enforcement (supplementary rule-making) or ascertaining facts to bring the law
into actual operation (contingent rule-making).199The conceptual treatment and limitations of
delegated rule-making were explained in the case of People v. Maceren200 as follows: Sec. 27. x x x.

The grant of the rule-making power to administrative agencies is a relaxation of the principle of xxxx
separation of powers and is an exception to the nondelegation of legislative powers.
Administrative regulations or "subordinate legislation" calculated to promote the public interest
are necessary because of "the growing complexity of modern life, the multiplication of the (2) The President shall have the power to veto any particular item or items in an appropriation,
subjects of governmental regulations, and the increased difficulty of administering the law." revenue, or tariff bill, but the veto shall not affect the item or items to which he does not object.

xxxx The presentment of appropriation, revenue or tariff bills to the President, wherein he may
exercise his power of item-veto, forms part of the "single, finely wrought and exhaustively
considered, procedures" for law-passage as specified under the Constitution.204 As stated in
Nevertheless, it must be emphasized that the rule-making power must be confined to details for Abakada, the final step in the law-making process is the "submission of the bill to the President
regulating the mode or proceeding to carry into effect the law as it has been enacted. The power for approval. Once approved, it takes effect as law after the required publication." 205
cannot be extended to amending or expanding the statutory requirements or to embrace matters
not covered by the statute. Rules that subvert the statute cannot be sanctioned. (Emphases
Elaborating on the President‘s item-veto power and its relevance as a check on the legislature,
supplied)
the Court, in Bengzon, explained that:206

b. Application.
The former Organic Act and the present Constitution of the Philippines make the Chief Executive
an integral part of the law-making power. His disapproval of a bill, commonly known as a veto, is
In the cases at bar, the Court observes that the 2013 PDAF Article, insofar as it confers post- essentially a legislative act. The questions presented to the mind of the Chief Executive are
enactment identification authority to individual legislators, violates the principle of non- precisely the same as those the legislature must determine in passing a bill, except that his will
delegability since said legislators are effectively allowed to individually exercise the power of be a broader point of view.
appropriation, which – as settled in Philconsa – is lodged in Congress.201 That the power to
appropriate must be exercised only through legislation is clear from Section 29(1), Article VI of
The Constitution is a limitation upon the power of the legislative department of the government,
the 1987 Constitution which states that: "No money shall be paid out of the Treasury except in
pursuance of an appropriation made by law." To understand what constitutes an act of but in this respect it is a grant of power to the executive department. The Legislature has the
appropriation, the Court, in Bengzon v. Secretary of Justice and Insular Auditor 202 (Bengzon), affirmative power to enact laws; the Chief Executive has the negative power by the constitutional
exercise of which he may defeat the will of the Legislature. It follows that the Chief Executive
held that the power of appropriation involves (a) the setting apart by law of a certain sum from
the public revenue for (b) a specified purpose. Essentially, under the 2013 PDAF Article, must find his authority in the Constitution. But in exercising that authority he may not be confined
to rules of strict construction or hampered by the unwise interference of the judiciary. The courts
will indulge every intendment in favor of the constitutionality of a veto in the same manner as In contrast, what beckons constitutional infirmity are appropriations which merely provide for a
they will presume the constitutionality of an act as originally passed by the Legislature. singular lump-sum amount to be tapped as a source of funding for multiple purposes. Since such
(Emphases supplied) appropriation type necessitates the further determination of both the actual amount to be
expended and the actual purpose of the appropriation which must still be chosen from the
multiple purposes stated in the law, it cannot be said that the appropriation law already indicates
The justification for the President‘s item-veto power rests on a variety of policy goals such as to
a "specific appropriation of money‖ and hence, without a proper line-item which the President
prevent log-rolling legislation,207 impose fiscal restrictions on the legislature, as well as to fortify
may veto. As a practical result, the President would then be faced with the predicament of either
the executive branch‘s role in the budgetary process.208 In Immigration and Naturalization
vetoing the entire appropriation if he finds some of its purposes wasteful or undesirable, or
Service v. Chadha, the US Supreme Court characterized the President‘s item-power as "a
approving the entire appropriation so as not to hinder some of its legitimate purposes. Finally, it
salutary check upon the legislative body, calculated to guard the community against the effects
may not be amiss to state that such arrangement also raises non-delegability issues considering
of factions, precipitancy, or of any impulse unfriendly to the public good, which may happen to
that the implementing authority would still have to determine, again, both the actual amount to
influence a majority of that body"; phrased differently, it is meant to "increase the chances in
be expended and the actual purpose of the appropriation. Since the foregoing determinations
favor of the community against the passing of bad laws, through haste, inadvertence, or
constitute the integral aspects of the power to appropriate, the implementing authority would, in
design."209
effect, be exercising legislative prerogatives in violation of the principle of non-delegability.

For the President to exercise his item-veto power, it necessarily follows that there exists a proper
b. Application.
"item" which may be the object of the veto. An item, as defined in the field of appropriations,
pertains to "the particulars, the details, the distinct and severable parts of the appropriation or of
the bill." In the case of Bengzon v. Secretary of Justice of the Philippine Islands, 210 the US In these cases, petitioners claim that "in the current x x x system where the PDAF is a lump-sum
Supreme Court characterized an item of appropriation as follows: appropriation, the legislator‘s identification of the projects after the passage of the GAA denies
the President the chance to veto that item later on."212 Accordingly, they submit that the "item
veto power of the President mandates that appropriations bills adopt line-item budgeting" and
An item of an appropriation bill obviously means an item which, in itself, is a specific
that "Congress cannot choose a mode of budgeting which effectively renders the
appropriation of money, not some general provision of law which happens to be put into an
constitutionally-given power of the President useless."213
appropriation bill. (Emphases supplied)

On the other hand, respondents maintain that the text of the Constitution envisions a process
On this premise, it may be concluded that an appropriation bill, to ensure that the President may
which is intended to meet the demands of a modernizing economy and, as such, lump-sum
be able to exercise his power of item veto, must contain "specific appropriations of money" and
appropriations are essential to financially address situations which are barely foreseen when a
not only "general provisions" which provide for parameters of appropriation.
GAA is enacted. They argue that the decision of the Congress to create some lump-sum
appropriations is constitutionally allowed and textually-grounded.214
Further, it is significant to point out that an item of appropriation must be an item characterized
by singular correspondence – meaning an allocation of a specified singular amount for a
The Court agrees with petitioners.
specified singular purpose, otherwise known as a "line-item."211 This treatment not only allows
the item to be consistent with its definition as a "specific appropriation of money" but also
ensures that the President may discernibly veto the same. Based on the foregoing formulation, Under the 2013 PDAF Article, the amount of ₱24.79 Billion only appears as a collective
the existing Calamity Fund, Contingent Fund and the Intelligence Fund, being appropriations allocation limit since the said amount would be further divided among individual legislators who
which state a specified amount for a specific purpose, would then be considered as "line- item" would then receive personal lump-sum allocations and could, after the GAA is passed,
appropriations which are rightfully subject to item veto. Likewise, it must be observed that an effectively appropriate PDAF funds based on their own discretion. As these intermediate
appropriation may be validly apportioned into component percentages or values; however, it is appropriations are made by legislators only after the GAA is passed and hence, outside of the
crucial that each percentage or value must be allocated for its own corresponding purpose for law, it necessarily means that the actual items of PDAF appropriation would not have been
such component to be considered as a proper line-item. Moreover, as Justice Carpio correctly written into the General Appropriations Bill and thus effectuated without veto consideration. This
pointed out, a valid appropriation may even have several related purposes that are by kind of lump-sum/post-enactment legislative identification budgeting system fosters the creation
accounting and budgeting practice considered as one purpose, e.g., MOOE (maintenance and of a budget within a budget" which subverts the prescribed procedure of presentment and
other operating expenses), in which case the related purposes shall be deemed sufficiently consequently impairs the President‘s power of item veto. As petitioners aptly point out, the
specific for the exercise of the President‘s item veto power. Finally, special purpose funds and above-described system forces the President to decide between (a) accepting the entire ₱24.79
discretionary funds would equally square with the constitutional mechanism of item-veto for as Billion PDAF allocation without knowing the specific projects of the legislators, which may or may
long as they follow the rule on singular correspondence as herein discussed. Anent special not be consistent with his national agenda and (b) rejecting the whole PDAF to the detriment of
purpose funds, it must be added that Section 25(4), Article VI of the 1987 Constitution requires all other legislators with legitimate projects.215
that the "special appropriations bill shall specify the purpose for which it is intended, and shall be
supported by funds actually available as certified by the National Treasurer, or t o be raised by a
Moreover, even without its post-enactment legislative identification feature, the 2013 PDAF
corresponding revenue proposal therein." Meanwhile, with respect to discretionary funds,
Article would remain constitutionally flawed since it would then operate as a prohibited form of
Section 2 5(6), Article VI of the 1987 Constitution requires that said funds "shall be disbursed
lump-sum appropriation above-characterized. In particular, the lump-sum amount of ₱24.79
only for public purposes to be supported by appropriate vouchers and subject to such guidelines
Billion would be treated as a mere funding source allotted for multiple purposes of spending, i.e.,
as may be prescribed by law."
scholarships, medical missions, assistance to indigents, preservation of historical materials,
construction of roads, flood control, etc. This setup connotes that the appropriation law leaves
the actual amounts and purposes of the appropriation for further determination and, therefore,
does not readily indicate a discernible item which may be subject to the President‘s power of Sec. 14. No Senator or Member of the House of Representatives may personally appear as
item veto. counsel before any court of justice or before the Electoral Tribunals, or quasi-judicial and other
administrative bodies. Neither shall he, directly or indirectly, be interested financially in any
contract with, or in any franchise or special privilege granted by the Government, or any
In fact, on the accountability side, the same lump-sum budgeting scheme has, as the CoA
subdivision, agency, or instrumentality thereof, including any government-owned or controlled
Chairperson relays, "limited state auditors from obtaining relevant data and information that
corporation, or its subsidiary, during his term of office. He shall not intervene in any matter
would aid in more stringently auditing the utilization of said Funds." 216 Accordingly, she
before any office of the Government for his pecuniary benefit or where he may be called upon to
recommends the adoption of a "line by line budget or amount per proposed program, activity or
act on account of his office. (Emphasis supplied)
project, and per implementing agency."217

Clearly, allowing legislators to intervene in the various phases of project implementation – a


Hence, in view of the reasons above-stated, the Court finds the 2013 PDAF Article, as well as all
matter before another office of government – renders them susceptible to taking undue
Congressional Pork Barrel Laws of similar operation, to be unconstitutional. That such budgeting
advantage of their own office.
system provides for a greater degree of flexibility to account for future contingencies cannot be
an excuse to defeat what the Constitution requires. Clearly, the first and essential truth of the
matter is that unconstitutional means do not justify even commendable ends. 218 The Court, however, cannot completely agree that the same post-enactment authority and/or the
individual legislator‘s control of his PDAF per se would allow him to perpetuate himself in office.
Indeed, while the Congressional Pork Barrel and a legislator‘s use thereof may be linked to this
c. Accountability.
area of interest, the use of his PDAF for re-election purposes is a matter which must be
analyzed based on particular facts and on a case-to-case basis.
Petitioners further relate that the system under which various forms of Congressional Pork Barrel
operate defies public accountability as it renders Congress incapable of checking itself or its
Finally, while the Court accounts for the possibility that the close operational proximity between
Members. In particular, they point out that the Congressional Pork Barrel "gives each legislator a
legislators and the Executive department, through the former‘s post-enactment participation,
direct, financial interest in the smooth, speedy passing of the yearly budget" which turns them
may affect the process of impeachment, this matter largely borders on the domain of politics and
"from fiscalizers" into "financially-interested partners."219 They also claim that the system has an
does not strictly concern the Pork Barrel System‘s intrinsic constitutionality. As such, it is an
effect on re- election as "the PDAF excels in self-perpetuation of elective officials." Finally, they
improper subject of judicial assessment.
add that the "PDAF impairs the power of impeachment" as such "funds are indeed quite useful,
‘to well, accelerate the decisions of senators.‘"220
In sum, insofar as its post-enactment features dilute congressional oversight and violate Section
14, Article VI of the 1987 Constitution, thus impairing public accountability, the 2013 PDAF
The Court agrees in part.
Article and other forms of Congressional Pork Barrel of similar nature are deemed as
unconstitutional.
The aphorism forged under Section 1, Article XI of the 1987 Constitution, which states that
"public office is a public trust," is an overarching reminder that every instrumentality of
4. Political Dynasties.
government should exercise their official functions only in accordance with the principles of the
Constitution which embodies the parameters of the people‘s trust. The notion of a public trust
connotes accountability,221 hence, the various mechanisms in the Constitution which are One of the petitioners submits that the Pork Barrel System enables politicians who are members
designed to exact accountability from public officers. of political dynasties to accumulate funds to perpetuate themselves in power, in contravention of
Section 26, Article II of the 1987 Constitution225 which states that:
Among others, an accountability mechanism with which the proper expenditure of public funds
may be checked is the power of congressional oversight. As mentioned in Sec. 26. The State shall guarantee equal access to opportunities for public service, and prohibit
Abakada,222 congressional oversight may be performed either through: (a) scrutiny based political dynasties as may be defined by law. (Emphasis and underscoring supplied)
primarily on Congress‘ power of appropriation and the budget hearings conducted in connection
with it, its power to ask heads of departments to appear before and be heard by either of its
At the outset, suffice it to state that the foregoing provision is considered as not self-executing
Houses on any matter pertaining to their departments and its power of confirmation; 223 or (b)
due to the qualifying phrase "as may be defined by law." In this respect, said provision does not,
investigation and monitoring of the implementation of laws pursuant to the power of Congress to
by and of itself, provide a judicially enforceable constitutional right but merely specifies guideline
conduct inquiries in aid of legislation.224
for legislative or executive action.226 Therefore, since there appears to be no standing law which
crystallizes the policy on political dynasties for enforcement, the Court must defer from ruling on
The Court agrees with petitioners that certain features embedded in some forms of this issue.
Congressional Pork Barrel, among others the 2013 PDAF Article, has an effect on congressional
oversight. The fact that individual legislators are given post-enactment roles in the
In any event, the Court finds the above-stated argument on this score to be largely speculative
implementation of the budget makes it difficult for them to become disinterested "observers"
since it has not been properly demonstrated how the Pork Barrel System would be able to
when scrutinizing, investigating or monitoring the implementation of the appropriation law. To a
propagate political dynasties.
certain extent, the conduct of oversight would be tainted as said legislators, who are vested with
post-enactment authority, would, in effect, be checking on activities in which they themselves
participate. Also, it must be pointed out that this very same concept of post-enactment 5. Local Autonomy.
authorization runs afoul of Section 14, Article VI of the 1987 Constitution which provides that:
The State‘s policy on local autonomy is principally stated in Section 25, Article II and Sections 2 inhabitants of such political units. The decision we reach today conforms not only to the letter of
and 3, Article X of the 1987 Constitution which read as follows: the pertinent laws but also to the spirit of the Constitution.229 (Emphases and underscoring
supplied)
ARTICLE II
In the cases at bar, petitioners contend that the Congressional Pork Barrel goes against the
constitutional principles on local autonomy since it allows district representatives, who are
Sec. 25. The State shall ensure the autonomy of local governments.
national officers, to substitute their judgments in utilizing public funds for local
development.230 The Court agrees with petitioners.
ARTICLE X
Philconsa described the 1994 CDF as an attempt "to make equal the unequal" and that "it is also
Sec. 2. The territorial and political subdivisions shall enjoy local autonomy. a recognition that individual members of Congress, far more than the President and their
congressional colleagues, are likely to be knowledgeable about the needs of their respective
constituents and the priority to be given each project."231 Drawing strength from this
Sec. 3. The Congress shall enact a local government code which shall provide for a more
pronouncement, previous legislators justified its existence by stating that "the relatively small
responsive and accountable local government structure instituted through a system of projects implemented under the Congressional Pork Barrel complement and link the national
decentralization with effective mechanisms of recall, initiative, and referendum, allocate among development goals to the countryside and grassroots as well as to depressed areas which are
the different local government units their powers, responsibilities, and resources, and provide for
overlooked by central agencies which are preoccupied with mega-projects.232 Similarly, in his
the qualifications, election, appointment and removal, term, salaries, powers and functions and August 23, 2013 speech on the "abolition" of PDAF and budgetary reforms, President Aquino
duties of local officials, and all other matters relating to the organization and operation of the mentioned that the Congressional Pork Barrel was originally established for a worthy goal, which
local units.
is to enable the representatives to identify projects for communities that the LGU concerned
cannot afford.233
Pursuant thereto, Congress enacted RA 7160,227 otherwise known as the "Local Government
Code of 1991" (LGC), wherein the policy on local autonomy had been more specifically
Notwithstanding these declarations, the Court, however, finds an inherent defect in the system
explicated as follows: which actually belies the avowed intention of "making equal the unequal." In particular, the Court
observes that the gauge of PDAF and CDF allocation/division is based solely on the fact of
Sec. 2. Declaration of Policy. – (a) It is hereby declared the policy of the State that the territorial office, without taking into account the specific interests and peculiarities of the district the
and political subdivisions of the State shall enjoy genuine and meaningful local autonomy to legislator represents. In this regard, the allocation/division limits are clearly not based on
enable them to attain their fullest development as self-reliant communities and make them more genuine parameters of equality, wherein economic or geographic indicators have been taken
effective partners in the attainment of national goals. Toward this end, the State shall provide for into consideration. As a result, a district representative of a highly-urbanized metropolis gets the
a more responsive and accountable local government structure instituted through a system of same amount of funding as a district representative of a far-flung rural province which would be
decentralization whereby local government units shall be given more powers, authority, relatively "underdeveloped" compared to the former. To add, what rouses graver scrutiny is that
responsibilities, and resources. The process of decentralization shall proceed from the National even Senators and Party-List Representatives – and in some years, even the Vice-President –
Government to the local government units. who do not represent any locality, receive funding from the Congressional Pork Barrel as well.
These certainly are anathema to the Congressional Pork Barrel‘s original intent which is "to
make equal the unequal." Ultimately, the PDAF and CDF had become personal funds under the
xxxx effective control of each legislator and given unto them on the sole account of their office.

(c) It is likewise the policy of the State to require all national agencies and offices to conduct The Court also observes that this concept of legislator control underlying the CDF and PDAF
periodic consultations with appropriate local government units, nongovernmental and people‘s conflicts with the functions of the various Local Development Councils (LDCs) which are already
organizations, and other concerned sectors of the community before any project or program is legally mandated to "assist the corresponding sanggunian in setting the direction of economic
implemented in their respective jurisdictions. (Emphases and underscoring supplied) and social development, and coordinating development efforts within its territorial
jurisdiction."234 Considering that LDCs are instrumentalities whose functions are essentially
The above-quoted provisions of the Constitution and the LGC reveal the policy of the State to geared towards managing local affairs,235 their programs, policies and resolutions should not be
empower local government units (LGUs) to develop and ultimately, become self-sustaining and overridden nor duplicated by individual legislators, who are national officers that have no law-
effective contributors to the national economy. As explained by the Court in Philippine Gamefowl making authority except only when acting as a body. The undermining effect on local autonomy
Commission v. Intermediate Appellate Court:228 caused by the post-enactment authority conferred to the latter was succinctly put by petitioners
in the following wise:236
This is as good an occasion as any to stress the commitment of the Constitution to the policy of
local autonomy which is intended to provide the needed impetus and encouragement to the With PDAF, a Congressman can simply bypass the local development council and initiate
development of our local political subdivisions as "self - reliant communities." In the words of projects on his own, and even take sole credit for its execution. Indeed, this type of personality-
Jefferson, "Municipal corporations are the small republics from which the great one derives its driven project identification has not only contributed little to the overall development of the
strength." The vitalization of local governments will enable their inhabitants to fully exploit their district, but has even contributed to "further weakening infrastructure planning and coordination
resources and more important, imbue them with a deepened sense of involvement in public efforts of the government."
affairs as members of the body politic. This objective could be blunted by undue interference by
the national government in purely local affairs which are best resolved by the officials and
Thus, insofar as individual legislators are authorized to intervene in purely local matters and To constitute an appropriation there must be money placed in a fund applicable to the
thereby subvert genuine local autonomy, the 2013 PDAF Article as well as all other similar forms designated purpose. The word appropriate means to allot, assign, set apart or apply to a
of Congressional Pork Barrel is deemed unconstitutional. particular use or purpose. An appropriation in the sense of the constitution means the setting
apart a portion of the public funds for a public purpose. No particular form of words is necessary
for the purpose, if the intention to appropriate is plainly manifested. (Emphases supplied)
With this final issue on the Congressional Pork Barrel resolved, the Court now turns to the
substantive issues involving the Presidential Pork Barrel.
Thus, based on the foregoing, the Court cannot sustain the argument that the appropriation must
be the "primary and specific" purpose of the law in order for a valid appropriation law to exist. To
C. Substantive Issues on the Presidential Pork Barrel.
reiterate, if a legal provision designates a determinate or determinable amount of money and
allocates the same for a particular public purpose, then the legislative intent to appropriate
1. Validity of Appropriation. becomes apparent and, hence, already sufficient to satisfy the requirement of an "appropriation
made by law" under contemplation of the Constitution.
Petitioners preliminarily assail Section 8 of PD 910 and Section 12 of PD1869 (now, amended
by PD 1993), which respectively provide for the Malampaya Funds and the Presidential Social Section 8 of PD 910 pertinently provides:
Fund, as invalid appropriations laws since they do not have the "primary and specific" purpose of
authorizing the release of public funds from the National Treasury. Petitioners submit that
Section 8. Appropriations. x x x
Section 8 of PD 910 is not an appropriation law since the "primary and specific‖ purpose of PD
910 is the creation of an Energy Development Board and Section 8 thereof only created a
Special Fund incidental thereto.237 In similar regard, petitioners argue that Section 12 of PD 1869 All fees, revenues and receipts of the Board from any and all sources including receipts from
is neither a valid appropriations law since the allocation of the Presidential Social Fund is merely service contracts and agreements such as application and processing fees, signature bonus,
incidental to the "primary and specific" purpose of PD 1869 which is the amendment of the discovery bonus, production bonus; all money collected from concessionaires, representing
Franchise and Powers of PAGCOR.238 In view of the foregoing, petitioners suppose that such unspent work obligations, fines and penalties under the Petroleum Act of 1949; as well as the
funds are being used without any valid law allowing for their proper appropriation in violation of government share representing royalties, rentals, production share on service contracts and
Section 29(1), Article VI of the 1987 Constitution which states that: "No money shall be paid out similar payments on the exploration, development and exploitation of energy resources, shall
of the Treasury except in pursuance of an appropriation made by law." 239 form part of a Special Fund to be used to finance energy resource development and exploitation
programs and projects of the government and for such other purposes as may be hereafter
directed by the President. (Emphases supplied)
The Court disagrees.

Whereas Section 12 of PD 1869, as amended by PD 1993, reads:


"An appropriation made by law‖ under the contemplation of Section 29(1), Article VI of the 1987
Constitution exists when a provision of law (a) sets apart a determinate or
determinable240 amount of money and (b) allocates the same for a particular public purpose. Sec. 12. Special Condition of Franchise. — After deducting five (5%) percent as Franchise Tax,
These two minimum designations of amount and purpose stem from the very definition of the the Fifty (50%) percent share of the Government in the aggregate gross earnings of the
word "appropriation," which means "to allot, assign, set apart or apply to a particular use or Corporation from this Franchise, or 60% if the aggregate gross earnings be less than
purpose," and hence, if written into the law, demonstrate that the legislative intent to appropriate ₱150,000,000.00 shall be set aside and shall accrue to the General Fund to finance the priority
exists. As the Constitution "does not provide or prescribe any particular form of words or infrastructure development projects and to finance the restoration of damaged or destroyed
religious recitals in which an authorization or appropriation by Congress shall be made, except facilities due to calamities, as may be directed and authorized by the Office of the President of
that it be ‘made by law,‘" an appropriation law may – according to Philconsa – be "detailed and the Philippines. (Emphases supplied)
as broad as Congress wants it to be" for as long as the intent to appropriate may be gleaned
from the same. As held in the case of Guingona, Jr.:241
Analyzing the legal text vis-à-vis the above-mentioned principles, it may then be concluded that
(a) Section 8 of PD 910, which creates a Special Fund comprised of "all fees, revenues, and
There is no provision in our Constitution that provides or prescribes any particular form of words receipts of the Energy Development Board from any and all sources" (a determinable amount)
or religious recitals in which an authorization or appropriation by Congress shall be made, "to be used to finance energy resource development and exploitation programs and projects of
except that it be "made by law," such as precisely the authorization or appropriation under the the government and for such other purposes as may be hereafter directed by the President" (a
questioned presidential decrees. In other words, in terms of time horizons, an appropriation may specified public purpose), and (b) Section 12 of PD 1869, as amended by PD 1993, which
be made impliedly (as by past but subsisting legislations) as well as expressly for the current similarly sets aside, "after deducting five (5%) percent as Franchise Tax, the Fifty (50%) percent
fiscal year (as by enactment of laws by the present Congress), just as said appropriation may be share of the Government in the aggregate gross earnings of PAGCOR, or 60%, if the aggregate
made in general as well as in specific terms. The Congressional authorization may be embodied gross earnings be less than ₱150,000,000.00" (also a determinable amount) "to finance the
in annual laws, such as a general appropriations act or in special provisions of laws of general or priority infrastructure development projects and x x x the restoration of damaged or destroyed
special application which appropriate public funds for specific public purposes, such as the facilities due to calamities, as may be directed and authorized by the Office of the President of
questioned decrees. An appropriation measure is sufficient if the legislative intention clearly and the Philippines" (also a specified public purpose), are legal appropriations under Section 29(1),
certainly appears from the language employed (In re Continuing Appropriations, 32 P. 272), Article VI of the 1987 Constitution.
whether in the past or in the present. (Emphases and underscoring supplied)
In this relation, it is apropos to note that the 2013 PDAF Article cannot be properly deemed as a
Likewise, as ruled by the US Supreme Court in State of Nevada v. La Grave: 242 legal appropriation under the said constitutional provision precisely because, as earlier stated, it
contains post-enactment measures which effectively create a system of intermediate contradicting respondents‘ own position that it is limited only to "energy resource development
appropriations. These intermediate appropriations are the actual appropriations meant for and exploitation programs and projects of the government."251 Thus, while Section 8 of PD 910
enforcement and since they are made by individual legislators after the GAA is passed, they may have passed the completeness test since the policy of energy development is clearly
occur outside the law. As such, the Court observes that the real appropriation made under the deducible from its text, the phrase "and for such other purposes as may be hereafter directed by
2013 PDAF Article is not the ₱24.79 Billion allocated for the entire PDAF, but rather the post- the President" under the same provision of law should nonetheless be stricken down as
enactment determinations made by the individual legislators which are, to repeat, occurrences unconstitutional as it lies independently unfettered by any sufficient standard of the delegating
outside of the law. Irrefragably, the 2013 PDAF Article does not constitute an "appropriation law. This notwithstanding, it must be underscored that the rest of Section 8, insofar as it allows
made by law" since it, in its truest sense, only authorizes individual legislators to appropriate in for the use of the Malampaya Funds "to finance energy resource development and exploitation
violation of the non-delegability principle as afore-discussed. programs and projects of the government," remains legally effective and subsisting. Truth be
told, the declared unconstitutionality of the aforementioned phrase is but an assurance that the
Malampaya Funds would be used – as it should be used – only in accordance with the avowed
2. Undue Delegation.
purpose and intention of PD 910.

On a related matter, petitioners contend that Section 8 of PD 910 constitutes an undue


As for the Presidential Social Fund, the Court takes judicial notice of the fact that Section 12 of
delegation of legislative power since the phrase "and for such other purposes as may be
PD 1869 has already been amended by PD 1993 which thus moots the parties‘ submissions on
hereafter directed by the President" gives the President "unbridled discretion to determine for
the same.252 Nevertheless, since the amendatory provision may be readily examined under the
what purpose the funds will be used."243 Respondents, on the other hand, urged the Court to
current parameters of discussion, the Court proceeds to resolve its constitutionality.
apply the principle of ejusdem generis to the same section and thus, construe the phrase "and
for such other purposes as may be hereafter directed by the President" to refer only to other
purposes related "to energy resource development and exploitation programs and projects of the Primarily, Section 12 of PD 1869, as amended by PD 1993, indicates that the Presidential Social
government."244 Fund may be used "to first, finance the priority infrastructure development projects and second,
to finance the restoration of damaged or destroyed facilities due to calamities, as may be
directed and authorized by the Office of the President of the Philippines." The Court finds that
The Court agrees with petitioners‘ submissions.
while the second indicated purpose adequately curtails the authority of the President to spend
the Presidential Social Fund only for restoration purposes which arise from calamities, the first
While the designation of a determinate or determinable amount for a particular public purpose is indicated purpose, however, gives him carte blanche authority to use the same fund for any
sufficient for a legal appropriation to exist, the appropriation law must contain adequate infrastructure project he may so determine as a "priority". Verily, the law does not supply a
legislative guidelines if the same law delegates rule-making authority to the Executive245 either definition of "priority in frastructure development projects" and hence, leaves the President
for the purpose of (a) filling up the details of the law for its enforcement, known as without any guideline to construe the same. To note, the delimitation of a project as one of
supplementary rule-making, or (b) ascertaining facts to bring the law into actual operation, "infrastructure" is too broad of a classification since the said term could pertain to any kind of
referred to as contingent rule-making.246 There are two (2) fundamental tests to ensure that the facility. This may be deduced from its lexicographic definition as follows: "the underlying
legislative guidelines for delegated rule-making are indeed adequate. The first test is called the framework of a system, especially public services and facilities (such as highways, schools,
"completeness test." Case law states that a law is complete when it sets forth therein the policy bridges, sewers, and water-systems) needed to support commerce as well as economic and
to be executed, carried out, or implemented by the delegate. On the other hand, the second test residential development."253 In fine, the phrase "to finance the priority infrastructure development
is called the "sufficient standard test." Jurisprudence holds that a law lays down a sufficient projects" must be stricken down as unconstitutional since – similar to the above-assailed
standard when it provides adequate guidelines or limitations in the law to map out the provision under Section 8 of PD 910 – it lies independently unfettered by any sufficient standard
boundaries of the delegate‘s authority and prevent the delegation from running riot. 247 To be of the delegating law. As they are severable, all other provisions of Section 12 of PD 1869, as
sufficient, the standard must specify the limits of the delegate‘s authority, announce the amended by PD 1993, remains legally effective and subsisting.
legislative policy, and identify the conditions under which it is to be implemented. 248
D. Ancillary Prayers. 1.
In view of the foregoing, the Court agrees with petitioners that the phrase "and for such other
purposes as may be hereafter directed by the President" under Section 8 of PD 910 constitutes
Petitioners’ Prayer to be Furnished Lists and Detailed Reports.
an undue delegation of legislative power insofar as it does not lay down a sufficient standard to
adequately determine the limits of the President‘s authority with respect to the purpose for which
the Malampaya Funds may be used. As it reads, the said phrase gives the President wide Aside from seeking the Court to declare the Pork Barrel System unconstitutional – as the Court
latitude to use the Malampaya Funds for any other purpose he may direct and, in effect, allows did so in the context of its pronouncements made in this Decision – petitioners equally pray that
him to unilaterally appropriate public funds beyond the purview of the law. That the subject the Executive Secretary and/or the DBM be ordered to release to the CoA and to the public: (a)
phrase may be confined only to "energy resource development and exploitation programs and "the complete schedule/list of legislators who have availed of their PDAF and VILP from the
projects of the government" under the principle of ejusdem generis, meaning that the general years 2003 to 2013, specifying the use of the funds, the project or activity and the recipient
word or phrase is to be construed to include – or be restricted to – things akin to, resembling, or entities or individuals, and all pertinent data thereto" (PDAF Use Schedule/List); 254 and (b) "the
of the same kind or class as those specifically mentioned,249 is belied by three (3) reasons: first, use of the Executive‘s lump-sum, discretionary funds, including the proceeds from the x x x
the phrase "energy resource development and exploitation programs and projects of the Malampaya Funds and remittances from the PAGCOR x x x from 2003 to 2013, specifying the x
government" states a singular and general class and hence, cannot be treated as a statutory x x project or activity and the recipient entities or individuals, and all pertinent data
reference of specific things from which the general phrase "for such other purposes" may be thereto"255 (Presidential Pork Use Report). Petitioners‘ prayer is grounded on Section 28, Article
limited; second, the said phrase also exhausts the class it represents, namely energy II and Section 7, Article III of the 1987 Constitution which read as follows:
development programs of the government;250 and, third, the Executive department has, in fact,
used the Malampaya Funds for non-energy related purposes under the subject phrase, thereby
ARTICLE II The request of the petitioners fails to meet this standard, there being no duty on the part of
respondent to prepare the list requested. (Emphases supplied)
Sec. 28. Subject to reasonable conditions prescribed by law, the State adopts and implements a
policy of full public disclosure of all its transactions involving public interest. In these cases, aside from the fact that none of the petitions are in the nature of mandamus
actions, the Court finds that petitioners have failed to establish a "a well-defined, clear and
certain legal right" to be furnished by the Executive Secretary and/or the DBM of their requested
ARTICLE III Sec. 7.
PDAF Use Schedule/List and Presidential Pork Use Report. Neither did petitioners assert any
law or administrative issuance which would form the bases of the latter‘s duty to furnish them
The right of the people to information on matters of public concern shall be recognized. Access with the documents requested. While petitioners pray that said information be equally released
to official records, and to documents and papers pertaining to official acts, transactions, or to the CoA, it must be pointed out that the CoA has not been impleaded as a party to these
decisions, as well as to government research data used as basis for policy development, shall cases nor has it filed any petition before the Court to be allowed access to or to compel the
be afforded the citizen, subject to such limitations as may be provided by law. release of any official document relevant to the conduct of its audit investigations. While the
Court recognizes that the information requested is a matter of significant public concern,
however, if only to ensure that the parameters of disclosure are properly foisted and so as not to
The Court denies petitioners‘ submission. unduly hamper the equally important interests of the government, it is constrained to deny
petitioners‘ prayer on this score, without prejudice to a proper mandamus case which they, or
Case law instructs that the proper remedy to invoke the right to information is to file a petition for even the CoA, may choose to pursue through a separate petition.
mandamus. As explained in the case of Legaspi v. Civil Service Commission:256
It bears clarification that the Court‘s denial herein should only cover petitioners‘ plea to be
While the manner of examining public records may be subject to reasonable regulation by the furnished with such schedule/list and report and not in any way deny them, or the general public,
government agency in custody thereof, the duty to disclose the information of public concern, access to official documents which are already existing and of public record. Subject to
and to afford access to public records cannot be discretionary on the part of said agencies. reasonable regulation and absent any valid statutory prohibition, access to these documents
Certainly, its performance cannot be made contingent upon the discretion of such agencies. should not be proscribed. Thus, in Valmonte, while the Court denied the application for
Otherwise, the enjoyment of the constitutional right may be rendered nugatory by any whimsical mandamus towards the preparation of the list requested by petitioners therein, it nonetheless
exercise of agency discretion. The constitutional duty, not being discretionary, its performance allowed access to the documents sought for by the latter, subject, however, to the custodian‘s
may be compelled by a writ of mandamus in a proper case. reasonable regulations,viz.:259

But what is a proper case for Mandamus to issue? In the case before Us, the public right to be In fine, petitioners are entitled to access to the documents evidencing loans granted by the
enforced and the concomitant duty of the State are unequivocably set forth in the Constitution. GSIS, subject to reasonable regulations that the latter may promulgate relating to the manner
and hours of examination, to the end that damage to or loss of the records may be avoided, that
undue interference with the duties of the custodian of the records may be prevented and that the
The decisive question on the propriety of the issuance of the writ of mandamus in this case is, right of other persons entitled to inspect the records may be insured Legaspi v. Civil Service
whether the information sought by the petitioner is within the ambit of the constitutional Commission, supra at p. 538, quoting Subido v. Ozaeta, 80 Phil. 383, 387. The petition, as to the
guarantee. (Emphases supplied) second and third alternative acts sought to be done by petitioners, is meritorious.

Corollarily, in the case of Valmonte v. Belmonte Jr.257 (Valmonte), it has been clarified that the However, the same cannot be said with regard to the first act sought by petitioners, i.e.,
right to information does not include the right to compel the preparation of "lists, abstracts,
summaries and the like." In the same case, it was stressed that it is essential that the "applicant
has a well -defined, clear and certain legal right to the thing demanded and that it is the "to furnish petitioners the list of the names of the Batasang Pambansa members belonging to
imperative duty of defendant to perform the act required." Hence, without the foregoing the UNIDO and PDP-Laban who were able to secure clean loans immediately before the
substantiations, the Court cannot grant a particular request for information. The pertinent February 7 election thru the intercession/marginal note of the then First Lady Imelda Marcos."
portions of Valmonte are hereunder quoted:258
The Court, therefore, applies the same treatment here.
Although citizens are afforded the right to information and, pursuant thereto, are entitled to
"access to official records," the Constitution does not accord them a right to compel custodians 2. Petitioners’ Prayer to Include Matters in Congressional Deliberations.
of official records to prepare lists, abstracts, summaries and the like in their desire to acquire
information on matters of public concern.
Petitioners further seek that the Court "order the inclusion in budgetary deliberations with the
Congress of all presently, off-budget, lump sum, discretionary funds including but not limited to,
It must be stressed that it is essential for a writ of mandamus to issue that the applicant has a proceeds from the x x x Malampaya Fund, remittances from the PAGCOR and the PCSO or the
well-defined, clear and certain legal right to the thing demanded and that it is the imperative duty
Executive‘s Social Funds."260
of defendant to perform the act required. The corresponding duty of the respondent to perform
the required act must be clear and specific Lemi v. Valencia, G.R. No. L-20768, November
29,1968,126 SCRA 203; Ocampo v. Subido, G.R. No. L-28344, August 27, 1976, 72 SCRA 443. Suffice it to state that the above-stated relief sought by petitioners covers a matter which is
generally left to the prerogative of the political branches of government. Hence, lest the Court
itself overreach, it must equally deny their prayer on this score.
3. Respondents’ Prayer to Lift TRO; Consequential Effects of Decision. and immediate effect of placing public funds beyond the control of the disbursing authority. In
fact, a SARO may even be withdrawn under certain circumstances which will prevent the actual
release of funds. On the other hand, the actual release of funds is brought about by the issuance
The final issue to be resolved stems from the interpretation accorded by the DBM to the concept
of the NCA,264 which is subsequent to the issuance of a SARO. As may be determined from the
of released funds. In response to the Court‘s September 10, 2013 TRO that enjoined the release
statements of the DBM representative during the Oral Arguments: 265
of the remaining PDAF allocated for the year 2013, the DBM issued Circular Letter No. 2013-8
dated September 27, 2013 (DBM Circular 2013-8) which pertinently reads as follows:
Justice Bernabe: Is the notice of allocation issued simultaneously with the SARO?
3.0 Nonetheless, PDAF projects funded under the FY 2013 GAA, where a Special Allotment
Release Order (SARO) has been issued by the DBM and such SARO has been obligated by the xxxx
implementing agencies prior to the issuance of the TRO, may continually be implemented and
disbursements thereto effected by the agencies concerned.
Atty. Ruiz: It comes after. The SARO, Your Honor, is only the go signal for the agencies to
obligate or to enter into commitments. The NCA, Your Honor, is already the go signal to the
Based on the text of the foregoing, the DBM authorized the continued implementation and treasury for us to be able to pay or to liquidate the amounts obligated in the SARO; so it comes
disbursement of PDAF funds as long as they are: first, covered by a SARO; and, second, that after. x x x The NCA, Your Honor, is the go signal for the MDS for the authorized government-
said SARO had been obligated by the implementing agency concerned prior to the issuance of disbursing banks to, therefore, pay the payees depending on the projects or projects covered by
the Court‘s September 10, 2013 TRO. the SARO and the NCA.

Petitioners take issue with the foregoing circular, arguing that "the issuance of the SARO does Justice Bernabe: Are there instances that SAROs are cancelled or revoked?
not yet involve the release of funds under the PDAF, as release is only triggered by the issuance
of a Notice of Cash Allocation [(NCA)]."261 As such, PDAF disbursements, even if covered by an
Atty. Ruiz: Your Honor, I would like to instead submit that there are instances that the SAROs
obligated SARO, should remain enjoined.
issued are withdrawn by the DBM.

For their part, respondents espouse that the subject TRO only covers "unreleased and
Justice Bernabe: They are withdrawn?
unobligated allotments." They explain that once a SARO has been issued and obligated by the
implementing agency concerned, the PDAF funds covered by the same are already "beyond the
reach of the TRO because they cannot be considered as ‘remaining PDAF.‘" They conclude that Atty. Ruiz: Yes, Your Honor x x x. (Emphases and underscoring supplied)
this is a reasonable interpretation of the TRO by the DBM.262
Thus, unless an NCA has been issued, public funds should not be treated as funds which have
The Court agrees with petitioners in part. been "released." In this respect, therefore, the disbursement of 2013 PDAF funds which are only
covered by obligated SAROs, and without any corresponding NCAs issued, must, at the time of
this Decision’s promulgation, be enjoined and consequently reverted to the unappropriated
At the outset, it must be observed that the issue of whether or not the Court‘s September 10,
surplus of the general fund. Verily, in view of the declared unconstitutionality of the 2013 PDAF
2013 TRO should be lifted is a matter rendered moot by the present Decision. The
Article, the funds appropriated pursuant thereto cannot be disbursed even though already
unconstitutionality of the 2013 PDAF Article as declared herein has the consequential effect of
obligated, else the Court sanctions the dealing of funds coming from an unconstitutional source.
converting the temporary injunction into a permanent one. Hence, from the promulgation of this
Decision, the release of the remaining PDAF funds for 2013, among others, is now permanently
enjoined. This same pronouncement must be equally applied to (a) the Malampaya Funds which have
been obligated but not released – meaning, those merely covered by a SARO – under the
phrase "and for such other purposes as may be hereafter directed by the President" pursuant to
The propriety of the DBM‘s interpretation of the concept of "release" must, nevertheless, be
Section 8 of PD 910; and (b) funds sourced from the Presidential Social Fund under the phrase
resolved as it has a practical impact on the execution of the current Decision. In particular, the
"to finance the priority infrastructure development projects" pursuant to Section 12 of PD 1869,
Court must resolve the issue of whether or not PDAF funds covered by obligated SAROs, at the
as amended by PD 1993, which were altogether declared by the Court as unconstitutional.
time this Decision is promulgated, may still be disbursed following the DBM‘s interpretation in
However, these funds should not be reverted to the general fund as afore-stated but instead,
DBM Circular 2013-8.
respectively remain under the Malampaya Funds and the Presidential Social Fund to be utilized
for their corresponding special purposes not otherwise declared as unconstitutional.
On this score, the Court agrees with petitioners‘ posturing for the fundamental reason that funds
covered by an obligated SARO are yet to be "released" under legal contemplation. A SARO, as
E. Consequential Effects of Decision.
defined by the DBM itself in its website, is "aspecific authority issued to identified agencies to
incur obligations not exceeding a given amount during a specified period for the purpose
indicated. It shall cover expenditures the release of which is subject to compliance with specific As a final point, it must be stressed that the Court‘s pronouncement anent the unconstitutionality
laws or regulations, or is subject to separate approval or clearance by competent authority." 263 of (a) the 2013 PDAF Article and its Special Provisions, (b) all other Congressional Pork Barrel
provisions similar thereto, and (c) the phrases (1) "and for such other purposes as may be
hereafter directed by the President" under Section 8 of PD 910, and (2) "to finance the priority
Based on this definition, it may be gleaned that a SARO only evinces the existence of an
infrastructure development projects" under Section 12 of PD 1869, as amended by PD 1993,
obligation and not the directive to pay. Practically speaking, the SARO does not have the direct
must only be treated as prospective in effect in view of the operative fact doctrine.
To explain, the operative fact doctrine exhorts the recognition that until the judiciary, in an execution, such as but not limited to the areas of project identification, modification and revision
appropriate case, declares the invalidity of a certain legislative or executive act, such act is of project identification, fund release and/or fund realignment, unrelated to the power of
presumed constitutional and thus, entitled to obedience and respect and should be properly congressional oversight; (c) all legal provisions of past and present Congressional Pork Barrel
enforced and complied with. As explained in the recent case of Commissioner of Internal Laws, such as the previous PDAF and CDF Articles and the various Congressional Insertions,
Revenue v. San Roque Power Corporation,266 the doctrine merely "reflects awareness that which confer/red personal, lump-sum allocations to legislators from which they are able to fund
precisely because the judiciary is the governmental organ which has the final say on whether or specific projects which they themselves determine; (d) all informal practices of similar import and
not a legislative or executive measure is valid, a period of time may have elapsed before it can effect, which the Court similarly deems to be acts of grave abuse of discretion amounting to lack
exercise the power of judicial review that may lead to a declaration of nullity. It would be to or excess of jurisdiction; and (e) the phrases (1) "and for such other purposes as may be
deprive the law of its quality of fairness and justice then, if there be no recognition of what had hereafter directed by the President" under Section 8 of Presidential Decree No. 910 and (2) "to
transpired prior to such adjudication."267 "In the language of an American Supreme Court finance the priority infrastructure development projects" under Section 12 of Presidential Decree
decision: ‘The actual existence of a statute, prior to such a determination of unconstitutionality, is No. 1869, as amended by Presidential Decree No. 1993, for both failing the sufficient standard
an operative fact and may have consequences which cannot justly be ignored.‘" 268 test in violation of the principle of non-delegability of legislative power.

For these reasons, this Decision should be heretofore applied prospectively. Accordingly, the Court‘s temporary injunction dated September 10, 2013 is hereby declared to
be PERMANENT. Thus, the disbursement/release of the remaining PDAF funds allocated for the
year 2013, as well as for all previous years, and the funds sourced from (1) the Malampaya
Conclusion
Funds under the phrase "and for such other purposes as may be hereafter directed by the
President" pursuant to Section 8 of Presidential Decree No. 910, and (2) the Presidential Social
The Court renders this Decision to rectify an error which has persisted in the chronicles of our Fund under the phrase "to finance the priority infrastructure development projects" pursuant to
history. In the final analysis, the Court must strike down the Pork Barrel System as Section 12 of Presidential Decree No. 1869, as amended by Presidential Decree No. 1993,
unconstitutional in view of the inherent defects in the rules within which it operates. To recount, which are, at the time this Decision is promulgated, not covered by Notice of Cash Allocations
insofar as it has allowed legislators to wield, in varying gradations, non-oversight, post- (NCAs) but only by Special Allotment Release Orders (SAROs), whether obligated or not, are
enactment authority in vital areas of budget execution, the system has violated the principle of hereby ENJOINED. The remaining PDAF funds covered by this permanent injunction shall not
separation of powers; insofar as it has conferred unto legislators the power of appropriation by be disbursed/released but instead reverted to the unappropriated surplus of the general fund,
giving them personal, discretionary funds from which they are able to fund specific projects while the funds under the Malampaya Funds and the Presidential Social Fund shall remain
which they themselves determine, it has similarly violated the principle of non-delegability of therein to be utilized for their respective special purposes not otherwise declared as
legislative power ; insofar as it has created a system of budgeting wherein items are not unconstitutional.
textualized into the appropriations bill, it has flouted the prescribed procedure of presentment
and, in the process, denied the President the power to veto items ; insofar as it has diluted the
On the other hand, due to improper recourse and lack of proper substantiation, the Court hereby
effectiveness of congressional oversight by giving legislators a stake in the affairs of budget
DENIES petitioners‘ prayer seeking that the Executive Secretary and/or the Department of
execution, an aspect of governance which they may be called to monitor and scrutinize, the
Budget and Management be ordered to provide the public and the Commission on Audit
system has equally impaired public accountability ; insofar as it has authorized legislators, who
complete lists/schedules or detailed reports related to the availments and utilization of the funds
are national officers, to intervene in affairs of purely local nature, despite the existence of
subject of these cases. Petitioners‘ access to official documents already available and of public
capable local institutions, it has likewise subverted genuine local autonomy ; and again, insofar
record which are related to these funds must, however, not be prohibited but merely subjected to
as it has conferred to the President the power to appropriate funds intended by law for energy-
the custodian‘s reasonable regulations or any valid statutory prohibition on the same. This denial
related purposes only to other purposes he may deem fit as well as other public funds under the
is without prejudice to a proper mandamus case which they or the Commission on Audit may
broad classification of "priority infrastructure development projects," it has once more
choose to pursue through a separate petition.
transgressed the principle of non-delegability.

The Court also DENIES petitioners prayer to order the inclusion of the funds subject of these
For as long as this nation adheres to the rule of law, any of the multifarious unconstitutional
cases in the budgetary deliberations of Congress as the same is a matter left to the prerogative
methods and mechanisms the Court has herein pointed out should never again be adopted in
of the political branches of government.
any system of governance, by any name or form, by any semblance or similarity, by any
influence or effect. Disconcerting as it is to think that a system so constitutionally unsound has
monumentally endured, the Court urges the people and its co-stewards in government to look Finally, the Court hereby DIRECTS all prosecutorial organs of the government to, within the
forward with the optimism of change and the awareness of the past. At a time of great civic bounds of reasonable dispatch, investigate and accordingly prosecute all government officials
unrest and vociferous public debate, the Court fervently hopes that its Decision today, while it and/or private individuals for possible criminal offenses related to the irregular, improper and/or
may not purge all the wrongs of society nor bring back what has been lost, guides this nation to unlawful disbursement/utilization of all funds under the Pork Barrel System.
the path forged by the Constitution so that no one may heretofore detract from its cause nor
stray from its course. After all, this is the Court‘s bounden duty and no other‘s.
This Decision is immediately executory but prospective in effect.

WHEREFORE, the petitions are PARTLY GRANTED. In view of the constitutional violations
SO ORDERED.
discussed in this Decision, the Court hereby declares as UNCONSTITUTIONAL: (a) the entire
2013 PDAF Article; (b) all legal provisions of past and present Congressional Pork Barrel Laws,
such as the previous PDAF and CDF Articles and the various Congressional Insertions, which
authorize/d legislators – whether individually or collectively organized into committees – to
intervene, assume or participate in any of the various post-enactment stages of the budget
G.R. No. 183891 October 19, 2011 The present motion for reconsideration rests on the following points:

ROMARICO J. MENDOZA, Petitioner, First. On January 7, 2010, during the pendency of the petitioner’s case before the
vs. Court, then President Gloria Macapagal-Arroyo signed RA No. 9903 into law. RA No.
PEOPLE OF THE PHILIPPINES, Respondent. 9903 mandates the effective withdrawal of all pending cases against employers who
would remit their delinquent contributions to the SSS within a specified period, viz.,
within six months after the law’s effectivity.4 The petitioner claims that in view of RA
R E SO L U T I O N
No. 9903 and its implementing rules, the settlement of his delinquent contributions in
2007 entitles him to an acquittal. He invokes the equal protection clause in support of
BRION, J.: his plea.

We resolve the motion for reconsideration filed by petitioner Romarico J. Mendoza seeking the Second. The petitioner alternatively prays that should the Court find his above
reversal of our Decision dated August 3, 2010. The Decision affirmed the petitioner’s conviction argument wanting, he should still be acquitted since the prosecution failed to prove all
for his failure to remit the Social Security Service (SSS) contributions of his employees. The the elements of the crime charged.
petitioner anchors the present motion on his supposed inclusion within the coverage of Republic
Act (RA) No. 9903 or the Social Security Condonation Law of 2009, whose passage the
Third. The petitioner prays that a fine be imposed, not imprisonment, should he be
petitioner claims to be a supervening event in his case. He further invokes the equal protection
found guilty.
clause in support of his motion.

The Solicitor General filed a Manifestation In Lieu of Comment and claims that the passage of
In our Decision dated August 3, 2010, we AFFIRMED, with modification, the decree of
RA No. 9903 constituted a supervening event in the petitioner’s case that supports the
conviction issued by both the trial and appellate courts for the petitioner’s violation of Section
petitioner’s acquittal "[a]fter a conscientious review of the case."5
22(a) and (d), in relation to Section 28 of RA No. 8282 or the Social Security Act of 1997. To
recall its highlights, our Decision emphasized that the petitioner readily admitted during trial that
he did not remit the SSS premium contributions of his employees at Summa Alta Tierra THE COURT’S RULING
Industries, Inc. from August 1998 to July 1999, in the amount of ₱239,756.80; inclusive of
penalties, this unremitted amount totaled to ₱421,151.09. The petitioner’s explanation for his
The petitioner’s arguments supporting his prayer for acquittal fail to convince us. However, we
failure to remit, which the trial court disbelieved, was that during this period, Summa Alta Tierra
find basis to allow waiver of the petitioner’s liability for accrued penalties.
Industries, Inc. shut down as a result of the general decline in the economy. The petitioner
pleaded good faith and lack of criminal intent as his defenses.
The petitioner’s liability for the crime is a settled matter
We ruled that the decree of conviction was founded on proof beyond reasonable doubt, based
on the following considerations: first, the remittance of employee contributions to the SSS is Upfront, we reject the petitioner’s claim that the prosecution failed to prove all the elements of
mandatory under RA No. 8282; and second, the failure to comply with a special law the crime charged. This is a matter that has been resolved in our Decision, and the petitioner did
being malum prohibitum, the defenses of good faith and lack of criminal intent are immaterial. not raise anything substantial to merit the reversal of our finding of guilt. To reiterate, the
petitioner’s conviction was based on his admission that he failed to remit his employees’
contribution to the SSS.
The petitioner further argued that since he was designated in the Information as a "proprietor,"
he was without criminal liability since "proprietors" are not among the corporate officers
specifically enumerated in Section 28(f) of RA No. 8282 to be criminally liable for the violation of The petitioner cannot benefit from the terms of RA No. 9903, which condone only employers
its provisions. We rejected this argument based on our ruling in Garcia v. Social Security who pay their delinquencies within six months from the law’s effectivity
Commission Legal and Collection.1 We ruled that to sustain the petitioner’s argument would be
to allow the unscrupulous to conveniently escape liability merely through the creative use of
managerial titles. We note that the petitioner does not ask for the reversal of his conviction based on the authority
of RA No. 9903; he avoids making a straightforward claim because this law plainly does not
apply to him or to others in the same situation. The clear intent of the law is to grant condonation
After taking into account the Indeterminate Penalty Law and Article 315 of the Revised Penal only to employers with delinquent contributions or pending cases for their delinquencies and who
Code, we MODIFIEDthe penalty originally imposed by the trial court2 and, instead, decreed the pay their delinquencies within the six (6)-month period set by the law. Mere payment of unpaid
penalty of four (4) years and two (2) months of prision correccional, as minimum, to twenty (20) contributions does not suffice; it is payment within, and only within, the six (6)-month availment
years of reclusion temporal, as maximum. period that triggers the applicability of RA No. 9903.

In the present motion for reconsideration, the petitioner points out that pending his appeal with True, the petitioner’s case was pending with us when RA No. 9903 was passed. Unfortunately
the Court of Appeals (CA), he voluntarily paid the SSS the amount of ₱239,756.80 to settle his for him, he paid his delinquent SSS contributions in 2007. By paying outside of the availment
delinquency.3 Note that the petitioner also gave notice of this payment to the CA via a Motion for period, the petitioner effectively placed himself outside the benevolent sphere of RA No. 9903.
Reconsideration and a Motion for New Trial. Although the People did not contest the fact of This is how the law is written: it condones employers — and only those employers — with
voluntary payment, the CA nevertheless denied the said motions. unpaid SSS contributions or with pending cases who pay within the six (6)-month period
following the law’s date of effectivity. Dura lex, sed lex.
The petitioner’s awareness that RA No. 9903 operates as discussed above is apparent in his Despite our discussion above, the petitioner’s move to have our Decision reconsidered is not
plea for equal protection. In his motion, he states that entirely futile. The one benefit the petitioner can obtain from RA No. 9903 is the waiver of his
accrued penalties, which remain unpaid in the amount of ₱181,394.29. This waiver is derived
from the last proviso of Section 4 of RA No. 9903:
[he] is entitled under the equal protection clause to the dismissal of the case against him since
he had already paid the subject delinquent contributions due to the SSS which accepted the
payment as borne by the official receipt it issued (please see Annex "A"). The equal protection Provided, further, That for reason of equity, employers who settled arrears in contributions
clause requires that similar subjects, [sic] should not be treated differently, so as to give undue before the effectivity of this Act shall likewise have their accrued penalties waived.
favor to some and unjustly discriminate against others. The petitioner is no more no less in the
same situation as the employer who would enjoy freedom from criminal prosecution upon
This proviso is applicable to the petitioner who settled his contributions long before the passage
payment in full of the delinquent contributions due and payable to the SSS within six months
of the law. Applied to the petitioner, therefore, RA No. 9903 only works to allow a waiver of his
from the effectivity of Republic Act No. 9903.6
accrued penalties, but not the reversal of his conviction.1avvphi1

The Court cannot amplify the scope of RA No. 9903 on the ground of equal protection, and
Referral to the Chief Executive for possible exercise of executive clemency
acquit the petitioner and other delinquent employers like him; it would in essence be an
amendment of RA No. 9903, an act of judicial legislation abjured by the trias politica principle.7
We realize that with the affirmation of the petitioner’s conviction for violation of RA No. 8282, he
stands to suffer imprisonment for four (4) years and two (2) months of prision correccional, as
RA No. 9903 creates two classifications of employers delinquent in remitting the SSS
minimum, to twenty (20) years of reclusion temporal, as maximum, notwithstanding the payment
contributions of their employees: (1) those delinquent employers who pay within the six (6)-
of his delinquent contribution.
month period (the former group), and (2) those delinquent employers who pay outside of this
availment period (the latter group). The creation of these two classes is obvious and unavoidable
when Section 2 and the last proviso of Section 48 of the law are read together. The same Under Article 5 of the Revised Penal Code,12 the courts are bound to apply the law as it is and
provisions show the law’s intent to limit the benefit of condonation to the former group only; had impose the proper penalty, no matter how harsh it might be. The same provision, however, gives
RA No. 9903 likewise intended to benefit the latter group, which includes the petitioner, it would the Court the discretion to recommend to the President actions it deems appropriate but are
have expressly declared so. Laws granting condonation constitute an act of benevolence on the beyond its power when it considers the penalty imposed as excessive. Although the petitioner
government’s part, similar to tax amnesty laws; their terms are strictly construed against the was convicted under a special penal law, the Court is not precluded from giving the Revised
applicants. Since the law itself excludes the class of employers to which the petitioner belongs, Penal Code suppletory application in light of Article 1013 of the same Code and our ruling in
no ground exists to justify his acquittal. An implementing rule or regulation must conform to and People v. Simon.14
be consistent with the provisions of the enabling statute; it cannot amend the law either by
abridging or expanding its scope.9
WHEREFORE, the Court PARTIALLY GRANTS petitioner Romarico J. Mendoza’s motion for
reconsideration. The Court AFFIRMS the petitioner’s conviction for violation of Section 22(a) and
For the same reason, we cannot grant the petitioner’s prayer to impose a fine in lieu of (d), in relation to Section 28 of Republic Act No. 8282, and the petitioner is thus sentenced to an
imprisonment; neither RA No. 8282 nor RA No. 9903 authorizes the Court to exercise this indeterminate prison term of four (4) years and two (2) months of prision correccional, as
option. minimum, to twenty (20) years of reclusion temporal, as maximum. In light of Section 4 of
Republic Act No. 9903, the petitioner’s liability for accrued penalties is considered WAIVED.
Considering the circumstances of the case, the Court transmits the case to the Chief Executive,
On the matter of equal protection, we stated in Tolentino v. Board of Accountancy, et al.10 that
through the Department of Justice, and RECOMMENDS the grant of executive clemency to the
the guarantee simply means "that no person or class of persons shall be denied the same
petitioner.
protection of the laws which is enjoyed by other persons or other classes in the same place and
in like circumstances." In People v. Cayat,11 we further summarized the jurisprudence on equal
protection in this wise: SO ORDERED.

It is an established principle of constitutional law that the guaranty of the equal protection of the
laws is not violated by a legislation based on reasonable classification. And the classification, to
be reasonable, (1) must rest on substantial distinctions; (2) must be germane to the purposes of
the law; (3) must not be limited to existing conditions only; and (4) must apply equally to all
members of the same class.

The difference in the dates of payment of delinquent contributions provides a substantial


distinction between the two classes of employers. In limiting the benefits of RA No. 9903 to
delinquent employers who pay within the six (6)-month period, the legislature refused to allow a
sweeping, non-discriminatory condonation to all delinquent employers, lest the policy behind RA
No. 8282 be undermined.1avvphi1

The petitioner is entitled to a waiver of his accrued penalties


G.R. No. 188550 August 19, 2013 be extended to a foreign corporation wishing to avail itself thereof, the latter should first invoke
the provisions of the tax treaty and prove that they indeed apply to the corporation.
DEUTSCHE BANK AG MANILA BRANCH, PETITIONER,
vs. THE CTA EN BANC RULING10
COMMISSIONER OF INTERNAL REVENUE, RESPONDENT.
The CTA En Banc affirmed the CTA Second Division’s Decision dated 29 August 2008 and
DECISION Resolution dated 14 January 2009. Citing Mirant, the CTA En Banc held that a ruling from the
ITAD of the BIR must be secured prior to the availment of a preferential tax rate under a tax
treaty. Applying the principle of stare decisis et non quieta movere, the CTA En Banc took into
SERENO, CJ.:
consideration that this Court had denied the Petition in G.R. No. 168531 filed by Mirant for failure
to sufficiently show any reversible error in the assailed judgment. 11 The CTA En Banc ruled that
This is a Petition for Review1 filed by Deutsche Bank AG Manila Branch (petitioner) under Rule once a case has been decided in one way, any other case involving exactly the same point at
45 of the 1997 Rules of Civil Procedure assailing the Court of Tax Appeals En Banc (CTA En issue should be decided in the same manner.
Banc) Decision2 dated 29 May 2009 and Resolution3 dated 1 July 2009 in C.T.A. EB No. 456.
The court likewise ruled that the 15-day rule for tax treaty relief application under RMO No. 1-
THE FACTS 2000 cannot be relaxed for petitioner, unlike in CBK Power Company Limited v. Commissioner
of Internal Revenue.12 In that case, the rule was relaxed and the claim for refund of excess final
withholding taxes was partially granted. While it issued a ruling to CBK Power Company Limited
In accordance with Section 28(A)(5)4 of the National Internal Revenue Code (NIRC) of 1997, after the payment of withholding taxes, the ITAD did not issue any ruling to petitioner even if it
petitioner withheld and remitted to respondent on 21 October 2003 the amount of PHP
filed a request for confirmation on 4 October 2005 that the remittance of branch profits to DB
67,688,553.51, which represented the fifteen percent (15%) branch profit remittance tax (BPRT) Germany is subject to a preferential tax rate of 10% pursuant to Article 10 of the RP-Germany
on its regular banking unit (RBU) net income remitted to Deutsche Bank Germany (DB Tax Treaty.
Germany) for 2002 and prior taxable years.5

ISSUE
Believing that it made an overpayment of the BPRT, petitioner filed with the BIR Large
Taxpayers Assessment and Investigation Division on 4 October 2005 an administrative claim for
refund or issuance of its tax credit certificate in the total amount of PHP 22,562,851.17. On the This Court is now confronted with the issue of whether the failure to strictly comply with RMO
same date, petitioner requested from the International Tax Affairs Division (ITAD) a confirmation No. 1-2000 will deprive persons or corporations of the benefit of a tax treaty.
of its entitlement to the preferential tax rate of 10% under the RP-Germany Tax Treaty.6
THE COURT’S RULING
Alleging the inaction of the BIR on its administrative claim, petitioner filed a Petition for
Review7 with the CTA on 18 October 2005. Petitioner reiterated its claim for the refund or
The Petition is meritorious.
issuance of its tax credit certificate for the amount of PHP 22,562,851.17 representing the
alleged excess BPRT paid on branch profits remittance to DB Germany.
Under Section 28(A)(5) of the NIRC, any profit remitted to its head office shall be subject to a tax
8 of 15% based on the total profits applied for or earmarked for remittance without any deduction
THE CTA SECOND DIVISION RULING
of the tax component. However, petitioner invokes paragraph 6, Article 10 of the RP-Germany
Tax Treaty, which provides that where a resident of the Federal Republic of Germany has a
After trial on the merits, the CTA Second Division found that petitioner indeed paid the total branch in the Republic of the Philippines, this branch may be subjected to the branch profits
amount of PHP 67,688,553.51 representing the 15% BPRT on its RBU profits amounting to PHP remittance tax withheld at source in accordance with Philippine law but shall not exceed 10% of
451,257,023.29 for 2002 and prior taxable years. Records also disclose that for the year 2003, the gross amount of the profits remitted by that branch to the head office.
petitioner remitted to DB Germany the amount of EURO 5,174,847.38 (or PHP 330,175,961.88
at the exchange rate of PHP 63.804:1 EURO), which is net of the 15% BPRT.
By virtue of the RP-Germany Tax Treaty, we are bound to extend to a branch in the Philippines,
remitting to its head office in Germany, the benefit of a preferential rate equivalent to 10% BPRT.
However, the claim of petitioner for a refund was denied on the ground that the application for a
tax treaty relief was not filed with ITAD prior to the payment by the former of its BPRT and actual
On the other hand, the BIR issued RMO No. 1-2000, which requires that any availment of the tax
remittance of its branch profits to DB Germany, or prior to its availment of the preferential rate of
treaty relief must be preceded by an application with ITAD at least 15 days before the
ten percent (10%) under the RP-Germany Tax Treaty provision. The court a quo held that
transaction. The Order was issued to streamline the processing of the application of tax treaty
petitioner violated the fifteen (15) day period mandated under Section III paragraph (2) of
relief in order to improve efficiency and service to the taxpayers. Further, it also aims to prevent
Revenue Memorandum Order (RMO) No. 1-2000.
the consequences of an erroneous interpretation and/or application of the treaty provisions (i.e.,
filing a claim for a tax refund/credit for the overpayment of taxes or for deficiency tax liabilities for
Further, the CTA Second Division relied on Mirant (Philippines) Operations Corporation (formerly underpayment).13
Southern Energy Asia-Pacific Operations [Phils.], Inc.) v. Commissioner of Internal
Revenue9 (Mirant) where the CTA En Banc ruled that before the benefits of the tax treaty may
The crux of the controversy lies in the implementation of RMO No. 1-2000.
Petitioner argues that, considering that it has met all the conditions under Article 10 of the RP- and treaties involved; more importantly, the disposition of that case was made only through a
Germany Tax Treaty, the CTA erred in denying its claim solely on the basis of RMO No. 1-2000. minute resolution.
The filing of a tax treaty relief application is not a condition precedent to the availment of a
preferential tax rate. Further, petitioner posits that, contrary to the ruling of the CTA, Mirant is not
Tax Treaty vs. RMO No. 1-2000
a binding judicial precedent to deny a claim for refund solely on the basis of noncompliance with
RMO No. 1-2000.
Our Constitution provides for adherence to the general principles of international law as part of
the law of the land.15The time-honored international principle of pacta sunt servanda demands
Respondent counters that the requirement of prior application under RMO No. 1-2000 is
the performance in good faith of treaty obligations on the part of the states that enter into the
mandatory in character. RMO No. 1-2000 was issued pursuant to the unquestioned authority of
agreement. Every treaty in force is binding upon the parties, and obligations under the treaty
the Secretary of Finance to promulgate rules and regulations for the effective implementation of
must be performed by them in good faith.16 More importantly, treaties have the force and effect
the NIRC. Thus, courts cannot ignore administrative issuances which partakes the nature of a
of law in this jurisdiction.17
statute and have in their favor a presumption of legality.

Tax treaties are entered into "to reconcile the national fiscal legislations of the contracting parties
The CTA ruled that prior application for a tax treaty relief is mandatory, and noncompliance with
and, in turn, help the taxpayer avoid simultaneous taxations in two different jurisdictions." 18 CIR
this prerequisite is fatal to the taxpayer’s availment of the preferential tax rate.
v. S.C. Johnson and Son, Inc. further clarifies that "tax conventions are drafted with a view
towards the elimination of international juridical double taxation, which is defined as the
We disagree. imposition of comparable taxes in two or more states on the same taxpayer in respect of the
same subject matter and for identical periods. The apparent rationale for doing away with double
taxation is to encourage the free flow of goods and services and the movement of capital,
A minute resolution is not a binding precedent
technology and persons between countries, conditions deemed vital in creating robust and
dynamic economies. Foreign investments will only thrive in a fairly predictable and reasonable
At the outset, this Court’s minute resolution on Mirant is not a binding precedent. The Court has international investment climate and the protection against double taxation is crucial in creating
clarified this matter in Philippine Health Care Providers, Inc. v. Commissioner of Internal such a climate."19
Revenue14 as follows:
Simply put, tax treaties are entered into to minimize, if not eliminate the harshness of
It is true that, although contained in a minute resolution, our dismissal of the petition was a international juridical double taxation, which is why they are also known as double tax treaty or
disposition of the merits of the case. When we dismissed the petition, we effectively affirmed the double tax agreements.
CA ruling being questioned. As a result, our ruling in that case has already become final. When
a minute resolution denies or dismisses a petition for failure to comply with formal and
"A state that has contracted valid international obligations is bound to make in its legislations
substantive requirements, the challenged decision, together with its findings of fact and legal
those modifications that may be necessary to ensure the fulfillment of the obligations
conclusions, are deemed sustained. But what is its effect on other cases?
undertaken."20 Thus, laws and issuances must ensure that the reliefs granted under tax treaties
are accorded to the parties entitled thereto. The BIR must not impose additional requirements
With respect to the same subject matter and the same issues concerning the same parties, it that would negate the availment of the reliefs provided for under international agreements. More
constitutes res judicata. However, if other parties or another subject matter (even with the same so, when the RP-Germany Tax Treaty does not provide for any pre-requisite for the availment of
parties and issues) is involved, the minute resolution is not binding precedent. Thus, in CIR v. the benefits under said agreement.
Baier-Nickel, the Court noted that a previous case, CIR v. Baier-Nickel involving the same
parties and the same issues, was previously disposed of by the Court thru a minute resolution
Likewise, it must be stressed that there is nothing in RMO No. 1-2000 which would indicate a
dated February 17, 2003 sustaining the ruling of the CA. Nonetheless, the Court ruled that the
deprivation of entitlement to a tax treaty relief for failure to comply with the 15-day period. We
previous case "ha(d) no bearing" on the latter case because the two cases involved different
recognize the clear intention of the BIR in implementing RMO No. 1-2000, but the CTA’s outright
subject matters as they were concerned with the taxable income of different taxable years.
denial of a tax treaty relief for failure to strictly comply with the prescribed period is not in
harmony with the objectives of the contracting state to ensure that the benefits granted under tax
Besides, there are substantial, not simply formal, distinctions between a minute resolution and a treaties are enjoyed by duly entitled persons or corporations.
decision. The constitutional requirement under the first paragraph of Section 14, Article VIII of
the Constitution that the facts and the law on which the judgment is based must be expressed
Bearing in mind the rationale of tax treaties, the period of application for the availment of tax
clearly and distinctly applies only to decisions, not to minute resolutions. A minute resolution is
treaty relief as required by RMO No. 1-2000 should not operate to divest entitlement to the relief
signed only by the clerk of court by authority of the justices, unlike a decision. It does not require
as it would constitute a violation of the duty required by good faith in complying with a tax treaty.
the certification of the Chief Justice. Moreover, unlike decisions, minute resolutions are not
The denial of the availment of tax relief for the failure of a taxpayer to apply within the prescribed
published in the Philippine Reports. Finally, the proviso of Section 4(3) of Article VIII speaks of a
period under the administrative issuance would impair the value of the tax treaty. At most, the
decision. Indeed, as a rule, this Court lays down doctrines or principles of law which constitute
application for a tax treaty relief from the BIR should merely operate to confirm the entitlement of
binding precedent in a decision duly signed by the members of the Court and certified by the
the taxpayer to the relief.
Chief Justice. (Emphasis supplied)

The obligation to comply with a tax treaty must take precedence over the objective of RMO No.
Even if we had affirmed the CTA in Mirant, the doctrine laid down in that Decision cannot bind
1-2000.1âwphi1 Logically, noncompliance with tax treaties has negative implications on
this Court in cases of a similar nature. There are differences in parties, taxes, taxable periods,
international relations, and unduly discourages foreign investors. While the consequences b. That on October 21, 2003, it filed its Monthly Remittance Return of Final Income
sought to be prevented by RMO No. 1-2000 involve an administrative procedure, these may be Taxes Withheld under BIR Form No. 1601-F and remitted the amount of
remedied through other system management processes, e.g., the imposition of a fine or penalty. ₱67,688,553.51 as branch profits remittance tax with the BIR; and
But we cannot totally deprive those who are entitled to the benefit of a treaty for failure to strictly
comply with an administrative issuance requiring prior application for tax treaty relief.
c. That on October 29, 2003, the Bangko Sentral ng Pilipinas having issued a
clearance, petitioner remitted to Frankfurt Head Office the amount of
Prior Application vs. Claim for Refund EUR5,174,847.38 (or ₱330,175,961.88 at 63.804 Peso/Euro) representing its 2002
profits remittance.22
Again, RMO No. 1-2000 was implemented to obviate any erroneous interpretation and/or
application of the treaty provisions. The objective of the BIR is to forestall assessments against The amount of PHP 67,688,553.51 paid by petitioner represented the 15% BPRT on its RBU net
corporations who erroneously availed themselves of the benefits of the tax treaty but are not income, due for remittance to DB Germany amounting to PHP 451,257,023.29 for 2002 and
legally entitled thereto, as well as to save such investors from the tedious process of claims for a prior taxable years.23
refund due to an inaccurate application of the tax treaty provisions. However, as earlier
discussed, noncompliance with the 15-day period for prior application should not operate to
Likewise, both the administrative and the judicial actions were filed within the two-year
automatically divest entitlement to the tax treaty relief especially in claims for refund.
prescriptive period pursuant to Section 229 of the NIRC.24

The underlying principle of prior application with the BIR becomes moot in refund cases, such as
Clearly, there is no reason to deprive petitioner of the benefit of a preferential tax rate of 10%
the present case, where the very basis of the claim is erroneous or there is excessive payment
BPRT in accordance with the RP-Germany Tax Treaty.
arising from non-availment of a tax treaty relief at the first instance. In this case, petitioner should
not be faulted for not complying with RMO No. 1-2000 prior to the transaction. It could not have
applied for a tax treaty relief within the period prescribed, or 15 days prior to the payment of its Petitioner is liable to pay only the amount of PHP 45,125,702.34 on its RBU net income
BPRT, precisely because it erroneously paid the BPRT not on the basis of the preferential tax amounting to PHP 451,257,023.29 for 2002 and prior taxable years, applying the 10% BPRT.
rate under Thus, it is proper to grant petitioner a refund ofthe difference between the PHP 67,688,553.51
(15% BPRT) and PHP 45,125,702.34 (10% BPRT) or a total of PHP 22,562,851.17.
the RP-Germany Tax Treaty, but on the regular rate as prescribed by the NIRC. Hence, the prior
application requirement becomes illogical. Therefore, the fact that petitioner invoked the WHEREFORE, premises considered, the instant Petition is GRANTED. Accordingly, the Court of
provisions of the RP-Germany Tax Treaty when it requested for a confirmation from the ITAD Tax Appeals En Banc Decision dated 29 May 2009 and Resolution dated 1 July 2009 are
before filing an administrative claim for a refund should be deemed substantial compliance with REVERSED and SET ASIDE. A new one is hereby entered ordering respondent Commissioner
RMO No. 1-2000. of Internal Revenue to refund or issue a tax credit certificate in favor of petitioner Deutsche Bank
AG Manila Branch the amount of TWENTY TWO MILLION FIVE HUNDRED SIXTY TWO
THOUSAND EIGHT HUNDRED FIFTY ONE PESOS AND SEVENTEEN CENTAVOS (PHP
Corollary thereto, Section 22921 of the NIRC provides the taxpayer a remedy for tax recovery
22,562,851.17), Philippine currency, representing the erroneously paid BPRT for 2002 and prior
when there has been an erroneous payment of tax.1âwphi1 The outright denial of petitioner’s
taxable years.
claim for a refund, on the sole ground of failure to apply for a tax treaty relief prior to the payment
of the BPRT, would defeat the purpose of Section 229.
SO ORDERED.
Petitioner is entitled to a refund

It is significant to emphasize that petitioner applied – though belatedly – for a tax treaty relief, in
substantial compliance with RMO No. 1-2000. A ruling by the BIR would have confirmed whether
petitioner was entitled to the lower rate of 10% BPRT pursuant to the RP-Germany Tax Treaty.

Nevertheless, even without the BIR ruling, the CTA Second Division found as follows:

Based on the evidence presented, both documentary and testimonial, petitioner was able to
establish the following facts:

a. That petitioner is a branch office in the Philippines of Deutsche Bank AG, a


corporation organized and existing under the laws of the Federal Republic of
Germany;
G.R. No. 221029 April 24, 2018 3. That recently, a case for divorce was filed by herein [petitioner] in Japan and after due
proceedings, a divorce decree dated December 6, 2011 was rendered by the Japanese Court x
REPUBLIC OF THE PHILIPPINES, Petitioner, x x;
v.
MARELYN TANEDO MANALO, Respondent.
4. That at present, by virtue of the said divorce decree, petitioner and her divorced Japanese
husband are no longer living together and in fact, petitioner and her daughter are living
DECISION
separately from said Japanese former husband;

PERALTA, J.:
5. That there is an imperative need to have the entry of marriage in the Civil Registry of San
Juan, Metro Manila cancelled, where the petitioner and the former Japanese husband's
This petition for review on certiorari under Rule 45 of the Rules of Court (Rules) seeks to reverse marriage was previously registered, in order that it would not appear anymore that petitioner is
and set aside the September 18, 2014 Decision1 and October 12, 2015 Resolution2 of the Court still married to the said Japanese national who is no longer her husband or is no longer married
of Appeals (CA) in CA-G.R. CV No. 100076. The dispositive portion of the Decision states: to her; furthermore, in the event that petitioner decides to be remarried, she shall not be
bothered and disturbed by said entry of marriage;
WHEREFORE, the instant appeal is GRANTED. The Decision dated 15 October 2012 of the
Regional Trial Court of Dagupan City, First Judicial Region, Branch 43, in SPEC. PROC. NO. 6. That this petition is filed principally for the purpose of causing the cancellation of entry of the
2012-0005 is REVERSED and SET ASIDE. marriage between the petitioner and the said Japanese national, pursuant to Rule 108 of the
Revised Rules of Court, which marriage was already dissolved by virtue of the aforesaid divorce
decree; [and]
Let a copy of this Decision be served on the Local Civil Registrar of San Juan, Metro Manila.

7. That petitioner prays, among others, that together with the cancellation of the said entry of her
SO ORDERED.3
marriage, that she be allowed to return and use. her maiden surname, MANALO.4

The facts are undisputed.


Manalo was allowed to testify in advance as she was scheduled to leave for Japan for her
employment. Among the documents that were offered and admitted were:
On January 10, 2012, respondent Marelyn Tanedo Manalo (Manalo) filed a petition for
cancellation of entry of marriage in the Civil Registry of San Juan, Metro Manila, by virtue of a
1. Court Order dated January 25, 2012, finding the petition and its attachments to be sufficient in
judgment of divorce rendered by a Japanese court.
form and in substance;

Finding the petition to be sufficient in form and in substance, Branch 43 of the Regional Trial
2. Affidavit of Publication;
Court (RTC) of Dagupan City set the case for initial hearing on April 25, 2012. The petition and
the notice of initial hearing were published once a week for three consecutive weeks in a
newspaper of general circulation. During the initial hearing, counsel for Manalo marked the 3. Issues of the Northern Journal dated February 21-27, 2012, February 28 - March 5, 2012, and
documentary evidence (consisting of the trial court's Order dated January 25, 2012, affidavit of March 6-12, 2012;
publication, and issues of the Northern Journal dated February 21-27, 2012, February 28 -
March 5, 2012, and March 6-12, 2012) for purposes of compliance with the jurisdictional
4. Certificate of Marriage between Manalo and her former Japanese husband;
requirements.

5. Divorce Decree of the Japanese court;


The Office of the Solicitor General (OSG) entered its appearance for petitioner Republic of the
Philippines authorizing the Office of the City Prosecutor of Dagupan to appear on its behalf.
Likewise, a Manifestation and Motion was filed questioning the title and/or caption of the petition 6. Authentication/Certificate issued by the Philippine Consulate General in Osaka, Japan of the
considering that, based on the allegations therein, the proper action should be a petition for Notification of Divorce; and
recognition and enforcement of a foreign judgment.
7. Acceptance of Certificate of Divorce.5
As a result, Manalo moved to admit an Amended Petition, which the court granted. The
Amended Petition, which captioned that it is also a petition for recognition and enforcement of
The OSG did not present any controverting evidence to rebut the allegations of Manalo.
foreign judgment, alleged:

2. That petitioner is previously married in the Philippines to a Japanese national named On October 15, 2012, the trial court denied the petition for lack of merit. In ruling that the divorce
obtained by Manalo in Japan should not be recognized, it opined that, based on Article 15 of the
YOSHINO MINORO as shown by their Marriage Contract x x x;
New Civil Code, the Philippine law "does not afford Filipinos the right to file for a divorce,
whether they are in the country or living abroad, if they are married to Filipinos or to foreigners,
or if they celebrated their marriage in the Philippines or in another country" and that unless
Filipinos "are naturalized as citizens of another country, Philippine laws shall have control over Paragraph 2 of Article 26 confers jurisdiction on Philippine courts to extend the effect of a foreign
issues related to Filipinos' family rights and duties, together with the determination of their divorce decree to a Filipino spouse without undergoing trial to determine the validity of the
condition and legal capacity to enter into contracts and civil relations, including marriages."6 dissolution of the marriage.20 It authorizes our courts to adopt the effects of a foreign divorce
decree precisely because the Philippines does not allow divorce. 21 Philippine courts cannot try
the case on the merits because it is tantamount to trying a divorce case. 22 Under the principles
On appeal, the CA overturned the RTC decision. It held that Article 26 of the Family Code of the
of comity, our jurisdiction recognizes a valid divorce obtained by a spouse of foreign nationality,
Philippines (Family Code) is applicable even if it was Manalo who filed for divorce against her
but the legal effects thereof, e.g., on custody, care and support of the children or property
Japanese husband because the decree they obtained makes the latter no longer married to the
relations of the spouses, must still be determined by our courts. 23
former, capacitating him to remarry. Conformably with Navarro, et al. v. Exec. Secretary Ermita,
et al.7 ruling that the meaning of the law should be based on the intent of the lawmakers and in
view of the legislative intent behind Article 26, it would be the height of injustice to consider According to Judge Alicia Sempio-Diy, a member of the Committee, the idea of the amendment
Manalo as still married to the Japanese national, who, in turn, is no longer married to her. For is to avoid the absurd situation of a Filipino as still being married to his or her alien spouse,
the appellate court, the fact that it was Manalo who filed the divorce case is inconsequential. although the latter is no longer married to the former because he or she had obtained a divorce
Cited as similar to this case was Van Dorn v. Judge Romillo, Jr.8 where the marriage between a abroad that is recognized by his or her national law.24 The aim was that it would solve the
foreigner and a Filipino was dissolved through a divorce filed abroad by the latter. problem of many Filipino women who, under the New Civil Code, are still considered married to
their alien husbands even after the latter have already validly divorced them under their (the
husbands') national laws and perhaps have already married again.25
The OSG filed a motion for reconsideration, but it was denied; hence, this petition.

In 2005, this Court concluded that Paragraph 2 of Article 26 applies to a case where, at the time
We deny the petition and partially affirm the CA decision.
of the celebration of the marriage, the parties were Filipino citizens, but later on, one of them
acquired foreign citizenship by naturalization, initiated a divorce proceeding, and obtained a
Divorce, the legal dissolution of a lawful union for a cause arising after marriage, are of two favorable decree. We held in Republic of the Phils. v. Orbecido III:26
types: (1) absolute divorce or a vinculo matrimonii, which terminates the marriage, and (2)
limited divorce or a mensa et thoro, which suspends it and leaves the bond in full force.9 In this
The jurisprudential answer lies latent in the 1998 case of Quita v. Court of Appeals. In Quita, the
jurisdiction, the following rules exist:
parties were, as in this case, Filipino citizens when they got married. The wife became a
naturalized American citizen in 1954 and obtained a divorce in the same year. The Court therein
1. Philippine law does not provide for absolute divorce; hence, our courts cannot grant it.10 hinted, by way of obiter dictum, that a Filipino divorced by his naturalized foreign spouse is no
longer married under Philippine law and can thus remarry.
2. Consistent with Articles 1511 and 1712 of the New Civil Code, the marital bond between two
Filipinos cannot be dissolved even by an absolute divorce obtained abroad. 13 Thus, taking into consideration the legislative intent and applying the rule of reason, we hold that
Paragraph 2 of Article 26 should be interpreted to include cases involving parties who, at the
time of the celebration of the marriage were Filipino citizens, but later on, one of them becomes
3. An absolute divorce obtained abroad by a couple, who are both aliens, may be recognized in naturalized as a foreign citizen and obtains a divorce decree. The Filipino spouse should
the Philippines, provided it is consistent with their respective national laws. 14 likewise be allowed to remarry as if the other party were a foreigner at the time of the
solemnization of the marriage. To rule otherwise would be to sanction absurdity and injustice. x x
4. In mixed marriages involving a Filipino and a foreigner, the former is allowed to contract a x
subsequent marriage in case the absolute divorce is validly obtained abroad by the alien spouse
capacitating him or her to remarry.15
If we are to give meaning to the legislative intent to avoid the absurd situation where the Filipino
spouse remains married to the alien spouse who, after obtaining a divorce is no longer married
On July 6, 1987, then President Corazon C. Aquino signed into law Executive Order (E.O.) No. to the Filipino spouse, then the instant case must be deemed as coming within the
209, otherwise known as The Family Code of the Philippines, which took effect on August 3, contemplation of Paragraph 2 of Article 26.
1988.16 Shortly thereafter, E.O. No. 227 was issued on July 17, 1987.17 Aside from amending
Articles 36 and 39 of the Family Code, a second paragraph was added to Article 26. 18 This In view of the foregoing, we state the twin elements for the application of Paragraph 2 of Article
provision was originally deleted by the Civil Code Revision Committee (Committee), but it was
26 as follows:
presented and approved at a Cabinet meeting after Pres. Aquino signed E.O. No. 209. 19 As
modified, Article 26 now states:
1. There is a valid marriage that has been celebrated between a Filipino citizen
and a foreigner; and
Art. 26. All marriages solemnized outside the Philippines, in accordance with the laws in force in
the country where they were solemnized, and valid there as such, shall also be valid in this
country, except those prohibited under Articles 35(1), (4), (5) and (6), 36, 37 and 38. 2. A valid divorce is obtained abroad by the alien spouse capacitating him or
her to remarry.

Where a marriage between a Filipino citizen and a foreigner is validly celebrated and a divorce is
thereafter validly obtained abroad by the alien spouse capacitating him or her to remarry, the The reckoning point is not the citizenship of the parties at the time of the celebration of the
Filipino spouse shall likewise have capacity to remarry under Philippine law. marriage, but their citizenship at the time a valid divorce is obtained abroad by the alien spouse
capacitating the latter to remarry.27
Now, the Court is tasked to resolve whether, under the same provision, a Filipino citizen has the considered contrary to our concept of public policy and morality. However, aliens may obtain
capacity to remarry under Philippine law after initiating a divorce proceeding abroad and divorces abroad, which may be recognized in the Philippines, provided they are valid according
obtaining a favorable judgment against his or her alien spouse who is capacitated to remarry. to their national law. In this case, the divorce in Nevada released private respondent from the
Specifically, Manalo pleads for the recognition and enforcement of the divorce decree rendered marriage from the standards of American law, under which divorce dissolves the marriage. As
by the Japanese court and for the cancellation of the entry of marriage in the local civil registry stated by the Federal Supreme Court of the United States in Atherton vs. Atherton, 45 L. Ed.
"in order that it would not appear anymore that [she] is still married to the said Japanese national 794, 799:
who is no longer her husband or is no longer married to her; [and], in the event that [she]
decides to be remarried, she shall not be bothered and disturbed by said entry of marriage," and
"The purpose and effect of a decree of divorce from the bond of matrimony by a court of
to return and to use her maiden surname.
competent jurisdiction are to change the existing status or domestic relation of husband and
wife, and to free them both from the bond. The marriage tie, when thus severed as to one party,
We rule in the affirmative. ceases to bind either. A husband without a wife, or a wife without a husband, is unknown to the
law. When the law provides, in the nature of a penalty, that the guilty party shall not marry again,
that party, as well as the other, is still absolutely freed from the bond of the former marriage."
Both Dacasin v. Dacasin28 and Van Dorn29 already recognized a foreign divorce decree that was
initiated and obtained by the Filipino spouse and extended its legal effects on the issues of child
custody and property relation, respectively. Thus, pursuant to his national law, private respondent is no longer the husband of petitioner. He
would have no standing to sue in the case below as petitioner's husband entitled to exercise
control over conjugal assets. As he is bound by the Decision of his own country's Court, which
In Dacasin, post-divorce, the former spouses executed an Agreement for the joint custody of
validly exercised jurisdiction over him, and whose decision he does not repudiate, he is
their minor daughter. Later on, the husband, who is a US citizen, sued his Filipino wife to enforce
estopped by his own representation before said Court from asserting his right over the alleged
the Agreement, alleging that it was only, the latter who exercised sole custody of their child. The
conjugal property.
trial court dismissed the action for lack of jurisdiction, on the ground, among others, that the
divorce decree is binding following the "nationality rule" prevailing in this jurisdiction. The
husband moved to reconsider, arguing that the divorce decree obtained by his former wife is To maintain, as private respondent does, that, under our laws, petitioner has to be considered
void, but it was denied. In ruling that the trial court has jurisdiction to entertain the suit but not to still married to private respondent and still subject to a wife's obligations under Article 109, et.
enforce the Agreement, which is void, this Court said: seq. of the Civil Code cannot be just. Petitioner should not be obliged to live together with,
observe respect and fidelity, and render support to private respondent. The latter should not
continue to be one of her heirs with possible rights to conjugal property. She should not be
Nor can petitioner rely on the divorce decree's alleged invalidity - not because the Illinois court
discriminated against in her own country if the ends of justice are to be served. 31
lacked jurisdiction or that the divorce decree violated Illinois law, but because the divorce was
obtained by his Filipino spouse - to support the Agreement's enforceability. The argument that
foreigners in this jurisdiction are not bound by foreign divorce decrees is hardly novel. Van Dorn In addition, the fact that a validly obtained foreign divorce initiated by the Filipino spouse can be
v. Romillo settled the matter by holding that an alien spouse of a Filipino is bound by a divorce recognized and given legal effects in the Philippines is implied from Our rulings in Fujiki v.
decree obtained abroad. There, we dismissed the alien divorcee's Philippine suit for accounting Marinay, et al.32 and Medina v. Koike.33
of alleged post-divorce conjugal property and rejected his submission that the foreign divorce
(obtained by the Filipino spouse) is not valid in this jurisdiction x x x.30
In Fujiki, the Filipino wife, with the help of her first husband, who is a Japanese national, was
able to obtain a judgment from Japan's family court, which declared the marriage between her
Van Dorn was decided before the Family Code took into effect. There, a complaint was filed by and her second husband, who is a Japanese national, void on the ground of bigamy. In
the ex-husband, who is a US citizen, against his Filipino wife to render an accounting of a resolving the issue of whether a husband or wife of a prior marriage can file a petition to
business that was alleged to be a conjugal property and to be declared with right to manage the recognize a foreign judgment nullifying the subsequent marriage between his or her spouse and
same. Van Dorn moved to dismiss the case on the ground that the cause of action was barred a foreign citizen on the ground of bigamy, We ruled:
by previous judgment in the divorce proceedings that she initiated, but the trial court denied the
motion. On his part, her ex-husband averred that the divorce decree issued by the Nevada court
Fujiki has the personality to file a petition to recognize the Japanese Family Court judgment
could not prevail over the prohibitive laws of the Philippines and its declared national policy; that
nullifying the marriage between Marinay and Maekara on the ground of bigamy because the
the acts and declaration of a foreign court cannot, especially if the same is contrary to public
judgment concerns his civil status as married to Marinay. For the same reason he has the
policy, divest Philippine courts of jurisdiction to entertain matters within its jurisdiction. In
personality to file a petition under Rule 108 to cancel the entry of marriage between Marinay and
dismissing the case filed by the alien spouse, the Court discussed the effect of the foreign
Maekara in the civil registry on the basis of the decree of the Japanese Family Court.
divorce on the parties and their conjugal property in the Philippines. Thus:

There is no doubt that the prior spouse has a personal and material interest in maintaining the
There can be no question as to the validity of that Nevada divorce in any of the States of the
integrity of the marriage he contracted and the property relations arising from it. There is also no
United States. The decree is binding on private respondent as an American citizen. For instance,
doubt that he is interested in the cancellation of an entry of a bigamous marriage in the civil
private respondent cannot sue petitioner, as her husband, in any State of the Union. What he is
registry, which compromises the public record of his marriage. The interest derives from the
contending in this case is that the divorce is not valid and binding in this jurisdiction, the same
substantive right of the spouse not only to preserve (or dissolve, in limited instances) his most
being contrary to local law and public policy.
intimate human relation, but also to protect his property interests that arise by operation of law
the moment he contracts marriage. These property interests in marriage include the right to be
It is true that owing to the nationality principle embodied in Article 15 of the Civil Code, only supported "in keeping with the financial capacity of the family" and preserving the property
Philippine nationals are covered by the policy against absolute divorces the same being regime of the marriage.
Property rights are already substantive rights protected by the Constitution, but a spouse's right aberration, and bearing in mind the principle that the intent or the spirit of the law is the law itself,
in a marriage extends further to relational rights recognized under Title III ("Rights and resort should be to the rule that the spirit of the law controls its letter.
Obligations between Husband and Wife") of the Family Code. x x x34
To reiterate, the purpose of Paragraph 2 of Article 26 is to avoid the absurd situation where the
On the other hand, in Medina, the Filipino wife and her Japanese husband jointly filed for Filipino spouse remains married to the alien spouse who, after a foreign divorce decree that is
divorce, which was granted. Subsequently, she filed a petition before the RTC for judicial effective in the country where it was rendered, is no longer married to the Filipino spouse. The
recognition of foreign divorce and declaration of capacity to remarry pursuant to Paragraph 2 of provision is a corrective measure to address an anomaly where the Filipino spouse is tied to the
Article 26. The RTC denied the petition on the ground that the foreign divorce decree and the marriage while the foreign spouse is free to marry under the laws of his or her
national law of the alien spouse recognizing his capacity to obtain a divorce decree must be country.42 Whether the Filipino spouse initiated the foreign divorce proceeding or not, a favorable
proven in accordance with Sections 24 and 25 of Rule 132 of the Revised Rules on Evidence. decree dissolving the marriage bond and capacitating his or her alien spouse to remarry will
This Court agreed and ruled that, consistent with Corpuz v. Sto. Tomas, et al.35 and Garcia v. have the same result: the Filipino spouse will effectively be without a husband or wife. A Filipino
Recio,36 the divorce decree and the national law of the alien spouse must be proven. Instead of who initiated a foreign divorce proceeding is in the same place and in "like circumstance as a
dismissing the case, We referred it to the CA for appropriate action including the reception of Filipino who is at the receiving end of an alien initiated proceeding. Therefore, the subject
evidence to determine and resolve the pertinent factual issues. provision should not make a distinction. In both instance, it is extended as a means to recognize
the residual effect of the foreign divorce decree on Filipinos whose marital ties to their alien
spouses are severed by operation of the latter's national law.
There is no compelling reason to deviate from the above-mentioned rulings. When this Court
recognized a foreign divorce decree that was initiated and obtained by the Filipino spouse and
extended its legal effects on the issues of child custody and property relation, it should not stop Conveniently invoking the nationality principle is erroneous. Such principle, found under Article
short in likewise acknowledging that one of the usual and necessary consequences of absolute 15 of the Civil Code, is not an absolute and unbending rule. In fact, the mere existence of
divorce is the right to remarry. Indeed, there is no longer a mutual obligation to live together and Paragraph 2 of Article 26 is a testament that the State may provide for an exception thereto.
observe fidelity. When the marriage tie is severed and ceased to exist, the civil status and the Moreover, blind adherence to the nationality principle must be disallowed if it would cause unjust
domestic relation of the former spouses change as both of them are freed from the marital bond. discrimination and oppression to certain classes of individuals whose rights are equally
protected by law. The courts have the duty to enforce the laws of divorce as written by the
Legislature only if they are constitutional.43
The dissent is of the view that, under the nationality principle, Manalo's personal status is
subject to Philippine law, which prohibits absolute divorce. Hence, the divorce decree which she
obtained under Japanese law cannot be given effect, as she is, without dispute, a national not of While the Congress is allowed a wide leeway in providing for a valid classification and that its
Japan, but of the Philippines. It is said that a contrary ruling will subvert not only the intention of decision is accorded recognition and respect by the courts of justice, such classification may be
the framers of the law, but also that of the Filipino people, as expressed in the Constitution. The subjected to judicial review.44 The deference stops where the classification violates a
Court is, therefore, bound to respect the prohibition until the legislature deems it fit to lift the fundamental right, or prejudices persons accorded special protection by the Constitution. 45 When
same. these violations arise, this Court must discharge its primary role as the vanguard of constitutional
guaranties, and require a stricter and more exacting adherence to constitutional limitations. 46 If a
legislative classification impermissibly interferes with the exercise of a fundamental right or
We beg to differ.
operates to the peculiar disadvantage of a suspect class strict judicial scrutiny is required since
it is presumed unconstitutional, and the burden is upon the government to prove that the
Paragraph 2 of Article 26 speaks of "a divorce x x x validly obtained abroad by the alien spouse classification is necessary to achieve a compelling state interest and that it is the least restrictive
capacitating him or her to remarry. " Based on a clear and plain reading of the provision, it only means to protect such interest.47
requires that there be a divorce validly obtained abroad. The letter of the law does not demand
that the alien spouse should be the one who initiated the proceeding wherein the divorce decree
"Fundamental rights" whose infringement leads to strict scrutiny under the equal protection
was granted. It does not distinguish whether the Filipino spouse is the petitioner or the
clause are those basic liberties explicitly or implicitly guaranteed in the Constitution. 48 It includes
respondent in the foreign divorce proceeding. The Court is bound by the words of the statute;
the right of procreation, the right to marry, the right to exercise free speech, political
neither can We put words in the mouths of the lawmakers.37 "The legislature is presumed to
expression, press, assembly, and so forth, the right to travel, and the right to vote. 49 On the other
know the meaning of the words, to have used words advisedly, and to have expressed its intent
hand, what constitutes compelling state interest is measured by the scale of rights and powers
by the use of such words as are found in the statute. Verba legis non est recedendum, or from
arrayed in the Constitution and calibrated by history.50 It is akin to the paramount interest of the
the words of a statute there should be no departure."38
state for which some individual liberties must give way, such as the promotion of public interest,
public safety or the general welfare.51 It essentially involves a public right or interest that,
Assuming, for the sake of argument, that the word "obtained" should be interpreted to mean that because of its primacy, overrides individual rights, and allows the former to take precedence
the divorce proceeding must be actually initiated by the alien spouse, still, the Court will not over the latter.52
follow the letter of the statute when to do so would depart from the true intent of the legislature or
would otherwise yield conclusions inconsistent with the general purpose of the act. 39 Laws have
Although the Family Code was not enacted by the Congress, the same principle applies with
ends to achieve, and statutes should be so construed as not to defeat but to carry out such ends
respect to the acts of the President, which have the force and effect of law unless declared
and purposes.40 As held in League of Cities of the Phils., et al. v. COMELEC, et al.:41
otherwise by the court. In this case, We find that Paragraph 2 of Article 26 violates one of the
essential requisites53 of the equal protection clause.54 Particularly, the limitation of the provision
The legislative intent is not at all times accurately reflected in the manner in which the resulting only to a foreign divorce decree initiated by the alien spouse is unreasonable as it is based on
law is couched. Thus, applying a verba legis or strictly literal interpretation of a statute may superficial, arbitrary, and whimsical classification.
render it meaningless and lead to inconvenience, an absurd situation or injustice. To obviate this
A Filipino who is married to another Filipino is not similarly situated with a Filipino who is married victims or at the losing end of mixed marriages. And Fourth, it is not for Us to prejudge the
to a foreign citizen. There are real, material and substantial differences between them. Ergo, motive behind a Filipino's decision to marry an alien national. In one case, it was said:
they should not be treated alike, both as to rights conferred and liabilities imposed. Without a
doubt, there are political, economic, cultural, and religious dissimilarities as well as varying legal
Motives for entering into a marriage are varied and complex. The State does not and cannot
systems and procedures, all too unfamiliar, that a Filipino national who is married to an alien
dictate on the kind of life that a couple chooses to lead. Any attempt to regulate their lifestyle
spouse has to contend with. More importantly, while a divorce decree obtained abroad by a
would go into the realm of their right to privacy and would raise serious constitutional questions.
Filipino against another Filipino is null and void, a divorce decree obtained by an alien against
The right to marital privacy allows married couples to structure their marriages in almost any way
his or her Filipino spouse is recognized if made in accordance with the national law of the
they see fit, to live together or live apart, to have children or no children, to love one another or
foreigner.55
not, and so on. Thus, marriages entered into for other purposes, limited or otherwise, such as
convenience, companionship, money, status, and title, provided that they comply with all the
On the contrary, there is no real and substantial difference between a Filipino who initiated a legal requisites, are equally valid. Love, though the ideal consideration in a marriage contract, is
foreign divorce proceedings and a Filipino who obtained a divorce decree upon the instance of not the only valid cause for marriage. Other considerations, not precluded by law, may validly
his or her alien spouse. In the eyes of the Philippine and foreign laws, both are considered as support a marriage.63
Filipinos who have the same rights and obligations in a alien land. The circumstances
surrounding them are alike. Were it not for Paragraph 2 of Article 26, both are still married to
The 1987 Constitution expresses that marriage, as an inviolable social institution, is the
their foreigner spouses who are no longer their wives/husbands. Hence, to make a distinction
foundation of the family and shall be protected by the State.64 Nevertheless, it was not meant to
between them based merely on the superficial difference of whether they initiated the divorce
be a general prohibition on divorce because Commissioner Jose Luis Martin C. Gascon, in
proceedings or not is utterly unfair. Indeed, the treatment gives undue favor to one and unjustly
response to a question by Father Joaquin G. Bernas during the deliberations of the 1986
discriminate against the other.
Constitutional Commission, was categorical about this point.65 Their exchange reveal as follows:

Further, the differentiation in Paragraph 2 of Article 26 is arbitrary. There is inequality in


MR. RAMA. Mr. Presiding Officer, may I ask that Commissioner Bernas be recognized.
treatment because a foreign divorce decree that was initiated and obtained by a Filipino citizen
against his or her alien spouse would not be recognized even if based on grounds similar to
Articles 35, 36, 37 and 38 of the Family Code.56 In filing for divorce based on these grounds, the THE PRESIDING OFFICER (Mr. Colayco). Commissioner Bernas is recognized.
Filipino spouse cannot be accused of invoking foreign law at whim, tantamount to insisting that
he or she should be governed with whatever law he or she chooses. The dissent's comment that
Manalo should be "reminded that all is not lost, for she may still pray for the severance of her FR. BERNAS. Just one question, and I am not sure if it has been categorically answered. I refer
marital ties before the RTC in accordance with the mechanisms now existing under the Family specifically to the proposal of Commissioner Gascon. Is this to be understood as a prohibition of
Code" is anything but comforting. For the guidance of the bench and the bar, it would have been a general law on divorce? His intention is to make this a prohibition so that the legislature cannot
better if the dissent discussed in detail what these "mechanisms" are and how they specifically pass a divorce law.
apply in Manalo's case as well as those who are similarly situated. If the dissent refers to a
petition for declaration of nullity or annulment of marriage, the reality is that there is no MR. GASCON. Mr. Presiding Officer, that was not primarily my intention. My intention was
assurance that our courts will automatically grant the same. Besides, such proceeding is primarily to encourage the social institution of marriage, but not necessarily discourage divorce.
duplicitous, costly, and protracted. All to the prejudice of our kababayan. But now that he mentioned the issue of divorce, my personal opinion is to discourage it, Mr.
Presiding Officer.
It is argued that the Court's liberal interpretation of Paragraph 2 of Article 26 encourages
Filipinos to marry foreigners, opening the floodgate to the indiscriminate practice of Filipinos FR. BERNAS. No. my question is more categorical. Does this carry the meaning of prohibiting a
marrying foreign nationals or initiating divorce proceedings against their alien spouses. divorce law?

The supposition is speculative and unfounded. MR. GASCON. No. Mr. Presiding Officer.

First, the dissent falls into a hasty generalization as no data whatsoever was shown to support FR. BERNAS. Thank you.66
what he intends to prove. Second, We adhere to the presumption of good faith in this
jurisdiction. Under the rules on evidence, it is disputably presumed (i.e., satisfactory if
uncontradicted and overcome by other evidence) that a person is innocent of crime or Notably, a law on absolute divorce is not new in our country. Effective March 11, 1917,
wrong,57 that a person intends the ordinary consequences of his voluntary acts, 58 that a person Philippine courts could grant an absolute divorce on the grounds of adultery on the part of the
takes ordinary care of his concerns,59 that acquiescence resulted from a belief that the thing wife or concubinage on the part of the husband by virtue of Act No. 2710 of the Philippine
acquiesced in was conformable to the law and fact,60 that a man and woman deporting Legislature.67 On March 25, 1943, pursuant to the authority conferred upon him by the
themselves as husband and wife have entered into a lawful contract of marriage, 61 and that the Commander-in-Chief of the Imperial Japanese Forces in the Philippines and with the approval of
law has been obeyed.62 It is whimsical to easily attribute any illegal, irregular or immoral conduct the latter, the Chairman of the Philippine Executive Commission promulgated an E.O. No. 141
on the part of a Filipino just because he or she opted to marry a foreigner instead of a fellow ("New Divorce Law"), which repealed Act No. 2710 and provided eleven grounds for absolute
Filipino. It is presumed that interracial unions are entered into out of genuine love and affection, divorce, such as intentional or unjustified desertion continuously for at least one year prior to the
rather than prompted by pure lust or profit. Third, We take judicial notice of the fact that Filipinos filing of the action, slander by deed or gross insult by one spouse against the other to such an
are relatively more forbearing and conservative in nature and that they are more often the extent as to make further living together impracticable, and a spouse's incurable
insanity.68 When the Philippines was liberated and the Commonwealth Government was
restored, it ceased to have force and effect and Act No. 2710 again prevailed. 69 From August 30, d. The consent of either party was obtained by force, intimidation or undue
1950, upon the effectivity of Republic Act No. 386 or the New Civil Code, an absolute divorce influence, unless the same having disappeared or ceased, such party
obtained by Filipino citizens, whether here or abroad, is no longer recognized. 70 thereafter freely cohabited with the other as husband and wife;
e. Either party was physically incapable of consummating the marriage with
the other and such incapacity continues or appears to be incurable; and
Through the years, there has been constant clamor from various sectors of the Philippine society
f. Either party was afflicted with a sexually transmissible infection found to be
to re-institute absolute divorce. As a matter of fact, in the current 17th Congress, House Bill
serious or appears to be incurable.
(H.B.) Nos. 116,711062,72 238073 and 602774 were filed in the House of Representatives. In
substitution of these bills, H.B. No. 7303 entitled "An Act Instituting Absolute Divorce and
Dissolution of Marriage in the Philippines" or the Absolute Divorce Act of 2018 was submitted by Provided, That the grounds mentioned in b, e and f existed either at the time of the marriage or
the House Committee on Population and Family Relations on February 28, 2018. It was supervening after the marriage.
approved on March 19, 2018 on Third Reading - with 134 in favor, 57 against, and 2
abstentions. Under the bill, the grounds for a judicial decree of absolute divorce are as follows:
1. When the spouses have been separated in fact for at least five (5) years at
the time the petition for absolute divorce is filed, and reconciliation is highly
1. The grounds for legal separation under Article 55 of the Family Code, modified or improbable;
amended, as follows: 2. Psychological incapacity of either spouse as provided for in Article 36 of the
Family Code, whether or not the incapacity was present at the time of the
celebration of the marriage or later;
a. Physical violence or grossly abusive conduct directed against the petitioner, a common child,
3. When one of the spouses undergoes a gender reassignment surgery or
or a child of the petitioner;
transitions from one sex to another, the other spouse is entitled to petition
b. Physical violence or moral pressure to compel the petitioner to change religious or political
for absolute divorce with the transgender or transsexual as respondent, or
affiliation;
vice-versa;
c. Attempt of respondent to corrupt or induce the petitioner, a common child, or a child of the
4. Irreconcilable marital differences and conflicts which have resulted in the
petitioner, to engage in prostitution, or connivance in such corruption or inducement;
total breakdown of the marriage beyond repair, despite earnest and
d. Final judgment sentencing the respondent to imprisonment of more than six (6) years, even if
repeated efforts at reconciliation.
pardoned;
e. Drug addiction or habitual alcoholism or chronic gambling of the respondent;
f. Homosexuality of the respondent; To be sure, a good number of the Filipinos led by the Roman Catholic Church react adversely to
g. Contracting by the respondent of a subsequent bigamous marriage, whether in the Philippines any attempt to enact a law on absolute divorce, viewing it as contrary to our customs, morals,
or abroad; and traditions that has looked upon marriage and family as an institution and their nature of
h. Marital infidelity or perversion or having a child with another person other than one's spouse permanence, inviolability, and solidarity. However, none of our laws should be based on any
during the marriage, except when upon the mutual agreement of the spouses, a child is born to religious law, doctrine, or teaching; otherwise, the separation of Church and State will be
them by in vitro or a similar procedure or when the wife bears a child after being a victim of violated.75
rape;
i. Attempt by the respondent against the life of the petitioner, a common child or a child of the
In the same breath that the establishment clause restricts what the government can do with
petitioner; and
religion, it also limits what religious sects can or cannot do. They can neither cause the
j. Abandonment of petitioner by respondent without justifiable cause for more than one (1) year.
government to adopt their particular doctrines as policy for everyone, nor can they cause the
government to restrict other groups. To do so, in simple terms, would cause the State to adhere
When the spouses are legally separated by judicial decree for more than two (2) years, either or to a particular religion and, thus, establish a state religion.76
both spouses can petition the proper court for an absolute divorce based on said judicial decree
of legal separation.
The Roman Catholic Church can neither impose its beliefs and convictions on the State and the
rest of the citizenry nor can it demand that the nation follow its beliefs, even if it sincerely
1. Grounds for annulment of marriage under Article 45 of the Family Code, restated as follows: believes that they are good for the country.77 While marriage is considered a sacrament, it has
civil and legal consequences which are governed by the Family Code. 78 It is in this aspect, bereft
of any ecclesiastical overtone, that the State has a legitimate right and interest to regulate.
a. The party in whose behalf it is sought to have the marriage annulled was
eighteen (18) years of age or over but below twenty-one (21), and the
marriage was solemnized without the consent of the parents, guardian or The declared State policy that marriage, as an inviolable social institution, is the foundation of
person having substitute parental authority over the party, in that order, the family and shall be protected by the State, should not be read in total isolation but must be
unless after attaining the age of twenty-one (21), such party freely cohabited harmonized with other constitutional provisions. Aside from strengthening the solidarity of the
with the other and both lived together as husband or wife; Filipino family, the State is equally mandated to actively promote its total development. 79 It is
b. either party was of unsound mind, unless such party after coming to reason, also obligated to defend, among others, the right of children to special protection from all forms
freely cohabited with the other as husband and wife; of neglect, abuse, cruelty, exploitation, and other conditions prejudicial to their
c. The consent of either party was obtained by fraud, unless such party development.80 To Our mind, the State cannot effectively enforce these obligations if We limit
afterwards with full knowledge of the facts constituting the fraud, freely the application of Paragraph 2 of Article 26 only to those foreign divorce initiated by the alien
cohabited with the other as husband and wife; spouse. It is not amiss to point that the women and children are almost always the helpless
victims of all forms of domestic abuse and violence. In fact, among the notable legislation
passed in order to minimize, if not eradicate, the menace are R.A. No. 6955 (prohibiting mail As judges, we are not automatons. We do not and must not unfeelingly apply the law as it is
order bride and similar practices), R.A. No. 9262 ("Anti-Violence Against Women and Their worded, yielding like robots to the literal command without regard to its cause and consequence.
Children Act of 2004"), R.A. No. 9710 ("The Magna Carta of Women"), R.A. No. 10354 ("The "Courts are apt to err by sticking too closely to the words of a law," so we are warned, by Justice
Responsible Parenthood and Reproductive Health Act of 2012"), and R.A. No. 9208 ("Anti- Holmes again, "where these words import a policy that goes beyond them."
Trafficking in Persons Act of 2003"), as amended by R.A. No. 10364 ("Expanded Anti-Trafficking
in Persons Act of 2012"). Moreover, in protecting and strengthening the Filipino family as a basic
xxxx
autonomous social institution, the Court must not lose sight of the constitutional mandate to
value the dignity of every human person, guarantee full respect for human rights, and ensure the
fundamental equality before the law of women and men.81 More than twenty centuries ago, Justinian defined justice "as the constant and perpetual wish to
render every one his due." That wish continues to motivate this Court when it assesses the facts
and the law in every case brought to it for decision. Justice is always an essential ingredient of
A prohibitive view of Paragraph 2 of Article 26 would do more harm than good. If We disallow a
its decisions. Thus when the facts warrant, we interpret the law in a way that will render justice,
Filipino citizen who initiated and obtained a foreign divorce from the coverage of Paragraph 2 of
presuming that it was the intention of the lawmaker, to begin with, that the law be dispensed with
Article 26 and still require him or her to first avail of the existing "mechanisms" under the Family
justice.86
Code, any subsequent relationship that he or she would enter in the meantime shall be
considered as illicit in the eyes of the Philippine law. Worse, any child born out of such "extra-
marital" affair has to suffer the stigma of being branded as illegitimate. Surely, these are just but Indeed, where the interpretation of a statute according to its exact and literal import would lead
a few of the adverse consequences, not only to the parent but also to the child, if We are to hold to mischievous results or contravene the clear purpose of the legislature, it should be construed
a restrictive interpretation of the subject provision. The irony is that the principle of inviolability of according to its spirit and reason, disregarding as far as necessary the letter of the law. 87 A
marriage under Section 2, Article XV of the Constitution is meant to be tilted in favor of marriage statute may, therefore, be extended to cases not within the literal meaning of its terms, so long
and against unions not formalized by marriage, but without denying State protection and as they come within its spirit or intent.88
assistance to live-in arrangements or to families formed according to indigenous customs. 82
The foregoing notwithstanding, We cannot yet write finis to this controversy by granting Manalo's
This Court should not turn a blind eye to the realities of the present time. With the advancement petition to recognize and enforce the divorce decree rendered by the Japanese court and to
of communication and information technology, as well as the improvement of the transportation cancel the entry of marriage in the Civil Registry of San Juan, Metro Manila.
system that almost instantly connect people from all over the world, mixed marriages have
become not too uncommon. Likewise, it is recognized that not all marriages are made in heaven
and that imperfect humans more often than not create imperfect unions. 83 Living in a flawed Jurisprudence has set guidelines before Philippine courts recognize a foreign judgment relating
world, the unfortunate reality for some is that the attainment of the individual's full human to the status of a marriage where one of the parties is a citizen of a foreign country. Presentation
potential and self-fulfillment is not found and achieved in the context of a marriage. Thus, it is solely of the divorce decree will not suffice.89 The fact of divorce must still first be
hypocritical to safeguard the quantity of existing marriages and, at the same time, brush aside proven.90 Before a foreign divorce decree can be recognized by our courts, the party pleading it
the truth that some of them are of rotten quality. must prove the divorce as a fact and demonstrate its conformity to the foreign law allowing it.91

Going back, We hold that marriage, being a mutual and shared commitment between two x x x Before a foreign judgment is given presumptive evidentiary value, the document must first
parties, cannot possibly be productive of any good to the society where one is considered be presented and admitted in evidence. A divorce obtained abroad is proven by the divorce
released from the marital bond while the other remains bound to it. 84 In reiterating that the decree itself. Indeed the best evidence of a judgment is the judgment itself. The decree purports
Filipino spouse should not be discriminated against in his or her own country if the ends of to be a written act or record of an act of an official body or tribunal of a foreign country.
justice are to be served, San Luis v. San Luis85 quoted:
Under Sections 24 and 25 of Rule 132, on the other hand, a writing or document may be proven
x x x In Alonzo v. Intermediate Appellate Court, the Court stated: as a public or official record of a foreign country by either (1) an official publication or (2) a copy
thereof attested by the officer having legal custody of the document. If the record is not kept in
the Philippines, such copy must be (a) accompanied by a certificate issued by the proper
But as has also been aptly observed, we test a law by its results; and likewise, we may add, by diplomatic or consular officer in the Philippine foreign service stationed in the foreign country in
its purposes. It is a cardinal rule that, in seeking the meaning of the law, the first concern of the which the record is kept and (b) authenticated by the seal of his office. 92
judge should be to discover in its provisions the intent of the lawmaker. Unquestionably, the law
should never be interpreted in such a way as to cause injustice as this is never within the
legislative intent. An indispensable part of that intent, in fact, for we presume the good motives of In granting Manalo's petition, the CA noted:
the legislature, is to render justice.
In this case, Petitioner was able to submit before the court a quo the 1) Decision of the
Thus, we interpret and apply the law not independently of but in consonance with justice. Law Japanese Court allowing the divorce; 2) the Authentication/Certificate issued by the Philippine
Consulate General in Osaka, Japan of the Decree of Divorce; and 3) Acceptance of Certificate
and justice are inseparable, and we must keep them so. To be sure, there are some laws that,
while generally valid, may seem arbitrary when applied in a particular case because of its of Divorce by Petitioner and the Japanese national. Under Rule 132, Sections 24 and 25, in
peculiar circumstances. In such a situation, we are not bound, because only of our nature and relation to Rule 39, Section 48 (b) of the Rules of Court, these documents sufficiently prove the
subject Divorce Decree as a fact. Thus, We are constrained to recognize the Japanese Court's
functions, to apply them just the same, in slavish obedience to their language. What we do
instead is find a balance between the word and the will, that justice may be done even as the judgment decreeing the divorce.93
law is obeyed.
If the opposing party fails to properly object, as in this case, the divorce decree is rendered
admissible as a written act of the foreign court.94 As it appears, the existence of the divorce
decree was not denied by the OSG; neither was the jurisdiction of the divorce court impeached
nor the validity of its proceedings challenged on the ground of collusion, fraud, or clear mistake
of fact or law, albeit an opportunity to do so.95

Nonetheless, the Japanese law on divorce must still be proved.

x x x The burden of proof lies with the "party who alleges the existence of a fact or thing
necessary in the prosecution or defense of an action." In civil cases, plaintiffs have the burden of
proving the material allegations of the complaint when those are denied by the answer; and
defendants have the burden of proving the material allegations in their answer when they
introduce new matters. x x x

It is well-settled in our jurisdiction that our courts cannot take judicial notice of foreign laws. Like
any other facts, they must be alleged and proved. x x x The power of judicial notice must be
exercised with caution, and every reasonable doubt upon the subject should be resolved in the
negative.96

Since the divorce was raised by Manalo, the burden of proving the pertinent Japanese law
validating it, as well as her former husband's capacity to remarry, fall squarely upon her.
Japanese laws on persons and family relations are not among those matters that Filipino judges
are supposed to know by reason of their judicial function.

WHEREFORE, the petition for review on certiorari is DENIED. The September 18, 2014
Decision and October 12, 2015 Resolution of the Court of Appeals in CA-G.R. CV No. 100076,
are AFFIRMED IN PART. The case is REMANDED to the court of origin for further proceedings
and reception of evidence as to the relevant Japanese law on divorce.

SO ORDERED.
G.R. No. 179267 June 25, 2013 Private respondent's claims

JESUS C. GARCIA, Petitioner, Private respondent married petitioner in 2002 when she was 34 years old and the former was
vs. eleven years her senior. They have three (3) children, namely: Jo-Ann J. Garcia, 17 years old,
THE HONORABLE RAY ALAN T. DRILON, Presiding Judge, Regional Trial Court-Branch who is the natural child of petitioner but whom private respondent adopted; Jessie Anthone J.
41, Bacolod City, and ROSALIE JAYPE-GARCIA, for herself and in behalf of minor Garcia, 6 years old; and Joseph Eduard J. Garcia, 3 years old. 8
children, namely: JO-ANN, JOSEPH EDUARD, JESSE ANTHONE, all surnamed
GARCIA, Respondents.
Private respondent described herself as a dutiful and faithful wife, whose life revolved around
her husband. On the other hand, petitioner, who is of Filipino-Chinese descent, is dominant,
DECISION controlling, and demands absolute obedience from his wife and children. He forbade private
respondent to pray, and deliberately isolated her from her friends. When she took up law, and
even when she was already working part time at a law office, petitioner trivialized her ambitions
PERLAS-BERNABE, J.:
and prevailed upon her to just stay at home. He was often jealous of the fact that his attractive
wife still catches the eye of some men, at one point threatening that he would have any man
Hailed as the bastion of Christianity in Asia, the Philippines boasts of 86.8 million Filipinos- or 93 eyeing her killed.9
percent of a total population of 93.3 million – adhering to the teachings of Jesus Christ.1 Yet, the
admonition for husbands to love their wives as their own bodies just as Christ loved the church
Things turned for the worse when petitioner took up an affair with a bank manager of Robinson's
and gave himself up for her2 failed to prevent, or even to curb, the pervasiveness of violence
Bank, Bacolod City, who is the godmother of one of their sons. Petitioner admitted to the affair
against Filipino women. The National Commission on the Role of Filipino Women (NCRFW)
when private respondent confronted him about it in 2004. He even boasted to the household
reported that, for the years 2000-2003, "female violence comprised more than 90o/o of all forms
help about his sexual relations with said bank manager. Petitioner told private respondent,
of abuse and violence and more than 90% of these reported cases were committed by the
though, that he was just using the woman because of their accounts with the bank. 10
women's intimate partners such as their husbands and live-in partners."3

Petitioner's infidelity spawned a series of fights that left private respondent physically and
Thus, on March 8, 2004, after nine (9) years of spirited advocacy by women's groups, Congress
emotionally wounded. In one of their quarrels, petitioner grabbed private respondent on both
enacted Republic Act (R.A.) No. 9262, entitled "An Act Defining Violence Against Women and
arms and shook her with such force that caused bruises and hematoma. At another time,
Their Children, Providing for Protective Measures for Victims, Prescribing Penalties Therefor,
petitioner hit private respondent forcefully on the lips that caused some bleeding. Petitioner
and for Other Purposes." It took effect on March 27, 2004. 4
sometimes turned his ire on their daughter, Jo-Ann, who had seen the text messages he sent to
his paramour and whom he blamed for squealing on him. He beat Jo-Ann on the chest and
R.A. 9262 is a landmark legislation that defines and criminalizes acts of violence against women slapped her many times. When private respondent decided to leave petitioner, Jo-Ann begged
and their children (VAWC) perpetrated by women's intimate partners, i.e, husband; former her mother to stay for fear that if the latter leaves, petitioner would beat her up. Even the small
husband; or any person who has or had a sexual or dating relationship, or with whom the boys are aware of private respondent's sufferings. Their 6-year-old son said that when he grows
woman has a common child.5 The law provides for protection orders from the barangay and the up, he would beat up his father because of his cruelty to private respondent. 11
courts to prevent the commission of further acts of VAWC; and outlines the duties and
responsibilities of barangay officials, law enforcers, prosecutors and court personnel, social
All the emotional and psychological turmoil drove private respondent to the brink of despair. On
workers, health care providers, and other local government officials in responding to complaints
December 17, 2005, while at home, she attempted suicide by cutting her wrist. She was found
of VAWC or requests for assistance.
by her son bleeding on the floor. Petitioner simply fled the house instead of taking her to the
hospital. Private respondent was hospitalized for about seven (7) days in which time petitioner
A husband is now before the Court assailing the constitutionality of R.A. 9262 as being violative never bothered to visit, nor apologized or showed pity on her. Since then, private respondent
of the equal protection and due process clauses, and an undue delegation of judicial power to has been undergoing therapy almost every week and is taking anti-depressant medications.12
barangay officials.
When private respondent informed the management of Robinson's Bank that she intends to file
The Factual Antecedents charges against the bank manager, petitioner got angry with her for jeopardizing the manager's
job. He then packed his things and told private respondent that he was leaving her for good. He
even told private respondent's mother, who lives with them in the family home, that private
On March 23, 2006, Rosalie Jaype-Garcia (private respondent) filed, for herself and in behalf of respondent should just accept his extramarital affair since he is not cohabiting with his paramour
her minor children, a verified petition6 (Civil Case No. 06-797) before the Regional Trial Court
and has not sired a child with her.13
(RTC) of Bacolod City for the issuance of a Temporary Protection Order (TPO) against her
husband, Jesus C. Garcia (petitioner), pursuant to R.A. 9262. She claimed to be a victim of
physical abuse; emotional, psychological, and economic violence as a result of marital infidelity Private respondent is determined to separate from petitioner but she is afraid that he would take
on the part of petitioner, with threats of deprivation of custody of her children and of financial her children from her and deprive her of financial support. Petitioner had previously warned her
support.7 that if she goes on a legal battle with him, she would not get a single centavo. 14

Petitioner controls the family businesses involving mostly the construction of deep wells. He is
the President of three corporations – 326 Realty Holdings, Inc., Negros Rotadrill Corporation,
and J-Bros Trading Corporation – of which he and private respondent are both stockholders. In should also be ordered to surrender any unlicensed firearms in his possession or
contrast to the absolute control of petitioner over said corporations, private respondent merely control.
draws a monthly salary of ₱20,000.00 from one corporation only, the Negros Rotadrill
Corporation. Household expenses amounting to not less than ₱200,000.00 a month are paid for
e) To pay full financial support for the Petitioner and the children, including rental of a
by private respondent through the use of credit cards, which, in turn, are paid by the same
house for them, and educational and medical expenses.
corporation together with the bills for utilities.15

f) Not to dissipate the conjugal business.


On the other hand, petitioner receives a monthly salary of ₱60,000.00 from Negros Rotadrill
Corporation, and enjoys unlimited cash advances and other benefits in hundreds of thousands of
pesos from the corporations.16 After private respondent confronted him about the affair, g) To render an accounting of all advances, benefits, bonuses and other cash he
petitioner forbade her to hold office at JBTC Building, Mandalagan, where all the businesses of received from all the corporations from 1 January 2006 up to 31 March 2006, which
the corporations are conducted, thereby depriving her of access to full information about said himself and as President of the corporations and his Comptroller, must submit to the
businesses. Until the filing of the petition a quo, petitioner has not given private respondent an Court not later than 2 April 2006. Thereafter, an accounting of all these funds shall be
accounting of the businesses the value of which she had helped raise to millions of pesos. 17 reported to the court by the Comptroller, copy furnished to the Petitioner, every 15
days of the month, under pain of Indirect Contempt of Court.
Action of the RTC of Bacolod City
h) To ensure compliance especially with the order granting support pendente lite, and
considering the financial resources of the Respondent and his threat that if the
Finding reasonable ground to believe that an imminent danger of violence against the private
Petitioner sues she will not get a single centavo, the Respondent is ordered to put up
respondent and her children exists or is about to recur, the RTC issued a TPO 18 on March 24,
a BOND TO KEEP THE PEACE in the amount of FIVE MILLION PESOS, in two
2006 effective for thirty (30) days, which is quoted hereunder:
sufficient sureties.

Respondent (petitioner herein), Jesus Chua Garcia, is hereby:


On April 24, 2006, upon motion19 of private respondent, the trial court issued an
amended TPO,20 effective for thirty (30) days, which included the following additional
a) Ordered to remove all his personal belongings from the conjugal dwelling or family provisions:
home within 24 hours from receipt of the Temporary Restraining Order and if he
refuses, ordering that he be removed by police officers from the conjugal dwelling; this
i) The petitioners (private respondents herein) are given the continued use of the
order is enforceable notwithstanding that the house is under the name of 236 Realty
Nissan Patrol and the Starex Van which they are using in Negros Occidental.
Holdings Inc. (Republic Act No. 9262 states "regardless of ownership"), this is to allow
the Petitioner (private respondent herein) to enter the conjugal dwelling without any
danger from the Respondent. j) The petitioners are given the continued use and occupation of the house in
Parañaque, the continued use of the Starex van in Metro Manila, whenever they go to
Manila.
After the Respondent leaves or is removed from the conjugal dwelling, or anytime the
Petitioner decides to return to the conjugal dwelling to remove things, the Petitioner
shall be assisted by police officers when re-entering the family home. k) Respondent is ordered to immediately post a bond to keep the peace, in two
sufficient sureties.
The Chief of Police shall also give the Petitioner police assistance on Sunday, 26
March 2006 because of the danger that the Respondent will attempt to take her l) To give monthly support to the petitioner provisionally fixed in the sum of One
children from her when he arrives from Manila and finds out about this suit. Hundred Fifty Thousand Pesos (Php 150,000.00) per month plus rental expenses of
Fifty Thousand Pesos (Php 50,000.00) per month until the matter of support could be
finally resolved.
b) To stay away from the petitioner and her children, mother and all her household
help and driver from a distance of 1,000 meters, and shall not enter the gate of the
subdivision where the Petitioner may be temporarily residing. Two days later, or on April 26, 2006, petitioner filed an Opposition to the Urgent Ex-Parte Motion
for Renewal of the TPO21 seeking the denial of the renewal of the TPO on the grounds that it did
not (1) comply with the three-day notice rule, and (2) contain a notice of hearing. He further
c) Not to harass, annoy, telephone, contact or otherwise communicate with the
asked that the TPO be modified by (1) removing one vehicle used by private respondent and
Petitioner, directly or indirectly, or through other persons, or contact directly or
returning the same to its rightful owner, the J-Bros Trading Corporation, and (2) cancelling or
indirectly her children, mother and household help, nor send gifts, cards, flowers,
reducing the amount of the bond from ₱5,000,000.00 to a more manageable level at
letters and the like. Visitation rights to the children may be subject of a modified TPO
₱100,000.00.
in the future.

Subsequently, on May 23, 2006, petitioner moved22 for the modification of the TPO to allow him
d) To surrender all his firearms including a .9MM caliber firearm and a Walther PPK
visitation rights to his children.
and ordering the Philippine National Police Firearms and Explosives Unit and the
Provincial Director of the PNP to cancel all the Respondent's firearm licenses. He
On May 24, 2006, the TPO was renewed and extended yet again, but subject only to the get her and her children's belongings. Finding some of her things inside a housemaid's (Sheryl
following modifications prayed for by private respondent: Jamola) bag in the maids' room, private respondent filed a case for qualified theft against
Jamola.27
a) That respondent (petitioner herein) return the clothes and other personal
belongings of Rosalie and her children to Judge Jesus Ramos, co-counsel for On August 23, 2006, the RTC issued a TPO,28 effective for thirty (30) days, which reads as
Petitioner, within 24 hours from receipt of the Temporary Protection Order by his follows:
counsel, otherwise be declared in Indirect Contempt of Court;
Respondent (petitioner herein), Jesus Chua Garcia, is hereby:
b) Respondent shall make an accounting or list of furniture and equipment in the
conjugal house in Pitimini St., Capitolville Subdivision, Bacolod City within 24 hours
1) Prohibited from threatening to commit or committing, personally or through another,
from receipt of the Temporary Protection Order by his counsel;
acts of violence against the offended party;

c) Ordering the Chief of the Women's Desk of the Bacolod City Police Headquarters to
2) Prohibited from harassing, annoying, telephoning, contacting or otherwise
remove Respondent from the conjugal dwelling within eight (8) hours from receipt of
communicating in any form with the offended party, either directly or indirectly;
the Temporary Protection Order by his counsel, and that he cannot return until 48
hours after the petitioners have left, so that the petitioner Rosalie and her
representatives can remove things from the conjugal home and make an inventory of 3) Required to stay away, personally or through his friends, relatives, employees or
the household furniture, equipment and other things in the conjugal home, which shall agents, from all the Petitioners Rosalie J. Garcia and her children, Rosalie J. Garcia's
be submitted to the Court. three brothers, her mother Primitiva Jaype, cook Novelita Caranzo, driver Romeo
Hontiveros, laundrywoman Mercedita Bornales, security guard Darwin Gayona and
the petitioner's other household helpers from a distance of 1,000 meters, and shall not
d) Deliver full financial support of Php200,000.00 and Php50,000.00 for rental and
enter the gate of the subdivision where the Petitioners are temporarily residing, as well
Php25,000.00 for clothes of the three petitioners (sic) children within 24 hours from
as from the schools of the three children; Furthermore, that respondent shall not
receipt of the Temporary Protection Order by his counsel, otherwise be declared in
contact the schools of the children directly or indirectly in any manner including,
indirect contempt of Court;
ostensibly to pay for their tuition or other fees directly, otherwise he will have access to
the children through the schools and the TPO will be rendered nugatory;
e) That respondent surrender his two firearms and all unlicensed firearms to the Clerk
of Court within 24 hours from receipt of the Temporary Protection Order by his
4) Directed to surrender all his firearms including .9MM caliber firearm and a Walther
counsel;
PPK to the Court;

f) That respondent shall pay petitioner educational expenses of the children upon
5) Directed to deliver in full financial support of Php200,000.00 a month and
presentation of proof of payment of such expenses. 23
Php50,000.00 for rental for the period from August 6 to September 6, 2006; and
support in arrears from March 2006 to August 2006 the total amount of
Claiming that petitioner continued to deprive them of financial support; failed to faithfully comply Php1,312,000.00;
with the TPO; and committed new acts of harassment against her and their children, private
respondent filed another application24 for the issuance of a TPO ex parte. She alleged inter
6) Directed to deliver educational expenses for 2006-2007 the amount of
Php75,000.00 and Php25,000.00;
alia that petitioner contrived a replevin suit against himself by J-Bros Trading, Inc., of which the
latter was purportedly no longer president, with the end in view of recovering the Nissan Patrol
7) Directed to allow the continued use of a Nissan Patrol with Plate No. FEW 508 and
and Starex Van used by private respondent and the children. A writ of replevin was served upon
a Starex van with Plate No. FFD 991 and should the respondent fail to deliver said
private respondent by a group of six or seven policemen with long firearms that scared the two
vehicles, respondent is ordered to provide the petitioner another vehicle which is the
small boys, Jessie Anthone and Joseph Eduard.25
one taken by J Bros Tading;

While Joseph Eduard, then three years old, was driven to school, two men allegedly attempted
8) Ordered not to dissipate, encumber, alienate, sell, lease or otherwise dispose of the
to kidnap him, which incident traumatized the boy resulting in his refusal to go back to school.
conjugal assets, or those real properties in the name of Jesus Chua Garcia only and
On another occasion, petitioner allegedly grabbed their daughter, Jo-Ann, by the arm and
those in which the conjugal partnership of gains of the Petitioner Rosalie J. Garcia and
threatened her.26 The incident was reported to the police, and Jo-Ann subsequently filed a
respondent have an interest in, especially the conjugal home located in No. 14,
criminal complaint against her father for violation of R.A. 7610, also known as the "Special
Pitimini St., Capitolville Subdivision, Bacolod City, and other properties which are
Protection of Children Against Child Abuse, Exploitation and Discrimination Act."
conjugal assets or those in which the conjugal partnership of gains of Petitioner
Rosalie J. Garcia and the respondent have an interest in and listed in Annexes "I," "I-
Aside from the replevin suit, petitioner's lawyers initiated the filing by the housemaids working at 1," and "I-2," including properties covered by TCT Nos. T-186325 and T-168814;
the conjugal home of a complaint for kidnapping and illegal detention against private
respondent. This came about after private respondent, armed with a TPO, went to said home to
9) Ordered that the Register of Deeds of Bacolod City and E.B. Magalona shall be The Issues
served a copy of this TEMPORARY PROTECTION ORDER and are ordered not to
allow the transfer, sale, encumbrance or disposition of these above-cited properties to
I.
any person, entity or corporation without the personal presence of petitioner Rosalie J.
Garcia, who shall affix her signature in the presence of the Register of Deeds, due to
the fear of petitioner Rosalie that her signature will be forged in order to effect the THE COURT OF APPEALS ERRED IN DISMISSING THE PETITION ON THE THEORY THAT
encumbrance or sale of these properties to defraud her or the conjugal partnership of THE ISSUE OF CONSTITUTIONALITY WAS NOT RAISED AT THE EARLIEST
gains. OPPORTUNITY AND THAT, THE PETITION CONSTITUTES A COLLATERAL ATTACK ON
THE VALIDITY OF THE LAW.
In its Order29 dated September 26, 2006, the trial court extended the aforequoted TPO for
another ten (10) days, and gave petitioner a period of five (5) days within which to show cause II.
why the TPO should not be renewed, extended, or modified. Upon petitioner's
manifestation,30 however, that he has not received a copy of private respondent's motion to
THE COURT OF APPEALS COMMITTED SERIOUS ERROR IN FAILING TO CONCLUDE
modify/renew the TPO, the trial court directed in its Order31 dated October 6, 2006 that petitioner
be furnished a copy of said motion. Nonetheless, an Order32 dated a day earlier, October 5, had THAT R.A. 9262 IS DISCRIMINATORY, UNJUST, AND VIOLATIVE OF THE EQUAL
already been issued renewing the TPO dated August 23, 2006. The pertinent portion is quoted PROTECTION CLAUSE.
hereunder:
III.
xxxx
THE COURT OF APPEALS COMMITTED GRAVE MISTAKE IN NOT FINDING THAT R.A.
x x x it appearing further that the hearing could not yet be finally terminated, the Temporary 9262 RUNS COUNTER TO THE DUE PROCESS CLAUSE OF THE CONSTITUTION.
Protection Order issued on August 23, 2006 is hereby renewed and extended for thirty (30) days
and continuously extended and renewed for thirty (30) days, after each expiration, until further IV.
orders, and subject to such modifications as may be ordered by the court.
THE COURT OF APPEALS ERRED IN NOT FINDING THAT THE LAW DOES VIOLENCE TO
After having received a copy of the foregoing Order, petitioner no longer submitted the required THE POLICY OF THE STATE TO PROTECT THE FAMILY AS A BASIC SOCIAL
comment to private respondent's motion for renewal of the TPO arguing that it would only be an INSTITUTION.
"exercise in futility."33
V.
Proceedings before the CA
THE COURT OF APPEALS SERIOUSLY ERRED IN NOT DECLARING R.A. No. 9262 AS
During the pendency of Civil Case No. 06-797, petitioner filed before the Court of Appeals (CA) INVALID AND UNCONSTITUTIONAL BECAUSE IT ALLOWS AN UNDUE DELEGATION OF
a petition34 for prohibition (CA-G.R. CEB-SP. No. 01698), with prayer for injunction and JUDICIAL POWER TO THE BARANGAY OFFICIALS.38
temporary restraining order, challenging (1) the constitutionality of R.A. 9262 for being violative
of the due process and the equal protection clauses, and (2) the validity of the modified TPO
issued in the civil case for being "an unwanted product of an invalid law." The Ruling of the Court

On May 26, 2006, the appellate court issued a 60-day Temporary Restraining Order36 (TRO) Before delving into the arguments propounded by petitioner against the constitutionality of R.A.
against the enforcement of the TPO, the amended TPOs and other orders pursuant thereto. 9262, we shall first tackle the propriety of the dismissal by the appellate court of the petition for
prohibition (CA-G.R. CEB-SP. No. 01698) filed by petitioner.

Subsequently, however, on January 24, 2007, the appellate court dismissed36 the petition for
failure of petitioner to raise the constitutional issue in his pleadings before the trial court in the As a general rule, the question of constitutionality must be raised at the earliest opportunity so
civil case, which is clothed with jurisdiction to resolve the same. Secondly, the challenge to the that if not raised in the pleadings, ordinarily it may not be raised in the trial, and if not raised in
validity the trial court, it will not be considered on appeal.39 Courts will not anticipate a question of
constitutional law in advance of the necessity of deciding it.40

of R.A. 9262 through a petition for prohibition seeking to annul the protection orders issued by
the trial court constituted a collateral attack on said law. In defending his failure to attack the constitutionality of R.A. 9262 before the RTC of Bacolod
City, petitioner argues that the Family Court has limited authority and jurisdiction that is
"inadequate to tackle the complex issue of constitutionality."41
His motion for reconsideration of the foregoing Decision having been denied in the
Resolution37 dated August 14, 2007, petitioner is now before us alleging that –
We disagree.
Family Courts have authority and jurisdiction to consider the constitutionality of a statute. Section 20 of A.M. No. 04-10-11-SC, the Rule on Violence Against Women and Their Children,
lays down a new kind of procedure requiring the respondent to file an opposition to the petition
and not an answer.49 Thus:
At the outset, it must be stressed that Family Courts are special courts, of the same level as
Regional Trial Courts. Under R.A. 8369, otherwise known as the "Family Courts Act of 1997,"
family courts have exclusive original jurisdiction to hear and decide cases of domestic violence SEC. 20. Opposition to petition. – (a) The respondent may file an opposition to the petition which
against women and children.42 In accordance with said law, the Supreme Court designated from he himself shall verify. It must be accompanied by the affidavits of witnesses and shall show
among the branches of the Regional Trial Courts at least one Family Court in each of several cause why a temporary or permanent protection order should not be issued.
key cities identified.43 To achieve harmony with the first mentioned law, Section 7 of R.A. 9262
now provides that Regional Trial Courts designated as Family Courts shall have original and
(b) Respondent shall not include in the opposition any counterclaim, cross-claim or third-party
exclusive jurisdiction over cases of VAWC defined under the latter law, viz:
complaint, but any cause of action which could be the subject thereof may be litigated in a
separate civil action. (Emphasis supplied)
SEC. 7. Venue. – The Regional Trial Court designated as a Family Court shall have original and
exclusive jurisdiction over cases of violence against women and their children under this law. In
We cannot subscribe to the theory espoused by petitioner that, since a counterclaim, cross-claim
the absence of such court in the place where the offense was committed, the case shall be filed
and third-party complaint are to be excluded from the opposition, the issue of constitutionality
in the Regional Trial Court where the crime or any of its elements was committed at the option of
cannot likewise be raised therein. A counterclaim is defined as any claim for money or other
the complainant. (Emphasis supplied)
relief which a defending party may have against an opposing party.50 A cross-claim, on the other
hand, is any claim by one party against a co-party arising out of the transaction or occurrence
Inspite of its designation as a family court, the RTC of Bacolod City remains possessed of that is the subject matter either of the original action or of a counterclaim therein.51Finally, a
authority as a court of general original jurisdiction to pass upon all kinds of cases whether civil, third-party complaint is a claim that a defending party may, with leave of court, file against a
criminal, special proceedings, land registration, guardianship, naturalization, admiralty or person not a party to the action for contribution, indemnity, subrogation or any other relief, in
insolvency.44 It is settled that RTCs have jurisdiction to resolve the constitutionality of a respect of his opponent's claim.52As pointed out by Justice Teresita J. Leonardo-De Castro, the
statute,45 "this authority being embraced in the general definition of the judicial power to unconstitutionality of a statute is not a cause of action that could be the subject of a
determine what are the valid and binding laws by the criterion of their conformity to the counterclaim, cross-claim or a third-party complaint. Therefore, it is not prohibited from being
fundamental law."46The Constitution vests the power of judicial review or the power to declare raised in the opposition in view of the familiar maxim expressio unius est exclusio alterius.
the constitutionality or validity of a law, treaty, international or executive agreement, presidential
decree, order, instruction, ordinance, or regulation not only in this Court, but in all RTCs. 47 We
Moreover, it cannot be denied that this issue affects the resolution of the case a quo because
said in J.M. Tuason and Co., Inc. v. CA48 that, "plainly the Constitution contemplates that the
the right of private respondent to a protection order is founded solely on the very statute the
inferior courts should have jurisdiction in cases involving constitutionality of any treaty or law, for
validity of which is being attacked53 by petitioner who has sustained, or will sustain, direct injury
it speaks of appellate review of final judgments of inferior courts in cases where such
as a result of its enforcement. The alleged unconstitutionality of R.A. 9262 is, for all intents and
constitutionality happens to be in issue." Section 5, Article VIII of the 1987 Constitution reads in
purposes, a valid cause for the non-issuance of a protection order.
part as follows:

That the proceedings in Civil Case No. 06-797 are summary in nature should not have deterred
SEC. 5. The Supreme Court shall have the following powers:
petitioner from raising the same in his Opposition. The question relative to the constitutionality of
a statute is one of law which does not need to be supported by evidence. 54 Be that as it may,
xxx Section 25 of A.M. No. 04-10-11-SC nonetheless allows the conduct of a hearing to determine
legal issues, among others, viz:
2. Review, revise, reverse, modify, or affirm on appeal or certiorari, as the law or the Rules of
Court may provide, final judgments and orders of lower courts in: SEC. 25. Order for further hearing. - In case the court determines the need for further hearing, it
may issue an order containing the following:
a. All cases in which the constitutionality or validity of any treaty, international or executive
agreement, law, presidential decree, proclamation, order, instruction, ordinance, or regulation is (a) Facts undisputed and admitted;
in question.
(b) Factual and legal issues to be resolved;
xxxx
(c) Evidence, including objects and documents that have been marked and will be
Thus, contrary to the posturing of petitioner, the issue of constitutionality of R.A. 9262 could presented;
have been raised at the earliest opportunity in his Opposition to the petition for protection order
before the RTC of Bacolod City, which had jurisdiction to determine the same, subject to the
(d) Names of witnesses who will be ordered to present their direct testimonies in the
review of this Court.
form of affidavits; and
(e) Schedule of the presentation of evidence by both parties which shall be done in Petitioner claims that since R.A. 9262 is intended to prevent and criminalize spousal and child
one day, to the extent possible, within the 30-day period of the effectivity of the abuse, which could very well be committed by either the husband or the wife, gender alone is
temporary protection order issued. (Emphasis supplied) not enough basis to deprive the husband/father of the remedies under the law. 60

To obviate potential dangers that may arise concomitant to the conduct of a hearing when A perusal of the deliberations of Congress on Senate Bill No. 2723,61 which became R.A. 9262,
necessary, Section 26 (b) of A.M. No. 04-10-11-SC provides that if a temporary protection order reveals that while the sponsor, Senator Luisa Pimentel-Ejercito (better known as Senator Loi
issued is due to expire, the trial court may extend or renew the said order for a period of thirty Estrada), had originally proposed what she called a "synthesized measure"62 – an amalgamation
(30) days each time until final judgment is rendered. It may likewise modify the extended or of two measures, namely, the "Anti-Domestic Violence Act" and the "Anti-Abuse of Women in
renewed temporary protection order as may be necessary to meet the needs of the parties. With Intimate Relationships Act"63 – providing protection to "all family members, leaving no one in
the private respondent given ample protection, petitioner could proceed to litigate the isolation" but at the same time giving special attention to women as the "usual victims" of
constitutional issues, without necessarily running afoul of the very purpose for the adoption of violence and abuse,64 nonetheless, it was eventually agreed that men be denied protection
the rules on summary procedure. under the same measure. We quote pertinent portions of the deliberations:

In view of all the foregoing, the appellate court correctly dismissed the petition for prohibition with Wednesday, December 10, 2003
prayer for injunction and temporary restraining order (CA-G.R. CEB - SP. No. 01698). Petitioner
may have proceeded upon an honest belief that if he finds succor in a superior court, he could
Senator Pangilinan. I just wanted to place this on record, Mr. President. Some women's groups
be granted an injunctive relief. However, Section 22(j) of A.M. No. 04-10-11-SC expressly
have expressed concerns and relayed these concerns to me that if we are to include domestic
disallows the filing of a petition for certiorari, mandamus or prohibition against any interlocutory
violence apart from against women as well as other members of the household, including
order issued by the trial court. Hence, the 60-day TRO issued by the appellate court in this case
children or the husband, they fear that this would weaken the efforts to address domestic
against the enforcement of the TPO, the amended TPOs and other orders pursuant thereto was
violence of which the main victims or the bulk of the victims really are the wives, the spouses or
improper, and it effectively hindered the case from taking its normal course in an expeditious and
the female partners in a relationship. We would like to place that on record. How does the good
summary manner.
Senator respond to this kind of observation?

As the rules stand, a review of the case by appeal or certiorari before judgment is prohibited.
Senator Estrada. Yes, Mr. President, there is this group of women who call themselves "WIIR"
Moreover, if the appeal of a judgment granting permanent protection shall not stay its
Women in Intimate Relationship. They do not want to include men in this domestic violence. But
enforcement,55 with more reason that a TPO, which is valid only for thirty (30) days at a
plenty of men are also being abused by women. I am playing safe so I placed here members of
time,56 should not be enjoined.
the family, prescribing penalties therefor and providing protective measures for victims. This
includes the men, children, live-in, common-law wives, and those related with the family.65
The mere fact that a statute is alleged to be unconstitutional or invalid, does not of itself entitle a
litigant to have the same enjoined.57 In Younger v. Harris, Jr.,58 the Supreme Court of the United
xxx
States declared, thus:

Wednesday, January 14, 2004


Federal injunctions against state criminal statutes, either in their entirety or with respect to their
separate and distinct prohibitions, are not to be granted as a matter of course, even if such
statutes are unconstitutional. No citizen or member of the community is immune from xxxx
prosecution, in good faith, for his alleged criminal acts. The imminence of such a prosecution
even though alleged to be unauthorized and, hence, unlawful is not alone ground for relief in
equity which exerts its extraordinary powers only to prevent irreparable injury to the plaintiff who The President Pro Tempore. x x x
seeks its aid. (Citations omitted)
Also, may the Chair remind the group that there was the discussion whether to limit this to
The sole objective of injunctions is to preserve the status quo until the trial court hears fully the women and not to families which was the issue of the AWIR group. The understanding that I
merits of the case. It bears stressing, however, that protection orders are granted ex parte so as have is that we would be having a broader scope rather than just women, if I remember
correctly, Madam sponsor.
to protect women and their children from acts of violence. To issue an injunction against such
orders will defeat the very purpose of the law against VAWC.
Senator Estrada. Yes, Mr. President.
Notwithstanding all these procedural flaws, we shall not shirk from our obligation to determine
novel issues, or issues of first impression, with far-reaching implications. We have, time and As a matter of fact, that was brought up by Senator Pangilinan during the interpellation period.
again, discharged our solemn duty as final arbiter of constitutional issues, and with more reason
now, in view of private respondent's plea in her Comment59 to the instant Petition that we should
put the challenge to the constitutionality of R.A. 9262 to rest. And so we shall. I think Senator Sotto has something to say to that.

Intent of Congress in enacting R.A. 9262. Senator Legarda. Mr. President, the reason I am in support of the measure. Do not get me
wrong. However, I believe that there is a need to protect women's rights especially in the
domestic environment.
As I said earlier, there are nameless, countless, voiceless women who have not had the Senator Sotto. I presume that the effect of the proposed amendment of Senator Legarda would
opportunity to file a case against their spouses, their live-in partners after years, if not decade, of be removing the "men and children" in this particular bill and focus specifically on women alone.
battery and abuse. If we broaden the scope to include even the men, assuming they can at all That will be the net effect of that proposed amendment. Hearing the rationale mentioned by the
be abused by the women or their spouses, then it would not equalize the already difficult distinguished sponsor, Sen. Luisa "Loi" Ejercito Estrada, I am not sure now whether she is
situation for women, Mr. President. inclined to accept the proposed amendment of Senator Legarda.

I think that the sponsor, based on our earlier conversations, concurs with this position. I am sure I am willing to wait whether she is accepting this or not because if she is going to accept this, I
that the men in this Chamber who love their women in their lives so dearly will agree with this will propose an amendment to the amendment rather than object to the amendment, Mr.
representation. Whether we like it or not, it is an unequal world. Whether we like it or not, no President.
matter how empowered the women are, we are not given equal opportunities especially in the
domestic environment where the macho Filipino man would always feel that he is stronger, more
xxxx
superior to the Filipino woman.

Senator Estrada. The amendment is accepted, Mr. President.


xxxx

The President Pro Tempore. Is there any objection?


The President Pro Tempore. What does the sponsor say?

xxxx
Senator Estrada. Mr. President, before accepting this, the committee came up with this bill
because the family members have been included in this proposed measure since the other
members of the family other than women are also possible victims of violence. While women are Senator Sotto. x x x May I propose an amendment to the amendment.
most likely the intended victims, one reason incidentally why the measure focuses on women,
the fact remains that in some relatively few cases, men also stand to be victimized and that
The President Pro Tempore. Before we act on the amendment?
children are almost always the helpless victims of violence. I am worried that there may not be
enough protection extended to other family members particularly children who are excluded.
Although Republic Act No. 7610, for instance, more or less, addresses the special needs of Senator Sotto. Yes, Mr. President.
abused children. The same law is inadequate. Protection orders for one are not available in said
law.
The President Pro Tempore. Yes, please proceed.

I am aware that some groups are apprehensive about granting the same protection to men,
fearing that they may use this law to justify their abusive behavior against women. However, we Senator Sotto. Mr. President, I am inclined to believe the rationale used by the distinguished
should also recognize that there are established procedures and standards in our courts which proponent of the amendment. As a matter of fact, I tend to agree. Kung may maaabuso, mas
give credence to evidentiary support and cannot just arbitrarily and whimsically entertain malamang iyong babae kaysa sa lalake. At saka iyong mga lalake, puwede na talagang magulpi
baseless complaints. iyan. Okey lang iyan. But I cannot agree that we remove the children from this particular
measure.

Mr. President, this measure is intended to harmonize family relations and to protect the family as
the basic social institution. Though I recognize the unequal power relations between men and So, if I may propose an amendment –
women in our society, I believe we have an obligation to uphold inherent rights and dignity of
both husband and wife and their immediate family members, particularly children. The President Pro Tempore. To the amendment.

While I prefer to focus mainly on women, I was compelled to include other family members as a Senator Sotto. – more than the women, the children are very much abused. As a matter of fact,
critical input arrived at after a series of consultations/meetings with various NGOs, experts, it is not limited to minors. The abuse is not limited to seven, six, 5-year-old children. I have seen
sports groups and other affected sectors, Mr. President. 14, 15-year-old children being abused by their fathers, even by their mothers. And it breaks my
heart to find out about these things.
Senator Sotto. Mr. President.
Because of the inadequate existing law on abuse of children, this particular measure will update
The President Pro Tempore. Yes, with the permission of the other senators. that. It will enhance and hopefully prevent the abuse of children and not only women.

Senator Sotto. Yes, with the permission of the two ladies on the Floor. SOTTO-LEGARDA AMENDMENTS

The President Pro Tempore. Yes, Sen. Vicente C. Sotto III is recognized. Therefore, may I propose an amendment that, yes, we remove the aspect of the men in the bill
but not the children.
Senator Legarda. I agree, Mr. President, with the Minority Leader. protection clause by favoring women over men as victims of violence and abuse to whom the
State extends its protection.
The President Pro Tempore. Effectively then, it will be women AND CHILDREN.
I. R.A. 9262 rests on substantial distinctions.
Senator Sotto. Yes, Mr. President.
The unequal power relationship between women and men; the fact that women are more likely
than men to be victims of violence; and the widespread gender bias and prejudice against
Senator Estrada. It is accepted, Mr. President.
women all make for real differences justifying the classification under the law. As Justice
McIntyre succinctly states, "the accommodation of differences ... is the essence of true
The President Pro Tempore. Is there any objection? [Silence] There being none, the equality."70
amendment, as amended, is approved.66
A. Unequal power relationship between men and women
It is settled that courts are not concerned with the wisdom, justice, policy, or expediency of a
statute.67 Hence, we dare not venture into the real motivations and wisdom of the members of
According to the Philippine Commission on Women (the National Machinery for Gender Equality
Congress in limiting the protection against violence and abuse under R.A. 9262 to women and
and Women's Empowerment), violence against women (VAW) is deemed to be closely linked
children only. No proper challenge on said grounds may be entertained in this proceeding.
with the unequal power relationship between women and men otherwise known as "gender-
Congress has made its choice and it is not our prerogative to supplant this judgment. The choice
based violence". Societal norms and traditions dictate people to think men are the leaders,
may be perceived as erroneous but even then, the remedy against it is to seek its amendment or
pursuers, providers, and take on dominant roles in society while women are nurturers, men's
repeal by the legislative. By the principle of separation of powers, it is the legislative that
companions and supporters, and take on subordinate roles in society. This perception leads to
determines the necessity, adequacy, wisdom and expediency of any law. 68 We only step in when
men gaining more power over women. With power comes the need to control to retain that
there is a violation of the Constitution. However, none was sufficiently shown in this case.
power. And VAW is a form of men's expression of controlling women to retain power.71

R.A. 9262 does not violate the guaranty of equal protection of the laws.
The United Nations, which has long recognized VAW as a human rights issue, passed its
Resolution 48/104 on the Declaration on Elimination of Violence Against Women on December
Equal protection simply requires that all persons or things similarly situated should be treated 20, 1993 stating that "violence against women is a manifestation of historically unequal power
alike, both as to rights conferred and responsibilities imposed. The oft-repeated disquisition in relations between men and women, which have led to domination over and discrimination
the early case of Victoriano v. Elizalde Rope Workers' Union69 is instructive: against women by men and to the prevention of the full advancement of women, and that
violence against women is one of the crucial social mechanisms by which women are forced into
subordinate positions, compared with men."72
The guaranty of equal protection of the laws is not a guaranty of equality in the application of the
laws upon all citizens of the state. It is not, therefore, a requirement, in order to avoid the
constitutional prohibition against inequality, that every man, woman and child should be affected Then Chief Justice Reynato S. Puno traced the historical and social context of gender-based
alike by a statute. Equality of operation of statutes does not mean indiscriminate operation on violence and developments in advocacies to eradicate VAW, in his remarks delivered during the
persons merely as such, but on persons according to the circumstances surrounding them. It Joint Launching of R.A. 9262 and its Implementing Rules last October 27, 2004, the pertinent
guarantees equality, not identity of rights. The Constitution does not require that things which are portions of which are quoted hereunder:
different in fact be treated in law as though they were the same. The equal protection clause
does not forbid discrimination as to things that are different. It does not prohibit legislation which
History reveals that most societies sanctioned the use of violence against women. The patriarch
is limited either in the object to which it is directed or by the territory within which it is to operate.
of a family was accorded the right to use force on members of the family under his control. I
quote the early studies:
The equal protection of the laws clause of the Constitution allows classification. Classification in
law, as in the other departments of knowledge or practice, is the grouping of things in
Traditions subordinating women have a long history rooted in patriarchy – the institutional rule of
speculation or practice because they agree with one another in certain particulars. A law is not
men. Women were seen in virtually all societies to be naturally inferior both physically and
invalid because of simple inequality. The very idea of classification is that of inequality, so that it
intellectually. In ancient Western societies, women whether slave, concubine or wife, were under
goes without saying that the mere fact of inequality in no manner determines the matter of
the authority of men. In law, they were treated as property.
constitutionality. All that is required of a valid classification is that it be reasonable, which means
that the classification should be based on substantial distinctions which make for real
differences; that it must be germane to the purpose of the law; that it must not be limited to The Roman concept of patria potestas allowed the husband to beat, or even kill, his wife if she
existing conditions only; and that it must apply equally to each member of the class. This Court endangered his property right over her. Judaism, Christianity and other religions oriented
has held that the standard is satisfied if the classification or distinction is based on a reasonable towards the patriarchal family strengthened the male dominated structure of society.
foundation or rational basis and is not palpably arbitrary. (Emphasis supplied)
English feudal law reinforced the tradition of male control over women. Even the eminent
Measured against the foregoing jurisprudential yardstick, we find that R.A. 9262 is based on a Blackstone has been quoted in his commentaries as saying husband and wife were one and that
valid classification as shall hereinafter be discussed and, as such, did not violate the equal one was the husband. However, in the late 1500s and through the entire 1600s, English
common law began to limit the right of husbands to chastise their wives. Thus, common law
developed the rule of thumb, which allowed husbands to beat their wives with a rod or stick no Finally in 1994, the United States Congress enacted the Violence Against Women Act.
thicker than their thumb.
In the International front, the women's struggle for equality was no less successful. The United
In the later part of the 19th century, legal recognition of these rights to chastise wives or inflict States Charter and the Universal Declaration of Human Rights affirmed the equality of all human
corporeal punishment ceased. Even then, the preservation of the family was given more beings. In 1979, the UN General Assembly adopted the landmark Convention on the Elimination
importance than preventing violence to women. of all Forms of Discrimination Against Women (CEDAW). In 1993, the UN General Assembly
also adopted the Declaration on the Elimination of Violence Against Women. World conferences
on the role and rights of women have been regularly held in Mexico City, Copenhagen, Nairobi
The metamorphosis of the law on violence in the United States followed that of the English
and Beijing. The UN itself established a Commission on the Status of Women.
common law. In 1871, the Supreme Court of Alabama became the first appellate court to strike
down the common law right of a husband to beat his wife:
The Philippines has been in cadence with the half – and full – steps of all these women's
movements. No less than Section 14, Article II of our 1987 Constitution mandates the State to
The privilege, ancient though it may be, to beat one's wife with a stick, to pull her hair, choke
recognize the role of women in nation building and to ensure the fundamental equality before the
her, spit in her face or kick her about the floor, or to inflict upon her like indignities, is not now
law of women and men. Our Senate has ratified the CEDAW as well as the Convention on the
acknowledged by our law... In person, the wife is entitled to the same protection of the law that
Rights of the Child and its two protocols. To cap it all, Congress, on March 8, 2004, enacted
the husband can invoke for himself.
Rep. Act No. 9262, entitled "An Act Defining Violence Against Women and Their Children,
Providing for Protective Measures for Victims, Prescribing Penalties therefor and for other
As time marched on, the women's advocacy movement became more organized. The Purposes." (Citations omitted)
temperance leagues initiated it. These leagues had a simple focus. They considered the evils of
alcoholism as the root cause of wife abuse. Hence, they demonstrated and picketed saloons,
B. Women are the "usual" and "most likely"
bars and their husbands' other watering holes. Soon, however, their crusade was joined by
suffragette movements, expanding the liberation movement's agenda. They fought for women's
right to vote, to own property, and more. Since then, the feminist movement was on the roll. victims of violence.

The feminist movement exposed the private invisibility of the domestic violence to the public At the time of the presentation of Senate Bill No. 2723, official statistics on violence against
gaze. They succeeded in transforming the issue into an important public concern. No less than women and children show that –
the United States Supreme Court, in 1992 case Planned Parenthood v. Casey, noted:
x x x physical injuries had the highest number of cases at 5,058 in 2002 representing 55.63% of
In an average 12-month period in this country, approximately two million women are the victims total cases reported (9,903). And for the first semester of 2003, there were 2,381 reported cases
of severe assaults by their male partners. In a 1985 survey, women reported that nearly one of out of 4,354 cases which represent 54.31%. xxx (T)he total number of women in especially
every eight husbands had assaulted their wives during the past year. The [American Medical difficult circumstances served by the Department of Social Welfare and Development (DSWD)
Association] views these figures as "marked underestimates," because the nature of these for the year 2002, there are 1,417 physically abused/maltreated cases out of the total of 5,608
incidents discourages women from reporting them, and because surveys typically exclude the cases. xxx (T)here are 1,091 DSWD cases out of a total number of 3,471 cases for the first
very poor, those who do not speak English well, and women who are homeless or in institutions semester of 2003. Female violence comprised more than 90% of all forms of abuse and violence
or hospitals when the survey is conducted. According to the AMA, "researchers on family and more than 90% of these reported cases were committed by the women's intimate partners
violence agree that the true incidence of partner violence is probably double the above such as their husbands and live-in partners.73
estimates; or four million severely assaulted women per year."
Recently, the Philippine Commission on Women presented comparative statistics on violence
Studies on prevalence suggest that from one-fifth to one-third of all women will be physically against women across an eight-year period from 2004 to August of 2011 with violations under
assaulted by a partner or ex-partner during their lifetime... Thus on an average day in the United R.A. 9262 ranking first among the different VAW categories since its implementation in
States, nearly 11,000 women are severely assaulted by their male partners. Many of these 2004,74 thus:
incidents involve sexual assault... In families where wife beating takes place, moreover, child
abuse is often present as well.
Table 1. Annual Comparative Statistics on Violence Against Women, 2004 - 2011*

Other studies fill in the rest of this troubling picture. Physical violence is only the most visible
form of abuse. Psychological abuse, particularly forced social and economic isolation of women,
is also common. Reported
2004 2005 2006 2007 2008 2009 2010 2011
Cases

Many victims of domestic violence remain with their abusers, perhaps because they perceive no
superior alternative...Many abused women who find temporary refuge in shelters return to their Rape 997 927 659 837 811 770 1,042 832
husbands, in large part because they have no other source of income... Returning to one's
abuser can be dangerous. Recent Federal Bureau of Investigation statistics disclose that 8.8
percent of all homicide victims in the United States are killed by their spouses...Thirty percent of
female homicide victims are killed by their male partners.
because many men will not even attempt to report the situation. In the United Kingdom, 32% of
Incestuous women who had ever experienced domestic violence did so four or five (or more) times,
38 46 26 22 28 27 19 23
Rape compared with 11% of the smaller number of men who had ever experienced domestic violence;
and women constituted 89% of all those who had experienced 4 or more incidents of domestic
violence.75Statistics in Canada show that spousal violence by a woman against a man is less
Attempted likely to cause injury than the other way around (18 percent versus 44 percent). Men, who
194 148 185 147 204 167 268 201
Rape experience violence from their spouses are much less likely to live in fear of violence at the
hands of their spouses, and much less likely to experience sexual assault. In fact, many cases of
physical violence by a woman against a spouse are in self-defense or the result of many years
Acts of
of physical or emotional abuse.76
580 536 382 358 445 485 745 625
Lasciviousness

While there are, indeed, relatively few cases of violence and abuse perpetrated against men in
the Philippines, the same cannot render R.A. 9262 invalid.
Physical
3,553 2,335 1,892 1,505 1,307 1,498 2,018 1,588
Injuries
In a 1960 case involving the violation of a city ordinance requiring drivers of animal-drawn
vehicles to pick up, gather and deposit in receptacles the manure emitted or discharged by their
vehicle-drawing animals in any public highways, streets, plazas, parks or alleys, said ordinance
Sexual
Harassment
53 37 38 46 18 54 83 63 was challenged as violative of the guaranty of equal protection of laws as its application is
limited to owners and drivers of vehicle-drawing animals and not to those animals, although not
utilized, but similarly pass through the same streets.
RA 9262 218 924 1,269 2,387 3,599 5,285 9,974 9,021
The ordinance was upheld as a valid classification for the reason that, while there may be non-
vehicle-drawing animals that also traverse the city roads, "but their number must be negligible
and their appearance therein merely occasional, compared to the rig-drawing ones, as not to
Threats 319 223 199 182 220 208 374 213
constitute a menace to the health of the community."77 The mere fact that the legislative
classification may result in actual inequality is not violative of the right to equal protection, for
every classification of persons or things for regulation by law produces inequality in some
Seduction 62 19 29 30 19 19 25 15 degree, but the law is not thereby rendered invalid.78

C. Gender bias and prejudices


Concubinage 121 102 93 109 109 99 158 128

From the initial report to the police through prosecution, trial, and sentencing, crimes against
women are often treated differently and less seriously than other crimes. This was argued by
RA 9208 17 11 16 24 34 152 190 62
then United States Senator Joseph R. Biden, Jr., now Vice President, chief sponsor of the
Violence Against Women Act (VAWA), in defending the civil rights remedy as a valid exercise of
the U.S. Congress' authority under the Commerce and Equal Protection Clauses. He stressed
Abduction
16 34 23 28 18 25 22
that the widespread gender bias in the U.S. has institutionalized historic prejudices against
/Kidnapping 29 victims of rape or domestic violence, subjecting them to "double victimization" – first at the hands
of the offender and then of the legal system.79

Unjust
Vexation
90 50 59 59 83 703 183 155 Our own Senator Loi Estrada lamented in her Sponsorship Speech for Senate Bill No. 2723 that
"(w)henever violence occurs in the family, the police treat it as a private matter and advise the
parties to settle the conflict themselves. Once the complainant brings the case to the prosecutor,
the latter is hesitant to file the complaint for fear that it might later be withdrawn. This lack of
Total 6,271 5,374 4,881 5,729 6,905 9,485 15,104 12,948 response or reluctance to be involved by the police and prosecution reinforces the escalating,
recurring and often serious nature of domestic violence."80

*2011 report covers only from January to August Sadly, our own courts, as well, have exhibited prejudices and biases against our women.

Source: Philippine National Police – Women and Children Protection Center (WCPC) In a recent case resolved on March 9, 2011, we fined RTC Judge Venancio J. Amila for Conduct
Unbecoming of a Judge. He used derogatory and irreverent language in reference to the
complainant in a petition for TPO and PPO under R.A. 9262, calling her as "only a live-in
On the other hand, no reliable estimates may be obtained on domestic abuse and violence partner" and presenting her as an "opportunist" and a "mistress" in an "illegitimate relationship."
against men in the Philippines because incidents thereof are relatively low and, perhaps,
Judge Amila even called her a "prostitute," and accused her of being motivated by "insatiable R.A. 9262 applies equally to all women and children who suffer violence and abuse. Section 3
greed" and of absconding with the contested property.81 Such remarks betrayed Judge Amila's thereof defines VAWC as:
prejudices and lack of gender sensitivity.
x x x any act or a series of acts committed by any person against a woman who is his wife,
The enactment of R.A. 9262 aims to address the discrimination brought about by biases and former wife, or against a woman with whom the person has or had a sexual or dating
prejudices against women. As emphasized by the CEDAW Committee on the Elimination of relationship, or with whom he has a common child, or against her child whether legitimate or
Discrimination against Women, addressing or correcting discrimination through specific illegitimate, within or without the family abode, which result in or is likely to result in physical,
measures focused on women does not discriminate against men. 82Petitioner's sexual, psychological harm or suffering, or economic abuse including threats of such acts,
contention,83 therefore, that R.A. 9262 is discriminatory and that it is an "anti-male," "husband- battery, assault, coercion, harassment or arbitrary deprivation of liberty. It includes, but is not
bashing," and "hate-men" law deserves scant consideration. As a State Party to the CEDAW, limited to, the following acts:
the Philippines bound itself to take all appropriate measures "to modify the social and cultural
patterns of conduct of men and women, with a view to achieving the elimination of prejudices
A. "Physical Violence" refers to acts that include bodily or physical harm;
and customary and all other practices which are based on the idea of the inferiority or the
superiority of either of the sexes or on stereotyped roles for men and women." 84 Justice Puno
correctly pointed out that "(t)he paradigm shift changing the character of domestic violence from B. "Sexual violence" refers to an act which is sexual in nature, committed against a woman or
a private affair to a public offense will require the development of a distinct mindset on the part of her child. It includes, but is not limited to:
the police, the prosecution and the judges."85
a) rape, sexual harassment, acts of lasciviousness, treating a woman or her
II. The classification is germane to the purpose of the law. child as a sex object, making demeaning and sexually suggestive remarks,
physically attacking the sexual parts of the victim's body, forcing her/him to
watch obscene publications and indecent shows or forcing the woman or
The distinction between men and women is germane to the purpose of R.A. 9262, which is to
her child to do indecent acts and/or make films thereof, forcing the wife and
address violence committed against women and children, spelled out in its Declaration of Policy,
mistress/lover to live in the conjugal home or sleep together in the same
as follows:
room with the abuser;

SEC. 2. Declaration of Policy. – It is hereby declared that the State values the dignity of women
b) acts causing or attempting to cause the victim to engage in any sexual
and children and guarantees full respect for human rights. The State also recognizes the need to
activity by force, threat of force, physical or other harm or threat of physical
protect the family and its members particularly women and children, from violence and threats to
or other harm or coercion;
their personal safety and security.

c) Prostituting the woman or child.


Towards this end, the State shall exert efforts to address violence committed against women
and children in keeping with the fundamental freedoms guaranteed under the Constitution and
the provisions of the Universal Declaration of Human Rights, the Convention on the Elimination C. "Psychological violence" refers to acts or omissions causing or likely to cause mental or
of All Forms of Discrimination Against Women, Convention on the Rights of the Child and other emotional suffering of the victim such as but not limited to intimidation, harassment, stalking,
international human rights instruments of which the Philippines is a party. damage to property, public ridicule or humiliation, repeated verbal abuse and marital infidelity. It
includes causing or allowing the victim to witness the physical, sexual or psychological abuse of
a member of the family to which the victim belongs, or to witness pornography in any form or to
In 1979, the U.N. General Assembly adopted the CEDAW, which the Philippines ratified on
witness abusive injury to pets or to unlawful or unwanted deprivation of the right to custody
August 5, 1981. Subsequently, the Optional Protocol to the CEDAW was also ratified by the
and/or visitation of common children.
Philippines on October 6, 2003.86 This Convention mandates that State parties shall accord to
women equality with men before the law87 and shall take all appropriate measures to eliminate
discrimination against women in all matters relating to marriage and family relations on the basis D. "Economic abuse" refers to acts that make or attempt to make a woman financially dependent
of equality of men and women.88 The Philippines likewise ratified the Convention on the Rights which includes, but is not limited to the following:
of the Child and its two protocols.89 It is, thus, bound by said Conventions and their respective
protocols.
1. withdrawal of financial support or preventing the victim from engaging in
any legitimate profession, occupation, business or activity, except in cases
III. The classification is not limited to existing wherein the other spouse/partner objects on valid, serious and moral
grounds as defined in Article 73 of the Family Code;
conditions only, and apply equally to all members
2. deprivation or threat of deprivation of financial resources and the right to
the use and enjoyment of the conjugal, community or property owned in
Moreover, the application of R.A. 9262 is not limited to the existing conditions when it was
common;
promulgated, but to future conditions as well, for as long as the safety and security of women
and their children are threatened by violence and abuse.
3. destroying household property;
4. controlling the victims' own money or properties or solely controlling the designated family or household member safety in the family residence, and to prevent the
conjugal money or properties. perpetrator from committing acts that jeopardize the employment and support of the victim. It
also enables the court to award temporary custody of minor children to protect the children from
violence, to prevent their abduction by the perpetrator and to ensure their financial support." 97
It should be stressed that the acts enumerated in the aforequoted provision are attributable to
research that has exposed the dimensions and dynamics of battery. The acts described here are
also found in the U.N. Declaration on the Elimination of Violence Against Women. 90 Hence, the The rules require that petitions for protection order be in writing, signed and verified by the
argument advanced by petitioner that the definition of what constitutes abuse removes the petitioner98 thereby undertaking full responsibility, criminal or civil, for every allegation therein.
difference between violent action and simple marital tiffs is tenuous. Since "time is of the essence in cases of VAWC if further violence is to be prevented," 99 the
court is authorized to issue ex parte a TPO after raffle but before notice and hearing when the
life, limb or property of the victim is in jeopardy and there is reasonable ground to believe that
There is nothing in the definition of VAWC that is vague and ambiguous that will confuse
the order is necessary to protect the victim from the immediate and imminent danger of VAWC
petitioner in his defense. The acts enumerated above are easily understood and provide
or to prevent such violence, which is about to recur.100
adequate contrast between the innocent and the prohibited acts. They are worded with sufficient
definiteness that persons of ordinary intelligence can understand what conduct is prohibited, and
need not guess at its meaning nor differ in its application.91 Yet, petitioner insists92that phrases There need not be any fear that the judge may have no rational basis to issue an ex parte order.
like "depriving or threatening to deprive the woman or her child of a legal right," "solely The victim is required not only to verify the allegations in the petition, but also to attach her
controlling the conjugal or common money or properties," "marital infidelity," and "causing witnesses' affidavits to the petition.101
mental or emotional anguish" are so vague that they make every quarrel a case of spousal
abuse. However, we have stressed that the "vagueness" doctrine merely requires a reasonable
The grant of a TPO ex parte cannot, therefore, be challenged as violative of the right to due
degree of certainty for the statute to be upheld – not absolute precision or mathematical
process. Just like a writ of preliminary attachment which is issued without notice and hearing
exactitude, as petitioner seems to suggest. Flexibility, rather than meticulous specificity, is
because the time in which the hearing will take could be enough to enable the defendant to
permissible as long as the metes and bounds of the statute are clearly delineated. An act will not
abscond or dispose of his property,102 in the same way, the victim of VAWC may already have
be held invalid merely because it might have been more explicit in its wordings or detailed in its
suffered harrowing experiences in the hands of her tormentor, and possibly even death, if notice
provisions.93
and hearing were required before such acts could be prevented. It is a constitutional
commonplace that the ordinary requirements of procedural due process must yield to the
There is likewise no merit to the contention that R.A. 9262 singles out the husband or father as necessities of protecting vital public interests,103among which is protection of women and
the culprit. As defined above, VAWC may likewise be committed "against a woman with whom children from violence and threats to their personal safety and security.
the person has or had a sexual or dating relationship." Clearly, the use of the gender-neutral
word "person" who has or had a sexual or dating relationship with the woman encompasses
It should be pointed out that when the TPO is issued ex parte, the court shall likewise order that
even lesbian relationships. Moreover, while the law provides that the offender be related or
notice be immediately given to the respondent directing him to file an opposition within five (5)
connected to the victim by marriage, former marriage, or a sexual or dating relationship, it does
days from service. Moreover, the court shall order that notice, copies of the petition and TPO be
not preclude the application of the principle of conspiracy under the Revised Penal Code (RPC).
served immediately on the respondent by the court sheriffs. The TPOs are initially effective for
Thus, in the case of Go-Tan v. Spouses Tan,94 the parents-in-law of Sharica Mari L. Go-Tan, the
thirty (30) days from service on the respondent.104
victim, were held to be proper respondents in the case filed by the latter upon the allegation that
they and their son (Go-Tan's husband) had community of design and purpose in tormenting her
by giving her insufficient financial support; harassing and pressuring her to be ejected from the Where no TPO is issued ex parte, the court will nonetheless order the immediate issuance and
family home; and in repeatedly abusing her verbally, emotionally, mentally and physically. service of the notice upon the respondent requiring him to file an opposition to the petition within
five (5) days from service. The date of the preliminary conference and hearing on the merits
shall likewise be indicated on the notice.105
R.A. 9262 is not violative of the
due process clause of the Constitution.
The opposition to the petition which the respondent himself shall verify, must be accompanied by
the affidavits of witnesses and shall show cause why a temporary or permanent protection order
Petitioner bewails the disregard of R.A. 9262, specifically in the issuance of POs, of all
should not be issued.106
protections afforded by the due process clause of the Constitution. Says he: "On the basis of
unsubstantiated allegations, and practically no opportunity to respond, the husband is stripped of
family, property, guns, money, children, job, future employment and reputation, all in a matter of It is clear from the foregoing rules that the respondent of a petition for protection order should be
seconds, without an inkling of what happened."95 apprised of the charges imputed to him and afforded an opportunity to present his side. Thus,
the fear of petitioner of being "stripped of family, property, guns, money, children, job, future
employment and reputation, all in a matter of seconds, without an inkling of what happened" is a
A protection order is an order issued to prevent further acts of violence against women and their
mere product of an overactive imagination. The essence of due process is to be found in the
children, their family or household members, and to grant other necessary reliefs. Its purpose is
reasonable opportunity to be heard and submit any evidence one may have in support of one's
to safeguard the offended parties from further harm, minimize any disruption in their daily life
defense. "To be heard" does not only mean verbal arguments in court; one may be heard also
and facilitate the opportunity and ability to regain control of their life. 96
through pleadings. Where opportunity to be heard, either through oral arguments or pleadings, is
accorded, there is no denial of procedural due process.107
"The scope of reliefs in protection orders is broadened to ensure that the victim or offended party
is afforded all the remedies necessary to curtail access by a perpetrator to the victim. This
serves to safeguard the victim from greater risk of violence; to accord the victim and any
It should be recalled that petitioner filed on April 26, 2006 an Opposition to the Urgent Ex-Parte not a subject for compromise. A process which involves parties mediating the issue of violence
Motion for Renewal of the TPO that was granted only two days earlier on April 24, 2006. implies that the victim is somehow at fault. In addition, mediation of issues in a proceeding for an
Likewise, on May 23, 2006, petitioner filed a motion for the modification of the TPO to allow him order of protection is problematic because the petitioner is frequently unable to participate
visitation rights to his children. Still, the trial court in its Order dated September 26, 2006, gave equally with the person against whom the protection order has been sought. (Emphasis
him five days (5) within which to show cause why the TPO should not be renewed or extended. supplied)
Yet, he chose not to file the required comment arguing that it would just be an "exercise in
futility," conveniently forgetting that the renewal of the questioned TPO was only for a limited
There is no undue delegation of
period (30 days) each time, and that he could prevent the continued renewal of said order if he
judicial power to barangay officials.
can show sufficient cause therefor. Having failed to do so, petitioner may not now be heard to
complain that he was denied due process of law.
Petitioner contends that protection orders involve the exercise of judicial power which, under the
Constitution, is placed upon the "Supreme Court and such other lower courts as may be
Petitioner next laments that the removal and exclusion of the respondent in the VAWC case from
established by law" and, thus, protests the delegation of power to barangay officials to issue
the residence of the victim, regardless of ownership of the residence, is virtually a "blank check"
protection orders.111 The pertinent provision reads, as follows:
issued to the wife to claim any property as her conjugal home.108

SEC. 14. Barangay Protection Orders (BPOs); Who May Issue and How. – Barangay Protection
The wording of the pertinent rule, however, does not by any stretch of the imagination suggest
Orders (BPOs) refer to the protection order issued by the Punong Barangay ordering the
that this is so. It states:
perpetrator to desist from committing acts under Section 5 (a) and (b) of this Act.1âwphi1 A
Punong Barangay who receives applications for a BPO shall issue the protection order to the
SEC. 11. Reliefs available to the offended party. -- The protection order shall include any, some applicant on the date of filing after ex parte determination of the basis of the application. If the
or all of the following reliefs: Punong Barangay is unavailable to act on the application for a BPO, the application shall be
acted upon by any available Barangay Kagawad. If the BPO is issued by a Barangay Kagawad,
the order must be accompanied by an attestation by the Barangay Kagawad that the Punong
xxxx
Barangay was unavailable at the time of the issuance of the BPO. BPOs shall be effective for
fifteen (15) days. Immediately after the issuance of an ex parte BPO, the Punong Barangay or
(c) Removing and excluding the respondent from the residence of the offended party, regardless Barangay Kagawad shall personally serve a copy of the same on the respondent, or direct any
of ownership of the residence, either temporarily for the purpose of protecting the offended party, barangay official to effect its personal service.
or permanently where no property rights are violated. If the respondent must remove personal
effects from the residence, the court shall direct a law enforcement agent to accompany the
The parties may be accompanied by a non-lawyer advocate in any proceeding before the
respondent to the residence, remain there until the respondent has gathered his things and
Punong Barangay.
escort him from the residence;

Judicial power includes the duty of the courts of justice to settle actual controversies involving
xxxx
rights which are legally demandable and enforceable, and to determine whether or not there has
been a grave abuse of discretion amounting to lack or excess of jurisdiction on the part of any
Indubitably, petitioner may be removed and excluded from private respondent's residence, branch or instrumentality of the Government.112 On the other hand, executive power "is generally
regardless of ownership, only temporarily for the purpose of protecting the latter. Such removal defined as the power to enforce and administer the laws. It is the power of carrying the laws into
and exclusion may be permanent only where no property rights are violated. How then can the practical operation and enforcing their due observance."113
private respondent just claim any property and appropriate it for herself, as petitioner seems to
suggest?
As clearly delimited by the aforequoted provision, the BPO issued by the Punong Barangay or,
in his unavailability, by any available Barangay Kagawad, merely orders the perpetrator to desist
The non-referral of a VAWC case from (a) causing physical harm to the woman or her child; and (2) threatening to cause the
to a mediator is justified. woman or her child physical harm. Such function of the Punong Barangay is, thus, purely
executive in nature, in pursuance of his duty under the Local Government Code to "enforce all
laws and ordinances," and to "maintain public order in the barangay."114
Petitioner argues that "by criminalizing run-of-the-mill arguments, instead of encouraging
mediation and counseling, the law has done violence to the avowed policy of the State to
"protect and strengthen the family as a basic autonomous social institution."109 We have held that "(t)he mere fact that an officer is required by law to inquire into the existence
of certain facts and to apply the law thereto in order to determine what his official conduct shall
be and the fact that these acts may affect private rights do not constitute an exercise of judicial
Under Section 23(c) of A.M. No. 04-10-11-SC, the court shall not refer the case or any issue powers."115
thereof to a mediator. The reason behind this provision is well-explained by the Commentary on
Section 311 of the Model Code on Domestic and Family Violence as follows: 110
In the same manner as the public prosecutor ascertains through a preliminary inquiry or
proceeding "whether there is reasonable ground to believe that an offense has been committed
This section prohibits a court from ordering or referring parties to mediation in a proceeding for
and the accused is probably guilty thereof," the Punong Barangay must determine reasonable
an order for protection. Mediation is a process by which parties in equivalent bargaining ground to believe that an imminent danger of violence against the woman and her children exists
positions voluntarily reach consensual agreement about the issue at hand. Violence, however, is
or is about to recur that would necessitate the issuance of a BPO. The preliminary investigation
conducted by the prosecutor is, concededly, an executive, not a judicial, function. The same
holds true with the issuance of a BPO.

We need not even belabor the issue raised by petitioner that since barangay officials and other
law enforcement agencies are required to extend assistance to victims of violence and abuse, it
would be very unlikely that they would remain objective and impartial, and that the chances of
acquittal are nil. As already stated, assistance by barangay officials and other law enforcement
agencies is consistent with their duty to enforce the law and to maintain peace and order.

Conclusion

Before a statute or its provisions duly challenged are voided, an unequivocal breach of, or a
clear conflict with the Constitution, not merely a doubtful or argumentative one, must be
demonstrated in such a manner as to leave no doubt in the mind of the Court. In other words,
the grounds for nullity must be beyond reasonable doubt.116 In the instant case, however, no
concrete evidence and convincing arguments were presented by petitioner to warrant a
declaration of the unconstitutionality of R.A. 9262, which is an act of Congress and signed into
law by the highest officer of the co-equal executive department. As we said in Estrada v.
Sandiganbayan, 117 courts must assume that the legislature is ever conscious of the borders and
edges of its plenary powers, and passed laws with full knowledge of the facts and for the
purpose of promoting what is right and advancing the welfare of the majority.

We reiterate here Justice Puno's observation that "the history of the women's movement against
domestic violence shows that one of its most difficult struggles was the fight against the violence
of law itself. If we keep that in mind, law will not again be a hindrance to the struggle of women
for equality but will be its fulfillment."118 Accordingly, the constitutionality of R.A. 9262 is, as it
should be, sustained.

WHEREFORE, the instant petition for review on certiorari is hereby DENIED for lack of merit.

SO ORDERED.
G.R. No. 101083 July 30, 1993 Philippines, taxpayers, and entitled to the full benefit, use and enjoyment of the natural resource
treasure that is the country's virgin tropical forests." The same was filed for themselves and
others who are equally concerned about the preservation of said resource but are "so numerous
JUAN ANTONIO, ANNA ROSARIO and JOSE ALFONSO, all surnamed OPOSA, minors,
that it is impracticable to bring them all before the Court." The minors further asseverate that
and represented by their parents ANTONIO and RIZALINA OPOSA, ROBERTA NICOLE
they "represent their generation as well as generations yet unborn."4 Consequently, it is prayed
SADIUA, minor, represented by her parents CALVIN and ROBERTA SADIUA, CARLO,
for that judgment be rendered:
AMANDA SALUD and PATRISHA, all surnamed FLORES, minors and represented by their
parents ENRICO and NIDA FLORES, GIANINA DITA R. FORTUN, minor, represented by
her parents SIGRID and DOLORES FORTUN, GEORGE II and MA. CONCEPCION, all . . . ordering defendant, his agents, representatives and other persons
surnamed MISA, minors and represented by their parents GEORGE and MYRA MISA, acting in his behalf to —
BENJAMIN ALAN V. PESIGAN, minor, represented by his parents ANTONIO and ALICE
PESIGAN, JOVIE MARIE ALFARO, minor, represented by her parents JOSE and MARIA
(1) Cancel all existing timber license agreements in the country;
VIOLETA ALFARO, MARIA CONCEPCION T. CASTRO, minor, represented by her parents
FREDENIL and JANE CASTRO, JOHANNA DESAMPARADO,
minor, represented by her parents JOSE and ANGELA DESAMPRADO, CARLO JOAQUIN (2) Cease and desist from receiving, accepting, processing, renewing or
T. NARVASA, minor, represented by his parents GREGORIO II and CRISTINE CHARITY approving new timber license agreements.
NARVASA, MA. MARGARITA, JESUS IGNACIO, MA. ANGELA and MARIE GABRIELLE, all
surnamed SAENZ, minors, represented by their parents ROBERTO and AURORA SAENZ,
KRISTINE, MARY ELLEN, MAY, GOLDA MARTHE and DAVID IAN, all surnamed KING, and granting the plaintiffs ". . . such other reliefs just and equitable under the premises." 5
minors, represented by their parents MARIO and HAYDEE KING, DAVID, FRANCISCO and
THERESE VICTORIA, all surnamed ENDRIGA, minors, represented by their parents The complaint starts off with the general averments that the Philippine archipelago of 7,100
BALTAZAR and TERESITA ENDRIGA, JOSE MA. and REGINA MA., all surnamed ABAYA, islands has a land area of thirty million (30,000,000) hectares and is endowed with rich, lush and
minors, represented by their parents ANTONIO and MARICA ABAYA, MARILIN, MARIO, verdant rainforests in which varied, rare and unique species of flora and fauna may be found;
JR. and MARIETTE, all surnamed CARDAMA, minors, represented by their parents MARIO these rainforests contain a genetic, biological and chemical pool which is irreplaceable; they are
and LINA CARDAMA, CLARISSA, ANN MARIE, NAGEL, and IMEE LYN, all surnamed also the habitat of indigenous Philippine cultures which have existed, endured and flourished
OPOSA, minors and represented by their parents RICARDO and MARISSA OPOSA, since time immemorial; scientific evidence reveals that in order to maintain a balanced and
PHILIP JOSEPH, STEPHEN JOHN and ISAIAH JAMES, all surnamed QUIPIT, minors, healthful ecology, the country's land area should be utilized on the basis of a ratio of fifty-four per
represented by their parents JOSE MAX and VILMI QUIPIT, BUGHAW CIELO, CRISANTO, cent (54%) for forest cover and forty-six per cent (46%) for agricultural, residential, industrial,
ANNA, DANIEL and FRANCISCO, all surnamed BIBAL, minors, represented by their commercial and other uses; the distortion and disturbance of this balance as a consequence of
parents FRANCISCO, JR. and MILAGROS BIBAL, and THE PHILIPPINE ECOLOGICAL deforestation have resulted in a host of environmental tragedies, such as (a) water shortages
NETWORK, INC., petitioners, resulting from drying up of the water table, otherwise known as the "aquifer," as well as of rivers,
vs. brooks and streams, (b) salinization of the water table as a result of the intrusion therein of salt
THE HONORABLE FULGENCIO S. FACTORAN, JR., in his capacity as the Secretary of the water, incontrovertible examples of which may be found in the island of Cebu and the
Department of Environment and Natural Resources, and THE HONORABLE ERIBERTO U. Municipality of Bacoor, Cavite, (c) massive erosion and the consequential loss of soil fertility and
ROSARIO, Presiding Judge of the RTC, Makati, Branch 66, respondents. agricultural productivity, with the volume of soil eroded estimated at one billion (1,000,000,000)
cubic meters per annum — approximately the size of the entire island of Catanduanes, (d) the
DAVIDE, JR., J.: endangering and extinction of the country's unique, rare and varied flora and fauna, (e) the
disturbance and dislocation of cultural communities, including the disappearance of the Filipino's
indigenous cultures, (f) the siltation of rivers and seabeds and consequential destruction of
In a broader sense, this petition bears upon the right of Filipinos to a balanced and healthful corals and other aquatic life leading to a critical reduction in marine resource productivity, (g)
ecology which the petitioners dramatically associate with the twin concepts of "inter-generational recurrent spells of drought as is presently experienced by the entire country, (h) increasing
responsibility" and "inter-generational justice." Specifically, it touches on the issue of whether the velocity of typhoon winds which result from the absence of windbreakers, (i) the floodings of
said petitioners have a cause of action to "prevent the misappropriation or impairment" of lowlands and agricultural plains arising from the absence of the absorbent mechanism of forests,
Philippine rainforests and "arrest the unabated hemorrhage of the country's vital life support (j) the siltation and shortening of the lifespan of multi-billion peso dams constructed and
systems and continued rape of Mother Earth." operated for the purpose of supplying water for domestic uses, irrigation and the generation of
electric power, and (k) the reduction of the earth's capacity to process carbon dioxide gases
which has led to perplexing and catastrophic climatic changes such as the phenomenon of
The controversy has its genesis in Civil Case No. 90-77 which was filed before Branch 66
global warming, otherwise known as the "greenhouse effect."
(Makati, Metro Manila) of the Regional Trial Court (RTC), National Capital Judicial Region. The
principal plaintiffs therein, now the principal petitioners, are all minors duly represented and
joined by their respective parents. Impleaded as an additional plaintiff is the Philippine Ecological Plaintiffs further assert that the adverse and detrimental consequences of continued and
Network, Inc. (PENI), a domestic, non-stock and non-profit corporation organized for the deforestation are so capable of unquestionable demonstration that the same may be submitted
purpose of, inter alia, engaging in concerted action geared for the protection of our environment as a matter of judicial notice. This notwithstanding, they expressed their intention to present
and natural resources. The original defendant was the Honorable Fulgencio S. Factoran, Jr., expert witnesses as well as documentary, photographic and film evidence in the course of the
then Secretary of the Department of Environment and Natural Resources (DENR). His trial.
substitution in this petition by the new Secretary, the Honorable Angel C. Alcala, was
subsequently ordered upon proper motion by the petitioners.1 The complaint2 was instituted as a
As their cause of action, they specifically allege that:
taxpayers' class suit3 and alleges that the plaintiffs "are all citizens of the Republic of the
CAUSE OF ACTION 16. Plaintiff have exhausted all administrative remedies with the defendant's
office. On March 2, 1990, plaintiffs served upon defendant a final demand to
cancel all logging permits in the country.
7. Plaintiffs replead by reference the foregoing allegations.

A copy of the plaintiffs' letter dated March 1, 1990 is hereto attached as


8. Twenty-five (25) years ago, the Philippines had some sixteen (16) million
Annex "B".
hectares of rainforests constituting roughly 53% of the country's land mass.

17. Defendant, however, fails and refuses to cancel the existing TLA's to the
9. Satellite images taken in 1987 reveal that there remained no more than
continuing serious damage and extreme prejudice of plaintiffs.
1.2 million hectares of said rainforests or four per cent (4.0%) of the
country's land area.
18. The continued failure and refusal by defendant to cancel the TLA's is an
act violative of the rights of plaintiffs, especially plaintiff minors who may be
10. More recent surveys reveal that a mere 850,000 hectares of virgin old-
left with a country that is desertified (sic), bare, barren and devoid of the
growth rainforests are left, barely 2.8% of the entire land mass of the
wonderful flora, fauna and indigenous cultures which the Philippines had
Philippine archipelago and about 3.0 million hectares of immature and
been abundantly blessed with.
uneconomical secondary growth forests.

19. Defendant's refusal to cancel the aforementioned TLA's is manifestly


11. Public records reveal that the defendant's, predecessors have granted
contrary to the public policy enunciated in the Philippine Environmental
timber license agreements ('TLA's') to various corporations to cut the
Policy which, in pertinent part, states that it is the policy of the State —
aggregate area of 3.89 million hectares for commercial logging purposes.

(a) to create, develop, maintain and improve conditions under which man
A copy of the TLA holders and the corresponding areas covered is hereto
and nature can thrive in productive and enjoyable harmony with each other;
attached as Annex "A".

(b) to fulfill the social, economic and other requirements of present and
12. At the present rate of deforestation, i.e. about 200,000 hectares per
future generations of Filipinos and;
annum or 25 hectares per hour — nighttime, Saturdays, Sundays and
holidays included — the Philippines will be bereft of forest resources after
the end of this ensuing decade, if not earlier. (c) to ensure the attainment of an environmental quality that is conductive to
a life of dignity and well-being. (P.D. 1151, 6 June 1977)
13. The adverse effects, disastrous consequences, serious injury and
irreparable damage of this continued trend of deforestation to the plaintiff 20. Furthermore, defendant's continued refusal to cancel the
minor's generation and to generations yet unborn are evident and aforementioned TLA's is contradictory to the Constitutional policy of the
incontrovertible. As a matter of fact, the environmental damages State to —
enumerated in paragraph 6 hereof are already being felt, experienced and
suffered by the generation of plaintiff adults.
a. effect "a more equitable distribution of opportunities, income and wealth"
and "make full and efficient use of natural resources (sic)." (Section 1,
14. The continued allowance by defendant of TLA holders to cut and Article XII of the Constitution);
deforest the remaining forest stands will work great damage and irreparable
injury to plaintiffs — especially plaintiff minors and their successors — who
b. "protect the nation's marine wealth." (Section 2, ibid);
may never see, use, benefit from and enjoy this rare and unique natural
resource treasure.
c. "conserve and promote the nation's cultural heritage and resources (sic)"
(Section 14, Article XIV, id.);
This act of defendant constitutes a misappropriation and/or impairment of
the natural resource property he holds in trust for the benefit of plaintiff
minors and succeeding generations. d. "protect and advance the right of the people to a balanced and healthful
ecology in accord with the rhythm and harmony of nature." (Section 16,
Article II, id.)
15. Plaintiffs have a clear and constitutional right to a balanced and healthful
ecology and are entitled to protection by the State in its capacity as
the parens patriae. 21. Finally, defendant's act is contrary to the highest law of humankind —
the natural law — and violative of plaintiffs' right to self-preservation and
perpetuation.
22. There is no other plain, speedy and adequate remedy in law other than in the country is a political question which should be properly addressed to the executive or
the instant action to arrest the unabated hemorrhage of the country's vital legislative branches of Government. They therefore assert that the petitioners' resources is not
life support systems and continued rape of Mother Earth. 6 to file an action to court, but to lobby before Congress for the passage of a bill that would ban
logging totally.
On 22 June 1990, the original defendant, Secretary Factoran, Jr., filed a Motion to Dismiss the
complaint based on two (2) grounds, namely: (1) the plaintiffs have no cause of action against As to the matter of the cancellation of the TLAs, respondents submit that the same cannot be
him and (2) the issue raised by the plaintiffs is a political question which properly pertains to the done by the State without due process of law. Once issued, a TLA remains effective for a certain
legislative or executive branches of Government. In their 12 July 1990 Opposition to the Motion, period of time — usually for twenty-five (25) years. During its effectivity, the same can neither be
the petitioners maintain that (1) the complaint shows a clear and unmistakable cause of action, revised nor cancelled unless the holder has been found, after due notice and hearing, to have
(2) the motion is dilatory and (3) the action presents a justiciable question as it involves the violated the terms of the agreement or other forestry laws and regulations. Petitioners'
defendant's abuse of discretion. proposition to have all the TLAs indiscriminately cancelled without the requisite hearing would be
violative of the requirements of due process.
On 18 July 1991, respondent Judge issued an order granting the aforementioned motion to
dismiss.7 In the said order, not only was the defendant's claim — that the complaint states no Before going any further, We must first focus on some procedural matters. Petitioners instituted
cause of action against him and that it raises a political question — sustained, the respondent Civil Case No. 90-777 as a class suit. The original defendant and the present respondents did
Judge further ruled that the granting of the relief prayed for would result in the impairment of not take issue with this matter. Nevertheless, We hereby rule that the said civil case is indeed a
contracts which is prohibited by the fundamental law of the land. class suit. The subject matter of the complaint is of common and general interest not just to
several, but to all citizens of the Philippines. Consequently, since the parties are so numerous, it,
becomes impracticable, if not totally impossible, to bring all of them before the court. We likewise
Plaintiffs thus filed the instant special civil action for certiorari under Rule 65 of the Revised
declare that the plaintiffs therein are numerous and representative enough to ensure the full
Rules of Court and ask this Court to rescind and set aside the dismissal order on the ground that
protection of all concerned interests. Hence, all the requisites for the filing of a valid class suit
the respondent Judge gravely abused his discretion in dismissing the action. Again, the parents
under Section 12, Rule 3 of the Revised Rules of Court are present both in the said civil case
of the plaintiffs-minors not only represent their children, but have also joined the latter in this
and in the instant petition, the latter being but an incident to the former.
case.8

This case, however, has a special and novel element. Petitioners minors assert that they
On 14 May 1992, We resolved to give due course to the petition and required the parties to
represent their generation as well as generations yet unborn. We find no difficulty in ruling that
submit their respective Memoranda after the Office of the Solicitor General (OSG) filed a
they can, for themselves, for others of their generation and for the succeeding generations, file a
Comment in behalf of the respondents and the petitioners filed a reply thereto.
class suit. Their personality to sue in behalf of the succeeding generations can only be based on
the concept of intergenerational responsibility insofar as the right to a balanced and healthful
Petitioners contend that the complaint clearly and unmistakably states a cause of action as it ecology is concerned. Such a right, as hereinafter expounded, considers
contains sufficient allegations concerning their right to a sound environment based on Articles the "rhythm and harmony of nature." Nature means the created world in its entirety.9 Such
19, 20 and 21 of the Civil Code (Human Relations), Section 4 of Executive Order (E.O.) No. 192 rhythm and harmony indispensably include, inter alia, the judicious disposition, utilization,
creating the DENR, Section 3 of Presidential Decree (P.D.) No. 1151 (Philippine Environmental management, renewal and conservation of the country's forest, mineral, land, waters, fisheries,
Policy), Section 16, Article II of the 1987 Constitution recognizing the right of the people to a wildlife, off-shore areas and other natural resources to the end that their exploration,
balanced and healthful ecology, the concept of generational genocide in Criminal Law and the development and utilization be equitably accessible to the present as well as future
concept of man's inalienable right to self-preservation and self-perpetuation embodied in natural generations. 10Needless to say, every generation has a responsibility to the next to preserve that
law. Petitioners likewise rely on the respondent's correlative obligation per Section 4 of E.O. No. rhythm and harmony for the full enjoyment of a balanced and healthful ecology. Put a little
192, to safeguard the people's right to a healthful environment. differently, the minors' assertion of their right to a sound environment constitutes, at the same
time, the performance of their obligation to ensure the protection of that right for the generations
to come.
It is further claimed that the issue of the respondent Secretary's alleged grave abuse of
discretion in granting Timber License Agreements (TLAs) to cover more areas for logging than
what is available involves a judicial question. The locus standi of the petitioners having thus been addressed, We shall now proceed to the
merits of the petition.
Anent the invocation by the respondent Judge of the Constitution's non-impairment clause,
petitioners maintain that the same does not apply in this case because TLAs are not contracts. After a careful perusal of the complaint in question and a meticulous consideration and
They likewise submit that even if TLAs may be considered protected by the said clause, it is well evaluation of the issues raised and arguments adduced by the parties, We do not hesitate to find
settled that they may still be revoked by the State when the public interest so requires. for the petitioners and rule against the respondent Judge's challenged order for having been
issued with grave abuse of discretion amounting to lack of jurisdiction. The pertinent portions of
the said order reads as follows:
On the other hand, the respondents aver that the petitioners failed to allege in their complaint a
specific legal right violated by the respondent Secretary for which any relief is provided by law.
They see nothing in the complaint but vague and nebulous allegations concerning an xxx xxx xxx
"environmental right" which supposedly entitles the petitioners to the "protection by the state in
its capacity as parens patriae." Such allegations, according to them, do not reveal a valid cause
After a careful and circumspect evaluation of the Complaint, the Court
of action. They then reiterate the theory that the question of whether logging should be permitted
cannot help but agree with the defendant. For although we believe that
plaintiffs have but the noblest of all intentions, it (sic) fell short of alleging, else would be lost not only for the present generation, but also for those to come — generations
with sufficient definiteness, a specific legal right they are seeking to enforce which stand to inherit nothing but parched earth incapable of sustaining life.
and protect, or a specific legal wrong they are seeking to prevent and
redress (Sec. 1, Rule 2, RRC). Furthermore, the Court notes that the
The right to a balanced and healthful ecology carries with it the correlative duty to refrain from
Complaint is replete with vague assumptions and vague conclusions based
impairing the environment. During the debates on this right in one of the plenary sessions of the
on unverified data. In fine, plaintiffs fail to state a cause of action in its
1986 Constitutional Commission, the following exchange transpired between Commissioner
Complaint against the herein defendant.
Wilfrido Villacorta and Commissioner Adolfo Azcuna who sponsored the section in question:

Furthermore, the Court firmly believes that the matter before it, being
MR. VILLACORTA:
impressed with political color and involving a matter of public policy, may not
be taken cognizance of by this Court without doing violence to the sacred
principle of "Separation of Powers" of the three (3) co-equal branches of the Does this section mandate the State to provide
Government. sanctions against all forms of pollution — air, water and
noise pollution?
The Court is likewise of the impression that it cannot, no matter how we
stretch our jurisdiction, grant the reliefs prayed for by the plaintiffs, i.e., to MR. AZCUNA:
cancel all existing timber license agreements in the country and to cease
and desist from receiving, accepting, processing, renewing or approving
new timber license agreements. For to do otherwise would amount to Yes, Madam President. The right to healthful (sic)
environment necessarily carries with it the correlative
"impairment of contracts" abhored (sic) by the fundamental law. 11
duty of not impairing the same and, therefore, sanctions
may be provided for impairment of environmental
We do not agree with the trial court's conclusions that the plaintiffs failed to allege with sufficient balance. 12
definiteness a specific legal right involved or a specific legal wrong committed, and that the
complaint is replete with vague assumptions and conclusions based on unverified data. A
reading of the complaint itself belies these conclusions. The said right implies, among many other things, the judicious management and conservation of
the country's forests.

The complaint focuses on one specific fundamental legal right — the right to a balanced and
healthful ecology which, for the first time in our nation's constitutional history, is solemnly Without such forests, the ecological or environmental balance would be irreversiby
disrupted.
incorporated in the fundamental law. Section 16, Article II of the 1987 Constitution explicitly
provides:
Conformably with the enunciated right to a balanced and healthful ecology and the right to
Sec. 16. The State shall protect and advance the right of the people to a health, as well as the other related provisions of the Constitution concerning the conservation,
balanced and healthful ecology in accord with the rhythm and harmony of development and utilization of the country's natural resources, 13 then President Corazon C.
nature. Aquino promulgated on 10 June 1987 E.O. No. 192, 14 Section 4 of which expressly mandates
that the Department of Environment and Natural Resources "shall be the primary government
agency responsible for the conservation, management, development and proper use of the
This right unites with the right to health which is provided for in the country's environment and natural resources, specifically forest and grazing lands, mineral,
preceding section of the same article: resources, including those in reservation and watershed areas, and lands of the public domain,
as well as the licensing and regulation of all natural resources as may be provided for by law in
order to ensure equitable sharing of the benefits derived therefrom for the welfare of the present
Sec. 15. The State shall protect and promote the right to health of the
and future generations of Filipinos." Section 3 thereof makes the following statement of policy:
people and instill health consciousness among them.

Sec. 3. Declaration of Policy. — It is hereby declared the policy of the State


While the right to a balanced and healthful ecology is to be found under the Declaration of
to ensure the sustainable use, development, management, renewal, and
Principles and State Policies and not under the Bill of Rights, it does not follow that it is less
conservation of the country's forest, mineral, land, off-shore areas and other
important than any of the civil and political rights enumerated in the latter. Such a right belongs
natural resources, including the protection and enhancement of the quality
to a different category of rights altogether for it concerns nothing less than self-preservation and
of the environment, and equitable access of the different segments of the
self-perpetuation — aptly and fittingly stressed by the petitioners — the advancement of which
population to the development and the use of the country's natural
may even be said to predate all governments and constitutions. As a matter of fact, these basic
resources, not only for the present generation but for future generations as
rights need not even be written in the Constitution for they are assumed to exist from the
well. It is also the policy of the state to recognize and apply a true value
inception of humankind. If they are now explicitly mentioned in the fundamental charter, it is
system including social and environmental cost implications relative to their
because of the well-founded fear of its framers that unless the rights to a balanced and healthful
utilization, development and conservation of our natural resources.
ecology and to health are mandated as state policies by the Constitution itself, thereby
highlighting their continuing importance and imposing upon the state a solemn obligation to
preserve the first and protect and advance the second, the day would not be too far when all
This policy declaration is substantially re-stated it Title XIV, Book IV of the Administrative Code A denial or violation of that right by the other who has the corelative duty or obligation to respect
of 1987,15 specifically in Section 1 thereof which reads: or protect the same gives rise to a cause of action. Petitioners maintain that the granting of the
TLAs, which they claim was done with grave abuse of discretion, violated their right to a
balanced and healthful ecology; hence, the full protection thereof requires that no further TLAs
Sec. 1. Declaration of Policy. — (1) The State shall ensure, for the benefit of
should be renewed or granted.
the Filipino people, the full exploration and development as well as the
judicious disposition, utilization, management, renewal and conservation of
the country's forest, mineral, land, waters, fisheries, wildlife, off-shore areas A cause of action is defined as:
and other natural resources, consistent with the necessity of maintaining a
sound ecological balance and protecting and enhancing the quality of the
. . . an act or omission of one party in violation of the legal right or rights of
environment and the objective of making the exploration, development and
the other; and its essential elements are legal right of the plaintiff, correlative
utilization of such natural resources equitably accessible to the different
obligation of the defendant, and act or omission of the defendant in violation
segments of the present as well as future generations.
of said legal right. 18

(2) The State shall likewise recognize and apply a true value system that
It is settled in this jurisdiction that in a motion to dismiss based on the ground that the complaint
takes into account social and environmental cost implications relative to the
fails to state a cause of action, 19 the question submitted to the court for resolution involves the
utilization, development and conservation of our natural resources.
sufficiency of the facts alleged in the complaint itself. No other matter should be considered;
furthermore, the truth of falsity of the said allegations is beside the point for the truth thereof is
The above provision stresses "the necessity of maintaining a sound ecological balance and deemed hypothetically admitted. The only issue to be resolved in such a case is: admitting such
protecting and enhancing the quality of the environment." Section 2 of the same Title, on the alleged facts to be true, may the court render a valid judgment in accordance with the prayer in
other hand, specifically speaks of the mandate of the DENR; however, it makes particular the complaint? 20 In Militante vs. Edrosolano, 21 this Court laid down the rule that the judiciary
reference to the fact of the agency's being subject to law and higher authority. Said section should "exercise the utmost care and circumspection in passing upon a motion to dismiss on the
provides: ground of the absence thereof [cause of action] lest, by its failure to manifest a correct
appreciation of the facts alleged and deemed hypothetically admitted, what the law grants or
recognizes is effectively nullified. If that happens, there is a blot on the legal order. The law itself
Sec. 2. Mandate. — (1) The Department of Environment and Natural
stands in disrepute."
Resources shall be primarily responsible for the implementation of the
foregoing policy.
After careful examination of the petitioners' complaint, We find the statements under the
introductory affirmative allegations, as well as the specific averments under the sub-heading
(2) It shall, subject to law and higher authority, be in charge of carrying out
CAUSE OF ACTION, to be adequate enough to show, prima facie, the claimed violation of their
the State's constitutional mandate to control and supervise the exploration,
rights. On the basis thereof, they may thus be granted, wholly or partly, the reliefs prayed for. It
development, utilization, and conservation of the country's natural
bears stressing, however, that insofar as the cancellation of the TLAs is concerned, there is the
resources.
need to implead, as party defendants, the grantees thereof for they are indispensable parties.

Both E.O. NO. 192 and the Administrative Code of 1987 have set the objectives which will serve
The foregoing considered, Civil Case No. 90-777 be said to raise a political question. Policy
as the bases for policy formulation, and have defined the powers and functions of the DENR.
formulation or determination by the executive or legislative branches of Government is not
squarely put in issue. What is principally involved is the enforcement of a right vis-a-vis policies
It may, however, be recalled that even before the ratification of the 1987 Constitution, specific already formulated and expressed in legislation. It must, nonetheless, be emphasized that the
statutes already paid special attention to the "environmental right" of the present and future political question doctrine is no longer, the insurmountable obstacle to the exercise of judicial
generations. On 6 June 1977, P.D. No. 1151 (Philippine Environmental Policy) and P.D. No. power or the impenetrable shield that protects executive and legislative actions from judicial
1152 (Philippine Environment Code) were issued. The former "declared a continuing policy of inquiry or review. The second paragraph of section 1, Article VIII of the Constitution states that:
the State (a) to create, develop, maintain and improve conditions under which man and nature
can thrive in productive and enjoyable harmony with each other, (b) to fulfill the social, economic
Judicial power includes the duty of the courts of justice to settle actual
and other requirements of present and future generations of Filipinos, and (c) to insure the
controversies involving rights which are legally demandable and
attainment of an environmental quality that is conducive to a life of dignity and well-being." 16 As
enforceable, and to determine whether or not there has been a grave abuse
its goal, it speaks of the "responsibilities of each generation as trustee and guardian of the
of discretion amounting to lack or excess of jurisdiction on the part of any
environment for succeeding generations." 17 The latter statute, on the other hand, gave flesh to
branch or instrumentality of the Government.
the said policy.

Commenting on this provision in his book, Philippine Political Law, 22 Mr. Justice Isagani A. Cruz,
Thus, the right of the petitioners (and all those they represent) to a balanced and healthful
a distinguished member of this Court, says:
ecology is as clear as the DENR's duty — under its mandate and by virtue of its powers and
functions under E.O. No. 192 and the Administrative Code of 1987 — to protect and advance the
said right. The first part of the authority represents the traditional concept of judicial
power, involving the settlement of conflicting rights as conferred as law. The
second part of the authority represents a broadening of judicial power to
enable the courts of justice to review what was before forbidden territory, to withdrawn whenever dictated by public interest or public welfare as in this
wit, the discretion of the political departments of the government. case.

As worded, the new provision vests in the judiciary, and particularly the A license is merely a permit or privilege to do what otherwise would be
Supreme Court, the power to rule upon even the wisdom of the decisions of unlawful, and is not a contract between the authority, federal, state, or
the executive and the legislature and to declare their acts invalid for lack or municipal, granting it and the person to whom it is granted; neither is it
excess of jurisdiction because tainted with grave abuse of discretion. The property or a property right, nor does it create a vested right; nor is it
catch, of course, is the meaning of "grave abuse of discretion," which is a taxation (37 C.J. 168). Thus, this Court held that the granting of license
very elastic phrase that can expand or contract according to the disposition does not create irrevocable rights, neither is it property or property rights
of the judiciary. (People vs. Ong Tin, 54 O.G. 7576).

In Daza vs. Singson, 23 Mr. Justice Cruz, now speaking for this Court, noted: We reiterated this pronouncement in Felipe Ysmael, Jr. & Co., Inc. vs. Deputy Executive
Secretary: 26
In the case now before us, the jurisdictional objection becomes even less
tenable and decisive. The reason is that, even if we were to assume that the . . . Timber licenses, permits and license agreements are the principal
issue presented before us was political in nature, we would still not be instruments by which the State regulates the utilization and disposition of
precluded from revolving it under the expanded jurisdiction conferred upon forest resources to the end that public welfare is promoted. And it can hardly
us that now covers, in proper cases, even the political question. Article VII, be gainsaid that they merely evidence a privilege granted by the State to
Section 1, of the Constitution clearly provides: . . . qualified entities, and do not vest in the latter a permanent or irrevocable
right to the particular concession area and the forest products therein. They
may be validly amended, modified, replaced or rescinded by the Chief
The last ground invoked by the trial court in dismissing the complaint is the non-impairment of
Executive when national interests so require. Thus, they are not deemed
contracts clause found in the Constitution. The court a quo declared that:
contracts within the purview of the due process of law clause [See Sections
3(ee) and 20 of Pres. Decree No. 705, as amended. Also, Tan v. Director of
The Court is likewise of the impression that it cannot, no matter how we Forestry, G.R. No. L-24548, October 27, 1983, 125 SCRA 302].
stretch our jurisdiction, grant the reliefs prayed for by the plaintiffs, i.e., to
cancel all existing timber license agreements in the country and to cease
Since timber licenses are not contracts, the non-impairment clause, which reads:
and desist from receiving, accepting, processing, renewing or approving
new timber license agreements. For to do otherwise would amount to
"impairment of contracts" abhored (sic) by the fundamental law. 24 Sec. 10. No law impairing, the obligation of contracts shall be passed. 27

We are not persuaded at all; on the contrary, We are amazed, if not shocked, by such a cannot be invoked.
sweeping pronouncement. In the first place, the respondent Secretary did not, for obvious
reasons, even invoke in his motion to dismiss the non-impairment clause. If he had done so, he
In the second place, even if it is to be assumed that the same are contracts, the instant case
would have acted with utmost infidelity to the Government by providing undue and unwarranted
does not involve a law or even an executive issuance declaring the cancellation or modification
benefits and advantages to the timber license holders because he would have forever bound the
of existing timber licenses. Hence, the non-impairment clause cannot as yet be invoked.
Government to strictly respect the said licenses according to their terms and conditions
Nevertheless, granting further that a law has actually been passed mandating cancellations or
regardless of changes in policy and the demands of public interest and welfare. He was aware
modifications, the same cannot still be stigmatized as a violation of the non-impairment clause.
that as correctly pointed out by the petitioners, into every timber license must be read Section 20
This is because by its very nature and purpose, such as law could have only been passed in the
of the Forestry Reform Code (P.D. No. 705) which provides:
exercise of the police power of the state for the purpose of advancing the right of the people to a
balanced and healthful ecology, promoting their health and enhancing the general welfare.
. . . Provided, That when the national interest so requires, the President may In Abe vs. Foster Wheeler
amend, modify, replace or rescind any contract, concession, permit, Corp. 28 this Court stated:
licenses or any other form of privilege granted herein . . .
The freedom of contract, under our system of government, is not meant to
Needless to say, all licenses may thus be revoked or rescinded by executive action. It be absolute. The same is understood to be subject to reasonable legislative
is not a contract, property or a property right protested by the due process clause of regulation aimed at the promotion of public health, moral, safety and
the Constitution. In Tan vs. Director of Forestry, 25 this Court held: welfare. In other words, the constitutional guaranty of non-impairment of
obligations of contract is limited by the exercise of the police power of the
State, in the interest of public health, safety, moral and general welfare.
. . . A timber license is an instrument by which the State regulates the
utilization and disposition of forest resources to the end that public welfare is
promoted. A timber license is not a contract within the purview of the due The reason for this is emphatically set forth in Nebia vs. New York, 29 quoted in Philippine
process clause; it is only a license or privilege, which can be validly American Life Insurance Co. vs. Auditor General,30 to wit:
Under our form of government the use of property and the making of
contracts are normally matters of private and not of public concern. The
general rule is that both shall be free of governmental interference. But
neither property rights nor contract rights are absolute; for government
cannot exist if the citizen may at will use his property to the detriment of his
fellows, or exercise his freedom of contract to work them harm. Equally
fundamental with the private right is that of the public to regulate it in the
common interest.

In short, the non-impairment clause must yield to the police power of the state. 31

Finally, it is difficult to imagine, as the trial court did, how the non-impairment clause could apply
with respect to the prayer to enjoin the respondent Secretary from receiving, accepting,
processing, renewing or approving new timber licenses for, save in cases of renewal, no
contract would have as of yet existed in the other instances. Moreover, with respect to renewal,
the holder is not entitled to it as a matter of right.

WHEREFORE, being impressed with merit, the instant Petition is hereby GRANTED, and the
challenged Order of respondent Judge of 18 July 1991 dismissing Civil Case No. 90-777 is
hereby set aside. The petitioners may therefore amend their complaint to implead as defendants
the holders or grantees of the questioned timber license agreements.

No pronouncement as to costs.

SO ORDERED.
G.R. No. 204819 April 8, 2014 Gabriel Juatas, Salvacion M. Monteiro, Emily R. Laws, Joseph R .
Laws & Katrina R. Laws, Petitioners,
JAMES M. IMBONG and LOVELY-ANN C. IMBONG, for themselves vs.
and in behalf of their minor children, LUCIA CARLOS IMBONG and HON. PAQUITO N. OCHOA, JR., Executive Secretary, HON.
BERNADETTE CARLOS IMBONG and MAGNIFICAT CHILD ENRIQUE T. ONA, Secretary, Department of Health, HON. ARMIN A.
DEVELOPMENT CENTER, INC., Petitioners, LUISTRO, Secretary, Department of Education, Culture and Sports,
vs. HON. CORAZON SOLIMAN, Secretary, Department of Social Welfare
HON. PAQUITO N. OCHOA, JR., Executive Secretary, HON. and Development, HON. MANUELA. ROXAS II, Secretary,
FLORENCIO B. ABAD, Secretary, Department of Budget and Department of Interior and Local Government, HON. FLORENCIO B.
Management, HON. ENRIQUE T. ONA, Secretary, Department of ABAD, Secretary, Department of Budget and Management, HON.
Health, HON. ARMIN A. LUISTRO, Secretary, Department of ARSENIO M. BALISACAN, Socio-Economic Planning Secretary and
Education, Culture and Sports and HON. MANUELA. ROXAS II, NEDA Director-General, THE PHILIPPINE COMMISSION ON WOMEN,
Secretary, Department of Interior and Local represented by its Chairperson, Remedios lgnacio-Rikken, THE
Government, Respondents. PHILIPPINE HEALTH INSURANCE CORPORATION, represented by
its President Eduardo Banzon, THE LEAGUE OF PROVINCES OF
x---------------------------------x THE PHILIPPINES, represented by its President Alfonso Umali, THE
LEAGUE OF CITIES OF THE PHILIPPINES, represented by its
President Oscar Rodriguez, and THE LEAGUE OF MUNICIPALITIES
G.R. No. 204934
OF THE PHILIPPINES, represented by its President Donato
Marcos,Respondents.
ALLIANCE FOR THE FAMILY FOUNDATION PHILIPPINES, INC.
[ALFI], represented by its President, Maria Concepcion S. Noche,
x---------------------------------x
Spouses Reynaldo S. Luistro & Rosie B . Luistro, Jose S. Sandejas
& Elenita S.A. Sandejas, Arturo M. Gorrez & Marietta C. Gorrez,
Salvador S. Mante, Jr. & Hazeleen L. Mante, Rolando M. Bautista & G.R. No. 204957
Maria Felisa S. Bautista, Desiderio Racho & Traquilina Racho, F
emand Antonio A. Tansingco & Carol Anne C. Tansingco for TASK FORCE FOR FAMILY AND LIFE VISAYAS, INC. and
themselves and on behalf of their minor children, Therese Antonette VALERIANO S. AVILA, Petitioners,
C. Tansingco, Lorenzo Jose C. Tansingco, Miguel F emando C. vs.
Tangsingco, Carlo Josemaria C. Tansingco & Juan Paolo C. HON. PAQUITO N. OCHOA, JR., Executive Secretary; HON.
Tansingco, Spouses Mariano V. Araneta & Eileen Z. Araneta for FLORENCIO B. ABAD, Secretary, Department of Budget and
themselves and on behalf of their minor children, Ramon Carlos Z. Management; HON. ENRIQUE T. ONA, Secretary, Department of
Araneta & Maya Angelica Z. Araneta, Spouses Renato C. Castor & Education; and HON. MANUELA. ROXAS II, Secretary, Department
Mildred C. Castor for themselves and on behalf of their minor of Interior and Local Government, Respondents.
children, Renz Jeffrey C. Castor, Joseph Ramil C. Castor, John Paul
C. Castor & Raphael C. Castor, Spouses Alexander R. Racho & Zara x---------------------------------x
Z. Racho for themselves and on behalf of their minor children
Margarita Racho, Mikaela Racho, Martin Racho, Mari Racho & G.R. No. 204988
Manolo Racho, Spouses Alfred R. Racho & Francine V. Racho for
themselves and on behalf of their minor children Michael Racho, SERVE LIFE CAGAYAN DE ORO CITY, INC., represented by Dr.
Mariana Racho, Rafael Racho, Maxi Racho, Chessie Racho & Laura Nestor B. Lumicao, M.D., as President and in his personal capacity,
Racho, Spouses David R. Racho & Armilyn A. Racho for themselves ROSEVALE FOUNDATION INC., represented by Dr. Rodrigo M.
and on behalf of their minor child Gabriel Racho, Mindy M. Juatas Alenton, M.D., as member of the school board and in his personal
and on behalf of her minor children Elijah Gerald Juatas and Elian capacity, ROSEMARIE R. ALENTON, IMELDA G. IBARRA, CPA,
LOVENIAP. NACES, Phd., ANTHONY G. NAGAC, EARL ANTHONY C. Osmundo C. Orlanes, Arsenio Z. Menor, Samuel J. Yap, Jaime F.
GAMBE and MARLON I. YAP,Petitioners, Mateo, Rolly Siguan, Dante E. Magdangal, Michael Eugenio O. Plana,
vs. Bienvenido C. Miguel, Jr., Landrito M. Diokno and Baldomero
OFFICE OF THE PRESIDENT, SENATE OF THE PHILIPPINES, Falcone, Petitioners,
HOUSE OF REPRESENTATIVES, HON. PAQUITO N. OCHOA, JR., vs.
Executive Secretary, HON. FLORENCIO B. ABAD, Secretary, HON. PAQUITO N. OCHOA, JR., Executive Secretary, HON.
Department of Budget and Management; HON. ENRIQUE T. ONA, FLORENCIO B. ABAD, Secretary, Department of Budget and
Secretary, Department of Health; HON. ARMIN A. LUISTRO, Management, HON. ENRIQUE T. ONA, Secretary, Department of
Secretary, Department of Education and HON. MANUELA. ROXAS II, Health, HON. ARMIN A. LUISTRO, Secretary, Department of
Secretary, Department of Interior and Local Education, HON. MANUELA. ROXAS II, Secretary, Department of
Government, Respondents. Interior and Local Government, HON. CORAZON J. SOLIMAN,
Secretary, Department of Social Welfare and Development, HON.
x---------------------------------x ARSENIO BALISACAN, Director-General, National Economic and
Development Authority, HON. SUZETTE H. LAZO, Director-General,
G.R. No. 205003 Food and Drugs Administration, THE BOARD OF DIRECTORS,
Philippine Health Insurance Corporation, and THE BOARD OF
COMMISSIONERS, Philippine Commission on Women, Respondents.
EXPEDITO A. BUGARIN, JR., Petitioner,
vs.
OFFICE OF THE PRESIDENT OF THE REPUBLIC OF THE x---------------------------------x
PHILIPPINES, HON. SENATE PRESIDENT, HON. SPEAKER OF THE
HOUSE OF REPRESENTATIVES and HON. SOLICITOR G.R. No. 205478
GENERAL, Respondents.
REYNALDO J. ECHAVEZ, M.D., JACQUELINE H. KING, M.D.,
x---------------------------------x CYNTHIA T. DOMINGO, M.D., AND JOSEPHINE MILLADO-LUMITAO,
M.D., collectively known as Doctors For Life, and ANTHONY PEREZ,
G.R. No. 205043 MICHAEL ANTHONY G. MAPA, CARLOS ANTONIO PALAD,
WILFREDO JOSE, CLAIRE NAVARRO, ANNA COSIO, and GABRIEL
DY LIACCO collectively known as Filipinos For Life, Petitioners,
EDUARDO B. OLAGUER and THE CATHOLIC XYBRSPACE
vs.
APOSTOLATE OF THE PHILIPPINES, Petitioners,
HON. PAQUITO N. OCHOA, JR., Executive Secretary; HON.
vs.
FLORENCIO B. ABAD, Secretary of the Department of Budget and
DOH SECRETARY ENRIQUE T. ONA, FDA DIRECTOR SUZETTE H.
Management; HON. ENRIQUE T. ONA, Secretary of the Department
LAZO, DBM SECRETARY FLORENCIO B. ABAD, DILG SECRETARY
of Health; HON. ARMIN A. LUISTRO, Secretary of the Department of
MANUELA. ROXAS II, DECS SECRETARY ARMIN A.
Education; and HON. MANUELA. ROXAS II, Secretary of the
LUISTRO, Respondents.
Department of Interior and Local Government, Respondents.
x---------------------------------x
x---------------------------------x
G.R. No. 205138
G.R. No. 205491
PHILIPPINE ALLIANCE OF XSEMINARIANS, INC. (PAX), herein
SPOUSES FRANCISCO S. TATAD AND MARIA FENNY C. TATAD &
represented by its National President, Atty. Ricardo M . Ribo, and in
ALA F. PAGUIA, for themselves, their Posterity, and the rest of
his own behalf, Atty. Lino E.A. Dumas, Romeo B. Almonte,
Filipino posterity, Petitioners,
vs. ROXAS and LOTA LAT-GUERRERO, Petitioners,
OFFICE OF THE PRESIDENT of the Republic of the vs.
Philippines, Respondent. HON. PAQUITO N. OCHOA, JR., Executive Secretary, HON.
FLORENCIO ABAD, Secretary, Department of Budget and
x---------------------------------x Management, HON. ENRIQUE T. ONA, Secretary, Department of
Health, HON. ARMIN A. LUISTRO, Secretary, Department of
G.R. No. 205720 Education, Culture and Sports and HON. MANUEL A. ROXAS II,
Secretary, Department of Interior and Local
Government, Respondents.
PRO-LIFE PHILIPPINES FOUNDATION, Inc., represented by Loma
Melegrito, as Executive Director, and in her personal capacity,
JOSELYN B. BASILIO, ROBERT Z. CORTES, ARIEL A. x---------------------------------x
CRISOSTOMO, JEREMY I. GATDULA, CRISTINA A. MONTES, RAUL
ANTONIO A. NIDOY, WINSTON CONRAD B. PADOJINOG, RUFINO L. G.R. No. 207172
POLICARPIO III, Petitioners,
vs. COUPLES FOR CHRIST FOUNDATION, INC., SPOUSES JUAN
OFFICE OF THE PRESIDENT, SENATE OF THE PHILIPPINES, CARLOS ARTADI SARMIENTO AND FRANCESCA ISABELLE
HOUSE OF REPRESENTATIVES, HON. PAQUITO N. OCHOA, JR., BESINGA-SARMIENTO, AND SPOUSES LUIS FRANCIS A.
Executive Secretary, HON. FLORENCIO B. ABAD, Secretary, RODRIGO, JR. and DEBORAH MARIE VERONICA N.
Department of Budget and Management, HON. ENRIQUE T. ONA, RODRIGO, Petitioners,
Secretary, Department of Health, HON. ARMIN A. LUISTRO, vs.
Secretary, Department of Education and HON. MANUEL A. ROXAS HON. PAQUITO N. OCHOA, JR., Executive Secretary, HON.
II, Secretary, Department of Interior and Local FLORENCIO B. ABAD, Secretary, Department of Budget and
Government, Respondents. Management, HON. ENRIQUE T. ONA, Secretary, Department of
Health, HON. ARMIN A. LUISTRO, Secretary, Department of
x---------------------------------x Education, Culture and Sports and HON. MANUELA. ROXAS II,
Secretary, Department of Interior and Local
G.R. No. 206355 Government, Respondents.

MILLENNIUM SAINT FOUNDATION, INC., ATTY. RAMON PEDROSA, x---------------------------------x


ATTY. CITA BORROMEO-GARCIA, STELLAACEDERA, ATTY.
BERTENI CATALUNA CAUSING, Petitioners, G.R. No. 207563
vs.
OFFICE OF THE PRESIDENT, OFFICE OF THE EXECUTIVE ALMARIM CENTI TILLAH and ABDULHUSSEIN M.
SECRETARY, DEPARTMENT OF HEALTH, DEPARTMENT OF KASHIM, Petitioners,
EDUCATION, Respondents. vs.
HON. PAQUITO N. OCHOA, JR., Executive Secretary, HON.
x---------------------------------x ENRIQUE T. ONA, Secretary of the Department of Health, and HON.
ARMIN A. LUISTRO,Secretary of the Department of Budget and
G.R. No. 207111 Management,Respondents.

JOHN WALTER B. JUAT, MARY M. IMBONG, ANTHONY VICTORIO DECISION


B. LUMICAO, JOSEPH MARTIN Q. VERDEJO, ANTONIA EMMA R.
MENDOZA, J.: Shortly after the President placed his imprimatur on the said law,
challengers from various sectors of society came knocking on the doors
Freedom of religion was accorded preferred status by the framers of our of the Court, beckoning it to wield the sword that strikes down
fundamental law. And this Court has consistently affirmed this preferred constitutional disobedience. Aware of the profound and lasting impact
status, well aware that it is "designed to protect the broadest possible that its decision may produce, the Court now faces the iuris controversy,
liberty of conscience, to allow each man to believe as his conscience as presented in fourteen (14) petitions and two (2) petitions- in-
directs, to profess his beliefs , and to live as he believes he ought to live, intervention, to wit:
consistent with the liberty of others and with the common good."1
(1) Petition for Certiorari and Prohibition,5 filed by spouses Attys.
To this day, poverty is still a major stumbling block to the nation's James M. Imbong and Lovely Ann C. Imbong, in their personal
emergence as a developed country, leaving our people beleaguered in a capacities as citizens, lawyers and taxpayers and on behalf of
state of hunger, illiteracy and unemployment. While governmental their minor children; and the Magnificat Child Leaming Center,
policies have been geared towards the revitalization of the economy, the Inc., a domestic, privately-owned educational institution
bludgeoning dearth in social services remains to be a problem that (Jmbong);
concerns not only the poor, but every member of society. The
government continues to tread on a trying path to the realization of its (2) Petition for Prohibition,6 filed by the Alliance for the Family
very purpose, that is, the general welfare of the Filipino people and the Foundation Philippines, Inc., through its president, Atty. Maria
development of the country as a whole. The legislative branch, as the Concepcion S. Noche7 and several others8 in their personal
main facet of a representative government, endeavors to enact laws and capacities as citizens and on behalf of the generations unborn
policies that aim to remedy looming societal woes, while the executive is (ALFI);
closed set to fully implement these measures and bring concrete and
substantial solutions within the reach of Juan dela Cruz. Seemingly (3) Petition for Certiorari,9 filed by the Task Force for Family and
distant is the judicial branch, oftentimes regarded as an inert Life Visayas, Inc., and Valeriano S. Avila, in their capacities as
governmental body that merely casts its watchful eyes on clashing citizens and taxpayers (Task Force Family);
stakeholders until it is called upon to adjudicate. Passive, yet reflexive
when called into action, the Judiciary then willingly embarks on its solemn (4) Petition for Certiorari and Prohibition,10 filed by Serve Life
duty to interpret legislation vis-a-vis the most vital and enduring principle Cagayan De Oro City, Inc.,11 Rosevale Foundation, Inc.,12 a
that holds Philippine society together - the supremacy of the Philippine domestic, privately-owned educational institution, and several
Constitution. others,13 in their capacities as citizens (Serve Life);

Nothing has polarized the nation more in recent years than the issues of (5) Petition,14 filed by Expedito A. Bugarin, Jr. in his capacity as a
population growth control, abortion and contraception. As in every citizen (Bugarin);
democratic society, diametrically opposed views on the subjects and their
perceived consequences freely circulate in various media. From
(6) Petition for Certiorari and Prohibition,15 filed by Eduardo
television debates2 to sticker campaigns,3 from rallies by socio-political
Olaguer and the Catholic Xybrspace Apostolate of the
activists to mass gatherings organized by members of the clergy4 - the
Philippines,16 in their capacities as a citizens and taxpayers
clash between the seemingly antithetical ideologies of the religious
(Olaguer);
conservatives and progressive liberals has caused a deep division in
every level of the society. Despite calls to withhold support thereto,
however, Republic Act (R.A.) No. 10354, otherwise known as the (7) Petition for Certiorari and Prohibition,17 filed by the Philippine
Responsible Parenthood and Reproductive Health Act of 2012 (RH Law), Alliance of Xseminarians Inc.,18 and several others19 in their
was enacted by Congress on December 21, 2012. capacities as citizens and taxpayers (PAX);
(8) Petition,20 filed by Reynaldo J. Echavez, M.D. and several purchase of hormonal contraceptives, intra-uterine devices and
others,21 in their capacities as citizens and taxpayers (Echavez); injectables which are abortives, in violation of Section 12, Article
II of the Constitution which guarantees protection of both the life
(9) Petition for Certiorari and Prohibition,22 filed by spouses of the mother and the life of the unborn from conception.35
Francisco and Maria Fenny C. Tatad and Atty. Alan F. Paguia, in
their capacities as citizens, taxpayers and on behalf of those yet • The RH Law violates the right to health and the right to
unborn. Atty. Alan F. Paguia is also proceeding in his capacity as protection against hazardous products. The petitioners posit that
a member of the Bar (Tatad); the RH Law provides universal access to contraceptives which
are hazardous to one's health, as it causes cancer and other
(10) Petition for Certiorari and Prohibition,23 filed by Pro-Life health problems.36
Philippines Foundation Inc.24 and several others,25 in their
capacities as citizens and taxpayers and on behalf of its • The RH Law violates the right to religious freedom. The
associates who are members of the Bar (Pro-Life); petitioners contend that the RH Law violates the constitutional
guarantee respecting religion as it authorizes the use of public
(11) Petition for Prohibition,26 filed by Millennium Saint funds for the procurement of contraceptives. For the petitioners,
Foundation, Inc.,27 Attys. Ramon Pedrosa, Cita Borromeo-Garcia, the use of public funds for purposes that are believed to be
Stella Acedera, and Berteni Catalufia Causing, in their capacities contrary to their beliefs is included in the constitutional mandate
as citizens, taxpayers and members of the Bar (MSF); ensuring religious freedom.37

(12) Petition for Certiorari and Prohibition,28 filed by John Walter It is also contended that the RH Law threatens conscientious objectors of
B. Juat and several others,29 in their capacities as citizens (Juat) ; criminal prosecution, imprisonment and other forms of punishment, as it
compels medical practitioners 1] to refer patients who seek advice on
(13) Petition for Certiorari and Prohibition,30 filed by Couples for reproductive health programs to other doctors; and 2] to provide full and
Christ Foundation, Inc. and several others,31in their capacities as correct information on reproductive health programs and service,
citizens (CFC); although it is against their religious beliefs and convictions.38

(14) Petition for Prohibition32 filed by Almarim Centi Tillah and In this connection, Section 5 .23 of the Implementing Rules and
Abdulhussein M. Kashim in their capacities as citizens and Regulations of the RH Law (RH-IRR),39 provides that skilled health
taxpayers (Tillah); and professionals who are public officers such as, but not limited to,
Provincial, City, or Municipal Health Officers, medical officers, medical
specialists, rural health physicians, hospital staff nurses, public health
(15) Petition-In-Intervention,33 filed by Atty. Samson S. Alcantara
nurses, or rural health midwives, who are specifically charged with the
in his capacity as a citizen and a taxpayer (Alcantara); and
duty to implement these Rules, cannot be considered as conscientious
objectors.40
(16) Petition-In-Intervention,34 filed by Buhay Hayaang Yumabong
(B UHAY) , an accredited political party.
It is also argued that the RH Law providing for the formulation of
mandatory sex education in schools should not be allowed as it is an
A perusal of the foregoing petitions shows that the petitioners are affront to their religious beliefs.41
assailing the constitutionality of RH Law on the following GROUNDS:
While the petit10ners recognize that the guarantee of religious freedom is
• The RH Law violates the right to life of the unborn. According to not absolute, they argue that the RH Law fails to satisfy the "clear and
the petitioners, notwithstanding its declared policy against present danger test" and the "compelling state interest test" to justify the
abortion, the implementation of the RH Law would authorize the
regulation of the right to free exercise of religion and the right to free to institutions owned and operated by religious groups, they are
speech.42 still forced to refer their patients to another healthcare facility
willing to perform the service or procedure.48
• The RH Law violates the constitutional provision on involuntary
servitude. According to the petitioners, the RH Law subjects • The RH Law intrudes into the zone of privacy of one's family
medical practitioners to involuntary servitude because, to be protected by the Constitution. It is contended that the RH Law
accredited under the PhilHealth program, they are compelled to providing for mandatory reproductive health education intrudes
provide forty-eight (48) hours of pro bona services for indigent upon their constitutional right to raise their children in accordance
women, under threat of criminal prosecution, imprisonment and with their beliefs.49
other forms of punishment.43
It is claimed that, by giving absolute authority to the person who will
The petitioners explain that since a majority of patients are covered by undergo reproductive health procedure, the RH Law forsakes any real
PhilHealth, a medical practitioner would effectively be forced to render dialogue between the spouses and impedes the right of spouses to
reproductive health services since the lack of PhilHealth accreditation mutually decide on matters pertaining to the overall well-being of their
would mean that the majority of the public would no longer be able to family. In the same breath, it is also claimed that the parents of a child
avail of the practitioners services.44 who has suffered a miscarriage are deprived of parental authority to
determine whether their child should use contraceptives.50
• The RH Law violates the right to equal protection of the law. It is
claimed that the RH Law discriminates against the poor as it • The RH Law violates the constitutional principle of non-
makes them the primary target of the government program that delegation of legislative authority. The petitioners question the
promotes contraceptive use. The petitioners argue that, rather delegation by Congress to the FDA of the power to determine
than promoting reproductive health among the poor, the RH Law whether a product is non-abortifacient and to be included in the
seeks to introduce contraceptives that would effectively reduce Emergency Drugs List (EDL).51
the number of the poor.45
• The RH Law violates the one subject/one bill rule provision
• The RH Law is "void-for-vagueness" in violation of the due under Section 26( 1 ), Article VI of the Constitution.52
process clause of the Constitution. In imposing the penalty of
imprisonment and/or fine for "any violation," it is vague because it • The RH Law violates Natural Law.53
does not define the type of conduct to be treated as "violation" of
the RH Law.46 • The RH Law violates the principle of Autonomy of Local
Government Units (LGUs) and the Autonomous Region of Muslim
In this connection, it is claimed that "Section 7 of the RH Law violates the Mindanao {ARMM). It is contended that the RH Law, providing for
right to due process by removing from them (the people) the right to reproductive health measures at the local government level and
manage their own affairs and to decide what kind of health facility they the ARMM, infringes upon the powers devolved to LGUs and the
shall be and what kind of services they shall offer."47 It ignores the ARMM under the Local Government Code and R.A . No. 9054.54
management prerogative inherent in corporations for employers to
conduct their affairs in accordance with their own discretion and Various parties also sought and were granted leave to file their respective
judgment. comments-in-intervention in defense of the constitutionality of the RH
Law. Aside from the Office of the Solicitor General (OSG) which
• The RH Law violates the right to free speech. To compel a commented on the petitions in behalf of the respondents,55 Congressman
person to explain a full range of family planning methods is plainly Edcel C. Lagman,56 former officials of the Department of Health Dr.
to curtail his right to expound only his own preferred way of family Esperanza I. Cabral, Jamie Galvez-Tan, and Dr. Alberto G.
planning. The petitioners note that although exemption is granted Romualdez,57 the Filipino Catholic Voices for Reproductive Health
(C4RH),58 Ana Theresa "Risa" Hontiveros,59 and Atty. Joan De Contraceptive Drugs and Devices." Although contraceptive drugs and
Venecia60 also filed their respective Comments-in-Intervention in devices were allowed, they could not be sold, dispensed or distributed
conjunction with several others. On June 4, 2013, Senator Pia Juliana S. "unless such sale, dispensation and distribution is by a duly licensed drug
Cayetano was also granted leave to intervene.61 store or pharmaceutical company and with the prescription of a qualified
medical practitioner."65
The respondents, aside from traversing the substantive arguments of the
petitioners, pray for the dismissal of the petitions for the principal reasons In addition, R.A. No. 5921,66 approved on June 21, 1969, contained
that 1] there is no actual case or controversy and, therefore, the issues provisions relative to "dispensing of abortifacients or anti-conceptional
are not yet ripe for judicial determination.; 2] some petitioners lack substances and devices." Under Section 37 thereof, it was provided that
standing to question the RH Law; and 3] the petitions are essentially "no drug or chemical product or device capable of provoking abortion or
petitions for declaratory relief over which the Court has no original preventing conception as classified by the Food and Drug Administration
jurisdiction. shall be delivered or sold to any person without a proper prescription by a
duly licensed physician."
Meanwhile, on March 15, 2013, the RH-IRR for the enforcement of the
assailed legislation took effect. On December 11, 1967, the Philippines, adhering to the UN Declaration
on Population, which recognized that the population problem should be
On March 19, 2013, after considering the issues and arguments raised, considered as the principal element for long-term economic development,
the Court issued the Status Quo Ante Order (SQAO), enjoining the enacted measures that promoted male vasectomy and tubal ligation to
effects and implementation of the assailed legislation for a period of one mitigate population growth.67 Among these measures included R.A. No.
hundred and twenty (120) days, or until July 17, 2013.62 6365, approved on August 16, 1971, entitled "An Act Establishing a
National Policy on Population, Creating the Commission on Population
On May 30, 2013, the Court held a preliminary conference with the and for Other Purposes. " The law envisioned that "family planning will be
counsels of the parties to determine and/or identify the pertinent issues made part of a broad educational program; safe and effective means will
raised by the parties and the sequence by which these issues were to be be provided to couples desiring to space or limit family size; mortality and
discussed in the oral arguments. On July 9 and 23, 2013, and on August morbidity rates will be further reduced."
6, 13, and 27, 2013, the cases were heard on oral argument. On July 16,
2013, the SQAO was ordered extended until further orders of the Court.63 To further strengthen R.A. No. 6365, then President Ferdinand E .
Marcos issued Presidential Decree. (P.D.) No. 79,68 dated December 8,
Thereafter, the Court directed the parties to submit their respective 1972, which, among others, made "family planning a part of a broad
memoranda within sixty (60) days and, at the same time posed several educational program," provided "family planning services as a part of
questions for their clarification on some contentions of the parties.64 over-all health care," and made "available all acceptable methods of
contraception, except abortion, to all Filipino citizens desirous of spacing,
limiting or preventing pregnancies."
The Status Quo Ante
Through the years, however, the use of contraceptives and family
(Population, Contraceptive and Reproductive Health Laws
planning methods evolved from being a component of demographic
management, to one centered on the promotion of public health,
Prior to the RH Law particularly, reproductive health.69 Under that policy, the country gave
priority to one's right to freely choose the method of family planning to be
Long before the incipience of the RH Law, the country has allowed the adopted, in conformity with its adherence to the commitments made in
sale, dispensation and distribution of contraceptive drugs and devices. As the International Conference on Population and Development.70 Thus, on
far back as June 18, 1966, the country enacted R.A. No. 4729 entitled August 14, 2009, the country enacted R.A. No. 9710 or "The Magna
"An Act to Regu,late the Sale, Dispensation, and/or Distribution of Carta for Women, " which, among others, mandated the State to provide
for comprehensive health services and programs for women, including using taxpayers' money. The State then will be the funder and provider of
family planning and sex education.71 all forms of family planning methods and the implementer of the program
by ensuring the widespread dissemination of, and universal access to, a
The RH Law full range of family planning methods, devices and supplies.74

Despite the foregoing legislative measures, the population of the country ISSUES
kept on galloping at an uncontrollable pace. From a paltry number of just
over 27 million Filipinos in 1960, the population of the country reached After a scrutiny of the various arguments and contentions of the parties,
over 76 million in the year 2000 and over 92 million in 2010.72 The the Court has synthesized and refined them to the following principal
executive and the legislative, thus, felt that the measures were still not issues:
adequate. To rein in the problem, the RH Law was enacted to provide
Filipinos, especially the poor and the marginalized, access and I. PROCEDURAL: Whether the Court may exercise its power of judicial
information to the full range of modem family planning methods, and to review over the controversy.
ensure that its objective to provide for the peoples' right to reproductive
health be achieved. To make it more effective, the RH Law made it 1] Power of Judicial Review
mandatory for health providers to provide information on the full range of
modem family planning methods, supplies and services, and for schools
2] Actual Case or Controversy
to provide reproductive health education. To put teeth to it, the RH Law
criminalizes certain acts of refusals to carry out its mandates.
3] Facial Challenge
Stated differently, the RH Law is an enhancement measure to fortify and
make effective the current laws on contraception, women's health and 4] Locus Standi
population control.
5] Declaratory Relief
Prayer of the Petitioners - Maintain the Status Quo
6] One Subject/One Title Rule
The petitioners are one in praying that the entire RH Law be declared
unconstitutional. Petitioner ALFI, in particular, argues that the II. SUBSTANTIVE: Whether the RH law is unconstitutional:
government sponsored contraception program, the very essence of the
RH Law, violates the right to health of women and the sanctity of life, 1] Right to Life
which the State is mandated to protect and promote. Thus, ALFI prays
that "the status quo ante - the situation prior to the passage of the RH 2] Right to Health
Law - must be maintained."73 It explains:
3] Freedom of Religion and the Right to Free Speech
x x x. The instant Petition does not question contraception and
contraceptives per se. As provided under Republic Act No. 5921 and 4] The Family
Republic Act No. 4729, the sale and distribution of contraceptives are
prohibited unless dispensed by a prescription duly licensed by a 5] Freedom of Expression and Academic Freedom
physician. What the Petitioners find deplorable and repugnant under the
RH Law is the role that the State and its agencies - the entire
bureaucracy, from the cabinet secretaries down to the barangay officials 6] Due Process
in the remotest areas of the country - is made to play in the
implementation of the contraception program to the fullest extent possible 7] Equal Protection
8] Involuntary Servitude of powers. To be clear, the separation of powers is a fundamental
principle in our system of government, which obtains not through express
9] Delegation of Authority to the FDA provision but by actual division in our Constitution. Each department of
the government has exclusive cognizance of matters within its jurisdiction
10] Autonomy of Local Govemments/ARMM and is supreme within its own sphere.81

DISCUSSION Thus, the 1987 Constitution provides that: (a) the legislative power shall
be vested in the Congress of the Philippines;82 (b) the executive power
shall be vested in the President of the Philippines;83 and (c) the judicial
Before delving into the constitutionality of the RH Law and its
power shall be vested in one Supreme Court and in such lower courts as
implementing rules, it behooves the Court to resolve some procedural
may be established by law.84 The Constitution has truly blocked out with
impediments.
deft strokes and in bold lines, the allotment of powers among the three
branches of government.85
I. PROCEDURAL ISSUE: Whether the Court can exercise its power of
judicial review over the controversy.
In its relationship with its co-equals, the Judiciary recognizes the doctrine
of separation of powers which imposes upon the courts proper restraint,
The Power of Judicial Review born of the nature of their functions and of their respect for the other
branches of government, in striking down the acts of the Executive or the
In its attempt to persuade the Court to stay its judicial hand, the OSG Legislature as unconstitutional. Verily, the policy is a harmonious blend of
asserts that it should submit to the legislative and political wisdom of courtesy and caution.86
Congress and respect the compromises made in the crafting of the RH
Law, it being "a product of a majoritarian democratic process"75 and It has also long been observed, however, that in times of social
"characterized by an inordinate amount of transparency."76 The OSG disquietude or political instability, the great landmarks of the Constitution
posits that the authority of the Court to review social legislation like the are apt to be forgotten or marred, if not entirely obliterated.87 In order to
RH Law by certiorari is "weak," since the Constitution vests the discretion address this, the Constitution impresses upon the Court to respect the
to implement the constitutional policies and positive norms with the acts performed by a co-equal branch done within its sphere of
political departments, in particular, with Congress.77 It further asserts that competence and authority, but at the same time, allows it to cross the line
in view of the Court's ruling in Southern Hemisphere v. Anti-Terrorism of separation - but only at a very limited and specific point - to determine
Council,78 the remedies of certiorari and prohibition utilized by the whether the acts of the executive and the legislative branches are null
petitioners are improper to assail the validity of the acts of the because they were undertaken with grave abuse of discretion.88 Thus,
legislature.79 while the Court may not pass upon questions of wisdom, justice or
expediency of the RH Law, it may do so where an attendant
Moreover, the OSG submits that as an "as applied challenge," it cannot unconstitutionality or grave abuse of discretion results.89 The Court must
prosper considering that the assailed law has yet to be enforced and demonstrate its unflinching commitment to protect those cherished rights
applied to the petitioners, and that the government has yet to distribute and principles embodied in the Constitution.
reproductive health devices that are abortive. It claims that the RH Law
cannot be challenged "on its face" as it is not a speech-regulating In this connection, it bears adding that while the scope of judicial power
measure.80 of review may be limited, the Constitution makes no distinction as to the
kind of legislation that may be subject to judicial scrutiny, be it in the form
In many cases involving the determination of the constitutionality of the of social legislation or otherwise. The reason is simple and goes back to
actions of the Executive and the Legislature, it is often sought that the the earlier point. The Court may pass upon the constitutionality of acts of
Court temper its exercise of judicial power and accord due respect to the the legislative and the executive branches, since its duty is not to review
wisdom of its co-equal branch on the basis of the principle of separation their collective wisdom but, rather, to make sure that they have acted in
consonance with their respective authorities and rights as mandated of participation - or instrument of intervention - of the judiciary in that
them by the Constitution. If after said review, the Court finds no balancing operation.95
constitutional violations of any sort, then, it has no more authority of
proscribing the actions under review.90 This is in line with Article VIII, Lest it be misunderstood, it bears emphasizing that the Court does not
Section 1 of the Constitution which expressly provides: have the unbridled authority to rule on just any and every claim of
constitutional violation. Jurisprudence is replete with the rule that the
Section 1. The judicial power shall be vested in one Supreme Court and power of judicial review is limited by four exacting requisites, viz : (a)
in such lower courts as may be established by law. there must be an actual case or controversy; (b) the petitioners must
possess locus standi; (c) the question of constitutionality must be raised
Judicial power includes the duty of the courts of justice to settle actual at the earliest opportunity; and (d) the issue of constitutionality must be
controversies involving rights which are legally demandable and the lis mota of the case.96
enforceable, and to determine whether or not there has been a grave
abuse of discretion amounting to lack or excess of jurisdiction on the part Actual Case or Controversy
of any branch or instrumentality of the Government. [Emphases supplied]
Proponents of the RH Law submit that the subj ect petitions do not
As far back as Tanada v. Angara,91 the Court has unequivocally declared present any actual case or controversy because the RH Law has yet to
that certiorari, prohibition and mandamus are appropriate remedies to be implemented.97 They claim that the questions raised by the petitions
raise constitutional issues and to review and/or prohibit/nullify, when are not yet concrete and ripe for adjudication since no one has been
proper, acts of legislative and executive officials, as there is no other charged with violating any of its provisions and that there is no showing
plain, speedy or adequate remedy in the ordinary course of law. This that any of the petitioners' rights has been adversely affected by its
ruling was later on applied in Macalintal v. COMELEC,92 Aldaba v. operation.98 In short, it is contended that judicial review of the RH Law is
COMELEC,93 Magallona v. Ermita,94 and countless others. In Tanada, the premature.
Court wrote:
An actual case or controversy means an existing case or controversy that
In seeking to nullify an act of the Philippine Senate on the ground that it is appropriate or ripe for determination, not conjectural or anticipatory,
contravenes the Constitution, the petition no doubt raises a justiciable lest the decision of the court would amount to an advisory opinion.99 The
controversy. Where an action of the legislative branch is seriously alleged rule is that courts do not sit to adjudicate mere academic questions to
to have infringed the Constitution, it becomes not only the right but in fact satisfy scholarly interest, however intellectually challenging. The
the duty of the judiciary to settle the dispute. "The question thus posed is controversy must be justiciable-definite and concrete, touching on the
judicial rather than political. The duty (to adjudicate) remains to assure legal relations of parties having adverse legal interests. In other words,
that the supremacy of the Constitution is upheld. " Once a "controversy the pleadings must show an active antagonistic assertion of a legal right,
as to the application or interpretation of constitutional provision is raised on the one hand, and a denial thereof, on the other; that is, it must
before this Court (as in the instant case), it becomes a legal issue which concern a real, tangible and not merely a theoretical question or issue.
the Court is bound by constitutional mandate to decide. [Emphasis There ought to be an actual and substantial controversy admitting of
supplied] specific relief through a decree conclusive in nature, as distinguished
from an opinion advising what the law would be upon a hypothetical state
In the scholarly estimation of former Supreme Court Justice Florentino of facts.100
Feliciano, "judicial review is essential for the maintenance and
enforcement of the separation of powers and the balancing of powers Corollary to the requirement of an actual case or controversy is the
among the three great departments of government through the definition requirement of ripeness.101 A question is ripe for adjudication when the act
and maintenance of the boundaries of authority and control between being challenged has had a direct adverse effect on the individual
them. To him, judicial review is the chief, indeed the only, medium of challenging it. For a case to be considered ripe for adjudication, it is a
prerequisite that something has then been accomplished or performed by
either branch before a court may come into the picture, and the petitioner In United States (US) constitutional law, a facial challenge, also known as
must allege the existence of an immediate or threatened injury to himself a First Amendment Challenge, is one that is launched to assail the
as a result of the challenged action. He must show that he has sustained validity of statutes concerning not only protected speech, but also all
or is immediately in danger of sustaining some direct injury as a result of other rights in the First Amendment.106 These include religious freedom,
the act complained of102 freedom of the press, and the right of the people to peaceably assemble,
and to petition the Government for a redress of grievances.107 After all, the
In The Province of North Cotabato v. The Government of the Republic of fundamental right to religious freedom, freedom of the press and peaceful
the Philippines,103 where the constitutionality of an unimplemented assembly are but component rights of the right to one's freedom of
Memorandum of Agreement on the Ancestral Domain (MOA-AD) was put expression, as they are modes which one's thoughts are externalized.
in question, it was argued that the Court has no authority to pass upon
the issues raised as there was yet no concrete act performed that could In this jurisdiction, the application of doctrines originating from the U.S.
possibly violate the petitioners' and the intervenors' rights. Citing has been generally maintained, albeit with some modifications. While this
precedents, the Court ruled that the fact of the law or act in question Court has withheld the application of facial challenges to strictly penal
being not yet effective does not negate ripeness. Concrete acts under a statues,108 it has expanded its scope to cover statutes not only regulating
law are not necessary to render the controversy ripe. Even a singular free speech, but also those involving religious freedom, and other
violation of the Constitution and/or the law is enough to awaken judicial fundamental rights.109 The underlying reason for this modification is
duty. simple. For unlike its counterpart in the U.S., this Court, under its
expanded jurisdiction, is mandated by the Fundamental Law not only to
In this case, the Court is of the view that an actual case or controversy settle actual controversies involving rights which are legally demandable
exists and that the same is ripe for judicial determination. Considering and enforceable, but also to determine whether or not there has been a
that the RH Law and its implementing rules have already taken effect and grave abuse of discretion amounting to lack or excess of jurisdiction on
that budgetary measures to carry out the law have already been passed, the part of any branch or instrumentality of the Government.110 Verily, the
it is evident that the subject petitions present a justiciable controversy. As framers of Our Constitution envisioned a proactive Judiciary, ever vigilant
stated earlier, when an action of the legislative branch is seriously with its duty to maintain the supremacy of the Constitution.
alleged to have infringed the Constitution, it not only becomes a right, but
also a duty of the Judiciary to settle the dispute.104 Consequently, considering that the foregoing petitions have seriously
alleged that the constitutional human rights to life, speech and religion
Moreover, the petitioners have shown that the case is so because and other fundamental rights mentioned above have been violated by the
medical practitioners or medical providers are in danger of being assailed legislation, the Court has authority to take cognizance of these
criminally prosecuted under the RH Law for vague violations thereof, kindred petitions and to determine if the RH Law can indeed pass
particularly public health officers who are threatened to be dismissed constitutional scrutiny. To dismiss these petitions on the simple expedient
from the service with forfeiture of retirement and other benefits. They that there exist no actual case or controversy, would diminish this Court
must, at least, be heard on the matter NOW. as a reactive branch of government, acting only when the Fundamental
Law has been transgressed, to the detriment of the Filipino people.
Facial Challenge
Locus Standi
The OSG also assails the propriety of the facial challenge lodged by the
subject petitions, contending that the RH Law cannot be challenged "on The OSG also attacks the legal personality of the petitioners to file their
its face" as it is not a speech regulating measure.105 respective petitions. It contends that the "as applied challenge" lodged by
the petitioners cannot prosper as the assailed law has yet to be enforced
The Court is not persuaded. and applied against them,111 and the government has yet to distribute
reproductive health devices that are abortive.112
The petitioners, for their part, invariably invoke the "transcendental is, after all, a procedural technicality which the Court has, on more than
importance" doctrine and their status as citizens and taxpayers in one occasion, waived or relaxed, thus allowing non-traditional plaintiffs,
establishing the requisite locus standi. such as concerned citizens, taxpayers, voters or legislators, to sue in the
public interest, albeit they may not have been directly injured by the
Locus standi or legal standing is defined as a personal and substantial operation of a law or any other government act. As held in Jaworski v.
interest in a case such that the party has sustained or will sustain direct PAGCOR:119
injury as a result of the challenged governmental act.113 It requires a
personal stake in the outcome of the controversy as to assure the Granting arguendo that the present action cannot be properly treated as
concrete adverseness which sharpens the presentation of issues upon a petition for prohibition, the transcendental importance of the issues
which the court so largely depends for illumination of difficult involved in this case warrants that we set aside the technical defects and
constitutional questions.114 take primary jurisdiction over the petition at bar. One cannot deny that the
issues raised herein have potentially pervasive influence on the social
In relation to locus standi, the "as applied challenge" embodies the rule and moral well being of this nation, specially the youth; hence, their
that one can challenge the constitutionality of a statute only if he asserts proper and just determination is an imperative need. This is in
a violation of his own rights. The rule prohibits one from challenging the accordance with the well-entrenched principle that rules of procedure are
constitutionality of the statute grounded on a violation of the rights of third not inflexible tools designed to hinder or delay, but to facilitate and
persons not before the court. This rule is also known as the prohibition promote the administration of justice. Their strict and rigid application,
against third-party standing.115 which would result in technicalities that tend to frustrate, rather than
promote substantial justice, must always be eschewed. (Emphasis
Transcendental Importance supplied)

Notwithstanding, the Court leans on the doctrine that "the rule on In view of the seriousness, novelty and weight as precedents, not only to
standing is a matter of procedure, hence, can be relaxed for non- the public, but also to the bench and bar, the issues raised must be
traditional plaintiffs like ordinary citizens, taxpayers, and legislators when resolved for the guidance of all. After all, the RH Law drastically affects
the public interest so requires, such as when the matter is of the constitutional provisions on the right to life and health, the freedom of
transcendental importance, of overreaching significance to society, or of religion and expression and other constitutional rights. Mindful of all these
paramount public interest."116 and the fact that the issues of contraception and reproductive health have
already caused deep division among a broad spectrum of society, the
Court entertains no doubt that the petitions raise issues of transcendental
In Coconut Oil Refiners Association, Inc. v. Torres,117 the Court held that
importance warranting immediate court adjudication. More importantly,
in cases of paramount importance where serious constitutional questions
considering that it is the right to life of the mother and the unborn which is
are involved, the standing requirement may be relaxed and a suit may be
primarily at issue, the Court need not wait for a life to be taken away
allowed to prosper even where there is no direct injury to the party
before taking action.
claiming the right of judicial review. In the first Emergency Powers
Cases,118 ordinary citizens and taxpayers were allowed to question the
constitutionality of several executive orders although they had only an The Court cannot, and should not, exercise judicial restraint at this time
indirect and general interest shared in common with the public. when rights enshrined in the Constitution are being imperilled to be
violated. To do so, when the life of either the mother or her child is at
stake, would lead to irreparable consequences.
With these said, even if the constitutionality of the RH Law may not be
assailed through an "as-applied challenge, still, the Court has time and
again acted liberally on the locus s tandi requirement. It has accorded Declaratory Relief
certain individuals standing to sue, not otherwise directly injured or with
material interest affected by a Government act, provided a constitutional The respondents also assail the petitions because they are essentially
issue of transcendental importance is invoked. The rule on locus standi petitions for declaratory relief over which the Court has no original
jurisdiction.120 Suffice it to state that most of the petitions are praying for The Court, thus, agrees with the petitioners' contention that the whole
injunctive reliefs and so the Court would just consider them as petitions idea of contraception pervades the entire RH Law. It is, in fact, the central
for prohibition under Rule 65, over which it has original jurisdiction. idea of the RH Law.126 Indeed, remove the provisions that refer to
Where the case has far-reaching implications and prays for injunctive contraception or are related to it and the RH Law loses its very
reliefs, the Court may consider them as petitions for prohibition under foundation.127 As earlier explained, "the other positive provisions such as
Rule 65.121 skilled birth attendance, maternal care including pre-and post-natal
services, prevention and management of reproductive tract infections
One Subject-One Title including HIV/AIDS are already provided for in the Magna Carta for
Women."128
The petitioners also question the constitutionality of the RH Law, claiming
that it violates Section 26(1 ), Article VI of the Constitution,122 prescribing Be that as it may, the RH Law does not violate the one subject/one bill
the one subject-one title rule. According to them, being one for rule. In Benjamin E. Cawaling, Jr. v. The Commission on Elections and
reproductive health with responsible parenthood, the assailed legislation Rep. Francis Joseph G Escudero, it was written:
violates the constitutional standards of due process by concealing its true
intent - to act as a population control measure.123 It is well-settled that the "one title-one subject" rule does not require the
Congress to employ in the title of the enactment language of such
To belittle the challenge, the respondents insist that the RH Law is not a precision as to mirror, fully index or catalogue all the contents and the
birth or population control measure,124 and that the concepts of minute details therein. The rule is sufficiently complied with if the title is
"responsible parenthood" and "reproductive health" are both interrelated comprehensive enough as to include the general object which the statute
as they are inseparable.125 seeks to effect, and where, as here, the persons interested are informed
of the nature, scope and consequences of the proposed law and its
Despite efforts to push the RH Law as a reproductive health law, the operation. Moreover, this Court has invariably adopted a liberal rather
Court sees it as principally a population control measure. The corpus of than technical construction of the rule "so as not to cripple or impede
the RH Law is geared towards the reduction of the country's population. legislation." [Emphases supplied]
While it claims to save lives and keep our women and children healthy, it
also promotes pregnancy-preventing products. As stated earlier, the RH In this case, a textual analysis of the various provisions of the law shows
Law emphasizes the need to provide Filipinos, especially the poor and that both "reproductive health" and "responsible parenthood" are
the marginalized, with access to information on the full range of modem interrelated and germane to the overriding objective to control the
family planning products and methods. These family planning methods, population growth. As expressed in the first paragraph of Section 2 of the
natural or modem, however, are clearly geared towards the prevention of RH Law:
pregnancy.
SEC. 2. Declaration of Policy. - The State recognizes and guarantees the
For said reason, the manifest underlying objective of the RH Law is to human rights of all persons including their right to equality and
reduce the number of births in the country. nondiscrimination of these rights, the right to sustainable human
development, the right to health which includes reproductive health, the
It cannot be denied that the measure also seeks to provide pre-natal and right to education and information, and the right to choose and make
post-natal care as well. A large portion of the law, however, covers the decisions for themselves in accordance with their religious convictions,
dissemination of information and provisions on access to medically-safe, ethics, cultural beliefs, and the demands of responsible parenthood.
non-abortifacient, effective, legal, affordable, and quality reproductive
health care services, methods, devices, and supplies, which are all The one subject/one title rule expresses the principle that the title of a law
intended to prevent pregnancy. must not be "so uncertain that the average person reading it would not be
informed of the purpose of the enactment or put on inquiry as to its
contents, or which is misleading, either in referring to or indicating one
subject where another or different one is really embraced in the act, or in confirms that abortifacients are not prohibited. Also considering that the
omitting any expression or indication of the real subject or scope of the FDA is not the agency that will actually supervise or administer the use of
act."129 these products and supplies to prospective patients, there is no way it
can truthfully make a certification that it shall not be used for abortifacient
Considering the close intimacy between "reproductive health" and purposes.133
"responsible parenthood" which bears to the attainment of the goal of
achieving "sustainable human development" as stated under its terms, Position of the Respondents
the Court finds no reason to believe that Congress intentionally sought to
deceive the public as to the contents of the assailed legislation. For their part, the defenders of the RH Law point out that the intent of the
Framers of the Constitution was simply the prohibition of abortion. They
II - SUBSTANTIVE ISSUES: contend that the RH Law does not violate the Constitution since the said
law emphasizes that only "non-abortifacient" reproductive health care
1-The Right to Life services, methods, devices products and supplies shall be made
Position of the Petitioners accessible to the public.134

The petitioners assail the RH Law because it violates the right to life and According to the OSG, Congress has made a legislative determination
health of the unborn child under Section 12, Article II of the Constitution. that contraceptives are not abortifacients by enacting the RH Law. As the
The assailed legislation allowing access to abortifacients/abortives RH Law was enacted with due consideration to various studies and
effectively sanctions abortion.130 consultations with the World Health Organization (WHO) and other
experts in the medical field, it is asserted that the Court afford deference
According to the petitioners, despite its express terms prohibiting and respect to such a determination and pass judgment only when a
abortion, Section 4(a) of the RH Law considers contraceptives that particular drug or device is later on determined as an abortive.135
prevent the fertilized ovum to reach and be implanted in the mother's
womb as an abortifacient; thus, sanctioning contraceptives that take For his part, respondent Lagman argues that the constitutional protection
effect after fertilization and prior to implantation, contrary to the intent of of one's right to life is not violated considering that various studies of the
the Framers of the Constitution to afford protection to the fertilized ovum WHO show that life begins from the implantation of the fertilized ovum.
which already has life. Consequently, he argues that the RH Law is constitutional since the law
specifically provides that only contraceptives that do not prevent the
They argue that even if Section 9 of the RH Law allows only "non- implantation of the fertilized ovum are allowed.136
abortifacient" hormonal contraceptives, intrauterine devices, injectables
and other safe, legal, non-abortifacient and effective family planning The Court's Position
products and supplies, medical research shows that contraceptives use
results in abortion as they operate to kill the fertilized ovum which already It is a universally accepted principle that every human being enjoys the
has life.131 right to life.137

As it opposes the initiation of life, which is a fundamental human good, Even if not formally established, the right to life, being grounded on
the petitioners assert that the State sanction of contraceptive use natural law, is inherent and, therefore, not a creation of, or dependent
contravenes natural law and is an affront to the dignity of man.132 upon a particular law, custom, or belief. It precedes and transcends any
authority or the laws of men.
Finally, it is contended that since Section 9 of the RH Law requires the
Food and Drug Administration (FDA) to certify that the product or supply In this jurisdiction, the right to life is given more than ample protection.
is not to be used as an abortifacient, the assailed legislation effectively Section 1, Article III of the Constitution provides:
Section 1. No person shall be deprived of life, liberty, or property without Section 12. The State recognizes the sanctity of family life and shall
due process of law, nor shall any person be denied the equal protection protect and strengthen the family as a basic autonomous social
of the laws. institution. It shall equally protect the life of the mother and the life of the
unborn from conception. The natural and primary right and duty of
As expounded earlier, the use of contraceptives and family planning parents in the rearing of the youth for civic efficiency and the
methods in the Philippines is not of recent vintage. From the enactment development of moral character shall receive the support of the
of R.A. No. 4729, entitled "An Act To Regulate The Sale, Dispensation, Government.
and/or Distribution of Contraceptive Drugs and Devices "on June 18,
1966, prescribing rules on contraceptive drugs and devices which Textually, the Constitution affords protection to the unborn from
prevent fertilization,138 to the promotion of male vasectomy and tubal conception. This is undisputable because before conception, there is no
ligation,139 and the ratification of numerous international agreements, the unborn to speak of. For said reason, it is no surprise that the Constitution
country has long recognized the need to promote population control is mute as to any proscription prior to conception or when life begins. The
through the use of contraceptives in order to achieve long-term economic problem has arisen because, amazingly, there are quarters who have
development. Through the years, however, the use of contraceptives and conveniently disregarded the scientific fact that conception is reckoned
other family planning methods evolved from being a component of from fertilization. They are waving the view that life begins at
demographic management, to one centered on the promotion of public implantation. Hence, the issue of when life begins.
health, particularly, reproductive health.140
In a nutshell, those opposing the RH Law contend that conception is
This has resulted in the enactment of various measures promoting synonymous with "fertilization" of the female ovum by the male
women's rights and health and the overall promotion of the family's well- sperm.142 On the other side of the spectrum are those who assert that
being. Thus, aside from R.A. No. 4729, R.A. No. 6365 or "The Population conception refers to the "implantation" of the fertilized ovum in the
Act of the Philippines" and R.A. No. 9710, otherwise known as the "The uterus.143
Magna Carta of Women" were legislated. Notwithstanding this paradigm
shift, the Philippine national population program has always been Plain and Legal Meaning
grounded two cornerstone principles: "principle of no-abortion" and the
"principle of non-coercion."141 As will be discussed later, these principles It is a canon in statutory construction that the words of the Constitution
are not merely grounded on administrative policy, but rather, originates should be interpreted in their plain and ordinary meaning. As held in the
from the constitutional protection expressly provided to afford protection recent case of Chavez v. Judicial Bar Council:144
to life and guarantee religious freedom.
One of the primary and basic rules in statutory construction is that where
When Life Begins* the words of a statute are clear, plain, and free from ambiguity, it must be
given its literal meaning and applied without attempted interpretation. It is
Majority of the Members of the Court are of the position that the question a well-settled principle of constitutional construction that the language
of when life begins is a scientific and medical issue that should not be employed in the Constitution must be given their ordinary meaning
decided, at this stage, without proper hearing and evidence. During the except where technical terms are employed. As much as possible, the
deliberation, however, it was agreed upon that the individual members of words of the Constitution should be understood in the sense they have in
the Court could express their own views on this matter. common use. What it says according to the text of the provision to be
construed compels acceptance and negates the power of the courts to
In this regard, the ponente, is of the strong view that life begins at alter it, based on the postulate that the framers and the people mean
fertilization. what they say. Verba legis non est recedendum - from the words of a
statute there should be no departure.
In answering the question of when life begins, focus should be made on
the particular phrase of Section 12 which reads:
The raison d' etre for the rule is essentially two-fold: First, because it is Records of the Constitutional Convention also shed light on the intention
assumed that the words in which constitutional provisions are couched of the Framers regarding the term "conception" used in Section 12,
express the objective sought to be attained; and second, because the Article II of the Constitution. From their deliberations, it clearly refers to
Constitution is not primarily a lawyer's document but essentially that of the moment of "fertilization." The records reflect the following:
the people, in whose consciousness it should ever be present as an
important condition for the rule of law to prevail. Rev. Rigos: In Section 9, page 3, there is a sentence which reads:

In conformity with the above principle, the traditional meaning of the word "The State shall equally protect the life of the mother and the life of the
"conception" which, as described and defined by all reliable and unborn from the moment of conception."
reputable sources, means that life begins at fertilization.
When is the moment of conception?
Webster's Third New International Dictionary describes it as the act of
becoming pregnant, formation of a viable zygote; the fertilization that xxx
results in a new entity capable of developing into a being like its
parents.145
Mr. Villegas: As I explained in the sponsorship speech, it is when the
ovum is fertilized by the sperm that there is human life. x x x.150
Black's Law Dictionary gives legal meaning to the term "conception" as
the fecundation of the female ovum by the male spermatozoon resulting
xxx
in human life capable of survival and maturation under normal
conditions.146
As to why conception is reckoned from fertilization and, as such, the
beginning of human life, it was explained:
Even in jurisprudence, an unborn child has already a legal personality. In
Continental Steel Manufacturing Corporation v. Hon. Accredited
Voluntary Arbitrator Allan S. Montano,147 it was written: Mr. Villegas: I propose to review this issue in a biological manner. The
first question that needs to be answered is: Is the fertilized ovum alive?
Biologically categorically says yes, the fertilized ovum is alive. First of all,
Life is not synonymous with civil personality. One need not acquire civil
like all living organisms, it takes in nutrients which it processes by itself. It
personality first before he/she could die. Even a child inside the womb
begins doing this upon fertilization. Secondly, as it takes in these
already has life. No less than the Constitution recognizes the life of the
nutrients, it grows from within. Thirdly, it multiplies itself at a geometric
unborn from conception, that the State must protect equally with the life
rate in the continuous process of cell division. All these processes are
of the mother. If the unborn already has life, then the cessation thereof
vital signs of life. Therefore, there is no question that biologically the
even prior to the child being delivered, qualifies as death. [Emphases in
fertilized ovum has life.
the original]
The second question: Is it human? Genetics gives an equally categorical
In Gonzales v. Carhart,148 Justice Anthony Kennedy, writing for the US
"yes." At the moment of conception, the nuclei of the ovum and the sperm
Supreme Court, said that the State "has respect for human life at all
rupture. As this happens 23 chromosomes from the ovum combine with
stages in the pregnancy" and "a legitimate and substantial interest in
23 chromosomes of the sperm to form a total of 46 chromosomes. A
preserving and promoting fetal life." Invariably, in the decision, the fetus
chromosome count of 46 is found only - and I repeat, only in human cells.
was referred to, or cited, as a baby or a child.149
Therefore, the fertilized ovum is human.
Intent of the Framers
Since these questions have been answered affirmatively, we must
conclude that if the fertilized ovum is both alive and human, then, as night
follows day, it must be human life. Its nature is human.151
Why the Constitution used the phrase "from the moment of conception" contraceptives deter the rooting of the ovum in the uterus. If fertilization
and not "from the moment of fertilization" was not because of doubt when has already occurred, the next process is for the fertilized ovum to travel
human life begins, but rather, because: towards the uterus and to take root. What happens with some
contraceptives is that they stop the opportunity for the fertilized ovum to
Mr. Tingson: x x x x the phrase from the moment of conception" was reach the uterus. Therefore, if we take the provision as it is proposed,
described by us here before with the scientific phrase "fertilized ovum" these so called contraceptives should be banned.
may be beyond the comprehension of some people; we want to use the
simpler phrase "from the moment of conception."152 Mr. Villegas: Yes, if that physical fact is established, then that is what is
called abortifacient and, therefore, would be unconstitutional and should
Thus, in order to ensure that the fertilized ovum is given ample protection be banned under this provision.
under the Constitution, it was discussed:
Mr. Gascon: Yes. So my point is that I do not think it is up to Congress to
Rev. Rigos: Yes, we think that the word "unborn" is sufficient for the state whether or not these certain contraceptives are abortifacient.
purpose of writing a Constitution, without specifying "from the moment of Scientifically and based on the provision as it is now proposed, they are
conception." already considered abortifacient.154

Mr. Davide: I would not subscribe to that particular view because From the deliberations above-quoted, it is apparent that the Framers of
according to the Commissioner's own admission, he would leave it to the Constitution emphasized that the State shall provide equal protection
Congress to define when life begins. So, Congress can define life to to both the mother and the unborn child from the earliest opportunity of
begin from six months after fertilization; and that would really be very, life, that is, upon fertilization or upon the union of the male sperm and the
very, dangerous. It is now determined by science that life begins from the female ovum. It is also apparent is that the Framers of the Constitution
moment of conception. There can be no doubt about it. So we should not intended that to prohibit Congress from enacting measures that would
give any doubt to Congress, too.153 allow it determine when life begins.

Upon further inquiry, it was asked: Equally apparent, however, is that the Framers of the Constitution did not
intend to ban all contraceptives for being unconstitutional. In fact,
Mr. Gascon: Mr. Presiding Officer, I would like to ask a question on that Commissioner Bernardo Villegas, spearheading the need to have a
point. Actually, that is one of the questions I was going to raise during the constitutional provision on the right to life, recognized that the
period of interpellations but it has been expressed already. The provision, determination of whether a contraceptive device is an abortifacient is a
as proposed right now states: question of fact which should be left to the courts to decide on based on
established evidence.155
The State shall equally protect the life of the mother and the life of the
unborn from the moment of conception. From the discussions above, contraceptives that kill or destroy the
fertilized ovum should be deemed an abortive and thus prohibited.
Conversely, contraceptives that actually prevent the union of the male
When it speaks of "from the moment of conception," does this mean
sperm and the female ovum, and those that similarly take action prior to
when the egg meets the sperm?
fertilization should be deemed non-abortive, and thus, constitutionally
permissible.
Mr. Villegas: Yes, the ovum is fertilized by the sperm.
As emphasized by the Framers of the Constitution:
Mr. Gascon: Therefore that does not leave to Congress the right to
determine whether certain contraceptives that we know today are
xxx xxx xxx
abortifacient or not because it is a fact that some of the so-called
Mr. Gascon: xx xx. As I mentioned in my speech on the US bases, I am Atty. Noche:
pro-life, to the point that I would like not only to protect the life of the
unborn, but also the lives of the millions of people in the world by fighting So, there is no life to be protected.
for a nuclear-free world. I would just like to be assured of the legal and
pragmatic implications of the term "protection of the life of the unborn Justice Bersamin:
from the moment of conception." I raised some of these implications this
afternoon when I interjected in the interpellation of Commissioner
To be protected.
Regalado. I would like to ask that question again for a categorical
answer.
Atty. Noche:
I mentioned that if we institutionalize the term "the life of the unborn from
the moment of conception" we are also actually saying "no," not "maybe," Under Section 12, yes.
to certain contraceptives which are already being encouraged at this
point in time. Is that the sense of the committee or does it disagree with Justice Bersamin:
me?
So you have no objection to condoms?
Mr. Azcuna: No, Mr. Presiding Officer, because contraceptives would be
preventive. There is no unborn yet. That is yet unshaped. Atty. Noche:

Mr. Gascon: Yes, Mr. Presiding Officer, but I was speaking more about Not under Section 12, Article II.
some contraceptives, such as the intra-uterine device which actually
stops the egg which has already been fertilized from taking route to the Justice Bersamin:
uterus. So if we say "from the moment of conception," what really occurs
is that some of these contraceptives will have to be unconstitutionalized. Even if there is already information that condoms sometimes have
porosity?
Mr. Azcuna: Yes, to the extent that it is after the fertilization.
Atty. Noche:
Mr. Gascon: Thank you, Mr. Presiding Officer.156
Well, yes, Your Honor, there are scientific findings to that effect, Your
The fact that not all contraceptives are prohibited by the 1987 Honor, but I am discussing here Section 12, Article II, Your Honor, yes.
Constitution is even admitted by petitioners during the oral arguments.
There it was conceded that tubal ligation, vasectomy, even condoms are Justice Bersamin:
not classified as abortifacients.157
Alright.
Atty. Noche:
Atty. Noche:
Before the union of the eggs, egg and the sperm, there is no life yet.
And it's not, I have to admit it's not an abortifacient, Your Honor.158
Justice Bersamin:
Medical Meaning
There is no life.
That conception begins at fertilization is not bereft of medical foundation. those new lives is never licit, no matter what the purported good outcome
Mosby s Medical, Nursing, and Allied Health Dictionary defines would be. In terms of biology and human embryology, a human being
conception as "the beginning of pregnancy usually taken to be the instant begins immediately at fertilization and after that, there is no point along
a spermatozoon enters an ovum and forms a viable zygote."159 the continuous line of human embryogenesis where only a "potential"
human being can be posited. Any philosophical, legal, or political
It describes fertilization as "the union of male and female gametes to form conclusion cannot escape this objective scientific fact.
a zygote from which the embryo develops."160
The scientific evidence supports the conclusion that a zygote is a human
The Textbook of Obstetrics (Physiological & Pathological organism and that the life of a new human being commences at a
Obstetrics),161 used by medical schools in the Philippines, also concludes scientifically well defined "moment of conception." This conclusion is
that human life (human person) begins at the moment of fertilization with objective, consistent with the factual evidence, and independent of any
the union of the egg and the sperm resulting in the formation of a new specific ethical, moral, political, or religious view of human life or of
individual, with a unique genetic composition that dictates all human embryos.164
developmental stages that ensue.
Conclusion: The Moment of Conception is Reckoned from
Similarly, recent medical research on the matter also reveals that: Fertilization
"Human development begins after the union of male and female gametes
or germ cells during a process known as fertilization (conception). In all, whether it be taken from a plain meaning, or understood under
Fertilization is a sequence of events that begins with the contact of a medical parlance, and more importantly, following the intention of the
sperm (spermatozoon) with a secondary oocyte (ovum) and ends with the Framers of the Constitution, the undeniable conclusion is that a zygote is
fusion of their pronuclei (the haploid nuclei of the sperm and ovum) and a human organism and that the life of a new human being commences at
the mingling of their chromosomes to form a new cell. This fertilized a scientifically well-defined moment of conception, that is, upon
ovum, known as a zygote, is a large diploid cell that is the beginning, or fertilization.
primordium, of a human being."162
For the above reasons, the Court cannot subscribe to the theory
The authors of Human Embryology & Teratology163 mirror the same advocated by Hon. Lagman that life begins at implantation.165 According to
position. They wrote: "Although life is a continuous process, fertilization is him, "fertilization and conception are two distinct and successive stages
a critical landmark because, under ordinary circumstances, a new, in the reproductive process. They are not identical and
genetically distinct human organism is thereby formed.... The synonymous."166 Citing a letter of the WHO, he wrote that "medical
combination of 23 chromosomes present in each pronucleus results in 46 authorities confirm that the implantation of the fertilized ovum is the
chromosomes in the zygote. Thus the diploid number is restored and the commencement of conception and it is only after implantation that
embryonic genome is formed. The embryo now exists as a genetic unity." pregnancy can be medically detected."167

In support of the RH Bill, The Philippine Medical Association came out This theory of implantation as the beginning of life is devoid of any legal
with a "Paper on the Reproductive Health Bill (Responsible Parenthood or scientific mooring. It does not pertain to the beginning of life but to the
Bill)" and therein concluded that: viability of the fetus. The fertilized ovum/zygote is not an inanimate object
- it is a living human being complete with DNA and 46
CONCLUSION chromosomes.168 Implantation has been conceptualized only for
convenience by those who had population control in mind. To adopt it
The PMA throws its full weight in supporting the RH Bill at the same time would constitute textual infidelity not only to the RH Law but also to the
that PMA maintains its strong position that fertilization is sacred because Constitution.
it is at this stage that conception, and thus human life, begins. Human
lives are sacred from the moment of conception, and that destroying Not surprisingly, even the OSG does not support this position.
If such theory would be accepted, it would unnervingly legitimize the (q) Reproductive health care refers to the access to a full range of
utilization of any drug or device that would prevent the implantation of the methods, facilities, services and supplies that contribute to reproductive
fetus at the uterine wall. It would be provocative and further aggravate health and well-being by addressing reproductive health-related
religious-based divisiveness. problems. It also includes sexual health, the purpose of which is the
enhancement of life and personal relations. The elements of reproductive
It would legally permit what the Constitution proscribes - abortion and health care include the following:
abortifacients.
xxx.
The RH Law and Abortion
(3) Proscription of abortion and management of abortion complications;
The clear and unequivocal intent of the Framers of the 1987 Constitution
in protecting the life of the unborn from conception was to prevent the xxx.
Legislature from enacting a measure legalizing abortion. It was so clear
that even the Court cannot interpret it otherwise. This intent of the 2] xx x.
Framers was captured in the record of the proceedings of the 1986
Constitutional Commission. Commissioner Bernardo Villegas, the Section 4. x x x.
principal proponent of the protection of the unborn from conception,
explained:
(s) Reproductive health rights refers to the rights of individuals and
couples, to decide freely and responsibly whether or not to have children;
The intention .. .is to make sure that there would be no pro-abortion laws the number, spacing and timing of their children; to make other decisions
ever passed by Congress or any pro-abortion decision passed by the concerning reproduction, free of discrimination, coercion and violence; to
Supreme Court.169 have the information and means to do so; and to attain the highest
standard of sexual health and reproductive health: Provided, however,
A reading of the RH Law would show that it is in line with this intent and That reproductive health rights do not include abortion, and access to
actually proscribes abortion. While the Court has opted not to make any abortifacients.
determination, at this stage, when life begins, it finds that the RH Law
itself clearly mandates that protection be afforded from the moment of 3] xx x.
fertilization. As pointed out by Justice Carpio, the RH Law is replete with
provisions that embody the policy of the law to protect to the fertilized
SEC. 29. Repealing Clause. - Except for prevailing laws against abortion,
ovum and that it should be afforded safe travel to the uterus for
any law, presidential decree or issuance, executive order, letter of
implantation.170
instruction, administrative order, rule or regulation contrary to or is
inconsistent with the provisions of this Act including Republic Act No.
Moreover, the RH Law recognizes that abortion is a crime under Article 7392, otherwise known as the Midwifery Act, is hereby repealed,
256 of the Revised Penal Code, which penalizes the destruction or modified or amended accordingly.
expulsion of the fertilized ovum. Thus:
The RH Law and Abortifacients
1] xx x.
In carrying out its declared policy, the RH Law is consistent in prohibiting
Section 4. Definition of Terms. - For the purpose of this Act, the following abortifacients. To be clear, Section 4(a) of the RH Law defines an
terms shall be defined as follows: abortifacient as:

xxx. Section 4. Definition of Terms - x x x x


(a) Abortifacient refers to any drug or device that induces abortion or the From the foregoing, the Court finds that inasmuch as it affords protection
destruction of a fetus inside the mother's womb or the prevention of the to the fertilized ovum, the RH Law does not sanction abortion. To repeat,
fertilized ovum to reach and be implanted in the mother's womb upon it is the Court's position that life begins at fertilization, not at implantation.
determination of the FDA. When a fertilized ovum is implanted in the uterine wall , its viability is
sustained but that instance of implantation is not the point of beginning of
As stated above, the RH Law mandates that protection must be afforded life. It started earlier. And as defined by the RH Law, any drug or device
from the moment of fertilization. By using the word " or," the RH Law that induces abortion, that is, which kills or destroys the fertilized ovum or
prohibits not only drugs or devices that prevent implantation, but also prevents the fertilized ovum to reach and be implanted in the mother's
those that induce abortion and those that induce the destruction of a womb, is an abortifacient.
fetus inside the mother's womb. Thus, an abortifacient is any drug or
device that either: Proviso Under Section 9 of the RH Law

(a) Induces abortion; or This notwithstanding, the Court finds that the proviso under Section 9 of
the law that "any product or supply included or to be included in the EDL
(b) Induces the destruction of a fetus inside the mother's womb; must have a certification from the FDA that said product and supply is
or made available on the condition that it is not to be used as an
abortifacient" as empty as it is absurd. The FDA, with all its expertise,
(c) Prevents the fertilized ovum to reach and be implanted in the cannot fully attest that a drug or device will not all be used as an
mother's womb, upon determination of the FDA. abortifacient, since the agency cannot be present in every instance when
the contraceptive product or supply will be used.171
Contrary to the assertions made by the petitioners, the Court finds that
the RH Law, consistent with the Constitution, recognizes that the Pursuant to its declared policy of providing access only to safe, legal and
fertilized ovum already has life and that the State has a bounden duty to non-abortifacient contraceptives, however, the Court finds that the
protect it. The conclusion becomes clear because the RH Law, first, proviso of Section 9, as worded, should bend to the legislative intent and
prohibits any drug or device that induces abortion (first kind), which, as mean that "any product or supply included or to be included in the EDL
discussed exhaustively above, refers to that which induces the killing or must have a certification from the FDA that said product and supply is
the destruction of the fertilized ovum, and, second, prohibits any drug or made available on the condition that it cannot be used as abortifacient."
device the fertilized ovum to reach and be implanted in the mother's Such a construction is consistent with the proviso under the second
womb (third kind). paragraph of the same section that provides:

By expressly declaring that any drug or device that prevents the fertilized Provided, further, That the foregoing offices shall not purchase or acquire
ovum to reach and be implanted in the mother's womb is an abortifacient by any means emergency contraceptive pills, postcoital pills,
(third kind), the RH Law does not intend to mean at all that life only abortifacients that will be used for such purpose and their other forms or
begins only at implantation, as Hon. Lagman suggests. It also does not equivalent.
declare either that protection will only be given upon implantation, as the
petitioners likewise suggest. Rather, it recognizes that: one, there is a Abortifacients under the RH-IRR
need to protect the fertilized ovum which already has life, and two, the
fertilized ovum must be protected the moment it becomes existent - all At this juncture, the Court agrees with ALFI that the authors of the RH-
the way until it reaches and implants in the mother's womb. After all, if life IRR gravely abused their office when they redefined the meaning of
is only recognized and afforded protection from the moment the fertilized abortifacient. The RH Law defines "abortifacient" as follows:
ovum implants - there is nothing to prevent any drug or device from killing
or destroying the fertilized ovum prior to implantation. SEC. 4. Definition of Terms. - For the purpose of this Act, the following
terms shall be defined as follows:
(a) Abortifacient refers to any drug or device that induces abortion or the conception/fertilization in violation of Article II, Section 12 of the
destruction of a fetus inside the mother's womb or the prevention of the Constitution. With such qualification in the RH-IRR, it appears to
fertilized ovum to reach and be implanted in the mother's womb upon insinuate that a contraceptive will only be considered as an "abortifacient"
determination of the FDA. if its sole known effect is abortion or, as pertinent here, the prevention of
the implantation of the fertilized ovum.
Section 3.0l (a) of the IRR, however, redefines "abortifacient" as:
For the same reason, this definition of "contraceptive" would permit the
Section 3.01 For purposes of these Rules, the terms shall be defined as approval of contraceptives which are actually abortifacients because of
follows: their fail-safe mechanism.174

a) Abortifacient refers to any drug or device that primarily induces Also, as discussed earlier, Section 9 calls for the certification by the FDA
abortion or the destruction of a fetus inside the mother's womb or the that these contraceptives cannot act as abortive. With this, together with
prevention of the fertilized ovum to reach and be implanted in the the definition of an abortifacient under Section 4 (a) of the RH Law and its
mother's womb upon determination of the Food and Drug Administration declared policy against abortion, the undeniable conclusion is that
(FDA). [Emphasis supplied] contraceptives to be included in the PNDFS and the EDL will not only be
those contraceptives that do not have the primary action of causing
Again in Section 3.0lG) of the RH-IRR, "contraceptive," is redefined, viz: abortion or the destruction of a fetus inside the mother's womb or the
prevention of the fertilized ovum to reach and be implanted in the
mother's womb, but also those that do not have the secondary action of
j) Contraceptive refers to any safe, legal, effective and scientifically
acting the same way.
proven modern family planning method, device, or health product,
whether natural or artificial, that prevents pregnancy but does not
primarily destroy a fertilized ovum or prevent a fertilized ovum from being Indeed, consistent with the constitutional policy prohibiting abortion, and
implanted in the mother's womb in doses of its approved indication as in line with the principle that laws should be construed in a manner that
determined by the Food and Drug Administration (FDA). its constitutionality is sustained, the RH Law and its implementing rules
must be consistent with each other in prohibiting abortion. Thus, the word
" primarily" in Section 3.0l(a) and G) of the RH-IRR should be declared
The above-mentioned section of the RH-IRR allows "contraceptives" and
void. To uphold the validity of Section 3.0l(a) and G) of the RH-IRR and
recognizes as "abortifacient" only those that primarily induce abortion or
prohibit only those contraceptives that have the primary effect of being an
the destruction of a fetus inside the mother's womb or the prevention of
abortive would effectively "open the floodgates to the approval of
the fertilized ovum to reach and be implanted in the mother's womb.172
contraceptives which may harm or destroy the life of the unborn from
conception/fertilization in violation of Article II, Section 12 of the
This cannot be done. Constitution."175

In this regard, the observations of Justice Brion and Justice Del Castillo To repeat and emphasize, in all cases, the "principle of no abortion"
are well taken. As they pointed out, with the insertion of the word embodied in the constitutional protection of life must be upheld.
"primarily," Section 3.0l(a) and G) of the RH-IRR173 must be struck down
for being ultra vires.
2-The Right to Health
Evidently, with the addition of the word "primarily," in Section 3.0l(a) and
The petitioners claim that the RH Law violates the right to health because
G) of the RH-IRR is indeed ultra vires. It contravenes Section 4(a) of the
it requires the inclusion of hormonal contraceptives, intrauterine devices,
RH Law and should, therefore, be declared invalid. There is danger that
injectables and family products and supplies in the National Drug
the insertion of the qualifier "primarily" will pave the way for the approval
Formulary and the inclusion of the same in the regular purchase of
of contraceptives which may harm or destroy the life of the unborn from
essential medicines and supplies of all national hospitals.176Citing various
studies on the matter, the petitioners posit that the risk of developing Section 12. The State shall establish and maintain an effective food and
breast and cervical cancer is greatly increased in women who use oral drug regulatory system and undertake appropriate health, manpower
contraceptives as compared to women who never use them. They point development, and research, responsive to the country's health needs and
out that the risk is decreased when the use of contraceptives is problems.
discontinued. Further, it is contended that the use of combined oral
contraceptive pills is associated with a threefold increased risk of venous Section 13. The State shall establish a special agency for disabled
thromboembolism, a twofold increased risk of ischematic stroke, and an person for their rehabilitation, self-development, and self-reliance, and
indeterminate effect on risk of myocardial infarction.177 Given the definition their integration into the mainstream of society.
of "reproductive health" and "sexual health" under Sections 4(p)178 and
(w)179 of the RH Law, the petitioners assert that the assailed legislation Finally, Section 9, Article XVI provides:
only seeks to ensure that women have pleasurable and satisfying sex
lives.180
Section 9. The State shall protect consumers from trade malpractices and
from substandard or hazardous products.
The OSG, however, points out that Section 15, Article II of the
Constitution is not self-executory, it being a mere statement of the
Contrary to the respondent's notion, however, these provisions are self-
administration's principle and policy. Even if it were self-executory, the
executing. Unless the provisions clearly express the contrary, the
OSG posits that medical authorities refute the claim that contraceptive
provisions of the Constitution should be considered self-executory. There
pose a danger to the health of women.181
is no need for legislation to implement these self-executing
provisions.182 In Manila Prince Hotel v. GSIS,183 it was stated:
The Court's Position
x x x Hence, unless it is expressly provided that a legislative act is
A component to the right to life is the constitutional right to health. In this necessary to enforce a constitutional mandate, the presumption now is
regard, the Constitution is replete with provisions protecting and that all provisions of the constitution are self-executing. If the
promoting the right to health. Section 15, Article II of the Constitution constitutional provisions are treated as requiring legislation instead of
provides: self-executing, the legislature would have the power to ignore and
practically nullify the mandate of the fundamental law. This can be
Section 15. The State shall protect and promote the right to health of the cataclysmic. That is why the prevailing view is, as it has always been,
people and instill health consciousness among them. that –

A portion of Article XIII also specifically provides for the States' duty to ... in case of doubt, the Constitution should be considered self-executing
provide for the health of the people, viz: rather than non-self-executing. . . . Unless the contrary is clearly
intended, the provisions of the Constitution should be considered self-
HEALTH executing, as a contrary rule would give the legislature discretion to
determine when, or whether, they shall be effective. These provisions
Section 11. The State shall adopt an integrated and comprehensive would be subordinated to the will of the lawmaking body, which could
approach to health development which shall endeavor to make essential make them entirely meaningless by simply refusing to pass the needed
goods, health and other social services available to all the people at implementing statute. (Emphases supplied)
affordable cost. There shall be priority for the needs of the
underprivileged, sick, elderly, disabled, women, and children. The State This notwithstanding, it bears mentioning that the petitioners, particularly
shall endeavor to provide free medical care to paupers. ALFI, do not question contraception and contraceptives per se.184 In fact,
ALFI prays that the status quo - under R.A. No. 5921 and R.A. No. 4729,
the sale and distribution of contraceptives are not prohibited when they
are dispensed by a prescription of a duly licensed by a physician - be "(a) "Contraceptive drug" is any medicine, drug, chemical, or
maintained.185 portion which is used exclusively for the purpose of preventing
fertilization of the female ovum: and
The legislative intent in the enactment of the RH Law in this regard is to
leave intact the provisions of R.A. No. 4729. There is no intention at all to "(b) "Contraceptive device" is any instrument, device, material, or
do away with it. It is still a good law and its requirements are still in to be agent introduced into the female reproductive system for the
complied with. Thus, the Court agrees with the observation of respondent primary purpose of preventing conception.
Lagman that the effectivity of the RH Law will not lead to the unmitigated
proliferation of contraceptives since the sale, distribution and "Sec. 3 Any person, partnership, or corporation, violating the provisions
dispensation of contraceptive drugs and devices will still require the of this Act shall be punished with a fine of not more than five hundred
prescription of a licensed physician. With R.A. No. 4729 in place, there pesos or an imprisonment of not less than six months or more than one
exists adequate safeguards to ensure the public that only contraceptives year or both in the discretion of the Court.
that are safe are made available to the public. As aptly explained by
respondent Lagman: "This Act shall take effect upon its approval.

D. Contraceptives cannot be "Approved: June 18, 1966"


dispensed and used without
prescription
111. Of the same import, but in a general manner, Section 25 of RA No.
5921 provides:
108. As an added protection to voluntary users of contraceptives, the
same cannot be dispensed and used without prescription.
"Section 25. Sale of medicine, pharmaceuticals, drugs and devices. No
medicine, pharmaceutical, or drug of whatever nature and kind or device
109. Republic Act No. 4729 or "An Act to Regulate the Sale, shall be compounded, dispensed, sold or resold, or otherwise be made
Dispensation, and/ or Distribution of Contraceptive Drugs and Devices" available to the consuming public except through a prescription drugstore
and Republic Act No. 5921 or "An Act Regulating the Practice of or hospital pharmacy, duly established in accordance with the provisions
Pharmacy and Setting Standards of Pharmaceutical Education in the of this Act.
Philippines and for Other Purposes" are not repealed by the RH Law and
the provisions of said Acts are not inconsistent with the RH Law.
112. With all of the foregoing safeguards, as provided for in the RH Law
and other relevant statutes, the pretension of the petitioners that the RH
110. Consequently, the sale, distribution and dispensation of Law will lead to the unmitigated proliferation of contraceptives, whether
contraceptive drugs and devices are particularly governed by RA No. harmful or not, is completely unwarranted and baseless.186 [Emphases in
4729 which provides in full: the Original. Underlining supplied.]

"Section 1. It shall be unlawful for any person, partnership, or corporation, In Re: Section 10 of the RH Law:
to sell, dispense or otherwise distribute whether for or without
consideration, any contraceptive drug or device, unless such sale,
The foregoing safeguards should be read in connection with Section 10
dispensation or distribution is by a duly licensed drug store or
of the RH Law which provides:
pharmaceutical company and with the prescription of a qualified medical
practitioner.
SEC. 10. Procurement and Distribution of Family Planning Supplies. -
The DOH shall procure, distribute to LGUs and monitor the usage of
"Sec. 2 . For the purpose of this Act:
family planning supplies for the whole country. The DOH shall coordinate
with all appropriate local government bodies to plan and implement this
procurement and distribution program. The supply and budget allotments been tested, evaluated, and approved by the FDA. The FDA, not
shall be based on, among others, the current levels and projections of the Congress, has the expertise to determine whether a particular hormonal
following: contraceptive or intrauterine device is safe and non-abortifacient. The
provision of the third sentence concerning the requirements for the
(a) Number of women of reproductive age and couples who want inclusion or removal of a particular family planning supply from the EDL
to space or limit their children; supports this construction.

(b) Contraceptive prevalence rate, by type of method used; and Stated differently, the provision in Section 9 covering the inclusion of
hormonal contraceptives, intra-uterine devices, injectables, and other
(c) Cost of family planning supplies. safe, legal, non-abortifacient and effective family planning products and
supplies by the National Drug Formulary in the EDL is not mandatory.
There must first be a determination by the FDA that they are in fact safe,
Provided, That LGUs may implement its own procurement, distribution
legal, non-abortifacient and effective family planning products and
and monitoring program consistent with the overall provisions of this Act
supplies. There can be no predetermination by Congress that the gamut
and the guidelines of the DOH.
of contraceptives are "safe, legal, non-abortifacient and effective" without
the proper scientific examination.
Thus, in the distribution by the DOH of contraceptive drugs and devices,
it must consider the provisions of R.A. No. 4729, which is still in effect,
3 -Freedom of Religion
and ensure that the contraceptives that it will procure shall be from a duly
and the Right to Free Speech
licensed drug store or pharmaceutical company and that the actual
dispensation of these contraceptive drugs and devices will done following
a prescription of a qualified medical practitioner. The distribution of Position of the Petitioners:
contraceptive drugs and devices must not be indiscriminately done. The
public health must be protected by all possible means. As pointed out by 1. On Contraception
Justice De Castro, a heavy responsibility and burden are assumed by the
government in supplying contraceptive drugs and devices, for it may be While contraceptives and procedures like vasectomy and tubal ligation
held accountable for any injury, illness or loss of life resulting from or are not covered by the constitutional proscription, there are those who,
incidental to their use.187 because of their religious education and background, sincerely believe
that contraceptives, whether abortifacient or not, are evil. Some of these
At any rate, it bears pointing out that not a single contraceptive has yet are medical practitioners who essentially claim that their beliefs prohibit
been submitted to the FDA pursuant to the RH Law. It behooves the not only the use of contraceptives but also the willing participation and
Court to await its determination which drugs or devices are declared by cooperation in all things dealing with contraceptive use. Petitioner PAX
the FDA as safe, it being the agency tasked to ensure that food and explained that "contraception is gravely opposed to marital chastity, it is
medicines available to the public are safe for public consumption. contrary to the good of the transmission of life, and to the reciprocal self-
Consequently, the Court finds that, at this point, the attack on the RH giving of the spouses; it harms true love and denies the sovereign rule of
Law on this ground is premature. Indeed, the various kinds of God in the transmission of Human life."188
contraceptives must first be measured up to the constitutional yardstick
as expounded herein, to be determined as the case presents itself. The petitioners question the State-sponsored procurement of
contraceptives, arguing that the expenditure of their taxes on
At this point, the Court is of the strong view that Congress cannot contraceptives violates the guarantee of religious freedom since
legislate that hormonal contraceptives and intra-uterine devices are safe contraceptives contravene their religious beliefs.189
and non-abortifacient. The first sentence of Section 9 that ordains their
inclusion by the National Drug Formulary in the EDL by using the 2. On Religious Accommodation and
mandatory "shall" is to be construed as operative only after they have The Duty to Refer
Petitioners Imbong and Luat note that while the RH Law attempts to emergency, risk or threat that endangers state interests. It does not
address religious sentiments by making provisions for a conscientious explain how the rights of the people (to equality, non-discrimination of
objector, the constitutional guarantee is nonetheless violated because the rights, sustainable human development, health, education, information,
law also imposes upon the conscientious objector the duty to refer the choice and to make decisions according to religious convictions, ethics,
patient seeking reproductive health services to another medical cultural beliefs and the demands of responsible parenthood) are being
practitioner who would be able to provide for the patient's needs. For the threatened or are not being met as to justify the impairment of religious
petitioners, this amounts to requiring the conscientious objector to freedom.194
cooperate with the very thing he refuses to do without violating his/her
religious beliefs.190 Finally, the petitioners also question Section 15 of the RH Law requiring
would-be couples to attend family planning and responsible parenthood
They further argue that even if the conscientious objector's duty to refer is seminars and to obtain a certificate of compliance. They claim that the
recognized, the recognition is unduly limited, because although it allows a provision forces individuals to participate in the implementation of the RH
conscientious objector in Section 23 (a)(3) the option to refer a patient Law even if it contravenes their religious beliefs.195 As the assailed law
seeking reproductive health services and information - no escape is dangles the threat of penalty of fine and/or imprisonment in case of non-
afforded the conscientious objector in Section 23 (a)(l) and (2), i.e. compliance with its provisions, the petitioners claim that the RH Law
against a patient seeking reproductive health procedures. They claim that forcing them to provide, support and facilitate access and information to
the right of other individuals to conscientiously object, such as: a) those contraception against their beliefs must be struck down as it runs afoul to
working in public health facilities referred to in Section 7; b) public officers the constitutional guarantee of religious freedom.
involved in the implementation of the law referred to in Section 23(b );
and c) teachers in public schools referred to in Section 14 of the RH Law, The Respondents' Positions
are also not recognize.191
The respondents, on the other hand, contend that the RH Law does not
Petitioner Echavez and the other medical practitioners meanwhile, provide that a specific mode or type of contraceptives be used, be it
contend that the requirement to refer the matter to another health care natural or artificial. It neither imposes nor sanctions any religion or
service provider is still considered a compulsion on those objecting belief.196 They point out that the RH Law only seeks to serve the public
healthcare service providers. They add that compelling them to do the act interest by providing accessible, effective and quality reproductive health
against their will violates the Doctrine of Benevolent Neutrality. Sections services to ensure maternal and child health, in line with the State's duty
9, 14 and 1 7 of the law are too secular that they tend to disregard the to bring to reality the social justice health guarantees of the
religion of Filipinos. Authorizing the use of contraceptives with abortive Constitution,197 and that what the law only prohibits are those acts or
effects, mandatory sex education, mandatory pro-bono reproductive practices, which deprive others of their right to reproductive
health services to indigents encroach upon the religious freedom of those health.198 They assert that the assailed law only seeks to guarantee
upon whom they are required.192 informed choice, which is an assurance that no one will be compelled to
violate his religion against his free will.199
Petitioner CFC also argues that the requirement for a conscientious
objector to refer the person seeking reproductive health care services to The respondents add that by asserting that only natural family planning
another provider infringes on one's freedom of religion as it forces the should be allowed, the petitioners are effectively going against the
objector to become an unwilling participant in the commission of a constitutional right to religious freedom, the same right they invoked to
serious sin under Catholic teachings. While the right to act on one's belief assail the constitutionality of the RH Law.200 In other words, by seeking the
may be regulated by the State, the acts prohibited by the RH Law are declaration that the RH Law is unconstitutional, the petitioners are asking
passive acts which produce neither harm nor injury to the public.193 that the Court recognize only the Catholic Church's sanctioned natural
family planning methods and impose this on the entire citizenry.201
Petitioner CFC adds that the RH Law does not show compelling state
interest to justify regulation of religious freedom because it mentions no
With respect to the duty to refer, the respondents insist that the same We, the sovereign Filipino people, imploring the aid of Almighty God, in
does not violate the constitutional guarantee of religious freedom, it being order to build a just and humane society, and establish a Government
a carefully balanced compromise between the interests of the religious that shall embody our ideals and aspirations, promote the common good,
objector, on one hand, who is allowed to keep silent but is required to conserve and develop our patrimony, and secure to ourselves and our
refer -and that of the citizen who needs access to information and who posterity, the blessings of independence and democracy under the rule of
has the right to expect that the health care professional in front of her will law and a regime of truth, justice, freedom, love, equality, and peace, do
act professionally. For the respondents, the concession given by the ordain and promulgate this Constitution.
State under Section 7 and 23(a)(3) is sufficient accommodation to the
right to freely exercise one's religion without unnecessarily infringing on The Filipino people in "imploring the aid of Almighty God " manifested
the rights of others.202 their spirituality innate in our nature and consciousness as a people,
shaped by tradition and historical experience. As this is embodied in the
Whatever burden is placed on the petitioner's religious freedom is preamble, it means that the State recognizes with respect the influence of
minimal as the duty to refer is limited in duration, location and impact.203 religion in so far as it instills into the mind the purest principles of
morality.205 Moreover, in recognition of the contributions of religion to
Regarding mandatory family planning seminars under Section 15 , the society, the 1935, 1973 and 1987 constitutions contain benevolent and
respondents claim that it is a reasonable regulation providing an accommodating provisions towards religions such as tax exemption of
opportunity for would-be couples to have access to information regarding church property, salary of religious officers in government institutions,
parenthood, family planning, breastfeeding and infant nutrition. It is and optional religious instructions in public schools.
argued that those who object to any information received on account of
their attendance in the required seminars are not compelled to accept The Framers, however, felt the need to put up a strong barrier so that the
information given to them. They are completely free to reject any State would not encroach into the affairs of the church, and vice-versa.
information they do not agree with and retain the freedom to decide on The principle of separation of Church and State was, thus, enshrined in
matters of family life without intervention of the State.204 Article II, Section 6 of the 1987 Constitution, viz:

For their part, respondents De Venecia et al., dispute the notion that Section 6. The separation of Church and State shall be inviolable.
natural family planning is the only method acceptable to Catholics and
the Catholic hierarchy. Citing various studies and surveys on the matter, Verily, the principle of separation of Church and State is based on mutual
they highlight the changing stand of the Catholic Church on contraception respect. Generally, the State cannot meddle in the internal affairs of the
1âwphi1

throughout the years and note the general acceptance of the benefits of church, much less question its faith and dogmas or dictate upon it. It
contraceptives by its followers in planning their families. cannot favor one religion and discriminate against another. On the other
hand, the church cannot impose its beliefs and convictions on the State
The Church and The State and the rest of the citizenry. It cannot demand that the nation follow its
beliefs, even if it sincerely believes that they are good for the country.
At the outset, it cannot be denied that we all live in a heterogeneous
society. It is made up of people of diverse ethnic, cultural and religious Consistent with the principle that not any one religion should ever be
beliefs and backgrounds. History has shown us that our government, in preferred over another, the Constitution in the above-cited provision
law and in practice, has allowed these various religious, cultural, social utilizes the term "church" in its generic sense, which refers to a temple, a
and racial groups to thrive in a single society together. It has embraced mosque, an iglesia, or any other house of God which metaphorically
minority groups and is tolerant towards all - the religious people of symbolizes a religious organization. Thus, the "Church" means the
different sects and the non-believers. The undisputed fact is that our religious congregations collectively.
people generally believe in a deity, whatever they conceived Him to be,
and to whom they call for guidance and enlightenment in crafting our Balancing the benefits that religion affords and the need to provide an
fundamental law. Thus, the preamble of the present Constitution reads: ample barrier to protect the State from the pursuit of its secular
objectives, the Constitution lays down the following mandate in Article III, religion within limits of utmost amplitude. It has been said that the religion
Section 5 and Article VI, Section 29 (2), of the 1987 Constitution: clauses of the Constitution are all designed to protect the broadest
possible liberty of conscience, to allow each man to believe as his
Section. 5. No law shall be made respecting an establishment of religion, conscience directs, to profess his beliefs, and to live as he believes he
or prohibiting the free exercise thereof. The free exercise and enjoyment ought to live, consistent with the liberty of others and with the common
of religious profession and worship, without discrimination or preference, good. Any legislation whose effect or purpose is to impede the
shall forever be allowed. No religious test shall be required for the observance of one or all religions, or to discriminate invidiously between
exercise of civil or political rights. the religions, is invalid, even though the burden may be characterized as
being only indirect. (Sherbert v. Verner, 374 U.S. 398, 10 L.ed.2d 965, 83
Section 29. S. Ct. 1970) But if the state regulates conduct by enacting, within its
power, a general law which has for its purpose and effect to advance the
state's secular goals, the statute is valid despite its indirect burden on
xxx.
religious observance, unless the state can accomplish its purpose without
imposing such burden. (Braunfeld v. Brown, 366 U.S. 599, 6 Led. 2d.
No public money or property shall be appropriated, applied, paid, or 563, 81 S. Ct. 144; McGowan v. Maryland, 366 U.S. 420, 444-5 and
employed, directly or indirectly, for the use, benefit, or support of any 449).
sect, church, denomination, sectarian institution, or system of religion, or
of any priest, preacher, minister, other religious teacher, or dignitary as
As expounded in Escritor,
such, except when such priest, preacher, minister, or dignitary is
assigned to the armed forces, or to any penal institution, or government
orphanage or leprosarium. The establishment and free exercise clauses were not designed to serve
contradictory purposes. They have a single goal-to promote freedom of
individual religious beliefs and practices. In simplest terms, the free
In short, the constitutional assurance of religious freedom provides two
exercise clause prohibits government from inhibiting religious beliefs with
guarantees: the Establishment Clause and the Free Exercise Clause.
penalties for religious beliefs and practice, while the establishment clause
prohibits government from inhibiting religious belief with rewards for
The establishment clause "principally prohibits the State from sponsoring religious beliefs and practices. In other words, the two religion clauses
any religion or favoring any religion as against other religions. It were intended to deny government the power to use either the carrot or
mandates a strict neutrality in affairs among religious the stick to influence individual religious beliefs and practices.210
groups."206 Essentially, it prohibits the establishment of a state religion and
the use of public resources for the support or prohibition of a religion.
Corollary to the guarantee of free exercise of one's religion is the
principle that the guarantee of religious freedom is comprised of two
On the other hand, the basis of the free exercise clause is the respect for parts: the freedom to believe, and the freedom to act on one's belief. The
the inviolability of the human conscience.207 Under this part of religious first part is absolute. As explained in Gerona v. Secretary of Education:211
freedom guarantee, the State is prohibited from unduly interfering with
the outside manifestations of one's belief and faith.208 Explaining the
The realm of belief and creed is infinite and limitless bounded only by
concept of religious freedom, the Court, in Victoriano v. Elizalde Rope
one's imagination and thought. So is the freedom of belief, including
Workers Union209 wrote:
religious belief, limitless and without bounds. One may believe in most
anything, however strange, bizarre and unreasonable the same may
The constitutional provisions not only prohibits legislation for the support appear to others, even heretical when weighed in the scales of orthodoxy
of any religious tenets or the modes of worship of any sect, thus or doctrinal standards. But between the freedom of belief and the
forestalling compulsion by law of the acceptance of any creed or the exercise of said belief, there is quite a stretch of road to travel.212
practice of any form of worship (U.S. Ballard, 322 U.S. 78, 88 L. ed.
1148, 1153), but also assures the free exercise of one's chosen form of
The second part however, is limited and subject to the awesome power the law. The case also used, albeit inappropriately, the "compelling state
of the State and can be enjoyed only with proper regard to the rights of interest" test. After Victoriano , German went back to the Gerona rule.
others. It is "subject to regulation where the belief is translated into Ebralinag then employed the "grave and immediate danger" test and
external acts that affect the public welfare."213 overruled the Gerona test. The fairly recent case of Iglesia ni Cristo went
back to the " clear and present danger" test in the maiden case of A
Legislative Acts and the merican Bible Society. Not surprisingly, all the cases which employed the
"clear and present danger" or "grave and immediate danger" test
Free Exercise Clause involved, in one form or another, religious speech as this test is often
used in cases on freedom of expression. On the other hand, the Gerona
and German cases set the rule that religious freedom will not prevail over
Thus, in case of conflict between the free exercise clause and the State,
established institutions of society and law. Gerona, however, which was
the Court adheres to the doctrine of benevolent neutrality. This has been
the authority cited by German has been overruled by Ebralinag which
clearly decided by the Court in Estrada v. Escritor, (Escritor)214 where it
employed the "grave and immediate danger" test . Victoriano was the
was stated "that benevolent neutrality-accommodation, whether
only case that employed the "compelling state interest" test, but as
mandatory or permissive, is the spirit, intent and framework underlying
explained previously, the use of the test was inappropriate to the facts of
the Philippine Constitution."215 In the same case, it was further explained
the case.
that"
The case at bar does not involve speech as in A merican Bible Society,
The benevolent neutrality theory believes that with respect to these
Ebralinag and Iglesia ni Cristo where the "clear and present danger" and
governmental actions, accommodation of religion may be allowed, not to
"grave and immediate danger" tests were appropriate as speech has
promote the government's favored form of religion, but to allow
easily discernible or immediate effects. The Gerona and German
individuals and groups to exercise their religion without hindrance. "The
doctrine, aside from having been overruled, is not congruent with the
purpose of accommodation is to remove a burden on, or facilitate the
benevolent neutrality approach, thus not appropriate in this jurisdiction.
exercise of, a person's or institution's religion."216 "What is sought under
Similar to Victoriano, the present case involves purely conduct arising
the theory of accommodation is not a declaration of unconstitutionality of
from religious belief. The "compelling state interest" test is proper where
a facially neutral law, but an exemption from its application or its
conduct is involved for the whole gamut of human conduct has different
'burdensome effect,' whether by the legislature or the courts."217
effects on the state's interests: some effects may be immediate and
short-term while others delayed and far-reaching. A test that would
In ascertaining the limits of the exercise of religious freedom, the protect the interests of the state in preventing a substantive evil, whether
compelling state interest test is proper.218Underlying the compelling state immediate or delayed, is therefore necessary. However, not any interest
interest test is the notion that free exercise is a fundamental right and that of the state would suffice to prevail over the right to religious freedom as
laws burdening it should be subject to strict scrutiny.219 In Escritor, it was this is a fundamental right that enjoys a preferred position in the hierarchy
written: of rights - "the most inalienable and sacred of all human rights", in the
words of Jefferson. This right is sacred for an invocation of the Free
Philippine jurisprudence articulates several tests to determine these Exercise Clause is an appeal to a higher sovereignty. The entire
limits. Beginning with the first case on the Free Exercise Clause, constitutional order of limited government is premised upon an
American Bible Society, the Court mentioned the "clear and present acknowledgment of such higher sovereignty, thus the Filipinos implore
danger" test but did not employ it. Nevertheless, this test continued to be the "aid of Almighty God in order to build a just and humane society and
cited in subsequent cases on religious liberty. The Gerona case then establish a government." As held in Sherbert, only the gravest abuses,
pronounced that the test of permissibility of religious freedom is whether endangering paramount interests can limit this fundamental right. A mere
it violates the established institutions of society and law. The Victoriano balancing of interests which balances a right with just a colorable state
case mentioned the "immediate and grave danger" test as well as the interest is therefore not appropriate. Instead, only a compelling interest of
doctrine that a law of general applicability may burden religious exercise the state can prevail over the fundamental right to religious liberty. The
provided the law is the least restrictive means to accomplish the goal of
test requires the state to carry a heavy burden, a compelling one, for to 2 . The State recognizes marriage as an inviolable social institution and
do otherwise would allow the state to batter religion, especially the less the foundation of the family which in turn is the foundation of the nation.
powerful ones until they are destroyed. In determining which shall prevail Pursuant thereto, the State shall defend:
between the state's interest and religious liberty, reasonableness shall be
the guide. The "compelling state interest" serves the purpose of revering (a) The right of spouses to found a family in accordance with their
religious liberty while at the same time affording protection to the religious convictions and the demands of responsible parenthood."
paramount interests of the state. This was the test used in Sherbert [Section 2, Declaration of Policy]
which involved conduct, i.e. refusal to work on Saturdays. In the end, the
"compelling state interest" test, by upholding the paramount interests of 3. The State shall promote and provide information and access, without
the state, seeks to protect the very state, without which, religious liberty bias, to all methods of family planning, including effective natural and
will not be preserved. [Emphases in the original. Underlining supplied.] modern methods which have been proven medically safe, legal, non-
abortifacient, and effective in accordance with scientific and evidence-
The Court's Position based medical research standards such as those registered and
approved by the FDA for the poor and marginalized as identified through
In the case at bench, it is not within the province of the Court to the NHTS-PR and other government measures of identifying
determine whether the use of contraceptives or one's participation in the marginalization: Provided, That the State shall also provide funding
support of modem reproductive health measures is moral from a religious support to promote modern natural methods of family planning, especially
standpoint or whether the same is right or wrong according to one's the Billings Ovulation Method, consistent with the needs of acceptors and
dogma or belief. For the Court has declared that matters dealing with their religious convictions. [Section 3(e), Declaration of Policy]
"faith, practice, doctrine, form of worship, ecclesiastical law, custom and
rule of a church ... are unquestionably ecclesiastical matters which are 4. The State shall promote programs that: (1) enable individuals and
outside the province of the civil courts."220 The jurisdiction of the Court couples to have the number of children they desire with due
extends only to public and secular morality. Whatever pronouncement consideration to the health, particularly of women, and the resources
the Court makes in the case at bench should be understood only in this available and affordable to them and in accordance with existing laws,
realm where it has authority. Stated otherwise, while the Court stands public morals and their religious convictions. [Section 3CDJ
without authority to rule on ecclesiastical matters, as vanguard of the
Constitution, it does have authority to determine whether the RH Law 5. The State shall respect individuals' preferences and choice of family
contravenes the guarantee of religious freedom. planning methods that are in accordance with their religious convictions
and cultural beliefs, taking into consideration the State's obligations under
At first blush, it appears that the RH Law recognizes and respects religion various human rights instruments. [Section 3(h)]
and religious beliefs and convictions. It is replete with assurances the no
one can be compelled to violate the tenets of his religion or defy his 6. Active participation by nongovernment organizations (NGOs) ,
religious convictions against his free will. Provisions in the RH Law women's and people's organizations, civil society, faith-based
respecting religious freedom are the following: organizations, the religious sector and communities is crucial to ensure
that reproductive health and population and development policies, plans,
1. The State recognizes and guarantees the human rights of all persons and programs will address the priority needs of women, the poor, and the
including their right to equality and nondiscrimination of these rights, the marginalized. [Section 3(i)]
right to sustainable human development, the right to health which
includes reproductive health, the right to education and information, and 7. Responsible parenthood refers to the will and ability of a parent to
the right to choose and make decisions for themselves in accordance respond to the needs and aspirations of the family and children. It is
with their religious convictions, ethics, cultural beliefs, and the demands likewise a shared responsibility between parents to determine and
of responsible parenthood. [Section 2, Declaration of Policy] achieve the desired number of children, spacing and timing of their
children according to their own family life aspirations, taking into account
psychological preparedness, health status, sociocultural and economic In a situation where the free exercise of religion is allegedly burdened by
concerns consistent with their religious convictions. [Section 4(v)] government legislation or practice, the compelling state interest test in
(Emphases supplied) line with the Court's espousal of the Doctrine of Benevolent Neutrality in
Escritor, finds application. In this case, the conscientious objector's claim
While the Constitution prohibits abortion, laws were enacted allowing the to religious freedom would warrant an exemption from obligations under
use of contraceptives. To some medical practitioners, however, the whole the RH Law, unless the government succeeds in demonstrating a more
idea of using contraceptives is an anathema. Consistent with the principle compelling state interest in the accomplishment of an important secular
of benevolent neutrality, their beliefs should be respected. objective. Necessarily so, the plea of conscientious objectors for
exemption from the RH Law deserves no less than strict scrutiny.
The Establishment Clause
In applying the test, the first inquiry is whether a conscientious objector's
and Contraceptives right to religious freedom has been burdened. As in Escritor, there is no
doubt that an intense tug-of-war plagues a conscientious objector. One
side coaxes him into obedience to the law and the abandonment of his
In the same breath that the establishment clause restricts what the
religious beliefs, while the other entices him to a clean conscience yet
government can do with religion, it also limits what religious sects can or
under the pain of penalty. The scenario is an illustration of the
cannot do with the government. They can neither cause the government
predicament of medical practitioners whose religious beliefs are
to adopt their particular doctrines as policy for everyone, nor can they not
incongruent with what the RH Law promotes.
cause the government to restrict other groups. To do so, in simple terms,
would cause the State to adhere to a particular religion and, thus,
establishing a state religion. The Court is of the view that the obligation to refer imposed by the RH
Law violates the religious belief and conviction of a conscientious
objector. Once the medical practitioner, against his will, refers a patient
Consequently, the petitioners are misguided in their supposition that the
seeking information on modem reproductive health products, services,
State cannot enhance its population control program through the RH Law
procedures and methods, his conscience is immediately burdened as he
simply because the promotion of contraceptive use is contrary to their
has been compelled to perform an act against his beliefs. As
religious beliefs. Indeed, the State is not precluded to pursue its
Commissioner Joaquin A. Bernas (Commissioner Bernas) has written, "at
legitimate secular objectives without being dictated upon by the policies
the basis of the free exercise clause is the respect for the inviolability of
of any one religion. One cannot refuse to pay his taxes simply because it
the human conscience.222
will cloud his conscience. The demarcation line between Church and
State demands that one render unto Caesar the things that are Caesar's
and unto God the things that are God's.221 Though it has been said that the act of referral is an opt-out clause, it is,
however, a false compromise because it makes pro-life health providers
complicit in the performance of an act that they find morally repugnant or
The Free Exercise Clause and the Duty to Refer
offensive. They cannot, in conscience, do indirectly what they cannot do
directly. One may not be the principal, but he is equally guilty if he abets
While the RH Law, in espousing state policy to promote reproductive the offensive act by indirect participation.
health manifestly respects diverse religious beliefs in line with the Non-
Establishment Clause, the same conclusion cannot be reached with
Moreover, the guarantee of religious freedom is necessarily intertwined
respect to Sections 7, 23 and 24 thereof. The said provisions commonly
with the right to free speech, it being an externalization of one's thought
mandate that a hospital or a medical practitioner to immediately refer a
and conscience. This in turn includes the right to be silent. With the
person seeking health care and services under the law to another
constitutional guarantee of religious freedom follows the protection that
accessible healthcare provider despite their conscientious objections
should be afforded to individuals in communicating their beliefs to others
based on religious or ethical beliefs.
as well as the protection for simply being silent. The Bill of Rights
guarantees the liberty of the individual to utter what is in his mind and the
liberty not to utter what is not in his mind.223 While the RH Law seeks to and in the performance of reproductive health procedures, the religious
provide freedom of choice through informed consent, freedom of choice freedom of health care service providers should be respected.
guarantees the liberty of the religious conscience and prohibits any
degree of compulsion or burden, whether direct or indirect, in the practice In the case of Islamic Da'wah Council of the Philippines, Inc. v. Office of
of one's religion.224 the Executive Secretary228 it was stressed:

In case of conflict between the religious beliefs and moral convictions of Freedom of religion was accorded preferred status by the framers of our
individuals, on one hand, and the interest of the State, on the other, to fundamental law. And this Court has consistently affirmed this preferred
provide access and information on reproductive health products, status, well aware that it is "designed to protect the broadest possible
services, procedures and methods to enable the people to determine the liberty of conscience, to allow each man to believe as his conscience
timing, number and spacing of the birth of their children, the Court is of directs, to profess his beliefs, and to live as he believes he ought to live,
the strong view that the religious freedom of health providers, whether consistent with the liberty of others and with the common good."10
public or private, should be accorded primacy. Accordingly, a
conscientious objector should be exempt from compliance with the The Court is not oblivious to the view that penalties provided by law
mandates of the RH Law. If he would be compelled to act contrary to his endeavour to ensure compliance. Without set consequences for either an
religious belief and conviction, it would be violative of "the principle of active violation or mere inaction, a law tends to be toothless and
non-coercion" enshrined in the constitutional right to free exercise of ineffectual. Nonetheless, when what is bartered for an effective
religion. implementation of a law is a constitutionally-protected right the Court
firmly chooses to stamp its disapproval. The punishment of a healthcare
Interestingly, on April 24, 2013, Scotland's Inner House of the Court of service provider, who fails and/or refuses to refer a patient to another, or
Session, found in the case of Doogan and Wood v. NHS Greater who declines to perform reproductive health procedure on a patient
Glasgow and Clyde Health Board,225 that the midwives claiming to be because incompatible religious beliefs, is a clear inhibition of a
conscientious objectors under the provisions of Scotland's Abortion Act of constitutional guarantee which the Court cannot allow.
1967, could not be required to delegate, supervise or support staff on
their labor ward who were involved in abortions.226 The Inner House stated The Implementing Rules and Regulation (RH-IRR)
"that if 'participation' were defined according to whether the person was
taking part 'directly' or ' indirectly' this would actually mean more
The last paragraph of Section 5.24 of the RH-IRR reads:
complexity and uncertainty."227
Provided, That skilled health professional such as provincial, city or
While the said case did not cover the act of referral, the applicable
municipal health officers, chiefs of hospital, head nurses, supervising
principle was the same - they could not be forced to assist abortions if it
midwives, among others, who by virtue of their office are specifically
would be against their conscience or will.
charged with the duty to implement the provisions of the RPRH Act and
these Rules, cannot be considered as conscientious objectors.
Institutional Health Providers
This is discriminatory and violative of the equal protection clause. The
The same holds true with respect to non-maternity specialty hospitals conscientious objection clause should be equally protective of the
and hospitals owned and operated by a religious group and health care religious belief of public health officers. There is no perceptible distinction
service providers. Considering that Section 24 of the RH Law penalizes why they should not be considered exempt from the mandates of the law.
such institutions should they fail or refuse to comply with their duty to The protection accorded to other conscientious objectors should equally
refer under Section 7 and Section 23(a)(3), the Court deems that it must apply to all medical practitioners without distinction whether they belong
be struck down for being violative of the freedom of religion. The same to the public or private sector. After all, the freedom to believe is intrinsic
applies to Section 23(a)(l) and (a)(2) in relation to Section 24, considering in every individual and the protective robe that guarantees its free
that in the dissemination of information regarding programs and services
exercise is not taken off even if one acquires employment in the Congressman Lagman:
government.
I will have to go over again the provisions, Your Honor.
It should be stressed that intellectual liberty occupies a place inferior to
none in the hierarchy of human values. The mind must be free to think Justice Mendoza:
what it wills, whether in the secular or religious sphere, to give expression
to its beliefs by oral discourse or through the media and, thus, seek other In other words, public health officers in contrast to the private
candid views in occasions or gatherings or in more permanent practitioners who can be conscientious objectors, skilled health
aggrupation. Embraced in such concept then are freedom of religion, professionals cannot be considered conscientious objectors. Do you
freedom of speech, of the press, assembly and petition, and freedom of agree with this? Is this not against the constitutional right to the religious
association.229 belief?

The discriminatory provision is void not only because no such exception Congressman Lagman:
is stated in the RH Law itself but also because it is violative of the equal
protection clause in the Constitution. Quoting respondent Lagman, if
Your Honor, if there is any conflict between the IRR and the law, the law
there is any conflict between the RH-IRR and the RH Law, the law must
must prevail.230
prevail.
Compelling State Interest
Justice Mendoza:
The foregoing discussion then begets the question on whether the
I'll go to another point. The RH law .. .in your Comment- in-Intervention
respondents, in defense of the subject provisions, were able to: 1]
on page 52, you mentioned RH Law is replete with provisions in
demonstrate a more compelling state interest to restrain conscientious
upholding the freedom of religion and respecting religious convictions.
objectors in their choice of services to render; and 2] discharge the
Earlier, you affirmed this with qualifications. Now, you have read, I
burden of proof that the obligatory character of the law is the least
presumed you have read the IRR-Implementing Rules and Regulations of
intrusive means to achieve the objectives of the law.
the RH Bill?
Unfortunately, a deep scrutiny of the respondents' submissions proved to
Congressman Lagman:
be in vain. The OSG was curiously silent in the establishment of a more
compelling state interest that would rationalize the curbing of a
Yes, Your Honor, I have read but I have to admit, it's a long IRR and I conscientious objector's right not to adhere to an action contrary to his
have not thoroughly dissected the nuances of the provisions. religious convictions. During the oral arguments, the OSG maintained the
same silence and evasion. The Transcripts of the Stenographic Notes
Justice Mendoza: disclose the following:

I will read to you one provision. It's Section 5.24. This I cannot find in the Justice De Castro:
RH Law. But in the IRR it says: " .... skilled health professionals such as
provincial, city or municipal health officers, chief of hospitals, head Let's go back to the duty of the conscientious objector to refer. ..
nurses, supervising midwives, among others, who by virtue of their office
are specifically charged with the duty to implement the provisions of the
Senior State Solicitor Hilbay:
RPRH Act and these Rules, cannot be considered as conscientious
objectors." Do you agree with this?
Yes, Justice.
Justice De Castro: contraceptive use is immediate and occurs the moment a patient seeks
consultation on reproductive health matters.
... which you are discussing awhile ago with Justice Abad. What is the
compelling State interest in imposing this duty to refer to a conscientious Moreover, granting that a compelling interest exists to justify the
objector which refuses to do so because of his religious belief? infringement of the conscientious objector's religious freedom, the
respondents have failed to demonstrate "the gravest abuses,
Senior State Solicitor Hilbay: endangering paramount interests" which could limit or override a person's
fundamental right to religious freedom. Also, the respondents have not
Ahh, Your Honor, .. presented any government effort exerted to show that the means it takes
to achieve its legitimate state objective is the least intrusive
means.234 Other than the assertion that the act of referring would only be
Justice De Castro:
momentary, considering that the act of referral by a conscientious
objector is the very action being contested as violative of religious
What is the compelling State interest to impose this burden? freedom, it behooves the respondents to demonstrate that no other
means can be undertaken by the State to achieve its objective without
Senior State Solicitor Hilbay: violating the rights of the conscientious objector. The health concerns of
women may still be addressed by other practitioners who may perform
In the first place, Your Honor, I don't believe that the standard is a reproductive health-related procedures with open willingness and
compelling State interest, this is an ordinary health legislation involving motivation. Suffice it to say, a person who is forced to perform an act in
professionals. This is not a free speech matter or a pure free exercise utter reluctance deserves the protection of the Court as the last vanguard
matter. This is a regulation by the State of the relationship between of constitutional freedoms.
medical doctors and their patients.231
At any rate, there are other secular steps already taken by the
Resultantly, the Court finds no compelling state interest which would limit Legislature to ensure that the right to health is protected. Considering
the free exercise clause of the conscientious objectors, however few in other legislations as they stand now, R.A . No. 4 729 or the Contraceptive
number. Only the prevention of an immediate and grave danger to the Act, R.A. No. 6365 or "The Population Act of the Philippines" and R.A.
security and welfare of the community can justify the infringement of No. 9710, otherwise known as "The Magna Carta of Women," amply
religious freedom. If the government fails to show the seriousness and cater to the needs of women in relation to health services and programs.
immediacy of the threat, State intrusion is constitutionally unacceptable.232 The pertinent provision of Magna Carta on comprehensive health
services and programs for women, in fact, reads:
Freedom of religion means more than just the freedom to believe. It also
means the freedom to act or not to act according to what one believes. Section 17. Women's Right to Health. - (a) Comprehensive Health
And this freedom is violated when one is compelled to act against one's Services. - The State shall, at all times, provide for a comprehensive,
belief or is prevented from acting according to one's belief.233 culture-sensitive, and gender-responsive health services and programs
covering all stages of a woman's life cycle and which addresses the
Apparently, in these cases, there is no immediate danger to the life or major causes of women's mortality and morbidity: Provided, That in the
health of an individual in the perceived scenario of the subject provisions. provision for comprehensive health services, due respect shall be
After all, a couple who plans the timing, number and spacing of the birth accorded to women's religious convictions, the rights of the spouses to
of their children refers to a future event that is contingent on whether or found a family in accordance with their religious convictions, and the
not the mother decides to adopt or use the information, product, method demands of responsible parenthood, and the right of women to protection
or supply given to her or whether she even decides to become pregnant from hazardous drugs, devices, interventions, and substances.
at all. On the other hand, the burden placed upon those who object to
Access to the following services shall be ensured:
(1) Maternal care to include pre- and post-natal services to women's health in government education and training programs, with due
address pregnancy and infant health and nutrition; regard to the following:

(2) Promotion of breastfeeding; (1) The natural and primary right and duty of parents in the
rearing of the youth and the development of moral character and
(3) Responsible, ethical, legal, safe, and effective methods of the right of children to be brought up in an atmosphere of morality
family planning; and rectitude for the enrichment and strengthening of character;

(4) Family and State collaboration in youth sexuality education (2) The formation of a person's sexuality that affirms human
and health services without prejudice to the primary right and duty dignity; and
of parents to educate their children;
(3) Ethical, legal, safe, and effective family planning methods
(5) Prevention and management of reproductive tract infections, including fertility awareness.
including sexually transmitted diseases, HIV, and AIDS;
As an afterthought, Asst. Solicitor General Hilbay eventually replied that
(6) Prevention and management of reproductive tract cancers like the compelling state interest was "Fifteen maternal deaths per day,
breast and cervical cancers, and other gynecological conditions hundreds of thousands of unintended pregnancies, lives changed, x x
and disorders; x."235 He, however, failed to substantiate this point by concrete facts and
figures from reputable sources.
(7) Prevention of abortion and management of pregnancy-related
complications; The undisputed fact, however, is that the World Health Organization
reported that the Filipino maternal mortality rate dropped to 48 percent
(8) In cases of violence against women and children, women and from 1990 to 2008, 236 although there was still no RH Law at that time.
children victims and survivors shall be provided with Despite such revelation, the proponents still insist that such number of
comprehensive health services that include psychosocial, maternal deaths constitute a compelling state interest.
therapeutic, medical, and legal interventions and assistance
towards healing, recovery, and empowerment; Granting that there are still deficiencies and flaws in the delivery of social
healthcare programs for Filipino women, they could not be solved by a
(9) Prevention and management of infertility and sexual measure that puts an unwarrantable stranglehold on religious beliefs in
dysfunction pursuant to ethical norms and medical standards; exchange for blind conformity.

(10) Care of the elderly women beyond their child-bearing years; Exception: Life Threatening Cases
and
All this notwithstanding, the Court properly recognizes a valid exception
(11) Management, treatment, and intervention of mental health set forth in the law. While generally healthcare service providers cannot
problems of women and girls. In addition, healthy lifestyle be forced to render reproductive health care procedures if doing it would
activities are encouraged and promoted through programs and contravene their religious beliefs, an exception must be made in life-
projects as strategies in the prevention of diseases. threatening cases that require the performance of emergency
procedures. In these situations, the right to life of the mother should be
given preference, considering that a referral by a medical practitioner
(b) Comprehensive Health Information and Education. - The State shall
would amount to a denial of service, resulting to unnecessarily placing
provide women in all sectors with appropriate, timely, complete, and
the life of a mother in grave danger. Thus, during the oral arguments,
accurate information and education on all the above-stated aspects of
Atty. Liban, representing CFC, manifested: "the forced referral clause that reject the information they find unacceptable, and retain the freedom to
we are objecting on grounds of violation of freedom of religion does not decide on matters of family life without the intervention of the State.
contemplate an emergency."237
4-The Family and the Right to Privacy
In a conflict situation between the life of the mother and the life of a child,
the doctor is morally obliged always to try to save both lives. If, however, Petitioner CFC assails the RH Law because Section 23(a) (2) (i) thereof
it is impossible, the resulting death to one should not be deliberate. Atty. violates the provisions of the Constitution by intruding into marital privacy
Noche explained: and autonomy. It argues that it cultivates disunity and fosters animosity in
the family rather than promote its solidarity and total development.240
Principle of Double-Effect. - May we please remind the principal author of
the RH Bill in the House of Representatives of the principle of double- The Court cannot but agree.
effect wherein intentional harm on the life of either the mother of the child
is never justified to bring about a "good" effect. In a conflict situation The 1987 Constitution is replete with provisions strengthening the family
between the life of the child and the life of the mother, the doctor is as it is the basic social institution. In fact, one article, Article XV, is
morally obliged always to try to save both lives. However, he can act in devoted entirely to the family.
favor of one (not necessarily the mother) when it is medically impossible
to save both, provided that no direct harm is intended to the other. If the
ARTICLE XV
above principles are observed, the loss of the child's life or the mother's
THE FAMILY
life is not intentional and, therefore, unavoidable. Hence, the doctor
would not be guilty of abortion or murder. The mother is never pitted
against the child because both their lives are equally valuable.238 Section 1. The State recognizes the Filipino family as the foundation of
the nation. Accordingly, it shall strengthen its solidarity and actively
promote its total development.
Accordingly, if it is necessary to save the life of a mother, procedures
endangering the life of the child may be resorted to even if is against the
religious sentiments of the medical practitioner. As quoted above, Section 2. Marriage, as an inviolable social institution, is the foundation of
whatever burden imposed upon a medical practitioner in this case would the family and shall be protected by the State.
have been more than justified considering the life he would be able to
save. Section 3. The State shall defend:

Family Planning Seminars The right of spouses to found a family in accordance with their religious
convictions and the demands of responsible parenthood;
Anent the requirement imposed under Section 15239 as a condition for the
issuance of a marriage license, the Court finds the same to be a The right of children to assistance, including proper care and nutrition,
reasonable exercise of police power by the government. A cursory and special protection from all forms of neglect, abuse, cruelty,
reading of the assailed provision bares that the religious freedom of the exploitation and other conditions prejudicial to their development;
petitioners is not at all violated. All the law requires is for would-be
spouses to attend a seminar on parenthood, family planning The right of the family to a family living wage and income; and
breastfeeding and infant nutrition. It does not even mandate the type of
family planning methods to be included in the seminar, whether they be The right of families or family assoc1at1ons to participate in the planning
natural or artificial. As correctly noted by the OSG, those who receive any and implementation of policies and programs that affect them.
information during their attendance in the required seminars are not
compelled to accept the information given to them, are completely free to In this case, the RH Law, in its not-so-hidden desire to control population
growth, contains provisions which tend to wreck the family as a solid
social institution. It bars the husband and/or the father from participating Decision-making involving a reproductive health procedure is a private
in the decision making process regarding their common future progeny. It matter which belongs to the couple, not just one of them. Any decision
likewise deprives the parents of their authority over their minor daughter they would reach would affect their future as a family because the size of
simply because she is already a parent or had suffered a miscarriage. the family or the number of their children significantly matters. The
decision whether or not to undergo the procedure belongs exclusively to,
The Family and Spousal Consent and shared by, both spouses as one cohesive unit as they chart their
own destiny. It is a constitutionally guaranteed private right. Unless it
Section 23(a) (2) (i) of the RH Law states: prejudices the State, which has not shown any compelling interest, the
State should see to it that they chart their destiny together as one family.
The following acts are prohibited:
As highlighted by Justice Leonardo-De Castro, Section 19( c) of R.A. No.
9710, otherwise known as the "Magna Carta for Women," provides that
(a) Any health care service provider, whether public or private, who shall:
women shall have equal rights in all matters relating to marriage and
...
family relations, including the joint decision on the number and spacing of
their children. Indeed, responsible parenthood, as Section 3(v) of the RH
(2) refuse to perform legal and medically-safe reproductive health Law states, is a shared responsibility between parents. Section 23(a)(2)(i)
procedures on any person of legal age on the ground of lack of consent of the RH Law should not be allowed to betray the constitutional mandate
or authorization of the following persons in the following instances: to protect and strengthen the family by giving to only one spouse the
absolute authority to decide whether to undergo reproductive health
(i) Spousal consent in case of married persons: provided, That in case of procedure.242
disagreement, the decision of the one undergoing the procedures shall
prevail. [Emphasis supplied] The right to chart their own destiny together falls within the protected
zone of marital privacy and such state intervention would encroach into
The above provision refers to reproductive health procedures like tubal the zones of spousal privacy guaranteed by the Constitution. In our
litigation and vasectomy which, by their very nature, should require jurisdiction, the right to privacy was first recognized in Marje v.
mutual consent and decision between the husband and the wife as they Mutuc,243 where the Court, speaking through Chief Justice Fernando, held
affect issues intimately related to the founding of a family. Section 3, Art. that "the right to privacy as such is accorded recognition independently of
XV of the Constitution espouses that the State shall defend the "right of its identification with liberty; in itself, it is fully deserving of constitutional
the spouses to found a family." One person cannot found a family. The protection."244 Marje adopted the ruling of the US Supreme Court in
right, therefore, is shared by both spouses. In the same Section 3, their Griswold v. Connecticut,245 where Justice William O. Douglas wrote:
right "to participate in the planning and implementation of policies and
programs that affect them " is equally recognized. We deal with a right of privacy older than the Bill of Rights -older than our
political parties, older than our school system. Marriage is a coming
The RH Law cannot be allowed to infringe upon this mutual decision- together for better or for worse, hopefully enduring, and intimate to the
making. By giving absolute authority to the spouse who would undergo a degree of being sacred. It is an association that promotes a way of life,
procedure, and barring the other spouse from participating in the decision not causes; a harmony in living, not political faiths; a bilateral loyalty, not
would drive a wedge between the husband and wife, possibly result in commercial or social projects. Yet it is an association for as noble a
bitter animosity, and endanger the marriage and the family, all for the purpose as any involved in our prior decisions.
sake of reducing the population. This would be a marked departure from
the policy of the State to protect marriage as an inviolable social Ironically, Griswold invalidated a Connecticut statute which made the use
institution.241 of contraceptives a criminal offense on the ground of its amounting to an
unconstitutional invasion of the right to privacy of married persons.
Nevertheless, it recognized the zone of privacy rightfully enjoyed by
couples. Justice Douglas in Grisworld wrote that "specific guarantees in The 1987 provision has added the adjective "primary" to modify the right
the Bill of Rights have penumbras, formed by emanations from those of parents. It imports the assertion that the right of parents is superior to
guarantees that help give them life and substance. Various guarantees that of the State.248 [Emphases supplied]
create zones of privacy."246
To insist on a rule that interferes with the right of parents to exercise
At any rate, in case of conflict between the couple, the courts will decide. parental control over their minor-child or the right of the spouses to
mutually decide on matters which very well affect the very purpose of
The Family and Parental Consent marriage, that is, the establishment of conjugal and family life, would
result in the violation of one's privacy with respect to his family. It would
Equally deplorable is the debarment of parental consent in cases where be dismissive of the unique and strongly-held Filipino tradition of
the minor, who will be undergoing a procedure, is already a parent or has maintaining close family ties and violative of the recognition that the State
had a miscarriage. Section 7 of the RH law provides: affords couples entering into the special contract of marriage to as one
unit in forming the foundation of the family and society.
SEC. 7. Access to Family Planning. – x x x.
The State cannot, without a compelling state interest, take over the role
of parents in the care and custody of a minor child, whether or not the
No person shall be denied information and access to family planning
latter is already a parent or has had a miscarriage. Only a compelling
services, whether natural or artificial: Provided, That minors will not be
state interest can justify a state substitution of their parental authority.
allowed access to modern methods of family planning without written
consent from their parents or guardian/s except when the minor is
already a parent or has had a miscarriage. First Exception: Access to Information

There can be no other interpretation of this provision except that when a Whether with respect to the minor referred to under the exception
minor is already a parent or has had a miscarriage, the parents are provided in the second paragraph of Section 7 or with respect to the
excluded from the decision making process of the minor with regard to consenting spouse under Section 23(a)(2)(i), a distinction must be made.
family planning. Even if she is not yet emancipated, the parental authority There must be a differentiation between access to information about
is already cut off just because there is a need to tame population growth. family planning services, on one hand, and access to the reproductive
health procedures and modern family planning methods themselves, on
the other. Insofar as access to information is concerned, the Court finds
It is precisely in such situations when a minor parent needs the comfort,
no constitutional objection to the acquisition of information by the minor
care, advice, and guidance of her own parents. The State cannot replace
referred to under the exception in the second paragraph of Section 7 that
her natural mother and father when it comes to providing her needs and
would enable her to take proper care of her own body and that of her
comfort. To say that their consent is no longer relevant is clearly anti-
unborn child. After all, Section 12, Article II of the Constitution mandates
family. It does not promote unity in the family. It is an affront to the
the State to protect both the life of the mother as that of the unborn child.
constitutional mandate to protect and strengthen the family as an
Considering that information to enable a person to make informed
inviolable social institution.
decisions is essential in the protection and maintenance of ones' health,
access to such information with respect to reproductive health must be
More alarmingly, it disregards and disobeys the constitutional mandate allowed. In this situation, the fear that parents might be deprived of their
that "the natural and primary right and duty of parents in the rearing of the parental control is unfounded because they are not prohibited to exercise
youth for civic efficiency and the development of moral character shall parental guidance and control over their minor child and assist her in
receive the support of the Government."247 In this regard, Commissioner deciding whether to accept or reject the information received.
Bernas wrote:
Second Exception: Life Threatening Cases
As in the case of the conscientious objector, an exception must be made At any rate, Section 12, Article II of the 1987 Constitution provides that
in life-threatening cases that require the performance of emergency the natural and primary right and duty of parents in the rearing of the
procedures. In such cases, the life of the minor who has already suffered youth for civic efficiency and development of moral character shall
a miscarriage and that of the spouse should not be put at grave risk receive the support of the Government. Like the 1973 Constitution and
simply for lack of consent. It should be emphasized that no person should the 1935 Constitution, the 1987 Constitution affirms the State recognition
be denied the appropriate medical care urgently needed to preserve the of the invaluable role of parents in preparing the youth to become
primordial right, that is, the right to life. productive members of society. Notably, it places more importance on the
role of parents in the development of their children by recognizing that
In this connection, the second sentence of Section 23(a)(2)(ii)249 should be said role shall be "primary," that is, that the right of parents in upbringing
struck down. By effectively limiting the requirement of parental consent to the youth is superior to that of the State.252
"only in elective surgical procedures," it denies the parents their right of
parental authority in cases where what is involved are "non-surgical It is also the inherent right of the State to act as parens patriae to aid
procedures." Save for the two exceptions discussed above, and in the parents in the moral development of the youth. Indeed, the Constitution
case of an abused child as provided in the first sentence of Section makes mention of the importance of developing the youth and their
23(a)(2)(ii), the parents should not be deprived of their constitutional right important role in nation building.253 Considering that Section 14 provides
of parental authority. To deny them of this right would be an affront to the not only for the age-appropriate-reproductive health education, but also
constitutional mandate to protect and strengthen the family. for values formation; the development of knowledge and skills in self-
protection against discrimination; sexual abuse and violence against
5 - Academic Freedom women and children and other forms of gender based violence and teen
pregnancy; physical, social and emotional changes in adolescents;
It is asserted that Section 14 of the RH Law, in relation to Section 24 women's rights and children's rights; responsible teenage behavior;
thereof, mandating the teaching of Age-and Development-Appropriate gender and development; and responsible parenthood, and that Rule 10,
Reproductive Health Education under threat of fine and/or imprisonment Section 11.01 of the RH-IRR and Section 4(t) of the RH Law itself
violates the principle of academic freedom . According to the petitioners, provides for the teaching of responsible teenage behavior, gender
these provisions effectively force educational institutions to teach sensitivity and physical and emotional changes among adolescents - the
reproductive health education even if they believe that the same is not Court finds that the legal mandate provided under the assailed provision
suitable to be taught to their students.250 Citing various studies conducted supplements, rather than supplants, the rights and duties of the parents
in the United States and statistical data gathered in the country, the in the moral development of their children.
petitioners aver that the prevalence of contraceptives has led to an
increase of out-of-wedlock births; divorce and breakdown of families; the Furthermore, as Section 14 also mandates that the mandatory
acceptance of abortion and euthanasia; the "feminization of poverty"; the reproductive health education program shall be developed in conjunction
aging of society; and promotion of promiscuity among the youth.251 with parent-teacher-community associations, school officials and other
interest groups, it could very well be said that it will be in line with the
At this point, suffice it to state that any attack on the validity of Section 14 religious beliefs of the petitioners. By imposing such a condition, it
of the RH Law is premature because the Department of Education, becomes apparent that the petitioners' contention that Section 14 violates
Culture and Sports has yet to formulate a curriculum on age-appropriate Article XV, Section 3(1) of the Constitution is without merit.254
reproductive health education. One can only speculate on the content,
manner and medium of instruction that will be used to educate the While the Court notes the possibility that educators might raise their
adolescents and whether they will contradict the religious beliefs of the objection to their participation in the reproductive health education
petitioners and validate their apprehensions. Thus, considering the program provided under Section 14 of the RH Law on the ground that the
premature nature of this particular issue, the Court declines to rule on its same violates their religious beliefs, the Court reserves its judgment
constitutionality or validity. should an actual case be filed before it.
6 - Due Process (n) Public health care service provider refers to: (1) public health care
institution, which is duly licensed and accredited and devoted primarily to
The petitioners contend that the RH Law suffers from vagueness and, the maintenance and operation of facilities for health promotion, disease
thus violates the due process clause of the Constitution. According to prevention, diagnosis, treatment and care of individuals suffering from
them, Section 23 (a)(l) mentions a "private health service provider" illness, disease, injury, disability or deformity, or in need of obstetrical or
among those who may be held punishable but does not define who is a other medical and nursing care; (2) public health care professional, who
"private health care service provider." They argue that confusion further is a doctor of medicine, a nurse or a midvvife; (3) public health worker
results since Section 7 only makes reference to a "private health care engaged in the delivery of health care services; or (4) barangay health
institution." worker who has undergone training programs under any accredited
government and NGO and who voluntarily renders primarily health care
The petitioners also point out that Section 7 of the assailed legislation services in the community after having been accredited to function as
exempts hospitals operated by religious groups from rendering such by the local health board in accordance with the guidelines
reproductive health service and modern family planning methods. It is promulgated by the Department of Health (DOH) .
unclear, however, if these institutions are also exempt from giving
reproductive health information under Section 23(a)(l), or from rendering Further, the use of the term "private health care institution" in Section 7 of
reproductive health procedures under Section 23(a)(2). the law, instead of "private health care service provider," should not be a
cause of confusion for the obvious reason that they are used
Finally, it is averred that the RH Law punishes the withholding, restricting synonymously.
and providing of incorrect information, but at the same time fails to define
"incorrect information." The Court need not belabor the issue of whether the right to be exempt
from being obligated to render reproductive health service and modem
The arguments fail to persuade. family planning methods, includes exemption from being obligated to give
reproductive health information and to render reproductive health
procedures. Clearly, subject to the qualifications and exemptions earlier
A statute or act suffers from the defect of vagueness when it lacks
discussed, the right to be exempt from being obligated to render
comprehensible standards that men of common intelligence must
reproductive health service and modem family planning methods,
necessarily guess its meaning and differ as to its application. It is
necessarily includes exemption from being obligated to give reproductive
repugnant to the Constitution in two respects: (1) it violates due process
health information and to render reproductive health procedures. The
for failure to accord persons, especially the parties targeted by it, fair
terms "service" and "methods" are broad enough to include the providing
notice of the conduct to avoid; and (2) it leaves law enforcers unbridled
of information and the rendering of medical procedures.
discretion in carrying out its provisions and becomes an arbitrary flexing
of the Government muscle.255 Moreover, in determining whether the words
used in a statute are vague, words must not only be taken in accordance The same can be said with respect to the contention that the RH Law
with their plain meaning alone, but also in relation to other parts of the punishes health care service providers who intentionally withhold, restrict
statute. It is a rule that every part of the statute must be interpreted with and provide incorrect information regarding reproductive health programs
reference to the context, that is, every part of it must be construed and services. For ready reference, the assailed provision is hereby
together with the other parts and kept subservient to the general intent of quoted as follows:
the whole enactment.256
SEC. 23. Prohibited Acts. - The following acts are prohibited:
As correctly noted by the OSG, in determining the definition of "private
health care service provider," reference must be made to Section 4(n) of (a) Any health care service provider, whether public or private, who shall:
the RH Law which defines a "public health service provider," viz:
(1) Knowingly withhold information or restrict the dissemination thereof,
and/ or intentionally provide incorrect information regarding programs and
services on reproductive health including the right to informed choice and clause, however, to provide for a more specific guaranty against any form
access to a full range of legal, medically-safe, non-abortifacient and of undue favoritism or hostility from the government. Arbitrariness in
effective family planning methods; general may be challenged on the basis of the due process clause. But if
the particular act assailed partakes of an unwarranted partiality or
From its plain meaning, the word "incorrect" here denotes failing to agree prejudice, the sharper weapon to cut it down is the equal protection
with a copy or model or with established rules; inaccurate, faulty; failing clause.
to agree with the requirements of duty, morality or propriety; and failing to
coincide with the truth. 257 On the other hand, the word "knowingly" means "According to a long line of decisions, equal protection simply requires
with awareness or deliberateness that is intentional.258 Used together in that all persons or things similarly situated should be treated alike, both
relation to Section 23(a)(l), they connote a sense of malice and ill motive as to rights conferred and responsibilities imposed." It "requires public
to mislead or misrepresent the public as to the nature and effect of bodies and inst itutions to treat similarly situated individuals in a similar
programs and services on reproductive health. Public health and safety manner." "The purpose of the equal protection clause is to secure every
demand that health care service providers give their honest and correct person within a state's jurisdiction against intentional and arbitrary
medical information in accordance with what is acceptable in medical discrimination, whether occasioned by the express terms of a statue or by
practice. While health care service providers are not barred from its improper execution through the state's duly constituted authorities." "In
expressing their own personal opinions regarding the programs and other words, the concept of equal justice under the law requires the state
services on reproductive health, their right must be tempered with the to govern impartially, and it may not draw distinctions between individuals
need to provide public health and safety. The public deserves no less. solely on differences that are irrelevant to a legitimate governmental
objective."
7-Egual Protection
The equal protection clause is aimed at all official state actions, not just
The petitioners also claim that the RH Law violates the equal protection those of the legislature. Its inhibitions cover all the departments of the
clause under the Constitution as it discriminates against the poor government including the political and executive departments, and
because it makes them the primary target of the government program extend to all actions of a state denying equal protection of the laws,
that promotes contraceptive use . They argue that, rather than promoting through whatever agency or whatever guise is taken.
reproductive health among the poor, the RH Law introduces
contraceptives that would effectively reduce the number of the poor. It, however, does not require the universal application of the laws to all
Their bases are the various provisions in the RH Law dealing with the persons or things without distinction. What it simply requires is equality
poor, especially those mentioned in the guiding principles259 and definition among equals as determined according to a valid classification. Indeed,
of terms260 of the law. the equal protection clause permits classification. Such classification,
however, to be valid must pass the test of reasonableness. The test has
They add that the exclusion of private educational institutions from the four requisites: (1) The classification rests on substantial distinctions; (2)
mandatory reproductive health education program imposed by the RH It is germane to the purpose of the law; (3) It is not limited to existing
Law renders it unconstitutional. conditions only; and (4) It applies equally to all members of the same
class. "Superficial differences do not make for a valid classification."
In Biraogo v. Philippine Truth Commission,261 the Court had the occasion
to expound on the concept of equal protection. Thus: For a classification to meet the requirements of constitutionality, it must
include or embrace all persons who naturally belong to the class. "The
One of the basic principles on which this government was founded is that classification will be regarded as invalid if all the members of the class
of the equality of right which is embodied in Section 1, Article III of the are not similarly treated, both as to rights conferred and obligations
1987 Constitution. The equal protection of the laws is embraced in the imposed. It is not necessary that the classification be made with absolute
concept of due process, as every unfair discrimination offends the symmetry, in the sense that the members of the class should possess the
requirements of justice and fair play. It has been embodied in a separate same characteristics in equal degree. Substantial similarity will suffice;
and as long as this is achieved, all those covered by the classification are poor in the implementation of government programs to promote basic
to be treated equally. The mere fact that an individual belonging to a reproductive health care.
class differs from the other members, as long as that class is
substantially distinguishable from all others, does not justify the non- With respect to the exclusion of private educational institutions from the
application of the law to him." mandatory reproductive health education program under Section 14,
suffice it to state that the mere fact that the children of those who are less
The classification must not be based on existing circumstances only, or fortunate attend public educational institutions does not amount to
so constituted as to preclude addition to the number included in the class. substantial distinction sufficient to annul the assailed provision. On the
It must be of such a nature as to embrace all those who may thereafter other hand, substantial distinction rests between public educational
be in similar circumstances and conditions. It must not leave out or institutions and private educational institutions, particularly because there
"underinclude" those that should otherwise fall into a certain is a need to recognize the academic freedom of private educational
classification. [Emphases supplied; citations excluded] institutions especially with respect to religious instruction and to consider
their sensitivity towards the teaching of reproductive health education.
To provide that the poor are to be given priority in the government's
reproductive health care program is not a violation of the equal protection 8-Involuntary Servitude
clause. In fact, it is pursuant to Section 11, Article XIII of the Constitution
which recognizes the distinct necessity to address the needs of the The petitioners also aver that the RH Law is constitutionally infirm as it
underprivileged by providing that they be given priority in addressing the violates the constitutional prohibition against involuntary servitude. They
health development of the people. Thus: posit that Section 17 of the assailed legislation requiring private and non-
government health care service providers to render forty-eight (48) hours
Section 11. The State shall adopt an integrated and comprehensive of pro bono reproductive health services, actually amounts to involuntary
approach to health development which shall endeavor to make essential servitude because it requires medical practitioners to perform acts
goods, health and other social services available to all the people at against their will.262
affordable cost. There shall be priority for the needs of the
underprivileged, sick, elderly, disabled, women, and children. The State The OSG counters that the rendition of pro bono services envisioned in
shall endeavor to provide free medical care to paupers. Section 17 can hardly be considered as forced labor analogous to
slavery, as reproductive health care service providers have the discretion
It should be noted that Section 7 of the RH Law prioritizes poor and as to the manner and time of giving pro bono services. Moreover, the
marginalized couples who are suffering from fertility issues and desire to OSG points out that the imposition is within the powers of the
have children. There is, therefore, no merit to the contention that the RH government, the accreditation of medical practitioners with PhilHealth
Law only seeks to target the poor to reduce their number. While the RH being a privilege and not a right.
Law admits the use of contraceptives, it does not, as elucidated above,
sanction abortion. As Section 3(1) explains, the "promotion and/or The point of the OSG is well-taken.
stabilization of the population growth rate is incidental to the
advancement of reproductive health." It should first be mentioned that the practice of medicine is undeniably
imbued with public interest that it is both a power and a duty of the State
Moreover, the RH Law does not prescribe the number of children a to control and regulate it in order to protect and promote the public
couple may have and does not impose conditions upon couples who welfare. Like the legal profession, the practice of medicine is not a right
intend to have children. While the petitioners surmise that the assailed but a privileged burdened with conditions as it directly involves the very
law seeks to charge couples with the duty to have children only if they lives of the people. A fortiori, this power includes the power of
would raise them in a truly humane way, a deeper look into its provisions Congress263 to prescribe the qualifications for the practice of professions
shows that what the law seeks to do is to simply provide priority to the or trades which affect the public welfare, the public health, the public
morals, and the public safety; and to regulate or control such professions Department of Health (DOH). Said Administration shall be under the
or trades, even to the point of revoking such right altogether.264 Office of the Secretary and shall have the following functions, powers and
duties:
Moreover, as some petitioners put it, the notion of involuntary servitude
connotes the presence of force, threats, intimidation or other similar "(a) To administer the effective implementation of this Act and of
means of coercion and compulsion.265 A reading of the assailed provision, the rules and regulations issued pursuant to the same;
however, reveals that it only encourages private and non- government
reproductive healthcare service providers to render pro bono service. "(b) To assume primary jurisdiction in the collection of samples of
Other than non-accreditation with PhilHealth, no penalty is imposed health products;
should they choose to do otherwise. Private and non-government
reproductive healthcare service providers also enjoy the liberty to choose "(c) To analyze and inspect health products in connection with the
which kind of health service they wish to provide, when, where and how implementation of this Act;
to provide it or whether to provide it all. Clearly, therefore, no compulsion,
force or threat is made upon them to render pro bono service against
"(d) To establish analytical data to serve as basis for the
their will. While the rendering of such service was made a prerequisite to
preparation of health products standards, and to recommend
accreditation with PhilHealth, the Court does not consider the same to be
standards of identity, purity, safety, efficacy, quality and fill of
an unreasonable burden, but rather, a necessary incentive imposed by
container;
Congress in the furtherance of a perceived legitimate state interest.
"(e) To issue certificates of compliance with technical
Consistent with what the Court had earlier discussed, however, it should
requirements to serve as basis for the issuance of appropriate
be emphasized that conscientious objectors are exempt from this
authorization and spot-check for compliance with regulations
provision as long as their religious beliefs and convictions do not allow
regarding operation of manufacturers, importers, exporters,
them to render reproductive health service, pro bona or otherwise.
distributors, wholesalers, drug outlets, and other establishments
and facilities of health products, as determined by the FDA;
9-Delegation of Authority to the FDA
"x x x
The petitioners likewise question the delegation by Congress to the FDA
of the power to determine whether or not a supply or product is to be
"(h) To conduct appropriate tests on all applicable health products
included in the Essential Drugs List (EDL).266
prior to the issuance of appropriate authorizations to ensure
safety, efficacy, purity, and quality;
The Court finds nothing wrong with the delegation. The FDA does not
only have the power but also the competency to evaluate, register and
"(i) To require all manufacturers, traders, distributors, importers,
cover health services and methods. It is the only government entity
exporters, wholesalers, retailers, consumers, and non-consumer
empowered to render such services and highly proficient to do so. It
users of health products to report to the FDA any incident that
should be understood that health services and methods fall under the
reasonably indicates that said product has caused or contributed
gamut of terms that are associated with what is ordinarily understood as
to the death, serious illness or serious injury to a consumer, a
"health products."
patient, or any person;
In this connection, Section 4 of R.A. No. 3 720, as amended by R.A. No.
"(j) To issue cease and desist orders motu propio or upon verified
9711 reads:
complaint for health products, whether or not registered with the
FDA Provided, That for registered health products, the cease and
SEC. 4. To carry out the provisions of this Act, there is hereby created an
office to be called the Food and Drug Administration (FDA) in the
desist order is valid for thirty (30) days and may be extended for As for the autonomy of local governments, the petitioners claim that the
sixty ( 60) days only after due process has been observed; RH Law infringes upon the powers devolved to local government units
(LGUs) under Section 17 of the Local Government Code. Said Section 17
"(k) After due process, to order the ban, recall, and/or withdrawal vested upon the LGUs the duties and functions pertaining to the delivery
of any health product found to have caused death, serious illness of basic services and facilities, as follows:
or serious injury to a consumer or patient, or is found to be
imminently injurious, unsafe, dangerous, or grossly deceptive, SECTION 17. Basic Services and Facilities. –
and to require all concerned to implement the risk management
plan which is a requirement for the issuance of the appropriate (a) Local government units shall endeavor to be self-reliant and
authorization; shall continue exercising the powers and discharging the duties
and functions currently vested upon them. They shall also
x x x. discharge the functions and responsibilities of national agencies
and offices devolved to them pursuant to this Code. Local
As can be gleaned from the above, the functions, powers and duties of government units shall likewise exercise such other powers and
the FDA are specific to enable the agency to carry out the mandates of discharge such other functions and responsibilities as are
the law. Being the country's premiere and sole agency that ensures the necessary, appropriate, or incidental to efficient and effective
safety of food and medicines available to the public, the FDA was provision of the basic services and facilities enumerated herein.
equipped with the necessary powers and functions to make it effective.
Pursuant to the principle of necessary implication, the mandate by (b) Such basic services and facilities include, but are not limited
Congress to the FDA to ensure public health and safety by permitting to, x x x.
only food and medicines that are safe includes "service" and "methods."
From the declared policy of the RH Law, it is clear that Congress While the aforementioned provision charges the LGUs to take on
intended that the public be given only those medicines that are proven the functions and responsibilities that have already been
medically safe, legal, non-abortifacient, and effective in accordance with devolved upon them from the national agencies on the aspect of
scientific and evidence-based medical research standards. The providing for basic services and facilities in their respective
philosophy behind the permitted delegation was explained in Echagaray jurisdictions, paragraph (c) of the same provision provides a
v. Secretary of Justice,267 as follows: categorical exception of cases involving nationally-funded
projects, facilities, programs and services.268 Thus:
The reason is the increasing complexity of the task of the government
and the growing inability of the legislature to cope directly with the many (c) Notwithstanding the provisions of subsection (b) hereof, public
problems demanding its attention. The growth of society has ramified its works and infrastructure projects and other facilities, programs
activities and created peculiar and sophisticated problems that the and services funded by the National Government under the
legislature cannot be expected reasonably to comprehend. Specialization annual General Appropriations Act, other special laws, pertinent
even in legislation has become necessary. To many of the problems executive orders, and those wholly or partially funded from
attendant upon present day undertakings, the legislature may not have foreign sources, are not covered under this Section, except in
the competence, let alone the interest and the time, to provide the those cases where the local government unit concerned is duly
required direct and efficacious, not to say specific solutions. designated as the implementing agency for such projects,
facilities, programs and services. [Emphases supplied]
10- Autonomy of Local Governments and the Autonomous Region
The essence of this express reservation of power by the national
of Muslim Mindanao (ARMM) government is that, unless an LGU is particularly designated as the
implementing agency, it has no power over a program for which funding
has been provided by the national government under the annual general
appropriations act, even if the program involves the delivery of basic With respect to the argument that the RH Law violates natural
services within the jurisdiction of the LGU.269 A complete relinquishment of law,276 suffice it to say that the Court does not duly recognize it as a legal
central government powers on the matter of providing basic facilities and basis for upholding or invalidating a law. Our only guidepost is the
services cannot be implied as the Local Government Code itself weighs Constitution. While every law enacted by man emanated from what is
against it.270 perceived as natural law, the Court is not obliged to see if a statute,
executive issuance or ordinance is in conformity to it. To begin with, it is
In this case, a reading of the RH Law clearly shows that whether it not enacted by an acceptable legitimate body. Moreover, natural laws are
pertains to the establishment of health care facilities,271 the hiring of skilled mere thoughts and notions on inherent rights espoused by theorists,
health professionals,272 or the training of barangay health workers,273 it will philosophers and theologists. The jurists of the philosophical school are
be the national government that will provide for the funding of its interested in the law as an abstraction, rather than in the actual law of the
implementation. Local autonomy is not absolute. The national past or present.277 Unless, a natural right has been transformed into a
government still has the say when it comes to national priority programs written law, it cannot serve as a basis to strike down a law. In Republic v.
which the local government is called upon to implement like the RH Law. Sandiganbayan,278 the very case cited by the petitioners, it was explained
that the Court is not duty-bound to examine every law or action and
Moreover, from the use of the word "endeavor," the LG Us are merely whether it conforms with both the Constitution and natural law. Rather,
encouraged to provide these services. There is nothing in the wording of natural law is to be used sparingly only in the most peculiar of
the law which can be construed as making the availability of these circumstances involving rights inherent to man where no law is
services mandatory for the LGUs. For said reason, it cannot be said that applicable.279
the RH Law amounts to an undue encroachment by the national
government upon the autonomy enjoyed by the local governments. At any rate, as earlier expounded, the RH Law does not sanction the
taking away of life. It does not allow abortion in any shape or form. It only
The ARMM seeks to enhance the population control program of the government by
providing information and making non-abortifacient contraceptives more
readily available to the public, especially to the poor.
The fact that the RH Law does not intrude in the autonomy of local
governments can be equally applied to the ARMM. The RH Law does not
infringe upon its autonomy. Moreover, Article III, Sections 6, 10 and 11 of Facts and Fallacies
R.A. No. 9054, or the organic act of the ARMM, alluded to by petitioner
Tillah to justify the exemption of the operation of the RH Law in the and the Wisdom of the Law
autonomous region, refer to the policy statements for the guidance of the
regional government. These provisions relied upon by the petitioners In general, the Court does not find the RH Law as unconstitutional insofar
simply delineate the powers that may be exercised by the regional as it seeks to provide access to medically-safe, non-abortifacient,
government, which can, in no manner, be characterized as an abdication effective, legal, affordable, and quality reproductive healthcare services,
by the State of its power to enact legislation that would benefit the methods, devices, and supplies. As earlier pointed out, however, the
general welfare. After all, despite the veritable autonomy granted the religious freedom of some sectors of society cannot be trampled upon in
ARMM, the Constitution and the supporting jurisprudence, as they now pursuit of what the law hopes to achieve. After all, the Constitutional
stand, reject the notion of imperium et imperio in the relationship between safeguard to religious freedom is a recognition that man stands
the national and the regional governments.274 Except for the express and accountable to an authority higher than the State.
implied limitations imposed on it by the Constitution, Congress cannot be
restricted to exercise its inherent and plenary power to legislate on all In conformity with the principle of separation of Church and State, one
subjects which extends to all matters of general concern or common religious group cannot be allowed to impose its beliefs on the rest of the
interest.275 society. Philippine modem society leaves enough room for diversity and
pluralism. As such, everyone should be tolerant and open-minded so that
11 - Natural Law
peace and harmony may continue to reign as we exist alongside each cold neutrality, the Court must carry out the delicate function of
other. interpreting the law, guided by the Constitution and existing legislation
and mindful of settled jurisprudence. The Court's function is therefore
As healthful as the intention of the RH Law may be, the idea does not limited, and accordingly, must confine itself to the judicial task of saying
escape the Court that what it seeks to address is the problem of rising what the law is, as enacted by the lawmaking body.281
poverty and unemployment in the country. Let it be said that the cause of
these perennial issues is not the large population but the unequal Be that as it may, it bears reiterating that the RH Law is a mere
distribution of wealth. Even if population growth is controlled, poverty will compilation and enhancement of the prior existing contraceptive and
remain as long as the country's wealth remains in the hands of the very reproductive health laws, but with coercive measures. Even if the Court
few. decrees the RH Law as entirely unconstitutional, there will still be the
Population Act (R.A. No. 6365), the Contraceptive Act (R.A. No. 4729)
At any rate, population control may not be beneficial for the country in the and the reproductive health for women or The Magna Carta of Women
long run. The European and Asian countries, which embarked on such a (R.A. No. 9710), sans the coercive provisions of the assailed legislation.
program generations ago , are now burdened with ageing populations. All the same, the principle of "no-abortion" and "non-coercion" in the
The number of their young workers is dwindling with adverse effects on adoption of any family planning method should be maintained.
their economy. These young workers represent a significant human
capital which could have helped them invigorate, innovate and fuel their WHEREFORE, the petitions are PARTIALLY GRANTED. Accordingly,
economy. These countries are now trying to reverse their programs, but the Court declares R.A. No. 10354 as NOT UNCONSTITUTIONAL
they are still struggling. For one, Singapore, even with incentives, is except with respect to the following provisions which are declared
failing. UNCONSTITUTIONAL:

And in this country, the economy is being propped up by remittances 1) Section 7 and the corresponding provision in the RH-IRR
from our Overseas Filipino Workers. This is because we have an ample insofar as they: a) require private health facilities and non-
supply of young able-bodied workers. What would happen if the country maternity specialty hospitals and hospitals owned and operated
would be weighed down by an ageing population and the fewer younger by a religious group to refer patients, not in an emergency or life-
generation would not be able to support them? This would be the threatening case, as defined under Republic Act No. 8344, to
situation when our total fertility rate would go down below the another health facility which is conveniently accessible; and b)
replacement level of two (2) children per woman.280 allow minor-parents or minors who have suffered a miscarriage
access to modem methods of family planning without written
Indeed, at the present, the country has a population problem, but the consent from their parents or guardian/s;
State should not use coercive measures (like the penal provisions of the
RH Law against conscientious objectors) to solve it. Nonetheless, the 2) Section 23(a)(l) and the corresponding provision in the RH-
policy of the Court is non-interference in the wisdom of a law. IRR, particularly Section 5 .24 thereof, insofar as they punish any
healthcare service provider who fails and or refuses to
x x x. But this Court cannot go beyond what the legislature has laid down. disseminate information regarding programs and services on
Its duty is to say what the law is as enacted by the lawmaking body. That reproductive health regardless of his or her religious beliefs.
is not the same as saying what the law should be or what is the correct
rule in a given set of circumstances. It is not the province of the judiciary 3) Section 23(a)(2)(i) and the corresponding provision in the RH-
to look into the wisdom of the law nor to question the policies adopted by IRR insofar as they allow a married individual, not in an
the legislative branch. Nor is it the business of this Tribunal to remedy emergency or life-threatening case, as defined under Republic
every unjust situation that may arise from the application of a particular Act No. 8344, to undergo reproductive health procedures without
law. It is for the legislature to enact remedial legislation if that would be the consent of the spouse;
necessary in the premises. But as always, with apt judicial caution and
4) Section 23(a)(2)(ii) and the corresponding provision in the RH-
IRR insofar as they limit the requirement of parental consent only
to elective surgical procedures.

5) Section 23(a)(3) and the corresponding provision in the RH-


IRR, particularly Section 5.24 thereof, insofar as they punish any
healthcare service provider who fails and/or refuses to refer a
patient not in an emergency or life-threatening case, as defined
under Republic Act No. 8344, to another health care service
provider within the same facility or one which is conveniently
accessible regardless of his or her religious beliefs;

6) Section 23(b) and the corresponding provision in the RH-IRR,


particularly Section 5 .24 thereof, insofar as they punish any
public officer who refuses to support reproductive health
programs or shall do any act that hinders the full implementation
of a reproductive health program, regardless of his or her
religious beliefs;

7) Section 17 and the corresponding prov1s10n in the RH-IRR


regarding the rendering of pro bona reproductive health service in
so far as they affect the conscientious objector in securing
PhilHealth accreditation; and

8) Section 3.0l(a) and Section 3.01 G) of the RH-IRR, which


added the qualifier "primarily" in defining abortifacients and
contraceptives, as they are ultra vires and, therefore, null and
void for contravening Section 4(a) of the RH Law and violating
Section 12, Article II of the Constitution.

The Status Quo Ante Order issued by the Court on March 19, 2013 as
extended by its Order, dated July 16, 2013 , is hereby LIFTED, insofar as
the provisions of R.A. No. 10354 which have been herein declared as
constitutional.

SO ORDERED.
G.R. No. 190582 April 8, 2010 This is a Petition for Certiorari under Rule 65 of the Rules of Court, with
an application for a writ of preliminary mandatory injunction, filed by Ang
ANG LADLAD LGBT PARTY represented herein by its Chair, Ladlad LGBT Party (Ang Ladlad) against the Resolutions of the
DANTON REMOTO, Petitioner, Commission on Elections (COMELEC) dated November 11, 20092 (the
vs. First Assailed Resolution) and December 16, 20093 (the Second Assailed
COMMISSION ON ELECTIONS Respondent. Resolution) in SPP No. 09-228 (PL) (collectively, the Assailed
Resolutions). The case has its roots in the COMELEC’s refusal to
DECISION accredit Ang Ladlad as a party-list organization under Republic Act (RA)
No. 7941, otherwise known as the Party-List System Act.4
DEL CASTILLO, J.:
Ang Ladlad is an organization composed of men and women who identify
themselves as lesbians, gays, bisexuals, or trans-gendered individuals
... [F]reedom to differ is not limited to things that do not matter much. That
(LGBTs). Incorporated in 2003, Ang Ladlad first applied for registration
would be a mere shadow of freedom. The test of its substance is the right
with the COMELEC in 2006. The application for accreditation was denied
to differ as to things that touch the heart of the existing order.
on the ground that the organization had no substantial membership base.
On August 17, 2009, Ang Ladlad again filed a Petition5 for registration
Justice Robert A. Jackson with the COMELEC.

West Virginia State Board of Education v. Barnette1 Before the COMELEC, petitioner argued that the LGBT community is a
marginalized and under-represented sector that is particularly
One unavoidable consequence of everyone having the freedom to disadvantaged because of their sexual orientation and gender identity;
choose is that others may make different choices – choices we would not that LGBTs are victims of exclusion, discrimination, and violence; that
make for ourselves, choices we may disapprove of, even choices that because of negative societal attitudes, LGBTs are constrained to hide
may shock or offend or anger us. However, choices are not to be legally their sexual orientation; and that Ang Ladlad complied with the 8-point
prohibited merely because they are different, and the right to disagree guidelines enunciated by this Court in Ang Bagong Bayani-OFW Labor
and debate about important questions of public policy is a core value Party v. Commission on Elections.6 Ang Ladlad laid out its national
protected by our Bill of Rights. Indeed, our democracy is built on genuine membership base consisting of individual members and organizational
recognition of, and respect for, diversity and difference in opinion. supporters, and outlined its platform of governance.7

Since ancient times, society has grappled with deep disagreements On November 11, 2009, after admitting the petitioner’s evidence, the
about the definitions and demands of morality. In many cases, where COMELEC (Second Division) dismissed the Petition on moral grounds,
moral convictions are concerned, harmony among those theoretically stating that:
opposed is an insurmountable goal. Yet herein lies the paradox –
philosophical justifications about what is moral are indispensable and yet x x x This Petition is dismissible on moral grounds. Petitioner defines the
at the same time powerless to create agreement. This Court recognizes, Filipino Lesbian, Gay, Bisexual and Transgender (LGBT) Community,
however, that practical solutions are preferable to ideological stalemates; thus:
accommodation is better than intransigence; reason more worthy than
rhetoric. This will allow persons of diverse viewpoints to live together, if
x x x a marginalized and under-represented sector that is particularly
not harmoniously, then, at least, civilly.
disadvantaged because of their sexual orientation and gender identity.
Factual Background
and proceeded to define sexual orientation as that which:
x x x refers to a person’s capacity for profound emotional, affectional and It also collides with Article 1306 of the Civil Code: ‘The contracting parties
sexual attraction to, and intimate and sexual relations with, individuals of may establish such stipulations, clauses, terms and conditions as they
a different gender, of the same gender, or more than one gender." may deem convenient, provided they are not contrary to law, morals,
good customs, public order or public policy. Art 1409 of the Civil Code
This definition of the LGBT sector makes it crystal clear that petitioner provides that ‘Contracts whose cause, object or purpose is contrary to
tolerates immorality which offends religious beliefs. In Romans 1:26, 27, law, morals, good customs, public order or public policy’ are inexistent
Paul wrote: and void from the beginning.

For this cause God gave them up into vile affections, for even their Finally to safeguard the morality of the Filipino community, the Revised
women did change the natural use into that which is against nature: And Penal Code, as amended, penalizes ‘Immoral doctrines, obscene
likewise also the men, leaving the natural use of the woman, burned in publications and exhibitions and indecent shows’ as follows:
their lust one toward another; men with men working that which is
unseemly, and receiving in themselves that recompense of their error Art. 201. Immoral doctrines, obscene publications and exhibitions, and
which was meet. indecent shows. — The penalty of prision mayor or a fine ranging from
six thousand to twelve thousand pesos, or both such imprisonment and
In the Koran, the hereunder verses are pertinent: fine, shall be imposed upon:

For ye practice your lusts on men in preference to women "ye are indeed 1. Those who shall publicly expound or proclaim doctrines
a people transgressing beyond bounds." (7.81) "And we rained down on openly contrary to public morals;
them a shower (of brimstone): Then see what was the end of those who
indulged in sin and crime!" (7:84) "He said: "O my Lord! Help Thou me 2. (a) The authors of obscene literature, published with their
against people who do mischief" (29:30). knowledge in any form; the editors publishing such literature; and
the owners/operators of the establishment selling the same;
As correctly pointed out by the Law Department in its Comment dated
October 2, 2008: (b) Those who, in theaters, fairs, cinematographs or any
other place, exhibit indecent or immoral plays, scenes,
The ANG LADLAD apparently advocates sexual immorality as indicated acts or shows, it being understood that the obscene
in the Petition’s par. 6F: ‘Consensual partnerships or relationships by literature or indecent or immoral plays, scenes, acts or
gays and lesbians who are already of age’. It is further indicated in par. shows, whether live or in film, which are prescribed by
24 of the Petition which waves for the record: ‘In 2007, Men Having Sex virtue hereof, shall include those which: (1) glorify
with Men or MSMs in the Philippines were estimated as 670,000 criminals or condone crimes; (2) serve no other purpose
(Genesis 19 is the history of Sodom and Gomorrah). but to satisfy the market for violence, lust or pornography;
(3) offend any race or religion; (4) tend to abet traffic in
Laws are deemed incorporated in every contract, permit, license, and use of prohibited drugs; and (5) are contrary to law,
relationship, or accreditation. Hence, pertinent provisions of the Civil public order, morals, good customs, established policies,
Code and the Revised Penal Code are deemed part of the requirement to lawful orders, decrees and edicts.
be complied with for accreditation.
3. Those who shall sell, give away or exhibit films, prints,
ANG LADLAD collides with Article 695 of the Civil Code which defines engravings, sculpture or literature which are offensive to morals.
nuisance as ‘Any act, omission, establishment, business, condition of
property, or anything else which x x x (3) shocks, defies; or Petitioner should likewise be denied accreditation not only for advocating
disregards decency or morality x x x immoral doctrines but likewise for not being truthful when it said that it "or
any of its nominees/party-list representatives have not violated or failed the attention of the nation because of their under representation. Until the
to comply with laws, rules, or regulations relating to the elections." time comes when Ladlad is able to justify that having mixed sexual
orientations and transgender identities is beneficial to the nation, its
Furthermore, should this Commission grant the petition, we will be application for accreditation under the party-list system will remain just
exposing our youth to an environment that does not conform to the that.
teachings of our faith. Lehman Strauss, a famous bible teacher and writer
in the U.S.A. said in one article that "older practicing homosexuals are a II. No substantial differentiation
threat to the youth." As an agency of the government, ours too is the
State’s avowed duty under Section 13, Article II of the Constitution to In the United States, whose equal protection doctrine pervades Philippine
protect our youth from moral and spiritual degradation.8 jurisprudence, courts do not recognize lesbians, gays, homosexuals, and
bisexuals (LGBT) as a "special class" of individuals. x x x Significantly, it
When Ang Ladlad sought reconsideration,9 three commissioners voted to has also been held that homosexuality is not a constitutionally protected
overturn the First Assailed Resolution (Commissioners Gregorio Y. fundamental right, and that "nothing in the U.S. Constitution discloses a
Larrazabal, Rene V. Sarmiento, and Armando Velasco), while three comparable intent to protect or promote the social or legal equality of
commissioners voted to deny Ang Ladlad’s Motion for Reconsideration homosexual relations," as in the case of race or religion or belief.
(Commissioners Nicodemo T. Ferrer, Lucenito N. Tagle, and Elias R.
Yusoph). The COMELEC Chairman, breaking the tie and speaking for xxxx
the majority in his Separate Opinion, upheld the First Assailed
Resolution, stating that: Thus, even if society’s understanding, tolerance, and acceptance of
LGBT’s is elevated, there can be no denying that Ladlad constituencies
I. The Spirit of Republic Act No. 7941 are still males and females, and they will remain either male or female
protected by the same Bill of Rights that applies to all citizens alike.
Ladlad is applying for accreditation as a sectoral party in the party-list
system. Even assuming that it has properly proven its under- xxxx
representation and marginalization, it cannot be said that Ladlad’s
expressed sexual orientations per se would benefit the nation as a whole. IV. Public Morals

Section 2 of the party-list law unequivocally states that the purpose of the x x x There is no question about not imposing on Ladlad Christian or
party-list system of electing congressional representatives is to enable Muslim religious practices. Neither is there any attempt to any particular
Filipino citizens belonging to marginalized and under-represented religious group’s moral rules on Ladlad. Rather, what are being adopted
sectors, organizations and parties, and who lack well-defined political as moral parameters and precepts are generally accepted public morals.
constituencies but who could contribute to the formulation and enactment They are possibly religious-based, but as a society, the Philippines
of appropriate legislation that will benefit the nation as a whole, to cannot ignore its more than 500 years of Muslim and Christian
become members of the House of Representatives. upbringing, such that some moral precepts espoused by said religions
have sipped [sic] into society and these are not publicly accepted moral
If entry into the party-list system would depend only on the ability of an norms.
organization to represent its constituencies, then all representative
organizations would have found themselves into the party-list race. But V. Legal Provisions
that is not the intention of the framers of the law. The party-list system is
not a tool to advocate tolerance and acceptance of misunderstood
But above morality and social norms, they have become part of the law of
persons or groups of persons. Rather, the party-list system is a tool for
the land. Article 201 of the Revised Penal Code imposes the penalty of
the realization of aspirations of marginalized individuals whose interests
prision mayor upon "Those who shall publicly expound or proclaim
are also the nation’s – only that their interests have not been brought to
doctrines openly contrary to public morals." It penalizes "immoral On January 26, 2010, Epifanio D. Salonga, Jr. filed his Motion to
doctrines, obscene publications and exhibition and indecent shows." "Ang Intervene18 which motion was granted on February 2, 2010.19
Ladlad" apparently falls under these legal provisions. This is clear from its
Petition’s paragraph 6F: "Consensual partnerships or relationships by The Parties’ Arguments
gays and lesbians who are already of age’ It is further indicated in par. 24
of the Petition which waves for the record: ‘In 2007, Men Having Sex with Ang Ladlad argued that the denial of accreditation, insofar as it justified
Men or MSMs in the Philippines were estimated as 670,000. Moreoever, the exclusion by using religious dogma, violated the constitutional
Article 694 of the Civil Code defines "nuisance" as any act, omission x x x guarantees against the establishment of religion. Petitioner also claimed
or anything else x x x which shocks, defies or disregards decency or that the Assailed Resolutions contravened its constitutional rights to
morality x x x." These are all unlawful.10 privacy, freedom of speech and assembly, and equal protection of laws,
as well as constituted violations of the Philippines’ international
On January 4, 2010, Ang Ladlad filed this Petition, praying that the Court obligations against discrimination based on sexual orientation.
annul the Assailed Resolutions and direct the COMELEC to grant Ang
Ladlad’s application for accreditation. Ang Ladlad also sought the The OSG concurred with Ang Ladlad’s petition and argued that the
issuance ex parte of a preliminary mandatory injunction against the COMELEC erred in denying petitioner’s application for registration since
COMELEC, which had previously announced that it would begin printing there was no basis for COMELEC’s allegations of immorality. It also
the final ballots for the May 2010 elections by January 25, 2010. opined that LGBTs have their own special interests and concerns which
should have been recognized by the COMELEC as a separate
On January 6, 2010, we ordered the Office of the Solicitor General (OSG) classification. However, insofar as the purported violations of petitioner’s
to file its Comment on behalf of COMELEC not later than 12:00 noon of freedom of speech, expression, and assembly were concerned, the OSG
January 11, 2010.11 Instead of filing a Comment, however, the OSG filed maintained that there had been no restrictions on these rights.
a Motion for Extension, requesting that it be given until January 16, 2010
to Comment.12 Somewhat surprisingly, the OSG later filed a Comment in In its Comment, the COMELEC reiterated that petitioner does not have a
support of petitioner’s application.13 Thus, in order to give COMELEC the concrete and genuine national political agenda to benefit the nation and
opportunity to fully ventilate its position, we required it to file its own that the petition was validly dismissed on moral grounds. It also argued
comment.14 The COMELEC, through its Law Department, filed its for the first time that the LGBT sector is not among the sectors
Comment on February 2, 2010.15 enumerated by the Constitution and RA 7941, and that petitioner made
untruthful statements in its petition when it alleged its national existence
In the meantime, due to the urgency of the petition, we issued a contrary to actual verification reports by COMELEC’s field personnel.
temporary restraining order on January 12, 2010, effective immediately
and continuing until further orders from this Court, directing the Our Ruling
COMELEC to cease and desist from implementing the Assailed
Resolutions.16
We grant the petition.
Also, on January 13, 2010, the Commission on Human Rights (CHR) filed
Compliance with the Requirements of the Constitution and Republic Act
a Motion to Intervene or to Appear as Amicus Curiae, attaching thereto
No. 7941
its Comment-in-Intervention.17 The CHR opined that the denial of Ang
Ladlad’spetition on moral grounds violated the standards and principles
of the Constitution, the Universal Declaration of Human Rights (UDHR), The COMELEC denied Ang Ladlad’s application for registration on the
and the International Covenant on Civil and Political Rights (ICCPR). On ground that the LGBT sector is neither enumerated in the Constitution
January 19, 2010, we granted the CHR’s motion to intervene. and RA 7941, nor is it associated with or related to any of the sectors in
the enumeration.
Respondent mistakenly opines that our ruling in Ang Bagong Bayani § Abra Gay Association
stands for the proposition that only those sectors specifically enumerated
in the law or related to said sectors (labor, peasant, fisherfolk, urban § Aklan Butterfly Brigade (ABB) – Aklan
poor, indigenous cultural communities, elderly, handicapped, women,
youth, veterans, overseas workers, and professionals) may be registered § Albay Gay Association
under the party-list system. As we explicitly ruled in Ang Bagong Bayani-
OFW Labor Party v. Commission on Elections,20 "the enumeration of
§ Arts Center of Cabanatuan City – Nueva Ecija
marginalized and under-represented sectors is not exclusive". The crucial
element is not whether a sector is specifically enumerated, but whether a
particular organization complies with the requirements of the Constitution § Boys Legion – Metro Manila
and RA 7941.
§ Cagayan de Oro People Like Us (CDO PLUS)
Respondent also argues that Ang Ladlad made untruthful statements in
its petition when it alleged that it had nationwide existence through its § Can’t Live in the Closet, Inc. (CLIC) – Metro Manila
members and affiliate organizations. The COMELEC claims that upon
verification by its field personnel, it was shown that "save for a few § Cebu Pride – Cebu City
isolated places in the country, petitioner does not exist in almost all
provinces in the country."21 § Circle of Friends

This argument that "petitioner made untruthful statements in its petition § Dipolog Gay Association – Zamboanga del Norte
when it alleged its national existence" is a new one; previously, the
COMELEC claimed that petitioner was "not being truthful when it said § Gay, Bisexual, & Transgender Youth Association (GABAY)
that it or any of its nominees/party-list representatives have not violated
or failed to comply with laws, rules, or regulations relating to the
§ Gay and Lesbian Activists Network for Gender Equality
elections." Nowhere was this ground for denial of petitioner’s (GALANG) – Metro Manila
accreditation mentioned or even alluded to in the Assailed Resolutions.
This, in itself, is quite curious, considering that the reports of petitioner’s
alleged non-existence were already available to the COMELEC prior to § Gay Men’s Support Group (GMSG) – Metro Manila
the issuance of the First Assailed Resolution. At best, this is irregular
procedure; at worst, a belated afterthought, a change in respondent’s § Gay United for Peace and Solidarity (GUPS) – Lanao del Norte
theory, and a serious violation of petitioner’s right to procedural due
process. § Iloilo City Gay Association – Iloilo City

Nonetheless, we find that there has been no misrepresentation. A § Kabulig Writer’s Group – Camarines Sur
cursory perusal of Ang Ladlad’s initial petition shows that it never claimed
to exist in each province of the Philippines. Rather, petitioner alleged that § Lesbian Advocates Philippines, Inc. (LEAP)
the LGBT community in the Philippines was estimated to constitute at
least 670,000 persons; that it had 16,100 affiliates and members around § LUMINA – Baguio City
the country, and 4,044 members in its electronic discussion group.22 Ang
Ladlad also represented itself to be "a national LGBT umbrella § Marikina Gay Association – Metro Manila
organization with affiliates around the Philippines composed of the
following LGBT networks:"
§ Metropolitan Community Church (MCC) – Metro Manila
§ Naga City Gay Association – Naga City guidelines in Ang Bagong Bayani. The difference, COMELEC claims, lies
in Ang Ladlad’s morality, or lack thereof.
§ ONE BACARDI
Religion as the Basis for Refusal to Accept Ang Ladlad’s Petition for
§ Order of St. Aelred (OSAe) – Metro Manila Registration

§ PUP LAKAN Our Constitution provides in Article III, Section 5 that "[n]o law shall be
made respecting an establishment of religion, or prohibiting the free
§ RADAR PRIDEWEAR exercise thereof." At bottom, what our non-establishment clause calls for
is "government neutrality in religious matters."24 Clearly, "governmental
reliance on religious justification is inconsistent with this policy of
§ Rainbow Rights Project (R-Rights), Inc. – Metro Manila
neutrality."25 We thus find that it was grave violation of the non-
establishment clause for the COMELEC to utilize the Bible and the Koran
§ San Jose del Monte Gay Association – Bulacan to justify the exclusion of Ang Ladlad.

§ Sining Kayumanggi Royal Family – Rizal Rather than relying on religious belief, the legitimacy of the Assailed
Resolutions should depend, instead, on whether the COMELEC is able to
§ Society of Transexual Women of the Philippines (STRAP) – advance some justification for its rulings beyond mere conformity to
Metro Manila religious doctrine. Otherwise stated, government must act for secular
purposes and in ways that have primarily secular effects. As we held in
§ Soul Jive – Antipolo, Rizal Estrada v. Escritor:26

§ The Link – Davao City x x x The morality referred to in the law is public and necessarily secular,
not religious as the dissent of Mr. Justice Carpio holds. "Religious
§ Tayabas Gay Association – Quezon teachings as expressed in public debate may influence the civil public
order but public moral disputes may be resolved only on grounds
§ Women’s Bisexual Network – Metro Manila articulable in secular terms." Otherwise, if government relies upon
religious beliefs in formulating public policies and morals, the resulting
§ Zamboanga Gay Association – Zamboanga City23 policies and morals would require conformity to what some might regard
as religious programs or agenda. The non-believers would therefore be
compelled to conform to a standard of conduct buttressed by a religious
Since the COMELEC only searched for the names ANG LADLAD LGBT belief, i.e., to a "compelled religion," anathema to religious freedom.
or LADLAD LGBT, it is no surprise that they found that petitioner had no Likewise, if government based its actions upon religious beliefs, it would
presence in any of these regions. In fact, if COMELEC’s findings are to tacitly approve or endorse that belief and thereby also tacitly disapprove
be believed, petitioner does not even exist in Quezon City, which is contrary religious or non-religious views that would not support the policy.
registered as Ang Ladlad’s principal place of business. As a result, government will not provide full religious freedom for all its
citizens, or even make it appear that those whose beliefs are
Against this backdrop, we find that Ang Ladlad has sufficiently disapproved are second-class citizens. 1avvphi1

demonstrated its compliance with the legal requirements for


accreditation. Indeed, aside from COMELEC’s moral objection and the In other words, government action, including its proscription of immorality
belated allegation of non-existence, nowhere in the records has the as expressed in criminal law like concubinage, must have a secular
respondent ever found/ruled that Ang Ladlad is not qualified to register as purpose. That is, the government proscribes this conduct because it is
a party-list organization under any of the requisites under RA 7941 or the "detrimental (or dangerous) to those conditions upon which depend the
existence and progress of human society" and not because the conduct not seen fit to criminalize homosexual conduct. Evidently, therefore,
is proscribed by the beliefs of one religion or the other. Although these "generally accepted public morals" have not been convincingly
admittedly, moral judgments based on religion might have a compelling transplanted into the realm of law.29
influence on those engaged in public deliberations over what actions
would be considered a moral disapprobation punishable by law. After all, The Assailed Resolutions have not identified any specific overt immoral
they might also be adherents of a religion and thus have religious act performed by Ang Ladlad. Even the OSG agrees that "there should
opinions and moral codes with a compelling influence on them; the have been a finding by the COMELEC that the group’s members have
human mind endeavors to regulate the temporal and spiritual institutions committed or are committing immoral acts."30 The OSG argues:
of society in a uniform manner, harmonizing earth with heaven.
Succinctly put, a law could be religious or Kantian or Aquinian or x x x A person may be sexually attracted to a person of the same gender,
utilitarian in its deepest roots, but it must have an articulable and of a different gender, or more than one gender, but mere attraction does
discernible secular purpose and justification to pass scrutiny of the not translate to immoral acts. There is a great divide between thought
religion clauses. x x x Recognizing the religious nature of the Filipinos and action. Reduction ad absurdum. If immoral thoughts could be
and the elevating influence of religion in society, however, the Philippine penalized, COMELEC would have its hands full of disqualification cases
constitution's religion clauses prescribe not a strict but a benevolent against both the "straights" and the gays." Certainly this is not the
neutrality. Benevolent neutrality recognizes that government must pursue intendment of the law.31
its secular goals and interests but at the same time strive to uphold
religious liberty to the greatest extent possible within flexible
Respondent has failed to explain what societal ills are sought to be
constitutional limits. Thus, although the morality contemplated by laws is
prevented, or why special protection is required for the youth. Neither has
secular, benevolent neutrality could allow for accommodation of morality
the COMELEC condescended to justify its position that petitioner’s
based on religion, provided it does not offend compelling state interests.27
admission into the party-list system would be so harmful as to irreparably
damage the moral fabric of society. We, of course, do not suggest that
Public Morals as a Ground to Deny Ang Ladlad’s Petition for Registration the state is wholly without authority to regulate matters concerning
morality, sexuality, and sexual relations, and we recognize that the
Respondent suggests that although the moral condemnation of government will and should continue to restrict behavior considered
homosexuality and homosexual conduct may be religion-based, it has detrimental to society. Nonetheless, we cannot countenance advocates
long been transplanted into generally accepted public morals. The who, undoubtedly with the loftiest of intentions, situate morality on one
COMELEC argues: end of an argument or another, without bothering to go through the rigors
of legal reasoning and explanation. In this, the notion of morality is
Petitioner’s accreditation was denied not necessarily because their group robbed of all value. Clearly then, the bare invocation of morality will not
consists of LGBTs but because of the danger it poses to the people remove an issue from our scrutiny.
especially the youth. Once it is recognized by the government, a sector
which believes that there is nothing wrong in having sexual relations with We also find the COMELEC’s reference to purported violations of our
individuals of the same gender is a bad example. It will bring down the penal and civil laws flimsy, at best; disingenuous, at worst. Article 694 of
standard of morals we cherish in our civilized society. Any society without the Civil Code defines a nuisance as "any act, omission, establishment,
a set of moral precepts is in danger of losing its own existence.28 condition of property, or anything else which shocks, defies, or disregards
decency or morality," the remedies for which are a prosecution under the
We are not blind to the fact that, through the years, homosexual conduct, Revised Penal Code or any local ordinance, a civil action, or abatement
and perhaps homosexuals themselves, have borne the brunt of societal without judicial proceedings.32 A violation of Article 201 of the Revised
disapproval. It is not difficult to imagine the reasons behind this censure – Penal Code, on the other hand, requires proof beyond reasonable doubt
religious beliefs, convictions about the preservation of marriage, family, to support a criminal conviction. It hardly needs to be emphasized that
and procreation, even dislike or distrust of homosexuals themselves and mere allegation of violation of laws is not proof, and a mere blanket
their perceived lifestyle. Nonetheless, we recall that the Philippines has
invocation of public morals cannot replace the institution of civil or a legitimate state interest that is sufficient to satisfy rational basis review
criminal proceedings and a judicial determination of liability or culpability. under the equal protection clause. The COMELEC’s differentiation, and
its unsubstantiated claim that Ang Ladlad cannot contribute to the
As such, we hold that moral disapproval, without more, is not a sufficient formulation of legislation that would benefit the nation, furthers no
governmental interest to justify exclusion of homosexuals from legitimate state interest other than disapproval of or dislike for a
participation in the party-list system. The denial of Ang disfavored group.
Ladlad’s registration on purely moral grounds amounts more to a
statement of dislike and disapproval of homosexuals, rather than a tool to From the standpoint of the political process, the lesbian, gay, bisexual,
further any substantial public interest. Respondent’s blanket justifications and transgender have the same interest in participating in the party-list
give rise to the inevitable conclusion that the COMELEC targets system on the same basis as other political parties similarly situated.
homosexuals themselves as a class, not because of any particular State intrusion in this case is equally burdensome. Hence, laws of
morally reprehensible act. It is this selective targeting that implicates our general application should apply with equal force to LGBTs, and they
equal protection clause. deserve to participate in the party-list system on the same basis as other
marginalized and under-represented sectors.
Equal Protection
It bears stressing that our finding that COMELEC’s act of differentiating
Despite the absolutism of Article III, Section 1 of our Constitution, which LGBTs from heterosexuals insofar as the party-list system is concerned
provides "nor shall any person be denied equal protection of the laws," does not imply that any other law distinguishing between heterosexuals
courts have never interpreted the provision as an absolute prohibition on and homosexuals under different circumstances would similarly fail. We
classification. "Equality," said Aristotle, "consists in the same treatment of disagree with the OSG’s position that homosexuals are a class in
similar persons."33 The equal protection clause guarantees that no person themselves for the purposes of the equal protection clause.38 We are not
or class of persons shall be deprived of the same protection of laws prepared to single out homosexuals as a separate class meriting special
which is enjoyed by other persons or other classes in the same place and or differentiated treatment. We have not received sufficient evidence to
in like circumstances.34 this effect, and it is simply unnecessary to make such a ruling today.
Petitioner itself has merely demanded that it be recognized under the
Recent jurisprudence has affirmed that if a law neither burdens a same basis as all other groups similarly situated, and that the COMELEC
fundamental right nor targets a suspect class, we will uphold the made "an unwarranted and impermissible classification not justified by
classification as long as it bears a rational relationship to some legitimate the circumstances of the case."
government end.35 In Central Bank Employees Association, Inc. v. Banko
Sentral ng Pilipinas,36 we declared that "[i]n our jurisdiction, the standard Freedom of Expression and Association
of analysis of equal protection challenges x x x have followed the ‘rational
basis’ test, coupled with a deferential attitude to legislative classifications Under our system of laws, every group has the right to promote its
and a reluctance to invalidate a law unless there is a showing of a clear agenda and attempt to persuade society of the validity of its position
and unequivocal breach of the Constitution."37 through normal democratic means.39 It is in the public square that deeply
held convictions and differing opinions should be distilled and deliberated
The COMELEC posits that the majority of the Philippine population upon. As we held in Estrada v. Escritor:40
considers homosexual conduct as immoral and unacceptable, and this
constitutes sufficient reason to disqualify the petitioner. Unfortunately for In a democracy, this common agreement on political and moral ideas is
the respondent, the Philippine electorate has expressed no such belief. distilled in the public square. Where citizens are free, every opinion,
No law exists to criminalize homosexual behavior or expressions or every prejudice, every aspiration, and every moral discernment has
parties about homosexual behavior. Indeed, even if we were to assume access to the public square where people deliberate the order of their life
that public opinion is as the COMELEC describes it, the asserted state together. Citizens are the bearers of opinion, including opinion shaped
interest here – that is, moral disapproval of an unpopular minority – is not by, or espousing religious belief, and these citizens have equal access to
the public square. In this representative democracy, the state is rights to expressive conduct. In order to justify the prohibition of a
prohibited from determining which convictions and moral judgments may particular expression of opinion, public institutions must show that their
be proposed for public deliberation. Through a constitutionally designed actions were caused by "something more than a mere desire to avoid the
process, the people deliberate and decide. Majority rule is a necessary discomfort and unpleasantness that always accompany an unpopular
principle in this democratic governance. Thus, when public deliberation viewpoint."43
on moral judgments is finally crystallized into law, the laws will largely
reflect the beliefs and preferences of the majority, i.e., the mainstream or With respect to freedom of association for the advancement of ideas and
median groups. Nevertheless, in the very act of adopting and accepting a beliefs, in Europe, with its vibrant human rights tradition, the European
constitution and the limits it specifies – including protection of religious Court of Human Rights (ECHR) has repeatedly stated that a political
freedom "not only for a minority, however small – not only for a majority, party may campaign for a change in the law or the constitutional
however large – but for each of us" – the majority imposes upon itself a structures of a state if it uses legal and democratic means and the
self-denying ordinance. It promises not to do what it otherwise could do: changes it proposes are consistent with democratic principles. The ECHR
to ride roughshod over the dissenting minorities. has emphasized that political ideas that challenge the existing order and
whose realization is advocated by peaceful means must be afforded a
Freedom of expression constitutes one of the essential foundations of a proper opportunity of expression through the exercise of the right of
democratic society, and this freedom applies not only to those that are association, even if such ideas may seem shocking or unacceptable to
favorably received but also to those that offend, shock, or disturb. Any the authorities or the majority of the population.44 A political group should
restriction imposed in this sphere must be proportionate to the legitimate not be hindered solely because it seeks to publicly debate controversial
aim pursued. Absent any compelling state interest, it is not for the political issues in order to find solutions capable of satisfying everyone
COMELEC or this Court to impose its views on the populace. Otherwise concerned.45 Only if a political party incites violence or puts forward
stated, the COMELEC is certainly not free to interfere with speech for no policies that are incompatible with democracy does it fall outside the
better reason than promoting an approved message or discouraging a protection of the freedom of association guarantee.46
disfavored one.
We do not doubt that a number of our citizens may believe that
This position gains even more force if one considers that homosexual homosexual conduct is distasteful, offensive, or even defiant. They are
conduct is not illegal in this country. It follows that both expressions entitled to hold and express that view. On the other hand, LGBTs and
concerning one’s homosexuality and the activity of forming a political their supporters, in all likelihood, believe with equal fervor that
association that supports LGBT individuals are protected as well. relationships between individuals of the same sex are morally equivalent
to heterosexual relationships. They, too, are entitled to hold and express
Other jurisdictions have gone so far as to categorically rule that even that view. However, as far as this Court is concerned, our democracy
overwhelming public perception that homosexual conduct violates public precludes using the religious or moral views of one part of the community
morality does not justify criminalizing same-sex conduct.41 European and to exclude from consideration the values of other members of the
United Nations judicial decisions have ruled in favor of gay rights community.
claimants on both privacy and equality grounds, citing general privacy
and equal protection provisions in foreign and international texts.42 To the Of course, none of this suggests the impending arrival of a golden age for
extent that there is much to learn from other jurisdictions that have gay rights litigants. It well may be that this Decision will only serve to
reflected on the issues we face here, such jurisprudence is certainly highlight the discrepancy between the rigid constitutional analysis of this
illuminating. These foreign authorities, while not formally binding on Court and the more complex moral sentiments of Filipinos. We do not
Philippine courts, may nevertheless have persuasive influence on the suggest that public opinion, even at its most liberal, reflect a clear-cut
Court’s analysis. strong consensus favorable to gay rights claims and we neither attempt
nor expect to affect individual perceptions of homosexuality through this
In the area of freedom of expression, for instance, United States courts Decision.
have ruled that existing free speech doctrines protect gay and lesbian
The OSG argues that since there has been neither prior restraint nor dynamically in its attempt to bring about a more just and humane world
subsequent punishment imposed on Ang Ladlad, and its members have order. For individuals and groups struggling with inadequate structural
not been deprived of their right to voluntarily associate, then there has and governmental support, international human rights norms are
been no restriction on their freedom of expression or association. The particularly significant, and should be effectively enforced in domestic
OSG argues that: legal systems so that such norms may become actual, rather than ideal,
standards of conduct.
There was no utterance restricted, no publication censored, or any
assembly denied. [COMELEC] simply exercised its authority to review Our Decision today is fully in accord with our international obligations to
and verify the qualifications of petitioner as a sectoral party applying to protect and promote human rights. In particular, we explicitly recognize
participate in the party-list system. This lawful exercise of duty cannot be the principle of non-discrimination as it relates to the right to electoral
said to be a transgression of Section 4, Article III of the Constitution. participation, enunciated in the UDHR and the ICCPR.

xxxx The principle of non-discrimination is laid out in Article 26 of the ICCPR,


as follows:
A denial of the petition for registration x x x does not deprive the
members of the petitioner to freely take part in the conduct of elections. Article 26
Their right to vote will not be hampered by said denial. In fact, the right to
vote is a constitutionally-guaranteed right which cannot be limited. All persons are equal before the law and are entitled without any
discrimination to the equal protection of the law. In this respect, the law
As to its right to be elected in a genuine periodic election, petitioner shall prohibit any discrimination and guarantee to all persons equal and
contends that the denial of Ang Ladlad’s petition has the clear and effective protection against discrimination on any ground such as race,
immediate effect of limiting, if not outrightly nullifying the capacity of its colour, sex, language, religion, political or other opinion, national or social
members to fully and equally participate in public life through origin, property, birth or other status.
engagement in the party list elections.
In this context, the principle of non-discrimination requires that laws of
This argument is puerile. The holding of a public office is not a right but a general application relating to elections be applied equally to all persons,
privilege subject to limitations imposed by law. x x x47 regardless of sexual orientation. Although sexual orientation is not
specifically enumerated as a status or ratio for discrimination in Article 26
The OSG fails to recall that petitioner has, in fact, established its of the ICCPR, the ICCPR Human Rights Committee has opined that the
qualifications to participate in the party-list system, and – as advanced by reference to "sex" in Article 26 should be construed to include "sexual
the OSG itself – the moral objection offered by the COMELEC was not a orientation."48Additionally, a variety of United Nations bodies have
limitation imposed by law. To the extent, therefore, that the petitioner has declared discrimination on the basis of sexual orientation to be prohibited
been precluded, because of COMELEC’s action, from publicly expressing under various international agreements.49
its views as a political party and participating on an equal basis in the
political process with other equally-qualified party-list candidates, we find The UDHR provides:
that there has, indeed, been a transgression of petitioner’s fundamental
rights. Article 21.

Non-Discrimination and International Law (1) Everyone has the right to take part in the government of his country,
directly or through freely chosen representatives.
In an age that has seen international law evolve geometrically in scope
and promise, international human rights law, in particular, has grown Likewise, the ICCPR states:
Article 25 legislative provisions which exclude any group or category of persons
from elective office.50
Every citizen shall have the right and the opportunity, without any of the
distinctions mentioned in article 2 and without unreasonable restrictions: We stress, however, that although this Court stands willing to assume the
responsibility of giving effect to the Philippines’ international law
(a) To take part in the conduct of public affairs, directly or through obligations, the blanket invocation of international law is not the panacea
freely chosen representatives; for all social ills. We refer now to the petitioner’s invocation of the
Yogyakarta Principles (the Application of International Human Rights Law
(b) To vote and to be elected at genuine periodic elections which In Relation to Sexual Orientation and Gender Identity),51 which petitioner
shall be by universal and equal suffrage and shall be held by declares to reflect binding principles of international law.
secret ballot, guaranteeing the free expression of the will of the
electors; At this time, we are not prepared to declare that these Yogyakarta
Principles contain norms that are obligatory on the Philippines. There are
(c) To have access, on general terms of equality, to public service declarations and obligations outlined in said Principles which are not
in his country. reflective of the current state of international law, and do not find basis in
any of the sources of international law enumerated under Article 38(1) of
the Statute of the International Court of Justice.52 Petitioner has not
As stated by the CHR in its Comment-in-Intervention, the scope of the
undertaken any objective and rigorous analysis of these alleged
right to electoral participation is elaborated by the Human Rights
principles of international law to ascertain their true status.
Committee in its General Comment No. 25 (Participation in Public Affairs
and the Right to Vote) as follows:
We also hasten to add that not everything that society – or a certain
segment of society – wants or demands is automatically a human right.
1. Article 25 of the Covenant recognizes and protects the right of every
This is not an arbitrary human intervention that may be added to or
citizen to take part in the conduct of public affairs, the right to vote and to
subtracted from at will. It is unfortunate that much of what passes for
be elected and the right to have access to public service. Whatever form
human rights today is a much broader context of needs that identifies
of constitution or government is in force, the Covenant requires States to
many social desires as rights in order to further claims that international
adopt such legislative and other measures as may be necessary to
law obliges states to sanction these innovations. This has the effect of
ensure that citizens have an effective opportunity to enjoy the rights it
diluting real human rights, and is a result of the notion that if "wants" are
protects. Article 25 lies at the core of democratic government based on
couched in "rights" language, then they are no longer controversial.
the consent of the people and in conformity with the principles of the
1avvphi1

Covenant.
Using even the most liberal of lenses, these Yogyakarta Principles,
consisting of a declaration formulated by various international law
xxxx
professors, are – at best – de lege ferenda – and do not constitute
binding obligations on the Philippines. Indeed, so much of contemporary
15. The effective implementation of the right and the opportunity to stand international law is characterized by the "soft law" nomenclature, i.e.,
for elective office ensures that persons entitled to vote have a free choice international law is full of principles that promote international
of candidates. Any restrictions on the right to stand for election, such as cooperation, harmony, and respect for human rights, most of which
minimum age, must be justifiable on objective and reasonable criteria. amount to no more than well-meaning desires, without the support of
Persons who are otherwise eligible to stand for election should not be either State practice or opinio juris.53
excluded by unreasonable or discriminatory requirements such as
education, residence or descent, or by reason of political affiliation. No
As a final note, we cannot help but observe that the social issues
person should suffer discrimination or disadvantage of any kind because
presented by this case are emotionally charged, societal attitudes are in
of that person's candidacy. States parties should indicate and explain the
flux, even the psychiatric and religious communities are divided in
opinion. This Court’s role is not to impose its own view of acceptable
behavior. Rather, it is to apply the Constitution and laws as best as it can,
uninfluenced by public opinion, and confident in the knowledge that our
democracy is resilient enough to withstand vigorous debate.

WHEREFORE, the Petition is hereby GRANTED. The Resolutions of the


Commission on Elections dated November 11, 2009 and December 16,
2009 in SPP No. 09-228 (PL) are hereby SET ASIDE. The Commission
on Elections is directed to GRANT petitioner’s application for party-list
accreditation.

SO ORDERED.
G.R. No. 80391 February 28, 1989 petitioner in his capacity as Speaker of the Assembly,
Region XII, in a letter which reads:
SULTAN ALIMBUSAR P. LIMBONA, petitioner,
vs. The Committee on Muslim Affairs well
CONTE MANGELIN, SALIC ALI, SALINDATO ALI, PILIMPINAS undertake consultations and dialogues
CONDING, ACMAD TOMAWIS, GERRY TOMAWIS, JESUS ORTIZ, with local government officials, civic,
ANTONIO DELA FUENTE, DIEGO PALOMARES, JR., RAUL religious organizations and traditional
DAGALANGIT, and BIMBO SINSUAT, respondents. leaders on the recent and present political
developments and other issues affecting
Ambrosio Padilla, Mempin & Reyes Law Offices for petitioner petitioner. Regions IX and XII.

Makabangkit B. Lanto for respondents. The result of the conference, consultations


and dialogues would hopefully chart the
autonomous governments of the two
regions as envisioned and may prod the
President to constitute immediately the
SARMIENTO, J.: Regional Consultative Commission as
mandated by the Commission.
The acts of the Sangguniang Pampook of Region XII are assailed in this
petition. The antecedent facts are as follows: You are requested to invite some
members of the Pampook Assembly of
1. On September 24, 1986, petitioner Sultan Alimbusar your respective assembly on November 1
Limbona was appointed as a member of the Sangguniang to 15, 1987, with venue at the Congress of
Pampook, Regional Autonomous Government, Region the Philippines. Your presence, unstinted
XII, representing Lanao del Sur. support and cooperation is (sic)
indispensable.
2. On March 12, 1987 petitioner was elected Speaker of
the Regional Legislative Assembly or Batasang Pampook 5. Consistent with the said invitation, petitioner sent a
of Central Mindanao (Assembly for brevity). telegram to Acting Secretary Johnny Alimbuyao of the
Assembly to wire all Assemblymen that there shall be no
3. Said Assembly is composed of eighteen (18) members. session in November as "our presence in the house
Two of said members, respondents Acmad Tomawis and committee hearing of Congress take (sic) precedence
Pakil Dagalangit, filed on March 23, 1987 with the over any pending business in batasang pampook ... ."
Commission on Elections their respective certificates of
candidacy in the May 11, 1987 congressional elections for 6. In compliance with the aforesaid instruction of the
the district of Lanao del Sur but they later withdrew from petitioner, Acting Secretary Alimbuyao sent to the
the aforesaid election and thereafter resumed again their members of the Assembly the following telegram:
positions as members of the Assembly.
TRANSMITTING FOR YOUR
4. On October 21, 1987 Congressman Datu Guimid INFORMATION AND GUIDANCE
Matalam, Chairman of the Committee on Muslim Affairs of TELEGRAM RECEIVED FROM
the House of Representatives, invited Mr. Xavier Razul, SPEAKER LIMBONA QUOTE
Pampook Speaker of Region XI, Zamboanga City and the CONGRESSMAN JIMMY MATALAM
CHAIRMAN OF THE HOUSE Assemblymen in attendance voted in the affirmative,
COMMITTEE ON MUSLIM AFFAIRS hence, the chair declared said seat of the Speaker
REQUESTED ME TO ASSIST SAID vacant. 8. On November 5, 1987, the session of the
COMMITTEE IN THE DISCUSSION OF Assembly resumed with the following Assemblymen
THE PROPOSED AUTONOMY present:
ORGANIC NOV. 1ST TO 15. HENCE
WERE ALL ASSEMBLYMEN THAT 1. Mangelen Conte-Presiding Officer
THERE SHALL BE NO SESSION IN
NOVEMBER AS OUR PRESENCE IN 2. Ali Salic
THE HOUSE COMMITTEE HEARING OF
CONGRESS TAKE PRECEDENCE
3. Ali Salindatu
OVER ANY PENDING BUSINESS IN
BATASANG PAMPOOK OF MATALAM
FOLLOWS UNQUOTE REGARDS. 4. Aratuc, Malik

7. On November 2, 1987, the Assembly held session in 5. Cajelo, Rene


defiance of petitioner's advice, with the following
assemblymen present: 6. Conding, Pilipinas (sic)

1. Sali, Salic 7. Dagalangit, Rakil

2. Conding, Pilipinas (sic) 8. Dela Fuente, Antonio

3. Dagalangit, Rakil 9. Ortiz, Jesus

4. Dela Fuente, Antonio 10 Palomares, Diego

5. Mangelen, Conte 11. Quijano, Jesus

6. Ortiz, Jesus 12. Sinsuat, Bimbo

7. Palomares, Diego 13. Tomawis, Acmad

8. Sinsuat, Bimbo 14. Tomawis, Jerry

9. Tomawis, Acmad An excerpt from the debates and proceeding of said


session reads:
10. Tomawis, Jerry
HON. DAGALANGIT: Mr. Speaker, Honorable Members
After declaring the presence of a quorum, the Speaker of the House, with the presence of our colleagues who
Pro-Tempore was authorized to preside in the session. have come to attend the session today, I move to call the
On Motion to declare the seat of the Speaker vacant, all names of the new comers in order for them to cast their
votes on the previous motion to declare the position of the Pending further proceedings, this Court, on January 19, 1988, received a
Speaker vacant. But before doing so, I move also that the resolution filed by the Sangguniang Pampook, "EXPECTING
designation of the Speaker Pro Tempore as the Presiding ALIMBUSAR P. LIMBONA FROM MEMBERSHIP OF THE
Officer and Mr. Johnny Evangelists as Acting Secretary in SANGGUNIANG PAMPOOK AUTONOMOUS REGION XII," 3 on the
the session last November 2, 1987 be reconfirmed in grounds, among other things, that the petitioner "had caused to be
today's session. prepared and signed by him paying [sic] the salaries and emoluments of
Odin Abdula, who was considered resigned after filing his Certificate of
HON. SALIC ALI: I second the motions. Candidacy for Congressmen for the First District of Maguindanao in the
last May 11, elections. . . and nothing in the record of the Assembly will
PRESIDING OFFICER: Any comment or objections on show that any request for reinstatement by Abdula was ever made . .
the two motions presented? Me chair hears none and the ." 4 and that "such action of Mr. Lim bona in paying Abdula his salaries
said motions are approved. ... and emoluments without authority from the Assembly . . . constituted a
usurpation of the power of the Assembly," 5 that the petitioner "had
recently caused withdrawal of so much amount of cash from the
Twelve (12) members voted in favor of the motion to
Assembly resulting to the non-payment of the salaries and emoluments
declare the seat of the Speaker vacant; one abstained
of some Assembly [sic]," 6 and that he had "filed a case before the
and none voted against. 1
Supreme Court against some members of the Assembly on question
which should have been resolved within the confines of the
Accordingly, the petitioner prays for judgment as follows: Assembly," 7 for which the respondents now submit that the petition had
become "moot and academic". 8
WHEREFORE, petitioner respectfully prays that-
The first question, evidently, is whether or not the expulsion of the
(a) This Petition be given due course; petitioner (pending litigation) has made the case moot and academic.

(b) Pending hearing, a restraining order or writ of We do not agree that the case has been rendered moot and academic by
preliminary injunction be issued enjoining respondents reason simply of the expulsion resolution so issued. For, if the petitioner's
from proceeding with their session to be held on expulsion was done purposely to make this petition moot and academic,
November 5, 1987, and on any day thereafter; and to preempt the Court, it will not make it academic.

(c) After hearing, judgment be rendered declaring the On the ground of the immutable principle of due process alone, we hold
proceedings held by respondents of their session on that the expulsion in question is of no force and effect. In the first place,
November 2, 1987 as null and void; there is no showing that the Sanggunian had conducted an investigation,
and whether or not the petitioner had been heard in his defense,
(d) Holding the election of petitioner as Speaker of said assuming that there was an investigation, or otherwise given the
Legislative Assembly or Batasan Pampook, Region XII opportunity to do so. On the other hand, what appears in the records is
held on March 12, 1987 valid and subsisting, and an admission by the Assembly (at least, the respondents) that "since
November, 1987 up to this writing, the petitioner has not set foot at the
(e) Making the injunction permanent. Sangguniang Pampook." 9 "To be sure, the private respondents aver that
"[t]he Assemblymen, in a conciliatory gesture, wanted him to come to
Petitioner likewise prays for such other relief as may be Cotabato City," 10 but that was "so that their differences could be threshed
just and equitable. 2 out and settled." 11Certainly, that avowed wanting or desire to thresh out
and settle, no matter how conciliatory it may be cannot be a substitute for
the notice and hearing contemplated by law.
While we have held that due process, as the term is known in its Constitution," 17 with legislative and executive machinery to exercise
administrative law, does not absolutely require notice and that a party the powers and responsibilities 18specified therein.
need only be given the opportunity to be heard, 12 it does not appear
herein that the petitioner had, to begin with, been made aware that he It requires the autonomous regional governments to "undertake all
had in fact stood charged of graft and corruption before his collegues. It internal administrative matters for the respective regions," 19 except to "act
cannot be said therefore that he was accorded any opportunity to rebut on matters which are within the jurisdiction and competence of the
their accusations. As it stands, then, the charges now levelled amount to National Government," 20 "which include, but are not limited to, the
mere accusations that cannot warrant expulsion. following:

In the second place, (the resolution) appears strongly to be a bare act of (1) National defense and security;
vendetta by the other Assemblymen against the petitioner arising from
what the former perceive to be abduracy on the part of the latter. Indeed, (2) Foreign relations;
it (the resolution) speaks of "a case [having been filed] [by the petitioner]
before the Supreme Court . . . on question which should have been
(3) Foreign trade;
resolved within the confines of the Assemblyman act which some
members claimed unnecessarily and unduly assails their integrity and
character as representative of the people" 13 an act that cannot possibly (4) Currency, monetary affairs, foreign exchange, banking
justify expulsion. Access to judicial remedies is guaranteed by the and quasi-banking, and external borrowing,
Constitution, 14 and, unless the recourse amounts to malicious
prosecution, no one may be punished for seeking redress in the courts. (5) Disposition, exploration, development, exploitation or
utilization of all natural resources;
We therefore order reinstatement, with the caution that should the past
acts of the petitioner indeed warrant his removal, the Assembly is (6) Air and sea transport
enjoined, should it still be so minded, to commence proper proceedings
therefor in line with the most elementary requirements of due process. (7) Postal matters and telecommunications;
And while it is within the discretion of the members of the Sanggunian to
punish their erring colleagues, their acts are nonetheless subject to the (8) Customs and quarantine;
moderating band of this Court in the event that such discretion is
exercised with grave abuse. (9) Immigration and deportation;

It is, to be sure, said that precisely because the Sangguniang (10) Citizenship and naturalization;
Pampook(s) are "autonomous," the courts may not rightfully intervene in
their affairs, much less strike down their acts. We come, therefore, to the
(11) National economic, social and educational planning;
second issue: Are the so-called autonomous governments of Mindanao,
and
as they are now constituted, subject to the jurisdiction of the national
courts? In other words, what is the extent of self-government given to the
two autonomous governments of Region IX and XII? (12) General auditing. 21

The autonomous governments of Mindanao were organized in Regions In relation to the central government, it provides that "[t]he President shall
IX and XII by Presidential Decree No. 1618 15 promulgated on July 25, have the power of general supervision and control over the Autonomous
1979. Among other things, the Decree established "internal Regions ..." 22
autonomy" 16 in the two regions "[w]ithin the framework of the national
sovereignty and territorial integrity of the Republic of the Philippines and
Now, autonomy is either decentralization of administration or xxx xxx xxx
decentralization of power. There is decentralization of administration
when the central government delegates administrative powers to political See. 15. Mere shall be created autonomous regions in
subdivisions in order to broaden the base of government power and in Muslim Mindanao and in the Cordilleras consisting of
the process to make local governments "more responsive and provinces, cities, municipalities, and geographical areas
accountable," 23 "and ensure their fullest development as self-reliant sharing common and distinctive historical and cultural
communities and make them more effective partners in the pursuit of heritage, economic and social structures, and other
national development and social progress." 24 At the same time, it relieves relevant characteristics within the framework of this
the central government of the burden of managing local affairs and Constitution and the national sovereignty as well as
enables it to concentrate on national concerns. The President exercises territorial integrity of the Republic of the Philippines. 31
"general supervision" 25 over them, but only to "ensure that local affairs
are administered according to law." 26 He has no control over their acts in An autonomous government that enjoys autonomy of the latter category
the sense that he can substitute their judgments with his own. 27 [CONST. (1987), art. X, sec. 15.] is subject alone to the decree of the
organic act creating it and accepted principles on the effects and limits of
Decentralization of power, on the other hand, involves an abdication of "autonomy." On the other hand, an autonomous government of the
political power in the favor of local governments units declare to be former class is, as we noted, under the supervision of the national
autonomous . In that case, the autonomous government is free to chart government acting through the President (and the Department of Local
its own destiny and shape its future with minimum intervention from Government). 32 If the Sangguniang Pampook (of Region XII), then, is
central authorities. According to a constitutional author, decentralization autonomous in the latter sense, its acts are, debatably beyond the
of power amounts to "self-immolation," since in that event, the domain of this Court in perhaps the same way that the internal acts, say,
autonomous government becomes accountable not to the central of the Congress of the Philippines are beyond our jurisdiction. But if it is
authorities but to its constituency. 28 autonomous in the former category only, it comes unarguably under our
jurisdiction. An examination of the very Presidential Decree creating the
But the question of whether or not the grant of autonomy Muslim autonomous governments of Mindanao persuades us that they were
Mindanao under the 1987 Constitution involves, truly, an effort to never meant to exercise autonomy in the second sense, that is, in which
decentralize power rather than mere administration is a question foreign the central government commits an act of self-immolation. Presidential
to this petition, since what is involved herein is a local government unit Decree No. 1618, in the first place, mandates that "[t]he President shall
constituted prior to the ratification of the present Constitution. Hence, the have the power of general supervision and control over Autonomous
Court will not resolve that controversy now, in this case, since no Regions."33 In the second place, the Sangguniang Pampook, their
controversy in fact exists. We will resolve it at the proper time and in the legislative arm, is made to discharge chiefly administrative services, thus:
proper case.
SEC. 7. Powers of the Sangguniang Pampook. The
Under the 1987 Constitution, local government units enjoy autonomy in Sangguniang Pampook shall exercise local legislative
these two senses, thus: powers over regional affairs within the framework of
national development plans, policies and goals, in the
Section 1. The territorial and political subdivisions of the following areas:
Republic of the Philippines are the provinces, cities,
municipalities, and barangays. Here shall be autonomous (1) Organization of regional administrative system;
regions in Muslim Mindanao ,and the Cordilleras as
hereinafter provided. 29 (2) Economic, social and cultural development of the
Autonomous Region;
Sec. 2. The territorial and political subdivisions shall enjoy
local autonomy. 30
(3) Agricultural, commercial and industrial programs for did so in violation of the Rules of the Sangguniang Pampook since the
the Autonomous Region; Assembly was then on recess; and (2) assuming that it was valid, his
ouster was ineffective nevertheless for lack of quorum.
(4) Infrastructure development for the Autonomous
Region; Upon the facts presented, we hold that the November 2 and 5, 1987
sessions were invalid. It is true that under Section 31 of the Region XII
(5) Urban and rural planning for the Autonomous Region; Sanggunian Rules, "[s]essions shall not be suspended or adjourned
except by direction of the Sangguniang Pampook," 35 but it provides
(6) Taxation and other revenue-raising measures as likewise that "the Speaker may, on [sic] his discretion, declare a recess of
provided for in this Decree; "short intervals." 36 Of course, there is disagreement between the
protagonists as to whether or not the recess called by the petitioner
effective November 1 through 15, 1987 is the "recess of short intervals"
(7) Maintenance, operation and administration of schools
referred to; the petitioner says that it is while the respondents insist that,
established by the Autonomous Region;
to all intents and purposes, it was an adjournment and that "recess" as
used by their Rules only refers to "a recess when arguments get heated
(8) Establishment, operation and maintenance of health, up so that protagonists in a debate can talk things out informally and
welfare and other social services, programs and facilities; obviate dissenssion [sic] and disunity. 37 The Court agrees with the
respondents on this regard, since clearly, the Rules speak of "short
(9) Preservation and development of customs, traditions, intervals." Secondly, the Court likewise agrees that the Speaker could not
languages and culture indigenous to the Autonomous have validly called a recess since the Assembly had yet to convene on
Region; and November 1, the date session opens under the same Rules. 38 Hence,
there can be no recess to speak of that could possibly interrupt any
(10) Such other matters as may be authorized by session. But while this opinion is in accord with the respondents' own, we
law,including the enactment of such measures as may be still invalidate the twin sessions in question, since at the time the
necessary for the promotion of the general welfare of the petitioner called the "recess," it was not a settled matter whether or not
people in the Autonomous Region. he could. do so. In the second place, the invitation tendered by the
Committee on Muslim Affairs of the House of Representatives provided a
The President shall exercise such powers as may be plausible reason for the intermission sought. Thirdly, assuming that a
necessary to assure that enactment and acts of the valid recess could not be called, it does not appear that the respondents
Sangguniang Pampook and the Lupong called his attention to this mistake. What appears is that instead, they
Tagapagpaganap ng Pook are in compliance with this opened the sessions themselves behind his back in an apparent act of
Decree, national legislation, policies, plans and programs. mutiny. Under the circumstances, we find equity on his side. For this
reason, we uphold the "recess" called on the ground of good faith.
The Sangguniang Pampook shall maintain liaison with the
Batasang Pambansa. 34 It does not appear to us, moreover, that the petitioner had resorted to the
aforesaid "recess" in order to forestall the Assembly from bringing about
Hence, we assume jurisdiction. And if we can make an inquiry in the his ouster. This is not apparent from the pleadings before us. We are
validity of the expulsion in question, with more reason can we review the convinced that the invitation was what precipitated it.
petitioner's removal as Speaker.
In holding that the "recess" in question is valid, we are not to be taken as
Briefly, the petitioner assails the legality of his ouster as Speaker on the establishing a precedent, since, as we said, a recess can not be validly
grounds that: (1) the Sanggunian, in convening on November 2 and 5, declared without a session having been first opened. In upholding the
1987 (for the sole purpose of declaring the office of the Speaker vacant), petitioner herein, we are not giving him a carte blanche to order recesses
in the future in violation of the Rules, or otherwise to prevent the lawful
meetings thereof.

Neither are we, by this disposition, discouraging the Sanggunian from


reorganizing itself pursuant to its lawful prerogatives. Certainly, it can do
so at the proper time. In the event that be petitioner should initiate
obstructive moves, the Court is certain that it is armed with enough
coercive remedies to thwart them. 39

In view hereof, we find no need in dwelling on the issue of quorum.

WHEREFORE, premises considered, the petition is GRANTED. The


Sangguniang Pampook, Region XII, is ENJOINED to (1) REINSTATE the
petitioner as Member, Sangguniang Pampook, Region XII; and (2)
REINSTATE him as Speaker thereof. No costs.

SO ORDERED.
On March 22, 2002, the RTC issued an Order8 setting the petition for hearing at 8:30 a.m. of
December 12 and 17, 2002 during which all persons concerned were enjoined to show cause, if
any, why the petition should not be granted. The entire petition and its annexes, including the
G.R. No. 170603 January 29, 2007 order, were ordered published once a week for three consecutive weeks in the Official Gazette
and also in a newspaper of general circulation in the City of Manila. The RTC likewise ordered
EDISON SO, Petitioner, that copies of the petition and notice be posted in public and conspicuous places in the Manila
vs. City Hall Building.9
REPUBLIC OF THE PHILIPPINES, Respondent.
Petitioner thus caused the publication of the above order, as well as the entire petition and its
DECISION annexes, in the Official Gazette on May 20, 200210 and May 27, 2002,11 and in Today, a
newspaper of general circulation in the City of Manila, on May 25, 2002 and June 1, 2002.
CALLEJO, SR., J.:
No one opposed the petition. During the hearing, petitioner presented Atty. Adasa, Jr. who
1
testified that he came to know petitioner in 1991 as the legal consultant and adviser of the So
Assailed in this Petition for Review on Certiorari is the Decision of the Court of Appeals (CA) in family’s business. He would usually attend parties and other social functions hosted by
CA-G.R. CV No. 80437 which reversed the Decision2 of the Regional Trial Court (RTC) of petitioner’s family. He knew petitioner to be obedient, hardworking, and possessed of good
Manila, Branch 8, in Naturalization Case No. 02-102984. Likewise assailed is the appellate moral character, including all the qualifications mandated by law. Atty. Adasa, Jr. further testified
court’s Resolution denying the Motion for Reconsideration of its Decision. that petitioner was gainfully employed and presently resides at No. 528 Lavezares Street,
Binondo, Manila; petitioner had been practicing Philippine tradition and those embodied in the
Antecedents Constitution; petitioner had been socially active, mingled with some of his neighbors and had
conducted himself in a proper and irreproachable manner during his entire stay in the
Philippines; and petitioner and his family observed Christmas and New Year and some
On February 28, 2002, petitioner Edison So filed before the RTC a Petition for occasions such as fiestas. According to the witness, petitioner was not disqualified under C.A.
Naturalization3 under Commonwealth Act (C.A.) No. 473, otherwise known as the Revised No. 473 to become a Filipino citizen: he is not opposed to organized government or believes in
Naturalization Law, as amended. He alleged the following in his petition: the use of force; he is not a polygamist and has not been convicted of a crime involving moral
turpitude; neither is he suffering from any mental alienation or any incurable disease. 12
He was born on February 17, 1982, in Manila; he is a Chinese citizen who has lived in No. 528
Lavezares St., Binondo, Manila, since birth; as an employee, he derives an average annual Another witness for petitioner, Mark Salcedo, testified that he has known petitioner for ten (10)
income of around P100,000.00 with free board and lodging and other benefits; he is single, able years; they first met at a birthday party in 1991. He and petitioner were classmates at the
to speak and write English, Chinese and Tagalog; he is exempt from the filing of Declaration of University of Santo Tomas (UST) where they took up Pharmacy. Petitioner was a member of
Intention to become a citizen of the Philippines pursuant to Section 6 of Commonwealth Act some school organizations and mingled well with friends.13 Salcedo further testified that he saw
(C.A.) No. 473, as amended, because he was born in the Philippines, and studied in a school petitioner twice a week, and during fiestas and special occasions when he would go to
recognized by the Government where Philippine history, government and culture are taught; he petitioner’s house. He has known petitioner to have resided in Manila since birth. Petitioner is
is a person of good moral character; he believes in the principles underlying the Philippine intelligent, a person of good moral character, and believes in the principles of the Philippine
constitution; he has conducted himself in a proper and irreproachable manner during the entire Constitution. Petitioner has a gainful occupation, has conducted himself in a proper and
period of his residence in the Philippines in his relation with the constituted government as well irreproachable manner and has all the qualifications to become a Filipino citizen.
as with the community in which he is living; he has mingled socially with the Filipinos and has
evinced a sincere desire to learn and embrace the customs, traditions and ideals of the Filipino
people; he has all the qualifications provided under Section 2 and none of the disqualifications Petitioner also testified and attempted to prove that he has all the qualifications and none of the
under Section 4 of C.A. No. 473, as amended; he is not opposed to organized government or disqualifications to become a citizen of the Philippines.
affiliated with any association or group of persons who uphold and teach doctrines opposing all
organized governments; he is not defending or teaching the necessity or propriety of violence, At the conclusion of his testimonial evidence, petitioner offered in evidence the following
personal assault or assassination for the success or predominance of men’s ideas; he is not a documents: (1) Certificate of Live Birth;14 (2) Alien Certificate of Registration;15 (3) Immigrant
polygamist or a believer in the practice of polygamy; he has not been convicted of any crime Certificate of Residence;16 (4) Elementary Pupil’s17 and High School Student’s18 Permanent
involving moral turpitude; he is not suffering from any incurable contagious diseases or from Record issued by Chang Kai Shek College; (5) Transcript of Record issued by the University of
mental alienation; the nation of which he is a citizen is not at war with the Philippines; it is his Santo Tomas;19 (6) Certification of Part-Time Employment dated November 20, 2002;20 (7)
intention in good faith to become a citizen of the Philippines and to renounce absolutely and Income Tax Returns and Certificate of Withholding Tax for the year 2001;21 (8) Certification from
forever all allegiance and fidelity to any foreign prince, potentate, state or sovereignty, and Metrobank that petitioner is a depositor;22 (9) Clearances that he has not been charged or
particularly to China; and he will reside continuously in the Philippines from the time of the filing convicted of any crime involving moral turpitude;23 and (10) Medical Certificates and Psychiatric
of the petition up to the time of his admission as citizen of the Philippines. The petition was Evaluation issued by the Philippine General Hospital.24 The RTC admitted all these in evidence.
docketed as Naturalization Case No. 02-102984.

The RTC granted the petition on June 4, 2003.25 The fallo of the decision reads:
Attached to the petition were the Joint Affidavit4 of Atty. Artemio Adasa, Jr. and Mark B. Salcedo;
and petitioner’s Certificate of Live Birth,5 Alien Certificate of Registration,6 and Immigrant
Certificate of Residence.7 WHEREFORE, judgment is hereby rendered GRANTING the petition and declaring that
petitioner EDISON SO has all the qualifications and none of the disqualifications to become a
Filipino citizen and he is hereby admitted as citizen of the Philippines, after taking the necessary In its Decision33 dated August 4, 2005, the CA set aside the ruling of the RTC and dismissed the
oath of allegiance, as soon as this decision becomes final, subject to payment of cost petition for naturalization without prejudice.34 According to the CA, petitioner’s two (2) witnesses
of P30,000.00. were not credible because they failed to mention specific details of petitioner’s life or character to
show how well they knew him; they merely "parroted" the provisions of the Naturalization Act
without clearly explaining their applicability to petitioner’s case. 35The appellate court likewise
SO ORDERED.26
ruled that petitioner failed to comply with the requirement of the law that the applicant must not
be less than 21 years of age on the day of the hearing of the petition; during the first hearing on
The trial court ruled that the witnesses for petitioner had known him for the period required by December 12, 2002, petitioner was only twenty (20) years, nine (9) months, and twenty five (25)
law, and they had affirmed that petitioner had all the qualifications and none of the days old, falling short of the requirement.36 The CA stated, however, that it was not its intention
disqualifications to become a Filipino citizen. Thus, the court concluded that petitioner had to forever close the door to any future application for naturalization which petitioner would file,
satisfactorily supported his petition with evidence. and that it believes that he would make a good Filipino citizen in due time, a decided asset to
this country.37
Respondent Republic of the Philippines, through the Office of the Solicitor General (OSG),
appealed the decision to the CA on the following grounds: Petitioner’s motion for reconsideration38 was denied in a Resolution39 dated November 24, 2005;
hence, the present petition grounded on the sole issue:
I.
WHETHER OR NOT THE HONORABLE COURT OF APPEALS COMMITTED REVERSIBLE
ERROR WHEN IT REVERSED THE DECISION OF THE REGIONAL TRIAL COURT OF
THE LOWER COURT ERRED IN GRANTING THE PETITION FOR NATURALIZATION MANILA.40
DESPITE THE FACT THAT THE TWO (2) CHARACTER WITNESSES, NAMELY: ARTEMIO
ADASA, JR. AND MARK SALCEDO WERE NOT QUALIFIED CHARACTER WITNESSES.
In support of his petition, petitioner reiterates the arguments he set forth in the Brief filed before
the CA.
II.

In its Comment41 on the petition, respondent countered that R.A. No. 9139 (which took effect on
PETITIONER IS NOT QUALIFIED TO BE ADMITTED AS CITIZEN OF THE PHILIPPINES.27 August 8, 2001 and where the applicant’s age requirement was lowered to eighteen (18) years
old), refers only to administrative naturalization filed with the Special Committee on
Respondent contended that based on the evidence on record, appellee failed to prove that he Naturalization; it does not apply to judicial naturalization before the court, as in the present
possesses all the qualifications under Section 2 and none of the disqualifications under Section case.42 Respondent, through the OSG, avers that its failure to oppose the petition before the
4 of C.A. No. 473. It insisted that his two (2) character witnesses did not know him well enough court a quo does not preclude it from appealing the decision of the RTC to the CA; it is even
to vouch for his fitness to become a Filipino citizen; they merely made general statements authorized to question an already final decision by filing a petition for cancellation of
without giving specific details about his character and moral conduct.28 The witnesses did not citizenship.43 Lastly, respondent reiterates its argument that petitioner’s character witnesses are
even reside in the same place as petitioner.29 Respondent likewise argued that petitioner himself not qualified to prove the former’s qualifications.
failed to prove that he is qualified to become a Filipino citizen because he did not give any
explanation or specific answers to the questions propounded by his lawyer. He merely answered In determining whether or not an applicant for naturalization is entitled to become a Filipino
"yes" or "no" or gave general statements in answer to his counsel’s questions. Thus, petitioner citizen, it is necessary to resolve the following issues: (1) whether or not R.A. No. 9139 applies
was unable to prove that he had all the qualifications and none of the disqualifications required to petitions for naturalization by judicial act; and (2) whether or not the witnesses presented by
by law to be a naturalized Filipino citizen.30
petitioner are "credible" in accordance with the jurisprudence and the definition and guidelines
set forth in C.A. No. 473.
On the other hand, petitioner averred that he graduated cum laude from the UST with the degree
of Bachelor of Science in Pharmacy. He is now on his second year as a medical student at the
The petition is denied for lack of merit.
UST Medicine and Surgery. He avers that the requirements for naturalization under C.A. No.
473, as amended by LOI 270, in relation to Presidential Decree Nos. 836 and 1379, had been
relaxed after the Philippine government entered into diplomatic relations with the People’s Naturalization signifies the act of formally adopting a foreigner into the political body of a nation
Republic of China; the requirements were further relaxed when Republic Act (R.A.) No. 9139 by clothing him or her with the privileges of a citizen. 44 Under current and existing laws, there are
was signed into law.31 Petitioner pointed out that the petition, with all its annexes, was published three ways by which an alien may become a citizen by naturalization: (a) administrative
in the official gazette and a newspaper of general circulation; notices were likewise sent to the naturalization pursuant to R.A. No. 9139; (b) judicial naturalization pursuant to C.A. No. 473, as
National Bureau of Investigation, Department of Justice, Department of Foreign Affairs, and the amended; and (c) legislative naturalization in the form of a law enacted by Congress bestowing
OSG. But none from these offices came forward to oppose the petition before the lower Philippine citizenship to an alien.45
court.32 Petitioner insisted that he has all the qualifications and none of the disqualifications to
become Filipino. This was clearly established by his witnesses.
Petitioner’s contention that the qualifications an applicant for naturalization should possess are
those provided for in R.A. No. 9139 and not those set forth in C.A. No. 473 is barren of merit.
In its Reply Brief, respondent alleged that R.A. No. 9139 applies to administrative naturalization The qualifications and disqualifications of an applicant for naturalization by judicial act are set
filed with the Special Committee on Naturalization. It insisted that even in the absence of any forth in Sections 246 and 447 of C.A. No. 473. On the other hand, Sections 348 and 449 of R.A. No.
opposition, a petition for naturalization may be dismissed.
9139 provide for the qualifications and disqualifications of an applicant for naturalization Petitioner’s witnesses, Atty. Adasa and Salcedo, did not testify on his specific acts; they did not
by administrative act. elaborate on his traits. Their testimonies do not convince the Court that they personally know
petitioner well and are therefore in a position to vouch for his qualifications. As correctly found by
the CA, the witnesses’ testimonies consisted mainly of general statements in answer to the
Indeed, R.A. No. 9139 was enacted as a remedial measure intended to make the process of
leading questions propounded by his counsel. What they conveniently did was to enumerate the
acquiring Philippine citizenship less tedious, less technical and more encouraging. 50 It likewise
qualifications as set forth in the law without giving specific details. The pertinent portion of Atty.
addresses the concerns of degree holders who, by reason of lack of citizenship requirement,
Adasa’s testimony follows:
cannot practice their profession, thus promoting "brain gain" for the Philippines.51 These
however, do not justify petitioner’s contention that the qualifications set forth in said law apply
even to applications for naturalization by judicial act. q Do you know the petitioner Edison So?

First. C.A. No. 473 and R.A. No. 9139 are separate and distinct laws – the former covers all a Yes, Sir.
aliens regardless of class while the latter covers native-born aliens who lived here in the
Philippines all their lives, who never saw any other country and all along thought that they were
q Will you please tell us how did you come to know him?
Filipinos; who have demonstrated love and loyalty to the Philippines and affinity to the customs
and traditions.52 To reiterate, the intention of the legislature in enacting R.A. No. 9139 was to
make the process of acquiring Philippine citizenship less tedious, less technical and more a Well I came to know him[,] the petitioner[,] when I was the legal consultant and adviser of their
encouraging which is administrative rather than judicial in nature. Thus, although the legislature family business and I used to ah (sic) me[e]t him during my visit to their place way back in 1991
believes that there is a need to liberalize the naturalization law of the Philippines, there is to 1992.
nothing from which it can be inferred that C.A. No. 473 was intended to be amended or repealed
by R.A. No. 9139. What the legislature had in mind was merely to prescribe another mode of
acquiring Philippine citizenship which may be availed of by native born aliens. The only q From that day of 1991 up to the present, is your relationship with the petitioner more or less
implication is that, a native born alien has the choice to apply for judicial or administrative contin[u]ous?
naturalization, subject to the prescribed qualifications and disqualifications.
a Yes, sir, because aside from the usual professional visit that I did to their family some social
In the instant case, petitioner applied for naturalization by judicial act, though at the time of the function was sponsored normally and I am (sic) invited and I used to attend.
filing of his petition, administrative naturalization under R.A. No. 9139 was already available.
Consequently, his application should be governed by C.A. No. 473. q During the birthday party of the petitioner, did you usually attend petitioner’s birthday?

Second. If the qualifications prescribed in R.A. No. 9139 would be made applicable even to a On several occasions I attend the birthday.
judicial naturalization, the coverage of the law would be broadened since it would then apply
even to aliens who are not native born. It must be stressed that R.A. No. 9139 applies only to
aliens who were born in the Philippines and have been residing here. q Will you please tell us where the petitioner resides at present?

Third. Applying the provisions of R.A. No. 9139 to judicial naturalization is contrary to the a At present the petitioner resides at No. 528 Lavezares Street, Binondo, Manila.
intention of the legislature to liberalize the naturalization procedure in the country. One of the
qualifications set forth in R.A. No. 9139 is that the applicant was born in the q Do you know for how long the petitioner resides in the Philippines?
Philippines and should have been residing herein since birth. Thus, one who was born here but
left the country, though resided for more than ten (10) years from the filing of the application is
also disqualified. On the other hand, if we maintain the distinct qualifications under each of the a As far as I personally known (sic) Your Honor is that since birth.
two laws, an alien who is not qualified under R.A. No. 9139 may still be naturalized under C.A.
No. 473. q During all the times that you have know[n] the petitioner, what is your impression of his
conduct?
Thus, absent a specific provision expressly amending C.A. No. 473, the law stands and the
qualifications and disqualifications set forth therein are maintained. a Well ah (sic) I have personally known him to be obedient and hard working individual and ah
(sic) he has a good moral character and he has been ah (sic) no adverse report concerning the
In any event, petitioner failed to prove that the witnesses he presented were competent to vouch character of the petitioner.
for his good moral character, and are themselves possessed of good moral character. It must be
stressed that character witnesses in naturalization proceedings stand as insurers of the q In your opinion does the petitioner has the qualifications necessary to become [a] citizen of the
applicant’s conduct and character. Thus, they ought to testify on specific facts and events Philippines?
justifying the inference that the applicant possesses all the qualifications and none of the
disqualifications provided by law.53
a Yes.

q Can you tell us why do you say so?


a I would say Your Honor that petitioner has posses (sic) all the qualifications mandated by law q Will you please inform the Honorable court under what circumstances did you come to know
and presently he is more than 21 years old and he has resided in the Philippines particularly in the petitioner?
the City of Manila contin[u]ously for more than ten (10) years and that since his birth; and that he
has good moral character and I have observed that ah (sic) he has been practicing Philippine
a I met him in a birthday party in 1991, Sir.
traditions and ah (sic) those embodied in the Philippine constitution and he has been socially
active and meddle (sic) some of his neighbors and ah (sic) I am sure he has desire to embrace
and learn the customs and ideas and traditions in the Philippine[s] and as I earlier mentioned q And from 1991 up to the present is your relationship with the petitioner more or less
that he conducted himself in proper and approachable (sic) manner during his entire residence contin[u]ous?
in our country and he has a gainful occupation.
a Yes, Sir.
q Will you please tell us what are these customs which the petitioner embraced?
q How often did you see the petitioner?
a Well I have observed that ah (sic) together with his family they used to ah observed (sic) the
usual Filipino celebration during Christmas and new year and some occasions such as fiestas.
a I see him twice a week, Sir.

q And do you know whether petitioner is not disqualified under Commonwealth Act to become
q And during this time that you met the petitioner, what did you usually do?
Filipino citizen of the Philippines (sic)?

a Ah there has been no incident or occasion which I learned that would disqualify of coming (sic) a We play some games, Sir. We play Patentero (sic).
the citizen of the Republic of the Philippines. I have noticed that ah (sic) he is qualified under
Commonwealth Act 473 as amended because he is not opposed to ah (sic) organized q Do you go to church together?
government. His family and himself does not believed (sic) in the use of force in the success of
his ideas and ah (sic) he is not a poligamist (sic) or believer in the practice of illegal and he has
not been convicted in any crime involving him in any crime (sic). and he is not suffering from any a Yes, Sir.
mental alienation or any incurable contidious (sic) disease. as provided for.
q During fiestas in your place, did the petitioner go?
q Will you please tell us why you know all these stage?
a Yes, Sir.
a Because of ah (sic) the personal attachment with his family we have continuously having ah
(sic) the usual contact with his family.54 q How about during fiestas in the place where the petitioner reside[s], did you also go during
fiestas?
It can thus be inferred that Atty. Adasa is close to petitioner’s family, but not specifically to
petitioner. Atty. Adasa’s statements refer to his observations on the family’s practices and not to a Yes, Sir.
petitioner in particular. Nothing in his testimony suggests that he was close to petitioner and
knew him well enough to vouch for his qualifications.
q During occasion in the house of the petitioner, are you invited?

Salcedo, on the other hand, testified thus:


a Yes, Sir.

q Now do you know the petitioner in this case Edison So?


q How many time[s] did you go to his (sic) residence of the petitioner?

a Yes, Sir.
a Twice a week, sir.

q Are you personally acquainted with him?


q Will you please tell us where the petitioner resides?

a Yes, Sir.
a The petitioner resides at 528 Lavezares Street, Tondo, Manila, Sir.

q How long have you known the petitioner?


q For how long does the petitioner reside in that address?

a I have known him for about ten (10) years, Sir.


a Since birth, Sir.
q During all the times that you have known the petitioner, will you please tell us your impression q Where is he studying?
of his conduct?
a In UST, Sir.
a He is a person of good moral, sir, and he believed in the principles of the Philippines (sic)
Constitution.
q Is he your classmate?

q Will you please cite one or two of these principles underlined the principles (sic) of the
a Yes, Sir.
Philippines (sic) Constitution?

q What was his course?


a Ah the Philippines is a Republican of the (sic) state, sovereignty preside (sic) over the people
and the government authority emanate from within; and the other one is the civilian government
is not supreme over the military. a Pharmacy, Sir.

q Now in your opinion does the petitioner have all the qualifications necessary to become a q So when you said he was the secretary he only works as part time secretary?
citizen of the Philippines?
a Yes, Sir.
a Yes, Sir.
q You said the petitioner meddle (sic) socially with the Filipinos?
q What are these qualifications?
a Yes, Sir.
a He is at least 21 years old, he is a person of good moral and has been residing in the
Philippines since birth.
q Will you please name at least one of those Filipinos the petitioner meddle (sic) with?

q What else?
a Samuel Falmera, Sir, Marlon Kahocom, Sir.

a He must be a Filipino and ah must practice the traditions and customs, Sir.
q Who else?

q Do you know whether the petitioner conducted himself in a proper and appraochable (sic)
a Elmer Ramos, Sir.
manner during the period of his residence in the Philippines?

a Yes, Sir. q Who else?

q Do you know if the petitioner has a gainful occupation? a Sharmaine Santos, Sir.

q You said the petitioner is of good moral character?


a Yes, Sir.

q What is the occupation of the petitioner? a Yes, Sir.

a Ah (sic) he is the secretary in a wood factory in Commonwealth, Sir. q Why do you know that?

a As a classmate I can see him I go with him and ah (sic) I can see that he has ah better
q And aside from being the secretary, what else did the petitioner do?
approached (sic) with other people and I can see that he mixed very well with friends.

a He help (sic) in the factory cargo, Sir.


q So during school days you see him everyday?

q Is the petitioner still a student?


a Yes, Sir.

a Yes, Sir.
q When there are no classes during the vacation you see the petitioner twice a week?
a Yes, Sir. q How about the other one which you mentioned?

q Does the petitioner (sic), do you think the petitioner is not disqualified to become the citizen of a Ah (sic) these are twisting, sir he represents the ah the (sic) school intercollegiate, Sir. 55
the Republic of the Philippines?
Again, Salcedo did not give specific details on petitioner’s qualifications.
a Yes, Sir, he is not disqualified, Sir.
In sum, petitioner’s witnesses clearly did not personally know him well enough; their testimonies
q Why do you say that he is not disqualified? do not satisfactorily establish that petitioner has all the qualifications and none of the
disqualifications prescribed by law.
a Because he abide [by] any law in the government, sir, ah (sic) he is not polygamus and he is
not convicted of any crime, Sir. In naturalization proceedings, it is the burden of the applicant to prove not only his own good
moral character but also the good moral character of his/her witnesses, who must be credible
persons.56 Within the purview of the naturalization law, a "credible person" is not only an
q Do you know ever the petitioner oppose to any organized government?
individual who has not been previously convicted of a crime; who is not a police character and
has no police record; who has not perjured in the past; or whose affidavit or testimony is not
a No, Sir. incredible. What must be credible is not the declaration made but the person making it. This
implies that such person must have a good standing in the community; that he is known to be
honest and upright; that he is reputed to be trustworthy and reliable; and that his word may be
q Do you know whether he believe[s] in the use of force in any such ideas? taken on its face value, as a good warranty of the applicant’s worthiness. 57

a No, Sir. The records likewise do not show that the character witnesses of petitioner are persons of good
standing in the community; that they are honest and upright, or reputed to be trustworthy and
q Do you know if the petitioner is a believer in the practice of polygamy? reliable. The most that was established was the educational attainment of the witnesses;
however, this cannot be equated with their credibility. In fine, petitioner focused on presenting
evidence tending to build his own good moral character and neglected to establish the credibility
a No, Sir. and good moral character of his witnesses.58

q Do you know whether the petitioner suffer[s] from mental alienation or incurable disease We do not agree with petitioner’s argument that respondent is precluded from questioning the
illnesses? RTC decision because of its failure to oppose the petition. A naturalization proceeding is not a
judicial adversary proceeding, and the decision rendered therein does not constitute res judicata.
a No, Sir. A certificate of naturalization may be cancelled if it is subsequently discovered that the applicant
obtained it by misleading the court upon any material fact. Law and jurisprudence even authorize
the cancellation of a certificate of naturalization upon grounds or conditions arising subsequent
q Why do you know? to the granting of the certificate.59 If the government can challenge a final grant of citizenship,
with more reason can it appeal the decision of the RTC within the reglementary period despite
a I know him personally, sir, I have been with him as my classmate, sir and ah (sic) he is a very its failure to oppose the petition before the lower court.
intelligent person, Sir.
Thus, petitioner failed to show full and complete compliance with the requirements of
q Is the petitioner a member also of any organization or association in your school? naturalization law. For this reason, we affirm the decision of the CA denying the petition for
naturalization without prejudice.
a Yes, Sir.
It must be stressed that admission to citizenship is one of the highest privileges that the Republic
of the Philippines can confer upon an alien. It is a privilege that should not be conferred except
q What organization? upon persons fully qualified for it, and upon strict compliance with the law.60

a He is a member of Wishten and a member of starget, Sir. IN LIGHT OF ALL THE FOREGOING, the petition is DENIED for lack of merit.

q What does starget means? SO ORDERED.

a Starget is an organization of Chinese community in UST, Sir.


G.R. No. 135083 May 26, 1999 Under Section 40(d) of the Local Government Code, those holding dual
citizenship are disqualified from running for any elective local position.
ERNESTO S. MERCADO, petitioner,
vs. WHEREFORE, the Commission hereby declares the respondent Eduardo
EDUARDO BARRIOS MANZANO and the COMMISSION ON ELECTIONS, respondents. Barrios Manzano DISQUALIFIED as candidate for Vice-Mayor of Makati
City.
MENDOZA, J.:
On May 8, 1998, private respondent filed a motion for reconsideration.3 The motion remained
pending even until after the election held on May 11, 1998.
Petitioner Ernesto S. Mercado and private respondent Eduardo B. Manzano were candidates for
vice mayor of the City of Makati in the May 11, 1998 elections. The other one was Gabriel V.
Daza III. The results of the election were as follows: Accordingly, pursuant to Omnibus Resolution No. 3044, dated May 10, 1998, of the COMELEC,
the board of canvassers tabulated the votes cast for vice mayor of Makati City but suspended
the proclamation of the winner.
Eduardo B. Manzano 103,853

On May 19, 1998, petitioner sought to intervene in the case for disqualification. 4 Petitioner's
Ernesto S. Mercado 100,894
motion was opposed by private respondent.

Gabriel V. Daza III 54,2751


The motion was not resolved. Instead, on August 31, 1998, the COMELEC en banc rendered its
resolution. Voting 4 to 1, with one commissioner abstaining, the COMELEC en banc reversed
The proclamation of private respondent was suspended in view of a pending petition for the ruling of its Second Division and declared private respondent qualified to run for vice mayor
disqualification filed by a certain Ernesto Mamaril who alleged that private respondent was not a of the City of Makati in the May 11, 1998 elections.5 The pertinent portions of the resolution of
citizen of the Philippines but of the United States. the COMELEC en banc read:

In its resolution, dated May 7, 1998,2 the Second Division of the COMELEC granted the petition As aforesaid, respondent Eduardo Barrios Manzano was born in San
of Mamaril and ordered the cancellation of the certificate of candidacy of private respondent on Francisco, California, U.S.A. He acquired US citizenship by operation of the
the ground that he is a dual citizen and, under §40(d) of the Local Government Code, persons United States Constitution and laws under the principle ofjus soli.
with dual citizenship are disqualified from running for any elective position. The COMELEC's
Second Division said:
He was also a natural born Filipino citizen by operation of the 1935
Philippine Constitution, as his father and mother were Filipinos at the time of
What is presented before the Commission is a petition for disqualification of his birth. At the age of six (6), his parents brought him to the Philippines
Eduardo Barrios Manzano as candidate for the office of Vice-Mayor of using an American passport as travel document. His parents also registered
Makati City in the May 11, 1998 elections. The petition is based on the him as an alien with the Philippine Bureau of Immigration. He was issued an
ground that the respondent is an American citizen based on the record of alien certificate of registration. This, however, did not result in the loss of his
the Bureau of Immigration and misrepresented himself as a natural-born Philippine citizenship, as he did not renounce Philippine citizenship and did
Filipino citizen. not take an oath of allegiance to the United States.

In his answer to the petition filed on April 27, 1998, the respondent admitted It is an undisputed fact that when respondent attained the age of majority,
that he is registered as a foreigner with the Bureau of Immigration under he registered himself as a voter, and voted in the elections of 1992, 1995
Alien Certificate of Registration No. B-31632 and alleged that he is a Filipino and 1998, which effectively renounced his US citizenship under American
citizen because he was born in 1955 of a Filipino father and a Filipino law. Under Philippine law, he no longer had U.S. citizenship.
mother. He was born in the United States, San Francisco, California,
September 14, 1955, and is considered in American citizen under US Laws.
At the time of the May 11, 1998 elections, the resolution of the Second
But notwithstanding his registration as an American citizen, he did not lose
Division, adopted on May 7, 1998, was not yet final. Respondent Manzano
his Filipino citizenship.
obtained the highest number of votes among the candidates for vice-mayor
of Makati City, garnering one hundred three thousand eight hundred fifty
Judging from the foregoing facts, it would appear that respondent Manzano three (103,853) votes over his closest rival, Ernesto S. Mercado, who
is born a Filipino and a US citizen. In other words, he holds dual citizenship. obtained one hundred thousand eight hundred ninety four (100,894) votes,
or a margin of two thousand nine hundred fifty nine (2,959) votes. Gabriel
Daza III obtained third place with fifty four thousand two hundred seventy
The question presented is whether under our laws, he is disqualified from five (54,275) votes. In applying election laws, it would be far better to err in
the position for which he filed his certificate of candidacy. Is he eligible for favor of the popular choice than be embroiled in complex legal issues
the office he seeks to be elected?
involving private international law which may well be settled before the
highest court (Cf. Frivaldo vs. Commission on Elections, 257 SCRA 727).
WHEREFORE, the Commission en banc hereby REVERSES the resolution Sec. 1. When proper and when may be permitted to intervene. — Any
of the Second Division, adopted on May 7, 1998, ordering the cancellation person allowed to initiate an action or proceeding may, before or during the
of the respondent's certificate of candidacy. trial of an action or proceeding, be permitted by the Commission, in its
discretion to intervene in such action or proceeding, if he has legal interest
in the matter in litigation, or in the success of either of the parties, or an
We declare respondent Eduardo Luis Barrios Manzano to be QUALIFIED as
interest against both, or when he is so situated as to be adversely affected
a candidate for the position of vice-mayor of Makati City in the May 11,
by such action or proceeding.
1998, elections.

xxx xxx xxx


ACCORDINGLY, the Commission directs the Makati City Board of
Canvassers, upon proper notice to the parties, to reconvene and proclaim
the respondent Eduardo Luis Barrios Manzano as the winning candidate for Sec. 3. Discretion of Commission. — In allowing or disallowing a motion for
vice-mayor of Makati City. intervention, the Commission or the Division, in the exercise of its discretion,
shall consider whether or not the intervention will unduly delay or prejudice
the adjudication of the rights of the original parties and whether or not the
Pursuant to the resolution of the COMELEC en banc, the board of canvassers, on the evening of
intervenor's rights may be fully protected in a separate action or proceeding.
August 31, 1998, proclaimed private respondent as vice mayor of the City of Makati.

Private respondent argues that petitioner has neither legal interest in the matter in
This is a petition for certiorari seeking to set aside the aforesaid resolution of the COMELEC en
litigation nor an interest to protect because he is "a defeated candidate for the vice-
banc and to declare private respondent disqualified to hold the office of vice mayor of Makati
mayoralty post of Makati City [who] cannot be proclaimed as the Vice-Mayor of Makati
City. Petitioner contends that —
City if the private respondent be ultimately disqualified by final and executory
judgment."
[T]he COMELEC en banc ERRED in holding that:
The flaw in this argument is it assumes that, at the time petitioner sought to intervene in the
A. Under Philippine law, Manzano was no longer a U.S. citizen when he: proceedings before the COMELEC, there had already been a proclamation of the results of the
election for the vice mayoralty contest for Makati City, on the basis of which petitioner came out
only second to private respondent. The fact, however, is that there had been no proclamation at
1. He renounced his U.S. citizenship when he attained that time. Certainly, petitioner had, and still has, an interest in ousting private respondent from
the age of majority when he was already 37 years old; the race at the time he sought to intervene. The rule in Labo v. COMELEC,6 reiterated in several
and,
cases,7 only applies to cases in which the election of the respondent is contested, and the
question is whether one who placed second to the disqualified candidate may be declared the
2. He renounced his U.S. citizenship when he (merely) winner. In the present case, at the time petitioner filed a "Motion for Leave to File Intervention"
registered himself as a voter and voted in the elections on May 20, 1998, there had been no proclamation of the winner, and petitioner's purpose was
of 1992, 1995 and 1998. precisely to have private respondent disqualified "from running for [an] elective local position"
under §40(d) of R.A. No. 7160. If Ernesto Mamaril (who originally instituted the disqualification
proceedings), a registered voter of Makati City, was competent to bring the action, so was
B. Manzano is qualified to run for and or hold the elective office of Vice- petitioner since the latter was a rival candidate for vice mayor of Makati City.
Mayor of the City of Makati;

Nor is petitioner's interest in the matter in litigation any less because he filed a motion for
C. At the time of the May 11, 1998 elections, the resolution of the Second intervention only on May 20, 1998, after private respondent had been shown to have garnered
Division adopted on 7 May 1998 was not yet final so that, effectively, the highest number of votes among the candidates for vice mayor. That petitioner had a right to
petitioner may not be declared the winner even assuming that Manzano is intervene at that stage of the proceedings for the disqualification against private respondent is
disqualified to run for and hold the elective office of Vice-Mayor of the City of clear from §6 of R.A. No. 6646, otherwise known as the Electoral Reform Law of 1987, which
Makati. provides:

We first consider the threshold procedural issue raised by private respondent Manzano — Any candidate who his been declared by final judgment to be disqualified
whether petitioner Mercado his personality to bring this suit considering that he was not an shall not be voted for, and the votes cast for him shall not be counted. If for
original party in the case for disqualification filed by Ernesto Mamaril nor was petitioner's motion any reason a candidate is not declared by final judgment before an election
for leave to intervene granted. to be disqualified and he is voted for and receives the winning number of
votes in such election, the Court or Commission shall continue with the trial
I. PETITIONER'S RIGHT TO BRING THIS SUIT and hearing of action, inquiry, or protest and, upon motion of the
complainant or any intervenor, may during the pendency thereof order the
suspension of the proclamation of such candidate whenever the evidence of
Private respondent cites the following provisions of Rule 8 of the Rules of Procedure of the guilt is strong.
COMELEC in support of his claim that petitioner has no right to intervene and, therefore, cannot
bring this suit to set aside the ruling denying his motion for intervention:
Under this provision, intervention may be allowed in proceedings for disqualification even after With respect to dual allegiance, Article IV, §5 of the Constitution provides: "Dual allegiance of
election if there has yet been no final judgment rendered. citizens is inimical to the national interest and shall be dealt with by law." This provision was
included in the 1987 Constitution at the instance of Commissioner Blas F. Ople who explained
its necessity as follows: 10
The failure of the COMELEC en banc to resolve petitioner's motion for intervention was
tantamount to a denial of the motion, justifying petitioner in filing the instant petition for certiorari.
As the COMELEC en banc instead decided the merits of the case, the present petition properly . . . I want to draw attention to the fact that dual allegiance is not dual
deals not only with the denial of petitioner's motion for intervention but also with the substantive citizenship. I have circulated a memorandum to the Bernas Committee
issues respecting private respondent's alleged disqualification on the ground of dual citizenship. according to which a dual allegiance — and I reiterate a dual allegiance —
is larger and more threatening than that of mere double citizenship which is
seldom intentional and, perhaps, never insidious. That is often a function of
This brings us to the next question, namely, whether private respondent Manzano possesses
the accident of mixed marriages or of birth on foreign soil. And so, I do not
dual citizenship and, if so, whether he is disqualified from being a candidate for vice mayor of
question double citizenship at all.
Makati City.

What we would like the Committee to consider is to take constitutional


II. DUAL CITIZENSHIP AS A GROUND FOR DISQUALIFICATION
cognizance of the problem of dual allegiance. For example, we all know
what happens in the triennial elections of the Federation of Filipino-Chinese
The disqualification of private respondent Manzano is being sought under §40 of the Local Chambers of Commerce which consists of about 600 chapters all over the
Government Code of 1991 (R.A. No. 7160), which declares as "disqualified from running for any country. There is a Peking ticket, as well as a Taipei ticket. Not widely
elective local position: . . . (d) Those with dual citizenship." This provision is incorporated in the known is the fact chat the Filipino-Chinese community is represented in the
Charter of the City of Makati. 8 Legislative Yuan of the Republic of China in Taiwan. And until recently,
sponsor might recall, in Mainland China in the People's Republic of China,
they have the Associated Legislative Council for overseas Chinese wherein
Invoking the maxim dura lex sed lex, petitioner, as well as the Solicitor General, who sides with
all of Southeast Asia including some European and Latin countries were
him in this case, contends that through §40(d) of the Local Government Code, Congress has represented, which was dissolved after several years because of diplomatic
"command[ed] in explicit terms the ineligibility of persons possessing dual allegiance to hold friction. At that time, the Filipino-Chinese were also represented in that
local elective office."
Overseas Council.

To begin with, dual citizenship is different from dual allegiance. The former arises when, as a When I speak of double allegiance, therefore, I speak of this unsettled kind
result of the concurrent application of the different laws of two or more states, a person is
of allegiance of Filipinos, of citizens who are already Filipinos but who, by
simultaneously considered a national by the said states.9 For instance, such a situation may their acts, may be said to be bound by a second allegiance, either to Peking
arise when a person whose parents are citizens of a state which adheres to the principle of jus or Taiwan. I also took close note of the concern expressed by some
sanguinis is born in a state which follows the doctrine of jus soli. Such a person, ipso facto and Commissioners yesterday, including Commissioner Villacorta, who were
without any voluntary act on his part, is concurrently considered a citizen of both states. concerned about the lack of guarantees of thorough assimilation, and
Considering the citizenship clause (Art. IV) of our Constitution, it is possible for the following especially Commissioner Concepcion who has always been worried about
classes of citizens of the Philippines to possess dual citizenship: minority claims on our natural resources.

(1) Those born of Filipino fathers and/or mothers in foreign countries which
Dull allegiance can actually siphon scarce national capital to Taiwan,
follow the principle of jus soli; Singapore, China or Malaysia, and this is already happening. Some of the
great commercial places in downtown Taipei are Filipino-owned, owned by
(2) Those born in the Philippines of Filipino mothers and alien fathers if by Filipino-Chinese — it is of common knowledge in Manila. It can mean a
the laws of their father's' country such children are citizens of that country; tragic capital outflow when we have to endure a capital famine which also
means economic stagnation, worsening unemployment and social unrest.
(3) Those who marry aliens if by the laws of the latter's country the former
are considered citizens, unless by their act or omission they are deemed to And so, this is exactly what we ask — that the Committee kindly consider
have renounced Philippine citizenship. incorporating a new section, probably Section 5, in the article on Citizenship
which will read as follows: DUAL ALLEGIANCE IS INIMICAL TO
CITIZENSHIP AND SHALL BE DEALT WITH ACCORDING TO LAW.
There may be other situations in which a citizen of the Philippines may, without performing any
act, be also a citizen of another state; but the above cases are clearly possible given the
constitutional provisions on citizenship. In another session of the Commission, Ople spoke on the problem of these citizens with dual
allegiance, thus: 11
Dual allegiance, on the other hand, refers to the situation in which a person simultaneously
owes, by some positive act, loyalty to two or more states. While dual citizenship is involuntary, . . . A significant number of Commissioners expressed their concern about
dual allegiance is the result of an individual's volition. dual citizenship in the sense that it implies a double allegiance under a
double sovereignty which some of us who spoke then in a freewheeling
debate thought would be repugnant to the sovereignty which pervades the SENATOR PIMENTEL. To my mind, Mr. President, it only means that at the
Constitution and to citizenship itself which implies a uniqueness and which moment when he would want to run for public office, he has to repudiate
elsewhere in the Constitution is defined in terms of rights and obligations one of his citizenships.
exclusive to that citizenship including, of course, the obligation to rise to the
defense of the State when it is threatened, and back of this, Commissioner
SENATOR ENRILE. Suppose he carries only a Philippine passport but the
Bernas, is, of course, the concern for national security. In the course of
country of origin or the country of the father claims that person,
those debates, I think some noted the fact that as a result of the wave of
nevertheless, as a citizen? No one can renounce. There are such countries
naturalizations since the decision to establish diplomatic relations with the
in the world.
People's Republic of China was made in 1975, a good number of these
naturalized Filipinos still routinely go to Taipei every October 10; and it is
asserted that some of them do renew their oath of allegiance to a foreign SENATOR PIMENTEL. Well, the very fact that he is running for public office
government maybe just to enter into the spirit of the occasion when the would, in effect, be an election for him of his desire to be considered as a
anniversary of the Sun Yat-Sen Republic is commemorated. And so, I have Filipino citizen.
detected a genuine and deep concern about double citizenship, with its
attendant risk of double allegiance which is repugnant to our sovereignty
and national security. I appreciate what the Committee said that this could SENATOR ENRILE. But, precisely, Mr. President, the Constitution does not
require an election. Under the Constitution, a person whose mother is a
be left to the determination of a future legislature. But considering the scale
of the problem, the real impact on the security of this country, arising from, citizen of the Philippines is, at birth, a citizen without any overt act to claim
let us say, potentially great numbers of double citizens professing double the citizenship.
allegiance, will the Committee entertain a proposed amendment at the
proper time that will prohibit, in effect, or regulate double citizenship? SENATOR PIMENTEL. Yes. What we are saying, Mr. President, is: Under
the Gentleman's example, if he does not renounce his other citizenship,
then he is opening himself to question. So, if he is really interested to run,
Clearly, in including §5 in Article IV on citizenship, the concern of the Constitutional Commission
was not with dual citizens per se but with naturalized citizens who maintain their allegiance to the first thing he should do is to say in the Certificate of Candidacy that: "I
their countries of origin even after their naturalization. Hence, the phrase "dual citizenship" in am a Filipino citizen, and I have only one citizenship."
R.A. No. 7160, §40(d) and in R.A. No. 7854, §20 must be understood as referring to "dual
allegiance." Consequently, persons with mere dual citizenship do not fall under this SENATOR ENRILE. But we are talking from the viewpoint of Philippine law,
disqualification. Unlike those with dual allegiance, who must, therefore, be subject to strict Mr. President. He will always have one citizenship, and that is the
process with respect to the termination of their status, for candidates with dual citizenship, it citizenship invested upon him or her in the Constitution of the Republic.
should suffice if, upon the filing of their certificates of candidacy, they elect Philippine citizenship
to terminate their status as persons with dual citizenship considering that their condition is the
unavoidable consequence of conflicting laws of different states. As Joaquin G. Bernas, one of SENATOR PIMENTEL. That is true, Mr. President. But if he exercises acts
the most perceptive members of the Constitutional Commission, pointed out: "[D]ual citizenship that will prove that he also acknowledges other citizenships, then he will
is just a reality imposed on us because we have no control of the laws on citizenship of other probably fall under this disqualification.
countries. We recognize a child of a Filipino mother. But whether she is considered a citizen of
another country is something completely beyond our control." 12 This is similar to the requirement that an applicant for naturalization must renounce "all
allegiance and fidelity to any foreign prince, potentate, state, or sovereignty" 14 of which at the
By electing Philippine citizenship, such candidates at the same time forswear allegiance to the time he is a subject or citizen before he can be issued a certificate of naturalization as a citizen
other country of which they are also citizens and thereby terminate their status as dual citizens. of the Philippines. In Parado v. Republic, 15 it was held:
It may be that, from the point of view of the foreign state and of its laws, such an individual has
not effectively renounced his foreign citizenship. That is of no moment as the following [W]hen a person applying for citizenship by naturalization takes an oath that
discussion on §40(d) between Senators Enrile and Pimentel clearly shows: 13 he renounce, his loyalty to any other country or government and solemnly
declares that he owes his allegiance to the Republic of the Philippines, the
SENATOR ENRILE. Mr. President, I would like to ask clarification of line 41, condition imposed by law is satisfied and compiled with. The determination
page 17: "Any person with dual citizenship" is disqualified to run for any whether such renunciation is valid or fully complies with the provisions of
elective local position. Under the present Constitution, Mr. President, our Naturalization Law lies within the province and is an exclusive
someone whose mother is a citizen of the Philippines but his father is a prerogative of our courts. The latter should apply the law duly enacted by
foreigner is a natural-born citizen of the Republic. There is no requirement the legislative department of the Republic. No foreign law may or should
that such a natural born citizen, upon reaching the age of majority, must interfere with its operation and application. If the requirement of the Chinese
elect or give up Philippine citizenship. Law of Nationality were to be read into our Naturalization Law, we would be
applying not what our legislative department has deemed it wise to require,
but what a foreign government has thought or intended to exact. That, of
On the assumption that this person would carry two passports, one course, is absurd. It must be resisted by all means and at all cost. It would
belonging to the country of his or her father and one belonging to the be a brazen encroachment upon the sovereign will and power of the people
Republic of the Philippines, may such a situation disqualify the person to run of this Republic.
for a local government position?
III. PETITIONER'S ELECTION OF PHILIPPINE CITIZENSHIP It is not disputed that on January 20, 1983 Frivaldo became an American.
Would the retroactivity of his repatriation not effectively give him dual
citizenship, which under Sec. 40 of the Local Government Code would
The record shows that private respondent was born in San Francisco, California on September
disqualify him "from running for any elective local position?" We answer this
4, 1955, of Filipino parents. Since the Philippines adheres to the principle of jus sanguinis, while
question in the negative, as there is cogent reason to hold that Frivaldo was
the United States follows the doctrine of jus soli, the parties agree that, at birth at least, he was a
really STATELESS at the time he took said oath of allegiance and even
national both of the Philippines and of the United States. However, the COMELEC en banc held
before that, when he ran for governor in 1988. In his Comment, Frivaldo
that, by participating in Philippine elections in 1992, 1995, and 1998, private respondent
wrote that he "had long renounced and had long abandoned his American
"effectively renounced his U.S. citizenship under American law," so that now he is solely a
citizenship — long before May 8, 1995. At best, Frivaldo was stateless in
Philippine national.
the interim — when he abandoned and renounced his US citizenship but
before he was repatriated to his Filipino citizenship."
Petitioner challenges this ruling. He argues that merely taking part in Philippine elections is not
sufficient evidence of renunciation and that, in any event, as the alleged renunciation was made
On this point, we quote from the assailed Resolution dated December 19,
when private respondent was already 37 years old, it was ineffective as it should have been
1995:
made when he reached the age of majority.

By the laws of the United States, petitioner Frivaldo lost


In holding that by voting in Philippine elections private respondent renounced his American
his American citizenship when he took his oath of
citizenship, the COMELEC must have in mind §349 of the Immigration and Nationality Act of the
allegiance to the Philippine Government when he ran
United States, which provided that "A person who is a national of the United States, whether by
for Governor in 1988, in 1992, and in 1995. Every
birth or naturalization, shall lose his nationality by: . . . (e) Voting in a political election in a foreign
certificate of candidacy contains an oath of allegiance
state or participating in an election or plebiscite to determine the sovereignty over foreign
to the Philippine Government.
territory." To be sure this provision was declared unconstitutional by the U.S. Supreme Court
in Afroyim v. Rusk 16 as beyond the power given to the U.S. Congress to regulate foreign
relations. However, by filing a certificate of candidacy when he ran for his present post, private These factual findings that Frivaldo has lost his foreign nationality long
respondent elected Philippine citizenship and in effect renounced his American citizenship. before the elections of 1995 have not been effectively rebutted by Lee.
Private respondent's certificate of candidacy, filed on March 27, 1998, contained the following Furthermore, it is basic that such findings of the Commission are conclusive
statements made under oath: upon this Court, absent any showing of capriciousness or arbitrariness or
abuse.
6. I AM A FILIPINO CITIZEN (STATE IF "NATURAL-
BORN" OR "NATURALIZED") NATURAL-BORN There is, therefore, no merit in petitioner's contention that the oath of allegiance contained in
private respondent's certificate of candidacy is insufficient to constitute renunciation that, to be
effective, such renunciation should have been made upon private respondent reaching the age
xxx xxx xxx
of majority since no law requires the election of Philippine citizenship to be made upon majority
age.
10. I AM A REGISTERED VOTER OF PRECINCT NO. 747-A, BARANGAY
SAN LORENZO, CITY/MUNICIPALITY OF MAKATI, PROVINCE OF NCR.
Finally, much is made of the fact that private respondent admitted that he is registered as an
American citizen in the Bureau of Immigration and Deportation and that he holds an American
11. I AM NOT A PERMANENT RESIDENT OF, OR IMMIGRANT TO, A passport which he used in his last travel to the United States on April 22, 1997. There is no merit
FOREIGN COUNTRY. in this. Until the filing of his certificate of candidacy on March 21, 1998, he had dual citizenship.
The acts attributed to him can be considered simply as the assertion of his American nationality
before the termination of his American citizenship. What this Court said in Aznar
12. I AM ELIGIBLE FOR THE OFFICE I SEEK TO BE ELECTED. I WILL v. COMELEC 18 applies mutatis mundatis to private respondent in the case at bar:
SUPPORT AND DEFEND THE CONSTITUTION OF THE PHILIPPINES
AND WILL MAINTAIN TRUE FAITH AND ALLEGIANCE THERETO; THAT I
WILL OBEY THE LAWS, LEGAL ORDERS AND DECREES . . . Considering the fact that admittedly Osmeña was both a Filipino and an
PROMULGATED BY THE DULY CONSTITUTED AUTHORITIES OF THE American, the mere fact that he has a Certificate staring he is an American
REPUBLIC OF THE PHILIPPINES; AND THAT I IMPOSE THIS does not mean that he is not still a Filipino. . . . [T]he Certification that he is
OBLIGATION UPON MYSELF VOLUNTARILY, WITHOUT MENTAL an American does not mean that he is not still a Filipino, possessed as he
RESERVATION OR PURPOSE OF EVASION. I HEREBY CERTIFY THAT is, of both nationalities or citizenships. Indeed, there is no express
THE FACTS STATED HEREIN ARE TRUE AND CORRECT OF MY OWN renunciation here of Philippine citizenship; truth to tell, there is even no
PERSONAL KNOWLEDGE. implied renunciation of said citizenship. When We consider that the
renunciation needed to lose Philippine citizenship must be "express," it
stands to reason that there can be no such loss of Philippine citizenship
The filing of such certificate of candidacy sufficed to renounce his American citizenship,
when there is no renunciation, either "express" or "implied."
effectively removing any disqualification he might have as a dual citizen. Thus, in Frivaldo
v. COMELEC it was held: 17
To recapitulate, by declaring in his certificate of candidacy that he is a Filipino citizen; that he is
not a permanent resident or immigrant of another country; that he will defend and support the
Constitution of the Philippines and bear true faith and allegiance thereto and that he does so
without mental reservation, private respondent has, as far as the laws of this country are
concerned, effectively repudiated his American citizenship and anything which he may have said
before as a dual citizen.

On the other hand, private respondent's oath of allegiance to the Philippines, when considered
with the fact that he has spent his youth and adulthood, received his education, practiced his
profession as an artist, and taken part in past elections in this country, leaves no doubt of his
election of Philippine citizenship.

His declarations will be taken upon the faith that he will fulfill his undertaking made under oath.
Should he betray that trust, there are enough sanctions for declaring the loss of his Philippine
citizenship through expatriation in appropriate proceedings. In Yu v. Defensor-Santiago, 19 we
sustained the denial of entry into the country of petitioner on the ground that, after taking his
oath as a naturalized citizen, he applied for the renewal of his Portuguese passport and declared
in commercial documents executed abroad that he was a Portuguese national. A similar
sanction can be taken against any one who, in electing Philippine citizenship, renounces his
foreign nationality, but subsequently does some act constituting renunciation of his Philippine
citizenship.

WHEREFORE, the petition for certiorari is DISMISSED for lack of merit.1âwphi1.nêt

SO ORDERED.
G.R. No. 195649 April 16, 2013 I am a natural born Filipino citizen / naturalized Filipino citizen.

CASAN MACODE MAQUILING, Petitioner, I am not a permanent resident of, or immigrant to, a foreign country.
vs.
COMMISSION ON ELECTIONS, ROMMEL ARNADO y CAGOCO, LINOG G.
I am eligible for the office I seek to be elected to.
BALUA, Respondents.

I will support and defend the Constitution of the Republic of the Philippines and will maintain true
DECISION
faith and allegiance thereto. I will obey the laws, legal orders and decrees promulgated by the
duly constituted authorities.
SERENO, CJ.:
I impose this obligation upon myself voluntarily without mental reservation or purpose of
THE CASE evasion.8

This is a Petition for Certiorari ender Rule 64 in conjunction with Rule 65 of the Rules of Court to On 28 April 2010, respondent Linog C. Balua (Balua), another mayoralty candidate, filed a
review the Resolutions of the Commission on Elections (COMELEC). The Resolution1 in SPA petition to disqualify Arnado and/or to cancel his certificate of candidacy for municipal mayor of
No. 10-1 09(DC) of the COMELEC First Division dated 5 October 201 0 is being assailed for Kauswagan, Lanao del Norte in connection with the 10 May 2010 local and national elections. 9
applying Section 44 of the Local Government Code while the Resolution2 of the COMELEC En
Banc dated 2 February 2011 is being questioned for finding that respondent Rommel Arnado y
Respondent Balua contended that Arnado is not a resident of Kauswagan, Lanao del Norte and
Cagoco (respondent Arnado/Arnado) is solely a Filipino citizen qualified to run for public office
that he is a foreigner, attaching thereto a certification issued by the Bureau of Immigration dated
despite his continued use of a U.S. passport.
23 April 2010 indicating the nationality of Arnado as "USA-American."10To further bolster his
claim of Arnado’s US citizenship, Balua presented in his Memorandum a computer-generated
FACTS travel record11 dated 03 December 2009 indicating that Arnado has been using his US Passport
No. 057782700 in entering and departing the Philippines. The said record shows that Arnado left
the country on 14 April 2009 and returned on 25 June 2009, and again departed on 29 July
Respondent Arnado is a natural born Filipino citizen.3 However, as a consequence of his
2009, arriving back in the Philippines on 24 November 2009.
subsequent naturalization as a citizen of the United States of America, he lost his Filipino
citizenship. Arnado applied for repatriation under Republic Act (R.A.) No. 9225 before the
Consulate General of the Philippines in San Franciso, USA and took the Oath of Allegiance to Balua likewise presented a certification from the Bureau of Immigration dated 23 April 2010,
the Republic of the Philippines on 10 July 2008.4 On the same day an Order of Approval of his certifying that the name "Arnado, Rommel Cagoco" appears in the available Computer
Citizenship Retention and Re-acquisition was issued in his favor.5 Database/Passenger manifest/IBM listing on file as of 21 April 2010, with the following pertinent
travel records:
The aforementioned Oath of Allegiance states:
DATE OF Arrival : 01/12/2010
I, Rommel Cagoco Arnado, solemnly swear that I will support and defend the Constitution of the
Republic of the Philippines and obey the laws and legal orders promulgated by the duly NATIONALITY : USA-AMERICAN
constituted authorities of the Philippines and I hereby declare that I recognize and accept the
supreme authority of the Philippines and will maintain true faith and allegiance thereto; and that I
PASSPORT : 057782700
impose this obligation upon myself voluntarily without mental reservation or purpose of evasion. 6

DATE OF Arrival : 03/23/2010


On 3 April 2009 Arnado again took his Oath of Allegiance to the Republic and executed an
Affidavit of Renunciation of his foreign citizenship, which states:
NATIONALITY : USA-AMERICAN
I, Rommel Cagoco Arnado, do solemnly swear that I absolutely and perpetually renounce all
allegiance and fidelity to the UNITED STATES OF AMERICA of which I am a citizen, and I PASSPORT : 05778270012
divest myself of full employment of all civil and political rights and privileges of the United States
of America.
On 30 April 2010, the COMELEC (First Division) issued an Order13 requiring the respondent to
personally file his answer and memorandum within three (3) days from receipt thereof.
I solemnly swear that all the foregoing statement is true and correct to the best of my knowledge
and belief.7
After Arnado failed to answer the petition, Balua moved to declare him in default and to present
evidence ex-parte.
On 30 November 2009, Arnado filed his Certificate of Candidacy for Mayor of Kauswagan,
Lanao del Norte, which contains, among others, the following statements:
Neither motion was acted upon, having been overtaken by the 2010 elections where Arnado Yu case, "a passport is defined as an official document of identity and nationality issued to a
garnered the highest number of votes and was subsequently proclaimed as the winning person intending to travel or sojourn in foreign countries." Surely, one who truly divested himself
candidate for Mayor of Kauswagan, Lanao del Norte. of US citizenship would not continue to avail of privileges reserved solely for US nationals. 19

It was only after his proclamation that Arnado filed his verified answer, submitting the following The dispositive portion of the Resolution rendered by the COMELEC
documents as evidence:14
First Division reads:
1. Affidavit of Renunciation and Oath of Allegiance to the Republic of the Philippines
dated 03 April 2009;
WHEREFORE, in view of the foregoing, the petition for disqualification and/or to cancel the
certificate of candidacy of Rommel C. Arnado is hereby GRANTED. Rommel C. Arnado’s
2. Joint-Affidavit dated 31 May 2010 of Engr. Virgil Seno, Virginia Branzuela, Leoncio proclamation as the winning candidate for Municipal Mayor of Kauswagan, Lanao del Nore is
Daligdig, and Jessy Corpin, all neighbors of Arnado, attesting that Arnado is a long- hereby ANNULLED. Let the order of succession under Section 44 of the Local Government
time resident of Kauswagan and that he has been conspicuously and continuously Code of 1991 take effect.20
residing in his family’s ancestral house in Kauswagan;
The Motion for Reconsideration and
3. Certification from the Punong Barangay of Poblacion, Kauswagan, Lanao del Norte the Motion for Intervention
dated 03 June 2010 stating that Arnado is a bona fide resident of his barangay and
that Arnado went to the United States in 1985 to work and returned to the Philippines
Arnado sought reconsideration of the resolution before the COMELEC En Banc on the ground
in 2009;
that "the evidence is insufficient to justify the Resolution and that the said Resolution is contrary
to law."21 He raised the following contentions:22
4. Certification dated 31 May 2010 from the Municipal Local Government Operations
Office of Kauswagan stating that Dr. Maximo P. Arnado, Sr. served as Mayor of
1. The finding that he is not a Filipino citizen is not supported by the evidence
Kauswagan, from January 1964 to June 1974 and from 15 February 1979 to 15 April
consisting of his Oath of Allegiance and the Affidavit of Renunciation, which show that
1986; and
he has substantially complied with the requirements of R.A. No. 9225;

5. Voter Certification issued by the Election Officer of Kauswagan certifying that


2. The use of his US passport subsequent to his renunciation of his American
Arnado has been a registered voter of Kauswagan since 03 April 2009.
citizenship is not tantamount to a repudiation of his Filipino citizenship, as he did not
perform any act to swear allegiance to a country other than the Philippines;
THE RULING OF THE COMELEC FIRST DIVISION
3. He used his US passport only because he was not informed of the issuance of his
Instead of treating the Petition as an action for the cancellation of a certificate of candidacy Philippine passport, and that he used his Philippine passport after he obtained it;
based on misrepresentation,15 the COMELEC First Division considered it as one for
disqualification. Balua’s contention that Arnado is a resident of the United States was dismissed
4. Balua’s petition to cancel the certificate of candidacy of Arnado was filed out of
upon the finding that "Balua failed to present any evidence to support his contention,"16 whereas
time, and the First Division’s treatment of the petition as one for disqualification
the First Division still could "not conclude that Arnado failed to meet the one-year residency
constitutes grave abuse of discretion amounting to excess of jurisdiction; 23
requirement under the Local Government Code."17

5. He is undoubtedly the people’s choice as indicated by his winning the elections;


In the matter of the issue of citizenship, however, the First Division disagreed with Arnado’s
claim that he is a Filipino citizen.18
6. His proclamation as the winning candidate ousted the COMELEC from jurisdiction
over the case; and
We find that although Arnado appears to have substantially complied with the requirements of
R.A. No. 9225, Arnado’s act of consistently using his US passport after renouncing his US
citizenship on 03 April 2009 effectively negated his Affidavit of Renunciation. 7. The proper remedy to question his citizenship is through a petition for quo warranto,
which should have been filed within ten days from his proclamation.
xxxx
Petitioner Casan Macode Maquiling (Maquiling), another candidate for mayor of Kauswagan,
and who garnered the second highest number of votes in the 2010 elections, intervened in the
Arnado’s continued use of his US passport is a strong indication that Arnado had no real
case and filed before the COMELEC En Banc a Motion for Reconsideration together with an
intention to renounce his US citizenship and that he only executed an Affidavit of Renunciation
Opposition to Arnado’s Amended Motion for Reconsideration. Maquiling argued that while the
to enable him to run for office. We cannot turn a blind eye to the glaring inconsistency between
First Division correctly disqualified Arnado, the order of succession under Section 44 of the Local
Arnado’s unexplained use of a US passport six times and his claim that he re-acquired his
Government Code is not applicable in this case. Consequently, he claimed that the cancellation
Philippine citizenship and renounced his US citizenship. As noted by the Supreme Court in the
of Arnado’s candidacy and the nullification of his proclamation, Maquiling, as the legitimate xxxx
candidate who obtained the highest number of lawful votes, should be proclaimed as the winner.
The respondent presented a plausible explanation as to the use of his US passport. Although he
Maquiling simultaneously filed his Memorandum with his Motion for Intervention and his Motion applied for a Philippine passport, the passport was only issued on June 18, 2009. However, he
for Reconsideration. Arnado opposed all motions filed by Maquiling, claiming that intervention is was not notified of the issuance of his Philippine passport so that he was actually able to get it
prohibited after a decision has already been rendered, and that as a second-placer, Maquiling about three (3) months later. Yet as soon as he was in possession of his Philippine passport, the
undoubtedly lost the elections and thus does not stand to be prejudiced or benefitted by the final respondent already used the same in his subsequent travels abroad. This fact is proven by the
adjudication of the case. respondent’s submission of a certified true copy of his passport showing that he used the same
for his travels on the following dates: January 31, 2010, April 16, 2010, May 20, 2010, January
12, 2010, March 31, 2010 and June 4, 2010. This then shows that the use of the US passport
RULING OF THE COMELEC EN BANC
was because to his knowledge, his Philippine passport was not yet issued to him for his use. As
probably pressing needs might be undertaken, the respondent used whatever is within his
In its Resolution of 02 February 2011, the COMELEC En Banc held that under Section 6 of control during that time.25
Republic Act No. 6646, the Commission "shall continue with the trial and hearing of the action,
inquiry or protest even after the proclamation of the candidate whose qualifications for office is
In his Separate Concurring Opinion, COMELEC Chairman Sixto Brillantes cited that the use of
questioned."
foreign passport is not one of the grounds provided for under Section 1 of Commonwealth Act
No. 63 through which Philippine citizenship may be lost.
As to Maquiling’s intervention, the COMELEC En Banc also cited Section 6 of R.A. No. 6646
which allows intervention in proceedings for disqualification even after elections if no final
"The application of the more assimilative principle of continuity of citizenship is more appropriate
judgment has been rendered, but went on further to say that Maquiling, as the second placer,
in this case. Under said principle, once a person becomes a citizen, either by birth or
would not be prejudiced by the outcome of the case as it agrees with the dispositive portion of
naturalization, it is assumed that he desires to continue to be a citizen, and this assumption
the Resolution of the First Division allowing the order of succession under Section 44 of the
stands until he voluntarily denationalizes or expatriates himself. Thus, in the instant case
Local Government Code to take effect.
respondent after reacquiring his Philippine citizenship should be presumed to have remained a
Filipino despite his use of his American passport in the absence of clear, unequivocal and
The COMELEC En Banc agreed with the treatment by the First Division of the petition as one for competent proof of expatriation. Accordingly, all doubts should be resolved in favor of retention
disqualification, and ruled that the petition was filed well within the period prescribed by of citizenship."26
law,24 having been filed on 28 April 2010, which is not later than 11 May 2010, the date of
proclamation.
On the other hand, Commissioner Rene V. Sarmiento dissented, thus:

However, the COMELEC En Banc reversed and set aside the ruling of the First Division and
Respondent evidently failed to prove that he truly and wholeheartedly abandoned his allegiance
granted Arnado’s Motion for Reconsideration, on the following premises:
to the United States. The latter’s continued use of his US passport and enjoyment of all the
privileges of a US citizen despite his previous renunciation of the afore-mentioned citizenship
First: runs contrary to his declaration that he chose to retain only his Philippine citizenship.
Respondent’s submission with the twin requirements was obviously only for the purpose of
complying with the requirements for running for the mayoralty post in connection with the May
By renouncing his US citizenship as imposed by R.A. No. 9225, the respondent embraced his 10, 2010 Automated National and Local Elections.
Philippine citizenship as though he never became a citizen of another country. It was at that
time, April 3, 2009, that the respondent became a pure Philippine Citizen again.
Qualifications for elective office, such as citizenship, are continuing requirements; once any of
them is lost during his incumbency, title to the office itself is deemed forfeited. If a candidate is
xxxx
not a citizen at the time he ran for office or if he lost his citizenship after his election to office, he
is disqualified to serve as such. Neither does the fact that respondent obtained the plurality of
The use of a US passport … does not operate to revert back his status as a dual citizen prior to votes for the mayoralty post cure the latter’s failure to comply with the qualification requirements
his renunciation as there is no law saying such. More succinctly, the use of a US passport does regarding his citizenship.
not operate to "un-renounce" what he has earlier on renounced. The First Division’s reliance in
the case of In Re: Petition for Habeas Corpus of Willy Yu v. Defensor-Santiago, et al. is Since a disqualified candidate is no candidate at all in the eyes of the law, his having received
misplaced. The petitioner in the said case is a naturalized citizen who, after taking his oath as a
the highest number of votes does not validate his election. It has been held that where a petition
naturalized Filipino, applied for the renewal of his Portuguese passport. Strict policy is for disqualification was filed before election against a candidate but was adversely resolved
maintained in the conduct of citizens who are not natural born, who acquire their citizenship by against him after election, his having obtained the highest number of votes did not make his
choice, thus discarding their original citizenship. The Philippine State expects strict conduct of
election valid. His ouster from office does not violate the principle of vox populi suprema est lex
allegiance to those who choose to be its citizens. In the present case, respondent is not a because the application of the constitutional and statutory provisions on disqualification is not a
naturalized citizen but a natural born citizen who chose greener pastures by working abroad and matter of popularity. To apply it is to breath[e] life to the sovereign will of the people who
then decided to repatriate to supposedly help in the progress of Kauswagan. He did not apply for
expressed it when they ratified the Constitution and when they elected their representatives who
a US passport after his renunciation. Thus the mentioned case is not on all fours with the case at enacted the law.27
bar.
THE PETITION BEFORE THE COURT Sec. 6. Effect of Disqualification Case. - Any candidate who has been declared by final judgment
to be disqualified shall not be voted for, and the votes cast for him shall not be counted. If for any
reason a candidate is not declared by final judgment before an election to be disqualified and he
Maquiling filed the instant petition questioning the propriety of declaring Arnado qualified to run
is voted for and receives the winning number of votes in such election, the Court or Commission
for public office despite his continued use of a US passport, and praying that Maquiling be
shall continue with the trial and hearing of the action, inquiry, or protest and, upon motion of the
proclaimed as the winner in the 2010 mayoralty race in Kauswagan, Lanao del Norte.
complainant or any intervenor, may during the pendency thereof order the suspension of the
proclamation of such candidate whenever the evidence of his guilt is strong.
Ascribing both grave abuse of discretion and reversible error on the part of the COMELEC En
Banc for ruling that Arnado is a Filipino citizen despite his continued use of a US passport,
Mercado v. Manzano28
Maquiling now seeks to reverse the finding of the COMELEC En Banc that Arnado is qualified to
run for public office.
clarified the right of intervention in a disqualification case. In that case, the Court said:
Corollary to his plea to reverse the ruling of the COMELEC En Banc or to affirm the First
Division’s disqualification of Arnado, Maquiling also seeks the review of the applicability of That petitioner had a right to intervene at that stage of the proceedings for the disqualification
Section 44 of the Local Government Code, claiming that the COMELEC committed reversible against private respondent is clear from Section 6 of R.A. No. 6646, otherwise known as the
error in ruling that "the succession of the vice mayor in case the respondent is disqualified is in Electoral Reforms Law of 1987, which provides: Any candidate who has been declared by final
order." judgment to be disqualified shall not be voted for, and the votes cast for him shall not be
counted. If for any reason a candidate is not declared by final judgment before an election to be
disqualified and he is voted for and receives the winning number of votes in such election, the
There are three questions posed by the parties before this Court which will be addressed
Court or Commission shall continue with the trial and hearing of the action, inquiry, or protest
seriatim as the subsequent questions hinge on the result of the first.
and, upon motion of the complainant or any intervenor, may during the pendency thereof order
the suspension of the proclamation of such candidate whenever the evidence of guilt is strong.
The first question is whether or not intervention is allowed in a disqualification case. Under this provision, intervention may be allowed in proceedings for disqualification even after
election if there has yet been no final judgment rendered.29
The second question is whether or not the use of a foreign passport after renouncing foreign
citizenship amounts to undoing a renunciation earlier made. Clearly then, Maquiling has the right to intervene in the case. The fact that the COMELEC En
Banc has already ruled that Maquiling has not shown that the requisites for the exemption to the
second-placer rule set forth in Sinsuat v. COMELEC30 are present and therefore would not be
A better framing of the question though should be whether or not the use of a foreign passport prejudiced by the outcome of the case, does not deprive Maquiling of the right to elevate the
after renouncing foreign citizenship affects one’s qualifications to run for public office.
matter before this Court.

The third question is whether or not the rule on succession in the Local Government Code is Arnado’s claim that the main case has attained finality as the original petitioner and respondents
applicable to this case. therein have not appealed the decision of the COMELEC En Banc, cannot be sustained. The
elevation of the case by the intervenor prevents it from attaining finality. It is only after this Court
OUR RULING has ruled upon the issues raised in this instant petition that the disqualification case originally
filed by Balua against Arnado will attain finality.
Intervention of a rival candidate in a
disqualification case is proper when The use of foreign passport after renouncing one’s foreign citizenship is a positive and
there has not yet been any voluntary act of representation as to one’s nationality and citizenship; it does not divest
proclamation of the winner. Filipino citizenship regained by repatriation but it recants the Oath of Renunciation
required to qualify one to run for an elective position.
Petitioner Casan Macode Maquiling intervened at the stage when respondent Arnado filed a
Motion for Reconsideration of the First Division Resolution before the COMELEC En Banc. As Section 5(2) of The Citizenship Retention and Re-acquisition Act of 2003 provides:
the candidate who garnered the second highest number of votes, Maquiling contends that he
has an interest in the disqualification case filed against Arnado, considering that in the event the Those who retain or re-acquire Philippine citizenship under this Act shall enjoy full civil and
latter is disqualified, the votes cast for him should be considered stray and the second-placer political rights and be subject to all attendant liabilities and responsibilities under existing laws of
should be proclaimed as the winner in the elections.
the Philippines and the following conditions:

It must be emphasized that while the original petition before the COMELEC is one for xxxx
cancellation of the certificate of candidacy and / or disqualification, the COMELEC First Division
and the COMELEC En Banc correctly treated the petition as one for disqualification.
(2)Those seeking elective public in the Philippines shall meet the qualification for holding such
public office as required by the Constitution and existing laws and, at the time of the filing of the
The effect of a disqualification case is enunciated in Section 6 of R.A. No. 6646:
certificate of candidacy, make a personal and sworn renunciation of any and all foreign before oath as a naturalized citizen, he applied for the renewal of his Portuguese passport and declared
any public officer authorized to administer an oath. in commercial documents executed abroad that he was a Portuguese national. A similar
sanction can be taken against anyone who, in electing Philippine citizenship, renounces his
foreign nationality, but subsequently does some act constituting renunciation of his Philippine
x x x31
citizenship.

Rommel Arnado took all the necessary steps to qualify to run for a public office. He took the
While the act of using a foreign passport is not one of the acts enumerated in Commonwealth
Oath of Allegiance and renounced his foreign citizenship. There is no question that after
Act No. 63 constituting renunciation and loss of Philippine citizenship,35 it is nevertheless an act
performing these twin requirements required under Section 5(2) of R.A. No. 9225 or the
which repudiates the very oath of renunciation required for a former Filipino citizen who is also a
Citizenship Retention and Re-acquisition Act of 2003, he became eligible to run for public office.
citizen of another country to be qualified to run for a local elective position.

Indeed, Arnado took the Oath of Allegiance not just only once but twice: first, on 10 July 2008
When Arnado used his US passport on 14 April 2009, or just eleven days after he renounced his
when he applied for repatriation before the Consulate General of the Philippines in San
American citizenship, he recanted his Oath of Renunciation36 that he "absolutely and perpetually
Francisco, USA, and again on 03 April 2009 simultaneous with the execution of his Affidavit of
renounce(s) all allegiance and fidelity to the UNITED STATES OF AMERICA" 37 and that he
Renunciation. By taking the Oath of Allegiance to the Republic, Arnado re-acquired his
"divest(s) himself of full employment of all civil and political rights and privileges of the United
Philippine citizenship. At the time, however, he likewise possessed American citizenship. Arnado
States of America."38
had therefore become a dual citizen.

We agree with the COMELEC En Banc that such act of using a foreign passport does not divest
After reacquiring his Philippine citizenship, Arnado renounced his American citizenship by
Arnado of his Filipino citizenship, which he acquired by repatriation. However, by representing
executing an Affidavit of Renunciation, thus completing the requirements for eligibility to run for
himself as an American citizen, Arnado voluntarily and effectively reverted to his earlier status as
public office.
a dual citizen. Such reversion was not retroactive; it took place the instant Arnado represented
himself as an American citizen by using his US passport.
By renouncing his foreign citizenship, he was deemed to be solely a Filipino citizen, regardless
of the effect of such renunciation under the laws of the foreign country.32
This act of using a foreign passport after renouncing one’s foreign citizenship is fatal to Arnado’s
bid for public office, as it effectively imposed on him a disqualification to run for an elective local
However, this legal presumption does not operate permanently and is open to attack when, after position.
renouncing the foreign citizenship, the citizen performs positive acts showing his continued
possession of a foreign citizenship.33
Arnado’s category of dual citizenship is that by which foreign citizenship is acquired through a
positive act of applying for naturalization. This is distinct from those considered dual citizens by
Arnado himself subjected the issue of his citizenship to attack when, after renouncing his foreign virtue of birth, who are not required by law to take the oath of renunciation as the mere filing of
citizenship, he continued to use his US passport to travel in and out of the country before filing the certificate of candidacy already carries with it an implied renunciation of foreign
his certificate of candidacy on 30 November 2009. The pivotal question to determine is whether citizenship.39 Dual citizens by naturalization, on the other hand, are required to take not only the
he was solely and exclusively a Filipino citizen at the time he filed his certificate of candidacy, Oath of Allegiance to the Republic of the Philippines but also to personally renounce foreign
thereby rendering him eligible to run for public office. citizenship in order to qualify as a candidate for public office.

Between 03 April 2009, the date he renounced his foreign citizenship, and 30 November 2009, By the time he filed his certificate of candidacy on 30 November 2009, Arnado was a dual citizen
the date he filed his COC, he used his US passport four times, actions that run counter to the enjoying the rights and privileges of Filipino and American citizenship. He was qualified to vote,
affidavit of renunciation he had earlier executed. By using his foreign passport, Arnado positively but by the express disqualification under Section 40(d) of the Local Government Code, 40 he was
and voluntarily represented himself as an American, in effect declaring before immigration not qualified to run for a local elective position.
authorities of both countries that he is an American citizen, with all attendant rights and
privileges granted by the United States of America.
In effect, Arnado was solely and exclusively a Filipino citizen only for a period of eleven days, or
from 3 April 2009 until 14 April 2009, on which date he first used his American passport after
The renunciation of foreign citizenship is not a hollow oath that can simply be professed at any renouncing his American citizenship.
time, only to be violated the next day. It requires an absolute and perpetual renunciation of the
foreign citizenship and a full divestment of all civil and political rights granted by the foreign
This Court has previously ruled that:
country which granted the citizenship.

Qualifications for public office are continuing requirements and must be possessed not only at
Mercado v. Manzano34 already hinted at this situation when the Court declared:
the time of appointment or election or assumption of office but during the officer's entire tenure.
Once any of the required qualifications is lost, his title may be seasonably challenged. x x x. 41
His declarations will be taken upon the faith that he will fulfill his undertaking made under oath.
Should he betray that trust, there are enough sanctions for declaring the loss of his Philippine
citizenship through expatriation in appropriate proceedings. In Yu v. Defensor-Santiago, we
sustained the denial of entry into the country of petitioner on the ground that, after taking his
The citizenship requirement for elective public office is a continuing one. It must be possessed We now resolve the next issue.
not just at the time of the renunciation of the foreign citizenship but continuously. Any act which
violates the oath of renunciation opens the citizenship issue to attack.
Resolving the third issue necessitates revisiting Topacio v. Paredes45 which is the jurisprudential
spring of the principle that a second-placer cannot be proclaimed as the winner in an election
We agree with the pronouncement of the COMELEC First Division that "Arnado’s act of contest. This doctrine must be re-examined and its soundness once again put to the test to
consistently using his US passport effectively negated his "Affidavit of Renunciation." 42 This address the ever-recurring issue that a second-placer who loses to an ineligible candidate
does not mean, that he failed to comply with the twin requirements under R.A. No. 9225, for he cannot be proclaimed as the winner in the elections.
in fact did.
The Facts of the case are as follows:
It was after complying with the requirements that he performed positive acts which effectively
disqualified him from running for an elective public office pursuant to Section 40(d) of the Local
On June 4, 1912, a general election was held in the town of Imus, Province of Cavite, to fill the
Government Code of 1991.
office of municipal president. The petitioner, Felipe Topacio, and the respondent, Maximo Abad,
were opposing candidates for that office. Topacio received 430 votes, and Abad 281. Abad
The purpose of the Local Government Code in disqualifying dual citizens from running for any contested the election upon the sole ground that Topacio was ineligible in that he was reelected
elective public office would be thwarted if we were to allow a person who has earlier renounced the second time to the office of the municipal president on June 4, 1912, without the four years
his foreign citizenship, but who subsequently represents himself as a foreign citizen, to hold any required by Act No. 2045 having intervened.46
public office.
Abad thus questioned the eligibility of To p a c i o on the basis of a statutory prohibition for
Arnado justifies the continued use of his US passport with the explanation that he was not seeking a second re-election absent the four year interruption.
notified of the issuance of his Philippine passport on 18 June 2009, as a result of which he was
only able to obtain his Philippine passport three (3) months later. 43
The often-quoted phrase in Topacio v. Paredes is that "the wreath of victory cannot be
transferred from an ineligible candidate to any other candidate when the sole question is the
The COMELEC En Banc differentiated Arnado from Willy Yu, the Portuguese national who eligibility of the one receiving a plurality of the legally cast ballots."47
sought naturalization as a Filipino citizen and later applied for the renewal of his Portuguese
passport. That Arnado did not apply for a US passport after his renunciation does not make his
This phrase is not even the ratio decidendi; it is a mere obiter dictum. The Court was comparing
use of a US passport less of an act that violated the Oath of Renunciation he took. It was still a
"the effect of a decision that a candidate is not entitled to the office because of fraud or
positive act of representation as a US citizen before the immigration officials of this country.
irregularities in the elections x x x with that produced by declaring a person ineligible to hold
such an office."
The COMELEC, in ruling favorably for Arnado, stated "Yet, as soon as he was in possession of
his Philippine passport, the respondent already used the same in his subsequent travels
The complete sentence where the phrase is found is part of a comparison and contrast between
abroad."44 We cannot agree with the COMELEC. Three months from June is September. If
the two situations, thus:
indeed, Arnado used his Philippine passport as soon as he was in possession of it, he would not
have used his US passport on 24 November 2009.
Again, the effect of a decision that a candidate is not entitled to the office because of fraud or
irregularities in the elections is quite different from that produced by declaring a person ineligible
Besides, Arnado’s subsequent use of his Philippine passport does not correct the fact that after
to hold such an office. In the former case the court, after an examination of the ballots may find
he renounced his foreign citizenship and prior to filing his certificate of candidacy, he used his
that some other person than the candidate declared to have received a plurality by the board of
US passport. In the same way that the use of his foreign passport does not undo his Oath of
canvassers actually received the greater number of votes, in which case the court issues its
Renunciation, his subsequent use of his Philippine passport does not undo his earlier use of his
mandamus to the board of canvassers to correct the returns accordingly; or it may find that the
US passport.
manner of holding the election and the returns are so tainted with fraud or illegality that it cannot
be determined who received a plurality of the legally cast ballots. In the latter case, no question
Citizenship is not a matter of convenience. It is a badge of identity that comes with attendant civil as to the correctness of the returns or the manner of casting and counting the ballots is before
and political rights accorded by the state to its citizens. It likewise demands the concomitant duty the deciding power, and generally the only result can be that the election fails entirely. In the
to maintain allegiance to one’s flag and country. While those who acquire dual citizenship by former, we have a contest in the strict sense of the word, because of the opposing parties are
choice are afforded the right of suffrage, those who seek election or appointment to public office striving for supremacy. If it be found that the successful candidate (according to the board of
are required to renounce their foreign citizenship to be deserving of the public trust. Holding canvassers) obtained a plurality in an illegal manner, and that another candidate was the real
public office demands full and undivided allegiance to the Republic and to no other. victor, the former must retire in favor of the latter. In the other case, there is not, strictly
speaking, a contest, as the wreath of victory cannot be transferred from an ineligible candidate
to any other candidate when the sole question is the eligibility of the one receiving a plurality of
We therefore hold that Arnado, by using his US passport after renouncing his American
the legally cast ballots. In the one case the question is as to who received a plurality of the
citizenship, has recanted the same Oath of Renunciation he took. Section 40(d) of the Local
legally cast ballots; in the other, the question is confined to the personal character and
Government Code applies to his situation. He is disqualified not only from holding the public
circumstances of a single individual.48 (Emphasis supplied)
office but even from becoming a candidate in the May 2010 elections.
Note that the sentence where the phrase is found starts with "In the other case, there is not, right to hold public office. The number of ballots cast in his favor cannot cure the defect of failure
strictly speaking, a contest" in contrast to the earlier statement, "In the former, we have a contest to qualify with the substantive legal requirements of eligibility to run for public office.
in the strict sense of the word, because of the opposing parties are striving for supremacy."
The popular vote does not cure the
The Court in Topacio v. Paredes cannot be said to have held that "the wreath of victory cannot ineligibility of a candidate.
be transferred from an ineligible candidate to any other candidate when the sole question is the
eligibility of the one receiving a plurality of the legally cast ballots."
The ballot cannot override the constitutional and statutory requirements for qualifications and
disqualifications of candidates. When the law requires certain qualifications to be possessed or
A proper reading of the case reveals that the ruling therein is that since the Court of First that certain disqualifications be not possessed by persons desiring to serve as elective public
Instance is without jurisdiction to try a disqualification case based on the eligibility of the person officials, those qualifications must be met before one even becomes a candidate. When a
who obtained the highest number of votes in the election, its jurisdiction being confined "to person who is not qualified is voted for and eventually garners the highest number of votes, even
determine which of the contestants has been duly elected" the judge exceeded his jurisdiction the will of the electorate expressed through the ballot cannot cure the defect in the qualifications
when he "declared that no one had been legally elected president of the municipality of Imus at of the candidate. To rule otherwise is to trample upon and rent asunder the very law that sets
the general election held in that town on 4 June 1912" where "the only question raised was forth the qualifications and disqualifications of candidates. We might as well write off our election
whether or not Topacio was eligible to be elected and to hold the office of municipal president." laws if the voice of the electorate is the sole determinant of who should be proclaimed worthy to
occupy elective positions in our republic.
The Court did not rule that Topacio was disqualified and that Abad as the second placer cannot
be proclaimed in his stead. The Court therein ruled: This has been, in fact, already laid down by the Court in Frivaldo v. COMELEC50 when we
pronounced:
For the foregoing reasons, we are of the opinion and so hold that the respondent judge
exceeded his jurisdiction in declaring in those proceedings that no one was elected municipal x x x. The fact that he was elected by the people of Sorsogon does not excuse this patent
president of the municipality of Imus at the last general election; and that said order and all violation of the salutary rule limiting public office and employment only to the citizens of this
subsequent proceedings based thereon are null and void and of no effect; and, although this country. The qualifications prescribed for elective office cannot be erased by the electorate
decision is rendered on respondents' answer to the order to show cause, unless respondents alone.
raised some new and additional issues, let judgment be entered accordingly in 5 days, without
costs. So ordered.49
The will of the people as expressed through the ballot cannot cure the vice of ineligibility,
especially if they mistakenly believed, as in this case, that the candidate was qualified.
On closer scrutiny, the phrase relied upon by a host of decisions does not even have a legal Obviously, this rule requires strict application when the deficiency is lack of citizenship. If a
basis to stand on. It was a mere pronouncement of the Court comparing one process with person seeks to serve in the Republic of the Philippines, he must owe his total loyalty to this
another and explaining the effects thereof. As an independent statement, it is even illogical. country only, abjuring and renouncing all fealty and fidelity to any other state.51 (Emphasis
supplied)
Let us examine the statement:
This issue has also been jurisprudentially clarified in Velasco v. COMELEC52 where the Court
ruled that the ruling in Quizon and Saya-ang cannot be interpreted without qualifications lest
"x x x the wreath of victory cannot be transferred from an ineligible candidate to any other
"Election victory x x x becomes a magic formula to bypass election eligibility requirements."53
candidate when the sole question is the eligibility of the one receiving a plurality of the legally
cast ballots."
We have ruled in the past that a candidate’s victory in the election may be considered a
sufficient basis to rule in favor of the candidate sought to be disqualified if the main issue
What prevents the transfer of the wreath of victory from the ineligible candidate to another
involves defects in the candidate’s certificate of candidacy. We said that while provisions relating
candidate?
to certificates of candidacy are mandatory in terms, it is an established rule of interpretation as
regards election laws, that mandatory provisions requiring certain steps before elections will be
When the issue being decided upon by the Court is the eligibility of the one receiving a plurality construed as directory after the elections, to give effect to the will of the people. We so ruled in
of the legally cast ballots and ineligibility is thereafter established, what stops the Court from Quizon v. COMELEC and Saya-ang v. COMELEC:
adjudging another eligible candidate who received the next highest number of votes as the
winner and bestowing upon him that "wreath?"
The present case perhaps presents the proper time and opportunity to fine-tune our above
ruling. We say this with the realization that a blanket and unqualified reading and application of
An ineligible candidate who receives the highest number of votes is a wrongful winner. By this ruling can be fraught with dangerous significance for the rule of law and the integrity of our
express legal mandate, he could not even have been a candidate in the first place, but by virtue elections. For one, such blanket/unqualified reading may provide a way around the law that
of the lack of material time or any other intervening circumstances, his ineligibility might not have effectively negates election requirements aimed at providing the electorate with the basic
been passed upon prior to election date. Consequently, he may have had the opportunity to hold information to make an informed choice about a candidate’s eligibility and fitness for office.
himself out to the electorate as a legitimate and duly qualified candidate. However,
notwithstanding the outcome of the elections, his ineligibility as a candidate remains unchanged.
The first requirement that may fall when an unqualified reading is made is Section 39 of the LGC
Ineligibility does not only pertain to his qualifications as a candidate but necessarily affects his
which specifies the basic qualifications of local government officials. Equally susceptive of being
rendered toothless is Section 74 of the OEC that sets out what should be stated in a COC. The electorate’s awareness of the candidate’s disqualification is not a prerequisite for the
Section 78 may likewise be emasculated as mere delay in the resolution of the petition to cancel disqualification to attach to the candidate. The very existence of a disqualifying circumstance
or deny due course to a COC can render a Section 78 petition useless if a candidate with false makes the candidate ineligible. Knowledge by the electorate of a candidate’s disqualification is
COC data wins. To state the obvious, candidates may risk falsifying their COC qualifications if not necessary before a qualified candidate who placed second to a disqualified one can be
they know that an election victory will cure any defect that their COCs may have. Election victory proclaimed as the winner. The second-placer in the vote count is actually the first-placer among
then becomes a magic formula to bypass election eligibility requirements. (Citations omitted) the qualified candidates.

What will stop an otherwise disqualified individual from filing a seemingly valid COC, concealing That the disqualified candidate has already been proclaimed and has assumed office is of no
any disqualification, and employing every strategy to delay any disqualification case filed against moment. The subsequent disqualification based on a substantive ground that existed prior to the
him so he can submit himself to the electorate and win, if winning the election will guarantee a filing of the certificate of candidacy voids not only the COC but also the proclamation.
disregard of constitutional and statutory provisions on qualifications and disqualifications of
candidates?
Section 6 of R.A. No. 6646 provides:

It is imperative to safeguard the expression of the sovereign voice through the ballot by ensuring
Section 6. Effect of Disqualification Case. - Any candidate who has been declared by final
that its exercise respects the rule of law. To allow the sovereign voice spoken through the ballot
judgment to be disqualified shall not be voted for, and the votes cast for him shall not be
to trump constitutional and statutory provisions on qualifications and disqualifications of
counted. If for any reason a candidate is not declared by final judgment before an election to be
candidates is not democracy or republicanism. It is electoral anarchy. When set rules are
disqualified and he is voted for and receives the winning number of votes in such election, the
disregarded and only the electorate’s voice spoken through the ballot is made to matter in the
Court or Commission shall continue with the trial and hearing of the action, inquiry, or protest
end, it precisely serves as an open invitation for electoral anarchy to set in.1âwphi1
and, upon motion of the complainant or any intervenor, may during the pendency thereof order
the suspension of the proclamation of such candidate whenever the evidence of his guilt is
Maquiling is not a second-placer as strong.
he obtained the highest number of
votes from among the qualified
There was no chance for Arnado’s proclamation to be suspended under this rule because
candidates.
Arnado failed to file his answer to the petition seeking his disqualification. Arnado only filed his
Answer on 15 June 2010, long after the elections and after he was already proclaimed as the
With Arnado’s disqualification, Maquiling then becomes the winner in the election as he obtained winner.
the highest number of votes from among the qualified candidates.
The disqualifying circumstance surrounding Arnado’s candidacy involves his citizenship. It does
We have ruled in the recent cases of Aratea v. COMELEC54 and Jalosjos v. COMELEC55 that a not involve the commission of election offenses as provided for in the first sentence of Section
void COC cannot produce any legal effect. 68 of the Omnibus Election Code, the effect of which is to disqualify the individual from
continuing as a candidate, or if he has already been elected, from holding the office.
Thus, the votes cast in favor of the ineligible candidate are not considered at all in determining
the winner of an election. The disqualifying circumstance affecting Arnado is his citizenship. As earlier discussed, Arnado
was both a Filipino and an American citizen when he filed his certificate of candidacy. He was a
dual citizen disqualified to run for public office based on Section 40(d) of the Local Government
Even when the votes for the ineligible candidate are disregarded, the will of the electorate is still
Code.
respected, and even more so. The votes cast in favor of an ineligible candidate do not constitute
the sole and total expression of the sovereign voice. The votes cast in favor of eligible and
legitimate candidates form part of that voice and must also be respected. Section 40 starts with the statement "The following persons are disqualified from running for any
elective local position." The prohibition serves as a bar against the individuals who fall under any
of the enumeration from participating as candidates in the election.
As in any contest, elections are governed by rules that determine the qualifications and
disqualifications of those who are allowed to participate as players. When there are participants
who turn out to be ineligible, their victory is voided and the laurel is awarded to the next in rank With Arnado being barred from even becoming a candidate, his certificate of candidacy is thus
who does not possess any of the disqualifications nor lacks any of the qualifications set in the rendered void from the beginning. It could not have produced any other legal effect except that
rules to be eligible as candidates. Arnado rendered it impossible to effect his disqualification prior to the elections because he filed
his answer to the petition when the elections were conducted already and he was already
proclaimed the winner.
There is no need to apply the rule cited in Labo v. COMELEC56 that when the voters are well
aware within the realm of notoriety of a candidate’s disqualification and still cast their votes in
favor said candidate, then the eligible candidate obtaining the next higher number of votes may To hold that such proclamation is valid is to negate the prohibitory character of the
be deemed elected. That rule is also a mere obiter that further complicated the rules affecting disqualification which Arnado possessed even prior to the filing of the certificate of candidacy.
qualified candidates who placed second to ineligible ones. The affirmation of Arnado's disqualification, although made long after the elections, reaches
back to the filing of the certificate of candidacy. Arnado is declared to be not a candidate at all in
the May 201 0 elections.
Arnado being a non-candidate, the votes cast in his favor should not have been counted. This
leaves Maquiling as the qualified candidate who obtained the highest number of votes.
Therefore, the rule on succession under the Local Government Code will not apply.

WHEREFORE, premises considered, the Petition is GRANTED. The Resolution of the


COMELEC En Bane dated 2 February 2011 is hereby ANNULLED and SET ASIDE.
Respondent ROMMEL ARNADO y CAGOCO is disqualified from running for any local elective
position. CASAN MACODE MAQUILING is hereby DECLARED the duly elected Mayor of
Kauswagan, Lanao del Norte in the 10 May 2010 elections.

This Decision is immediately executory.

Let a copy of this Decision be served personally upon the parties and the Commission on
Elections.

No pronouncement as to costs.

SO ORDERED.
G.R. No. 195649 July 2, 2013 of Philippine citizenship. Section 5(2)6 requires those who have re-acquired Philippine
citizenship and who seek elective public office, to renounce any and all foreign citizenship.
CASAN MACODE MACQUILING, PETITIONER,
vs. This requirement of renunciation of any and all foreign citizenship, when read together with
COMMISSION ON ELECTIONS, ROMMEL ARNADO Y CAGOCO, AND LINOG G. Section 40(d) of the Local Government Code7 which disqualifies those with dual citizenship from
BALUA. RESPONDENTS. running for any elective local position, indicates a policy that anyone who seeks to run for public
office must be solely and exclusively a Filipino citizen. To allow a former Filipino who reacquires
Philippine citizenship to continue using a foreign passport – which indicates the recognition of a
RESOLUTION
foreign state of the individual as its national – even after the Filipino has renounced his foreign
citizenship, is to allow a complete disregard of this policy.
SERENO, J.:
Further, we respectfully disagree that the majority decision rules on a situation of doubt.
This Resolution resolves the Motion for Reconsideration filed by respondent on May 10, 2013
and the Supplemental Motion for Reconsideration filed on May 20, 2013.
Indeed, there is no doubt that Section 40(d) of the Local Government Code disqualifies those
with dual citizenship from running for local elective positions.
We are not unaware that the term of office of the local officials elected in the May 2010 elections
has already ended on June 30, 2010. Arnado, therefore, has successfully finished his term of
There is likewise no doubt that the use of a passport is a positive declaration that one is a citizen
office. While the relief sought can no longer be granted, ruling on the motion for reconsideration
of the country which issued the passport, or that a passport proves that the country which issued
is important as it will either affirm the validity of Arnado’s election or affirm that Arnado never
it recognizes the person named therein as its national.
qualified to run for public office.

It is unquestioned that Arnado is a natural born Filipino citizen, or that he acquired American
Respondent failed to advance any argument to support his plea for the reversal of this Court’s
citizenship by naturalization. There is no doubt that he reacquired his Filipino citizenship by
Decision dated April 16, 2013. Instead, he presented his accomplishments as the Mayor of
taking his Oath of Allegiance to the Philippines and that he renounced his American citizenship.
Kauswagan, Lanao del Norte and reiterated that he has taken the Oath of Allegiance not only
It is also indubitable that after renouncing his American citizenship, Arnado used his U.S.
twice but six times. It must be stressed, however, that the relevant question is the efficacy of his
passport at least six times.
renunciation of his foreign citizenship and not the taking of the Oath of Allegiance to the
Republic of the Philippines. Neither do his accomplishments as mayor affect the question before
this Court. If there is any remaining doubt, it is regarding the efficacy of Arnado’s renunciation of his
American citizenship when he subsequently used his U.S. passport. The renunciation of foreign
citizenship must be complete and unequivocal. The requirement that the renunciation must be
Respondent cites Section 349 of the Immigration and Naturalization Act of the United States as
made through an oath emphasizes the solemn duty of the one making the oath of renunciation
having the effect of expatriation when he executed his Affidavit of Renunciation of American
to remain true to what he has sworn to. Allowing the subsequent use of a foreign passport
Citizenship on April 3, 2009 and thus claims that he was divested of his American citizenship. If
because it is convenient for the person to do so is rendering the oath a hollow act. It devalues
indeed, respondent was divested of all the rights of an American citizen, the fact that he was still
the act of taking of an oath, reducing it to a mere ceremonial formality.
able to use his US passport after executing his Affidavit of Renunciation repudiates this claim.

The dissent states that the Court has effectively left Arnado "a man without a
The Court cannot take judicial notice of foreign laws,1 which must be presented as public
country".1âwphi1 On the contrary, this Court has, in fact, found Arnado to have more than one.
documents2 of a foreign country and must be "evidenced by an official publication
Nowhere in the decision does it say that Arnado is not a Filipino citizen. What the decision
thereof."3 Mere reference to a foreign law in a pleading does not suffice for it to be considered in
merely points out is that he also possessed another citizenship at the time he filed his certificate
deciding a case.
of candidacy.

Respondent likewise contends that this Court failed to cite any law of the United States
Well-settled is the rule that findings of fact of administrative bodies will not be interfered with by
"providing that a person who is divested of American citizenship thru an Affidavit of Renunciation
the courts in the absence of grave abuse of discretion on the part of said agencies, or unless the
will re-acquire such American citizenship by using a US Passport issued prior to expatriation." 4
aforementioned findings are not supported by substantial evidence.8 They are accorded not only
great respect but even finality, and are binding upon this Court, unless it is shown that the
American law does not govern in this jurisdiction. Instead, Section 40(d) of the Local administrative body had arbitrarily disregarded or misapprehended evidence before it to such an
Government Code calls for application in the case before us, given the fact that at the time extent as to compel a contrary conclusion had such evidence been properly appreciated. 9
Arnado filed his certificate of candidacy, he was not only a Filipino citizen but, by his own
declaration, also an American citizen. It is the application of this law and not of any foreign law
Nevertheless, it must be emphasized that COMELEC First Division found that Arnado used his
that serves as the basis for Arnado’s disqualification to run for any local elective position.
U.S. Passport at least six times after he renounced his American citizenship. This was debunked
by the COMELEC En Banc, which found that Arnado only used his U.S. passport four times, and
With all due respect to the dissent, the declared policy of Republic Act No. (RA) 9225 is that "all which agreed with Arnado’s claim that he only used his U.S. passport on those occasions
Philippine citizens who become citizens of another country shall be deemed not to have lost their because his Philippine passport was not yet issued. The COMELEC En Banc argued that
Philippine citizenship under the conditions of this Act."5 This policy pertains to the reacquisition Arnado was able to prove that he used his Philippine passport for his travels on the following
dates: 12 January 2010, 31 January 2010, 31 March 2010, 16 April 2010, 20 May 2010, and 4
June 2010.

None of these dates coincide with the two other dates indicated in the certification issued by the
Bureau of Immigration showing that on 21 January 2010 and on 23 March 2010, Arnado arrived
in the Philippines using his U.S. Passport No. 057782700 which also indicated therein that his
nationality is USA-American. Adding these two travel dates to the travel record provided by the
Bureau of Immigration showing that Arnado also presented his U.S. passport four times (upon
departure on 14 April 2009, upon arrival on 25 June 2009, upon departure on 29 July 2009 and
upon arrival on 24 November 2009), these incidents sum up to six.

The COMELEC En Banc concluded that "the use of the US passport was because to his
knowledge, his Philippine passport was not yet issued to him for his use."10 This conclusion,
however, is not supported by the facts. Arnado claims that his Philippine passport was issued on
18 June 2009. The records show that he continued to use his U.S. passport even after he
already received his Philippine passport. Arnado’s travel records show that he presented his
U.S. passport on 24 November 2009, on 21 January 2010, and on 23 March 2010. These facts
were never refuted by Arnado.

Thus, the ruling of the COMELEC En Banc is based on a misapprehension of the facts that the
use of the U.S. passport was discontinued when Arnado obtained his Philippine passport.
Arnado’s continued use of his U.S. passport cannot be considered as isolated acts contrary to
what the dissent wants us to believe.

It must be stressed that what is at stake here is the principle that only those who are exclusively
Filipinos are qualified to run for public office. If we allow dual citizens who wish to run for public
office to renounce their foreign citizenship and afterwards continue using their foreign passports,
we are creating a special privilege for these dual citizens, thereby effectively junking the
prohibition in Section 40(d) of the Local Government Code.

WHEREFORE, the Motion for Reconsideration and the Supplemental Motion for
Reconsideration are hereby DENIED with finality.

SO ORDERED.
G.R. No. 199113 March 18, 2015 On June 3, 2008, the CENRO issued an order rejecting petitioner’s MLA. It ruled that petitioner’s
subsequent re-acquisition of Philippine citizenship did not cure the defect in his MLA which was
void ab initio.8
RENATO M. DAVID, Petitioner,
vs.
EDITHA A. AGBAY and PEOPLE OF THE PHILIPPINES, Respondents. In the meantime, on July 26, 2010, the petition for review filed by petitioner was denied by the
DOJ which held that the presence of the elements of the crime of falsification of public document
suffices to warrant indictment of the petitioner notwithstanding the absence of any proof that he
DECISION
gained or intended to injure a third person in committing the act of falsification.9 Consequently,
an information for Falsification of Public Document was filed before the MTC (Criminal Case No.
VILLARAMA, JR., J.: 2012) and a warrant of arrest was issued against the petitioner.

This is a petition for review under Rule 45 seeking to reverse the Order1 dated October 8, 2011 On February 11, 2011, after the filing of the Information and before his arrest, petitioner filed an
of the Regional Trial Court (RTC) of Pinamalayan, Oriental Mindoro, which denied the petition Urgent Motion for Re-Determination of Probable Cause10 in the MTC. Interpreting the provisions
for certiorari filed by Renato(petitioner)M. David. Petitioner assailed the Order2 dated March 22, of the law relied upon by petitioner, the said court denied the motion, holding that R.A. 9225
2011 of the Municipal Trial Court (MTC) of Socorro, Oriental Mindoro denying his motion for makes a distinction between those who became foreign citizens during its effectivity, and those
redetermination of probable cause. who lost their Philippine citizenship before its enactment when the governing law was
Commonwealth Act No. 6311 (CA 63). Since the crime for which petitioner was charged was
alleged and admitted to have been committed on April 12, 2007 before he had re- acquired his
The factual antecedents: Philippine citizenship, the MTC concluded that petitioner was at that time still a Canadian citizen.
Thus, the MTC ordered:
In 1974, petitioner migrated to Canada where he became a Canadian citizen by naturalization.
Upon their retirement, petitioner and his wife returned to the Philippines. Sometime in 2000, they WHEREFORE, for lack of jurisdiction over the person of the accused, and for lack of merit, the
purchased a 600-square meter lot along the beach in Tambong, Gloria, Oriental Mindoro where motion is DENIED.
they constructed a residential house. However, in the year 2004, they came to know that the
portion where they built their house is public land and part of the salvage zone.
SO ORDERED.12
On April 12, 2007, petitioner filed a Miscellaneous Lease Application3 (MLA) over the subject
land with the Department of Environment and Natural Resources (DENR) at the Community In his motion for reconsideration,13 petitioner questioned the foregoing order denying him relief
Environment and Natural Resources Office (CENRO) in Socorro. In the said application, on the ground of lack of jurisdiction and insisted that the issue raised is purely legal. He argued
petitioner indicated that he is a Filipino citizen. that since his application had yet to receive final evaluation and action by the DENR Region IV-B
office in Manila, it is academic to ask the citizenship of the applicant (petitioner) who had re-
acquired Philippine citizenship six months after he applied for lease of public land. The MTC
Private respondent Editha A. Agbay opposed the application on the ground that petitioner, a denied the motion for reconsideration.14
Canadian citizen, is disqualified to own land. She also filed a criminal complaint for falsification
of public documents under Article 172 of the Revised Penal Code (RPC) (I.S. No. 08-6463)
against the petitioner. Dissatisfied, petitioner elevated the case to the RTC via a petition15 for certiorari under Rule 65,
alleging grave abuse of discretion on the part of the MTC. He asserted that first, jurisdiction over
the person of an accused cannot be a pre-condition for the re-determination of probable cause
Meanwhile, petitioner re-acquired his Filipino citizenship under the provisions of Republic Act by the court that issues a warrant of arrest; and second, the March 22, 2011 Order disregarded
No. 9225,4 (R.A. 9225) as evidenced by Identification Certificate No. 266-10-075 issued by the the legal fiction that once a natural-born Filipino citizen who had been naturalized in another
Consulate General of the Philippines (Toronto) on October 11, 2007. country re-acquires his citizenship under R.A. 9225, his Filipino citizenship is thus deemed not to
have been lost on account of said naturalization.
In his defense, petitioner averred that at the time he filed his application, he had intended to re-
acquire Philippine citizenship and that he had been assured by a CENRO officer that he could In his Comment and Opposition,16 the prosecutor emphasized that the act of falsification was
declare himself as a Filipino. He further alleged that he bought the property from the Agbays
already consummated as petitioner has not yet re-acquired his Philippine citizenship, and his
who misrepresented to him that the subject property was titled land and they have the right and subsequent oath to re-acquire Philippine citizenship will only affect his citizenship status and not
authority to convey the same. The dispute had in fact led to the institution of civil and criminal his criminal act which was long consummated prior to said oath of allegiance.
suits between him and private respondent’s family.

On October 8, 2011, the RTC issued the assailed Order denying the petition for certiorari after
On January 8, 2008,6 the Office of the Provincial Prosecutor issued its Resolution7 finding finding no grave abuse of discretion committed by the lower court, thus:
probable cause to indict petitioner for violation of Article 172 of the RPC and recommending the
filing of the corresponding information in court. Petitioner challenged the said resolution in a
petition for review he filed before the Department of Justice (DOJ). ACCORDINGLY, the petition is hereby DENIED. At any rate petitioner is not left without any
remedy or recourse because he can proceed to trial where he can make use of his claim to be a
Filipino citizen as his defense to be adjudicated in a full blown trial, and in case of conviction, to
appeal such conviction.
SO ORDERED.17 by reason of their naturalization as citizens of a foreign country are hereby deemed to
have reacquired Philippine citizenship upon taking the following oath of allegiance to the
Republic:
Petitioner is now before us arguing that –

"I ______________________, solemnly swear (or affirm) that I will support and defend the
A. By supporting the prosecution of the petitioner for falsification, the lower court has
Constitution of the Republic of the Philippines and obey the laws and legal orders promulgated
disregarded the undisputed fact that petitioner is a natural-born Filipino citizen, and
by the duly constituted authorities of the Philippines; and I hereby declare that I recognize and
that by re-acquiring the same status under R.A. No. 9225 he was by legal fiction
accept the supreme authority of the Philippines and will maintain true faith and allegiance
"deemed not to have lost" it at the time of his naturalization in Canada and through the
thereto; and that I impose this obligation upon myself voluntarily without mental reservation or
time when he was said to have falsely claimed Philippine citizenship.
purpose of evasion."

B. By compelling petitioner to first return from his legal residence in Canada and to
Natural-born citizens of the Philippines who, after the effectivity of this Act, become citizens of
surrender or allow himself to be arrested under a warrant for his alleged false claim to
a foreign country shall retain their Philippine citizenship upon taking the aforesaid
Philippine citizenship, the lower court has pre-empted the right of petitioner through
oath. (Emphasis supplied)
his wife and counsel to question the validity of the said warrant of arrest against him
before the same is implemented, which is tantamount to a denial of due process. 18
While Section 2 declares the general policy that Filipinos who have become citizens of another
country shall be deemed "not to have lost their Philippine citizenship," such is qualified by the
In his Comment, the Solicitor General contends that petitioner’s argument regarding the
phrase "under the conditions of this Act." Section 3 lays down such conditions for two categories
retroactivity of R.A. 9225 is without merit.1âwphi1 It is contended that this Court’s rulings
of natural-born Filipinos referred to in the first and second paragraphs. Under the first paragraph
in Frivaldo v. Commission on Elections19 and Altarejos v. Commission on Elections20 on the
are those natural-born Filipinos who have lost their citizenship by naturalization in a foreign
retroactivity of one’s re- acquisition of Philippine citizenship to the date of filing his application
country who shall re-acquire their Philippine citizenship upon taking the oath of allegiance to the
therefor cannot be applied to the case of herein petitioner. Even assuming for the sake of
Republic of the Philippines. The second paragraph covers those natural-born Filipinos who
argument that such doctrine applies in the present situation, it will still not work for petitioner’s
became foreign citizens after R.A. 9225 took effect, who shall retain their Philippine citizenship
cause for the simple reason that he had not alleged, much less proved, that he had already
upon taking the same oath. The taking of oath of allegiance is required for both categories of
applied for reacquisition of Philippine citizenship before he made the declaration in the Public
natural-born Filipino citizens who became citizens of a foreign country, but the terminology used
Land Application that he is a Filipino. Moreover, it is stressed that in falsification of public
is different, "re-acquired" for the first group, and "retain" for the second group.
document, it is not necessary that the idea of gain or intent to injure a third person be present.
As to petitioner’s defense of good faith, such remains to be a defense which may be properly
raised and proved in a full- blown trial. The law thus makes a distinction between those natural-born Filipinos who became foreign
citizens before and after the effectivity of R.A. 9225. Although the heading of Section 3 is
"Retention of Philippine Citizenship", the authors of the law intentionally employed the terms "re-
On the issue of jurisdiction over the person of accused (petitioner), the Solicitor General opines
acquire" and "retain" to describe the legal effect of taking the oath of allegiance to the Republic
that in seeking an affirmative relief from the MTC when he filed his Urgent Motion for Re-
of the Philippines. This is also evident from the title of the law using both re-acquisition and
determination of Probable Cause, petitioner is deemed to have submitted his person to the said
retention.
court’s jurisdiction by his voluntary appearance. Nonetheless, the RTC correctly ruled that the
lower court committed no grave abuse of discretion in denying the petitioner’s motion after a
judicious, thorough and personal evaluation of the parties’ arguments contained in their In fine, for those who were naturalized in a foreign country, they shall be deemed to have re-
respective pleadings, and the evidence submitted before the court. acquired their Philippine citizenship which was lost pursuant to CA 63, under which
naturalization in a foreign country is one of the ways by which Philippine citizenship may be lost.
As its title declares, R.A. 9225 amends CA 63 by doing away with the provision in the old law
In sum, the Court is asked to resolve whether (1) petitioner may be indicted for falsification for
which takes away Philippine citizenship from natural-born Filipinos who become naturalized
representing himself as a Filipino in his Public Land Application despite his subsequent re-
citizens of other countries and allowing dual citizenship, 21 and also provides for the procedure for
acquisition of Philippine citizenship under the provisions of R.A. 9225; and (2) the MTC properly
re-acquiring and retaining Philippine citizenship. In the case of those who became foreign
denied petitioner’s motion for re-determination of probable cause on the ground of lack of
citizens after R.A. 9225 took effect, they shall retain Philippine citizenship despite having
jurisdiction over the person of the accused (petitioner).
acquired foreign citizenship provided they took the oath of allegiance under the new law.

R.A. 9225, otherwise known as the "Citizenship Retention and Re- acquisition Act of 2003," was
Petitioner insists we should not distinguish between re-acquisition and retention in R.A. 9225. He
signed into law by President Gloria Macapagal-Arroyo on August 29, 2003. Sections 2 and 3 of
asserts that in criminal cases, that interpretation of the law which favors the accused is preferred
said law read:
because it is consistent with the constitutional presumption of innocence, and in this case it
becomes more relevant when a seemingly difficult question of law is expected to have been
SEC. 2. Declaration of Policy.–It is hereby declared the policy of the State that all Philippine understood by the accused, who is a non-lawyer, at the time of the commission of the alleged
citizens who become citizens of another country shall be deemed not to have lost their offense. He further cites the letter-reply dated January 31, 201122 of the Bureau of Immigration
Philippine citizenship under the conditions of this Act. (BI) to his query, stating that his status as a natural-born Filipino will be governed by Section 2 of
R.A. 9225.
SEC. 3. Retention of Philippine Citizenship.–Any provision of law to the contrary
notwithstanding, natural-born citizens of the Philippines who have lost their Philippine citizenship These contentions have no merit.
That the law distinguishes between re-acquisition and retention of Philippine citizenship was Section 3 who lost Philippine citizenship by naturalization in a foreign country. As the new law
made clear in the discussion of the Bicameral Conference Committee on the Disagreeing allows dual citizenship, he was able to re-acquire his Philippine citizenship by taking the required
Provisions of House Bill No. 4720 and Senate Bill No. 2130 held on August 18, 2003, where oath of allegiance.
Senator Franklin Drilon was responding to the query of Representative Exequiel Javier:
For the purpose of determining the citizenship of petitioner at the time of filing his MLA, it is not
REP. JAVIER. I have some questions in Section 3. Here, under Section 3 of the Senate version, necessary to discuss the rulings in Frivaldo and Altarejos on the retroactivity of such
"Any provision of law on the contrary notwithstanding, natural-born citizens of the Philippines reacquisition because R.A. 9225 itself treats those of his category as having already lost
who, after the effectivity of this Act, shall… and so forth, ano, shall retain their Philippine Philippine citizenship, in contradistinction to those natural-born Filipinos who became foreign
citizenship. citizens after R.A. 9225 came into force. In other words, Section 2 declaring the policy that
considers Filipinos who became foreign citizens as not to have lost their Philippine citizenship,
should be read together with Section 3, the second paragraph of which clarifies that such policy
Now in the second paragraph, natural-born citizens who have lost their citizenship by reason of
governs all cases after the new law’s effectivity.
their naturalization after the effectivity of this Act are deemed to have reacquired…

As to the letter-reply of BI, it simply quoted Section 2 of R.A. 9225 without any reference to
THE CHAIRMAN (SEN. DRILON). Prior to the effectivity.
Section 3 on the particular application of reacquisition and retention to Filipinos who became
foreign citizens before and after the effectivity of R.A. 9225.
REP. JAVIER. Well, you have two kinds of natural-born citizens here. Natural-born citizens who
acquired foreign citizenship after the effectivity of this act are considered to have retained their
Petitioner’s plea to adopt the interpretation most favorable to the accused is likewise misplaced.
citizenship. But natural-born citizens who lost their Filipino citizenship before the effectivity of this
Courts adopt an interpretation more favorable to the accused following the time-honored
act are considered to have reacquired. May I know the distinction? Do you mean to say that
principle that penal statutes are construed strictly against the State and liberally in favor of the
natural-born citizens who became, let’s say, American citizens after the effectivity of this act are
accused.23 R.A. 9225, however, is not a penal law.
considered natural-born?

Falsification of documents under paragraph 1, Article 17224 in relation to Article 17125 of the RPC
Now in the second paragraph are the natural-born citizens who lost their citizenship before the
refers to falsification by a private individual, or a public officer or employee who did not take
effectivity of this act are no longer natural born citizens because they have just reacquired their
advantage of his official position, of public, private, or commercial documents. The elements of
citizenship. I just want to know this distinction, Mr. Chairman.
falsification of documents under paragraph 1, Article 172 of the RPC are:

THE CHAIRMAN (SEN. DRILON). The title of the Senate version is precisely retention and
(1)that the offender is a private individual or a public officer or employee who did not
reacquisition. The reacquisition will apply to those who lost their Philippine citizenship by
take advantage of his official position;
virtue of Commonwealth Act 63.Upon the effectivity -- assuming that we can agree on this,
upon the effectivity of this new measure amending Commonwealth Act 63, the Filipinos who lost
their citizenship is deemed to have reacquired their Philippine citizenship upon the effectivity of (2)that he committed any of the acts of falsification enumerated in Article 171 of the
the act. RPC; and

The second aspect is the retention of Philippine citizenship applying to future (3)that the falsification was committed in a public, official or commercial document. 26
instances. So that’s the distinction.
Petitioner made the untruthful statement in the MLA, a public document, that he is a Filipino
REP. JAVIER. Well, I’m just asking this question because we are here making distinctions citizen at the time of the filing of said application, when in fact he was then still a Canadian
between natural-born citizens. Because this is very important for certain government positions, citizen. Under CA 63, the governing law at the time he was naturalized as Canadian citizen,
‘no, because natural-born citizens are only qualified for a specific… naturalization in a foreign country was among those ways by which a natural-born citizen loses
his Philippine citizenship. While he re-acquired Philippine citizenship under R.A. 9225 six
months later, the falsification was already a consummated act, the said law having no retroactive
THE CHAIRMAN (SEN. DRILON). That is correct.
effect insofar as his dual citizenship status is concerned. The MTC therefore did not err in finding
probable cause for falsification of public document under Article 172, paragraph 1.
REP. JAVIER. ...positions under the Constitution and under the law.
The MTC further cited lack of jurisdiction over the person of petitioner accused as ground for
THE CHAIRMAN (SEN. DRILON). Yes. We can get to that later on. It’s one of the provisions, denying petitioner’s motion for re- determination of probable cause, as the motion was filed prior
yes. But just for purposes of the explanation, Congressman Javier, that is our to his arrest. However, custody of the law is not required for the adjudication of reliefs other than
conceptualization. Reacquired for those who previously lost [Filipino citizenship] by an application for bail.27 In Miranda v. Tuliao,28 which involved a motion to quash warrant of
virtue of Commonwealth Act 63, and retention for those in the future. (Emphasis supplied) arrest, this Court discussed the distinction between custody of the law and jurisdiction over the
person, and held that jurisdiction over the person of the accused is deemed waived when he
files any pleading seeking an affirmative relief, except in cases when he invokes the special
Considering that petitioner was naturalized as a Canadian citizen prior to the effectivity of R.A.
jurisdiction of the court by impugning such jurisdiction over his person. Thus:
9225, he belongs to the first category of natural- born Filipinos under the first paragraph of
In arguing, on the other hand, that jurisdiction over their person was already acquired by their WHEREFORE, the petition is DENIED. The Order dated October 8, 2011 of the Regional Trial
filing of the above Urgent Motion, petitioners invoke our pronouncement, through Justice Florenz Court of Pinamalayan, Oriental Mindoro in Civil Case No. SCA-07-11 (Criminal Case No. 2012)
D. Regalado, in Santiago v. Vasquez: is hereby AFFIRMED and UPHELD.

The voluntary appearance of the accused, whereby the court acquires jurisdiction over his With costs against the petitioner.
person, is accomplished either by his pleading to the merits (such as by filing a motion to quash
or other pleadings requiring the exercise of the court’s jurisdiction thereover, appearing for
SO ORDERED.
arraignment, entering trial) or by filing bail. On the matter of bail, since the same is intended to
obtain the provisional liberty of the accused, as a rule the same cannot be posted before custody
of the accused has been acquired by the judicial authorities either by his arrest or voluntary
surrender.

Our pronouncement in Santiago shows a distinction between custody of the law and jurisdiction
over the person. Custody of the law is required before the court can act upon the application for
bail, but is not required for the adjudication of other reliefs sought by the defendant where the
mere application therefor constitutes a waiver of the defense of lack of jurisdiction over the
person of the accused. Custody of the law is accomplished either by arrest or voluntary
surrender, while jurisdiction over the person of the accused is acquired upon his arrest or
voluntary appearance. One can be under the custody of the law but not yet subject to the
jurisdiction of the court over his person, such as when a person arrested by virtue of a warrant
files a motion before arraignment to quash the warrant. On the other hand, one can be subject to
the jurisdiction of the court over his person, and yet not be in the custody of the law, such as
when an accused escapes custody after his trial has commenced. Being in the custody of the
law signifies restraint on the person, who is thereby deprived of his own will and liberty, binding
him to become obedient to the will of the law. Custody of the law is literally custody over the
body of the accused. It includes, but is not limited to, detention.

xxxx

While we stand by our above pronouncement in Pico insofar as it concerns bail, we clarify
that, as a general rule, one who seeks an affirmative relief is deemed to have submitted to
the jurisdiction of the court. As we held in the aforecited case of Santiago, seeking an
affirmative relief in court, whether in civil or criminal proceedings, constitutes voluntary
appearance.

xxxx

To recapitulate what we have discussed so far, in criminal cases, jurisdiction over the
person of the accused is deemed waived by the accused when he files any pleading
seeking an affirmative relief, except in cases when he invokes the special jurisdiction of
the court by impugning such jurisdiction over his person.Therefore, in narrow cases
involving special appearances, an accused can invoke the processes of the court even though
there is neither jurisdiction over the person nor custody of the law. However, if a person invoking
the special jurisdiction of the court applies for bail, he must first submit himself to the custody of
the law.29 (Emphasis supplied)

Considering that petitioner sought affirmative relief in filing his motion for re-determination of
probable cause, the MTC clearly erred in stating that it lacked jurisdiction over his person.
Notwithstanding such erroneous ground stated in the MTC's order, the RTC correctly ruled that
no grave abuse of discretion was committed by the MTC in denying the said motion for lack of
merit.
G.R. No. 187567 February 15, 2012 WHEREFORE, the instant petition is hereby GRANTED. Petitioner Nora Fe Sagun y Chan is
hereby DECLARED [a] FILIPINO CITIZEN, having chosen or elected Filipino citizenship.
THE REPUBLIC OF THE PHILIPPINES, Petitioner,
vs. Upon payment of the required fees, the Local Civil Registrar of Baguio City is hereby directed to
NORA FE SAGUN, Respondent. annotate [on] her birth certificate, this judicial declaration of Filipino citizenship of said petitioner.

DECISION IT IS SO ORDERED.7

VILLARAMA, JR., J.: Contending that the lower court erred in so ruling, petitioner, through the OSG, directly filed the
instant recourse viaa petition for review on certiorari before us. Petitioner raises the following
issues:
Before us is a petition for review on certiorari filed by the Solicitor General on behalf of the
Republic of the Philippines, seeking the reversal of the April 3, 2009 Decision1 of the Regional
Trial Court (RTC), Branch 3, of Baguio City in Spcl. Pro. Case No. 17-R. The RTC granted the I
petition2 filed by respondent Nora Fe Sagun entitled "In re: Judicial Declaration of Election of
Filipino Citizenship, Nora Fe Sagun v. The Local Civil Registrar of Baguio City."
Whether or not an action or proceeding for judicial declaration of Philippine citizenship
is procedurally and jurisdictionally permissible; and,
The facts follow:
II
Respondent is the legitimate child of Albert S. Chan, a Chinese national, and Marta Borromeo, a
Filipino citizen. She was born on August 8, 1959 in Baguio City3 and did not elect Philippine
Whether or not an election of Philippine citizenship, made twelve (12) years after
citizenship upon reaching the age of majority. In 1992, at the age of 33 and after getting married
reaching the age of majority, is considered to have been made "within a reasonable
to Alex Sagun, she executed an Oath of Allegiance4 to the Republic of the Philippines. Said
time" as interpreted by jurisprudence.8
document was notarized by Atty. Cristeta Leung on December 17, 1992, but was not recorded
and registered with the Local Civil Registrar of Baguio City.
Petitioner argues that respondent’s petition before the RTC was improper on two counts: for
one, law and jurisprudence clearly contemplate no judicial action or proceeding for the
Sometime in September 2005, respondent applied for a Philippine passport. Her application was
declaration of Philippine citizenship; and for another, the pleaded registration of the oath of
denied due to the citizenship of her father and there being no annotation on her birth certificate
allegiance with the local civil registry and its annotation on respondent’s birth certificate are the
that she has elected Philippine citizenship. Consequently, she sought a judicial declaration of
ministerial duties of the registrar; hence, they require no court order. Petitioner asserts that
her election of Philippine citizenship and prayed that the Local Civil Registrar of Baguio City be
respondent’s petition before the trial court seeking a judicial declaration of her election of
ordered to annotate the same on her birth certificate.
Philippine citizenship undeniably entails a determination and consequent declaration of her
status as a Filipino citizen which is not allowed under our legal system. Petitioner also argues
In her petition, respondent averred that she was raised as a Filipino, speaks Ilocano and that if respondent’s intention in filing the petition is ultimately to have her oath of allegiance
Tagalog fluently and attended local schools in Baguio City, including Holy Family Academy and registered with the local civil registry and annotated on her birth certificate, then she does not
the Saint Louis University. Respondent claimed that despite her part-Chinese ancestry, she have to resort to court proceedings.
always thought of herself as a Filipino. She is a registered voter of Precinct No. 0419A of
Barangay Manuel A. Roxas in Baguio City and had voted in local and national elections as
Petitioner further argues that even assuming that respondent’s action is sanctioned, the trial
shown in the Voter Certification5 issued by Atty. Maribelle Uminga of the Commission on
court erred in finding respondent as having duly elected Philippine citizenship since her
Elections of Baguio City.
purported election was not in accordance with the procedure prescribed by law and was not
made within a "reasonable time." Petitioner points out that while respondent executed an oath of
She asserted that by virtue of her positive acts, she has effectively elected Philippine citizenship allegiance before a notary public, there was no affidavit of her election of Philippine citizenship.
and such fact should be annotated on her record of birth so as to entitle her to the issuance of a Additionally, her oath of allegiance which was not registered with the nearest local civil registry
Philippine passport. was executed when she was already 33 years old or 12 years after she reached the age of
majority. Accordingly, it was made beyond the period allowed by law.
On August 7, 2007, the Office of the Solicitor General (OSG) entered its appearance as counsel
for the Republic of the Philippines and authorized the City Prosecutor of Baguio City to appear in In her Comment,9 respondent avers that notwithstanding her failure to formally elect Filipino
the above mentioned case.6 However, no comment was filed by the City Prosecutor. citizenship upon reaching the age of majority, she has in fact effectively elected Filipino
citizenship by her performance of positive acts, among which is the exercise of the right of
suffrage. She claims that she had voted and participated in all local and national elections from
After conducting a hearing, the trial court rendered the assailed Decision on April 3, 2009
the time she was of legal age. She also insists that she is a Filipino citizen despite the fact that
granting the petition and declaring respondent a Filipino citizen. The fallo of the decision reads:
her "election" of Philippine citizenship was delayed and unregistered.
In reply,10 petitioner argues that the special circumstances invoked by respondent, like her said right, and a remedy, granted or sanctioned by law, for said breach of right. As an incident
continuous and uninterrupted stay in the Philippines, her having been educated in schools in the only of the adjudication of the rights of the parties to a controversy, the court may pass upon,
country, her choice of staying here despite the naturalization of her parents as American and make a pronouncement relative to their status. Otherwise, such a pronouncement is beyond
citizens, and her being a registered voter, cannot confer on her Philippine citizenship as the law judicial power. x x x
specifically provides the requirements for acquisition of Philippine citizenship by election.
Clearly, it was erroneous for the trial court to make a specific declaration of respondent’s Filipino
Essentially, the issues for our resolution are: (1) whether respondent’s petition for declaration of citizenship as such pronouncement was not within the court’s competence.
election of Philippine citizenship is sanctioned by the Rules of Court and jurisprudence; (2)
whether respondent has effectively elected Philippine citizenship in accordance with the
As to the propriety of respondent’s petition seeking a judicial declaration of election of Philippine
procedure prescribed by law.
citizenship, it is imperative that we determine whether respondent is required under the law to
make an election and if so, whether she has complied with the procedural requirements in the
The petition is meritorious. election of Philippine citizenship.

At the outset, it is necessary to stress that a direct recourse to this Court from the decisions, final When respondent was born on August 8, 1959, the governing charter was the 1935 Constitution,
resolutions and orders of the RTC may be taken where only questions of law are raised or which declares as citizens of the Philippines those whose mothers are citizens of the Philippines
involved. There is a question of law when the doubt or difference arises as to what the law is on and elect Philippine citizenship upon reaching the age of majority. Sec. 1, Art. IV of the 1935
a certain state of facts, which does not call for an examination of the probative value of the Constitution reads:
evidence presented by the parties-litigants. On the other hand, there is a question of fact when
the doubt or controversy arises as to the truth or falsity of the alleged facts. Simply put, when
Section 1. The following are citizens of the Philippines:
there is no dispute as to fact, the question of whether the conclusion drawn therefrom is correct
or not, is a question of law.11
xxxx
In the present case, petitioner assails the propriety of the decision of the trial court declaring
respondent a Filipino citizen after finding that respondent was able to substantiate her election of (4) Those whose mothers are citizens of the Philippines and, upon reaching the age of majority,
Filipino citizenship. Petitioner contends that respondent’s petition for judicial declaration of elect Philippine citizenship.
election of Philippine citizenship is procedurally and jurisdictionally impermissible. Verily,
petitioner has raised questions of law as the resolution of these issues rest solely on what the
law provides given the attendant circumstances. Under Article IV, Section 1(4) of the 1935 Constitution, the citizenship of a legitimate child born
of a Filipino mother and an alien father followed the citizenship of the father, unless, upon
reaching the age of majority, the child elected Philippine citizenship. The right to elect Philippine
In granting the petition, the trial court stated: citizenship was recognized in the 1973 Constitution when it provided that "[t]hose who elect
Philippine citizenship pursuant to the provisions of the Constitution of nineteen hundred and
thirty-five" are citizens of the Philippines.17 Likewise, this recognition by the 1973
This Court believes that petitioner was able to fully substantiate her petition regarding her
Constitution was carried over to the 1987 Constitution which states that "[t]hose born before
election of Filipino citizenship, and the Local Civil Registrar of Baguio City should be ordered to
January 17, 1973 of Filipino mothers, who elect Philippine citizenship upon reaching the age of
annotate in her birth certificate her election of Filipino citizenship. This Court adds that the
majority" are Philippine citizens.18 It should be noted, however, that the 1973 and 1987
petitioner’s election of Filipino citizenship should be welcomed by this country and people
Constitutional provisions on the election of Philippine citizenship should not be understood as
because the petitioner has the choice to elect citizenship of powerful countries like the United
having a curative effect on any irregularity in the acquisition of citizenship for those covered by
States of America and China, however, petitioner has chosen Filipino citizenship because she
the 1935 Constitution. If the citizenship of a person was subject to challenge under the old
grew up in this country, and has learned to love the Philippines. Her choice of electing Filipino
charter, it remains subject to challenge under the new charter even if the judicial challenge had
citizenship is, in fact, a testimony that many of our people still wish to live in the Philippines, and
not been commenced before the effectivity of the new Constitution. 19
are very proud of our country.

Being a legitimate child, respondent’s citizenship followed that of her father who is Chinese,
WHEREFORE, the instant petition is hereby GRANTED. Petitioner Nora Fe Sagun y Chan is
unless upon reaching the age of majority, she elects Philippine citizenship. It is a settled rule that
hereby DECLARED as FILIPINO CITIZEN, having chosen or elected Filipino citizenship. 12
only legitimate children follow the citizenship of the father and that illegitimate children are under
the parental authority of the mother and follow her nationality.20 An illegitimate child of Filipina
For sure, this Court has consistently ruled that there is no proceeding established by law, or the need not perform any act to confer upon him all the rights and privileges attached to citizens of
Rules for the judicial declaration of the citizenship of an individual. 13 There is no specific the Philippines; he automatically becomes a citizen himself. 21 But in the case of respondent, for
legislation authorizing the institution of a judicial proceeding to declare that a given person is part her to be considered a Filipino citizen, she must have validly elected Philippine citizenship upon
of our citizenry.14 This was our ruling in Yung Uan Chu v. Republic15citing the early case of Tan reaching the age of majority.
v. Republic of the Philippines,16 where we clearly stated:
Commonwealth Act (C.A.) No. 625,22 enacted pursuant to Section 1(4), Article IV of the 1935
Under our laws, there can be no action or proceeding for the judicial declaration of the Constitution, prescribes the procedure that should be followed in order to make a valid election
citizenship of an individual. Courts of justice exist for settlement of justiciable controversies, of Philippine citizenship, to wit:
which imply a given right, legally demandable and enforceable, an act or omission violative of
Section 1. The option to elect Philippine citizenship in accordance with subsection (4), [S]ection WHEREFORE, the petition is GRANTED. The Decision dated April 3, 2009 of the Regional Trial
1, Article IV, of the Constitution shall be expressed in a statement to be signed and sworn to by Court, Branch 3 of Baguio City in Spcl. Pro. Case No. 17-R is REVERSED and SET ASIDE. The
the party concerned before any officer authorized to administer oaths, and shall be filed with the petition for judicial declaration of election of Philippine citizenship filed by respondent Nora Fe
nearest civil registry. The said party shall accompany the aforesaid statement with the oath of Sagun is hereby DISMISSED for lack of merit.
allegiance to the Constitution and the Government of the Philippines.
No costs.
Based on the foregoing, the statutory formalities of electing Philippine citizenship are: (1) a
statement of election under oath; (2) an oath of allegiance to the Constitution and Government of
SO ORDERED.
the Philippines; and (3) registration of the statement of election and of the oath with the nearest
civil registry.23

Furthermore, no election of Philippine citizenship shall be accepted for registration under C.A.
No. 625 unless the party exercising the right of election has complied with the requirements of
the Alien Registration Act of 1950. In other words, he should first be required to register as an
alien.24 Pertinently, the person electing Philippine citizenship is required to file a petition with the
Commission of Immigration and Deportation (now Bureau of Immigration) for the cancellation of
his alien certificate of registration based on his aforesaid election of Philippine citizenship and
said Office will initially decide, based on the evidence presented the validity or invalidity of said
election.25 Afterwards, the same is elevated to the Ministry (now Department) of Justice for final
determination and review.26 1âwphi1

It should be stressed that there is no specific statutory or procedural rule which authorizes the
direct filing of a petition for declaration of election of Philippine citizenship before the courts. The
special proceeding provided under Section 2, Rule 108 of the Rules of Court on Cancellation or
Correction of Entries in the Civil Registry, merely allows any interested party to file an action for
cancellation or correction of entry in the civil registry, i.e., election, loss and recovery of
citizenship, which is not the relief prayed for by the respondent.

Be that as it may, even if we set aside this procedural infirmity, still the trial court’s conclusion
that respondent duly elected Philippine citizenship is erroneous since the records undisputably
show that respondent failed to comply with the legal requirements for a valid election.
Specifically, respondent had not executed a sworn statement of her election of Philippine
citizenship. The only documentary evidence submitted by respondent in support of her claim of
alleged election was her oath of allegiance, executed 12 years after she reached the age of
majority, which was unregistered. As aptly pointed out by the petitioner, even
assuming arguendo that respondent’s oath of allegiance suffices, its execution was not within a
reasonable time after respondent attained the age of majority and was not registered with the
nearest civil registry as required under Section 1 of C.A. No. 625. The phrase "reasonable time"
has been interpreted to mean that the election should be made generally within three (3) years
from reaching the age of majority.27 Moreover, there was no satisfactory explanation proffered by
respondent for the delay and the failure to register with the nearest local civil registry.

Based on the foregoing circumstances, respondent clearly failed to comply with the procedural
requirements for a valid and effective election of Philippine citizenship. Respondent cannot
assert that the exercise of suffrage and the participation in election exercises constitutes a
positive act of election of Philippine citizenship since the law specifically lays down the
requirements for acquisition of citizenship by election. The mere exercise of suffrage, continuous
and uninterrupted stay in the Philippines, and other similar acts showing exercise of Philippine
citizenship cannot take the place of election of Philippine citizenship. Hence, respondent cannot
now be allowed to seek the intervention of the court to confer upon her Philippine citizenship
when clearly she has failed to validly elect Philippine citizenship. As we held in Ching,28 the
prescribed procedure in electing Philippine citizenship is certainly not a tedious and painstaking
process. All that is required of the elector is to execute an affidavit of election of Philippine
citizenship and, thereafter, file the same with the nearest civil registry. Having failed to comply
with the foregoing requirements, respondent’s petition before the trial court must be denied.

Anda mungkin juga menyukai